You are on page 1of 431

Manifestations of Democracy and Republicanism

Existence of bill of rights


-Province of North Cotabato v GRP Peace Panel on Ancestral Domain, 568 SCRA 402 (2008) (Associative
State/Bangsamoro)

Subject of these consolidated cases is the extent of the powers of the President in pursuing the peace process. While
the facts surrounding this controversy center on the armed conflict in Mindanao between the government and the Moro
Islamic Liberation Front (MILF), the legal issue involved has a bearing on all areas in the country where there has been a
long-standing armed conflict. Yet again, the Court is tasked to perform a delicate balancing act. It must uncompromisingly
delineate the bounds within which the President may lawfully exercise her discretion, but it must do so in strict adherence
to the Constitution, lest its ruling unduly restricts the freedom of action vested by that same Constitution in the Chief
Executive precisely to enable her to pursue the peace process effectively.

I. FACTUAL ANTECEDENTS OF THE PETITIONS

On August 5, 2008, the Government of the Republic of the Philippines (GRP) and the MILF, through the Chairpersons of
their respective peace negotiating panels, were scheduled to sign a Memorandum of Agreement on the Ancestral Domain
(MOA-AD) Aspect of the GRP-MILF Tripoli Agreement on Peace of 2001 in Kuala Lumpur, Malaysia.

The MILF is a rebel group which was established in March 1984 when, under the leadership of the late Salamat Hashim, it
splintered from the Moro National Liberation Front (MNLF) then headed by Nur Misuari, on the ground, among others, of
what Salamat perceived to be the manipulation of the MNLF away from an Islamic basis towards Marxist-Maoist
orientations.1

The signing of the MOA-AD between the GRP and the MILF was not to materialize, however, for upon motion of
petitioners, specifically those who filed their cases before the scheduled signing of the MOA-AD, this Court issued a
Temporary Restraining Order enjoining the GRP from signing the same.

The MOA-AD was preceded by a long process of negotiation and the concluding of several prior agreements between the
two parties beginning in 1996, when the GRP-MILF peace negotiations began. On July 18, 1997, the GRP and MILF
Peace Panels signed the Agreement on General Cessation of Hostilities. The following year, they signed the General
Framework of Agreement of Intent on August 27, 1998.

The Solicitor General, who represents respondents, summarizes the MOA-AD by stating that the same contained, among
others, the commitment of the parties to pursue peace negotiations, protect and respect human rights, negotiate with
sincerity in the resolution and pacific settlement of the conflict, and refrain from the use of threat or force to attain undue
advantage while the peace negotiations on the substantive agenda are on-going. 2

Early on, however, it was evident that there was not going to be any smooth sailing in the GRP-MILF peace process.
Towards the end of 1999 up to early 2000, the MILF attacked a number of municipalities in Central Mindanao and, in
March 2000, it took control of the town hall of Kauswagan, Lanao del Norte. 3 In response, then President Joseph Estrada
declared and carried out an "all-out-war" against the MILF.

When President Gloria Macapagal-Arroyo assumed office, the military offensive against the MILF was suspended and the
government sought a resumption of the peace talks. The MILF, according to a leading MILF member, initially responded
with deep reservation, but when President Arroyo asked the Government of Malaysia through Prime Minister Mahathir
Mohammad to help convince the MILF to return to the negotiating table, the MILF convened its Central Committee to
seriously discuss the matter and, eventually, decided to meet with the GRP. 4

The parties met in Kuala Lumpur on March 24, 2001, with the talks being facilitated by the Malaysian government, the
parties signing on the same date the Agreement on the General Framework for the Resumption of Peace Talks Between
the GRP and the MILF. The MILF thereafter suspended all its military actions. 5

Formal peace talks between the parties were held in Tripoli, Libya from June 20-22, 2001, the outcome of which was the
GRP-MILF Tripoli Agreement on Peace (Tripoli Agreement 2001) containing the basic principles and agenda on the
following aspects of the negotiation: Security Aspect, Rehabilitation Aspect, and Ancestral Domain Aspect. With regard
to the Ancestral Domain Aspect, the parties in Tripoli Agreement 2001 simply agreed "that the same be discussed further
by the Parties in their next meeting."

A second round of peace talks was held in Cyberjaya, Malaysia on August 5-7, 2001 which ended with the signing of
the Implementing Guidelines on the Security Aspect of the Tripoli Agreement 2001 leading to a ceasefire status between
the parties. This was followed by the Implementing Guidelines on the Humanitarian Rehabilitation and Development
Aspects of the Tripoli Agreement 2001, which was signed on May 7, 2002 at Putrajaya, Malaysia. Nonetheless, there were
many incidence of violence between government forces and the MILF from 2002 to 2003.

Meanwhile, then MILF Chairman Salamat Hashim passed away on July 13, 2003 and he was replaced by Al Haj Murad,
who was then the chief peace negotiator of the MILF. Murad's position as chief peace negotiator was taken over by
Mohagher Iqbal.6
In 2005, several exploratory talks were held between the parties in Kuala Lumpur, eventually leading to the crafting of the
draft MOA-AD in its final form, which, as mentioned, was set to be signed last August 5, 2008.

II. STATEMENT OF THE PROCEEDINGS

Before the Court is what is perhaps the most contentious "consensus" ever embodied in an instrument - the MOA-AD
which is assailed principally by the present petitions bearing docket numbers 183591, 183752, 183893, 183951 and
183962.

Commonly impleaded as respondents are the GRP Peace Panel on Ancestral Domain 7 and the Presidential Adviser on
the Peace Process (PAPP) Hermogenes Esperon, Jr.

On July 23, 2008, the Province of North Cotabato 8 and Vice-Governor Emmanuel Piñol filed a petition, docketed as G.R.
No. 183591, for Mandamus and Prohibition with Prayer for the Issuance of Writ of Preliminary Injunction and Temporary
Restraining Order.9 Invoking the right to information on matters of public concern, petitioners seek to compel respondents
to disclose and furnish them the complete and official copies of the MOA-AD including its attachments, and to prohibit the
slated signing of the MOA-AD, pending the disclosure of the contents of the MOA-AD and the holding of a public
consultation thereon. Supplementarily, petitioners pray that the MOA-AD be declared unconstitutional. 10

This initial petition was followed by another one, docketed as G.R. No. 183752, also for Mandamus and Prohibition 11 filed
by the City of Zamboanga,12 Mayor Celso Lobregat, Rep. Ma. Isabelle Climaco and Rep. Erico Basilio Fabian who likewise
pray for similar injunctive reliefs. Petitioners herein moreover pray that the City of Zamboanga be excluded from the
Bangsamoro Homeland and/or Bangsamoro Juridical Entity and, in the alternative, that the MOA-AD be declared null and
void.

By Resolution of August 4, 2008, the Court issued a Temporary Restraining Order commanding and directing public
respondents and their agents to cease and desist from formally signing the MOA-AD. 13 The Court also required the
Solicitor General to submit to the Court and petitioners the official copy of the final draft of the MOA-AD, 14 to which she
complied.15

Meanwhile, the City of Iligan16 filed a petition for Injunction and/or Declaratory Relief, docketed as G.R. No. 183893,
praying that respondents be enjoined from signing the MOA-AD or, if the same had already been signed, from
implementing the same, and that the MOA-AD be declared unconstitutional. Petitioners herein additionally implead
Executive Secretary Eduardo Ermita as respondent.

The Province of Zamboanga del Norte,17 Governor Rolando Yebes, Vice-Governor Francis Olvis, Rep. Cecilia Jalosjos-
Carreon, Rep. Cesar Jalosjos, and the members 18 of the Sangguniang Panlalawigan of Zamboanga del Norte filed on
August 15, 2008 a petition for Certiorari, Mandamus and Prohibition, 19 docketed as G.R. No. 183951. They pray, inter alia,
that the MOA-AD be declared null and void and without operative effect, and that respondents be enjoined from executing
the MOA-AD.

On August 19, 2008, Ernesto Maceda, Jejomar Binay, and Aquilino Pimentel III filed a petition for Prohibition, 20 docketed
as G.R. No. 183962, praying for a judgment prohibiting and permanently enjoining respondents from formally signing and
executing the MOA-AD and or any other agreement derived therefrom or similar thereto, and nullifying the MOA-AD for
being unconstitutional and illegal. Petitioners herein additionally implead as respondent the MILF Peace Negotiating Panel
represented by its Chairman Mohagher Iqbal.

Various parties moved to intervene and were granted leave of court to file their petitions-/comments-in-intervention.
Petitioners-in-Intervention include Senator Manuel A. Roxas, former Senate President Franklin Drilon and Atty. Adel
Tamano, the City of Isabela21 and Mayor Cherrylyn Santos-Akbar, the Province of Sultan Kudarat 22 and Gov. Suharto
Mangudadatu, the Municipality of Linamon in Lanao del Norte, 23 Ruy Elias Lopez of Davao City and of the Bagobo
tribe, Sangguniang Panlungsod member Marino Ridao and businessman Kisin Buxani, both of Cotabato City; and lawyers
Carlo Gomez, Gerardo Dilig, Nesario Awat, Joselito Alisuag, Richalex Jagmis, all of Palawan City. The Muslim Legal
Assistance Foundation, Inc. (Muslaf) and the Muslim Multi-Sectoral Movement for Peace and Development (MMMPD)
filed their respective Comments-in-Intervention.

By subsequent Resolutions, the Court ordered the consolidation of the petitions. Respondents filed Comments on the
petitions, while some of petitioners submitted their respective Replies.

Respondents, by Manifestation and Motion of August 19, 2008, stated that the Executive Department shall thoroughly
review the MOA-AD and pursue further negotiations to address the issues hurled against it, and thus moved to dismiss the
cases. In the succeeding exchange of pleadings, respondents' motion was met with vigorous opposition from petitioners.

The cases were heard on oral argument on August 15, 22 and 29, 2008 that tackled the following principal issues:

1. Whether the petitions have become moot and academic

(i) insofar as the mandamus aspect is concerned, in view of the disclosure of official copies of the final draft
of the Memorandum of Agreement (MOA); and
(ii) insofar as the prohibition aspect involving the Local Government Units is concerned, if it is considered
that consultation has become fait accompli with the finalization of the draft;

2. Whether the constitutionality and the legality of the MOA is ripe for adjudication;

3. Whether respondent Government of the Republic of the Philippines Peace Panel committed grave abuse of
discretion amounting to lack or excess of jurisdiction when it negotiated and initiated the MOA vis-à-vis ISSUES
Nos. 4 and 5;

4. Whether there is a violation of the people's right to information on matters of public concern (1987 Constitution,
Article III, Sec. 7) under a state policy of full disclosure of all its transactions involving public interest (1987
Constitution, Article II, Sec. 28) including public consultation under Republic Act No. 7160 (LOCAL
GOVERNMENT CODE OF 1991)[;]

If it is in the affirmative, whether prohibition under Rule 65 of the 1997 Rules of Civil Procedure is an appropriate
remedy;

5. Whether by signing the MOA, the Government of the Republic of the Philippines would be BINDING itself

a) to create and recognize the Bangsamoro Juridical Entity (BJE) as a separate state, or a juridical,
territorial or political subdivision not recognized by law;

b) to revise or amend the Constitution and existing laws to conform to the MOA;

c) to concede to or recognize the claim of the Moro Islamic Liberation Front for ancestral domain in
violation of Republic Act No. 8371 (THE INDIGENOUS PEOPLES RIGHTS ACT OF 1997), particularly
Section 3(g) & Chapter VII (DELINEATION, RECOGNITION OF ANCESTRAL DOMAINS)[;]

If in the affirmative, whether the Executive Branch has the authority to so bind the Government of the Republic of
the Philippines;

6. Whether the inclusion/exclusion of the Province of North Cotabato, Cities of Zamboanga, Iligan and Isabela, and
the Municipality of Linamon, Lanao del Norte in/from the areas covered by the projected Bangsamoro Homeland is
a justiciable question; and

7. Whether desistance from signing the MOA derogates any prior valid commitments of the Government of the
Republic of the Philippines.24

The Court, thereafter, ordered the parties to submit their respective Memoranda. Most of the parties submitted their
memoranda on time.

III. OVERVIEW OF THE MOA-AD

As a necessary backdrop to the consideration of the objections raised in the subject five petitions and six petitions-in-
intervention against the MOA-AD, as well as the two comments-in-intervention in favor of the MOA-AD, the Court takes an
overview of the MOA.

The MOA-AD identifies the Parties to it as the GRP and the MILF.

Under the heading "Terms of Reference" (TOR), the MOA-AD includes not only four earlier agreements between the GRP
and MILF, but also two agreements between the GRP and the MNLF: the 1976 Tripoli Agreement, and the Final Peace
Agreement on the Implementation of the 1976 Tripoli Agreement, signed on September 2, 1996 during the administration
of President Fidel Ramos.

The MOA-AD also identifies as TOR two local statutes - the organic act for the Autonomous Region in Muslim Mindanao
(ARMM)25 and the Indigenous Peoples Rights Act (IPRA),26 and several international law instruments - the ILO Convention
No. 169 Concerning Indigenous and Tribal Peoples in Independent Countries in relation to the UN Declaration on the
Rights of the Indigenous Peoples, and the UN Charter, among others.

The MOA-AD includes as a final TOR the generic category of "compact rights entrenchment emanating from the regime
of dar-ul-mua'hada (or territory under compact) and dar-ul-sulh (or territory under peace agreement) that partakes the
nature of a treaty device."

During the height of the Muslim Empire, early Muslim jurists tended to see the world through a simple dichotomy: there
was the dar-ul-Islam (the Abode of Islam) and dar-ul-harb (the Abode of War). The first referred to those lands where
Islamic laws held sway, while the second denoted those lands where Muslims were persecuted or where Muslim laws
were outlawed or ineffective.27 This way of viewing the world, however, became more complex through the centuries as
the Islamic world became part of the international community of nations.
As Muslim States entered into treaties with their neighbors, even with distant States and inter-governmental organizations,
the classical division of the world into dar-ul-Islam and dar-ul-harb eventually lost its meaning. New terms were drawn up
to describe novel ways of perceiving non-Muslim territories. For instance, areas like dar-ul-mua'hada (land of compact)
and dar-ul-sulh (land of treaty) referred to countries which, though under a secular regime, maintained peaceful and
cooperative relations with Muslim States, having been bound to each other by treaty or agreement. Dar-ul-aman (land
of order), on the other hand, referred to countries which, though not bound by treaty with Muslim States, maintained
freedom of religion for Muslims.28

It thus appears that the "compact rights entrenchment" emanating from the regime of dar-ul-mua'hada and dar-ul-
sulh simply refers to all other agreements between the MILF and the Philippine government - the Philippines being the
land of compact and peace agreement - that partake of the nature of a treaty device, "treaty" being broadly defined as
"any solemn agreement in writing that sets out understandings, obligations, and benefits for both parties which provides
for a framework that elaborates the principles declared in the [MOA-AD]."29

The MOA-AD states that the Parties "HAVE AGREED AND ACKNOWLEDGED AS FOLLOWS," and starts with its main
body.

The main body of the MOA-AD is divided into four strands, namely, Concepts and Principles, Territory,
Resources, and Governance.

A. CONCEPTS AND PRINCIPLES

This strand begins with the statement that it is "the birthright of all Moros and all Indigenous peoples of Mindanao to
identify themselves and be accepted as ‘Bangsamoros.'" It defines "Bangsamoro people" as the natives or original
inhabitants of Mindanao and its adjacent islands including Palawan and the Sulu archipelago at the time of conquest or
colonization, and their descendants whether mixed or of full blood, including their spouses.30

Thus, the concept of "Bangsamoro," as defined in this strand of the MOA-AD, includes not only "Moros" as traditionally
understood even by Muslims,31 but all indigenous peoples of Mindanao and its adjacent islands. The MOA-AD adds that
the freedom of choice of indigenous peoples shall be respected. What this freedom of choice consists in has not been
specifically defined.

The MOA-AD proceeds to refer to the "Bangsamoro homeland," the ownership of which is vested exclusively in the
Bangsamoro people by virtue of their prior rights of occupation.32 Both parties to the MOA-AD acknowledge that ancestral
domain does not form part of the public domain.33

The Bangsamoro people are acknowledged as having the right to self-governance, which right is said to be rooted on
ancestral territoriality exercised originally under the suzerain authority of their sultanates and the Pat a Pangampong ku
Ranaw. The sultanates were described as states or "karajaan/kadatuan" resembling a body politic endowed with all the
elements of a nation-state in the modern sense.34

The MOA-AD thus grounds the right to self-governance of the Bangsamoro people on the past suzerain authority of the
sultanates. As gathered, the territory defined as the Bangsamoro homeland was ruled by several sultanates and,
specifically in the case of the Maranao, by the Pat a Pangampong ku Ranaw, a confederation of independent principalities
(pangampong) each ruled by datus and sultans, none of whom was supreme over the others. 35

The MOA-AD goes on to describe the Bangsamoro people as "the ‘First Nation' with defined territory and with a system of
government having entered into treaties of amity and commerce with foreign nations."

The term "First Nation" is of Canadian origin referring to the indigenous peoples of that territory, particularly those known
as Indians. In Canada, each of these indigenous peoples is equally entitled to be called "First Nation," hence, all of them
are usually described collectively by the plural "First Nations." 36 To that extent, the MOA-AD, by identifying the
Bangsamoro people as "the First Nation" - suggesting its exclusive entitlement to that designation - departs from the
Canadian usage of the term.

The MOA-AD then mentions for the first time the "Bangsamoro Juridical Entity" (BJE) to which it grants the authority
and jurisdiction over the Ancestral Domain and Ancestral Lands of the Bangsamoro.37

B. TERRITORY

The territory of the Bangsamoro homeland is described as the land mass as well as the maritime, terrestrial, fluvial and
alluvial domains, including the aerial domain and the atmospheric space above it, embracing the Mindanao-Sulu-Palawan
geographic region.38

More specifically, the core of the BJE is defined as the present geographic area of the ARMM - thus constituting the
following areas: Lanao del Sur, Maguindanao, Sulu, Tawi-Tawi, Basilan, and Marawi City. Significantly, this core
also includes certain municipalities of Lanao del Norte that voted for inclusion in the ARMM in the 2001 plebiscite.39

Outside of this core, the BJE is to cover other provinces, cities, municipalities and barangays, which are grouped into two
categories, Category A and Category B. Each of these areas is to be subjected to a plebiscite to be held on different
dates, years apart from each other. Thus, Category A areas are to be subjected to a plebiscite not later than twelve (12)
months following the signing of the MOA-AD.40 Category B areas, also called "Special Intervention Areas," on the other
hand, are to be subjected to a plebiscite twenty-five (25) years from the signing of a separate agreement - the
Comprehensive Compact.41

The Parties to the MOA-AD stipulate that the BJE shall have jurisdiction over all natural resources within its
"internal waters," defined as extending fifteen (15) kilometers from the coastline of the BJE area; 42 that the BJE shall also
have "territorial waters," which shall stretch beyond the BJE internal waters up to the baselines of the Republic of the
Philippines (RP) south east and south west of mainland Mindanao; and that within these territorial waters, the BJE and the
"Central Government" (used interchangeably with RP) shall exercise joint jurisdiction, authority and management over all
natural resources.43 Notably, the jurisdiction over the internal waters is not similarly described as "joint."

The MOA-AD further provides for the sharing of minerals on the territorial waters between the Central Government and
the BJE, in favor of the latter, through production sharing and economic cooperation agreement. 44 The activities which the
Parties are allowed to conduct on the territorial waters are enumerated, among which are the exploration and utilization of
natural resources, regulation of shipping and fishing activities, and the enforcement of police and safety measures. 45 There
is no similar provision on the sharing of minerals and allowed activities with respect to the internal waters of the BJE.

C. RESOURCES

The MOA-AD states that the BJE is free to enter into any economic cooperation and trade relations with foreign countries
and shall have the option to establish trade missions in those countries. Such relationships and understandings, however,
are not to include aggression against the GRP. The BJE may also enter into environmental cooperation agreements. 46

The external defense of the BJE is to remain the duty and obligation of the Central Government. The Central Government
is also bound to "take necessary steps to ensure the BJE's participation in international meetings and events" like those of
the ASEAN and the specialized agencies of the UN. The BJE is to be entitled to participate in Philippine official missions
and delegations for the negotiation of border agreements or protocols for environmental protection and equitable sharing
of incomes and revenues involving the bodies of water adjacent to or between the islands forming part of the ancestral
domain.47

With regard to the right of exploring for, producing, and obtaining all potential sources of energy, petroleum, fossil fuel,
mineral oil and natural gas, the jurisdiction and control thereon is to be vested in the BJE "as the party having control
within its territorial jurisdiction." This right carries the proviso that, "in times of national emergency, when public interest so
requires," the Central Government may, for a fixed period and under reasonable terms as may be agreed upon by both
Parties, assume or direct the operation of such resources.48

The sharing between the Central Government and the BJE of total production pertaining to natural resources is to be
75:25 in favor of the BJE.49

The MOA-AD provides that legitimate grievances of the Bangsamoro people arising from any unjust dispossession of their
territorial and proprietary rights, customary land tenures, or their marginalization shall be acknowledged. Whenever
restoration is no longer possible, reparation is to be in such form as mutually determined by the Parties. 50

The BJE may modify or cancel the forest concessions, timber licenses, contracts or agreements, mining concessions,
Mineral Production and Sharing Agreements (MPSA), Industrial Forest Management Agreements (IFMA), and other land
tenure instruments granted by the Philippine Government, including those issued by the present ARMM.51

D. GOVERNANCE

The MOA-AD binds the Parties to invite a multinational third-party to observe and monitor the implementation of
the Comprehensive Compact. This compact is to embody the "details for the effective enforcement" and "the
mechanisms and modalities for the actual implementation" of the MOA-AD. The MOA-AD explicitly provides that the
participation of the third party shall not in any way affect the status of the relationship between the Central Government
and the BJE.52

The "associative" relationship


between the Central Government
and the BJE

The MOA-AD describes the relationship of the Central Government and the BJE as "associative," characterized by shared
authority and responsibility. And it states that the structure of governance is to be based on executive, legislative, judicial,
and administrative institutions with defined powers and functions in the Comprehensive Compact.

The MOA-AD provides that its provisions requiring "amendments to the existing legal framework" shall take effect upon
signing of the Comprehensive Compact and upon effecting the aforesaid amendments, with due regard to the non-
derogation of prior agreements and within the stipulated timeframe to be contained in the Comprehensive Compact. As
will be discussed later, much of the present controversy hangs on the legality of this provision.

The BJE is granted the power to build, develop and maintain its own institutions inclusive of civil service, electoral,
financial and banking, education, legislation, legal, economic, police and internal security force, judicial system and
correctional institutions, the details of which shall be discussed in the negotiation of the comprehensive compact.
As stated early on, the MOA-AD was set to be signed on August 5, 2008 by Rodolfo Garcia and Mohagher Iqbal,
Chairpersons of the Peace Negotiating Panels of the GRP and the MILF, respectively. Notably, the penultimate paragraph
of the MOA-AD identifies the signatories as "the representatives of the Parties," meaning the GRP and MILF themselves,
and not merely of the negotiating panels.53 In addition, the signature page of the MOA-AD states that it is "WITNESSED
BY" Datuk Othman Bin Abd Razak, Special Adviser to the Prime Minister of Malaysia, "ENDORSED BY" Ambassador
Sayed Elmasry, Adviser to Organization of the Islamic Conference (OIC) Secretary General and Special Envoy for Peace
Process in Southern Philippines, and SIGNED "IN THE PRESENCE OF" Dr. Albert G. Romulo, Secretary of Foreign
Affairs of RP and Dato' Seri Utama Dr. Rais Bin Yatim, Minister of Foreign Affairs, Malaysia, all of whom were scheduled
to sign the Agreement last August 5, 2008.

Annexed to the MOA-AD are two documents containing the respective lists cum maps of the provinces, municipalities, and
barangays under Categories A and B earlier mentioned in the discussion on the strand on TERRITORY.

IV. PROCEDURAL ISSUES

A. RIPENESS

The power of judicial review is limited to actual cases or controversies. 54 Courts decline to issue advisory opinions or to
resolve hypothetical or feigned problems, or mere academic questions. 55 The limitation of the power of judicial review to
actual cases and controversies defines the role assigned to the judiciary in a tripartite allocation of power, to assure that
the courts will not intrude into areas committed to the other branches of government. 56

An actual case or controversy involves a conflict of legal rights, an assertion of opposite legal claims, susceptible of
judicial resolution as distinguished from a hypothetical or abstract difference or dispute. There must be a contrariety of
legal rights that can be interpreted and enforced on the basis of existing law and jurisprudence. 57 The Court can decide the
constitutionality of an act or treaty only when a proper case between opposing parties is submitted for judicial
determination.58

Related to the requirement of an actual case or controversy is the requirement of ripeness. A question is ripe for
adjudication when the act being challenged has had a direct adverse effect on the individual challenging it. 59 For a case to
be considered ripe for adjudication, it is a prerequisite that something had then been accomplished or performed by either
branch before a court may come into the picture, 60 and the petitioner must allege the existence of an immediate or
threatened injury to itself as a result of the challenged action. 61 He must show that he has sustained or is immediately in
danger of sustaining some direct injury as a result of the act complained of. 62

The Solicitor General argues that there is no justiciable controversy that is ripe for judicial review in the present petitions,
reasoning that

The unsigned MOA-AD is simply a list of consensus points subject to further negotiations and legislative
enactments as well as constitutional processes aimed at attaining a final peaceful agreement. Simply put, the
MOA-AD remains to be a proposal that does not automatically create legally demandable rights and
obligations until the list of operative acts required have been duly complied with. x x x

xxxx

In the cases at bar, it is respectfully submitted that this Honorable Court has no authority to pass upon issues
based on hypothetical or feigned constitutional problems or interests with no concrete bases. Considering
the preliminary character of the MOA-AD, there are no concrete acts that could possibly violate petitioners' and
intervenors' rights since the acts complained of are mere contemplated steps toward the formulation of a final
peace agreement. Plainly, petitioners and intervenors' perceived injury, if at all, is merely imaginary and illusory
apart from being unfounded and based on mere conjectures. (Underscoring supplied)

The Solicitor General cites63 the following provisions of the MOA-AD:

TERRITORY

xxxx

2. Toward this end, the Parties enter into the following stipulations:

xxxx

d. Without derogating from the requirements of prior agreements, the Government stipulates to conduct and
deliver, using all possible legal measures, within twelve (12) months following the signing of the MOA-AD, a
plebiscite covering the areas as enumerated in the list and depicted in the map as Category A attached herein (the
"Annex"). The Annex constitutes an integral part of this framework agreement. Toward this end, the Parties shall
endeavor to complete the negotiations and resolve all outstanding issues on the Comprehensive Compact within
fifteen (15) months from the signing of the MOA-AD.

xxxx
GOVERNANCE

xxxx

7. The Parties agree that mechanisms and modalities for the actual implementation of this MOA-AD shall be spelt
out in the Comprehensive Compact to mutually take such steps to enable it to occur effectively.

Any provisions of the MOA-AD requiring amendments to the existing legal framework shall come into force upon
the signing of a Comprehensive Compact and upon effecting the necessary changes to the legal framework with
due regard to non-derogation of prior agreements and within the stipulated timeframe to be contained in the
Comprehensive Compact.64 (Underscoring supplied)

The Solicitor General's arguments fail to persuade.

Concrete acts under the MOA-AD are not necessary to render the present controversy ripe. In Pimentel, Jr. v.
Aguirre,65 this Court held:

x x x [B]y the mere enactment of the questioned law or the approval of the challenged action, the dispute is said to
have ripened into a judicial controversy even without any other overt act. Indeed, even a singular violation of the
Constitution and/or the law is enough to awaken judicial duty.

xxxx

By the same token, when an act of the President, who in our constitutional scheme is a coequal of Congress, is
seriously alleged to have infringed the Constitution and the laws x x x settling the dispute becomes the duty and
the responsibility of the courts.66

In Santa Fe Independent School District v. Doe,67 the United States Supreme Court held that the challenge to the
constitutionality of the school's policy allowing student-led prayers and speeches before games was ripe for adjudication,
even if no public prayer had yet been led under the policy, because the policy was being challenged as unconstitutional on
its face.68

That the law or act in question is not yet effective does not negate ripeness. For example, in New York v. United
States,69 decided in 1992, the United States Supreme Court held that the action by the State of New York challenging the
provisions of the Low-Level Radioactive Waste Policy Act was ripe for adjudication even if the questioned provision was
not to take effect until January 1, 1996, because the parties agreed that New York had to take immediate action to avoid
the provision's consequences.70

The present petitions pray for Certiorari, 71 Prohibition, and Mandamus. Certiorari and Prohibition are remedies granted by
law when any tribunal, board or officer has acted, in the case of certiorari, or is proceeding, in the case of prohibition,
without or in excess of its jurisdiction or with grave abuse of discretion amounting to lack or excess of
jurisdiction.72 Mandamus is a remedy granted by law when any tribunal, corporation, board, officer or person unlawfully
neglects the performance of an act which the law specifically enjoins as a duty resulting from an office, trust, or station, or
unlawfully excludes another from the use or enjoyment of a right or office to which such other is entitled. 73 Certiorari,
Mandamus and Prohibition are appropriate remedies to raise constitutional issues and to review and/or prohibit/nullify,
when proper, acts of legislative and executive officials. 74

The authority of the GRP Negotiating Panel is defined by Executive Order No. 3 (E.O. No. 3), issued on February 28,
2001.75 The said executive order requires that "[t]he government's policy framework for peace, including the systematic
approach and the administrative structure for carrying out the comprehensive peace process x x x be governed by this
Executive Order."76

The present petitions allege that respondents GRP Panel and PAPP Esperon drafted the terms of the MOA-AD without
consulting the local government units or communities affected, nor informing them of the proceedings. As will be
discussed in greater detail later, such omission, by itself, constitutes a departure by respondents from their mandate under
E.O. No. 3.

Furthermore, the petitions allege that the provisions of the MOA-AD violate the Constitution. The MOA-AD provides that
"any provisions of the MOA-AD requiring amendments to the existing legal framework shall come into force upon the
signing of a Comprehensive Compact and upon effecting the necessary changes to the legal framework," implying an
amendment of the Constitution to accommodate the MOA-AD. This stipulation, in effect, guaranteed to the MILF the
amendment of the Constitution.  Such act constitutes another violation of its authority . Again, these points will be discussed
in more detail later.

As the petitions allege acts or omissions on the part of respondent that exceed their authority, by violating their duties
under E.O. No. 3 and the provisions of the Constitution and statutes, the petitions make a prima facie case for Certiorari,
Prohibition, and Mandamus, and an actual case or controversy ripe for adjudication exists. When an act of a branch of
government is seriously alleged to have infringed the Constitution, it becomes not only the right but in fact the
duty of the judiciary to settle the dispute.77

B. LOCUS STANDI
For a party to have locus standi, one must allege "such a personal stake in the outcome of the controversy as to assure
that concrete adverseness which sharpens the presentation of issues upon which the court so largely depends for
illumination of difficult constitutional questions." 78

Because constitutional cases are often public actions in which the relief sought is likely to affect other persons, a
preliminary question frequently arises as to this interest in the constitutional question raised. 79

When suing as a citizen, the person complaining must allege that he has been or is about to be denied some right or
privilege to which he is lawfully entitled or that he is about to be subjected to some burdens or penalties by reason of the
statute or act complained of.80 When the issue concerns a public right, it is sufficient that the petitioner is a citizen and has
an interest in the execution of the laws.81

For a taxpayer, one is allowed to sue where there is an assertion that public funds are illegally disbursed or deflected to an
illegal purpose, or that there is a wastage of public funds through the enforcement of an invalid or unconstitutional
law.82 The Court retains discretion whether or not to allow a taxpayer's suit. 83

In the case of a legislator or member of Congress, an act of the Executive that injures the institution of Congress causes a
derivative but nonetheless substantial injury that can be questioned by legislators. A member of the House of
Representatives has standing to maintain inviolate the prerogatives, powers and privileges vested by the Constitution in
his office.84

An organization may be granted standing to assert the rights of its members, 85 but the mere invocation by the Integrated
Bar of the Philippines or any member of the legal profession of the duty to preserve the rule of law does not suffice to
clothe it with standing.86

As regards a local government unit (LGU), it can seek relief in order to protect or vindicate an interest of its own, and of
the other LGUs.87

Intervenors, meanwhile, may be given legal standing upon showing of facts that satisfy the requirements of the law
authorizing intervention,88 such as a legal interest in the matter in litigation, or in the success of either of the parties.

In any case, the Court has discretion to relax the procedural technicality on locus standi, given the liberal attitude it has
exercised, highlighted in the case of David v. Macapagal-Arroyo,89 where technicalities of procedure were brushed aside,
the constitutional issues raised being of paramount public interest or of transcendental importance deserving the attention
of the Court in view of their seriousness, novelty and weight as precedents. 90 The Court's forbearing stance on locus
standi on issues involving constitutional issues has for its purpose the protection of fundamental rights.

In not a few cases, the Court, in keeping with its duty under the Constitution to determine whether the other branches of
government have kept themselves within the limits of the Constitution and the laws and have not abused the discretion
given them, has brushed aside technical rules of procedure. 91

In the petitions at bar, petitioners Province of North Cotabato (G.R. No. 183591) Province of Zamboanga del
Norte (G.R. No. 183951), City of Iligan (G.R. No. 183893) and City of Zamboanga (G.R. No. 183752) and petitioners-in-
intervention Province of Sultan Kudarat, City of Isabela and Municipality of Linamon have locus standi in view of the
direct and substantial injury that they, as LGUs, would suffer as their territories, whether in whole or in part, are to be
included in the intended domain of the BJE. These petitioners allege that they did not vote for their inclusion in the ARMM
which would be expanded to form the BJE territory. Petitioners' legal standing is thus beyond doubt.

In G.R. No. 183962, petitioners Ernesto Maceda, Jejomar Binay and Aquilino Pimentel III would have no standing as
citizens and taxpayers for their failure to specify that they would be denied some right or privilege or there would be
wastage of public funds. The fact that they are a former Senator, an incumbent mayor of Makati City, and a resident of
Cagayan de Oro, respectively, is of no consequence. Considering their invocation of the transcendental importance of the
issues at hand, however, the Court grants them standing.

Intervenors Franklin Drilon and Adel Tamano, in alleging their standing as taxpayers, assert that government funds
would be expended for the conduct of an illegal and unconstitutional plebiscite to delineate the BJE territory. On that score
alone, they can be given legal standing. Their allegation that the issues involved in these petitions are of "undeniable
transcendental importance" clothes them with added basis for their personality to intervene in these petitions.

With regard to Senator Manuel Roxas, his standing is premised on his being a member of the Senate and a citizen to
enforce compliance by respondents of the public's constitutional right to be informed of the MOA-AD, as well as on a
genuine legal interest in the matter in litigation, or in the success or failure of either of the parties. He thus possesses the
requisite standing as an intervenor.

With respect to Intervenors Ruy Elias Lopez, as a former congressman of the 3rd district of Davao City, a taxpayer and a
member of the Bagobo tribe; Carlo B. Gomez, et al., as members of the IBP Palawan chapter, citizens and
taxpayers; Marino Ridao, as taxpayer, resident and member of the Sangguniang Panlungsod of Cotabato City; and Kisin
Buxani, as taxpayer, they failed to allege any proper legal interest in the present petitions. Just the same, the Court
exercises its discretion to relax the procedural technicality on locus standi given the paramount public interest in the issues
at hand.
Intervening respondents Muslim Multi-Sectoral Movement for Peace and Development, an advocacy group for justice
and the attainment of peace and prosperity in Muslim Mindanao; and Muslim Legal Assistance Foundation Inc., a non-
government organization of Muslim lawyers, allege that they stand to be benefited or prejudiced, as the case may be, in
the resolution of the petitions concerning the MOA-AD, and prays for the denial of the petitions on the grounds therein
stated. Such legal interest suffices to clothe them with standing.

B. MOOTNESS

Respondents insist that the present petitions have been rendered moot with the satisfaction of all the reliefs prayed for by
petitioners and the subsequent pronouncement of the Executive Secretary that "[n]o matter what the Supreme Court
ultimately decides[,] the government will not sign the MOA."92

In lending credence to this policy decision, the Solicitor General points out that the President had already disbanded the
GRP Peace Panel.93

In David v. Macapagal-Arroyo,94 this Court held that the "moot and academic" principle not being a magical formula that
automatically dissuades courts in resolving a case, it will decide cases, otherwise moot and academic, if it finds that (a)
there is a grave violation of the Constitution; 95 (b) the situation is of exceptional character and paramount public interest is
involved;96 (c) the constitutional issue raised requires formulation of controlling principles to guide the bench, the bar, and
the public;97 and (d) the case is capable of repetition yet evading review. 98

Another exclusionary circumstance that may be considered is where there is a voluntary cessation of the activity
complained of by the defendant or doer. Thus, once a suit is filed and the doer voluntarily ceases the challenged conduct,
it does not automatically deprive the tribunal of power to hear and determine the case and does not render the case moot
especially when the plaintiff seeks damages or prays for injunctive relief against the possible recurrence of the violation. 99

The present petitions fall squarely into these exceptions to thus thrust them into the domain of judicial review. The grounds
cited above in David are just as applicable in the present cases as they were, not only in David, but also in Province of
Batangas v. Romulo100 and Manalo v. Calderon101 where the Court similarly decided them on the merits, supervening
events that would ordinarily have rendered the same moot notwithstanding.

Petitions not mooted

Contrary then to the asseverations of respondents, the non-signing of the MOA-AD and the eventual dissolution of the
GRP Peace Panel did not moot the present petitions. It bears emphasis that the signing of the MOA-AD did not push
through due to the Court's issuance of a Temporary Restraining Order.

Contrary too to respondents' position, the MOA-AD cannot be considered a mere "list of consensus points," especially
given its nomenclature, the need to have it signed or initialed by all the parties concerned on August 5, 2008, and
the far-reaching Constitutional implications of these "consensus points," foremost of which is the creation of the BJE.

In fact, as what will, in the main, be discussed, there is a commitment on the part of respondents to amend and effect
necessary changes to the existing legal framework for certain provisions of the MOA-AD to take effect.
Consequently, the present petitions are not confined to the terms and provisions of the MOA-AD, but to other  on-
going and future negotiations and agreements necessary for its realization . The petitions have not, therefore, been
rendered moot and academic simply by the public disclosure of the MOA-AD, 102 the manifestation that it will not be signed
as well as the disbanding of the GRP Panel not withstanding.

Petitions are imbued with paramount public interest

There is no gainsaying that the petitions are imbued with paramount public interest, involving a significant part of the
country's territory and the wide-ranging political modifications of affected LGUs. The assertion that the MOA-AD is
subject to further legal enactments including possible Constitutional amendments more than ever provides
impetus for the Court to formulate controlling principles to guide the bench, the bar, the public and, in this case,
the government and its negotiating entity.

Respondents cite Suplico v. NEDA, et al.103 where the Court did not "pontificat[e] on issues which no longer legitimately
constitute an actual case or controversy [as this] will do more harm than good to the nation as a whole."

The present petitions must be differentiated from Suplico. Primarily, in Suplico, what was assailed and eventually
cancelled was a stand-alone government procurement contract for a national broadband network involving a one-time
contractual relation between two parties-the government and a private foreign corporation. As the issues therein involved
specific government procurement policies and standard principles on contracts, the majority opinion in Suplico found
nothing exceptional therein, the factual circumstances being peculiar only to the transactions and parties involved in the
controversy.

The MOA-AD is part of a series of agreements

In the present controversy, the MOA-AD is a significant part of a series of agreements necessary to carry out the
Tripoli Agreement 2001. The MOA-AD which dwells on the Ancestral Domain Aspect of said Tripoli Agreement is the third
such component to be undertaken following the implementation of the Security Aspect in August 2001 and
the Humanitarian, Rehabilitation and Development Aspect in May 2002.

Accordingly, even if the Executive Secretary, in his Memorandum of August 28, 2008 to the Solicitor General, has stated
that "no matter what the Supreme Court ultimately decides[,] the government will not sign the MOA[-AD]," mootness will
not set in in light of the terms of the Tripoli Agreement 2001.

Need to formulate principles-guidelines

Surely, the present MOA-AD can be renegotiated or another one will be drawn up to carry out the Ancestral Domain
Aspect of the Tripoli Agreement 2001, in another or in any form, which could contain similar or significantly drastic
provisions. While the Court notes the word of the Executive Secretary that the government "is committed to securing an
agreement that is both constitutional and equitable because that is the only way that long-lasting peace can be assured," it
is minded to render a decision on the merits in the present petitions to formulate controlling principles to guide the
bench, the bar, the public and, most especially, the government in negotiating with the MILF regarding Ancestral
Domain.

Respondents invite the Court's attention to the separate opinion of then Chief Justice Artemio Panganiban in Sanlakas v.
Reyes104 in which he stated that the doctrine of "capable of repetition yet evading review" can override mootness,
"provided the party raising it in a proper case has been and/or continue to be prejudiced or damaged as a direct result of
their issuance." They contend that the Court must have jurisdiction over the subject matter for the doctrine to be invoked.

The present petitions all contain prayers for Prohibition over which this Court exercises original jurisdiction. While G.R. No.
183893 (City of Iligan v. GRP) is a petition for Injunction and Declaratory Relief, the Court will treat it as one for Prohibition
as it has far reaching implications and raises questions that need to be resolved. 105 At all events, the Court has jurisdiction
over most if not the rest of the petitions.

Indeed, the present petitions afford a proper venue for the Court to again apply the doctrine immediately referred to as
what it had done in a number of landmark cases.106 There is a reasonable expectation that petitioners, particularly the
Provinces of North Cotabato, Zamboanga del Norte and Sultan Kudarat, the Cities of Zamboanga, Iligan and Isabela, and
the Municipality of Linamon, will again be subjected to the same problem in the future as respondents' actions are capable
of repetition, in another or any form.

It is with respect to the prayers for Mandamus that the petitions have become moot, respondents having, by Compliance
of August 7, 2008, provided this Court and petitioners with official copies of the final draft of the MOA-AD and its annexes.
Too, intervenors have been furnished, or have procured for themselves, copies of the MOA-AD.

V. SUBSTANTIVE ISSUES

As culled from the Petitions and Petitions-in-Intervention, there are basically two SUBSTANTIVE issues to be resolved,
one relating to the manner in which the MOA-AD was negotiated and finalized, the other relating to its provisions, viz:

1. Did respondents violate constitutional and statutory provisions on public consultation and the right to information when
they negotiated and later initialed the MOA-AD?

2. Do the contents of the MOA-AD violate the Constitution and the laws?

ON THE FIRST SUBSTANTIVE ISSUE

Petitioners invoke their constitutional right to information on matters of public concern, as provided in Section 7,
Article III on the Bill of Rights:

Sec. 7. The right of the people to information on matters of public concern shall be recognized. Access to official
records, and to documents, and papers pertaining to official acts, transactions, or decisions, as well as to
government research data used as basis for policy development, shall be afforded the citizen, subject to such
limitations as may be provided by law.107

As early as 1948, in Subido v. Ozaeta,108 the Court has recognized the statutory right to examine and inspect public
records, a right which was eventually accorded constitutional status.

The right of access to public documents, as enshrined in both the 1973 Constitution and the 1987 Constitution, has been
recognized as a self-executory constitutional right. 109

In the 1976 case of Baldoza v. Hon. Judge Dimaano,110 the Court ruled that access to public records is predicated on the
right of the people to acquire information on matters of public concern since, undoubtedly, in a democracy, the pubic has a
legitimate interest in matters of social and political significance.

x x x The incorporation of this right in the Constitution is a recognition of the fundamental role of free exchange of
information in a democracy. There can be no realistic perception by the public of the nation's problems, nor a meaningful
democratic decision-making if they are denied access to information of general interest. Information is needed to enable
the members of society to cope with the exigencies of the times. As has been aptly observed: "Maintaining the flow of
such information depends on protection for both its acquisition and its dissemination since, if either process is interrupted,
the flow inevitably ceases." x x x111

In the same way that free discussion enables members of society to cope with the exigencies of their time, access to
information of general interest aids the people in democratic decision-making by giving them a better perspective of the
vital issues confronting the nation112 so that they may be able to criticize and participate in the affairs of the government in
a responsible, reasonable and effective manner. It is by ensuring an unfettered and uninhibited exchange of ideas among
a well-informed public that a government remains responsive to the changes desired by the people. 113

The MOA-AD is a matter of public concern

That the subject of the information sought in the present cases is a matter of public concern 114 faces no serious challenge.
In fact, respondents admit that the MOA-AD is indeed of public concern.115 In previous cases, the Court found that the
regularity of real estate transactions entered in the Register of Deeds, 116 the need for adequate notice to the public of the
various laws,117 the civil service eligibility of a public employee,118 the proper management of GSIS funds allegedly used to
grant loans to public officials,119 the recovery of the Marcoses' alleged ill-gotten wealth, 120 and the identity of party-list
nominees,121 among others, are matters of public concern. Undoubtedly, the MOA-AD subject of the present cases is of
public concern, involving as it does the sovereignty and territorial integrity of the State, which directly affects the
lives of the public at large.

Matters of public concern covered by the right to information include steps and negotiations leading to the consummation
of the contract. In not distinguishing as to the executory nature or commercial character of agreements, the Court has
categorically ruled:

x x x [T]he right to information "contemplates inclusion of negotiations leading to the consummation of the
transaction." Certainly, a consummated contract is not a requirement for the exercise of the right to information.
Otherwise, the people can never exercise the right if no contract is consummated, and if one is consummated, it
may be too late for the public to expose its defects.

Requiring a consummated contract will keep the public in the dark until the contract, which may be grossly
disadvantageous to the government or even illegal, becomes fait accompli. This negates the State policy of full
transparency on matters of public concern, a situation which the framers of the Constitution could not have
intended. Such a requirement will prevent the citizenry from participating in the public discussion of
any proposed contract, effectively truncating a basic right enshrined in the Bill of Rights. We can allow neither an
emasculation of a constitutional right, nor a retreat by the State of its avowed "policy of full disclosure of all its
transactions involving public interest." 122 (Emphasis and italics in the original)

Intended as a "splendid symmetry"123 to the right to information under the Bill of Rights is the policy of public disclosure
under Section 28, Article II of the Constitution reading:

Sec. 28. Subject to reasonable conditions prescribed by law, the State adopts and implements a policy of full
public disclosure of all its transactions involving public interest. 124

The policy of full public disclosure enunciated in above-quoted Section 28 complements the right of access to information


on matters of public concern found in the Bill of Rights. The right to information guarantees the right of the people to
demand information, while Section 28 recognizes the duty of officialdom to give information even if nobody demands.125

The policy of public disclosure establishes a concrete ethical principle for the conduct of public affairs in a genuinely open
democracy, with the people's right to know as the centerpiece. It is a mandate of the State to be accountable by following
such policy.126 These provisions are vital to the exercise of the freedom of expression and essential to hold public officials
at all times accountable to the people.127

Whether Section 28 is self-executory, the records of the deliberations of the Constitutional Commission so disclose:

MR. SUAREZ. And since this is not self-executory, this policy will not be enunciated or will not be in force and
effect until after Congress shall have provided it.

MR. OPLE. I expect it to influence the climate of public ethics immediately but, of course, the implementing law will
have to be enacted by Congress, Mr. Presiding Officer. 128

The following discourse, after Commissioner Hilario Davide, Jr., sought clarification on the issue, is enlightening.

MR. DAVIDE. I would like to get some clarifications on this. Mr. Presiding Officer, did I get the Gentleman correctly
as having said that this is not a self-executing provision? It would require a legislation by Congress to implement?

MR. OPLE. Yes. Originally, it was going to be self-executing, but I accepted an amendment from Commissioner
Regalado, so that the safeguards on national interest are modified by the clause "as may be provided by law"

MR. DAVIDE. But as worded, does it not mean that this will immediately take effect and Congress may
provide for reasonable safeguards on the sole ground national interest?
MR. OPLE. Yes. I think so, Mr. Presiding Officer, I said earlier that it should immediately influence the
climate of the conduct of public affairs but, of course, Congress here may no longer pass a law revoking it, or if
this is approved, revoking this principle, which is inconsistent with this policy. 129 (Emphasis supplied)

Indubitably, the effectivity of the policy of public disclosure need not await the passing of a statute. As Congress
cannot revoke this principle, it is merely directed to provide for "reasonable safeguards." The complete and effective
exercise of the right to information necessitates that its complementary provision on public disclosure derive the same self-
executory nature. Since both provisions go hand-in-hand, it is absurd to say that the broader 130 right to information on
matters of public concern is already enforceable while the correlative duty of the State to disclose its transactions involving
public interest is not enforceable until there is an enabling law. Respondents cannot thus point to the absence of an
implementing legislation as an excuse in not effecting such policy.

An essential element of these freedoms is to keep open a continuing dialogue or process of communication between the
government and the people. It is in the interest of the State that the channels for free political discussion be maintained to
the end that the government may perceive and be responsive to the people's will. 131 Envisioned to be corollary to the twin
rights to information and disclosure is the design for feedback mechanisms.

MS. ROSARIO BRAID. Yes. And lastly, Mr. Presiding Officer, will the people be able to participate? Will the
government provide feedback mechanisms so that the people can participate and can react where the
existing media facilities are not able to provide full feedback mechanisms to the government? I suppose
this will be part of the government implementing operational mechanisms.

MR. OPLE. Yes. I think through their elected representatives and that is how these courses take place. There is a
message and a feedback, both ways.

xxxx

MS. ROSARIO BRAID. Mr. Presiding Officer, may I just make one last sentence?

I think when we talk about the feedback network, we are not talking about public officials but also network
of private business o[r] community-based organizations that will be reacting. As a matter of fact, we will put
more credence or credibility on the private network of volunteers and voluntary community-based organizations.
So I do not think we are afraid that there will be another OMA in the making. 132 (Emphasis supplied)

The imperative of a public consultation, as a species of the right to information, is evident in the "marching orders" to
respondents. The mechanics for the duty to disclose information and to conduct public consultation regarding the peace
agenda and process is manifestly provided by E.O. No. 3. 133 The preambulatory clause of E.O. No. 3 declares that there is
a need to further enhance the contribution of civil society to the comprehensive peace process by institutionalizing the
people's participation.

One of the three underlying principles of the comprehensive peace process is that it "should be community-based,
reflecting the sentiments, values and principles important to all Filipinos" and "shall be defined not by the government
alone, nor by the different contending groups only, but by all Filipinos as one community." 134 Included as a component of
the comprehensive peace process is consensus-building and empowerment for peace, which includes "continuing
consultations on both national and local levels to build consensus for a peace agenda and process, and the mobilization
and facilitation of people's participation in the peace process."135

Clearly, E.O. No. 3 contemplates not just the conduct of a plebiscite to effectuate "continuing" consultations,
contrary to respondents' position that plebiscite is "more than sufficient consultation."136

Further, E.O. No. 3 enumerates the functions and responsibilities of the PAPP, one of which is to "[c]onduct regular
dialogues with the National Peace Forum (NPF) and other peace partners to seek relevant information, comments,
recommendations as well as to render appropriate and timely reports on the progress of the comprehensive peace
process."137 E.O. No. 3 mandates the establishment of the NPF to be "the principal forum for the PAPP to consult with and
seek advi[c]e from the peace advocates, peace partners and concerned sectors of society on both national and local
levels, on the implementation of the comprehensive peace process, as well as for government[-]civil society dialogue and
consensus-building on peace agenda and initiatives." 138

In fine, E.O. No. 3 establishes petitioners' right to be consulted on the peace agenda, as a corollary to the
constitutional right to information and disclosure.

PAPP Esperon committed grave abuse of discretion

The PAPP committed grave abuse of discretion  when he failed to carry out the pertinent consultation. The furtive
process by which the MOA-AD was designed and crafted  runs contrary to and in excess of the legal authority , and
amounts to a whimsical, capricious, oppressive, arbitrary and despotic exercise thereof.

The Court may not, of course, require the PAPP to conduct the consultation in a particular way or manner. It may,
however, require him to comply with the law and discharge the functions within the authority granted by the President.139
Petitioners are not claiming a seat at the negotiating table, contrary to respondents' retort in justifying the denial of
petitioners' right to be consulted. Respondents' stance manifests the manner by which they treat the salient provisions of
E.O. No. 3 on people's participation. Such disregard of the express mandate of the President is not much different from
superficial conduct toward token provisos that border on classic lip service. 140 It illustrates a gross evasion of positive duty
and a virtual refusal to perform the duty enjoined.

As for respondents' invocation of the doctrine of executive privilege, it is not tenable under the premises. The argument
defies sound reason when contrasted with E.O. No. 3's explicit provisions on continuing consultation and dialogue on
both national and local levels. The executive order even recognizes the exercise of the public's right even before the
GRP makes its official recommendations or before the government proffers its definite propositions.141 It bear emphasis
that E.O. No. 3 seeks to elicit relevant advice, information, comments and recommendations from the people through
dialogue.

AT ALL EVENTS, respondents effectively waived the defense of executive privilege in view of their unqualified disclosure
of the official copies of the final draft of the MOA-AD. By unconditionally complying with the Court's August 4, 2008
Resolution, without a prayer for the document's disclosure in camera, or without a manifestation that it was complying
therewith ex abundante ad cautelam.

Petitioners' assertion that the Local Government Code (LGC) of 1991 declares it a State policy to "require all national
agencies and offices to conduct periodic consultations with appropriate local government units, non-governmental and
people's organizations, and other concerned sectors of the community before any project or program is implemented in
their respective jurisdictions"142 is well-taken. The LGC chapter on intergovernmental relations puts flesh into this avowed
policy:

Prior Consultations Required. - No project or program shall be implemented by government authorities  unless the
consultations mentioned in Sections 2 (c) and 26 hereof are complied with, and prior approval of the sanggunian
concerned is obtained: Provided, That occupants in areas where such projects are to be implemented shall not be
evicted unless appropriate relocation sites have been provided, in accordance with the provisions of the
Constitution.143 (Italics and underscoring supplied)

In Lina, Jr. v. Hon. Paño,144 the Court held that the above-stated policy and above-quoted provision of the LGU apply only
to national programs or projects which are to be implemented in a particular local community. Among the programs and
projects covered are those that are critical to the environment and human ecology including those that may call for
the eviction of a particular group of people residing in the locality where these will be implemented.145 The MOA-AD is one
peculiar program that unequivocally and unilaterally vests ownership of a vast territory to the Bangsamoro
people,146 which could pervasively and drastically result to the diaspora or displacement of a great number of
inhabitants from their total environment.

With respect to the indigenous cultural communities/indigenous peoples (ICCs/IPs), whose interests are represented
herein by petitioner Lopez and are adversely affected by the MOA-AD, the ICCs/IPs have, under the IPRA, the right to
participate fully at all levels of decision-making in matters which may affect their rights, lives and destinies. 147 The MOA-
AD, an instrument recognizing ancestral domain, failed to justify its non-compliance with the clear-cut mechanisms
ordained in said Act,148 which entails, among other things, the observance of the free and prior informed consent of the
ICCs/IPs.

Notably, the IPRA does not grant the Executive Department or any government agency the power to delineate and
recognize an ancestral domain claim by mere agreement or compromise. The recognition of the ancestral domain is
the raison d'etre of the MOA-AD, without which all other stipulations or "consensus points" necessarily must fail. In
proceeding to make a sweeping declaration on ancestral domain, without complying with the IPRA, which is cited as one
of the TOR of the MOA-AD, respondents clearly transcended the boundaries of their authority. As it seems, even the
heart of the MOA-AD is still subject to necessary changes to the legal framework. While paragraph 7 on Governance
suspends the effectivity of all provisions requiring changes to the legal framework, such clause is itself invalid, as will be
discussed in the following section.

Indeed, ours is an open society, with all the acts of the government subject to public scrutiny and available always to
public cognizance. This has to be so if the country is to remain democratic, with sovereignty residing in the people and all
government authority emanating from them. 149

ON THE SECOND SUBSTANTIVE ISSUE

With regard to the provisions of the MOA-AD, there can be no question that they cannot all be accommodated under the
present Constitution and laws. Respondents have admitted as much in the oral arguments before this Court, and the
MOA-AD itself recognizes the need to amend the existing legal framework to render effective at least some of its
provisions. Respondents, nonetheless, counter that the MOA-AD is free of any legal infirmity because any provisions
therein which are inconsistent with the present legal framework will not be effective until the necessary changes to that
framework are made. The validity of this argument will be considered later. For now, the Court shall pass upon how

The MOA-AD is inconsistent with the Constitution and laws as presently worded.

In general, the objections against the MOA-AD center on the extent of the powers conceded therein to the BJE. Petitioners
assert that the powers granted to the BJE exceed those granted to any local government under present laws, and even go
beyond those of the present ARMM. Before assessing some of the specific powers that would have been vested in the
BJE, however, it would be useful to turn first to a general idea that serves as a unifying link to the different provisions of
the MOA-AD, namely, the international law concept of association. Significantly, the MOA-AD explicitly alludes to this
concept, indicating that the Parties actually framed its provisions with it in mind.

Association is referred to in paragraph 3 on TERRITORY, paragraph 11 on RESOURCES, and paragraph 4 on


GOVERNANCE. It is in the last mentioned provision, however, that the MOA-AD most clearly uses it to describe
the envisioned relationship between the BJE and the Central Government.

4. The relationship between the Central Government and the Bangsamoro juridical entity shall
be associative characterized by shared authority and responsibility with a structure of governance based on
executive, legislative, judicial and administrative institutions with defined powers and functions in the
comprehensive compact. A period of transition shall be established in a comprehensive peace compact specifying
the relationship between the Central Government and the BJE. (Emphasis and underscoring supplied)

The nature of the "associative" relationship may have been intended to be defined more precisely in the still to be forged
Comprehensive Compact. Nonetheless, given that there is a concept of "association" in international law, and the MOA-
AD - by its inclusion of international law instruments in its TOR- placed itself in an international legal context, that concept
of association may be brought to bear in understanding the use of the term "associative" in the MOA-AD.

Keitner and Reisman state that

[a]n association is formed when two states of unequal power voluntarily establish durable links. In the basic
model, one state, the associate, delegates certain responsibilities to the other, the principal, while
maintaining its international status as a state. Free associations represent a middle ground between
integration and independence. x x x150 (Emphasis and underscoring supplied)

For purposes of illustration, the Republic of the Marshall Islands and the Federated States of Micronesia (FSM), formerly
part of the U.S.-administered Trust Territory of the Pacific Islands, 151 are associated states of the U.S. pursuant to a
Compact of Free Association. The currency in these countries is the U.S. dollar, indicating their very close ties with the
U.S., yet they issue their own travel documents, which is a mark of their statehood. Their international legal status as
states was confirmed by the UN Security Council and by their admission to UN membership.

According to their compacts of free association, the Marshall Islands and the FSM generally have the capacity to conduct
foreign affairs in their own name and right, such capacity extending to matters such as the law of the sea, marine
resources, trade, banking, postal, civil aviation, and cultural relations. The U.S. government, when conducting its foreign
affairs, is obligated to consult with the governments of the Marshall Islands or the FSM on matters which it (U.S.
government) regards as relating to or affecting either government.

In the event of attacks or threats against the Marshall Islands or the FSM, the U.S. government has the authority and
obligation to defend them as if they were part of U.S. territory. The U.S. government, moreover, has the option of
establishing and using military areas and facilities within these associated states and has the right to bar the military
personnel of any third country from having access to these territories for military purposes.

It bears noting that in U.S. constitutional and international practice, free association is understood as an international
association between sovereigns. The Compact of Free Association is a treaty which is subordinate to the associated
nation's national constitution, and each party may terminate the association consistent with the right of independence. It
has been said that, with the admission of the U.S.-associated states to the UN in 1990, the UN recognized that the
American model of free association is actually based on an underlying status of independence. 152

In international practice, the "associated state" arrangement has usually been used as a transitional device of former
colonies on their way to full independence. Examples of states that have passed through the status of associated states
as a transitional phase are Antigua, St. Kitts-Nevis-Anguilla, Dominica, St. Lucia, St. Vincent and Grenada. All have since
become independent states.153

Back to the MOA-AD, it contains many provisions which are consistent with the international legal concept of  association,
specifically the following: the BJE's capacity to enter into economic and trade relations with foreign countries, the
commitment of the Central Government to ensure the BJE's participation in meetings and events in the ASEAN and the
specialized UN agencies, and the continuing responsibility of the Central Government over external defense. Moreover,
the BJE's right to participate in Philippine official missions bearing on negotiation of border agreements, environmental
protection, and sharing of revenues pertaining to the bodies of water adjacent to or between the islands forming part of the
ancestral domain, resembles the right of the governments of FSM and the Marshall Islands to be consulted by the U.S.
government on any foreign affairs matter affecting them.

These provisions of the MOA indicate, among other things, that the Parties aimed to vest in the BJE the status of
an associated state or, at any rate, a status closely approximating it.

The concept of association is not recognized under the present Constitution

No province, city, or municipality, not even the ARMM, is recognized under our laws as having an "associative"
relationship with the national government. Indeed, the concept implies powers that go beyond anything ever granted by
the Constitution to any local or regional government. It also implies the recognition of the associated entity as a state. The
Constitution, however, does not contemplate any state in this jurisdiction other than the Philippine State, much less does it
provide for a transitory status that aims to prepare any part of Philippine territory for independence.

Even the mere concept animating many of the MOA-AD's provisions, therefore, already requires for its validity the
amendment of constitutional provisions, specifically the following provisions of Article X:

SECTION 1. The territorial and political subdivisions of the Republic of the Philippines are the provinces, cities,
municipalities, and barangays. There shall be autonomous regions in Muslim Mindanao and the Cordilleras as
hereinafter provided.

SECTION 15. There shall be created autonomous regions in Muslim Mindanao and in the Cordilleras consisting of
provinces, cities, municipalities, and geographical areas sharing common and distinctive historical and cultural
heritage, economic and social structures, and other relevant characteristics within the framework of this
Constitution and the national sovereignty as well as territorial integrity of the Republic of the Philippines.

The BJE is a far more powerful


entity than the autonomous region
recognized in the Constitution

It is not merely an expanded version of the ARMM, the status of its relationship with the national government being
fundamentally different from that of the ARMM. Indeed, BJE is a state in all but name as it meets the criteria of a state
laid down in the Montevideo Convention,154 namely, a permanent population, a defined territory, a government, and
a capacity to enter into relations with other states.

Even assuming arguendo that the MOA-AD would not necessarily sever any portion of Philippine territory, the spirit
animating it - which has betrayed itself by its use of the concept of association - runs counter to the national
sovereignty and territorial integrity of the Republic.

The defining concept underlying the relationship between the national government and the BJE being itself
contrary to the present Constitution, it is not surprising that many of the specific provisions of the MOA-AD on
the formation and powers of the BJE are in conflict with the Constitution and the laws.

Article X, Section 18 of the Constitution provides that "[t]he creation of the autonomous region shall be effective when
approved by a majority of the votes cast by the constituent units in a plebiscite called for the purpose, provided that only
provinces, cities, and geographic areas voting favorably in such plebiscite shall be included in the autonomous
region." (Emphasis supplied)

As reflected above, the BJE is more of a state than an autonomous region. But even assuming that it is covered by the
term "autonomous region" in the constitutional provision just quoted, the MOA-AD would still be in conflict with it. Under
paragraph 2(c) on TERRITORY in relation to 2(d) and 2(e), the present geographic area of the ARMM and, in addition, the
municipalities of Lanao del Norte which voted for inclusion in the ARMM during the 2001 plebiscite - Baloi, Munai,
Nunungan, Pantar, Tagoloan and Tangkal - are automatically part of the BJE without need of another plebiscite, in
contrast to the areas under Categories A and B mentioned earlier in the overview. That the present components of the
ARMM and the above-mentioned municipalities voted for inclusion therein in 2001, however, does  not render another
plebiscite unnecessary under the Constitution, precisely because what these areas voted for then was their inclusion in
the ARMM, not the BJE.

The MOA-AD, moreover, would not


comply with Article X, Section 20 of
the Constitution

since that provision defines the powers of autonomous regions as follows:

SECTION 20. Within its territorial jurisdiction and subject to the provisions of this Constitution and national laws,
the organic act of autonomous regions shall provide for legislative powers over:

(1) Administrative organization;

(2) Creation of sources of revenues;

(3) Ancestral domain and natural resources;

(4) Personal, family, and property relations;

(5) Regional urban and rural planning development;

(6) Economic, social, and tourism development;

(7) Educational policies;


(8) Preservation and development of the cultural heritage; and

(9) Such other matters as may be authorized by law for the promotion of the general welfare of the people of the
region. (Underscoring supplied)

Again on the premise that the BJE may be regarded as an autonomous region, the MOA-AD would require an amendment
that would expand the above-quoted provision. The mere passage of new legislation pursuant to sub-paragraph No. 9 of
said constitutional provision would not suffice, since any new law that might vest in the BJE the powers found in the MOA-
AD must, itself, comply with other provisions of the Constitution. It would not do, for instance, to merely pass legislation
vesting the BJE with treaty-making power in order to accommodate paragraph 4 of the strand on RESOURCES which
states: "The BJE is free to enter into any economic cooperation and trade relations with foreign countries: provided,
however, that such relationships and understandings do not include aggression against the Government of the Republic of
the Philippines x x x." Under our constitutional system, it is only the President who has that power. Pimentel v. Executive
Secretary155 instructs:

In our system of government, the President, being the head of state, is regarded as the sole organ and authority
in external relations and is the country's sole representative with foreign nations. As the chief architect of
foreign policy, the President acts as the country's mouthpiece with respect to international affairs. Hence, the
President is vested with the authority to deal with foreign states and governments, extend or withhold
recognition, maintain diplomatic relations, enter into treaties, and otherwise transact the business of
foreign relations. In the realm of treaty-making, the President has the sole authority to negotiate with other
states. (Emphasis and underscoring supplied)

Article II, Section 22 of the Constitution must also be amended if the scheme envisioned in the MOA-AD is to be
effected. That constitutional provision states: "The State recognizes and promotes the rights of indigenous cultural
communities within the framework of national unity and development." (Underscoring
supplied) An associative arrangement does not uphold national unity . While there may be a semblance of unity because of
the associative ties between the BJE and the national government, the act of placing a portion of Philippine territory in a
status which, in international practice, has generally been a preparation for independence, is certainly not conducive
to national unity.

Besides being irreconcilable with the Constitution, the MOA-AD is also inconsistent with prevailing statutory law,
among which are R.A. No. 9054156 or the Organic Act of the ARMM, and the IPRA.157

Article X, Section 3 of the Organic Act of the ARMM is a bar to the adoption of the definition of "Bangsamoro
people" used in the MOA-AD. Paragraph 1 on Concepts and Principles states:

1. It is the birthright of all Moros and all Indigenous peoples of Mindanao to identify themselves and be
accepted as "Bangsamoros". The Bangsamoro people refers to those who are natives or original inhabitants
of Mindanao and its adjacent islands including Palawan and the Sulu archipelago at the time of conquest or
colonization of its descendants whether mixed or of full blood. Spouses and their descendants are classified as
Bangsamoro. The freedom of choice of the Indigenous people shall be respected. (Emphasis and underscoring
supplied)

This use of the term Bangsamoro sharply contrasts with that found in the Article X, Section 3 of the Organic Act, which,
rather than lumping together the identities of the Bangsamoro and other indigenous peoples living in Mindanao,
clearly distinguishes between Bangsamoro people and Tribal peoples, as follows:

"As used in this Organic Act, the phrase "indigenous cultural community" refers to Filipino citizens residing in
the autonomous region who are:

(a) Tribal peoples. These are citizens whose social, cultural and economic conditions distinguish them from other
sectors of the national community; and

(b) Bangsa Moro people. These are citizens who are believers in Islam and who have retained some or all of
their own social, economic, cultural, and political institutions."

Respecting the IPRA, it lays down the prevailing procedure for the delineation and recognition of ancestral domains. The
MOA-AD's manner of delineating the ancestral domain of the Bangsamoro people is a clear departure from that
procedure. By paragraph 1 of Territory, the Parties simply agree that, subject to the delimitations in the agreed Schedules,
"[t]he Bangsamoro homeland and historic territory refer to the land mass as well as the maritime, terrestrial, fluvial and
alluvial domains, and the aerial domain, the atmospheric space above it, embracing the Mindanao-Sulu-Palawan
geographic region."

Chapter VIII of the IPRA, on the other hand, lays down a detailed procedure, as illustrated in the following provisions
thereof:

SECTION 52. Delineation Process. - The identification and delineation of ancestral domains shall be done in
accordance with the following procedures:

xxxx
b) Petition for Delineation. - The process of delineating a specific perimeter may be initiated by the NCIP with the
consent of the ICC/IP concerned, or through a Petition for Delineation filed with the NCIP, by a majority of the
members of the ICCs/IPs;

c) Delineation Proper. - The official delineation of ancestral domain boundaries including census of all community
members therein, shall be immediately undertaken by the Ancestral Domains Office upon filing of the application
by the ICCs/IPs concerned. Delineation will be done in coordination with the community concerned and shall at all
times include genuine involvement and participation by the members of the communities concerned;

d) Proof Required. - Proof of Ancestral Domain Claims shall include the testimony of elders or community under
oath, and other documents directly or indirectly attesting to the possession or occupation of the area since time
immemorial by such ICCs/IPs in the concept of owners which shall be any one (1) of the following authentic
documents:

1) Written accounts of the ICCs/IPs customs and traditions;

2) Written accounts of the ICCs/IPs political structure and institution;

3) Pictures showing long term occupation such as those of old improvements, burial grounds, sacred
places and old villages;

4) Historical accounts, including pacts and agreements concerning boundaries entered into by the
ICCs/IPs concerned with other ICCs/IPs;

5) Survey plans and sketch maps;

6) Anthropological data;

7) Genealogical surveys;

8) Pictures and descriptive histories of traditional communal forests and hunting grounds;

9) Pictures and descriptive histories of traditional landmarks such as mountains, rivers, creeks, ridges,
hills, terraces and the like; and

10) Write-ups of names and places derived from the native dialect of the community.

e) Preparation of Maps. - On the basis of such investigation and the findings of fact based thereon, the Ancestral
Domains Office of the NCIP shall prepare a perimeter map, complete with technical descriptions, and a description
of the natural features and landmarks embraced therein;

f) Report of Investigation and Other Documents. - A complete copy of the preliminary census and a report of
investigation, shall be prepared by the Ancestral Domains Office of the NCIP;

g) Notice and Publication. - A copy of each document, including a translation in the native language of the
ICCs/IPs concerned shall be posted in a prominent place therein for at least fifteen (15) days. A copy of the
document shall also be posted at the local, provincial and regional offices of the NCIP, and shall be published in a
newspaper of general circulation once a week for two (2) consecutive weeks to allow other claimants to file
opposition thereto within fifteen (15) days from date of such publication: Provided, That in areas where no such
newspaper exists, broadcasting in a radio station will be a valid substitute: Provided, further, That mere posting
shall be deemed sufficient if both newspaper and radio station are not available;

h) Endorsement to NCIP. - Within fifteen (15) days from publication, and of the inspection process, the Ancestral
Domains Office shall prepare a report to the NCIP endorsing a favorable action upon a claim that is deemed to
have sufficient proof. However, if the proof is deemed insufficient, the Ancestral Domains Office shall require the
submission of additional evidence: Provided, That the Ancestral Domains Office shall reject any claim that is
deemed patently false or fraudulent after inspection and verification: Provided, further, That in case of rejection,
the Ancestral Domains Office shall give the applicant due notice, copy furnished all concerned, containing the
grounds for denial. The denial shall be appealable to the NCIP: Provided, furthermore, That in cases where there
are conflicting claims among ICCs/IPs on the boundaries of ancestral domain claims, the Ancestral Domains
Office shall cause the contending parties to meet and assist them in coming up with a preliminary resolution of the
conflict, without prejudice to its full adjudication according to the section below.

xxxx

To remove all doubts about the irreconcilability of the MOA-AD with the present legal system, a discussion of not only the
Constitution and domestic statutes, but also of international law is in order, for

Article II, Section 2 of the Constitution states that the Philippines "adopts the generally accepted principles of
international law as part of the law of the land."
Applying this provision of the Constitution, the Court, in Mejoff v. Director of Prisons,158 held that the Universal Declaration
of Human Rights is part of the law of the land on account of which it ordered the release on bail of a detained alien of
Russian descent whose deportation order had not been executed even after two years. Similarly, the Court in Agustin v.
Edu159 applied the aforesaid constitutional provision to the 1968 Vienna Convention on Road Signs and Signals.

International law has long recognized the right to self-determination of "peoples," understood not merely as the entire
population of a State but also a portion thereof. In considering the question of whether the people of Quebec had a right to
unilaterally secede from Canada, the Canadian Supreme Court in REFERENCE RE SECESSION OF QUEBEC 160 had
occasion to acknowledge that "the right of a people to self-determination is now so widely recognized in international
conventions that the principle has acquired a status beyond ‘convention' and is considered a general principle of
international law."

Among the conventions referred to are the International Covenant on Civil and Political Rights 161 and the International
Covenant on Economic, Social and Cultural Rights 162 which state, in Article 1 of both covenants, that all peoples, by virtue
of the right of self-determination, "freely determine their political status and freely pursue their economic, social, and
cultural development."

The people's right to self-determination should not, however, be understood as extending to a unilateral right of secession.
A distinction should be made between the right of internal and external self-determination. REFERENCE RE SECESSION
OF QUEBEC is again instructive:

"(ii) Scope of the Right to Self-determination

126. The recognized sources of international law establish that the right to self-determination of a people is
normally fulfilled through internal self-determination - a people's pursuit of its political, economic, social
and cultural development within the framework of an existing state.  A right to external self-determination
(which in this case potentially takes the form of the assertion of a right to unilateral secession) arises in
only the most extreme of cases and, even then, under carefully defined circumstances. x x x

External self-determination can be defined as in the following statement from the Declaration on Friendly


Relations, supra, as

The establishment of a sovereign and independent State, the free association or integration with an
independent State or the emergence into any other political status freely determined by a people constitute
modes of implementing the right of self-determination by that people. (Emphasis added)

127. The international law principle of self-determination has evolved within a framework of respect for the
territorial integrity of existing states. The various international documents that support the existence of a
people's right to self-determination also contain parallel statements supportive of the conclusion that the exercise
of such a right must be sufficiently limited to prevent threats to an existing state's territorial integrity or the stability
of relations between sovereign states.

x x x x (Emphasis, italics and underscoring supplied)

The Canadian Court went on to discuss the exceptional cases in which the right to external self-determination can arise,
namely, where a people is under colonial rule, is subject to foreign domination or exploitation outside a colonial context,
and - less definitely but asserted by a number of commentators - is blocked from the meaningful exercise of its right to
internal self-determination. The Court ultimately held that the population of Quebec had no right to secession, as the same
is not under colonial rule or foreign domination, nor is it being deprived of the freedom to make political choices and
pursue economic, social and cultural development, citing that Quebec is equitably represented in legislative, executive and
judicial institutions within Canada, even occupying prominent positions therein.

The exceptional nature of the right of secession is further exemplified in the REPORT OF THE INTERNATIONAL
COMMITTEE OF JURISTS ON THE LEGAL ASPECTS OF THE AALAND ISLANDS QUESTION. 163 There, Sweden
presented to the Council of the League of Nations the question of whether the inhabitants of the Aaland Islands should be
authorized to determine by plebiscite if the archipelago should remain under Finnish sovereignty or be incorporated in the
kingdom of Sweden. The Council, before resolving the question, appointed an International Committee composed of three
jurists to submit an opinion on the preliminary issue of whether the dispute should, based on international law, be entirely
left to the domestic jurisdiction of Finland. The Committee stated the rule as follows:

x x x [I]n the absence of express provisions in international treaties, the right of disposing of national territory
is essentially an attribute of the sovereignty of every State. Positive International Law does not recognize
the right of national groups, as such, to separate themselves from the State of which they form part by the
simple expression of a wish, any more than it recognizes the right of other States to claim such a
separation. Generally speaking, the grant or refusal of the right to a portion of its population of determining
its own political fate by plebiscite or by some other method, is, exclusively, an attribute of the sovereignty
of every State which is definitively constituted. A dispute between two States concerning such a question,
under normal conditions therefore, bears upon a question which International Law leaves entirely to the domestic
jurisdiction of one of the States concerned. Any other solution would amount to an infringement of sovereign rights
of a State and would involve the risk of creating difficulties and a lack of stability which would not only be contrary
to the very idea embodied in term "State," but would also endanger the interests of the international community. If
this right is not possessed by a large or small section of a nation, neither can it be held by the State to which the
national group wishes to be attached, nor by any other State. (Emphasis and underscoring supplied)

The Committee held that the dispute concerning the Aaland Islands did not refer to a question which is left by international
law to the domestic jurisdiction of Finland, thereby applying the exception rather than the rule elucidated above. Its ground
for departing from the general rule, however, was a very narrow one, namely, the Aaland Islands agitation originated at a
time when Finland was undergoing drastic political transformation. The internal situation of Finland was, according to the
Committee, so abnormal that, for a considerable time, the conditions required for the formation of a sovereign State did
not exist. In the midst of revolution, anarchy, and civil war, the legitimacy of the Finnish national government was disputed
by a large section of the people, and it had, in fact, been chased from the capital and forcibly prevented from carrying out
its duties. The armed camps and the police were divided into two opposing forces. In light of these circumstances, Finland
was not, during the relevant time period, a "definitively constituted" sovereign state. The Committee, therefore, found that
Finland did not possess the right to withhold from a portion of its population the option to separate itself - a right which
sovereign nations generally have with respect to their own populations.

Turning now to the more specific category of indigenous peoples, this term has been used, in scholarship as well as
international, regional, and state practices, to refer to groups with distinct cultures, histories, and connections to land
(spiritual and otherwise) that have been forcibly incorporated into a larger governing society. These groups are regarded
as "indigenous" since they are the living descendants of pre-invasion inhabitants of lands now dominated by others.
Otherwise stated, indigenous peoples, nations, or communities are culturally distinctive groups that find themselves
engulfed by settler societies born of the forces of empire and conquest. 164 Examples of groups who have been regarded as
indigenous peoples are the Maori of New Zealand and the aboriginal peoples of Canada.

As with the broader category of "peoples," indigenous peoples situated within states do not have a general right to
independence or secession from those states under international law, 165 but they do have rights amounting to what was
discussed above as the right to internal self-determination.

In a historic development last September 13, 2007, the UN General Assembly adopted the United Nations Declaration on
the Rights of Indigenous Peoples (UN DRIP) through General Assembly Resolution 61/295. The vote was 143 to 4, the
Philippines being included among those in favor, and the four voting against being Australia, Canada, New Zealand, and
the U.S. The Declaration clearly recognized the right of indigenous peoples to self-determination, encompassing the
right to autonomy or self-government, to wit:

Article 3

Indigenous peoples have the right to self-determination. By virtue of that right they freely determine their political
status and freely pursue their economic, social and cultural development.

Article 4

Indigenous peoples, in exercising their right to self-determination, have the right to autonomy or self-


government in matters relating to their internal and local affairs, as well as ways and means for financing
their autonomous functions.

Article 5

Indigenous peoples have the right to maintain and strengthen their distinct political, legal, economic, social and
cultural institutions, while retaining their right to participate fully, if they so choose, in the political, economic, social
and cultural life of the State.

Self-government, as used in international legal discourse pertaining to indigenous peoples, has been understood as
equivalent to "internal self-determination."166 The extent of self-determination provided for in the UN DRIP is more
particularly defined in its subsequent articles, some of which are quoted hereunder:

Article 8

1. Indigenous peoples and individuals have the right not to be subjected to forced assimilation or destruction of
their culture.

2. States shall provide effective mechanisms for prevention of, and redress for:

(a) Any action which has the aim or effect of depriving them of their integrity as distinct peoples, or of
their cultural values or ethnic identities;

(b) Any action which has the aim or effect of dispossessing them of their lands, territories or resources;

(c) Any form of forced population transfer which has the aim or effect of violating or undermining any of
their rights;

(d) Any form of forced assimilation or integration;


(e) Any form of propaganda designed to promote or incite racial or ethnic discrimination directed against
them.

Article 21

1. Indigenous peoples have the right, without discrimination, to the improvement of their economic and social
conditions, including, inter alia, in the areas of education, employment, vocational training and retraining, housing,
sanitation, health and social security.

2. States shall take effective measures and, where appropriate, special measures to ensure continuing
improvement of their economic and social conditions. Particular attention shall be paid to the rights and special
needs of indigenous elders, women, youth, children and persons with disabilities.

Article 26

1. Indigenous peoples have the right to the lands, territories and resources which they have traditionally
owned, occupied or otherwise used or acquired.

2. Indigenous peoples have the right to own, use, develop and control the lands, territories and resources that they
possess by reason of traditional ownership or other traditional occupation or use, as well as those which they have
otherwise acquired.

3. States shall give legal recognition and protection to these lands, territories and resources. Such recognition
shall be conducted with due respect to the customs, traditions and land tenure systems of the indigenous peoples
concerned.

Article 30

1. Military activities shall not take place in the lands or territories of indigenous peoples, unless justified by a
relevant public interest or otherwise freely agreed with or requested by the indigenous peoples concerned.

2. States shall undertake effective consultations with the indigenous peoples concerned, through appropriate
procedures and in particular through their representative institutions, prior to using their lands or territories for
military activities.

Article 32

1. Indigenous peoples have the right to determine and develop priorities and strategies for the development or use
of their lands or territories and other resources.

2. States shall consult and cooperate in good faith with the indigenous peoples concerned through their own
representative institutions in order to obtain their free and informed consent prior to the approval of any project
affecting their lands or territories and other resources, particularly in connection with the development, utilization or
exploitation of mineral, water or other resources.

3. States shall provide effective mechanisms for just and fair redress for any such activities, and appropriate
measures shall be taken to mitigate adverse environmental, economic, social, cultural or spiritual impact.

Article 37

1. Indigenous peoples have the right to the recognition, observance and enforcement of treaties, agreements and
other constructive arrangements concluded with States or their successors and to have States honour and respect
such treaties, agreements and other constructive arrangements.

2. Nothing in this Declaration may be interpreted as diminishing or eliminating the rights of indigenous peoples
contained in treaties, agreements and other constructive arrangements.

Article 38

States in consultation and cooperation with indigenous peoples, shall take the appropriate measures, including
legislative measures, to achieve the ends of this Declaration.

Assuming that the UN DRIP, like the Universal Declaration on Human Rights, must now be regarded as embodying
customary international law - a question which the Court need not definitively resolve here - the obligations enumerated
therein do not strictly require the Republic to grant the Bangsamoro people, through the instrumentality of the BJE, the
particular rights and powers provided for in the MOA-AD. Even the more specific provisions of the UN DRIP are general in
scope, allowing for flexibility in its application by the different States.

There is, for instance, no requirement in the UN DRIP that States now guarantee indigenous peoples their own police and
internal security force. Indeed, Article 8 presupposes that it is the State which will provide protection for indigenous
peoples against acts like the forced dispossession of their lands - a function that is normally performed by police officers. If
the protection of a right so essential to indigenous people's identity is acknowledged to be the responsibility of the State,
then surely the protection of rights less significant to them as such peoples would also be the duty of States. Nor is there
in the UN DRIP an acknowledgement of the right of indigenous peoples to the aerial domain and atmospheric space. What
it upholds, in Article 26 thereof, is the right of indigenous peoples to the lands, territories and resources which they
have traditionally owned, occupied or otherwise used or acquired.

Moreover, the UN DRIP, while upholding the right of indigenous peoples to autonomy, does not obligate States to grant
indigenous peoples the near-independent status of an associated state. All the rights recognized in that document are
qualified in Article 46 as follows:

1. Nothing in this Declaration may be interpreted as implying for any State, people, group or person any right to
engage in any activity or to perform any act contrary to the Charter of the United Nations or construed as
authorizing or encouraging any action which would dismember or impair, totally or in part, the territorial
integrity or political unity of sovereign and independent States.

Even if the UN DRIP were considered as part of the law of the land pursuant to Article II, Section 2 of the Constitution, it
would not suffice to uphold the validity of the MOA-AD so as to render its compliance with other laws unnecessary.

It is, therefore, clear that the MOA-AD contains numerous provisions that cannot be reconciled with the
Constitution and the laws as presently worded. Respondents proffer, however, that the signing of the MOA-AD alone
would not have entailed any violation of law or grave abuse of discretion on their part, precisely because it stipulates that
the provisions thereof inconsistent with the laws shall not take effect until these laws are amended. They cite paragraph 7
of the MOA-AD strand on GOVERNANCE quoted earlier, but which is reproduced below for convenience:

7. The Parties agree that the mechanisms and modalities for the actual implementation of this MOA-AD shall be
spelt out in the Comprehensive Compact to mutually take such steps to enable it to occur effectively.

Any provisions of the MOA-AD requiring amendments to the existing legal framework shall come into force upon
signing of a Comprehensive Compact and upon effecting the necessary changes to the legal framework with due
regard to non derogation of prior agreements and within the stipulated timeframe to be contained in the
Comprehensive Compact.

Indeed, the foregoing stipulation keeps many controversial provisions of the MOA-AD from coming into force until the
necessary changes to the legal framework are effected. While the word "Constitution" is not mentioned in the
provision now under consideration or anywhere else in the MOA-AD, the term "legal framework" is certainly
broad enough to include the Constitution.

Notwithstanding the suspensive clause, however, respondents, by their mere act of incorporating in the MOA-AD the
provisions thereof regarding the associative relationship between the BJE and the Central Government, have already
violated the Memorandum of Instructions From The President dated March 1, 2001, which states that the "negotiations
shall be conducted in accordance with x x x the principles of the sovereignty and territorial integrity of the Republic of the
Philippines." (Emphasis supplied) Establishing an associative relationship between the BJE and the Central Government
is, for the reasons already discussed, a preparation for independence, or worse, an implicit acknowledgment of an
independent status already prevailing.

Even apart from the above-mentioned Memorandum, however, the MOA-AD is defective because the suspensive clause
is invalid, as discussed below.

The authority of the GRP Peace Negotiating Panel to negotiate with the MILF is founded on E.O. No. 3, Section 5(c),
which states that there shall be established Government Peace Negotiating Panels for negotiations with different rebel
groups to be "appointed by the President as her official emissaries to conduct negotiations, dialogues, and face-to-face
discussions with rebel groups." These negotiating panels are to report to the President, through the PAPP on the conduct
and progress of the negotiations.

It bears noting that the GRP Peace Panel, in exploring lasting solutions to the Moro Problem through its negotiations with
the MILF, was not restricted by E.O. No. 3 only to those options available under the laws as they presently stand. One of
the components of a comprehensive peace process, which E.O. No. 3 collectively refers to as the "Paths to Peace," is the
pursuit of social, economic, and political reforms which may require new legislation or even constitutional amendments.
Sec. 4(a) of E.O. No. 3, which reiterates Section 3(a), of E.O. No. 125, 167 states:

SECTION 4. The Six Paths to Peace. - The components of the comprehensive peace process comprise the
processes known as the "Paths to Peace". These component processes are interrelated and not mutually
exclusive, and must therefore be pursued simultaneously in a coordinated and integrated fashion. They shall
include, but may not be limited to, the following:

a. PURSUIT OF SOCIAL, ECONOMIC AND POLITICAL REFORMS. This component involves the vigorous
implementation of various policies, reforms, programs and projects aimed at addressing the root causes
of internal armed conflicts and social unrest. This may require administrative action, new legislation or
even constitutional amendments.
x x x x (Emphasis supplied)

The MOA-AD, therefore, may reasonably be perceived as an attempt of respondents to address, pursuant to this provision
of E.O. No. 3, the root causes of the armed conflict in Mindanao. The E.O. authorized them to "think outside the box," so
to speak. Hence, they negotiated and were set on signing the MOA-AD that included various social, economic, and
political reforms which cannot, however, all be accommodated within the present legal framework, and which thus would
require new legislation and constitutional amendments.

The inquiry on the legality of the "suspensive clause," however, cannot stop here, because it must be asked whether the
President herself may exercise the power delegated to the GRP Peace Panel under E.O. No. 3, Sec. 4(a).

The President cannot delegate a power that she herself does not possess. May the President, in the course of peace
negotiations, agree to pursue reforms that would require new legislation and constitutional amendments, or should the
reforms be restricted only to those solutions which the present laws allow? The answer to this question requires a
discussion of the extent of the President's power to conduct peace negotiations.

That the authority of the President to conduct peace negotiations with rebel groups is not explicitly mentioned in the
Constitution does not mean that she has no such authority. In Sanlakas v. Executive Secretary,168 in issue was the
authority of the President to declare a state of rebellion - an authority which is not expressly provided for in the
Constitution. The Court held thus:

"In her ponencia in Marcos v. Manglapus, Justice Cortes put her thesis into jurisprudence. There, the Court, by a
slim 8-7 margin, upheld the President's power to forbid the return of her exiled predecessor. The rationale for the
majority's ruling rested on the President's

. . . unstated residual powers which are implied from the grant of executive power and which
are necessary for her to comply with her duties under the Constitution. The powers of the
President are not limited to what are expressly enumerated in the article on the Executive
Department and in scattered provisions of the Constitution. This is so, notwithstanding the avowed
intent of the members of the Constitutional Commission of 1986 to limit the powers of the President as a
reaction to the abuses under the regime of Mr. Marcos, for the result was a limitation of specific powers of
the President, particularly those relating to the commander-in-chief clause, but not a diminution of the
general grant of executive power.

Thus, the President's authority to declare a state of rebellion springs in the main from her powers as chief
executive and, at the same time, draws strength from her Commander-in-Chief powers. x x x (Emphasis and
underscoring supplied)

Similarly, the President's power to conduct peace negotiations is implicitly included in her powers as Chief Executive and
Commander-in-Chief. As Chief Executive, the President has the general responsibility to promote public peace, and as
Commander-in-Chief, she has the more specific duty to prevent and suppress rebellion and lawless violence. 169

As the experience of nations which have similarly gone through internal armed conflict will show, however, peace is rarely
attained by simply pursuing a military solution. Oftentimes, changes as far-reaching as a fundamental reconfiguration of
the nation's constitutional structure is required. The observations of Dr. Kirsti Samuels are enlightening, to wit:

x x x [T]he fact remains that a successful political and governance transition must form the core of any post-conflict
peace-building mission. As we have observed in Liberia and Haiti over the last ten years, conflict cessation without
modification of the political environment, even where state-building is undertaken through technical electoral
assistance and institution- or capacity-building, is unlikely to succeed. On average, more than 50 percent of states
emerging from conflict return to conflict. Moreover, a substantial proportion of transitions have resulted in weak or
limited democracies.

The design of a constitution and its constitution-making process can play an important role in the political and
governance transition. Constitution-making after conflict is an opportunity to create a common vision of the future
of a state and a road map on how to get there. The constitution can be partly a peace agreement and partly a
framework setting up the rules by which the new democracy will operate. 170

In the same vein, Professor Christine Bell, in her article on the nature and legal status of peace agreements, observed that
the typical way that peace agreements establish or confirm mechanisms for demilitarization and demobilization is by
linking them to new constitutional structures addressing governance, elections, and legal and human rights
institutions.171

In the Philippine experience, the link between peace agreements and constitution-making has been recognized by no less
than the framers of the Constitution. Behind the provisions of the Constitution on autonomous regions 172 is the framers'
intention to implement a particular peace agreement, namely, the Tripoli Agreement of 1976 between the GRP and the
MNLF, signed by then Undersecretary of National Defense Carmelo Z. Barbero and then MNLF Chairman Nur Misuari.

MR. ROMULO. There are other speakers; so, although I have some more questions, I will reserve my right to ask
them if they are not covered by the other speakers. I have only two questions.
I heard one of the Commissioners say that local autonomy already exists in the Muslim region; it is working
very well; it has, in fact, diminished a great deal of the problems. So, my question is: since that already exists,
why do we have to go into something new?

MR. OPLE. May I answer that on behalf of Chairman Nolledo. Commissioner Yusup Abubakar is right that certain
definite steps have been taken to implement the provisions of the Tripoli Agreement with respect to an
autonomous region in Mindanao. This is a good first step, but there is no question that this is merely a
partial response to the Tripoli Agreement itself and to the fuller standard of regional autonomy
contemplated in that agreement, and now by state policy. 173(Emphasis supplied)

The constitutional provisions on autonomy and the statutes enacted pursuant to them have, to the credit of their drafters,
been partly successful. Nonetheless, the Filipino people are still faced with the reality of an on-going conflict between the
Government and the MILF. If the President is to be expected to find means for bringing this conflict to an end and to
achieve lasting peace in Mindanao, then she must be given the leeway to explore, in the course of peace negotiations,
solutions that may require changes to the Constitution for their implementation. Being uniquely vested with the power to
conduct peace negotiations with rebel groups, the President is in a singular position to know the precise nature of their
grievances which, if resolved, may bring an end to hostilities.

The President may not, of course, unilaterally implement the solutions that she considers viable, but she may not be
prevented from submitting them as recommendations to Congress, which could then, if it is minded, act upon them
pursuant to the legal procedures for constitutional amendment and revision. In particular, Congress would have the option,
pursuant to Article XVII, Sections 1 and 3 of the Constitution, to propose the recommended amendments or revision to the
people, call a constitutional convention, or submit to the electorate the question of calling such a convention.

While the President does not possess constituent powers - as those powers may be exercised only by Congress, a
Constitutional Convention, or the people through initiative and referendum - she may submit proposals for constitutional
change to Congress in a manner that does not involve the arrogation of constituent powers.

In Sanidad v. COMELEC,174 in issue was the legality of then President Marcos' act of directly submitting proposals for
constitutional amendments to a referendum, bypassing the interim National Assembly which was the body vested by the
1973 Constitution with the power to propose such amendments. President Marcos, it will be recalled, never convened the
interim National Assembly. The majority upheld the President's act, holding that "the urges of absolute necessity"
compelled the President as the agent of the people to act as he did, there being no interim National Assembly to propose
constitutional amendments. Against this ruling, Justices Teehankee and Muñoz Palma vigorously dissented. The Court's
concern at present, however, is not with regard to the point on which it was then divided in that controversial case, but on
that which was not disputed by either side.

Justice Teehankee's dissent, 175 in particular, bears noting. While he disagreed that the President may directly submit
proposed constitutional amendments to a referendum, implicit in his opinion is a recognition that he would have upheld the
President's action along with the majority had the President convened the interim National Assembly and coursed his
proposals through it. Thus Justice Teehankee opined:

"Since the Constitution provides for the organization of the essential departments of government, defines and
delimits the powers of each and prescribes the manner of the exercise of such powers, and the constituent power
has not been granted to but has been withheld from the President or Prime Minister, it follows that the President's
questioned decrees proposing and submitting constitutional amendments directly to the people (without the
intervention of the interim National Assembly in whom the power is expressly vested) are devoid of
constitutional and legal basis."176 (Emphasis supplied)

From the foregoing discussion, the principle may be inferred that the President - in the course of conducting peace
negotiations - may validly consider implementing even those policies that require changes to the Constitution, but she
may not unilaterally implement them without the intervention of Congress, or act in any way as if the assent of that
body were assumed as a certainty.

Since, under the present Constitution, the people also have the power to directly propose amendments through initiative
and referendum, the President may also submit her recommendations to the people, not as a formal proposal to be voted
on in a plebiscite similar to what President Marcos did in Sanidad, but for their independent consideration of whether these
recommendations merit being formally proposed through initiative.

These recommendations, however, may amount to nothing more than the President's suggestions to the people, for any
further involvement in the process of initiative by the Chief Executive may vitiate its character as a genuine
"people's initiative." The only initiative recognized by the Constitution is that which truly proceeds from the people. As the
Court stated in Lambino v. COMELEC:177

"The Lambino Group claims that their initiative is the ‘people's voice.' However, the Lambino Group unabashedly
states in ULAP Resolution No. 2006-02, in the verification of their petition with the COMELEC, that ‘ULAP
maintains its unqualified support to the agenda of Her Excellency President Gloria Macapagal-Arroyo for
constitutional reforms.' The Lambino Group thus admits that their ‘people's' initiative is an ‘unqualified support to
the agenda' of the incumbent President to change the Constitution. This forewarns the Court to be wary of
incantations of ‘people's voice' or ‘sovereign will' in the present initiative."
It will be observed that the President has authority, as stated in her oath of office, 178 only to preserve and defend the
Constitution. Such presidential power does not, however, extend to allowing her to change the Constitution, but simply to
recommend proposed amendments or revision. As long as she limits herself to recommending these changes and submits
to the proper procedure for constitutional amendments and revision, her mere recommendation need not be construed as
an unconstitutional act.

The foregoing discussion focused on the President's authority to propose constitutional amendments, since her authority
to propose new legislation is not in controversy. It has been an accepted practice for Presidents in this jurisdiction to
propose new legislation. One of the more prominent instances the practice is usually done is in the yearly State of the
Nation Address of the President to Congress. Moreover, the annual general appropriations bill has always been based on
the budget prepared by the President, which - for all intents and purposes - is a proposal for new legislation coming from
the President.179

The "suspensive clause" in the MOA-AD viewed in light of the above-discussed standards

Given the limited nature of the President's authority to propose constitutional amendments, she cannot guarantee to any
third party that the required amendments will eventually be put in place, nor even be submitted to a plebiscite. The most
she could do is submit these proposals as recommendations either to Congress or the people, in whom constituent
powers are vested.

Paragraph 7 on Governance of the MOA-AD states, however, that all provisions thereof which cannot be reconciled with
the present Constitution and laws "shall come into force upon signing of a Comprehensive Compact and upon effecting
the necessary changes to the legal framework." This stipulation does not bear the marks of a suspensive condition -
defined in civil law as a future and uncertain event - but of a term. It is not a question of whether the necessary changes
to the legal framework will be effected, but when. That there is no uncertainty being contemplated is plain from what
follows, for the paragraph goes on to state that the contemplated changes shall be "with due regard to non derogation of
prior agreements and within the stipulated timeframe to be contained in the Comprehensive Compact."

Pursuant to this stipulation, therefore, it is mandatory for the GRP to effect the changes to the legal framework
contemplated in the MOA-AD - which changes would include constitutional amendments, as discussed earlier. It bears
noting that,

By the time these changes are put in place, the MOA-AD itself would be counted among the "prior agreements"
from which there could be no derogation.

What remains for discussion in the Comprehensive Compact would merely be the implementing details for these
"consensus points" and, notably, the deadline for effecting the contemplated changes to the legal framework.

Plainly, stipulation-paragraph 7 on GOVERNANCE is inconsistent with the limits of the President's authority to
propose constitutional amendments, it being a virtual guarantee that the Constitution and the laws of the Republic of
the Philippines will certainly be adjusted to conform to all the "consensus points" found in the MOA-AD. Hence, it must be
struck down as unconstitutional.

A comparison between the "suspensive clause" of the MOA-AD with a similar provision appearing in the 1996 final peace
agreement between the MNLF and the GRP is most instructive.

As a backdrop, the parties to the 1996 Agreement stipulated that it would be implemented in two phases. Phase I covered
a three-year transitional period involving the putting up of new administrative structures through Executive Order, such as
the Special Zone of Peace and Development (SZOPAD) and the Southern Philippines Council for Peace and
Development (SPCPD), while Phase II covered the establishment of the new regional autonomous government through
amendment or repeal of R.A. No. 6734, which was then the Organic Act of the ARMM.

The stipulations on Phase II consisted of specific agreements on the structure of the expanded autonomous region
envisioned by the parties. To that extent, they are similar to the provisions of the MOA-AD. There is, however, a crucial
difference between the two agreements. While the MOA-AD virtually guarantees that the "necessary changes to the
legal framework" will be put in place, the GRP-MNLF final peace agreement states thus: "Accordingly, these provisions
[on Phase II] shall be recommended by the GRP to Congress for incorporation in the amendatory or repealing law."

Concerns have been raised that the MOA-AD would have given rise to a binding international law obligation on the part of
the Philippines to change its Constitution in conformity thereto, on the ground that it may be considered either as a binding
agreement under international law, or a unilateral declaration of the Philippine government to the international community
that it would grant to the Bangsamoro people all the concessions therein stated. Neither ground finds sufficient support in
international law, however.

The MOA-AD, as earlier mentioned in the overview thereof, would have included foreign dignitaries as signatories. In
addition, representatives of other nations were invited to witness its signing in Kuala Lumpur. These circumstances readily
lead one to surmise that the MOA-AD would have had the status of a binding international agreement had it been signed.
An examination of the prevailing principles in international law, however, leads to the contrary conclusion.

The Decision on Challenge to Jurisdiction: Lomé Accord Amnesty 180 (the Lomé Accord case) of the Special Court of Sierra
Leone is enlightening. The Lomé Accord was a peace agreement signed on July 7, 1999 between the Government of
Sierra Leone and the Revolutionary United Front (RUF), a rebel group with which the Sierra Leone Government had been
in armed conflict for around eight years at the time of signing. There were non-contracting signatories to the agreement,
among which were the Government of the Togolese Republic, the Economic Community of West African States, and the
UN.

On January 16, 2002, after a successful negotiation between the UN Secretary-General and the Sierra Leone
Government, another agreement was entered into by the UN and that Government whereby the Special Court of Sierra
Leone was established. The sole purpose of the Special Court, an international court, was to try persons who bore the
greatest responsibility for serious violations of international humanitarian law and Sierra Leonean law committed in the
territory of Sierra Leone since November 30, 1996.

Among the stipulations of the Lomé Accord was a provision for the full pardon of the members of the RUF with respect to
anything done by them in pursuit of their objectives as members of that organization since the conflict began.

In the Lomé Accord case, the Defence argued that the Accord created an internationally binding obligation not to
prosecute the beneficiaries of the amnesty provided therein, citing, among other things, the participation of foreign
dignitaries and international organizations in the finalization of that agreement. The Special Court, however, rejected this
argument, ruling that the Lome Accord is not a treaty and that it can only create binding obligations and rights between the
parties in municipal law, not in international law. Hence, the Special Court held, it is ineffective in depriving an international
court like it of jurisdiction.

"37. In regard to the nature of a negotiated settlement of an internal armed conflict it is easy to assume and to
argue with some degree of plausibility, as Defence counsel for the defendants seem to have done, that the
mere fact that in addition to the parties to the conflict, the document formalizing the settlement is signed
by foreign heads of state or their representatives and representatives of international organizations,
means the agreement of the parties is internationalized so as to create obligations in international law.

xxxx

40. Almost every conflict resolution will involve the parties to the conflict and the mediator or facilitator of the
settlement, or persons or bodies under whose auspices the settlement took place but who are not at all parties to
the conflict, are not contracting parties and who do not claim any obligation from the contracting parties or incur
any obligation from the settlement.

41. In this case, the parties to the conflict are the lawful authority of the State and the RUF which has no
status of statehood and is to all intents and purposes a faction within the state. The non-contracting
signatories of the Lomé Agreement were moral guarantors of the principle that, in the terms of Article
XXXIV of the Agreement, "this peace agreement is implemented with integrity and in good faith by both
parties". The moral guarantors assumed no legal obligation. It is recalled that the UN by its representative
appended, presumably for avoidance of doubt, an understanding of the extent of the agreement to be
implemented as not including certain international crimes.

42. An international agreement in the nature of a treaty must create rights and obligations regulated by
international law so that a breach of its terms will be a breach determined under international law which will also
provide principle means of enforcement. The Lomé Agreement created neither rights nor obligations capable
of being regulated by international law. An agreement such as the Lomé Agreement which brings to an
end an internal armed conflict no doubt creates a factual situation of restoration of peace that the
international community acting through the Security Council may take note of. That, however, will not
convert it to an international agreement which creates an obligation enforceable in international, as
distinguished from municipal, law. A breach of the terms of such a peace agreement resulting in resumption of
internal armed conflict or creating a threat to peace in the determination of the Security Council may indicate a
reversal of the factual situation of peace to be visited with possible legal consequences arising from the new
situation of conflict created. Such consequences such as action by the Security Council pursuant to Chapter VII
arise from the situation and not from the agreement, nor from the obligation imposed by it. Such action cannot be
regarded as a remedy for the breach. A peace agreement which settles an internal armed conflict cannot be
ascribed the same status as one which settles an international armed conflict which, essentially, must be
between two or more warring States. The Lomé Agreement cannot be characterised as an international
instrument. x x x" (Emphasis, italics and underscoring supplied)

Similarly, that the MOA-AD would have been signed by representatives of States and international organizations not
parties to the Agreement would not have sufficed to vest in it a binding character under international law.

In another vein, concern has been raised that the MOA-AD would amount to a unilateral declaration of the Philippine
State, binding under international law, that it would comply with all the stipulations stated therein, with the result that it
would have to amend its Constitution accordingly regardless of the true will of the people. Cited as authority for this view
is Australia v. France,181 also known as the Nuclear Tests Case, decided by the International Court of Justice (ICJ).

In the Nuclear Tests Case, Australia challenged before the ICJ the legality of France's nuclear tests in the South Pacific.
France refused to appear in the case, but public statements from its President, and similar statements from other French
officials including its Minister of Defence, that its 1974 series of atmospheric tests would be its last, persuaded the ICJ to
dismiss the case.182 Those statements, the ICJ held, amounted to a legal undertaking addressed to the international
community, which required no acceptance from other States for it to become effective.
Essential to the ICJ ruling is its finding that the French government intended to be bound to the international community in
issuing its public statements, viz:

43. It is well recognized that declarations made by way of unilateral acts, concerning legal or factual situations,
may have the effect of creating legal obligations. Declarations of this kind may be, and often are, very
specific. When it is the intention of the State making the declaration that it should become bound
according to its terms, that intention confers on the declaration the character of a legal undertaking, the
State being thenceforth legally required to follow a course of conduct consistent with the declaration. An
undertaking of this kind, if given publicly, and with an intent to be bound, even though not made within the context
of international negotiations, is binding. In these circumstances, nothing in the nature of a quid pro quo nor any
subsequent acceptance of the declaration, nor even any reply or reaction from other States, is required for the
declaration to take effect, since such a requirement would be inconsistent with the strictly unilateral nature of the
juridical act by which the pronouncement by the State was made.

44. Of course, not all unilateral acts imply obligation; but a State may choose to take up a certain position
in relation to a particular matter with the intention of being bound-the intention is to be ascertained by
interpretation of the act. When States make statements by which their freedom of action is to be limited, a
restrictive interpretation is called for.

xxxx

51. In announcing that the 1974 series of atmospheric tests would be the last, the French Government
conveyed to the world at large, including the Applicant, its intention effectively to terminate these tests. It
was bound to assume that other States might take note of these statements and rely on their being
effective. The validity of these statements and their legal consequences must be considered within the
general framework of the security of international intercourse, and the confidence and trust which are so
essential in the relations among States. It is from the actual substance of these statements, and from the
circumstances attending their making, that the legal implications of the unilateral act must be
deduced. The objects of these statements are clear and they were addressed to the international
community as a whole, and the Court holds that they constitute an undertaking possessing legal
effect. The Court considers *270 that the President of the Republic, in deciding upon the effective cessation of
atmospheric tests, gave an undertaking to the international community to which his words were addressed. x x x
(Emphasis and underscoring supplied)

As gathered from the above-quoted ruling of the ICJ, public statements of a state representative may be construed as
a unilateral declaration only when the following conditions are present: the statements were clearly addressed to the
international community, the state intended to be bound to that community by its statements, and that not to give legal
effect to those statements would be detrimental to the security of international intercourse. Plainly, unilateral declarations
arise only in peculiar circumstances.

The limited applicability of the Nuclear Tests Case ruling was recognized in a later case decided by the ICJ
entitled Burkina Faso v. Mali,183 also known as the Case Concerning the Frontier Dispute. The public declaration subject of
that case was a statement made by the President of Mali, in an interview by a foreign press agency, that Mali would abide
by the decision to be issued by a commission of the Organization of African Unity on a frontier dispute then pending
between Mali and Burkina Faso.

Unlike in the Nuclear Tests Case, the ICJ held that the statement of Mali's President was not a unilateral act with legal
implications. It clarified that its ruling in the Nuclear Tests case rested on the peculiar circumstances surrounding the
French declaration subject thereof, to wit:

40. In order to assess the intentions of the author of a unilateral act, account must be taken of all the factual
circumstances in which the act occurred. For example, in the Nuclear Tests cases, the Court took the view that
since the applicant States were not the only ones concerned at the possible continuance of atmospheric
testing by the French Government, that Government's unilateral declarations had ‘conveyed to the world
at large, including the Applicant, its intention effectively to terminate these tests‘ (I.C.J. Reports 1974, p.
269, para. 51; p. 474, para. 53). In the particular circumstances of those cases, the French Government
could not express an intention to be bound otherwise than by unilateral declarations. It is difficult to see
how it could have accepted the terms of a negotiated solution with each of the applicants without thereby
jeopardizing its contention that its conduct was lawful. The circumstances of the present case are
radically different. Here, there was nothing to hinder the Parties from manifesting an intention to accept
the binding character of the conclusions of the Organization of African Unity Mediation Commission by
the normal method: a formal agreement on the basis of reciprocity. Since no agreement of this kind was
concluded between the Parties, the Chamber finds that there are no grounds to interpret the declaration made by
Mali's head of State on 11 April 1975 as a unilateral act with legal implications in regard to the present case.
(Emphasis and underscoring supplied)

Assessing the MOA-AD in light of the above criteria, it would not have amounted to a unilateral declaration on the part of
the Philippine State to the international community. The Philippine panel did not draft the same with the clear intention of
being bound thereby to the international community as a whole or to any State, but only to the MILF. While there were
States and international organizations involved, one way or another, in the negotiation and projected signing of the MOA-
AD, they participated merely as witnesses or, in the case of Malaysia, as facilitator. As held in the Lomé Accord case, the
mere fact that in addition to the parties to the conflict, the peace settlement is signed by representatives of states and
international organizations does not mean that the agreement is internationalized so as to create obligations in
international law.

Since the commitments in the MOA-AD were not addressed to States, not to give legal effect to such commitments would
not be detrimental to the security of international intercourse - to the trust and confidence essential in the relations among
States.

In one important respect, the circumstances surrounding the MOA-AD are closer to that of Burkina Faso wherein, as
already discussed, the Mali President's statement was not held to be a binding unilateral declaration by the ICJ. As in that
case, there was also nothing to hinder the Philippine panel, had it really been its intention to be bound to other States, to
manifest that intention by formal agreement. Here, that formal agreement would have come about by the inclusion in the
MOA-AD of a clear commitment to be legally bound to the international community, not just the MILF, and by an equally
clear indication that the signatures of the participating states-representatives would constitute an acceptance of that
commitment. Entering into such a formal agreement would not have resulted in a loss of face for the Philippine
government before the international community, which was one of the difficulties that prevented the French Government
from entering into a formal agreement with other countries. That the Philippine panel did not enter into such a formal
agreement suggests that it had no intention to be bound to the international community. On that ground, the MOA-AD
may not be considered a unilateral declaration under international law .

The MOA-AD not being a document that can bind the Philippines under international law notwithstanding, respondents'
almost consummated act of guaranteeing amendments to the legal framework is, by itself, sufficient to constitute
grave abuse of discretion. The grave abuse lies not in the fact that they considered, as a solution to the Moro Problem,
the creation of a state within a state, but in their brazen willingness to guarantee that Congress and the sovereign
Filipino people would give their imprimatur to their solution. Upholding such an act would amount to authorizing a
usurpation of the constituent powers vested only in Congress, a Constitutional Convention, or the people themselves
through the process of initiative, for the only way that the Executive can ensure the outcome of the amendment process is
through an undue influence or interference with that process.

The sovereign people may, if it so desired, go to the extent of giving up a portion of its own territory to the Moros for the
sake of peace, for it can change the Constitution in any it wants, so long as the change is not inconsistent with what, in
international law, is known as Jus Cogens.184 Respondents, however, may not preempt it in that decision.

SUMMARY

The petitions are ripe for adjudication. The failure of respondents to consult the local government units or communities
affected constitutes a departure by respondents from their mandate under E.O. No. 3. Moreover, respondents exceeded
their authority by the mere act of guaranteeing amendments to the Constitution. Any alleged violation of the Constitution
by any branch of government is a proper matter for judicial review.

As the petitions involve constitutional issues which are of paramount public interest or of transcendental importance, the
Court grants the petitioners, petitioners-in-intervention and intervening respondents the requisite locus standi in keeping
with the liberal stance adopted in David v. Macapagal-Arroyo.

Contrary to the assertion of respondents that the non-signing of the MOA-AD and the eventual dissolution of the GRP
Peace Panel mooted the present petitions, the Court finds that the present petitions provide an exception to the "moot and
academic" principle in view of (a) the grave violation of the Constitution involved; (b) the exceptional character of the
situation and paramount public interest; (c) the need to formulate controlling principles to guide the bench, the bar, and the
public; and (d) the fact that the case is capable of repetition yet evading review.

The MOA-AD is a significant part of a series of agreements necessary to carry out the GRP-MILF Tripoli Agreement on
Peace signed by the government and the MILF back in June 2001. Hence, the present MOA-AD can be renegotiated or
another one drawn up that could contain similar or significantly dissimilar provisions compared to the original.

The Court, however, finds that the prayers for mandamus have been rendered moot in view of the respondents' action in
providing the Court and the petitioners with the official copy of the final draft of the MOA-AD and its annexes.

The people's right to information on matters of public concern under Sec. 7, Article III of the Constitution is in splendid
symmetry with the state policy of full public disclosure of all its transactions involving public interest under Sec. 28, Article
II of the Constitution. The right to information guarantees the right of the people to demand information, while Section 28
recognizes the duty of officialdom to give information even if nobody demands. The complete and effective exercise of the
right to information necessitates that its complementary provision on public disclosure derive the same self-executory
nature, subject only to reasonable safeguards or limitations as may be provided by law.

The contents of the MOA-AD is a matter of paramount public concern involving public interest in the highest order. In
declaring that the right to information contemplates steps and negotiations leading to the consummation of the contract,
jurisprudence finds no distinction as to the executory nature or commercial character of the agreement.

An essential element of these twin freedoms is to keep a continuing dialogue or process of communication between the
government and the people. Corollary to these twin rights is the design for feedback mechanisms. The right to public
consultation was envisioned to be a species of these public rights.
At least three pertinent laws animate these constitutional imperatives and justify the exercise of the people's right to be
consulted on relevant matters relating to the peace agenda.

One, E.O. No. 3 itself is replete with mechanics for continuing consultations on both national and local levels and for a
principal forum for consensus-building. In fact, it is the duty of the Presidential Adviser on the Peace Process to conduct
regular dialogues to seek relevant information, comments, advice, and recommendations from peace partners and
concerned sectors of society.

Two, Republic Act No. 7160 or the Local Government Code of 1991 requires all national offices to conduct consultations
before any project or program critical to the environment and human ecology including those that may call for the eviction
of a particular group of people residing in such locality, is implemented therein. The MOA-AD is one peculiar program that
unequivocally and unilaterally vests ownership of a vast territory to the Bangsamoro people, which could pervasively and
drastically result to the diaspora or displacement of a great number of inhabitants from their total environment.

Three, Republic Act No. 8371 or the Indigenous Peoples Rights Act of 1997 provides for clear-cut procedure for the
recognition and delineation of ancestral domain, which entails, among other things, the observance of the free and prior
informed consent of the Indigenous Cultural Communities/Indigenous Peoples. Notably, the statute does not grant the
Executive Department or any government agency the power to delineate and recognize an ancestral domain claim by
mere agreement or compromise.

The invocation of the doctrine of executive privilege as a defense to the general right to information or the specific right to
consultation is untenable. The various explicit legal provisions fly in the face of executive secrecy. In any event,
respondents effectively waived such defense after it unconditionally disclosed the official copies of the final draft of the
MOA-AD, for judicial compliance and public scrutiny.

In sum, the Presidential Adviser on the Peace Process committed grave abuse of discretion when he failed to carry out the
pertinent consultation process, as mandated by E.O. No. 3, Republic Act No. 7160, and Republic Act No. 8371. The furtive
process by which the MOA-AD was designed and crafted runs contrary to and in excess of the legal authority, and
amounts to a whimsical, capricious, oppressive, arbitrary and despotic exercise thereof. It illustrates a gross evasion of
positive duty and a virtual refusal to perform the duty enjoined.

The MOA-AD cannot be reconciled with the present Constitution and laws. Not only its specific provisions but the
very concept underlying them, namely, the associative relationship envisioned between the GRP and the
BJE, are unconstitutional, for the concept presupposes that the associated entity is a state and implies that the same is
on its way to independence.

While there is a clause in the MOA-AD stating that the provisions thereof inconsistent with the present legal framework will
not be effective until that framework is amended, the same does not cure its defect. The inclusion of provisions in the
MOA-AD establishing an associative relationship between the BJE and the Central Government is, itself, a violation of the
Memorandum of Instructions From The President dated March 1, 2001, addressed to the government peace panel.
Moreover, as the clause is worded, it virtually guarantees that the necessary amendments to the Constitution and the laws
will eventually be put in place. Neither the GRP Peace Panel nor the President herself is authorized to make such a
guarantee. Upholding such an act would amount to authorizing a usurpation of the constituent powers vested only in
Congress, a Constitutional Convention, or the people themselves through the process of initiative, for the only way that the
Executive can ensure the outcome of the amendment process is through an undue influence or interference with that
process.

While the MOA-AD would not amount to an international agreement or unilateral declaration binding on the Philippines
under international law, respondents' act of guaranteeing amendments is, by itself, already a constitutional violation that
renders the MOA-AD fatally defective.

WHEREFORE, respondents' motion to dismiss is DENIED. The main and intervening petitions are GIVEN DUE COURSE
and hereby GRANTED.

The Memorandum of Agreement on the Ancestral Domain Aspect of the GRP-MILF Tripoli Agreement on Peace of 2001 is
declared contrary to law and the Constitution.

SO ORDERED

Incorporation Doctrine vs Transformation Doctrine


-Deutsche Bank AG Manila Branch v CIR, 704 SCRA 216 (2013)

This is a Petition for Review1 filed by Deutsche Bank AG Manila Branch (petitioner) under Rule 45 of the 1997 Rules of
Civil Procedure assailing the Court of Tax Appeals En Banc (CTA En Banc) Decision 2 dated 29 May 2009 and
Resolution3 dated 1 July 2009 in C.T.A. EB No. 456.

THE FACTS

In accordance with Section 28(A)(5)4 of the National Internal Revenue Code (NIRC) of 1997, petitioner withheld and
remitted to respondent on 21 October 2003 the amount of PHP 67,688,553.51, which represented the fifteen percent
(15%) branch profit remittance tax (BPRT) on its regular banking unit (RBU) net income remitted to Deutsche Bank
Germany (DB Germany) for 2002 and prior taxable years. 5

Believing that it made an overpayment of the BPRT, petitioner filed with the BIR Large Taxpayers Assessment and
Investigation Division on 4 October 2005 an administrative claim for refund or issuance of its tax credit certificate in the
total amount of PHP 22,562,851.17. On the same date, petitioner requested from the International Tax Affairs Division
(ITAD) a confirmation of its entitlement to the preferential tax rate of 10% under the RP-Germany Tax Treaty. 6

Alleging the inaction of the BIR on its administrative claim, petitioner filed a Petition for Review 7 with the CTA on 18
October 2005. Petitioner reiterated its claim for the refund or issuance of its tax credit certificate for the amount of PHP
22,562,851.17 representing the alleged excess BPRT paid on branch profits remittance to DB Germany.

THE CTA SECOND DIVISION RULING8

After trial on the merits, the CTA Second Division found that petitioner indeed paid the total amount of PHP 67,688,553.51
representing the 15% BPRT on its RBU profits amounting to PHP 451,257,023.29 for 2002 and prior taxable years.
Records also disclose that for the year 2003, petitioner remitted to DB Germany the amount of EURO 5,174,847.38 (or
PHP 330,175,961.88 at the exchange rate of PHP 63.804:1 EURO), which is net of the 15% BPRT.

However, the claim of petitioner for a refund was denied on the ground that the application for a tax treaty relief was not
filed with ITAD prior to the payment by the former of its BPRT and actual remittance of its branch profits to DB Germany,
or prior to its availment of the preferential rate of ten percent (10%) under the RP-Germany Tax Treaty provision. The
court a quo held that petitioner violated the fifteen (15) day period mandated under Section III paragraph (2) of Revenue
Memorandum Order (RMO) No. 1-2000.

Further, the CTA Second Division relied on Mirant (Philippines) Operations Corporation (formerly Southern Energy Asia-
Pacific Operations [Phils.], Inc.) v. Commissioner of Internal Revenue 9 (Mirant) where the CTA En Banc ruled that before
the benefits of the tax treaty may be extended to a foreign corporation wishing to avail itself thereof, the latter should first
invoke the provisions of the tax treaty and prove that they indeed apply to the corporation.

THE CTA EN BANC RULING10

The CTA En Banc affirmed the CTA Second Division’s Decision dated 29 August 2008 and Resolution dated 14 January
2009. Citing Mirant, the CTA En Banc held that a ruling from the ITAD of the BIR must be secured prior to the availment of
a preferential tax rate under a tax treaty. Applying the principle of stare decisis et non quieta movere, the CTA En Banc
took into consideration that this Court had denied the Petition in G.R. No. 168531 filed by Mirant for failure to sufficiently
show any reversible error in the assailed judgment. 11 The CTA En Banc ruled that once a case has been decided in one
way, any other case involving exactly the same point at issue should be decided in the same manner.

The court likewise ruled that the 15-day rule for tax treaty relief application under RMO No. 1-2000 cannot be relaxed for
petitioner, unlike in CBK Power Company Limited v. Commissioner of Internal Revenue. 12 In that case, the rule was
relaxed and the claim for refund of excess final withholding taxes was partially granted. While it issued a ruling to CBK
Power Company Limited after the payment of withholding taxes, the ITAD did not issue any ruling to petitioner even if it
filed a request for confirmation on 4 October 2005 that the remittance of branch profits to DB Germany is subject to a
preferential tax rate of 10% pursuant to Article 10 of the RP-Germany Tax Treaty.

ISSUE

This Court is now confronted with the issue of whether the failure to strictly comply with RMO No. 1-2000 will deprive
persons or corporations of the benefit of a tax treaty.

THE COURT’S RULING

The Petition is meritorious.

Under Section 28(A)(5) of the NIRC, any profit remitted to its head office shall be subject to a tax of 15% based on the
total profits applied for or earmarked for remittance without any deduction of the tax component. However, petitioner
invokes paragraph 6, Article 10 of the RP-Germany Tax Treaty, which provides that where a resident of the Federal
Republic of Germany has a branch in the Republic of the Philippines, this branch may be subjected to the branch profits
remittance tax withheld at source in accordance with Philippine law but shall not exceed 10% of the gross amount of the
profits remitted by that branch to the head office.

By virtue of the RP-Germany Tax Treaty, we are bound to extend to a branch in the Philippines, remitting to its head office
in Germany, the benefit of a preferential rate equivalent to 10% BPRT.

On the other hand, the BIR issued RMO No. 1-2000, which requires that any availment of the tax treaty relief must be
preceded by an application with ITAD at least 15 days before the transaction. The Order was issued to streamline the
processing of the application of tax treaty relief in order to improve efficiency and service to the taxpayers. Further, it also
aims to prevent the consequences of an erroneous interpretation and/or application of the treaty provisions (i.e., filing a
claim for a tax refund/credit for the overpayment of taxes or for deficiency tax liabilities for underpayment). 13
The crux of the controversy lies in the implementation of RMO No. 1-2000.

Petitioner argues that, considering that it has met all the conditions under Article 10 of the RP-Germany Tax Treaty, the
CTA erred in denying its claim solely on the basis of RMO No. 1-2000. The filing of a tax treaty relief application is not a
condition precedent to the availment of a preferential tax rate. Further, petitioner posits that, contrary to the ruling of the
CTA, Mirant is not a binding judicial precedent to deny a claim for refund solely on the basis of noncompliance with RMO
No. 1-2000.

Respondent counters that the requirement of prior application under RMO No. 1-2000 is mandatory in character. RMO No.
1-2000 was issued pursuant to the unquestioned authority of the Secretary of Finance to promulgate rules and regulations
for the effective implementation of the NIRC. Thus, courts cannot ignore administrative issuances which partakes the
nature of a statute and have in their favor a presumption of legality.

The CTA ruled that prior application for a tax treaty relief is mandatory, and noncompliance with this prerequisite is fatal to
the taxpayer’s availment of the preferential tax rate.

We disagree.

A minute resolution is not a binding precedent

At the outset, this Court’s minute resolution on Mirant is not a binding precedent. The Court has clarified this matter in
Philippine Health Care Providers, Inc. v. Commissioner of Internal Revenue 14 as follows:

It is true that, although contained in a minute resolution, our dismissal of the petition was a disposition of the merits of the
case. When we dismissed the petition, we effectively affirmed the CA ruling being questioned. As a result, our ruling in that
case has already become final. When a minute resolution denies or dismisses a petition for failure to comply with formal
and substantive requirements, the challenged decision, together with its findings of fact and legal conclusions, are deemed
sustained. But what is its effect on other cases?

With respect to the same subject matter and the same issues concerning the same parties, it constitutes res judicata.
However, if other parties or another subject matter (even with the same parties and issues) is involved, the minute
resolution is not binding precedent. Thus, in CIR v. Baier-Nickel, the Court noted that a previous case, CIR v. Baier-Nickel
involving the same parties and the same issues, was previously disposed of by the Court thru a minute resolution dated
February 17, 2003 sustaining the ruling of the CA. Nonetheless, the Court ruled that the previous case "ha(d) no bearing"
on the latter case because the two cases involved different subject matters as they were concerned with the taxable
income of different taxable years.

Besides, there are substantial, not simply formal, distinctions between a minute resolution and a decision. The
constitutional requirement under the first paragraph of Section 14, Article VIII of the Constitution that the facts and the law
on which the judgment is based must be expressed clearly and distinctly applies only to decisions, not to minute
resolutions. A minute resolution is signed only by the clerk of court by authority of the justices, unlike a decision. It does
not require the certification of the Chief Justice. Moreover, unlike decisions, minute resolutions are not published in the
Philippine Reports. Finally, the proviso of Section 4(3) of Article VIII speaks of a decision. Indeed, as a rule, this Court lays
down doctrines or principles of law which constitute binding precedent in a decision duly signed by the members of the
Court and certified by the Chief Justice. (Emphasis supplied)

Even if we had affirmed the CTA in Mirant, the doctrine laid down in that Decision cannot bind this Court in cases of a
similar nature. There are differences in parties, taxes, taxable periods, and treaties involved; more importantly, the
disposition of that case was made only through a minute resolution.

Tax Treaty vs. RMO No. 1-2000

Our Constitution provides for adherence to the general principles of international law as part of the law of the land. 15 The
time-honored international principle of pacta sunt servanda demands the performance in good faith of treaty obligations on
the part of the states that enter into the agreement. Every treaty in force is binding upon the parties, and obligations under
the treaty must be performed by them in good faith.16 More importantly, treaties have the force and effect of law in this
jurisdiction.17

Tax treaties are entered into "to reconcile the national fiscal legislations of the contracting parties and, in turn, help the
taxpayer avoid simultaneous taxations in two different jurisdictions." 18 CIR v. S.C. Johnson and Son, Inc. further clarifies
that "tax conventions are drafted with a view towards the elimination of international juridical double taxation, which is
defined as the imposition of comparable taxes in two or more states on the same taxpayer in respect of the same subject
matter and for identical periods. The apparent rationale for doing away with double taxation is to encourage the free flow
of goods and services and the movement of capital, technology and persons between countries, conditions deemed vital
in creating robust and dynamic economies. Foreign investments will only thrive in a fairly predictable and reasonable
international investment climate and the protection against double taxation is crucial in creating such a climate." 19

Simply put, tax treaties are entered into to minimize, if not eliminate the harshness of international juridical double taxation,
which is why they are also known as double tax treaty or double tax agreements.
"A state that has contracted valid international obligations is bound to make in its legislations those modifications that may
be necessary to ensure the fulfillment of the obligations undertaken." 20 Thus, laws and issuances must ensure that the
reliefs granted under tax treaties are accorded to the parties entitled thereto. The BIR must not impose additional
requirements that would negate the availment of the reliefs provided for under international agreements. More so, when
the RP-Germany Tax Treaty does not provide for any pre-requisite for the availment of the benefits under said agreement.

Likewise, it must be stressed that there is nothing in RMO No. 1-2000 which would indicate a deprivation of entitlement to
a tax treaty relief for failure to comply with the 15-day period. We recognize the clear intention of the BIR in implementing
RMO No. 1-2000, but the CTA’s outright denial of a tax treaty relief for failure to strictly comply with the prescribed period
is not in harmony with the objectives of the contracting state to ensure that the benefits granted under tax treaties are
enjoyed by duly entitled persons or corporations.

Bearing in mind the rationale of tax treaties, the period of application for the availment of tax treaty relief as required by
RMO No. 1-2000 should not operate to divest entitlement to the relief as it would constitute a violation of the duty required
by good faith in complying with a tax treaty. The denial of the availment of tax relief for the failure of a taxpayer to apply
within the prescribed period under the administrative issuance would impair the value of the tax treaty. At most, the
application for a tax treaty relief from the BIR should merely operate to confirm the entitlement of the taxpayer to the relief.

The obligation to comply with a tax treaty must take precedence over the objective of RMO No. 1-2000.  Logically,
1âwphi1

noncompliance with tax treaties has negative implications on international relations, and unduly discourages foreign
investors. While the consequences sought to be prevented by RMO No. 1-2000 involve an administrative procedure, these
may be remedied through other system management processes, e.g., the imposition of a fine or penalty. But we cannot
totally deprive those who are entitled to the benefit of a treaty for failure to strictly comply with an administrative issuance
requiring prior application for tax treaty relief.

Prior Application vs. Claim for Refund

Again, RMO No. 1-2000 was implemented to obviate any erroneous interpretation and/or application of the treaty
provisions. The objective of the BIR is to forestall assessments against corporations who erroneously availed themselves
of the benefits of the tax treaty but are not legally entitled thereto, as well as to save such investors from the tedious
process of claims for a refund due to an inaccurate application of the tax treaty provisions. However, as earlier discussed,
noncompliance with the 15-day period for prior application should not operate to automatically divest entitlement to the tax
treaty relief especially in claims for refund.

The underlying principle of prior application with the BIR becomes moot in refund cases, such as the present case, where
the very basis of the claim is erroneous or there is excessive payment arising from non-availment of a tax treaty relief at
the first instance. In this case, petitioner should not be faulted for not complying with RMO No. 1-2000 prior to the
transaction. It could not have applied for a tax treaty relief within the period prescribed, or 15 days prior to the payment of
its BPRT, precisely because it erroneously paid the BPRT not on the basis of the preferential tax rate under

the RP-Germany Tax Treaty, but on the regular rate as prescribed by the NIRC. Hence, the prior application requirement
becomes illogical. Therefore, the fact that petitioner invoked the provisions of the RP-Germany Tax Treaty when it
requested for a confirmation from the ITAD before filing an administrative claim for a refund should be deemed substantial
compliance with RMO No. 1-2000.

Corollary thereto, Section 22921 of the NIRC provides the taxpayer a remedy for tax recovery when there has been an
erroneous payment of tax.  The outright denial of petitioner’s claim for a refund, on the sole ground of failure to apply for a
1âwphi1

tax treaty relief prior to the payment of the BPRT, would defeat the purpose of Section 229.

Petitioner is entitled to a refund

It is significant to emphasize that petitioner applied – though belatedly – for a tax treaty relief, in substantial compliance
with RMO No. 1-2000. A ruling by the BIR would have confirmed whether petitioner was entitled to the lower rate of 10%
BPRT pursuant to the RP-Germany Tax Treaty.

Nevertheless, even without the BIR ruling, the CTA Second Division found as follows:

Based on the evidence presented, both documentary and testimonial, petitioner was able to establish the following facts:

a. That petitioner is a branch office in the Philippines of Deutsche Bank AG, a corporation organized and existing
under the laws of the Federal Republic of Germany;

b. That on October 21, 2003, it filed its Monthly Remittance Return of Final Income Taxes Withheld under BIR
Form No. 1601-F and remitted the amount of ₱67,688,553.51 as branch profits remittance tax with the BIR; and

c. That on October 29, 2003, the Bangko Sentral ng Pilipinas having issued a clearance, petitioner remitted to
Frankfurt Head Office the amount of EUR5,174,847.38 (or ₱330,175,961.88 at 63.804 Peso/Euro) representing its
2002 profits remittance.22

The amount of PHP 67,688,553.51 paid by petitioner represented the 15% BPRT on its RBU net income, due for
remittance to DB Germany amounting to PHP 451,257,023.29 for 2002 and prior taxable years. 23
Likewise, both the administrative and the judicial actions were filed within the two-year prescriptive period pursuant to
Section 229 of the NIRC.24

Clearly, there is no reason to deprive petitioner of the benefit of a preferential tax rate of 10% BPRT in accordance with
the RP-Germany Tax Treaty.

Petitioner is liable to pay only the amount of PHP 45,125,702.34 on its RBU net income amounting to PHP
451,257,023.29 for 2002 and prior taxable years, applying the 10% BPRT. Thus, it is proper to grant petitioner a refund
ofthe difference between the PHP 67,688,553.51 (15% BPRT) and PHP 45,125,702.34 (10% BPRT) or a total of PHP
22,562,851.17.

WHEREFORE, premises considered, the instant Petition is GRANTED. Accordingly, the Court of Tax Appeals En Banc
Decision dated 29 May 2009 and Resolution dated 1 July 2009 are REVERSED and SET ASIDE. A new one is hereby
entered ordering respondent Commissioner of Internal Revenue to refund or issue a tax credit certificate in favor of
petitioner Deutsche Bank AG Manila Branch the amount of TWENTY TWO MILLION FIVE HUNDRED SIXTY TWO
THOUSAND EIGHT HUNDRED FIFTY ONE PESOS AND SEVENTEEN CENTAVOS (PHP 22,562,851.17), Philippine
currency, representing the erroneously paid BPRT for 2002 and prior taxable years.

SO ORDERED.

Freedom of Religion, Article III, Section 5


-Republic v Manalo, GR 221029, Apr 4, 2018

This petition for review on certiorari under Rule 45 of the Rules of Court (Rules) seeks to reverse and set aside the
September 18, 2014 Decision  and October 12, 2015 Resolution  of the Court of Appeals (CA) in CA-G.R. CV No. 100076.
1 2

The dispositive portion of the Decision states:

WHEREFORE, the instant appeal is GRANTED. The Decision dated 15 October 2012 of the Regional Trial Court of
Dagupan City, First Judicial Region, Branch 43, in SPEC. PROC. NO. 2012-0005 is REVERSED and SET ASIDE.

Let a copy of this Decision be served on the Local Civil Registrar of San Juan, Metro Manila.

SO ORDERED. 3

The facts are undisputed.

On January 10, 2012, respondent Marelyn Tanedo Manalo (Manalo) filed a petition for cancellation of

Entry of marriage in the Civil Registry of San Juan , Metro Manila, by virtueof a judgment of divorce Japanese court.

Finding the petition to be sufficient in form and in substance, Branch 43 of the Regional Trial Court (RTC) of Dagupan City
set the case for initial hearing on April 25, 2012. The petition and the notice of initial hearing were published once a week
for three consecutive weeks in newspaper of general circulation. During the initial hearing, counsel for Manalo marked the
documentary evidence (consisting of the trial courts Order dated January 25, 2012, affidavit of publication, and issues of
the Northern Journal dated February 21-27, 2012, February 28 - March 5, 2012, and March 6-12, 2012) for purposes of
compliance with the jurisdictional requirements.

The Office of the Solicitor General (OSG) entered its appearance for petitioner Republic of the Philippines authorizing the
Office of the City Prosecutor of Dagupan to appear on its behalf. Likewise, a Manifestation and Motion was filed
questioning the title and/or caption of the petition considering that based on the allegations therein, the proper action
should be a petition for recognition and enforcement of a foreign judgment.

As a result, Manalo moved to admit an Amended Petition, which the court granted. The Amended Petition, which
captioned that if it is also a petition for recognition and enforcement of foreign judgment alleged:

2. That petitioner is previously married in the Philippines to a Japanese national named YOSHINO MINORO as shown by
their Marriage Contract xxx;

3. That recently, a case for divorce was filed by herein [petitioner] in Japan and after die proceedings, a divorce decree
dated December 6, 2011 was rendered by the Japanese Court x x x;

4. That at present, by virtue of the said divorce decree, petitioner and her divorce Japanese husband are no longer living
together and in fact, petitioner and her daughter are living separately from said Japanese former husband;

5. That there is an imperative need to have the entry of marriage in Civil Registry of San Juan, Metro Manila cancelled,
where the petitioner and the former Japanese husband's marriage was previously registered, in order that it would not
appear anymore that petitioner is still married to the said Japanese national who is no longer her husband or is no longer
married to her, she shall not be bothered and disturbed by aid entry of marriage;
6. That this petition is filed principally for the purpose of causing the cancellation of entry of the marriage between the
petitioner and the said Japanese national, pursuant to Rule 108 of the Revised Rules of Court, which marriage was
already dissolved by virtue of the aforesaid divorce decree; [and]

7. That petitioner prays, among others, that together with the cancellation of the said entry of her marriage, that she be
allowed to return and use her maiden surname, MANALO. 4

Manalo was allowed to testify in advance as she was scheduled to leave for Japan for her employment. Among the
documents that were offered and admitted were:

1. Court Order dated January 25, 2012, finding the petition and its attachments to be sufficient in form and in substance;

2. Affidavit of Publication;

3. Issues of the Northern Journal dated February 21-27, 2012, February 28 - March 5, 2012, and March 6-12, 2012;

4. Certificate of Marriage between Manalo and her former Japanese husband;

5. Divorce Decree of Japanese court;

6. Authentication/Certificate issued by the Philippine Consulate General in Osaka, Japan of the Notification of Divorce; and

7. Acceptance of Certificate of Divorce. 5

The OSG did not present any controverting evidence to rebut the allegations of Manalo.

On October 15, 2012, the trial court denied the petition for lack of merit. In ruling that the divorce obtained by Manalo in
Japan should not be recognized, it opined that, based on Article 15 of the New Civil Code, the Philippine law "does not
afford Filipinos the right to file for a divorce whether they are in the country or living abroad, if they are married to Filipinos
or to foreigners, or if they celebrated their marriage in the Philippines or in another country" and that unless Filipinos "are
naturalized as citizens of another country, Philippine laws shall have control over issues related to Filipinos' family rights
and duties, together with the determination of their condition and legal capacity to enter into contracts and civil relations,
inclusing marriages." 6

On appeal, the CA overturned the RTC decision. It held that Article 26 of the Family Code of the Philippines (Family Code)
is applicable even if it was Manalo who filed for divorce against her Japanese husband because the decree may obtained
makes the latter no longer married to the former, capacitating him to remarry. Conformably with Navarro, et al. V. Exec.
Secretary Ermita, et al.  ruling that the meaning of the law should be based on the intent of the lawmakers and in view of
7

the legislative intent behind Article 26, it would be height of injustice to consider Manalo as still married to the Japanese
national, who, in turn, is no longer married to her. For the appellate court, the fact that it was Manalo who filed the divorce
case is inconsequential. Cited as similar to this case was Van Dorn v. Judge Romilo, Jr.  where the mariage between a
8

foreigner an a Filipino was dissolved filed abroad by the latter.

The OSG filed a motion for reconsideration, but it was denied; hence, this petition.

We deny the petition and partially affirm the CA decision.

Divorce, the legal dissolution of a lawful union for a cause arising after the marriage, are of two types: (1) absolute divorce
or a vinculo matrimonii, which terminates the marriage, and (2) limited divorce or a mensa et thoro, which suspends it and
leaves the bond in full force.  In this jurisdiction, the following rules exist:
9

1. Philippine law does not provide for absolute divorce; hence, our courts cannot grant it. 10

2. Consistent with Articles 15  and 17  of the New Civil Code, the marital bond between two Filipinos cannot be
11 12

dissolved even by an absolute divorce obtained abroad. 13

3. An absolute divorce obtained abroad by a couple, who both aliens, may be recognized in the Philippines,
provided it is consistent with their respective national laws. 14

4. In mixed marriages involving a Filipino and a foreigner, the former is allowed to contract a subsequent marriage
in case the absolute divorce is validly obtained abroad by the alien spouse capacitating him or her to remarry. 15

On July 6, 1987, then President Corazon C. Aquino signed into law Executive Order (E.O.) No. 209, otherwise known as
the Family Code of the Philippines, which took effect on August 3, 1988.  Shortly thereafter , E.O. No. 227 was issued on
16

July 17, 1987.  Aside from amending Articles 36 and 39 of the Family Code, a second paragraph was added to Article
17

26.  This provision was originally deleted by the Civil Code Revision Committee (Committee),but it was presented and
18

approved at a Cabinet meeting after Pres. Aquino signed E.O. No. 209.  As modified, Article 26 now states:
19
Art. 26. All marriages solemnized outside the Philippines, in accordance with the laws in force in the where country where
they were solemnized, and valid there as such, shall also be valid in this country, except those prohibited under Articles
35(1), (4), (5) and (6), 36, 37 and 38.

Where a marriage between Filipino citizen and a foreigner is validly celebrated and a divorce is thereafter validly obtained
abroad by the alien spouse capacitating him her to remarry under Philippine law.

Paragraph 2 of Article 26 confers jurisdiction on Philippine courts to extend the effect of a foreign divorce decree to a
Filipino spouse without undergoing trial to determine the validity of the dissolution of the marriage.  It authorizes our courts
20

to adopt the effects of a foreign divorce decree precisely because the Philippines does not allow divorce.  Philippine
21

courts cannot try the case on the merits because it is tantamount to trying a divorce case.  Under the principles of comity,
22

our jurisdiction recognizes a valid divorce obtained by the spouse of foreign nationality, but the legal effects thereof, e.g.,
on custody, care and support of the children or property relations of the spouses, must still be determined by our courts. 23

According to Judge Alicia Sempio-Diy, a member of the Committee, the idea of the amendment is to avoid the absurd
situation of a Filipino as still being married to his or her alien spouse, although the latter is no longer married to the former
because he or she had obtained a divorce abroad that is recognized by his or national law.  The aim was that it would
24

solved the problem of many Filipino women who, under the New Civil Code, are still considered married to their alien
husbands even after the latter have already validly divorced them under their (the husbands') national laws and perhaps
have already married again. 25

In 2005, this Court concluded that Paragraph 2 of Article 26 applies to a case where, at the time of the celebration of the
marriage, the parties were Filipino citizens, but later on, one of them acquired foreign citizenship by naturalization, initiated
a divorce proceeding, and obtained a favorable decree. We held in Republic of the Phils. v. Orbecido III: 26

The jurisprudential answer lies latent in the 1998 case of Quita v. Court of Appeals. In Quita, the parties were, as in this
case, Filipino citizens when they got married. The wife became naturalized American citizen n 1954 and obtained a
divorce in the same year. The court therein hinted, by the way of obiter dictum, that a Filipino divorced by his naturalized
foreign spouse is no longer married under Philippine law and can thus remarry.

Thus, taking into consideration the legislative intent and applying the rule of reason, we hold that Paragraph 2 of Article 26
should be interpreted to include cases involving parties who, at the time of the celebration of the marriage were Filipino
citizens, but later on, one of them becomes naturalized as foreign citizen and obtains divorce decree. The Filipino spouse
should likewise be allowed to remarry as if the other party were foreigner at the time of the solemnization of the marriage.
To rule otherwise would be to sanction absurdity and injustice. x x x

If we are to give meaning to the legislative intent to avoid the absurd situation where the Filipino spouse remains married
to the alien spouse who after obtaining a divorce is no longer married to the Filipino spouse, then the instant case must be
deemed as coming within the contemplation of Paragraph 2 of Article 26.

In view of the foregoing, we state the twin elements for the application of Paragraph 2 of Article 26 as follows:

1. There is a valid marriage that has been celebrated between a Filipino citizen and a foreigner; and

2. A valid divorce is obtained abroad by the alien spouse capacitating him or her to remarry.

The reckoning point is not the citizenship of the parties at the time of the celebration of marriage, but their citizenship at
the time valid divorced obtained abroad by the alien spouse capacitating the latter to remarry.

Now, the Court is tasked to resolve whether, under the same provision, a Filipino citizen has the capacity to remarry under
Philippine law after initiating a divorce proceeding abroad and obtaining a favorable judgment against his or her alien
spouse who is capacitated to remarry. Specifically, Manalo pleads for the recognition of enforcement of the divorced
decree rendered by the Japanese court and for the cancellation of the entry of marriage in the local civil registry " in order
that it would not appear anymore that she is still married to the said Japanese national who is no longer her husband or is
no longer married to her; [and], in the event that [she] decides to be remarried, she shall not be bothered and disturbed by
said entry of marriage," and to use her maiden surname.

We rule in the affirmative.

Both Dacasin v. Dacasin  and Van Dorn  already recognized a foreign divorce decree that was initiated and obtained by
28 29

the Filipino spouse and extended its legal effects on the issues of child custody and property relation, respectively.

In Dacasin, post-divorce, the former spouses executed an Agreement for the joint custody of their minor daughter. Later
on, the husband who is a US citizen, sued his Filipino wife enforce the Agreement, alleging that it was only the latter who
exercised sole custody of their child. The trial court dismissed the action for lack of jurisdiction, on the ground, among
others, that the divorce decree is binding following the "nationality rule" prevailing in this jurisdiction. The husband moved
to reconsider, arguing that the divorce decree obtained by his former wife is void, but it was denied. In ruling that the trial
court has jurisdiction to entertain the suit bu not to enforce the Agreement, which is void, this Court said:

Nor can petitioner rely on the divorce decree's alleged invalidity - not because the Illinois court lacked jurisdiction or that
the divorced decree violated Illinois law, but because the divorce was obtained by his Filipino spouse - to support the
Agreement's enforceability . The argument that foreigners in this jurisdiction are not bound by foreign divorce decrees is
hardly novel. Van Dron v. Romillo settled the matter by holding that an alien spouse of a Filipino is bound by a divorce
decree obtained abroad. There, we dismissed the alien divorcee's Philippine suit for accounting of alleged post-divorce
conjugal property and rejected his submission that the foreign divorce (obtained by the Filipino spouse) is not valid in this
jurisdiction x x x. 30

Van Dorn was decided before the Family Code took into effect. There, a complaint was filed by the ex-husband , who is a
US citizen, against his Filipino wife to render an accounting of a business that was alleged to be a conjugal property and to
be declared with right to manage the same. Van Dorn moved to dismiss the case on the ground that the cause of action
was barred by previous judgment in the divorce proceedings that she initiated, but the trial court denied the motion. On his
part, her ex-husband averred that the divorce decree issued by the Nevada court could not prevail over the prohibitive
laws of the Philippines and its declared national policy; that the acts and declaration of a foreign court cannot, especially if
the same is contrary to public policy, divest Philippine courts of jurisdiction to entertain matters within its jurisdiction . In
dismissing the case filed by the alien spouse, the Court discussed the effect of the foreign divorce on the parties and their
conjugal property in the Philippines. Thus:

There can be no question as to the validity of that Nevada divorce in any of the States of the United States. The decree is
binding on private respondent as an American citizen. For instance, private respondent cannot sue petitioner, as her
husband, in any State of the Union. What he is contending in this case is that the divorce is not valid and binding in this
jurisdiction, the same being contrary to local law and public policy.

Is it true that owing to the nationality principle embodied in Article 15 of the Civil Code, only Philippine nationals are
covered by the policy and morality. However, aliens may obtain divorce abroad, which may be recognized in the
Philippines, provided they are valid according to their national law. In this case, the divorce in Nevada released private
respondent from the marriage from standards of American law, under which divorce dissolves the marriage. As stated
by the Federal Supreme Court of the United States in Atherton vs. Atherton, 45 L. Ed. 794,799:

"The purpose and effect of a decree of divorce from the bond of matrimony by a court of competent jurisdiction are to
change the existing status or domestic relation of husband and wife, and to free them both from the bond. The marriage
tie, when thus severed as stone party, ceases to bind either. A husband without a wife, or a wife without a husband, is
unknown to the law. When the law provides in the nature of penalty, that the guilty party shall not marry again, that party,
as well as the other, is still absolutely feed from the bond of the former marriage."

Thus, pursuant to his national law, private respondent is no longer the husband of petitioner. He would have no standing
to sue in the case below as petitioner's husband entitled to exercise control over conjugal assets. As he is estopped by his
own representation before said court from asserting his right over the alleged conjugal property.

To maintain, as private respondent does, that under our laws, petitioner has to be considered still married to private
respondent and still subject to a wife's obligations under Article 109, et. seq. of the Civil Code cannot be just. Petitioner
should not be obliged to live together with, observe respect and fidelity, and render support to private respondent. The
latter should not continue to be one of her heirs with possible rights to conjugal property. She should not be discriminated
against in her own country if the ends of justice are to be served. 31

In addition, the fact that a validity obtained foreign divorce initiated by the Filipino spouse can be recognized and given
legal effects in the Philippines is implied from Our rulings in Fujiki v. Marinay, et al.  and Medina v. Koike.
32 33

In Fujiki, the Filipino wife, with the help of her husband, who is a Japanese national, was able to obtain a judgment from
Japan's family court. Which declared the marriage between her and her second husband, who is a Japanese national,
void on the ground of bigamy. In resolving the issue of whether a husband or wife of a prior marriage can file a petition to
recognize a foreign judgment nullifying the subsequent marriage between his her spouse and a foreign citizen on the
ground of bigamy, We ruled:

Fujiki has the personality to file a petition to recognize the Japanese Family Court judgment nullifying the marriage
between Marinay and Maekara on the ground of bigamy because the judgment concerns his civil status as married to
Marinay. For the same reason he has the personality to file a petition under Rule 108 to cancel the entry of marriage
between Marinay and Maekara in the civil registry on the basis of the decree of the Japanese Family Court.

There is no doubt that the prior spouse has a personal and material interest in maintaining the integrity of the marriage he
contracted and the property relations arising from it. There is also no doubt that he is interested in the cancellation of an
entry of a bigamous marriage in the civil registry, which compromises the public record of his marriage. The interest
derives from the substantive right of the spouse not only to preserve (or dissolve, in limited instances) his most intimate
human relation, but also to protect his property interests that arise by operation of law the moment he contracts marriage.
These property interests in marriage included the right to be supported "in keeping with the financial capacity of the family"
and preserving the property regime of the marriage.

Property rights are already substantive rights protected by the Constitution, but a spouse's right in a marriage extends
further to relational rights recognized under Title III ("Rights and Obligations between Husband and Wife") of the Family
Code. x x x 34

On the other hand, in Medina, the Filipino wife and her Japanese husband jointly filed for divorce, which was
granted.  Subsequently, she filed a petition before the RTC for judicial recognition of foreign divorce and declaration of
1âwphi1
capacity to remarry pursuant to Paragraph 2 of Article 26. The RTC denied the petition on the ground that the foreign
divorce decree and the national law of the alien spouse recognizing his capacity to obtain a divorce must be proven in
accordance with Sections 24 and 25 of Rule 132 of the Revised Rules on Evidence. This Court agreed and ruled that,
consistent with Corpuz v. Sto. Tomas, et al.  and Garcia v. Recio,  the divorce decree and the national law of the alien
35 36

spouse must be proven. Instead of dismissing the case, We referred it to the CA for appropriate action including the
reception of evidence to determine and resolve the pertinent factual issues.

There is no compelling reason to deviate from the above-mentioned rulings. When this Court recognized a foreign divorce
decree that was initiated and obtained by the Filipino spouse and extended its legal effects on the issues of child custody
and property relation, it should not stop short in a likewise acknowledging that one of the usual and necessary
consequences of absolute divorce is the right to remarry. Indeed, there is no longer a mutual obligation to live together
and observe fidelity. When the marriage tie is severed and ceased to exist, the civil status and the domestic relation of the
former spouses change as both of them are freed from the marital bond.

The dissent is of the view that, under the nationality principle, Manalo's personal status is subject to Philippine law, which
prohibits absolute divorce. Hence, the divorce decree which she obtained under Japanese law cannot be given effect, as
she is, without dispute, a national not of Japan, bit of the Philippines. It is said that that a contrary ruling will subvert not
only the intention of the framers of the law, but also that of the Filipino peopl, as expressed in the Constitution. The Court
is, therefore, bound to respect the prohibition until the legislature deems it fit to lift the same.

We beg to differ.

Paragraph 2 of Artilce 26 speaksof "a divorce x x x validly obtained abroad by the alien spouse capacitating him or her to
remarry." Based on a clear and plain reading of the provision, it only requires that there be a divorce validly obtained
abroad. The letter of the law does not demand that the alien spouse should be the one who initiated the proceeding
wherein the divorce decree was granted. It does not distinguish whether the Filipino spouse is the petitioner or the
respondent in the foreign divorce proceeding. The Court is bound by the words of the statute; neither can We put words in
the mouth of lawmakers.  The legislature is presumed to know the meaning of the words to have used words advisely and
37

to have expressed its intent by the use of such words as are found in the statute. Verba legis non est recedendum, or from
the words if a statute there should be departure." 38

Assuming, for the sake of argument, that the word "obtained" should be interpreted to mean that the divorce proceeding
must be actually initiated by the alien spouse, still, the Court will not follow the letter of the statute when to do so would
depart from the true intent of the legislature or would otherwise yield conclusions inconsistent with the general purpose of
the act.  Law have ends to achieve, and statutes should be so construed as not to defeat but to carry out such ends and
39

purposes.  As held in League of Cities of the Phils. et al. v. COMELEC et. al.:
40 41

The legislative intent is not at all times accurately reflected in the manner in which the resulting law is couched. Thus,
applying a verba legis or strictly literal interpretation of a statute may render it meaningless and lead to inconvience, an
absurd situation or injustice. To obviate this aberration, and bearing in mind the principle that the intent or the spirit of the
law is the law itself, resort should be to the rule that the spirit of the law control its letter.

To reiterate, the purpose of Paragraph 2 of Article 26 is to avoid the absurd situation where the Filipino spouse remains
married to the alien spouse who, after a foreign divorce decree that is effective in the country where it was rendered, is no
longer married to the Filipino spouse. The provision is a corrective measure is free to marry under the laws of his or her
countr.  Whether the Filipino spouse initiated the foreign divorce proceeding or not, a favorable decree dissolving the
42

marriage bond and capacitating his or her alien spouse to remarry will have the same result: the Filipino spouse will
effectively be without a husband or wife. A Filipino who initiated a foreign divorce proceeding is in the same place and in
like circumstances as a Filipino who is at the receiving end of an alien initiated proceeding. Therefore, the subject
provision should not make a distinction. In both instance, it is extended as a means to recognize the residual effect of the
foreign divorce decree on a Filipinos whose marital ties to their alien spouses are severed by operations of their alien
spouses are severed by operation on the latter's national law.

Conveniently invoking the nationality principle is erroneous. Such principle, found under Article 15 of the City Code, is not
an absolute and unbending rule. In fact, the mer e existence of Paragraph 2 of Article 26 is a testament that the State may
provide for an exception thereto. Moreover, blind adherence to the nationality principle must be disallowed if it would
cause unjust discrimination and oppression to certain classes of individuals whose rights are equally protected by law. The
courts have the duty to enforce the laws of divorce as written by the Legislature only if they are constitutional. 43

While the Congress is allowed a wide leeway in providing for a valid classification and that its decision is accorded
recognition and respect by the court of justice, such classification may be subjected to judicial review.  The deference
44

stops where the classification violates a fundamental right, or prejudices persons accorded special protection by the
Constitution.  When these violations arise, this Court must discharge its primary role as the vanguard of constitutional
45

guaranties, and require a stricter and more exacting adherence to constitutional limitations.  If a legislative classification
46

impermissibly interferes with the exercise of a fundamental right or operates to the peculiar disadvantage of a suspect
class strict judicial scrutiny is required since it is presumed unconstitutional, and the burden is upon the government to
prove that the classification is necessary to achieve a compelling state interest and that it is the least restrictive means to
protect such interest. 47

"Fundamental rights" whose infringement leads to strict scrutiny under the equal protection clause are those basic liberties
explicitly or implicitly guaranteed in the Constitution.  It includes the right to free speech, political expression, press,
48

assembly, and forth, the right to travel, and the right to vote.  On the other hand, what constitutes compelling state interest
49
is measured by the scale rights and powers arrayed in the Constitution and calibrated by history.  It is akin to the
50

paramount interest of the state for which some individual liberties must give way, such as the promotion of public interest,
public safety or the general welfare.  It essentially involves a public right or interest that, because of its primacy, overrides
51

individual rights, and allows the former to take precedence over the latter. 52

Although the Family Code was not enacted by the Congress, the same principle applies with respect to the acts of the
President which have the force and effect of law unless declared otherwise by the court. In this case, We find that
Paragraph 2 of Article 26 violates one of the essential requisites  of the equal protection clause.  Particularly, the limitation
53 54

of the provision only to a foreign divorce decree initiated by the alien spouse is unreasonable as it is based on superficial,
arbitrary, and whimsical classification.

A Filipino who is married to another Filipino is not similarly situated with a Filipino who is married to a foreign citizen. There
are real, material and substantial differences between them. Ergo, they should not be treated alike, both as to rights
conferred and liabilities imposed. Without a doubt, there are political, economic cultural, and religious dissimilarities as well
as varying legal systems and procedures, all too unfamiliar, that a Filipino national who is married to an alien spouse has
to contend with. More importantly, while a divorce decree obtained abroad by a Filipino against another Filipino is null and
void, a divorce decree obtained by an alien against his her Filipino spouse is recognized if made in accordance with the
national law of the foreigner. 55

On the contrary, there is no real and substantial difference between a Filipino who initiated a foreign divorce proceedings a
Filipino who obtained a divorce decree upon the instance of his or her alien spouse . In the eyes of the Philippine and
foreign laws, both are considered as Filipinos who have the same rights and obligations in a alien land. The circumstances
surrounding them are alike. Were it not for Paragraph 2 of Article 26, both are still married to their foreigner spouses who
are no longer their wives/husbands. Hence, to make a distinction between them based merely on the superficial difference
of whether they initiated the divorce proceedings or not is utterly unfair. Indeed, the treatment gives undue favor to one
and unjustly discriminate against the other.

Further, the differentiation in Paragraph 2 Article 26 is arbitrary. There is inequality in treatment because a foreign divorce
decree that was initiated and obtained by a Filipino citizen against his or her alien spouse would not be recognized even if
based on grounds similar to Articles 35, 36, 37 and 38 of the Family Code.  In filing for divorce based on these grounds,
56

the Filipino spouse cannot be accused of invoking foreign law at whim, tantamount to insisting that he or she should be
governed with whatever law he or she chooses. The dissent's comment that Manalo should be "reminded that all is not
lost, for she may still pray for the severance of her martial ties before the RTC in accordance with the mechanism now
existing under the Family Code" is anything but comforting. For the guidance of the bench and the bar, it would have been
better if the dissent discussed in detail what these "mechanism" are and how they specifically apply in Manalo's case as
well as those who are similarly situated. If the dissent refers to a petition for declaration of nullity or annulment of marriage,
the reality is that there is no assurance that our courts will automatically grant the same. Besides, such proceeding is
duplicitous, costly, and protracted. All to the prejudice of our kababayan.

It is argued that the Court's liberal interpretation of Paragraph 2 of Artilce 26 encourages Filipinos to marry foreigners,
opening the floodgate to the indiscriminate practice of Filipinos marrying foreign nationals or initiating divorce proceedings
against their alien spouses.

The supposition is speculative and unfounded.

First, the dissent falls into a hasty generalization as no data whatsoever was sworn to support what he intends to
prove. Second, We adhere to the presumption of good faith in this jurisdiction. Under the rules on evidence, it is disputable
presumed (i.e., satisfactory if uncontradicted and overcome by other evidence) that a person is innocent of crime or
wrong,  that a person takes ordinary care of his concerns,  that acquiescence resulted from a belief that the thing
57 59

acquiesced in was conformable to the law and fact,   that a man and woman deporting themselves as husband and wife
60

have entered into a lawful contract of marriage,  and that the law has been obeyed.  It is whimsical to easily attribute any
61 62

illegal, irregular or immoral conduct on the part of a Filipino just because he or she opted to marry a foreigner instead of a
fellow Filipino. It is presumed that interracial unions are entered into out of genuine love and affection, rather than
prompted by pure lust or profit. Third, We take judicial notice of the fact that Filipinos are relatively more forbearing and
conservative in nature and that they are more often the victims or losing end of mixed marriages. And Fourth, it is not for
Us to prejudge the motive behind Filipino's decision to marry an alien national. In one case, it was said:

Motive for entering into a marriage are varied and complex. The State does not and cannot dictated on the kind of life that
a couple chooses to lead. Any attempt to regulate their lifestyle would go into the realm of their right to privacy and would
raise serious constitutional questions. The right marital privacy allows married couples to structure their marriages in
almost any way they see it fit, to live together or live apart, to have children or no children, to love one another or not, and
so on. Thus, marriages entered into for other purposes, limited or otherwise, such as convenience, companionship,
money, status, and title, provided that they comply with all the legal requisites, are equally valid. Love, though the ideal
consideration in a marriage contract, is not the only valid cause for marriage. Other considerations, not precluded by law,
may validly support a marriage. 63

The 1987 Constitution expresses that marriage, as an inviolable social institution, is the foundation of the family and shall
be protected by the State.  Nevertheless, it was not meant to be a general prohibition on divorce because Commissioner
64

Jose Luis Martin C. Gascon, in response to a question by Father Joaquin G. Bernas during the deliberations of the 1986
Constitutional Commission, was categorical about this point.  Their exchange reveal as follows:
65

MR. RAMA. Mr. Presiding Officer, may I ask that Commissioner Bernas be recognized.
THE PRESIDING OFFICER (Mr. Colayco). Commissioner Bernas is recognized.

FR. BERNAS. Just one question, and I am not sure if it has been categorically answered. I refer specifically to the
proposal of Commissioner Gascon. Is this be understood as a prohibition of a general law on divorce? His intention is to
make this a prohibition so that the legislature cannot pass a divorce law.

MR. GASCON. Mr. Presding Officer, that was not primarily my intention. My intention was primarily to encourage the
social institution of marriage, but not necessarily discourage divorce. But now that the mentioned the issue of divorce, my
personal opinion is to discourage it. Mr. Presiding Officer.

FR. BERNAS. No my question is more categorical. Does this carry the meaning of prohibiting a divorce law?

MR. GASCON. No Mr. Presiding Officer.

FR. BERNAS. Thank you. 66

Notably, a law on absolute divorce is not new in our country. Effectivity March 11, 1917, Philippine courts could grant an
absolute divorce in the grounds of adultery on the part of the wife or concubinage on the part of the husband by virtue of
Act No. 2710 of the Philippine Legislature.  On March 25, 1943, pursuant to the authority conferred upon him by the
67

Commander-in-Chief fo the Imperial Japanese Forces in the Philippines and with the approval of the latter, the Chairman
of the Philippine Executive Commission promulgated an E.O. No. 141 ("New Divorce Law"), which repealed Act No. 2710
and provided eleven ground for absolute divorce, such as intentional or unjustified desertion continuously for at least one
year prior to the filing of the action, slander by deed or gross insult by one spouse against the other to such an extent as to
make further living together impracticable, and a spouse's incurable insanity.  When the Philippines was liberated and the
68

Commonwealth Government was restored, it ceased to have force and effect and Act No. 2710 again prevailed.  From 69

August 30, 1950, upon the effectivity of Republic Act No. 836 or the New Civil Code, an absolute divorce obatined by
Filipino citizens, whether here or abroad, is no longer recognized. 70

Through the years, there has been constant clamor from various sectors of the Philippine society to re-institute absolute
divorce. As a matte of fcat, in the currnet 17th Congress, House Bill (H.B.) Nos. 116  1062  2380  and 6027  were filed in
71 72 73 74

the House of representatives. In substitution of these bills, H.B. No. 7303 entitled "An Act Instituting Absolute Divorce and
Dissolution of Marriage in the Philippines" or the Absolute Divorce Act of 2018 was submitted by the House Committee on
Population

And Family Relations of February 8, 2018. It was approved on March 19, 2018 on Third Reading - with 134 in favor, 57
against, and 2 absentations. Under the bill, the grounds for a judicial decree of absolute divorce are as follows:

1. The grounds for legal separation under Article 55 of the Family Code, modified or amended, as follows:

a. Physical violence or grossly abusive conduct directed against the petitioner, a common child, or a child of the
petitioner;

b. Physical violence or moral pressure to compel the petitioner to change religious or political affiliation;

c. Attempt of respondent to corrupt or induce the petitioner, a common child, or a child of a petitioner, to engage in
prostitution, or connivance in such corruption or inducement;

d. Final judgment sentencing the respondent to imprisonment of more than six (6) years, even if pardoned;

e. Drug addiction or habitual alchoholism ro chronic gambling of respondent;

f. Homosexuality of the respondent;

g. Contracting by the respondent of a subsequent bigamous marriage, whether in the Philippines or abroad;

h. Marital infidelity or perversion or having a child with another person other than one's spouse during the
marriage, except when upon the mutual agreement of the spouses, a child is born to them by in vitro or a similar
procedure or when the wife bears a child after being a victim of rape;

i. attempt by the respondent against the life of the petitioner, a common child or a child of a petitioner; and

j. Abandonment of petitioner by respondent without justifiable cause for more than one (1) year.

When the spouses are legally separated by judicial decree for more thath two (2) years, either or both spouses can
petition the proper court for an absolute divorce based on said judicial decree of legal separation.

1. Grounds for annulment of marriage under Article 45 of the Family Code restated as follows:

a. The party in whose behalf it is sought to have the marriage annulled was eighteen (18) years of age or over but
below twety-one (21), and the marriage was solemnized without the consent of the parents guradian or personl
having substitute parental authority over the party, in that order, unless after attaining the age of twenty-one (21)
such party freely cohabited with the other and both lived together as husband and wife;

b. either party was of unsound mind, unless such party after coming to reason, freely cohabited with the other as
husband and wife;

c. The consent of either party was obtained by fraud, unless such party afterwards with full knowledge of the facts
constituting the fraud, freely cohabited with the other husband and wife;

d. consent of either party was obtained by force, intimidation or undue influence, unless the same having
disappeared or ceased, such party thereafter freely cohabited with the other as husband and wife;

e. Either party was physically incapable of consummating the marriage with the other and such incapacity
continues or appears to be incurable; and

f. Either part was afflicted with the sexually transmissible infection found to be serious or appears to be incurable.

Provided, That the ground mentioned in b, e and f existed either at the time of the marriage or supervening after the
marriage.

1. When the spouses have been separated in fact for at least five (5) years at the time the petition for absolute divorce is
filed, and the reconciliation is highly improbable;

2. Psychological incapacity of either spouse as provided for in Article 36 of the Family Code, whether or not the incapacity
was present at the time of the celebration of the marriage or later;

3. When one of the spouses undergoes a gender reassignment surgery or transition from one sex to another, the other
spouse is entitled to petition for absolute divorce with the transgender or transsexual as respondent, or vice-versa;

4. Irreconcilable marital differences and conflicts which have resulted in the total breakdown of the marriage beyond
repair, despite earnest and repeated efforts at reconciliation.

To be sure, a good number of Filipinos led by the Roman Catholic Church react adversely to any attempt to enact a law on
absolute divorce, viewing it as contrary to our customs, morals, and traditions that has looked upon marriage and family as
an institution and their nature of permanence,

In the same breath that the establishment clause restricts what the government can do with religion, it also limits what
religious sects can or cannot do. They can neither cause the government to adopt their particular doctrines as policy for
everyone, nor can they cause the government to restrict other groups. To do so, in simple terms, would cause the State to
adhere to a particular religion and, thus establish a state religion.
76

The Roman Catholic Church can neither impose its beliefs and convictions on the State and the rest of the citizenry nor
can it demand that the nation follow its beliefs, even if it is sincerely believes that they are good for country.  While
77

marriage is considered a sacrament, it has civil and legal consequences which are governed by the Family Code.  It is in 78

this aspect, bereft of any ecclesiastical overtone, that the State has a legitimate right and interest to regulate.

The declared State policy that marriage, as an inviolable social institution, is a foundation of the family and shall be
protected by the State, should not be read in total isolation but must be harmonized with other constitutional provision.
Aside from strengthening the solidarity of the Filipino family, the State is equally mandated to actively promote its total
development.  It is also obligated to defend, among others, the right of children to special protection from all forms of
79

neglect, abuse, cruelty, exploitation, and other conditions prejudicial to their development.  To Our mind, the State cannot
80

effectively enforce these obligation s if We limit the application of Paragraph 2 or Article 26 only those foreign divorce
initiated by the alien spouse. It is not amiss to point that the women and children are almost always the helpless victims of
all forms of domestic abuse and violence. In fact, among the notable legislation passed in order to minimize, if not
eradicate, the menace are R.A. No. 9262 ("Anti-Violence Against Women and Their Children Act of 2004") R.A. No.
9710 ("The Magna Carta of Women"), R.A. No 10354 ("The Responsible Parenthood and Reproductive Health Act of
2012") and R.A. No 9208 ("Anti-Trafficking in Person Act of 2003"), as amended by R.A. No. 10364 ("ExpandedAnti-
Trafficking in Persons Act of 2012"). Moreover, in protecting and strengthening the Filipino family as a basic autonomous
social institution, the Court must not lose sight of the constitutional mandate to value the dignity of every human person,
guarantee full respect for human rights, and ensure the fundamental equality before the law of women and men. 81

A prohibitive view of Paragraph 2 of Article 26 would do more harm than good. If We disallow a Filipino citizen who
initiated and obtained a foreign divorce from the coverage of Paragraph 2 Article 26 and still require him or her to first avail
of the existing "mechanisms" under the Family Code, any subsequent relationship that he or she would enter in the
meantime shall be considered as illicit in the eyes of the Philippine law. Worse, any child born out such "extra-marital"
affair has to suffer the stigma of being branded as illegitimate. Surely, these are just but a few of the adverse
consequences, not only to the parent but also to the child, if We are to hold a restrictive interpretation of the subject
provision. The irony is that the principle of inviolability of marriage under Section 2, Article XV of the Constitution is meant
to be tilted in favor of marriage and against unions not formalized by marriage, but without denying State protection and
assistance to live-in arrangements or to families formed according to indigenous customs. 82
This Court should not turn a blind eye to the realities of the present time. With the advancement of communication and
information technology, as well as the improvement of the transportation system that almost instantly connect people from
all over the world, mixed marriages have become not too uncommon. Likewise, it is recognized that not all marriages are
made in heaven and that imperfect humans more often than not create imperfect unions.  Living in a flawed world, the
83

unfortunate reality for some is that the attainment of the individual's full human potential and self fulfillment is not found
and achieved in the context of a marriage. Thus it is hypocritical to safeguard the quantity of existing marriages and, at the
same time, brush aside the truth that some of them are rotten quality.

Going back, we hold that marriage, being a mutual and shared commitment between two parties, cannot possibly be
productive of any good to the society where one is considered released from the marital bond while the other remains
bound to it.  In reiterating that the Filipino spouse should not be discriminated against in his or her own country if the ends
84

of justice are to be served, San Luis v. San Luis  quoted:


85

x x x In Alonzo v. Intermediate Applellate Court, the Court stated:

But as has also been aptly observed, we test a law by its results: and likewise, we may add, by its purposes. It is a
cardinal rule that, in seeking the meaning of the law, the first concern of the judge should be to discover in its provisions
the intent of the lawmaker. Unquestionably, the law should never be interpreted in such a way as to cause injustice as this
is never within the legislative intent. An indispensable part of that intent, in fact, for we presume the good motives of the
legislature, is to render justice.

Thus, we interpret and apply the law not independently of but in consonance with justice. Law and justice are inseparable,
and we must keep them so. To be sure, there are some laws that, while generally valid, may seem arbitrary when applied
in a particular case because only of our nature and functions, to apply them just the same, in slavish obedience to their
language. What we do instead is find a balance between the sord and the will, that justice may be done even as the law is
obeyed.

As judges, we are not automatons. We do not and must not unfeelingly apply the law as it worded, yielding like robots to
the literal command without regard to its cause and consequence. "Courts are apt to err by sticking too closely to the
words of law," so we are warned, by Justice Holmes agaian, "where these words import a policy that goes beyond them."

xxxx

More that twenty centuries ago, Justinian defined justice "as the constant and perpetual wish to render every one of his
due." That wish continues to motivate this Court when it assesses the facts and the law in ever case brought to it for
decisions. Justice is always an essential ingredient of its decisions. Thus when the facts warrant, we interpret the law in a
way that will render justice, presuming that it was the intention if the lawmaker, to begin with, that the law be dispensed
with justice. 86

Indeed, where the interpretation of a statute according to its exact and literal import would lead to mischievous results or
contravene the clear purpose of the legislature, it should be construed according to its spirit and reason, disregarding as
far as necessary the letter of the law.  A statute may therefore, be extended to cases not within the literal meaning of its
87

terms, so long as they come within its spirit or intent.


88

The foregoing notwithstanding, We cannot yet write finis to this controversy by granting Manalo's petition to recognize and
enforce the divorce decree rendered by the Japanese court and to cancel the entry of marriage in the Civil Registry of San
Juan, Metro Manila.

Jurisprudence has set guidelines before the Philippine courts recognize a foreign judgment relating to the status of a
marriage where one of the parties is a citizen of foreign country. Presentation solely of the divorce decree will not
suffice.  The fact of divorce must still first be proven.  Before a a foreign divorce decree can be recognized by our courts,
89 90

the party pleading it must prove the divorce as a fact and demonstrate its conformity to the foreign law allowing it. 91

x x x Before a foreign judgment is given presumptive evidentiary value, the document must first be presented and admitted
in evidence. A divorce obtained abroad is proven by the divorce decree itself. The decree purports to be written act or
record of an act of an official body or tribunal of foreign country.

Under Sections 24 and 25 of Rule 132, on the other hand, a writing or document may be proven as a public or official
record of a foreign country by either (1) an official publication or (2) a copy thereof attested by the officer having legal
custody of the document. If the record is not kept in the Philippines, such copy must be (a) accompanied by a certificate
issued by the proper diplomatic or consular officer in the Philippine foreign service stationed in the foreign country in which
the record is kept and (b)authenticated by the seal of his office.
92

In granting Manalo's petition, the CA noted:

In this case, Petitioner was able to submit before the court a quo the 1) Decision of the Japanese Court allowing the
divorce; 2) the Authentication/Certificate issued by the Philippines Consulate General in Osaka, Japan of the Decree of
Divorce; and 3) Acceptance of Certificate of Divorce byu the Petitioner and the Japanese national. Under Rule 132,
Sections 24 and 25, in relation to Rule 39, Section 48 (b) of the Rules of Court, these documents sufficiently prove the
subject Divorce Decree as a fact. Thus, We are constrained to recognize the Japanese Court's judgment decreeing the
divorce.93
If the opposing party fails to properly object, as in this case, the divorce decree is rendered admissible a a written act of
the foreign court.  As it appears, the existence of the divorce decree was not denied by the OSG; neither was the
94

jurisdiction of the divorce court impeached nor the validity of its proceedings challenged on the ground of collusion, fraud,
or clear mistake of fact or law, albeit an opportunity to do so.95

Nonetheless, the Japanese law on divorce must still be proved.

x x x The burden of proof lies with the "party who alleges the existence of a fact or thing necessary in the prosecution or
defense of an action." In civil cases, plaintiffs have the burden of proving the material defendants have the burden of
proving the material allegations in their answer when they introduce new matters. x x x

It is well-settled in our jurisdiction that our courts cannot take judicial notice of foreign laws. Like any other facts, they must
alleged and proved. x x x The power of judicial notice must be exercise d with caution, and every reasonable doubt upon
the subject should be resolved in the negative. 96

Since the divorce was raised by Manalo, the burden of proving the pertinent Japanese law validating it, as well as her
former husband's capacity to remarry, fall squarely upon her. Japanese laws on persons and family relations are not
among those matters that Filipino judges are supposed to know by reason of their judicial function.

WHEREFORE, the petition for review on certiorari is DENIED. The September 18, 2014 Decision and October 12, 2015
Resolution if the Court of Appeals in CA G.R. CV. No. 100076, are AFFIRMED IN PART. The case is REMANDED to the
court of origin for further proceedings and reception of evidence as to the relevant Japanese law on divorce.

SO ORDERED

Treaty vs Executive Agreement


-Saguisag v Executive Secretary Ochoa, GR 212426, Jul 26, 2016 (EDCA)

The petitions  before this Court question the constitutionality of the Enhanced Defense Cooperation Agreement (EDCA)
1

between the Republic of the Philippines and the United States of America (U.S.). Petitioners allege that respondents
committed grave abuse of discretion amounting to lack or excess of jurisdiction when they entered into EDCA with the
U.S.,  claiming that the instrument violated multiple constitutional provisions.  In reply, respondents argue that petitioners
2 3

lack standing to bring the suit. To support the legality of their actions, respondents invoke the 1987 Constitution, treaties,
and judicial precedents. 4

A proper analysis of the issues requires this Court to lay down at the outset the basic parameters of the constitutional
powers and roles of the President and the Senate in respect of the above issues. A more detailed discussion of these
powers and roles will be made in the latter portions.

I. BROAD CONSTITUTIONAL CONTEXT OF THE POWERS OF THE PRESIDENT: DEFENSE, FOREIGN RELATIONS,
AND EDCA

A. The Prime Duty of the State and the Consolidation of Executive Power in the President

Mataimtim kong pinanunumpaan (o pinatotohanan) na tutuparin ko nang buong katapatan at sigasig ang aking mga
tungkulin bilang Pangulo (o Pangalawang Pangulo o Nanunungkulang Pangulo) ng Pilipinas, pangangalagaan at
ipagtatanggol ang kanyang Konstitusyon, ipatutupad ang mga batas nito, magiging makatarungan sa bawat tao, at
itatalaga ang aking sarili sa paglilingkod sa Bansa. Kasihan nawa aka ng Diyos.

- Panunumpa sa Katungkulan ng Pangulo ng Pilipinas ayon sa Saligang Batas 5

The 1987 Constitution has "vested the executive power in the President of the Republic of the Philippines."  While the
6

vastness of the executive power that has been consolidated in the person of the President cannot be expressed fully in
one provision, the Constitution has stated the prime duty of the government, of which the President is the head:

The prime duty of the Government is to serve and protect the people. The Government may call upon the people to
defend the State and, in the fulfillment thereof, all citizens may be required, under conditions provided by law, to render
personal military or civil service.  (Emphases supplied)
7

B. The duty to protect the territory and the citizens of the Philippines, the power to call upon the people to defend
the State, and the President as Commander-in-Chief

The duty to protect the State and its people must be carried out earnestly and effectively throughout the whole territory of
the Philippines in accordance with the constitutional provision on national territory. Hence, the President of the Philippines,
as the sole repository of executive power, is the guardian of the Philippine archipelago, including all the islands and waters
embraced therein and all other territories over which it has sovereignty or jurisdiction. These territories consist of its
terrestrial, fluvial, and aerial domains; including its territorial sea, the seabed, the subsoil, the insular shelves, and other
submarine areas; and the waters around, between, and connecting the islands of the archipelago, regardless of their
breadth and dimensions. 8
To carry out this important duty, the President is equipped with authority over the Armed Forces of the Philippines
(AFP),  which is the protector of the people and the state. The AFP's role is to secure the sovereignty of the State and the
9

integrity of the national territory.  In addition, the Executive is constitutionally empowered to maintain peace and order;
10

protect life, liberty, and property; and promote the general welfare. 11

In recognition of these powers, Congress has specified that the President must oversee, ensure, and reinforce our
defensive capabilities against external and internal threats  and, in the same vein, ensure that the country is adequately
12

prepared for all national and local emergencies arising from natural and man-made disasters. 13

To be sure, this power is limited by the Constitution itself. To illustrate, the President may call out the AFP to prevent or
suppress instances of lawless violence, invasion or rebellion,  but not suspend the privilege of the writ of habeas corpus
14

for a period exceeding 60 days, or place the Philippines or any part thereof under martial law exceeding that same span.
In the exercise of these powers, the President is also duty-bound to submit a report to Congress, in person or in writing,
within 48 hours from the proclamation of martial law or the suspension of the privilege of the writ of habeas corpus; and
Congress may in turn revoke the proclamation or suspension. The same provision provides for the Supreme Court's
review of the factual basis for the proclamation or suspension, as well as the promulgation of the decision within 30 days
from filing.

C. The power and duty to conduct foreign relations

The President also carries the mandate of being the sole organ in the conduct of foreign relations.  Since every state has
15

the capacity to interact with and engage in relations with other sovereign states,  it is but logical that every state must vest
16

in an agent the authority to represent its interests to those other sovereign states.

The conduct of foreign relations is full of complexities and consequences, sometimes with life and death significance to the
nation especially in times of war. It can only be entrusted to that department of government which can act on the basis of
the best available information and can decide with decisiveness. x x x It is also the President who possesses the most
comprehensive and the most confidential information about foreign countries for our diplomatic and consular officials
regularly brief him on meaningful events all over the world. He has also unlimited access to ultra-sensitive military
intelligence data. In fine, the presidential role in foreign affairs is dominant and the President is traditionally accorded a
wider degree of discretion in the conduct of foreign affairs. The regularity, nay, validity of his actions are adjudged under
less stringent standards, lest their judicial repudiation lead to breach of an international obligation, rupture of state
relations, forfeiture of confidence, national embarrassment and a plethora of other problems with equally undesirable
consequences. 17

The role of the President in foreign affairs is qualified by the Constitution in that the Chief Executive must give paramount
importance to the sovereignty of the nation, the integrity of its territory, its interest, and the right of the sovereign Filipino
people to self-determination.  In specific provisions, the President's power is also limited, or at least shared, as in Section
18

2 of Article II on the conduct of war; Sections 20 and 21 of Article VII on foreign loans, treaties, and international
agreements; Sections 4(2) and 5(2)(a) of Article VIII on the judicial review of executive acts; Sections 4 and 25 of Article
XVIII on treaties and international agreements entered into prior to the Constitution and on the presence of foreign military
troops, bases, or facilities.

D. The relationship between the two major presidential functions and the role of the Senate

Clearly, the power to defend the State and to act as its representative in the international sphere inheres in the person of
the President. This power, however, does not crystallize into absolute discretion to craft whatever instrument the Chief
Executive so desires. As previously mentioned, the Senate has a role in ensuring that treaties or international agreements
the President enters into, as contemplated in Section 21 of Article VII of the Constitution, obtain the approval of two-thirds
of its members.

Previously, treaties under the 1973 Constitution required ratification by a majority of the Batasang Pambansa,  except in
19

instances wherein the President "may enter into international treaties or agreements as the national welfare and interest
may require."  This left a large margin of discretion that the President could use to bypass the Legislature altogether. This
20

was a departure from the 1935 Constitution, which explicitly gave the President the power to enter into treaties only with
the concurrence of two-thirds of all the Members of the Senate.  The 1987 Constitution returned the Senate's power  and,
21 22

with it, the legislative's traditional role in foreign affairs.


23

The responsibility of the President when it comes to treaties and international agreements under the present Constitution
is therefore shared with the Senate. This shared role, petitioners claim, is bypassed by EDCA.

II. HISTORICAL ANTECEDENTS OF EDCA

A. U.S. takeover of Spanish colonization and its military bases, and the transition to Philippine independence

The presence of the U.S. military forces in the country can be traced to their pivotal victory in the 1898 Battle of Manila
Bay during the Spanish-American War.  Spain relinquished its sovereignty over the Philippine Islands in favor of the U.S.
24

upon its formal surrender a few months later.  By 1899, the Americans had consolidated a military administration in the
25

archipelago. 26
When it became clear that the American forces intended to impose colonial control over the Philippine Islands, General
Emilio Aguinaldo immediately led the Filipinos into an all-out war against the U.S.  The Filipinos were ultimately defeated 27

in the Philippine-American War, which lasted until 1902 and led to the downfall of the first Philippine Republic.  The 28

Americans henceforth began to strengthen their foothold in the country.  They took over and expanded the former29

Spanish Naval Base in Subic Bay, Zambales, and put up a cavalry post called Fort Stotsenberg in Pampanga, now known
as Clark Air Base. 30

When talks of the eventual independence of the Philippine Islands gained ground, the U.S. manifested the desire to
maintain military bases and armed forces in the country.  The U.S. Congress later enacted the Hare-Hawes-Cutting Act of
31

1933, which required that the proposed constitution of an independent Philippines recognize the right of the U.S. to
maintain the latter's armed forces and military bases.  The Philippine Legislature rejected that law, as it also gave the U.S.
32

the power to unilaterally designate any part of Philippine territory as a permanent military or naval base of the U.S. within
two years from complete independence. 33

The U.S. Legislature subsequently crafted another law called the Tydings-McDuffie Act or the Philippine Independence
Act of 1934. Compared to the old Hare-Hawes-Cutting Act, the new law provided for the surrender to the Commonwealth
Government of "all military and other reservations" of the U.S. government in the Philippines, except "naval reservations
and refueling stations."  Furthermore, the law authorized the U.S. President to enter into negotiations for the adjustment
34

and settlement of all questions relating to naval reservations and fueling stations within two years after the Philippines
would have gained independence.  Under the Tydings-McDuffie Act, the U.S. President would proclaim the American
35

withdrawal and surrender of sovereignty over the islands 10 years after the inauguration of the new government in the
Philippines.  This law eventually led to the promulgation of the 1935 Philippine Constitution.
36

The original plan to surrender the military bases changed.  At the height of the Second World War, the Philippine and the
37

U.S. Legislatures each passed resolutions authorizing their respective Presidents to negotiate the matter of retaining
military bases in the country after the planned withdrawal of the U.S.  Subsequently, in 1946, the countries entered into
38

the Treaty of General Relations, in which the U.S. relinquished all control and sovereignty over the Philippine
Islands, except the areas that would be covered by the American military bases in the country.  This treaty eventually led 39

to the creation of the post-colonial legal regime on which would hinge the continued presence of U.S. military forces until
1991: the Military Bases Agreement (MBA) of 1947, the Military Assistance Agreement of 1947, and the Mutual Defense
Treaty (MDT) of 1951. 40

B. Former legal regime on the presence of U.S. armed forces in the territory of an independent Philippines (1946-
1991)

Soon after the Philippines was granted independence, the two countries entered into their first military arrangement
pursuant to the Treaty of General Relations - the 1947 MBA.  The Senate concurred on the premise of "mutuality of
41

security interest,"  which provided for the presence and operation of 23 U.S. military bases in the Philippines for 99 years
42

or until the year 2046.  The treaty also obliged the Philippines to negotiate with the U.S. to allow the latter to expand the
43

existing bases or to acquire new ones as military necessity might require. 44

A number of significant amendments to the 1947 MBA were made.  With respect to its duration, the parties entered into
45

the Ramos-Rusk Agreement of 1966, which reduced the term of the treaty from 99 years to a total of 44 years or until
1991.  Concerning the number of U.S. military bases in the country, the Bohlen-Serrano Memorandum of Agreement
46

provided for the return to the Philippines of 17 U.S. military bases covering a total area of 117,075 hectares.  Twelve 47

years later, the U.S. returned Sangley Point in Cavite City through an exchange of notes.  Then, through the Romulo- 48

Murphy Exchange of Notes of 1979, the parties agreed to the recognition of Philippine sovereignty over Clark and Subic
Bases and the reduction of the areas that could be used by the U.S. military.  The agreement also provided for the 49

mandatory review of the treaty every five years.  In 1983, the parties revised the 1947 MBA through the Romualdez-
50

Armacost Agreement.  The revision pertained to the operational use of the military bases by the U.S. government within
51

the context of Philippine sovereignty,  including the need for prior consultation with the Philippine government on the
52

former' s use of the bases for military combat operations or the establishment of long-range missiles. 53

Pursuant to the legislative authorization granted under Republic Act No. 9,  the President also entered into the 1947 54

Military Assistance Agreement  with the U.S. This executive agreement established the conditions under which U.S.
55

military assistance would be granted to the Philippines,  particularly the provision of military arms, ammunitions, supplies,
56

equipment, vessels, services, and training for the latter's defense forces.  An exchange of notes in 1953 made it clear that
57

the agreement would remain in force until terminated by any of the parties. 58

To further strengthen their defense and security relationship,  the Philippines and the U.S. next entered into the MDT in
59

1951. Concurred in by both the Philippine  and the U.S.  Senates, the treaty has two main features: first, it allowed for
60 61

mutual assistance in maintaining and developing their individual and collective capacities to resist an armed
attack;  and second, it provided for their mutual self-defense in the event of an armed attack against the territory of either
62

party.  The treaty was premised on their recognition that an armed attack on either of them would equally be a threat to
63

the security of the other. 64

C. Current legal regime on the presence of U.S. armed forces in the country

In view of the impending expiration of the 1947 MBA in 1991, the Philippines and the U.S. negotiated for a possible
renewal of their defense and security relationship.  Termed as the Treaty of Friendship, Cooperation and Security, the
65

countries sought to recast their military ties by providing a new framework for their defense cooperation and the use of
Philippine installations.  One of the proposed provisions included an arrangement in which U.S. forces would be granted
66
the use of certain installations within the Philippine naval base in Subic.  On 16 September 1991, the Senate rejected the
67

proposed treaty. 68

The consequent expiration of the 1947 MBA and the resulting paucity of any formal agreement dealing with the treatment
of U.S. personnel in the Philippines led to the suspension in 1995 of large-scale joint military exercises.  In the meantime,
69

the respective governments of the two countries agreed  to hold joint exercises at a substantially reduced level.  The
70 71

military arrangements between them were revived in 1999 when they concluded the first Visiting Forces Agreement
(VFA).72

As a "reaffirm[ation] [of the] obligations under the MDT,"  the VFA has laid down the regulatory mechanism for the
73

treatment of U.S. military and civilian personnel visiting the country.  It contains provisions on the entry and departure of
74

U.S. personnel; the purpose, extent, and limitations of their activities; criminal and disciplinary jurisdiction; the waiver of
certain claims; the importation and exportation of equipment, materials, supplies, and other pieces of property owned by
the U.S. government; and the movement of U.S. military vehicles, vessels, and aircraft into and within the country.  The 75

Philippines and the U.S. also entered into a second counterpart agreement (VFA II), which in turn regulated the treatment
of Philippine military and civilian personnel visiting the U.S.  The Philippine Senate concurred in the first VFA on 27 May
76

1999.77

Beginning in January 2002, U.S. military and civilian personnel started arriving in Mindanao to take part in joint military
exercises with their Filipino counterparts.  Called Balikatan, these exercises involved trainings aimed at simulating joint
78

military maneuvers pursuant to the MDT. 79

In the same year, the Philippines and the U.S. entered into the Mutual Logistics Support Agreement to "further the
interoperability, readiness, and effectiveness of their respective military forces"  in accordance with the MDT, the Military
80

Assistance Agreement of 1953, and the VFA.  The new agreement outlined the basic terms, conditions, and procedures
81

for facilitating the reciprocal provision of logistics support, supplies, and services between the military forces of the two
countries.  The phrase "logistics support and services" includes billeting, operations support, construction and use of
82

temporary structures, and storage services during an approved activity under the existing military arrangements.  Already
83

extended twice, the agreement will last until 2017. 84

D. The Enhanced Defense Cooperation Agreement

EDCA authorizes the U.S. military forces to have access to and conduct activities within certain "Agreed Locations" in the
country. It was not transmitted to the Senate on the executive's understanding that to do so was no longer
necessary.  Accordingly, in June 2014, the Department of Foreign Affairs (DFA) and the U.S. Embassy exchanged
85

diplomatic notes confirming the completion of all necessary internal requirements for the agreement to enter into force in
the two countries. 86

According to the Philippine government, the conclusion of EDCA was the result of intensive and comprehensive
negotiations in the course of almost two years.  After eight rounds of negotiations, the Secretary of National Defense and
87

the U.S. Ambassador to the Philippines signed the agreement on 28 April 2014.  President Benigno S. Aquino III ratified
88

EDCA on 6 June 2014.  The OSG clarified during the oral arguments  that the Philippine and the U.S. governments had
89 90

yet to agree formally on the specific sites of the Agreed Locations mentioned in the agreement.

Two petitions for certiorari were thereafter filed before us assailing the constitutionality of EDCA. They primarily argue that
it should have been in the form of a treaty concurred in by the Senate, not an executive agreement.

On 10 November 2015, months after the oral arguments were concluded and the parties ordered to file their respective
memoranda, the Senators adopted Senate Resolution No. (SR) 105.  The resolution expresses the "strong sense"  of the
91 92

Senators that for EDCA to become valid and effective, it must first be transmitted to the Senate for deliberation and
concurrence.

III. ISSUES

Petitioners mainly seek a declaration that the Executive Department committed grave abuse of discretion in entering into
EDCA in the form of an executive agreement. For this reason, we cull the issues before us:

A. Whether the essential requisites for judicial review are present

B. Whether the President may enter into an executive agreement on foreign military bases, troops, or
facilities

C. Whether the provisions under EDCA are consistent with the Constitution, as well as with existing laws
and treaties

IV. DISCUSSION

A. Whether the essential requisites for judicial review have been satisfied

Petitioners are hailing this Court's power of judicial review in order to strike down EDCA for violating the Constitution. They
stress that our fundamental law is explicit in prohibiting the presence of foreign military forces in the country, except under
a treaty concurred in by the Senate. Before this Court may begin to analyze the constitutionality or validity of an official act
of a coequal branch of government, however, petitioners must show that they have satisfied all the essential requisites for
judicial review.93

Distinguished from the general notion of judicial power, the power of judicial review specially refers to both the authority
and the duty of this Court to determine whether a branch or an instrumentality of government has acted beyond the scope
of the latter's constitutional powers.  As articulated in Section 1, Article VIII of the Constitution, the power of judicial review
94

involves the power to resolve cases in which the questions concern the constitutionality or validity of any treaty,
international or executive agreement, law, presidential decree, proclamation, order, instruction, ordinance, or
regulation.  In Angara v. Electoral Commission, this Court exhaustively discussed this "moderating power" as part of the
95

system of checks and balances under the Constitution. In our fundamental law, the role of the Court is to determine
whether a branch of government has adhered to the specific restrictions and limitations of the latter's power: 96

The separation of powers is a fundamental principle in our system of government. It obtains not through express provision
but by actual division in our Constitution. Each department of the government has exclusive cognizance of matters
within its jurisdiction, and is supreme within its own sphere. But it does not follow from the fact that the three powers
are to be kept separate and distinct that the Constitution intended them to be absolutely unrestrained and independent of
each other. The Constitution has provided for an elaborate system of checks and balances to secure coordination in
the workings of the various departments of the government. x x x. And the judiciary in turn, with the Supreme Court as
the final arbiter, effectively checks the other departments in the exercise of its power to determine the law, and
hence to declare executive and legislative acts void if violative of the Constitution.

xxxx

As any human production, our Constitution is of course lacking perfection and perfectibility, but as much as it was within
the power of our people, acting through their delegates to so provide, that instrument which is the expression of their
sovereignty however limited, has established a republican government intended to operate and function as a
harmonious whole, under a system of checks and balances, and subject to specific limitations and restrictions
provided in the said instrument. The Constitution sets forth in no uncertain language the restrictions and
limitations upon governmental powers and agencies. If these restrictions and limitations are transcended it would
be inconceivable if the Constitution had not provided for a mechanism by which to direct the course of
government along constitutional channels, for then the distribution of powers would be mere verbiage, the bill of
rights mere expressions of sentiment, and the principles of good government mere political apothegms. Certainly,
the limitations and restrictions embodied in our Constitution are real as they should be in any living constitution. x x x. In
our case, this moderating power is granted, if not expressly, by clear implication from section 2 of article VIII of [the 1935]
Constitution.

The Constitution is a definition of the powers of government. Who is to determine the nature, scope and extent of such
powers? The Constitution itself has provided for the instrumentality of the judiciary as the rational way. And when the
judiciary mediates to allocate constitutional boundaries, it does not assert any superiority over the other
departments; it does not in reality nullify or invalidate an act of the legislature, but only asserts the solemn and
sacred obligation assigned to it by the Constitution to determine conflicting claims of authority under the
Constitution and to establish for the parties in an actual controversy the rights which that instrument secures and
guarantees to them. This is in truth all that is involved in what is termed "judicial supremacy" which properly is the
power of judicial review under the Constitution. x x x x. (Emphases supplied)

The power of judicial review has since been strengthened in the 1987 Constitution. The scope of that power has been
extended to the determination of whether in matters traditionally considered to be within the sphere of appreciation of
another branch of government, an exercise of discretion has been attended with grave abuse.  The expansion of this
97

power has made the political question doctrine "no longer the insurmountable obstacle to the exercise of judicial power or
the impenetrable shield that protects executive and legislative actions from judicial inquiry or review." 98

This moderating power, however, must be exercised carefully and only if it cannot be completely avoided. We stress that
our Constitution is so incisively designed that it identifies the spheres of expertise within which the different branches of
government shall function and the questions of policy that they shall resolve.  Since the power of judicial review involves
99

the delicate exercise of examining the validity or constitutionality of an act of a coequal branch of government, this Court
must continually exercise restraint to avoid the risk of supplanting the wisdom of the constitutionally appointed actor with
that of its own.100

Even as we are left with no recourse but to bare our power to check an act of a coequal branch of government - in this
case the executive - we must abide by the stringent requirements for the exercise of that power under the
Constitution. Demetria v. Alba  and Francisco v. House of Representatives  cite the "pillars" of the limitations on the
101 102

power of judicial review as enunciated in the concurring opinion of U.S. Supreme Court Justice Brandeis in Ashwander v.
Tennessee Valley Authority.  Francisco  redressed these "pillars" under the following categories:
103 104

1. That there be absolute necessity of deciding a case

2. That rules of constitutional law shall be formulated only as required by the facts of the case

3. That judgment may not be sustained on some other ground


4. That there be actual injury sustained by the party by reason of the operation of the statute

5. That the parties are not in estoppel

6. That the Court upholds the presumption of constitutionality

(Emphases supplied)

These are the specific safeguards laid down by the Court when it exercises its power of judicial review.  Guided by these
105

pillars, it may invoke the power only when the following four stringent requirements are satisfied: (a) there is an actual
case or controversy; (b) petitioners possess locus standi; (c) the question of constitutionality is raised at the earliest
opportunity; and (d) the issue of constitutionality is the lis mota of the case.  Of these four, the first two conditions will be
106

the focus of our discussion.

1. Petitioners have shown the presence of an actual case or controversy.

The OSG maintains  that there is no actual case or controversy that exists, since the Senators have not been deprived of
107

the opportunity to invoke the privileges of the institution they are representing. It contends that the nonparticipation of the
Senators in the present petitions only confirms that even they believe that EDCA is a binding executive agreement that
does not require their concurrence.

It must be emphasized that the Senate has already expressed its position through SR 105.  Through the Resolution, the
108

Senate has taken a position contrary to that of the OSG. As the body tasked to participate in foreign affairs by ratifying
treaties, its belief that EDCA infringes upon its constitutional role indicates that an actual controversy - albeit brought to the
Court by non-Senators, exists.

Moreover, we cannot consider the sheer abstention of the Senators from the present proceedings as basis for finding that
there is no actual case or controversy before us. We point out that the focus of this requirement is the ripeness for
adjudication of the matter at hand, as opposed to its being merely conjectural or anticipatory.  The case must involve a
109

definite and concrete issue involving real parties with conflicting legal rights and legal claims admitting of specific relief
through a decree conclusive in nature.  It should not equate with a mere request for an opinion or advice on what the law
110

would be upon an abstract, hypothetical, or contingent state of facts.  As explained in Angara v. Electoral Commission:
111 112

[The] power of judicial review is limited to actual cases and controversies to be exercised after full opportunity of
argument by the parties, and limited further to the constitutional question raised or the very lis mota presented. Any
attempt at abstraction could only lead to dialectics and barren legal questions and to sterile conclusions of
wisdom, justice or expediency of legislation. More than that, courts accord the presumption of constitutionality to
legislative enactments, not only because the legislature is presumed to abide by the Constitution but also because
the judiciary in the determination of actual cases and controversies must reflect the wisdom and justice of the
people as expressed through their representatives in the executive and legislative departments of the
government. (Emphases supplied)

We find that the matter before us involves an actual case or controversy that is already ripe for adjudication. The
Executive Department has already sent an official confirmation to the U.S. Embassy that "all internal requirements of the
Philippines x x x have already been complied with."  By this exchange of diplomatic notes, the Executive Department
113

effectively performed the last act required under Article XII(l) of EDCA before the agreement entered into force. Section
25, Article XVIII of the Constitution, is clear that the presence of foreign military forces in the country shall only be allowed
by virtue of a treaty concurred in by the Senate. Hence, the performance of an official act by the Executive Department
that led to the entry into force of an executive agreement was sufficient to satisfy the actual case or controversy
requirement.

2. While petitioners Saguisag et. al., do not have legal standing, they nonetheless raise issues involving matters
of transcendental importance.

The question of locus standi or legal standing focuses on the determination of whether those assailing the governmental
act have the right of appearance to bring the matter to the court for adjudication.  They must show that they have a
114

personal and substantial interest in the case, such that they have sustained or are in immediate danger of sustaining,
some direct injury as a consequence of the enforcement of the challenged governmental act.  Here, "interest" in the
115

question involved must be material - an interest that is in issue and will be affected by the official act - as distinguished
from being merely incidental or general.  Clearly, it would be insufficient to show that the law or any governmental act is
116

invalid, and that petitioners stand to suffer in some indefinite way.  They must show that they have a particular interest in
117

bringing the suit, and that they have been or are about to be denied some right or privilege to which they are lawfully
entitled, or that they are about to be subjected to some burden or penalty by reason of the act complained of.  The reason
118

why those who challenge the validity of a law or an international agreement are required to allege the existence of a
personal stake in the outcome of the controversy is "to assure the concrete adverseness which sharpens the presentation
of issues upon which the court so largely depends for illumination of difficult constitutional questions." 119

The present petitions cannot qualify as citizens', taxpayers', or legislators' suits; the Senate as a body has the requisite
standing, but considering that it has not formally filed a pleading to join the suit, as it merely conveyed to the Supreme
Court its sense that EDCA needs the Senate's concurrence to be valid, petitioners continue to suffer from lack of standing.
In assailing the constitutionality of a governmental act, petitioners suing as citizens may dodge the requirement of having
to establish a direct and personal interest if they show that the act affects a public right.  In arguing that they have legal
120

standing, they claim  that the case they have filed is a concerned citizen's suit. But aside from general statements that the
121

petitions involve the protection of a public right, and that their constitutional rights as citizens would be violated, they fail to
make any specific assertion of a particular public right that would be violated by the enforcement of EDCA. For their
failure to do so, the present petitions cannot be considered by the Court as citizens' suits that would justify a
disregard of the aforementioned requirements.

In claiming that they have legal standing as taxpayers, petitioners  aver that the implementation of EDCA would result in
122

the unlawful use of public funds. They emphasize that Article X(1) refers to an appropriation of funds; and that the
agreement entails a waiver of the payment of taxes, fees, and rentals. During the oral arguments, however, they admitted
that the government had not yet appropriated or actually disbursed public funds for the purpose of implementing the
agreement.  The OSG, on the other hand, maintains that petitioners cannot sue as taxpayers.  Respondent explains that
123 124

EDCA is neither meant to be a tax measure, nor is it directed at the disbursement of public funds.

A taxpayer's suit concerns a case in which the official act complained of directly involves the illegal disbursement of public
funds derived from taxation.  Here, those challenging the act must specifically show that they have sufficient interest in
125

preventing the illegal expenditure of public money, and that they will sustain a direct injury as a result of the enforcement
of the assailed act.  Applying that principle to this case, they must establish that EDCA involves the exercise by
126

Congress of its taxing or spending powers. 127

We agree with the OSG that the petitions cannot qualify as taxpayers' suits. We emphasize that a taxpayers' suit
contemplates a situation in which there is already an appropriation or a disbursement of public funds.  A reading of Article
128

X(l) of EDCA would show that there has been neither an appropriation nor an authorization of disbursement of funds. The
cited provision reads:

All obligations under this Agreement are subject to the availability of appropriated funds authorized for these
purposes. (Emphases supplied)

This provision means that if the implementation of EDCA would require the disbursement of public funds, the money must
come from appropriated funds that are specifically authorized for this purpose. Under the agreement, before there can
even be a disbursement of public funds, there must first be a legislative action. Until and unless the Legislature
appropriates funds for EDCA, or unless petitioners can pinpoint a specific item in the current budget that allows
expenditure under the agreement, we cannot at this time rule that there is in fact an appropriation or a
disbursement of funds that would justify the filing of a taxpayers' suit.

Petitioners Bayan et al. also claim  that their co-petitioners who are party-list representatives have the standing to
129

challenge the act of the Executive Department, especially if it impairs the constitutional prerogatives, powers, and
privileges of their office. While they admit that there is no incumbent Senator who has taken part in the present petition,
they nonetheless assert that they also stand to sustain a derivative but substantial injury as legislators. They argue that
under the Constitution, legislative power is vested in both the Senate and the House of Representatives; consequently, it
is the entire Legislative Department that has a voice in determining whether or not the presence of foreign military should
be allowed. They maintain that as members of the Legislature, they have the requisite personality to bring a suit,
especially when a constitutional issue is raised.

The OSG counters  that petitioners do not have any legal standing to file the suits concerning the lack of Senate
130

concurrence in EDCA. Respondent emphasizes that the power to concur in treaties and international agreements is an
"institutional prerogative" granted by the Constitution to the Senate. Accordingly, the OSG argues that in case of an
allegation of impairment of that power, the injured party would be the Senate as an institution or any of its incumbent
members, as it is the Senate's constitutional function that is allegedly being violated.

The legal standing of an institution of the Legislature or of any of its Members has already been recognized by this Court
in a number of cases.  What is in question here is the alleged impairment of the constitutional duties and powers granted
131

to, or the impermissible intrusion upon the domain of, the Legislature or an institution thereof.  In the case of suits initiated
132

by the legislators themselves, this Court has recognized their standing to question the validity of any official action that
they claim infringes the prerogatives, powers, and privileges vested by the Constitution in their office.  As aptly explained
133

by Justice Perfecto in Mabanag v. Lopez Vito: 134

Being members of Congress, they are even duty bound to see that the latter act within the bounds of the
Constitution which, as representatives of the people, they should uphold, unless they are to commit a flagrant betrayal
of public trust. They are representatives of the sovereign people and it is their sacred duty to see to it that the
fundamental law embodying the will of the sovereign people is not trampled upon. (Emphases supplied)

We emphasize that in a legislators' suit, those Members of Congress who are challenging the official act have standing
only to the extent that the alleged violation impinges on their right to participate in the exercise of the powers of the
institution of which they are members.  Legislators have the standing "to maintain inviolate the prerogatives, powers, and
135

privileges vested by the Constitution in their office and are allowed to sue to question the validity of any official action,
which they claim infringes their prerogatives as legislators."  As legislators, they must clearly show that there was a direct
136

injury to their persons or the institution to which they belong. 137

As correctly argued by respondent, the power to concur in a treaty or an international agreement is an institutional
prerogative granted by the Constitution to the Senate, not to the entire Legislature. In Pimentel v. Office of the Executive
Secretary, this Court did not recognize the standing of one of the petitioners therein who was a member of the House of
Representatives. The petition in that case sought to compel the transmission to the Senate for concurrence of the signed
text of the Statute of the International Criminal Court. Since that petition invoked the power of the Senate to grant or
withhold its concurrence in a treaty entered into by the Executive Department, only then incumbent Senator Pimentel was
allowed to assert that authority of the Senate of which he was a member.

Therefore, none of the initial petitioners in the present controversy has the standing to maintain the suits as
legislators.

Nevertheless, this Court finds that there is basis for it to review the act of the Executive for the following reasons.

In any case, petitioners raise issues involving matters of transcendental importance.

Petitioners  argue that the Court may set aside procedural technicalities, as the present petition tackles issues that are of
138

transcendental importance. They point out that the matter before us is about the proper exercise of the Executive
Department's power to enter into international agreements in relation to that of the Senate to concur in those agreements.
They also assert that EDCA would cause grave injustice, as well as irreparable violation of the Constitution and of the
Filipino people's rights.

The OSG, on the other hand, insists  that petitioners cannot raise the mere fact that the present petitions involve matters
139

of transcendental importance in order to cure their inability to comply with the constitutional requirement of standing.
Respondent bewails the overuse of "transcendental importance" as an exception to the traditional requirements of
constitutional litigation. It stresses that one of the purposes of these requirements is to protect the Supreme Court from
unnecessary litigation of constitutional questions.

In a number of cases,  this Court has indeed taken a liberal stance towards the requirement of legal standing, especially
140

when paramount interest is involved. Indeed, when those who challenge the official act are able to craft an issue of
transcendental significance to the people, the Court may exercise its sound discretion and take cognizance of the suit. It
may do so in spite of the inability of the petitioners to show that they have been personally injured by the operation of a
law or any other government act.

While this Court has yet to thoroughly delineate the outer limits of this doctrine, we emphasize that not every other case,
however strong public interest may be, can qualify as an issue of transcendental importance. Before it can be impelled to
brush aside the essential requisites for exercising its power of judicial review, it must at the very least consider a number
of factors: (1) the character of the funds or other assets involved in the case; (2) the presence of a clear case of disregard
of a constitutional or statutory prohibition by the public respondent agency or instrumentality of the government; and (3)
the lack of any other party that has a more direct and specific interest in raising the present questions. 141

An exhaustive evaluation of the memoranda of the parties, together with the oral arguments, shows that petitioners have
presented serious constitutional issues that provide ample justification for the Court to set aside the rule on standing. The
transcendental importance of the issues presented here is rooted in the Constitution itself. Section 25, Article XVIII thereof,
cannot be any clearer: there is a much stricter mechanism required before foreign military troops, facilities, or bases may
be allowed in the country. The DFA has already confirmed to the U.S. Embassy that "all internal requirements of the
Philippines x x x have already been complied with."  It behooves the Court in this instance to take a liberal stance towards
142

the rule on standing and to determine forthwith whether there was grave abuse of discretion on the part of the Executive
Department.

We therefore rule that this case is a proper subject for judicial review.

B. Whether the President may enter into an executive agreement on foreign military bases, troops, or
facilities

C. Whether the provisions under EDCA are consistent with the Constitution, as well as with existing laws
and treaties

Issues B and C shall be discussed together infra.

1. The role of the President as the executor of the law includes the duty to defend the State, for which purpose he
may use that power in the conduct of foreign relations

Historically, the Philippines has mirrored the division of powers in the U.S. government. When the Philippine government
was still an agency of the Congress of the U.S., it was as an agent entrusted with powers categorized as executive,
legislative, and judicial, and divided among these three great branches.  By this division, the law implied that the divided
143

powers cannot be exercised except by the department given the power. 144

This divide continued throughout the different versions of the Philippine Constitution and specifically vested the supreme
executive power in the Governor-General of the Philippines,  a position inherited by the President of the Philippines when
145

the country attained independence. One of the principal functions of the supreme executive is the responsibility for the
faithful execution of the laws as embodied by the oath of office.  The oath of the President prescribed by the 1987
146

Constitution reads thus:


I do solemnly swear (or affirm) that I will faithfully and conscientiously fulfill my duties as President (or Vice-
President or Acting President) of the Philippines, preserve and defend its Constitution, execute its laws, do justice to
every man, and consecrate myself to the service of the Nation. So help me God. (In case of affirmation, last sentence will
be omitted.)  (Emphases supplied)
147

This Court has interpreted the faithful execution clause as an obligation imposed on the President, and not a separate
grant of power.  Section 1 7, Article VII of the Constitution, expresses this duty in no uncertain terms and includes it in the
148

provision regarding the President's power of control over the executive department, viz:

The President shall have control of all the executive departments, bureaus, and offices. He shall ensure that the laws be
faithfully executed.

The equivalent provisions in the next preceding Constitution did not explicitly require this oath from the President. In the
1973 Constitution, for instance, the provision simply gives the President control over the ministries.  A similar language,
149

not in the form of the President's oath, was present in the 1935 Constitution, particularly in the enumeration of executive
functions.  By 1987, executive power was codified not only in the Constitution, but also in the Administrative Code:
150 151

SECTION 1. Power of Control. - The President shall have control of all the executive departments, bureaus, and
offices. He shall ensure that the laws be faithfully executed. (Emphasis supplied)

Hence, the duty to faithfully execute the laws of the land is inherent in executive power and is intimately related to the
other executive functions. These functions include the faithful execution of the law in autonomous regions;  the right to
152

prosecute crimes;  the implementation of transportation projects;  the duty to ensure compliance with treaties, executive
153 154

agreements and executive orders;  the authority to deport undesirable aliens;  the conferment of national awards under
155 156

the President's jurisdiction;  and the overall administration and control of the executive department.
157 158

These obligations are as broad as they sound, for a President cannot function with crippled hands, but must be capable of
securing the rule of law within all territories of the Philippine Islands and be empowered to do so within constitutional limits.
Congress cannot, for instance, limit or take over the President's power to adopt implementing rules and regulations for a
law it has enacted. 159

More important, this mandate is self-executory by virtue of its being inherently executive in nature.  As Justice Antonio T.
160

Carpio previously wrote, 161

[i]f the rules are issued by the President in implementation or execution of self-executory constitutional powers vested in
the President, the rule-making power of the President is not a delegated legislative power. The most important self-
executory constitutional power of the President is the President's constitutional duty and mandate to "ensure that the laws
be faithfully executed." The rule is that the President can execute the law without any delegation of power from the
legislature.

The import of this characteristic is that the manner of the President's execution of the law, even if not expressly
granted by the law, is justified by necessity and limited only by law, since the President must "take necessary and
proper steps to carry into execution the law."  Justice George Malcolm states this principle in a grand manner:
162 163

The executive should be clothed with sufficient power to administer efficiently the affairs of state. He should have complete
control of the instrumentalities through whom his responsibility is discharged. It is still true, as said by Hamilton, that "A
feeble executive implies a feeble execution of the government. A feeble execution is but another phrase for a bad
execution; and a government ill executed, whatever it may be in theory, must be in practice a bad government." The
mistakes of State governments need not be repeated here.

xxxx

Every other consideration to one side, this remains certain - The Congress of the United States clearly intended that the
Governor-General's power should be commensurate with his responsibility. The Congress never intended that the
Governor-General should be saddled with the responsibility of administering the government and of executing the laws but
shorn of the power to do so. The interests of the Philippines will be best served by strict adherence to the basic principles
of constitutional government.

In light of this constitutional duty, it is the President's prerogative to do whatever is legal and necessary for Philippine
defense interests. It is no coincidence that the constitutional provision on the faithful execution clause was followed by that
on the President's commander-in-chief powers,  which are specifically granted during extraordinary events of lawless
164

violence, invasion, or rebellion. And this duty of defending the country is unceasing, even in times when there is no state
of lawlesss violence, invasion, or rebellion. At such times, the President has full powers to ensure the faithful execution of
the laws.

It would therefore be remiss for the President and repugnant to the faithful-execution clause of the Constitution to do
nothing when the call of the moment requires increasing the military's defensive capabilities, which could include forging
alliances with states that hold a common interest with the Philippines or bringing an international suit against an offending
state.
The context drawn in the analysis above has been termed by Justice Arturo D. Brion's Dissenting Opinion as the
beginning of a "patent misconception."  His dissent argues that this approach taken in analyzing the President's role as
165

executor of the laws is preceded by the duty to preserve and defend the Constitution, which was allegedly overlooked. 166

In arguing against the approach, however, the dissent grossly failed to appreciate the nuances of the analysis, if read
holistically and in context. The concept that the President cannot function with crippled hands and therefore can disregard
the need for Senate concurrence in treaties  was never expressed or implied. Rather, the appropriate reading of the
167

preceding analysis shows that the point being elucidated is the reality that the President's duty to execute the laws and
protect the Philippines is inextricably interwoven with his foreign affairs powers, such that he must resolve issues imbued
with both concerns to the full extent of his powers, subject only to the limits supplied by law. In other words, apart from an
expressly mandated limit, or an implied limit by virtue of incompatibility, the manner of execution by the President must be
given utmost deference. This approach is not different from that taken by the Court in situations with fairly similar contexts.

Thus, the analysis portrayed by the dissent does not give the President authority to bypass constitutional safeguards and
limits. In fact, it specifies what these limitations are, how these limitations are triggered, how these limitations function, and
what can be done within the sphere of constitutional duties and limitations of the President.

Justice Brion's dissent likewise misinterprets the analysis proffered when it claims that the foreign relations power of the
President should not be interpreted in isolation.  The analysis itself demonstrates how the foreign affairs function, while
168

mostly the President's, is shared in several instances, namely in Section 2 of Article II on the conduct of war; Sections 20
and 21 of Article VII on foreign loans, treaties, and international agreements; Sections 4(2) and 5(2)(a) of Article VIII on the
judicial review of executive acts; Sections 4 and 25 of Article XVIII on treaties and international agreements entered into
prior to the Constitution and on the presence of foreign military troops, bases, or facilities.

In fact, the analysis devotes a whole subheading to the relationship between the two major presidential functions and the
role of the Senate in it.

This approach of giving utmost deference to presidential initiatives in respect of foreign affairs is not novel to the Court.
The President's act of treating EDCA as an executive agreement is not the principal power being analyzed as the
Dissenting Opinion seems to suggest. Rather, the preliminary analysis is in reference to the expansive power of foreign
affairs. We have long treated this power as something the Courts must not unduly restrict. As we stated recently in Vinuya
v. Romulo:

To be sure, not all cases implicating foreign relations present political questions, and courts certainly possess the authority
to construe or invalidate treaties and executive agreements. However, the question whether the Philippine government
should espouse claims of its nationals against a foreign government is a foreign relations matter, the authority for which is
demonstrably committed by our Constitution not to the courts but to the political branches. In this case, the Executive
Department has already decided that it is to the best interest of the country to waive all claims of its nationals for
reparations against Japan in the Treaty of Peace of 1951. The wisdom of such decision is not for the courts to question.
Neither could petitioners herein assail the said determination by the Executive Department via the instant petition for
certiorari.

In the seminal case of US v. Curtiss-Wright Export Corp., the US Supreme Court held that "[t]he President is the sole
organ of the nation in its external relations, and its sole representative with foreign relations."

It is quite apparent that if, in the maintenance of our international relations, embarrassment - perhaps serious
embarrassment - is to be avoided and success for our aims achieved, congressional legislation which is to be
made effective through negotiation and inquiry within the international field must often accord to the President a
degree of discretion and freedom from statutory restriction which would not be admissible where
domestic affairs alone involved. Moreover, he, not Congress, has the better opportunity of knowing the
conditions which prevail in foreign countries, and especially is this true in time of war. He has his confidential
sources of information. He has his agents in the form of diplomatic, consular and other officials ....

This ruling has been incorporated in our jurisprudence through Bavan v. Executive Secretary  and Pimentel v.
Executive Secretary; its overreaching principle was, perhaps, best articulated in (now Chief) Justice Puno's dissent in
Secretary of Justice v. Lantion:

. . . The conduct of foreign relations is full of complexities and consequences, sometimes with life and death
significance to the nation especially in times of war. It can only be entrusted to that department of government
which can act on the basis of the best available information and can decide with decisiveness .... It is also the
President who possesses the most comprehensive and the most confidential information about foreign countries
for our diplomatic and consular officials regularly brief him on meaningful events all over the world. He has also
unlimited access to ultra-sensitive military intelligence data. In fine, the presidential role in foreign affairs is
dominant and the President is traditionally accorded a wider degree of discretion in the conduct of foreign
affairs. The regularity, nay, validity of his actions are adjudged under less stringent standards, lest their
judicial repudiation lead to breach of an international obligation, rupture of state relations, forfeiture of
confidence, national embarrassment and a plethora of other problems with equally undesirable
consequences.  (Emphases supplied)
169

Understandably, this Court must view the instant case with the same perspective and understanding, knowing full well the
constitutional and legal repercussions of any judicial overreach.
2. The plain meaning of the Constitution prohibits the entry of foreign military bases, troops or facilities, except
by way of a treaty concurred in by the Senate - a clear limitation on the President's dual role as defender of the
State and as sole authority in foreign relations.

Despite the President's roles as defender of the State and sole authority in foreign relations, the 1987 Constitution
expressly limits his ability in instances when it involves the entry of foreign military bases, troops or facilities. The initial
limitation is found in Section 21 of the provisions on the Executive Department: "No treaty or international agreement shall
be valid and effective unless concurred in by at least two-thirds of all the Members of the Senate." The specific limitation is
given by Section 25 of the Transitory Provisions, the full text of which reads as follows:

SECTION 25. After the expiration in 1991 of the Agreement between the Republic of the Philippines and the United States
of America concerning Military Bases, foreign military bases, troops, or facilities shall not be allowed in the Philippines
except under a treaty duly concurred in by the Senate and, when the Congress so requires, ratified by a majority of the
votes cast by the people in a national referendum held for that purpose, and recognized as a treaty by the other
contracting State.

It is quite plain that the Transitory Provisions of the 1987 Constitution intended to add to the basic requirements of a treaty
under Section 21 of Article VII. This means that both provisions must be read as additional limitations to the President's
overarching executive function in matters of defense and foreign relations.

3. The President, however, may enter into an executive agreement on foreign military bases, troops, or facilities, if
(a) it is not the instrument that allows the presence of foreign military bases, troops, or facilities; or (b) it merely
aims to implement an existing law or treaty.

Again we refer to Section 25, Article XVIII of the Constitution:

SECTION 25. After the expiration in 1991 of the Agreement between the Republic of the Philippines and the United States
of America concerning Military Bases, foreign military bases, troops, or facilities shall not be allowed in the
Philippines except under a treaty duly concurred in by the Senate and, when the Congress so requires, ratified by a
majority of the votes cast by the people in a national referendum held for that purpose, and recognized as a treaty by the
other contracting State. (Emphases supplied)

In view of this provision, petitioners argue  that EDCA must be in the form of a "treaty" duly concurred in by the Senate.
170

They stress that the Constitution is unambigous in mandating the transmission to the Senate of all international
agreements concluded after the expiration of the MBA in 1991 - agreements that concern the presence of foreign military
bases, troops, or facilities in the country. Accordingly, petitioners maintain that the Executive Department is not given the
choice to conclude agreements like EDCA in the form of an executive agreement.

This is also the view of the Senate, which, through a majority vote of 15 of its members - with 1 against and 2 abstaining -
says in SR 105  that EDCA must be submitted to the Senate in the form of a treaty for concurrence by at least two-thirds
171

of all its members.

The Senate cites two constitutional provisions (Article VI, Section 21 and Article XVIII, Section 25) to support its position.
Compared with the lone constitutional provision that the Office of the Solicitor General (OSG) cites, which is Article XVIII,
Section 4(2), which includes the constitutionality of "executive agreement(s)" among the cases subject to the Supreme
Court's power of judicial review, the Constitution clearly requires submission of EDCA to the Senate. Two specific
provisions versus one general provision means that the specific provisions prevail. The term "executive agreement" is "a
term wandering alone in the Constitution, bereft of provenance and an unidentified constitutional mystery."

The author of SR 105, Senator Miriam Defensor Santiago, upon interpellation even added that the MDT, which the
Executive claims to be partly implemented through EDCA, is already obsolete.

There are two insurmountable obstacles to this Court's agreement with SR 105, as well as with the comment on
interpellation made by Senator Santiago.

First, the concept of "executive agreement" is so well-entrenched in this Court's pronouncements on the powers of the
President. When the Court validated the concept of "executive agreement," it did so with full knowledge of the Senate's
role in concurring in treaties. It was aware of the problematique of distinguishing when an international agreement needed
Senate concurrence for validity, and when it did not; and the Court continued to validate the existence of "executive
agreements" even after the 1987 Constitution.  This follows a long line of similar decisions upholding the power of the
172

President to enter into an executive agreement. 173

Second, the MDT has not been rendered obsolescent, considering that as late as 2009,  this Court continued to
174

recognize its validity.

Third, to this Court, a plain textual reading of Article XIII, Section 25, inevitably leads to the conclusion that it applies only
to a proposed agreement between our government and a foreign government, whereby military bases, troops, or facilities
of such foreign government would be "allowed" or would "gain entry" Philippine territory.

Note that the provision "shall not be allowed" is a negative injunction. This wording signifies that the President is not
authorized by law to allow foreign military bases, troops, or facilities to enter the Philippines, except under a treaty
concurred in by the Senate. Hence, the constitutionally restricted authority pertains to the entry of the bases, troops, or
facilities, and not to the activities to be done after entry.

Under the principles of constitutional construction, of paramount consideration is the plain meaning of the language
expressed in the Constitution, or the verba legis rule.  It is presumed that the provisions have been carefully crafted in
175

order to express the objective it seeks to attain.  It is incumbent upon the Court to refrain from going beyond the plain
176

meaning of the words used in the Constitution. It is presumed that the framers and the people meant what they said when
they said it, and that this understanding was reflected in the Constitution and understood by the people in the way it was
meant to be understood when the fundamental law was ordained and promulgated.  As this Court has often said:
177

We look to the language of the document itself in our search for its meaning. We do not of course stop there, but that is
where we begin. It is to be assumed that the words in which constitutional provisions are couched express the objective
sought to be attained. They are to be given their ordinary meaning except where technical terms are employed in
which case the significance thus attached to them prevails. As the Constitution is not primarily a lawyer's document, it
being essential for the rule of law to obtain that it should ever be present in the people's consciousness, its language as
much as possible should be understood in the sense they have in common use. What it says according to the text of
the provision to be construed compels acceptance and negates the power of the courts to alter it, based on the postulate
that the framers and the people mean what they say. Thus, these are the cases where the need for construction is
reduced to a minimum.  (Emphases supplied)
178

It is only in those instances in which the constitutional provision is unclear, ambiguous, or silent that further construction
must be done to elicit its meaning.  In Ang Bagong Bayani-OFW v. Commission on Elections,  we reiterated this guiding
179 180

principle:

it [is] safer to construe the Constitution from what appears upon its face. The proper interpretation therefore
depends more on how it was understood by the people adopting it than in the framers' understanding thereof.
(Emphases supplied)

The effect of this statement is surprisingly profound, for, if taken literally, the phrase "shall not be allowed in the
Philippines" plainly refers to the entry of bases, troops, or facilities in the country. The Oxford English Dictionary defines
the word "allow" as a transitive verb that means "to permit, enable"; "to give consent to the occurrence of or relax restraint
on (an action, event, or activity)"; "to consent to the presence or attendance of (a person)"; and, when with an adverbial of
place, "to permit (a person or animal) to go, come, or be in, out, near, etc."  Black's Law Dictionary defines the term as
181

one that means "[t]o grant, approve, or permit." 182

The verb "allow" is followed by the word "in," which is a preposition used to indicate "place or position in space or anything
having material extension: Within the limits or bounds of, within (any place or thing)."  That something is the Philippines,
183

which is the noun that follows.

It is evident that the constitutional restriction refers solely to the initial entry of the foreign military bases, troops, or
facilities. Once entry is authorized, the subsequent acts are thereafter subject only to the limitations provided by the rest of
the Constitution and Philippine law, and not to the Section 25 requirement of validity through a treaty.

The VFA has already allowed the entry of troops in the Philippines. This Court stated in Lim v. Executive Secretary:

After studied reflection, it appeared farfetched that the ambiguity surrounding the meaning of the word "activities" arose
from accident. In our view, it was deliberately made that way to give both parties a certain leeway in negotiation. In this
manner, visiting US forces may sojourn in Philippine territory for purposes other than military. As conceived, the
joint exercises may include training on new techniques of patrol and surveillance to protect the nation's marine resources,
sea search-and-rescue operations to assist vessels in distress, disaster relief operations, civic action projects such as the
building of school houses, medical and humanitarian missions, and the like.

Under these auspices, the VFA gives legitimacy to the current Balikatan exercises. It is only logical to assume that
"Balikatan 02-1," a "mutual anti- terrorism advising, assisting and training exercise," falls under the umbrella of sanctioned
or allowable activities in the context of the agreement. Both the history and intent of the Mutual Defense Treaty and the
VFA support the conclusion that combat-related activities -as opposed to combat itself-such as the one subject of the
instant petition, are indeed authorized.  (Emphasis supplied)
184

Moreover, the Court indicated that the Constitution continues to govern the conduct of foreign military troops in the
Philippines,  readily implying the legality of their initial entry into the country.
185

The OSG emphasizes that EDCA can be in the form of an executive agreement, since it merely involves "adjustments in
detail" in the implementation of the MDT and the VFA.  It points out that there are existing treaties between the
186

Philippines and the U.S. that have already been concurred in by the Philippine Senate and have thereby met the
requirements of the Constitution under Section 25. Because of the status of these prior agreements, respondent
emphasizes that EDCA need not be transmitted to the Senate.

The aforecited Dissenting Opinion of Justice Brion disagrees with the ponencia's application of verba legis construction to
the words of Article XVIII, Section 25.  It claims that the provision is "neither plain, nor that simple."  To buttress its
187 188

disagreement, the dissent states that the provision refers to a historical incident, which is the expiration of the 1947
MBA.  Accordingly, this position requires questioning the circumstances that led to the historical event, and the meaning
189

of the terms under Article XVIII, Section 25.

This objection is quite strange. The construction technique of verba legis is not inapplicable just because a provision has a
specific historical context. In fact, every provision of the Constitution has a specific historical context. The purpose of
constitutional and statutory construction is to set tiers of interpretation to guide the Court as to how a particular provision
functions. Verba legis is of paramount consideration, but it is not the only consideration. As this Court has often said:

We look to the language of the document itself in our search for its meaning. We do not of course stop there, but that is
where we begin. It is to be assumed that the words in which constitutional provisions are couched express the objective
sought to be attained. They are to be given their ordinary meaning except where technical terms are employed in
which case the significance thus attached to them prevails. As the Constitution is not primarily a lawyer's document, it
being essential for the rule of law to obtain that it should ever be present in the people's consciousness, its language as
much as possible should be understood in the sense they have in common use. What it says according to the text of
the provision to be construed compels acceptance and negates the power of the courts to alter it, based on the postulate
that the framers and the people mean what they say. Thus, these are the cases where the need for construction is
reduced to a minimum.  (Emphases supplied)
190

As applied, verba legis aids in construing the ordinary meaning of terms. In this case, the phrase being construed is "shall
not be allowed in the Philippines" and not the preceding one referring to "the expiration in 1991 of the Agreement between
the Republic of the Philippines and the United States of America concerning Military Bases, foreign military bases, troops,
or facilities." It is explicit in the wording of the provision itself that any interpretation goes beyond the text itself and into the
discussion of the framers, the context of the Constitutional Commission's time of drafting, and the history of the 1947
MBA. Without reference to these factors, a reader would not understand those terms. However, for the phrase "shall not
be allowed in the Philippines," there is no need for such reference. The law is clear. No less than the Senate understood
this when it ratified the VFA.

4. The President may generally enter into executive agreements subject to limitations defined by the Constitution
and may be in furtherance of a treaty already concurred in by the Senate.

We discuss in this section why the President can enter into executive agreements.

It would be helpful to put into context the contested language found in Article XVIII, Section 25. Its more exacting
requirement was introduced because of the previous experience of the country when its representatives felt compelled to
consent to the old MBA.  They felt constrained to agree to the MBA in fulfilment of one of the major conditions for the
191

country to gain independence from the U.S.  As a result of that experience, a second layer of consent for agreements that
192

allow military bases, troops and facilities in the country is now articulated in Article XVIII of our present Constitution.

This second layer of consent, however, cannot be interpreted in such a way that we completely ignore the intent of our
constitutional framers when they provided for that additional layer, nor the vigorous statements of this Court that affirm the
continued existence of that class of international agreements called "executive agreements."

The power of the President to enter into binding executive agreements without Senate concurrence is already well-
established in this jurisdiction.  That power has been alluded to in our present and past Constitutions,  in various
193 194

statutes,  in Supreme Court decisions,  and during the deliberations of the Constitutional Commission.  They cover a
195 196 197

wide array of subjects with varying scopes and purposes,  including those that involve the presence of foreign military
198

forces in the country. 199

As the sole organ of our foreign relations  and the constitutionally assigned chief architect of our foreign policy,  the
200 201

President is vested with the exclusive power to conduct and manage the country's interface with other states and
governments. Being the principal representative of the Philippines, the Chief Executive speaks and listens for the nation;
initiates, maintains, and develops diplomatic relations with other states and governments; negotiates and enters into
international agreements; promotes trade, investments, tourism and other economic relations; and settles international
disputes with other states. 202

As previously discussed, this constitutional mandate emanates from the inherent power of the President to enter into
agreements with other states, including the prerogative to conclude binding executive agreements that do not require
further Senate concurrence. The existence of this presidential power  is so well-entrenched that Section 5(2)(a), Article
203

VIII of the Constitution, even provides for a check on its exercise. As expressed below, executive agreements are among
those official governmental acts that can be the subject of this Court's power of judicial review:

(2) Review, revise, reverse, modify, or affirm on appeal or certiorari, as the law or the Rules of Court may
provide, final judgments and orders of lower courts in:

(a) All cases in which the constitutionality or validity of any treaty, international or executive


agreement, law, presidential decree, proclamation, order, instruction, ordinance, or regulation is in
question. (Emphases supplied)

In Commissioner of Customs v. Eastern Sea Trading, executive agreements are defined as "international agreements
embodying adjustments of detail carrying out well-established national policies and traditions and those involving
arrangements of a more or less temporary nature."  In Bayan Muna v. Romulo, this Court further clarified that executive
204
agreements can cover a wide array of subjects that have various scopes and purposes.  They are no longer limited to the
205

traditional subjects that are usually covered by executive agreements as identified in Eastern Sea Trading. The Court
thoroughly discussed this matter in the following manner:

The categorization of subject matters that may be covered by international agreements mentioned in Eastern Sea


Trading is not cast in stone. x x x.

As may be noted, almost half a century has elapsed since the Court rendered its decision in Eastern Sea
Trading. Since then, the conduct of foreign affairs has become more complex and the domain of international law
wider, as to include such subjects as human rights, the environment, and the sea. In fact, in the US alone, the executive
agreements executed by its President from 1980 to 2000 covered subjects such as defense, trade, scientific
cooperation, aviation, atomic energy, environmental cooperation, peace corps, arms limitation, and nuclear
safety, among others. Surely, the enumeration in Eastern Sea Trading cannot circumscribe the option of each
state on the matter of which the international agreement format would be convenient to serve its best interest. As
Francis Sayre said in his work referred to earlier:

. . . It would be useless to undertake to discuss here the large variety of executive agreements as such concluded
from time to time. Hundreds of executive agreements, other than those entered into under the trade-agreement act, have
been negotiated with foreign governments. . . . They cover such subjects as the inspection of vessels, navigation dues,
income tax on shipping profits, the admission of civil air craft, custom matters and commercial relations generally,
international claims, postal matters, the registration of trademarks and copyrights, etc .... (Emphases Supplied)

One of the distinguishing features of executive agreements is that their validity and effectivity are not affected by a lack of
Senate concurrence.  This distinctive feature was recognized as early as in Eastern Sea Trading (1961), viz:
206

Treaties are formal documents which require ratification with the approval of two-thirds of the Senate. Executive


agreements become binding through executive action without the need of a vote by the Senate or by Congress.

xxxx

[T]he right of the Executive to enter into binding agreements without the necessity of subsequent Congressional
approval has been confirmed by long usage. From the earliest days of our history we have entered into executive
agreements covering such subjects as commercial and consular relations, most-favored-nation rights, patent rights,
trademark and copyright protection, postal and navigation arrangements and the settlement of claims. The validity of
these has never been seriously questioned by our courts. (Emphases Supplied)

That notion was carried over to the present Constitution. In fact, the framers specifically deliberated on whether the
general term "international agreement" included executive agreements, and whether it was necessary to include an
express proviso that would exclude executive agreements from the requirement of Senate concurrence. After noted
constitutionalist Fr. Joaquin Bernas quoted the Court's ruling in Eastern Sea Trading, the Constitutional Commission
members ultimately decided that the term "international agreements" as contemplated in Section 21, Article VII, does not
include executive agreements, and that a proviso is no longer needed. Their discussion is reproduced below: 207

MS. AQUINO: Madam President, first I would like a clarification from the Committee. We have retained the words
"international agreement" which I think is the correct judgment on the matter because an international agreement is
different from a treaty. A treaty is a contract between parties which is in the nature of international agreement and also a
municipal law in the sense that the people are bound. So there is a conceptual difference. However, I would like to be
clarified if the international agreements include executive agreements.

MR. CONCEPCION: That depends upon the parties. All parties to these international negotiations stipulate the conditions
which are necessary for the agreement or whatever it may be to become valid or effective as regards the parties.

MS. AQUINO: Would that depend on the parties or would that depend on the nature of the executive agreement?
According to common usage, there are two types of executive agreement: one is purely proceeding from an
executive act which affects external relations independent of the legislative and the other is an executive act in
pursuance of legislative authorization. The first kind might take the form of just conventions or exchanges of notes
or protocol while the other, which would be pursuant to the legislative authorization, may be in the nature of
commercial agreements.

MR. CONCEPCION: Executive agreements are generally made to implement a treaty already enforced or to


determine the details for the implementation of the treaty. We are speaking of executive agreements, not international
agreements.

MS. AQUINO: I am in full agreement with that, except that it does not cover the first kind of executive agreement which is
just protocol or an exchange of notes and this would be in the nature of reinforcement of claims of a citizen against a
country, for example.

MR. CONCEPCION: The Commissioner is free to require ratification for validity insofar as the Philippines is concerned.

MS. AQUINO: It is my humble submission that we should provide, unless the Committee explains to us otherwise,
an explicit proviso which would except executive agreements from the requirement of concurrence of two-thirds of
the Members of the Senate. Unless I am enlightened by the Committee I propose that tentatively, the sentence should
read. "No treaty or international agreement EXCEPT EXECUTIVE AGREEMENTS shall be valid and effective."

FR. BERNAS: I wonder if a quotation from the Supreme Court decision [in Eastern Sea Trading] might help clarify
this:

The right of the executive to enter into binding agreements without the necessity of subsequent Congressional
approval has been confirmed by long usage. From the earliest days of our history, we have entered into executive
agreements covering such subjects as commercial and consular relations, most favored nation rights, patent rights,
trademark and copyright protection, postal and navigation arrangements and the settlement of claims. The validity of this
has never been seriously questioned by our Courts.

Agreements with respect to the registration of trademarks have been concluded by the executive of various countries
under the Act of Congress of March 3, 1881 (21 Stat. 502) . . . International agreements involving political issues or
changes of national policy and those involving international agreements of a permanent character usually take the
form of treaties. But international agreements embodying adjustments of detail, carrying out well established national
policies and traditions and those involving arrangements of a more or less temporary nature usually take the form
of executive agreements.

MR. ROMULO: Is the Commissioner, therefore, excluding the executive agreements?

FR. BERNAS: What we are referring to, therefore, when we say international agreements which need concurrence by
at least two-thirds are those which are permanent in nature.

MS. AQUINO: And it may include commercial agreements which are executive agreements essentially but which are
proceeding from the authorization of Congress. If that is our understanding, then I am willing to withdraw that amendment.

FR. BERNAS: If it is with prior authorization of Congress, then it does not need subsequent concurrence by
Congress.

MS. AQUINO: In that case, I am withdrawing my amendment.

MR. TINGSON: Madam President.

THE PRESIDENT: Is Commissioner Aquino satisfied?

MS. AQUINO: Yes. There is already an agreement among us on the definition of "executive agreements" and that
would make unnecessary any explicit proviso on the matter.

xxx

MR. GUINGONA: I am not clear as to the meaning of "executive agreements" because I heard that these executive
agreements must rely on treaties. In other words, there must first be treaties.

MR. CONCEPCION: No, I was speaking about the common use, as executive agreements being the implementation of
treaties, details of which do not affect the sovereignty of the State.

MR. GUINGONA: But what about the matter of permanence, Madam President? Would 99 years be considered
permanent? What would be the measure of permanency? I do not conceive of a treaty that is going to be forever, so there
must be some kind of a time limit.

MR. CONCEPCION: I suppose the Commissioner's question is whether this type of agreement should be included in a
provision of the Constitution requiring the concurrence of Congress.

MR. GUINGONA: It depends on the concept of the executive agreement of which I am not clear. If the executive
agreement partakes of the nature of a treaty, then it should also be included.

MR. CONCEPCION: Whether it partakes or not of the nature of a treaty, it is within the power of the Constitutional
Commission to require that.

MR. GUINGONA: Yes. That is why I am trying to clarify whether the words "international agreements" would
include executive agreements.

MR. CONCEPCION: No, not necessarily; generally no.

xxx

MR. ROMULO: I wish to be recognized first. I have only one question. Do we take it, therefore, that as far as the
Committee is concerned, the term "international agreements" does not include the term "executive
agreements" as read by the Commissioner in that text?
FR. BERNAS: Yes. (Emphases Supplied)

The inapplicability to executive agreements of the requirements under Section 21 was again recognized in Bayan v.
Zamora and in Bayan Muna v. Romulo. These cases, both decided under the aegis of the present Constitution,
quoted Eastern Sea Trading in reiterating that executive agreements are valid and binding even without the concurrence
of the Senate.

Executive agreements may dispense with the requirement of Senate concurrence because of the legal mandate with
which they are concluded. As culled from the afore-quoted deliberations of the Constitutional Commission, past Supreme
Court Decisions, and works of noted scholars,  executive agreements merely involve arrangements on the
208

implementation of existing policies, rules, laws, or agreements. They are concluded (1) to adjust the details of a
treaty;  (2) pursuant to or upon confirmation by an act of the Legislature;  or (3) in the exercise of the President's
209 210

independent powers under the Constitution.  The raison d'etre of executive agreements hinges on prior constitutional or
211

legislative authorizations.

The special nature of an executive agreement is not just a domestic variation in international agreements. International
practice has accepted the use of various forms and designations of international agreements, ranging from the traditional
notion of a treaty - which connotes a formal, solemn instrument - to engagements concluded in modem, simplified forms
that no longer necessitate ratification.  An international agreement may take different forms: treaty, act, protocol,
212

agreement, concordat, compromis d'arbitrage, convention, covenant, declaration, exchange of notes, statute, pact,


charter, agreed minute, memorandum of agreement, modus vivendi, or some other form.  Consequently, under
213

international law, the distinction between a treaty and an international agreement or even an executive agreement is
irrelevant for purposes of determining international rights and obligations.

However, this principle does not mean that the domestic law distinguishing treaties, international
agreements, and executive agreements is relegated to a mere variation in form, or that the constitutional requirement of
Senate concurrence is demoted to an optional constitutional directive. There remain two very important features that
distinguish treaties from executive agreements and translate them into terms of art in the domestic setting.

First, executive agreements must remain traceable to an express or implied authorization under the Constitution, statutes,
or treaties. The absence of these precedents puts the validity and effectivity of executive agreements under serious
question for the main function of the Executive is to enforce the Constitution and the laws enacted by the Legislature, not
to defeat or interfere in the performance of these rules.  In turn, executive agreements cannot create new international
214

obligations that are not expressly allowed or reasonably implied in the law they purport to implement.

Second, treaties are, by their very nature, considered superior to executive agreements. Treaties are products of the acts
of the Executive and the Senate  unlike executive agreements, which are solely executive actions.  Because of
215 216

legislative participation through the Senate, a treaty is regarded as being on the same level as a statute.  If there is an
217

irreconcilable conflict, a later law or treaty takes precedence over one that is prior.  An executive agreement is treated
218

differently. Executive agreements that are inconsistent with either a law or a treaty are considered ineffective.  Both types
219

of international agreement are nevertheless subject to the supremacy of the Constitution. 220

This rule does not imply, though, that the President is given carte blanche to exercise this discretion. Although the Chief
Executive wields the exclusive authority to conduct our foreign relations, this power must still be exercised within the
context and the parameters set by the Constitution, as well as by existing domestic and international laws. There are
constitutional provisions that restrict or limit the President's prerogative in concluding international agreements, such as
those that involve the following:

a. The policy of freedom from nuclear weapons within Philippine territory 221

b. The fixing of tariff rates, import and export quotas, tonnage and wharfage dues, and other duties or imposts,
which must be pursuant to the authority granted by Congress 222

c. The grant of any tax exemption, which must be pursuant to a law concurred in by a majority of all the Members
of Congress 223

d. The contracting or guaranteeing, on behalf of the Philippines, of foreign loans that must be previously concurred
in by the Monetary Board 224

e. The authorization of the presence of foreign military bases, troops, or facilities in the country must be in the form
of a treaty duly concurred in by the Senate. 225

f. For agreements that do not fall under paragraph 5, the concurrence of the Senate is required, should the form of
the government chosen be a treaty.

5. The President had the choice to enter into EDCA by way of an executive agreement or a treaty.

No court can tell the President to desist from choosing an executive agreement over a treaty to embody an international
agreement, unless the case falls squarely within Article VIII, Section 25.
As can be gleaned from the debates among the members of the Constitutional Commission, they were aware that legally
binding international agreements were being entered into by countries in forms other than a treaty. At the same time, it is
clear that they were also keen to preserve the concept of "executive agreements" and the right of the President to enter
into such agreements.

What we can glean from the discussions of the Constitutional Commissioners is that they understood the following
realities:

1. Treaties, international agreements, and executive agreements are all constitutional manifestations of the
conduct of foreign affairs with their distinct legal characteristics.

a. Treaties are formal contracts between the Philippines and other States-parties, which are in the nature
of international agreements, and also of municipal laws in the sense of their binding nature. 226

b. International agreements are similar instruments, the provisions of which may require the ratification of a
designated number of parties thereto. These agreements involving political issues or changes in national
policy, as well as those involving international agreements of a permanent character, usually take the form
of treaties. They may also include commercial agreements, which are executive agreements essentially,
but which proceed from previous authorization by Congress, thus dispensing with the requirement of
concurrence by the Senate. 227

c. Executive agreements are generally intended to implement a treaty already enforced or to determine the
details of the implementation thereof that do not affect the sovereignty of the State.228

2. Treaties and international agreements that cannot be mere executive agreements must, by constitutional
decree, be concurred in by at least two-thirds of the Senate.

3. However, an agreement - the subject of which is the entry of foreign military troops, bases, or facilities - is
particularly restricted. The requirements are that it be in the form of a treaty concurred in by the Senate; that when
Congress so requires, it be ratified by a majority of the votes cast by the people in a national referendum held for
that purpose; and that it be recognized as a treaty by the other contracting State.

4. Thus, executive agreements can continue to exist as a species of international agreements.

That is why our Court has ruled the way it has in several cases.

In Bayan Muna v. Romulo, we ruled that the President acted within the scope of her constitutional authority and discretion
when she chose to enter into the RP-U.S. Non-Surrender Agreement in the form of an executive agreement, instead of a
treaty, and in ratifying the agreement without Senate concurrence. The Court en banc discussed this intrinsic presidential
prerogative as follows:

Petitioner parlays the notion that the Agreement is of dubious validity, partaking as it does of the nature of a treaty; hence,
it must be duly concurred in by the Senate. x x x x. Pressing its point, petitioner submits that the subject of the Agreement
does not fall under any of the subject-categories that xx x may be covered by an executive agreement, such as
commercial/consular relations, most-favored nation rights, patent rights, trademark and copyright protection, postal and
navigation arrangements and settlement of claims.

The categorization of subject matters that may be covered by international agreements mentioned in Eastern Sea
Trading is not cast in stone. There are no hard and fast rules on the propriety of entering, on a given subject, into a
treaty or an executive agreement as an instrument of international relations. The primary consideration in the
choice of the form of agreement is the parties' intent and desire to craft an international agreement in the form they
so wish to further their respective interests. Verily, the matter of form takes a back seat when it comes to
effectiveness and binding effect of the enforcement of a treaty or an executive agreement, as the parties in either
international agreement each labor under the pacta sunt servanda principle.

xxxx

But over and above the foregoing considerations is the fact that - save for the situation and matters contemplated in Sec.
25, Art. XVIII of the Constitution - when a treaty is required, the Constitution does not classify any subject, like that
involving political issues, to be in the form of, and ratified as, a treaty. What the Constitution merely prescribes is that
treaties need the concurrence of the Senate by a vote defined therein to complete the ratification process.

xxxx

x x x. As the President wields vast powers and influence, her conduct in the external affairs of the nation is,
as Bayan would put it, "executive altogether." The right of the President to enter into or ratify binding executive
agreements has been confirmed by long practice.

In thus agreeing to conclude the Agreement thru E/N BF0-028-03, then President Gloria Macapagal-Arroyo,
represented by the Secretary of Foreign Affairs, acted within the scope of the authority and discretion vested in her
by the Constitution. At the end of the day, the President - by ratifying, thru her deputies, the non-surrender
agreement - did nothing more than discharge a constitutional duty and exercise a prerogative that pertains to her
office. (Emphases supplied)

Indeed, in the field of external affairs, the President must be given a larger measure of authority and wider discretion,
subject only to the least amount of checks and restrictions under the Constitution.  The rationale behind this power and
229

discretion was recognized by the Court in Vinuya v. Executive Secretary, cited earlier. 230

Section 9 of Executive Order No. 459, or the Guidelines in the Negotiation of International Agreements and its Ratification,
thus, correctly reflected the inherent powers of the President when it stated that the DFA "shall determine whether an
agreement is an executive agreement or a treaty."

Accordingly, in the exercise of its power of judicial review, the Court does not look into whether an international agreement
should be in the form of a treaty or an executive agreement, save in cases in which the Constitution or a statute requires
otherwise. Rather, in view of the vast constitutional powers and prerogatives granted to the President in the field of foreign
affairs, the task of the Court is to determine whether the international agreement is consistent with the applicable
limitations.

6. Executive agreements may cover the matter of foreign military forces if it merely involves detail adjustments.

The practice of resorting to executive agreements in adjusting the details of a law or a treaty that already deals with the
presence of foreign military forces is not at all unusual in this jurisdiction. In fact, the Court has already implicitly
acknowledged this practice in Lim v. Executive Secretary.  In that case, the Court was asked to scrutinize the
231

constitutionality of the Terms of Reference of the Balikatan 02-1 joint military exercises, which sought to implement the
VFA. Concluded in the form of an executive agreement, the Terms of Reference detailed the coverage of the term
"activities" mentioned in the treaty and settled the matters pertaining to the construction of temporary structures for the
U.S. troops during the activities; the duration and location of the exercises; the number of participants; and the extent of
and limitations on the activities of the U.S. forces. The Court upheld the Terms of Reference as being consistent with the
VFA. It no longer took issue with the fact that the Balikatan Terms of Reference was not in the form of a treaty concurred
in by the Senate, even if it dealt with the regulation of the activities of foreign military forces on Philippine territory.

In Nicolas v. Romulo,  the Court again impliedly affirmed the use of an executive agreement in an attempt to adjust the
232

details of a provision of the VFA. The Philippines and the U.S. entered into the Romulo-Kenney Agreement, which
undertook to clarify the detention of a U.S. Armed Forces member, whose case was pending appeal after his conviction by
a trial court for the crime of rape. In testing the validity of the latter agreement, the Court precisely alluded to one of the
inherent limitations of an executive agreement: it cannot go beyond the terms of the treaty it purports to implement. It was
eventually ruled that the Romulo-Kenney Agreement was "not in accord" with the VFA, since the former was squarely
inconsistent with a provision in the treaty requiring that the detention be "by Philippine authorities." Consequently, the
Court ordered the Secretary of Foreign Affairs to comply with the VFA and "forthwith negotiate with the United States
representatives for the appropriate agreement on detention facilities under Philippine authorities as provided in Art. V, Sec.
10 of the VFA. "233

Culling from the foregoing discussions, we reiterate the following pronouncements to guide us in resolving the present
controversy:

1. Section 25, Article XVIII of the Constitution, contains stringent requirements that must be fulfilled by the
international agreement allowing the presence of foreign military bases, troops, or facilities in the Philippines: (a)
the agreement must be in the form of a treaty, and (b) it must be duly concurred in by the Senate.

2. If the agreement is not covered by the above situation, then the President may choose the form of the
agreement (i.e., either an executive agreement or a treaty), provided that the agreement dealing with foreign
military bases, troops, or facilities is not the principal agreement that first allows their entry or presence in the
Philippines.

3. The executive agreement must not go beyond the parameters, limitations, and standards set by the law and/or
treaty that the former purports to implement; and must not unduly expand the international obligation expressly
mentioned or necessarily implied in the law or treaty.

4. The executive agreement must be consistent with the Constitution, as well as with existing laws and treaties.

In light of the President's choice to enter into EDCA in the form of an executive agreement, respondents carry the burden
of proving that it is a mere implementation of existing laws and treaties concurred in by the Senate. EDCA must thus be
carefully dissected to ascertain if it remains within the legal parameters of a valid executive agreement.

7. EDCA is consistent with the content, purpose, and framework of the MDT and the VFA

The starting point of our analysis is the rule that "an executive agreement xx x may not be used to amend a
treaty."  In Lim v. Executive Secretary and in Nicolas v. Romulo, the Court approached the question of the validity of
234

executive agreements by comparing them with the general framework and the specific provisions of the treaties they seek
to implement.
In Lim, the Terms of Reference of the joint military exercises was scrutinized by studying "the framework of the treaty
antecedents to which the Philippines bound itself,"  i.e., the MDT and the VFA. The Court proceeded to examine the
235

extent of the term "activities" as contemplated in Articles 1  and II  of the VFA. It later on found that the term "activities"
236 237

was deliberately left undefined and ambiguous in order to permit "a wide scope of undertakings subject only to the
approval of the Philippine government"  and thereby allow the parties "a certain leeway in negotiation."  The Court
238 239

eventually ruled that the Terms of Reference fell within the sanctioned or allowable activities, especially in the context of
the VFA and the MDT.

The Court applied the same approach to Nicolas v. Romulo. It studied the provisions of the VFA on custody and detention
to ascertain the validity of the Romulo-Kenney Agreement.  It eventually found that the two international agreements were
240

not in accord, since the Romulo-Kenney Agreement had stipulated that U.S. military personnel shall be detained at the
U.S. Embassy Compound and guarded by U.S. military personnel, instead of by Philippine authorities. According to the
Court, the parties "recognized the difference between custody during the trial and detention after conviction."  Pursuant to
241

Article V(6) of the VFA, the custody of a U.S. military personnel resides with U.S. military authorities during trial. Once
there is a finding of guilt, Article V(l0) requires that the confinement or detention be "by Philippine authorities."

Justice Marvic M.V.F. Leonen's Dissenting Opinion posits that EDCA "substantially modifies or amends the VFA"  and 242

follows with an enumeration of the differences between EDCA and the VFA. While these arguments will be rebutted more
fully further on, an initial answer can already be given to each of the concerns raised by his dissent.

The first difference emphasized is that EDCA does not only regulate visits as the VFA does, but allows temporary
stationing on a rotational basis of U.S. military personnel and their contractors in physical locations with permanent
facilities and pre-positioned military materiel.

This argument does not take into account that these permanent facilities, while built by U.S. forces, are to be owned by the
Philippines once constructed.  Even the VFA allowed construction for the benefit of U.S. forces during their temporary
243

visits.

The second difference stated by the dissent is that EDCA allows the prepositioning of military materiel, which can include
various types of warships, fighter planes, bombers, and vessels, as well as land and amphibious vehicles and their
corresponding ammunition. 244

However, the VFA clearly allows the same kind of equipment, vehicles, vessels, and aircraft to be brought into the country.
Articles VII and VIII of the VFA contemplates that U.S. equipment, materials, supplies, and other property are imported
into or acquired in the Philippines by or on behalf of the U.S. Armed Forces; as are vehicles, vessels, and aircraft operated
by or for U.S. forces in connection with activities under the VFA. These provisions likewise provide for the waiver of the
specific duties, taxes, charges, and fees that correspond to these equipment.

The third difference adverted to by the Justice Leonen's dissent is that the VFA contemplates the entry of troops for
training exercises, whereas EDCA allows the use of territory for launching military and paramilitary operations conducted
in other states.  The dissent of Justice Teresita J. Leonardo-De Castro also notes that VFA was intended for non-combat
245

activides only, whereas the entry and activities of U.S. forces into Agreed Locations were borne of military necessity or
had a martial character, and were therefore not contemplated by the VFA. 246

This Court's jurisprudence however established in no uncertain terms that combat-related activities, as opposed to actual
combat, were allowed under the MDT and VFA, viz:

Both the history and intent of the Mutual Defense Treaty and the VFA support the conclusion that combat-related activities
as opposed to combat itself such as the one subject of the instant petition, are indeed authorized. 247

Hence, even if EDCA was borne of military necessity, it cannot be said to have strayed from the intent of the VFA since
EDCA's combat-related components are allowed under the treaty.

Moreover, both the VFA and EDCA are silent on what these activities actually are. Both the VFA and EDCA deal with the
presence of U.S. forces within the Philippines, but make no mention of being platforms for activity beyond Philippine
territory. While it may be that, as applied, military operations under either the VFA or EDCA would be carried out in the
future the scope of judicial review does not cover potential breaches of discretion but only actual occurrences or blatantly
illegal provisions. Hence, we cannot invalidate EDCA on the basis of the potentially abusive use of its provisions.

The fourth difference is that EDCA supposedly introduces a new concept not contemplated in the VFA or the MDT: Agreed
Locations, Contractors, Pre-positioning, and Operational Control. 248

As previously mentioned, these points shall be addressed fully and individually in the latter analysis of EDCA's provisions.
However, it must already be clarified that the terms and details used by an implementing agreement need not be found in
the mother treaty. They must be sourced from the authority derived from the treaty, but are not necessarily expressed
word-for-word in the mother treaty. This concern shall be further elucidated in this Decision.

The fifth difference highlighted by the Dissenting Opinion is that the VFA does not have provisions that may be construed
as a restriction on or modification of obligations found in existing statues, including the jurisdiction of courts, local
autonomy, and taxation. Implied in this argument is that EDCA contains such restrictions or modifications. 249
This last argument cannot be accepted in view of the clear provisions of EDCA. Both the VFA and EDCA ensure
Philippine jurisdiction in all instances contemplated by both agreements, with the exception of those outlined by the VFA in
Articles III-VI. In the VFA, taxes are clearly waived whereas in EDCA, taxes are assumed by the government as will be
discussed later on. This fact does not, therefore, produce a diminution of jurisdiction on the part of the Philippines, but
rather a recognition of sovereignty and the rights that attend it, some of which may be waived as in the cases under
Articles III-VI of the VFA.

Taking off from these concerns, the provisions of EDCA must be compared with those of the MDT and the VFA, which are
the two treaties from which EDCA allegedly draws its validity.

"Authorized presence" under the VFA versus "authorized activities" under EDCA: (1) U.S. personnel and (2) U.S.
contractors

The OSG argues  that EDCA merely details existing policies under the MDT and the VFA. It explains that EDCA
250

articulates the principle of defensive preparation embodied in Article II of the MDT; and seeks to enhance the defensive,
strategic, and technological capabilities of both parties pursuant to the objective of the treaty to strengthen those
capabilities to prevent or resist a possible armed attack. Respondent also points out that EDCA simply implements Article I
of the VFA, which already allows the entry of U.S. troops and personnel into the country. Respondent stresses this Court's
recognition in Lim v. Executive Secretary that U.S. troops and personnel are authorized to conduct activities that promote
the goal of maintaining and developing their defense capability.

Petitioners contest  the assertion that the provisions of EDCA merely implement the MDT. According to them, the treaty
251

does not specifically authorize the entry of U.S. troops in the country in order to maintain and develop the individual and
collective capacities of both the Philippines and the U.S. to resist an armed attack. They emphasize that the treaty was
concluded at a time when there was as yet no specific constitutional prohibition on the presence of foreign military forces
in the country.

Petitioners also challenge the argument that EDCA simply implements the VFA. They assert that the agreement covers
only short-term or temporary visits of U.S. troops "from time to time" for the specific purpose of combined military
exercises with their Filipino counterparts. They stress that, in contrast, U.S. troops are allowed under EDCA to perform
activities beyond combined military exercises, such as those enumerated in Articles 111(1) and IV(4) thereof.
Furthermore, there is some degree of permanence in the presence of U.S. troops in the country, since the effectivity of
EDCA is continuous until terminated. They proceed to argue that while troops have a "rotational" presence, this scheme in
fact fosters their permanent presence.

a. Admission of U.S. military and civilian personnel into Philippine territory is already allowed under the VFA

We shall first deal with the recognition under EDCA of the presence in the country of three distinct classes of individuals
who will be conducting different types of activities within the Agreed Locations: (1) U.S. military personnel; (2) U.S. civilian
personnel; and (3) U.S. contractors. The agreement refers to them as follows:

"United States personnel" means United States military and civilian personnel temporarily in the territory of the
Philippines in connection with activities approved by the Philippines, as those terms are defined in the VFA. 252

"United States forces" means the entity comprising United States personnel and all property, equipment, and
materiel of the United States Armed Forces present in the territory of the Philippines. 253

"United States contractors" means companies and firms, and their employees, under contract or subcontract to or
on behalf of the United States Department of Defense. United States contractors are not included as part of the definition
of United States personnel in this Agreement, including within the context of the VFA. 254

United States forces may contract for any materiel, supplies, equipment, and services (including construction) to be
furnished or undertaken in the territory of the Philippines without restriction as to choice of contractor, supplier, or
person who provides such materiel, supplies, equipment, or services. Such contracts shall be solicited, awarded, and
administered in accordance with the laws and regulations of the United States.  (Emphases Supplied)
255

A thorough evaluation of how EDCA is phrased clarities that the agreement does not deal with the entry into the
country of U.S. personnel and contractors per se. While Articles I(l)(b)  and II(4)  speak of "the right to access and
256 257

use" the Agreed Locations, their wordings indicate the presumption that these groups have already been allowed entry
into Philippine territory, for which, unlike the VFA, EDCA has no specific provision. Instead, Article II of the latter simply
alludes to the VFA in describing U.S. personnel, a term defined under Article I of the treaty as follows:

As used in this Agreement, "United States personnel" means United States military and civilian personnel temporarily in
the Philippines in connection with activities approved by the Philippine Government. Within this definition:

1. The term "military personnel" refers to military members of the United States Army, Navy, Marine Corps,
Air Force, and Coast Guard.

2. The term "civilian personnel" refers to individuals who are neither nationals of nor ordinarily resident in
the Philippines and who are employed by the United States armed forces or who are accompanying the
United States armed forces, such as employees of the American Red Cross and the United Services
Organization. 258

Article II of EDCA must then be read with Article III of the VFA, which provides for the entry accommodations to be
accorded to U.S. military and civilian personnel:

1. The Government of the Philippines shall facilitate the admission of United States personnel and their


departure from the Philippines in connection with activities covered by this agreement.

2. United States military personnel shall be exempt from passport and visa regulations upon entering and
departing the Philippines.

3. The following documents only, which shall be required in respect of United States military personnel who enter
the Philippines; xx xx.

4. United States civilian personnel shall be exempt from visa requirements but shall present, upon
demand, valid passports upon entry and departure of the Philippines. (Emphases Supplied)

By virtue of Articles I and III of the VFA, the Philippines already allows U.S. military and civilian personnel to be
"temporarily in the Philippines," so long as their presence is "in connection with activities approved by the Philippine
Government." The Philippines, through Article III, even guarantees that it shall facilitate the admission of U.S. personnel
into the country and grant exemptions from passport and visa regulations. The VFA does not even limit their temporary
presence to specific locations.

Based on the above provisions, the admission and presence of U.S. military and civilian personnel in Philippine
territory are already allowed under the VFA, the treaty supposedly being implemented by EDCA. What EDCA has
effectively done, in fact, is merely provide the mechanism to identify the locations in which U.S. personnel may perform
allowed activities pursuant to the VFA. As the implementing agreement, it regulates and limits the presence of U.S.
personnel in the country.

b. EDCA does not provide the legal basis for admission of U.S. contractors into Philippine territory; their entry must be
sourced from extraneous Philippine statutes and regulations for the admission of alien employees or business persons.

Of the three aforementioned classes of individuals who will be conducting certain activities within the Agreed Locations,
we note that only U.S. contractors are not explicitly mentioned in the VFA. This does not mean, though, that the
recognition of their presence under EDCA is ipso facto an amendment of the treaty, and that there must be Senate
concurrence before they are allowed to enter the country.

Nowhere in EDCA are U.S. contractors guaranteed immediate admission into the Philippines. Articles III and IV, in fact,
merely grant them the right of access to, and the authority to conduct certain activities within the Agreed Locations. Since
Article II(3) of EDCA specifically leaves out U.S. contractors from the coverage of the VFA, they shall not be granted the
same entry accommodations and privileges as those enjoyed by U.S. military and civilian personnel under the VFA.

Consequently, it is neither mandatory nor obligatory on the part of the Philippines to admit U.S. contractors into the
country.  We emphasize that the admission of aliens into Philippine territory is "a matter of pure permission and simple
259

tolerance which creates no obligation on the part of the government to permit them to stay."  Unlike U.S. personnel who
260

are accorded entry accommodations, U.S. contractors are subject to Philippine immigration laws.  The latter must comply
261

with our visa and passport regulations  and prove that they are not subject to exclusion under any provision of Philippine
262

immigration laws.  The President may also deny them entry pursuant to his absolute and unqualified power to prohibit or
263

prevent the admission of aliens whose presence in the country would be inimical to public interest. 264

In the same vein, the President may exercise the plenary power to expel or deport U.S. contractors  as may be265

necessitated by national security, public safety, public health, public morals, and national interest.  They may also be
266

deported if they are found to be illegal or undesirable aliens pursuant to the Philippine Immigration Act  and the Data
267

Privacy Act.  In contrast, Article 111(5) of the VFA requires a request for removal from the Philippine government before a
268

member of the U.S. personnel may be "dispos[ed] xx x outside of the Philippines."

c. Authorized activities of U.S. military and civilian personnel within Philippine territory are in furtherance of the MDT and
the VFA

We begin our analysis by quoting the relevant sections of the MDT and the VFA that pertain to the activities in which U.S.
military and civilian personnel may engage:

MUTUAL DEFENSE TREATY

Article II

In order more effectively to achieve the objective of this Treaty, the Parties separately and jointly by self-help and mutual
aid will maintain and develop their individual and collective capacity to resist armed attack.
Article III

The Parties, through their Foreign Ministers or their deputies, will consult together from time to time regarding
the implementation of this Treaty and whenever in the opinion of either of them the territorial integrity, political
independence or security of either of the Parties is threatened by external armed attack in the Pacific.

VISITING FORCES AGREEMENT

Preamble

xxx

Reaffirming their obligations under the Mutual Defense Treaty of August 30, 1951;

Noting that from time to time elements of the United States armed forces may visit the Republic of the Philippines;

Considering that cooperation between the United States and the Republic of the Philippines promotes their common
security interests;

xxx

Article I - Definitions

As used in this Agreement, "United States personnel" means United States military and civilian personnel temporarily in
the Philippines in connection with activities approved by the Philippine Government. Within this definition: xx x

Article II - Respect for Law

It is the duty of United States personnel to respect the laws of the Republic of the Philippines and to abstain from
any activity inconsistent with the spirit of this agreement, and, in particular, from any political activity in the
Philippines. The Government of the United States shall take all measures within its authority to ensure that this is done.

Article VII - Importation and Exportation

1. United States Government equipment, materials, supplies, and other property imported into or acquired in the
Philippines by or on behalf of the United States armed forces in connection with activities to which this agreement
applies, shall be free of all Philippine duties, taxes and other similar charges. Title to such property shall remain with the
United States, which may remove such property from the Philippines at any time, free from export duties, taxes, and other
similar charges. x x x.

Article VIII - Movement of Vessels and Aircraft

1. Aircraft operated by or for the United States armed forces may enter the Philippines upon approval of the
Government of the Philippines in accordance with procedures stipulated in implementing arrangements.

2. Vessels operated by or for the United States armed forces may enter the Philippines upon approval of the
Government of the Philippines. The movement of vessels shall be in accordance with international custom and
practice governing such vessels, and such agreed implementing arrangements as necessary. x x x (Emphases
Supplied)

Manifest in these provisions is the abundance of references to the creation of further "implementing arrangements"
including the identification of "activities [to be] approved by the Philippine Government." To determine the parameters of
these implementing arrangements and activities, we referred to the content, purpose, and framework of the MDT and the
VFA.

By its very language, the MDT contemplates a situation in which both countries shall engage in joint activities, so that they
can maintain and develop their defense capabilities. The wording itself evidently invites a reasonable construction that
the joint activities shall involve joint military trainings, maneuvers, and exercises. Both the interpretation  and the
269

subsequent practice  of the parties show that the MDT independently allows joint military exercises in the country. Lim v.
270

Executive Secretary  and Nicolas v. Romulo  recognized that Balikatan exercises, which are activities that seek to
271 272

enhance and develop the strategic and technological capabilities of the parties to resist an armed attack, "fall squarely
under the provisions of the RP-US MDT."  In Lim, the Court especially noted that the Philippines and the U.S. continued
273

to conduct joint military exercises even after the expiration of the MBA and even before the conclusion of the VFA.  These
274

activities presumably related to the Status of Forces Agreement, in which the parties agreed on the status to be accorded
to U.S. military and civilian personnel while conducting activities in the Philippines in relation to the MDT.
275

Further, it can be logically inferred from Article V of the MDT that these joint activities may be conducted on Philippine or
on U.S. soil. The article expressly provides that the term armed attack includes "an armed attack on the metropolitan
territory of either of the Parties, or on the island territories under its jurisdiction in the Pacific or on its armed forces,
public vessels or aircraft in the Pacific." Surely, in maintaining and developing our defense capabilities, an assessment
or training will need to be performed, separately and jointly by self-help and mutual aid, in the territories of the contracting
parties. It is reasonable to conclude that the assessment of defense capabilities would entail understanding the terrain,
wind flow patterns, and other environmental factors unique to the Philippines.

It would also be reasonable to conclude that a simulation of how to respond to attacks in vulnerable areas would be part of
the training of the parties to maintain and develop their capacity to resist an actual armed attack and to test and validate
the defense plan of the Philippines. It is likewise reasonable to imagine that part of the training would involve an analysis
of the effect of the weapons that may be used and how to be prepared for the eventuality. This Court recognizes that all of
this may require training in the area where an armed attack might be directed at the Philippine territory.

The provisions of the MDT must then be read in conjunction with those of the VFA.

Article I of the VFA indicates that the presence of U.S. military and civilian personnel in the Philippines is "in connection
with activities approved by the Philippine Government." While the treaty does not expressly enumerate or detail the nature
of activities of U.S. troops in the country, its Preamble makes explicit references to the reaffirmation of the obligations of
both countries under the MDT. These obligations include the strengthening of international and regional security in the
Pacific area and the promotion of common security interests.

The Court has already settled in Lim v. Executive Secretary that the phrase "activities approved by the Philippine
Government" under Article I of the VFA was intended to be ambiguous in order to afford the parties flexibility to adjust the
details of the purpose of the visit of U.S. personnel.  In ruling that the Terms of Reference for the Balikatan Exercises in
276

2002 fell within the context of the treaty, this Court explained:

After studied reflection, it appeared farfetched that the ambiguity surrounding the meaning of the word "activities"
arose from accident. In our view, it was deliberately made that way to give both parties a certain leeway in
negotiation. In this manner, visiting US forces may sojourn in Philippine territory for purposes other than military.
As conceived, the joint exercises may include training on new techniques of patrol and surveillance to protect the nation's
marine resources, sea search-and-rescue operations to assist vessels in distress, disaster relief operations, civic action
projects such as the building of school houses, medical and humanitarian missions, and the like.

Under these auspices, the VFA gives legitimacy to the current Balikatan exercises. It is only logical to assume that
"Balikatan 02-1," a "mutual anti-terrorism advising, assisting and training exercise," falls under the umbrella of
sanctioned or allowable activities in the context of the agreement. Both the history and intent of the Mutual Defense
Treaty and the VFA support the conclusion that combat-related activities - as opposed to combat itself- such as the one
subject of the instant petition, are indeed authorized. (Emphases Supplied)

The joint report of the Senate committees on foreign relations and on national defense and security further explains the
wide range and variety of activities contemplated in the VFA, and how these activities shall be identified: 277

These joint exercises envisioned in the VFA are not limited to combat-related activities; they have a wide range and
variety. They include exercises that will reinforce the AFP's ability to acquire new techniques of patrol and
surveillance to protect the country's maritime resources; sea-search and rescue operations to assist ships in distress;
and disaster-relief operations to aid the civilian victims of natural calamities, such as earthquakes, typhoons and tidal
waves.

xxxx

Joint activities under the VFA will include combat maneuvers; training in aircraft maintenance and equipment repair; civic-
action projects; and consultations and meetings of the Philippine-U.S. Mutual Defense Board. It is at the level of the
Mutual Defense Board-which is headed jointly by the Chief of Staff of the AFP and the Commander in Chief of the U.S.
Pacific Command-that the VFA exercises are planned. Final approval of any activity involving U.S. forces is,
however, invariably given by the Philippine Government.

xxxx

Siazon clarified that it is not the VFA by itself that determines what activities will be conducted between the armed
forces of the U.S. and the Philippines. The VFA regulates and provides the legal framework for the presence,
conduct and legal status of U.S. personnel while they are in the country for visits, joint exercises and other related
activities. (Emphases Supplied)

What can be gleaned from the provisions of the VFA, the joint report of the Senate committees on foreign
relations and on national defense and security, and the ruling of this Court in Lim is that the "activities" referred
to in the treaty are meant to be specified and identified infurther agreements. EDCA is one such agreement.

EDCA seeks to be an instrument that enumerates the Philippine-approved activities of U.S. personnel referred to in the
VFA. EDCA allows U.S. military and civilian personnel to perform "activities approved by the Philippines, as those terms
are defined in the VFA"  and clarifies that these activities include those conducted within the Agreed Locations:
278

1. Security cooperation exercises; joint and combined training activities; humanitarian assistance and disaster relief
activities; and such other activities as may be agreed upon by the Parties 279
2. Training; transit; support and related activities; refueling of aircraft; bunkering of vessels; temporary maintenance of
vehicles, vessels, and aircraft; temporary accommodation of personnel; communications; prepositioning of equipment,
supplies, and materiel; deployment of forces and materiel; and such other activities as the Parties may agree 280

3. Exercise of operational control over the Agreed Locations for construction activities and other types of activity, including
alterations and improvements thereof 281

4. Exercise of all rights and authorities within the Agreed Locations that are necessary for their operational control or
defense, including the adoption of apfropriate measures to protect U.S. forces and contractors 282

5. Use of water, electricity, and other public utilities 283

6. Operation of their own telecommunication systems, including the utilization of such means and services as are required
to ensure the full ability to operate telecommunication systems, as well as the use of the necessary radio spectrum
allocated for this purpose 284

According to Article I of EDCA, one of the purposes of these activities is to maintain and develop, jointly and by mutual aid,
the individual and collective capacities of both countries to resist an armed attack. It further states that the activities are in
furtherance of the MDT and within the context of the VFA.

We note that these planned activities are very similar to those under the Terms of Reference  mentioned in Lim. Both
285

EDCA and the Terms of Reference authorize the U.S. to perform the following: (a) participate in training exercises; (b)
retain command over their forces; (c) establish temporary structures in the country; (d) share in the use of their respective
resources, equipment and other assets; and (e) exercise their right to self-defense. We quote the relevant portion of the
Terms and Conditions as follows: 286

I. POLICY LEVEL

xxxx

No permanent US basing and support facilities shall be established. Temporary structures such as those for troop
billeting, classroom instruction and messing may be set up for use by RP and US Forces during the Exercise.

The Exercise shall be implemented jointly by RP and US Exercise Co-Directors under the authority of the Chief of Staff,
AFP. In no instance will US Forces operate independently during field training exercises (FTX). AFP and US Unit
Commanders will retain command over their respective forces under the overall authority of the Exercise Co-
Directors. RP and US participants shall comply with operational instructions of the AFP during the FTX.

The exercise shall be conducted and completed within a period of not more than six months, with the projected
participation of 660 US personnel and 3,800 RP Forces. The Chief of Staff, AFP shall direct the Exercise Co-Directors to
wind up and terminate the Exercise and other activities within the six month Exercise period.

The Exercise is a mutual counter-terrorism advising, assisting and training Exercise relative to Philippine efforts
against the ASG, and will be conducted on the Island of Basilan. Further advising, assisting and training exercises shall be
conducted in Malagutay and the Zamboanga area. Related activities in Cebu will be for support of the Exercise.

xx xx.

US exercise participants shall not engage in combat, without prejudice to their right of self-defense.

These terms of Reference are for purposes of this Exercise only and do not create additional legal obligations between the
US Government and the Republic of the Philippines.

II. EXERCISE LEVEL

1. TRAINING

a. The Exercise shall involve the conduct of mutual military assisting, advising and training of RP and US
Forces with the primary objective of enhancing the operational capabilities of both forces to combat terrorism.

b. At no time shall US Forces operate independently within RP territory.

c. Flight plans of all aircraft involved in the exercise will comply with the local air traffic regulations.

2. ADMINISTRATION & LOGISTICS

xxxx
a. RP and US participating forces may share, in accordance with their respective laws and regulations, in the use of
their resources, equipment and other assets. They will use their respective logistics channels. x x x. (Emphases
Supplied)

After a thorough examination of the content, purpose, and framework of the MDT and the VFA, we find that EDCA has
remained within the parameters set in these two treaties. Just like the Terms of Reference mentioned in Lim, mere
adjustments in detail to implement the MDT and the VFA can be in the form of executive agreements.

Petitioners assert  that the duration of the activities mentioned in EDCA is no longer consistent with the temporary nature
287

of the visits as contemplated in the VFA. They point out that Article XII(4) of EDCA has an initial term of 10 years, a term
automatically renewed unless the Philippines or the U.S. terminates the agreement. According to petitioners, such length
of time already has a badge of permanency.

In connection with this, Justice Teresita J. Leonardo-De Castro likewise argues in her Concurring and Dissenting Opinion
that the VFA contemplated mere temporary visits from U.S. forces, whereas EDCA allows an unlimited period for U.S.
forces to stay in the Philippines.288

However, the provisions of EDCA directly contradict this argument by limiting itself to 10 years of effectivity. Although this
term is automatically renewed, the process for terminating the agreement is unilateral and the right to do so automatically
accrues at the end of the 10 year period. Clearly, this method does not create a permanent obligation.

Drawing on the reasoning in Lim, we also believe that it could not have been by chance that the VFA does not include a
maximum time limit with respect to the presence of U.S. personnel in the country. We construe this lack of specificity as a
deliberate effort on the part of the Philippine and the U.S. governments to leave out this aspect and reserve it for the
"adjustment in detail" stage of the implementation of the treaty. We interpret the subsequent, unconditional concurrence of
the Senate in the entire text of the VFA as an implicit grant to the President of a margin of appreciation in determining the
duration of the "temporary" presence of U.S. personnel in the country.

Justice Brion's dissent argues that the presence of U.S. forces under EDCA is "more permanent" in nature.  However,289

this argument has not taken root by virtue of a simple glance at its provisions on the effectivity period. EDCA does not
grant permanent bases, but rather temporary rotational access to facilities for efficiency. As Professor Aileen S.P. Baviera
notes:

The new EDCA would grant American troops, ships and planes rotational access to facilities of the Armed Forces of the
Philippines – but not permanent bases which are prohibited under the Philippine Constitution - with the result of reducing
response time should an external threat from a common adversary crystallize. 290

EDCA is far from being permanent in nature compared to the practice of states as shown in other defense cooperation
agreements. For example, Article XIV(l) of the U.S.-Romania defense agreement provides the following:

This Agreement is concluded for an indefinite period and shall enter into force in accordance with the internal laws of
each Party x x x. (emphasis supplied)

Likewise, Article 36(2) of the US-Poland Status of Forces Agreement reads:

This Agreement has been concluded for an indefinite period of time. It may be terminated by written notification by
either Party and in that event it terminates 2 years after the receipt of the notification. (emphasis supplied)

Section VIII of US.-Denmark Mutual Support Agreement similarly provides:

8.1 This Agreement, which consists of a Preamble, SECTIONs I-VIII, and Annexes A and B, shall become effective on the
date of the last signature affixed below and shall remain in force until terminated by the Parties, provided that it may
be terminated by either Party upon 180 days written notice of its intention to do so to the other Party. (emphasis supplied)

On the other hand, Article XXI(3) of the US.-Australia Force Posture Agreement provides a longer initial term:

3. This Agreement shall have an initial term of 25 years and thereafter shall continue in force, but may be terminated
by either Party at any time upon one year's written notice to the other Party through diplomatic channels. (emphasis
supplied)

The phrasing in EDCA is similar to that in the U.S.-Australia treaty but with a term less than half of that is provided in the
latter agreement. This means that EDCA merely follows the practice of other states in not specifying a non-extendible
maximum term. This practice, however, does not automatically grant a badge of permanency to its terms. Article XII(4) of
EDCA provides very clearly, in fact, that its effectivity is for an initial term of 10 years, which is far shorter than the terms of
effectivity between the U.S. and other states. It is simply illogical to conclude that the initial, extendible term of 10 years
somehow gives EDCA provisions a permanent character.

The reasoning behind this interpretation is rooted in the constitutional role of the President who, as Commander-in-Chief
of our armed forces, is the principal strategist of the nation and, as such, duty-bound to defend our national sovereignty
and territorial integrity;  who, as chief architect of our foreign relations, is the head policymaker tasked to assess, ensure,
291

and protect our national security and interests;  who holds the most comprehensive and most confidential information
292
about foreign countries  that may affect how we conduct our external affairs; and who has unrestricted access to highly
293

classified military intelligence data  that may threaten the life of the nation. Thus, if after a geopolitical prognosis of
294

situations affecting the country, a belief is engendered that a much longer period of military training is needed, the
President must be given ample discretion to adopt necessary measures including the flexibility to set an extended
timetable.

Due to the sensitivity and often strict confidentiality of these concerns, we acknowledge that the President may not always
be able to candidly and openly discuss the complete situation being faced by the nation. The Chief Executive's hands
must not be unduly tied, especially if the situation calls for crafting programs and setting timelines for approved activities.
These activities may be necessary for maintaining and developing our capacity to resist an armed attack, ensuring our
national sovereignty and territorial integrity, and securing our national interests. If the Senate decides that the President is
in the best position to define in operational terms the meaning of temporary in relation to the visits, considered individually
or in their totality, the Court must respect that policy decision. If the Senate feels that there is no need to set a time limit to
these visits, neither should we.

Evidently, the fact that the VFA does not provide specificity in regard to the extent of the "temporary" nature of the visits of
U.S. personnel does not suggest that the duration to which the President may agree is unlimited. Instead, the boundaries
of the meaning of the term temporary in Article I of the treaty must be measured depending on the purpose of each visit or
activity.  That purpose must be analyzed on a case-by-case basis depending on the factual circumstances surrounding
295

the conclusion of the implementing agreement. While the validity of the President's actions will be judged under less
stringent standards, the power of this Court to determine whether there was grave abuse of discretion remains
unimpaired.

d. Authorized activities performed by US. contractors within Philippine territory - who were legitimately permitted to enter
the country independent of EDCA - are subject to relevant Philippine statutes and regulations and must be consistent with
the MDT and the VFA

Petitioners also raise  concerns about the U.S. government's purported practice of hiring private security contractors in
296

other countries. They claim that these contractors - one of which has already been operating in Mindanao since 2004 -
have been implicated in incidents or scandals in other parts of the globe involving rendition, torture and other human rights
violations. They also assert that these contractors employ paramilitary forces in other countries where they are operating.

Under Articles III and IV of EDCA, U.S. contractors are authorized to perform only the following activities:

1. Training; transit; support and related activities; refueling of aircraft; bunkering of vessels; temporary
maintenance of vehicles, vessels, and aircraft; temporary accommodation of personnel; communications;
prepositioning of equipment, supplies, and materiel; deployment of forces and materiel; and such other activities
as the Parties may agree 297

2. Prepositioning and storage of defense equipment, supplies, and materiel, including delivery, management,
inspection, use, maintenance, and removal of such equipment, supplies and materiel 298

3. Carrying out of matters in accordance with, and to the extent permissible under, U.S. laws, regulations, and
policies 299

EDCA requires that all activities within Philippine territory be in accordance with Philippine law. This means that certain
privileges denied to aliens are likewise denied to foreign military contractors. Relevantly, providing security  and carrying,
300

owning, and possessing firearms  are illegal for foreign civilians.


301

The laws in place already address issues regarding the regulation of contractors. In the 2015 Foreign Investment Negative
list,  the Executive Department has already identified corporations that have equity restrictions in Philippine jurisdiction.
302

Of note is No. 5 on the list - private security agencies that cannot have any foreign equity by virtue of Section 4 of Republic
Act No. 5487;  and No. 15, which regulates contracts for the construction of defense-related structures based on
303

Commonwealth Act No. 541.

Hence, any other entity brought into the Philippines by virtue of EDCA must subscribe to corporate and civil requirements
imposed by the law, depending on the entity's corporate structure and the nature of its business.

That Philippine laws extraneous to EDCA shall govern the regulation of the activities of U.S. contractors has been clear
even to some of the present members of the Senate.

For instance, in 2012, a U.S. Navy contractor, the Glenn Marine, was accused of spilling fuel in the waters off Manila
Bay.  The Senate Committee on Foreign Relations and the Senate Committee on Environment and Natural Resources
304

chairperson claimed environmental and procedural violations by the contractor.  The U.S. Navy investigated the
305

contractor and promised stricter guidelines to be imposed upon its contractors.  The statement attributed to Commander
306

Ron Steiner of the public affairs office of the U.S. Navy's 7th Fleet - that U.S. Navy contractors are bound by Philippine
laws - is of particular relevance. The statement acknowledges not just the presence of the contractors, but also the U.S.
position that these contractors are bound by the local laws of their host state. This stance was echoed by other U.S. Navy
representatives. 307
This incident simply shows that the Senate was well aware of the presence of U.S. contractors for the purpose of fulfilling
the terms of the VFA. That they are bound by Philippine law is clear to all, even to the U.S.

As applied to EDCA, even when U.S. contractors are granted access to the Agreed Locations, all their activities must be
consistent with Philippine laws and regulations and pursuant to the MDT and the VFA.

While we recognize the concerns of petitioners, they do not give the Court enough justification to strike down EDCA.
In Lim v. Executive Secretary, we have already explained that we cannot take judicial notice of claims aired in news
reports, "not because of any issue as to their truth, accuracy, or impartiality, but for the simple reason that facts must be
established in accordance with the rules of evidence."  What is more, we cannot move one step ahead and speculate that
308

the alleged illegal activities of these contractors in other countries would take place in the Philippines with certainty. As can
be seen from the above discussion, making sure that U.S. contractors comply with Philippine laws is a function of law
enforcement. EDCA does not stand in the way of law enforcement.

Nevertheless, we emphasize that U.S. contractors are explicitly excluded from the coverage of the VFA. As visiting aliens,
their entry, presence, and activities are subject to all laws and treaties applicable within the Philippine territory. They may
be refused entry or expelled from the country if they engage in illegal or undesirable activities. There is nothing that
prevents them from being detained in the country or being subject to the jurisdiction of our courts. Our penal laws,  labor
309

laws,  and immigrations laws  apply to them and therefore limit their activities here. Until and unless there is another law
310 311

or treaty that specifically deals with their entry and activities, their presence in the country is subject to unqualified
Philippine jurisdiction.

EDCA does not allow the presence of U.S.-owned or -controlled military facilities and bases in the Philippines

Petitioners Saguisag et al. claim that EDCA permits the establishment of U.S. military bases through the "euphemistically"
termed "Agreed Locations. "  Alluding to the definition of this term in Article II(4) of EDCA, they point out that these
312

locations are actually military bases, as the definition refers to facilities and areas to which U.S. military forces have
access for a variety of purposes. Petitioners claim that there are several badges of exclusivity in the use of the Agreed
Locations by U.S. forces. First, Article V(2) of EDCA alludes to a "return" of these areas once they are no longer needed
by U.S. forces, indicating that there would be some transfer of use. Second, Article IV(4) ofEDCA talks about American
forces' unimpeded access to the Agreed Locations for all matters relating to the prepositioning and storage of U.S. military
equipment, supplies, and materiel. Third, Article VII of EDCA authorizes U.S. forces to use public utilities and to operate
their own telecommunications system.

a. Preliminary point on badges of exclusivity

As a preliminary observation, petitioners have cherry-picked provisions of EDCA by presenting so-called "badges of
exclusivity," despite the presence of contrary provisions within the text of the agreement itself.

First, they clarify the word "return" in Article V(2) of EDCA. However, the use of the word "return" is within the context of a
lengthy provision. The provision as a whole reads as follows:

The United States shall return to the Philippines any Agreed Locations, or any portion thereof, including non-relocatable
structures and assemblies constructed, modified, or improved by the United States, once no longer required by United
States forces for activities under this Agreement. The Parties or the Designated Authorities shall consult regarding the
terms of return of any Agreed Locations, including possible compensation for improvements or construction.

The context of use is "required by United States forces for activities under this Agreement." Therefore, the return of an
Agreed Location would be within the parameters of an activity that the Mutual Defense Board (MDB) and the Security
Engagement Board (SEB) would authorize. Thus, possession by the U.S. prior to its return of the Agreed Location would
be based on the authority given to it by a joint body co-chaired by the "AFP Chief of Staff and Commander, U.S. PACOM
with representatives from the Philippines' Department of National Defense and Department of Foreign Affairs sitting as
members."  The terms shall be negotiated by both the Philippines and the U.S., or through their Designated Authorities.
313

This provision, seen as a whole, contradicts petitioners' interpretation of the return as a "badge of exclusivity." In fact, it
shows the cooperation and partnership aspect of EDCA in full bloom.

Second, the term "unimpeded access" must likewise be viewed from a contextual perspective. Article IV(4) states that
U.S. forces and U.S. contractors shall have "unimpeded access to Agreed Locations for all matters relating to the
prepositioning and storage of defense equipment, supplies, and materiel, including delivery, management, inspection, use,
maintenance, and removal of such equipment, supplies and materiel."

At the beginning of Article IV, EDCA states that the Philippines gives the U.S. the authority to bring in these equipment,
supplies, and materiel through the MDB and SEB security mechanism. These items are owned by the U.S.,  are 314

exclusively for the use of the U.S.  and, after going through the joint consent mechanisms of the MDB and the SEB, are
315

within the control of the U.S.  More importantly, before these items are considered prepositioned, they must have gone
316

through the process of prior authorization by the MDB and the SEB and given proper notification to the AFP. 317

Therefore, this "unimpeded access" to the Agreed Locations is a necessary adjunct to the ownership, use, and control of
the U.S. over its own equipment, supplies, and materiel and must have first been allowed by the joint mechanisms in play
between the two states since the time of the MDT and the VFA. It is not the use of the Agreed Locations that is
exclusive per se; it is mere access to items in order to exercise the rights of ownership granted by virtue of the Philippine
Civil Code. 318

As for the view that EDCA authorizes U.S. forces to use public utilities and to operate their own telecommunications
system, it will be met and answered in part D, infra.

Petitioners also point out  that EDCA is strongly reminiscent of and in fact bears a one-to-one correspondence with the
319

provisions of the 1947 MBA. They assert that both agreements (a) allow similar activities within the area; (b) provide for
the same "species of ownership" over facilities; and (c) grant operational control over the entire area. Finally, they
argue  that EDCA is in fact an implementation of the new defense policy of the U.S. According to them, this policy was
320

not what was originally intended either by the MDT or by the VFA.

On these points, the Court is not persuaded.

The similar activities cited by petitioners  simply show that under the MBA, the U.S. had the right to construct, operate,
321

maintain, utilize, occupy, garrison, and control the bases. The so-called parallel provisions of EDCA allow only operational
control over the Agreed Locations specifically for construction activities. They do not allow the overarching power to
operate, maintain, utilize, occupy, garrison, and control a base with full discretion. EDCA in fact limits the rights of the U.S.
in respect of every activity, including construction, by giving the MDB and the SEB the power to determine the details of all
activities such as, but not limited to, operation, maintenance, utility, occupancy, garrisoning, and control. 322

The "species of ownership" on the other hand, is distinguished by the nature of the property. For immovable property
constructed or developed by the U.S., EDCA expresses that ownership will automatically be vested to the
Philippines.  On the other hand, for movable properties brought into the Philippines by the U.S., EDCA provides that
323

ownership is retained by the latter. In contrast, the MBA dictates that the U.S. retains ownership over immovable and
movable properties.

To our mind, both EDCA and the MBA simply incorporate what is already the law of the land in the Philippines. The Civil
Code's provisions on ownership, as applied, grant the owner of a movable property full rights over that property, even if
located in another person's property. 324

The parallelism, however, ends when the situation involves facilities that can be considered immovable. Under the MBA,
the U.S. retains ownership if it paid for the facility.  Under EDCA, an immovable is owned by the Philippines, even if built
325

completely on the back of U.S. funding.  This is consistent with the constitutional prohibition on foreign land ownership.
326 327

Despite the apparent similarity, the ownership of property is but a part of a larger whole that must be considered before
the constitutional restriction is violated. Thus, petitioners' points on operational control will be given more attention in the
discussion below. The arguments on policy are, however, outside the scope of judicial review and will not be discussed

Moreover, a direct comparison of the MBA and EDCA will result in several important distinctions that would allay suspicion
that EDCA is but a disguised version of the MBA.

b. There are substantial matters that the US. cannot do under EDCA, but which it was authorized to do under the 1947
MBA

The Philippine experience with U.S. military bases under the 1947 MBA is simply not possible under EDCA for a number
of important reasons.

First,  in the 1947 MBA, the U.S. retained all rights of jurisdiction in and over Philippine territory occupied by
American bases. In contrast, the U.S. under EDCA does not enjoy any such right over any part of the Philippines in
which its forces or equipment may be found. Below is a comparative table between the old treaty and EDCA:

1947 MBA/ 1946 Treaty of General EDCA


Relations

1947 MBA, Art. I(1): EDCA, preamble:

The Government of the Republic of Affirming that the Parties share an


the Philippines (hereinafter referred to as the understanding for the United States not to
Philippines) grants to the Government of establish a permanent military presence or
the United States of America (hereinafter base in the territory of the Philippines;
referred to as the United States) the right to
retain the use of the bases in the xxxx
Philippines listed in Annex A attached hereto.
Recognizing that all United States access to
1947 MBA, Art. XVII(2): and use of facilities and areas will be at the
invitation of the Philippines and with full
All buildings and structures which respect for the Philippine Constitution and
are erected by the United States in the
bases shall be the property of the United Philippine laws;
States and may be removed by it before the
expiration of this Agreement or the earlier xxxx
relinquishment of the base on which the
structures are situated. There shall be no EDCA, Art. II(4):
obligation on the part of the Philippines or of
the United States to rebuild or repair any
"Agreed Locations" means facilities and
destruction or damage inflicted from any
areas that are provided by the Government
cause whatsoever on any of the said buildings
of the Philippines through the AFP and that
or structures owned or used by the United
United States forces, United States
States in the bases. x x x x.
contractors, and others as mutually agreed,
shall have the right to access and use
1946 Treaty of Gen. Relations, Art. I: pursuant to this Agreement. Such Agreed
Locations may be listed in an annex to be
The United States of America agrees to appended to this Agreement, and may be
withdraw and surrender, and does hereby further described in implementing
withdraw and surrender, all rights of arrangements.
possession, supervision, jurisdiction,
control or sovereignty existing and EDCA, Art. V:
exercised by the United States of America in
and over the territory and the people of the
1. The Philippines shall retain ownership of
Philippine Islands, except the use of such
and title to Agreed Locations.
bases, necessary appurtenances to such
bases, and the rights incident thereto, as
the United States of America, by agreement xxxx
with the Republic of the Philippines may deem
necessary to retain for the mutual protection of 4. All buildings, non-relocatable structures,
the Republic of the Philippines and of the and assemblies affixed to the land in the
United States of America. x x x. Agreed Locations, including ones altered or
improved by United States forces, remain the
property of the Philippines. Permanent
buildings constructed by United States forces
become the property of the Philippines, once
constructed, but shall be used by United
States forces until no longer required by
United States forces.

Second,  in the bases agreement, the U.S. and the Philippines were visibly not on equal footing when it came to
deciding whether to expand or to increase the number of bases, as the Philippines may be compelled to negotiate with
the U.S. the moment the latter requested an expansion of the existing bases or to acquire additional bases. In EDCA,
U.S. access is purely at the invitation of the Philippines.

1947 MBA/ 1946 Treaty of General EDCA


Relations

1947 MBA, Art.I(3): EDCA, preamble:

The Philippines agree to enter into Recognizing that all United States access to


negotiations with the United States at the and use of facilities and areas will be at the
latter's request, to permit the United invitation of the Philippines and with full
States to expand such bases, to exchange respect for the Philippine Constitution and
such bases for other bases, to acquire Philippine laws;
additional bases, or relinquish rights to
bases, as any of such exigencies may be xxxx
required by military necessity.
EDCA. Art. II(4):
1946 Treaty of Gen. Relations, Art. I:
"Agreed Locations" means facilities and
The United States of America agrees to areas that are provided by the Government
withdraw and surrender, and does hereby of the Philippines through the AFP and that
withdraw and surrender, all rights of United States forces, United States
possession, supervision, jurisdiction, contractors, and others as mutually agreed,
control or sovereignty existing and shall have the right to access and use
exercised by the United States of America in pursuant to this Agreement. Such Agreed
and over the territory and the people of the Locations may be listed in an annex to be
Philippine Islands, except the use of such appended to this Agreement, and may be
bases, necessary appurtenances to such further described in implementing
bases, and the rights incident thereto, as the arrangements.
United States of America, by agreement
with the Republic of the Philippines may
deem necessary to retain for the mutual
protection of the Republic of the Philippines
and of the United States of America. x x x.

Third,  in EDCA, the Philippines is- guaranteed access over the entire area of the Agreed Locations. On the other
hand, given that the U.S. had complete control over its military bases under the 1947 MBA, the treaty did not provide
for any express recognition of the right of access of Philippine authorities. Without that provision and in light of the
retention of U.S. sovereignty over the old military bases, the U.S. could effectively prevent Philippine authorities
from entering those bases.

1947 MBA EDCA

EDCA, Art. III(5):


No equivalent provision.
The Philippine Designated Authority and its
authorized representative shall have access
to the entire area of the Agreed Locations.
Such access shall be provided promptly
consistent with operational safety and security
requirements in accordance with agreed
procedures developed by the Parties.

Fourth,  in the bases agreement, the U.S. retained the right, power, and authority over the establishment, use,
operation, defense, and control of military bases, including the limits of territorial waters and air space adjacent to or
in the vicinity of those bases. The only standard used in determining the extent of its control was military necessity.
On the other hand, there is no such grant of power or authority under EDCA. It merely allows the U.S. to exercise
operational control over the construction of Philippine-owned structures and facilities:

1947 MBA EDCA

1947 MBA, Art.I(2): EDCA, Art. III(4):

The Philippines agrees to permit the United The Philippines hereby grants to the United
States, upon notice to the Philippines, to use States, through bilateral security
such of those bases listed in Annex B as the mechanisms, such as the MDB and
United States determines to be required by SEB, operational control of Agreed
military necessity. Locations for construction
activities and authority to undertake such
1947 MBA, Art. III(1): activities on, and make alterations and
improvements to, Agreed Locations. United
It is mutually agreed that the United States forces shall consult on issues
States shall have the rights, power and regarding such construction, alterations,
authority within the bases which and improvements based on the Parties'
are necessary for the establishment, use, shared intent that the technical requirements
operation and defense thereof or and construction standards of any such
appropriate for the control thereof and all projects undertaken by or on behalf of United
the rights, power and authority within the States forces should be consistent with the
limits of territorial waters and air space requirements and standards of both Parties.
adjacent to, or in the vicinity of, the bases
which are necessary to provide access to
them, or appropriate for their control.

Fifth,  the U.S. under the bases agreement was given the authority to use Philippine territory for additional staging
areas, bombing and gunnery ranges. No such right is given under EDCA, as seen below:

1947 MBA EDCA

1947 MBA, Art. VI: EDCA, Art. III(1):

The United States shall, subject to previous With consideration of the views of


agreement with the Philippines, have the right the Parties,
to use land and coastal sea areas of the Philippines hereby authorizes and
appropriate size and location for periodic agrees that United States forces, United
maneuvers, for additional staging areas, States contractors, and vehicles, vessels, and
bombing and gunnery ranges, and for such aircraft operated by or for United States forces
intermediate airfields as may be required for may conduct the following activities with
safe and efficient air operations. Operations in
respect to Agreed Locations: training; transit;
such areas shall be carried on with due regard support and related activities; refueling of
and safeguards for the public safety. aircraft; bunkering of vessels; temporary
maintenance of vehicles, vessels, and aircraft;
1947 MBA, Art.I(2): temporary accommodation of personnel;
communications; prepositioning of equipment,
The Philippines agrees to permit the United supplies, and materiel; deploying forces and
States, upon notice to the Philippines, to use materiel; and such other activities as the
such of those bases listed in Annex B as the Parties may agree.
United States determines to be required by
military necessity.

Sixth,  under the MBA, the U.S. was given the right, power, and authority to control and prohibit the movement and
operation of all types of vehicles within the vicinity of the bases. The U.S. does not have any right, power, or
authority to do so under EDCA.

1947 MBA EDCA

1947 MBA, Art. 111(2)(c)


No equivalent provision.
Such rights, power and authority shall
include, inter alia, the right, power and
authority: x x x x to control (including the
right to prohibit) in so far as may be required
for the efficient operation and safety of the
bases, and within the limits of military
necessity, anchorages, moorings, landings,
takeoffs, movements and operation of
ships and water-borne craft, aircraft and
other vehicles on water, in the air or on
land comprising

Seventh,  under EDCA, the U.S. is merely given temporary access to land and facilities (including roads, ports, and
airfields). On the other hand, the old treaty gave the U.S. the right to improve and deepen the harbors, channels,
entrances, and anchorages; and to construct or maintain necessary roads and bridges that would afford it access to its
military bases.

1947 MBA EDCA

1947 MBA, Art. III(2)(b): EDCA, Art. III(2):

Such rights, power and authority shall When requested, the Designated Authority of
include, inter alia, the right, power and the Philippines shall assist in facilitating
authority: x x x x to improve and deepen transit or temporary access by United States
the harbors, channels, entrances and forces to public land and facilities (including
anchorages, and to construct or maintain roads, ports, and airfields), including those
necessary roads and bridges affording owned or controlled by local governments, and
access to the bases. to other land and facilities (including roads,
ports, and airfields).

Eighth,  in the 1947 MBA, the U.S. was granted the automatic right to use any and all public utilities, services and
facilities, airfields, ports, harbors, roads, highways, railroads, bridges, viaducts, canals, lakes, rivers, and streams in
the Philippines in the same manner that Philippine military forces enjoyed that right. No such arrangement appears in
EDCA. In fact, it merely extends to U.S. forces temporary access to public land and facilities when requested:

1947 MBA EDCA

1947 MBA, Art. VII: EDCA, Art. III(2):

It is mutually agreed that the United States When requested, the Designated Authority of
may employ and use for United States the Philippines shall assist in facilitating
military forces any and all public utilities, transit or temporary access by United States
other services and facilities, airfields, ports, forces to public land and facilities (including
harbors, roads, highways, railroads, bridges, roads, ports, and airfields), including those
viaducts, canals, lakes, rivers and streams in owned or controlled by local governments, and
the Philippines under conditions no less to other land and facilities (including roads,
favorable than those that may ports, and airfields).
be applicable from time to time to the military
forces of the Philippines.

Ninth, under EDCA, the U.S. no longer has the right, power, and authority to construct, install, maintain, and
employ any  type of facility, weapon, substance, device, vessel or vehicle, or system unlike in the old treaty. EDCA
merely grants the U.S., through bilateral security mechanisms, the authority to undertake construction, alteration, or
improvements on the Philippine-owned Agreed Locations.

1947 MBA EDCA

1947 MBA, Art. III(2)(e): EDCA, Art. III(4):

Such rights, power and authority shall The Philippines hereby grants to the United
include, inter alia, the right, power and States, through bilateral security
authority: x x x x to construct, install, mechanisms, such as the MDB and SEB,
maintain, and employ on any base any operational control of Agreed Locations for
type of facilities, weapons, substance, construction activities and authority to
device, vessel or vehicle on or under the undertake such activities on, and make
ground, in the air or on or under the water that alterations and improvements to, Agreed
may be requisite or appropriate, including Locations. United States forces shall consult
meteorological systems, aerial and water on issues regarding such construction,
navigation lights, radio and radar apparatus alterations, and improvements based on the
and electronic devices, of any desired power, Parties' shared intent that the technical
type of emission and frequency. requirements and construction standards of
any such projects undertaken by or on behalf
of United States forces should be consistent
with the requirements and standards of both
Parties.

Tenth,  EDCA does not allow the U.S. to acquire, by condemnation or expropriation proceedings, real property
belonging to any private person. The old military bases agreement gave this right to the U.S. as seen below:

1947 MBA EDCA

1947 MBA, Art. XXII(l):


No equivalent provision.
Whenever it is necessary to acquire by

condemnation or expropriation
proceedings real property belonging to any
private persons, associations or corporations
located in bases named in Annex A and
Annex B in order to carry out the purposes of
this Agreement, the Philippines will institute
and prosecute such condemnation or
expropriation proceedings in accordance with
the laws of the Philippines. The United States
agrees to reimburse the Philippines for all the
reasonable expenses, damages and costs
therebv incurred, including the value of the
property as determined by the Court. In
addition, subject to the mutual agreement of
the two Governments, the United States will
reimburse the Philippines for the reasonable
costs of transportation and removal of any
occupants displaced or ejected by reason of
the condemnation or expropriation.
Eleventh,  EDCA does not allow the U.S. to unilaterally bring into the country non-Philippine nationals who are under
its employ, together with their families, in connection with the construction, maintenance, or operation of the bases.
EDCA strictly adheres to the limits under the VFA.

1947 MBA EDCA

1947 MBA, Art. XI(l): EDCA, Art. II:

It is mutually agreed that the United States 1. "United States personnel" means United


shall have the right to bring into the States military and civilian
Philippines members of the United States personnel temporarily in the territory of the
military forces and the United States Philippines in connection with activities
nationals employed by or under a contract approved by the Philippines, as those terms
with the United States together with their are defined in the VFA.
families, and technical personnel of other
nationalities (not being persons excluded by x xx x
the laws of the Philippines) in connection with
the construction, maintenance, or operation of 3. "United States contractors" means
the bases. The United States shall make companies and firms, and their employees,
suitable arrangements so that such persons under contract or subcontract to or on behalf
may be readily identified and their status of the United States Department of Defense.
established when necessary by the Philippine United States contractors are not included as
authorities. Such persons, other than part of the definition of United States
members of the United States armed forces in personnel in this Agreement, including
uniform, shall present their travel documents within the context of the VFA.
to the appropriate Philippine authorities for
visas, it being understood that no objection
 
will be made to their travel to the
Philippines as non-immigrants.

Twelfth,  EDCA does not allow the U.S. to exercise jurisdiction over any offense committed by any person within the
Agreed Locations, unlike in the former military bases:

1947 MBA EDCA

1947 MBA, Art. XIII(l)(a):


No equivalent provision.
The Philippines consents that the United

States shall have the right to exercise


jurisdiction over the following offenses:
(a) Any offense committed by any
person within any base except where the
offender and offended parties are both
Philippine citizens (not members of the armed
forces of the United States on active duty) or
the offense is against the security of the
Philippines.

Thirteenth,  EDCA does not allow the U.S. to operate military post exchange (PX) facilities, which is free of customs
duties and taxes, unlike what the expired MBA expressly allowed. Parenthetically, the PX store has become the
cultural icon of U.S. military presence in the country.

1947 MBA EDCA

1947 MBA, Art. XVIII(l):


No equivalent provision.
It is mutually agreed that the United States

shall have the right to establish on bases,


free of all licenses; fees; sales, excise or
other taxes, or imposts; Government
agencies, including concessions, such
as sales commissaries and post
exchanges; messes and social clubs, for the
exclusive use of the United States military
forces and authorized civilian personnel
and their families. The merchandise or
services sold or dispensed by such agencies
shall be free of all taxes, duties and
inspection by the Philippine
authorities. Administrative measures shall be
taken by the appropriate authorities of the
United States to prevent the resale of goods
which are sold under the provisions of this
Article to persons not entitled to buy goods at
such agencies and, generally, to prevent
abuse of the privileges granted under this
Article. There shall be cooperation between
such authorities and the Philippines to this
end.

In sum, EDCA is a far cry from a basing agreement as was understood by the people at the time that the 1987
Constitution was adopted.

Nevertheless, a comprehensive review of what the Constitution means by "foreign military bases" and "facilities" is
required before EDCA can be deemed to have passed judicial scrutiny.

c. The meaning of military facilities and bases

An appreciation of what a military base is, as understood by the Filipino people in 1987, would be vital in determining
whether EDCA breached the constitutional restriction.

Prior to the drafting of the 1987 Constitution, the last definition of "military base" was provided under Presidential Decree
No. (PD) 1227.  Unlawful entry into a military base is punishable under the decree as supported by Article 281 of the
328

Revised Penal Code, which itself prohibits the act of trespass.

Section 2 of the law defines the term in this manner: "'[M]ilitary base' as used in this decree means any military, air, naval,
or coast guard reservation, base, fort, camp, arsenal, yard, station, or installation in the Philippines."

Commissioner Tadeo, in presenting his objections to U.S. presence in the Philippines before the 1986 Constitutional
Commission, listed the areas that he considered as military bases:

1,000 hectares Camp O'Donnel

20,000 hectares Crow Valley Weapon's Range

55,000 hectares Clark Air Base

150 hectares Wallace Air Station

400 hectares John Hay Air Station

15,000 hectares Subic Naval Base

1,000 hectares San Miguel Naval Communication

750 hectares Radio Transmitter in Capas, Tarlac

900 hectares Radio Bigot Annex at Bamban, Tarlac 329

The Bases Conversion and Development Act of 1992 described its coverage in its Declaration of Policies:

Sec. 2. Declaration of Policies. - It is hereby declared the policy of the Government to accelerate the sound and balanced
conversion into alternative productive uses of the Clark and Subic military reservations and their extensions (John Hay
Station, Wallace Air Station, O'Donnell Transmitter Station, San Miguel Naval Communications Station and Capas Relay
Station), to raise funds by the sale of portions of Metro Manila military camps, and to apply said funds as provided herein
for the development and conversion to productive civilian use of the lands covered under the 194 7 Military Bases
Agreement between the Philippines and the United States of America, as amended. 330

The result of the debates and subsequent voting is Section 25, Article XVIII of the Constitution, which specifically restricts,
among others, foreign military facilities or bases. At the time of its crafting of the Constitution, the 1986 Constitutional
Commission had a clear idea of what exactly it was restricting. While the term "facilities and bases" was left undefined, its
point of reference was clearly those areas covered by the 1947 MBA as amended.
Notably, nearly 30 years have passed since then, and the ever-evolving world of military technology and geopolitics has
surpassed the understanding of the Philippine people in 1986. The last direct military action of the U.S. in the region was
the use of Subic base as the staging ground for Desert Shield and Desert Storm during the Gulf War.  In 1991, the
331

Philippine Senate rejected the successor treaty of the 1947 MBA that would have allowed the continuation of U.S. bases
in the Philippines.

Henceforth, any proposed entry of U.S. forces into the Philippines had to evolve likewise, taking into consideration the
subsisting agreements between both parties, the rejection of the 1991 proposal, and a concrete understanding of what
was constitutionally restricted. This trend birthed the VFA which, as discussed, has already been upheld by this Court.

The latest agreement is EDCA, which proposes a novel concept termed "Agreed Locations."

By definition, Agreed Locations are

facilities and areas that are provided by the Government of the Philippines through the AFP and that United States forces,
United States contractors, and others as mutually agreed, shall have the right to access and use pursuant to this
Agreement. Such Agreed Locations may be listed in an annex to be appended to this Agreement, and may be further
described in implementing arrangements. 332

Preliminarily, respondent already claims that the proviso that the Philippines shall retain ownership of and title to the
Agreed Locations means that EDCA is "consistent with Article II of the VFA which recognizes Philippine sovereignty and
jurisdiction over locations within Philippine territory.333

By this interpretation, respondent acknowledges that the contention of petitioners springs from an understanding that the
Agreed Locations merely circumvent the constitutional restrictions. Framed differently, the bone of contention is whether
the Agreed Locations are, from a legal perspective, foreign military facilities or bases. This legal framework triggers
Section 25, Article XVIII, and makes Senate concurrence a sine qua non.

Article III of EDCA provides for Agreed Locations, in which the U.S. is authorized by the Philippines to "conduct the
following activities: "training; transit; support and related activities; refueling of aircraft; bunkering of vessels; temporary
maintenance of vehicles, vessels and aircraft; temporary accommodation of personnel; communications; prepositioning of
equipment, supplies and materiel; deploying forces and materiel; and such other activities as the Parties may agree."

This creation of EDCA must then be tested against a proper interpretation of the Section 25 restriction.

d. Reasons for the constitutional requirements and legal standards for constitutionally compatible military bases and
facilities

Section 25 does not define what is meant by a "foreign military facility or base." While it specifically alludes to U.S. military
facilities and bases that existed during the framing of the Constitution, the provision was clearly meant to apply to those
bases existing at the time and to any future facility or base. The basis for the restriction must first be deduced from the
spirit of the law, in order to set a standard for the application of its text, given the particular historical events preceding the
agreement.

Once more, we must look to the 1986 Constitutional Commissioners to glean, from their collective wisdom, the intent of
Section 25. Their speeches are rich with history and wisdom and present a clear picture of what they considered in the
crafting the provision.

SPEECH OF COMMISSIONER REGALADO 334

xxxx

We have been regaled here by those who favor the adoption of the anti-bases provisions with what purports to be an
objective presentation of the historical background of the military bases in the Philippines. Care appears, however, to have
been taken to underscore the inequity in their inception as well as their implementation, as to seriously reflect on the
supposed objectivity of the report. Pronouncements of military and civilian officials shortly after World War II are quoted in
support of the proposition on neutrality; regrettably, the implication is that the same remains valid today, as if the world
and international activity stood still for the last 40 years.

We have been given inspired lectures on the effect of the presence of the military bases on our sovereignty -
whether in its legal or political sense is not clear - and the theory that any country with foreign bases in its
territory cannot claim to be fully sovereign or completely independent. I was not aware that the concepts of
sovereignty and independence have now assumed the totality principle, such that a willing assumption of some
delimitations in the exercise of some aspects thereof would put that State in a lower bracket of nationhood.

xxxx

We have been receiving a continuous influx of materials on the pros and cons on the advisability of having military bases
within our shores. Most of us who, only about three months ago, were just mulling the prospects of these varying
contentions are now expected, like armchair generals, to decide not only on the geopolitical aspects and contingent
implications of the military bases but also on their political, social, economic and cultural impact on our national life. We
are asked to answer a plethora of questions, such as: 1) whether the bases are magnets of nuclear attack or are
deterrents to such attack; 2) whether an alliance or mutual defense treaty is a derogation of our national sovereignty; 3)
whether criticism of us by Russia, Vietnam and North Korea is outweighed by the support for us of the ASEAN countries,
the United States, South Korea, Taiwan, Australia and New Zealand; and 4) whether the social, moral and legal problems
spawned by the military bases and their operations can be compensated by the economic benefits outlined in papers
which have been furnished recently to all of us. 335

xxxx

Of course, one side of persuasion has submitted categorical, unequivocal and forceful assertions of their positions. They
are entitled to the luxury of the absolutes. We are urged now to adopt the proposed declaration as a "golden,"
"unique" and "last" opportunity for Filipinos to assert their sovereign rights. Unfortunately, I have never been
enchanted by superlatives, much less for the applause of the moment or the ovation of the hour. Nor do I look forward to
any glorious summer after a winter of political discontent. Hence, if I may join Commissioner Laurel, I also invoke a caveat
not only against the tyranny of labels but also the tyranny of slogans.336

xxxx

SPEECH OF COMMISSIONER SUAREZ 337

MR. SUAREZ: Thank you, Madam President.

I am quite satisfied that the crucial issues involved in the resolution of the problem of the removal of foreign bases from the
Philippines have been adequately treated by previous speakers. Let me, therefore, just recapitulate the arguments
adduced in favor of a foreign bases-free Philippines:

1. That every nation should be free to shape its own destiny without outside interference;

2. That no lasting peace and no true sovereignty would ever be achieved so long as there are foreign military
forces in our country;

3. That the presence of foreign military bases deprives us of the very substance of national sovereignty and
this is a constant source of national embarrassment and an insult to our national dignity and selfrespect as a
nation;

4. That these foreign military bases unnecessarily expose our country to devastating nuclear attacks;

5. That these foreign military bases create social problems and are designed to perpetuate the strangle-hold of
United States interests in our national economy and development;

6. That the extraterritorial rights enjoyed by these foreign bases operate to deprive our country of jurisdiction
over civil and criminal offenses committed within our own national territory and against Filipinos;

7. That the bases agreements are colonial impositions and dictations upon our helpless country; and

8. That on the legal viewpoint and in the ultimate analysis, all the bases agreements are null and void ab
initio, especially because they did not count the sovereign consent and will of the Filipino people.338

xxxx

In the real sense, Madam President, if we in the Commission could accommodate the provisions I have cited, what is our
objection to include in our Constitution a matter as priceless as the nationalist values we cherish? A matter of the gravest
concern for the safety and survival of this nation indeed deserves a place in our Constitution.

xxxx

x x x Why should we bargain away our dignity and our self-respect as a nation and the future of generations to come
with thirty pieces of silver?
339

SPEECH OF COMMISSIONER BENNAGEN 340

xxxx

The underlying principle of military bases and nuclear weapons wherever they are found and whoever owns them is
that those are for killing people or for terrorizing humanity. This objective by itself at any point in history is morally
repugnant. This alone is reason enough for us to constitutionalize the ban on foreign military bases and on nuclear
weapons. 341

SPEECH OF COMMISSIONER BACANI 342


xxxx

x x x Hence, the remedy to prostitution does not seem to be primarily to remove the bases because even if the
bases are removed, the girls mired in poverty will look for their clientele elsewhere. The remedy to the problem of
prostitution lies primarily elsewhere - in an alert and concerned citizenry, a healthy economy and a sound education in
values. 343

SPEECH OF COMMISSIONER JAMIR 344

xxxx

One of the reasons advanced against the maintenance of foreign military bases here is that they impair portions
of our sovereignty. While I agree that our country's sovereignty should not be impaired, I also hold the view that there
are times when it is necessary to do so according to the imperatives of national interest. There are precedents to this
effect. Thus, during World War II, England leased its bases in the West Indies and in Bermuda for 99 years to the United
States for its use as naval and air bases. It was done in consideration of 50 overaged destroyers which the United States
gave to England for its use in the Battle of the Atlantic.

A few years ago, England gave the Island of Diego Garcia to the United States for the latter's use as a naval base in the
Indian Ocean. About the same time, the United States obtained bases in Spain, Egypt and Israel. In doing so, these
countries, in effect, contributed to the launching of a preventive defense posture against possible trouble in the Middle
East and in the Indian Ocean for their own protection. 345

SPEECH OF COMMISSIONER TINGSON 346

xxxx

In the case of the Philippines and the other Southeast Asian nations, the presence of American troops in the country is a
projection of America's security interest. Enrile said that nonetheless, they also serve, although in an incidental and
secondary way, the security interest of the Republic of the Philippines and the region. Yes, of course, Mr. Enrile also
echoes the sentiments of most of us in this Commission, namely: It is ideal for us as an independent and sovereign
nation to ultimately abrogate the RP-US military treaty and, at the right time, build our own air and naval might. 347

xxxx

Allow me to say in summation that I am for the retention of American military bases in the Philippines provided
that such an extension from one period to another shall be concluded upon concurrence of the parties, and such
extension shall be based on justice, the historical amity of the people of the Philippines and the United States and
their common defense interest. 348

SPEECH OF COMMISSIONER ALONTO 349

xxxx

Madam President, sometime ago after this Commission started with this task of framing a constitution, I read a statement
of President Aquino to the effect that she is for the removal of the U.S. military bases in this country but that the removal of
the U.S. military bases should not be done just to give way to other foreign bases. Today, there are two world
superpowers, both vying to control any and all countries which have importance to their strategy for world domination. The
Philippines is one such country.

Madam President, I submit that I am one of those ready to completely remove any vestiges of the days of
enslavement, but not prepared to erase them if to do so would merely leave a vacuum to be occupied by a far worse
type.350

SPEECH OF COMMISSIONER GASCON 351

xxxx

Let us consider the situation of peace in our world today. Consider our brethren in the Middle East, in Indo-China, Central
America, in South Africa - there has been escalation of war in some of these areas because of foreign intervention which
views these conflicts through the narrow prism of the East-West conflict. The United States bases have been used as
springboards for intervention in some of these conflicts. We should not allow ourselves to be party to the warlike
mentality of these foreign interventionists. We must always be on the side of peace – this means that we should not
always rely on military solution.
352

xxxx

x x x The United States bases, therefore, are springboards for intervention in our own internal affairs and in the
affairs of other nations in this region.
xxxx

Thus, I firmly believe that a self-respecting nation should safeguard its fundamental freedoms which should logically be
declared in black and white in our fundamental law of the land - the Constitution. Let us express our desire for national
sovereignty so we may be able to achieve national self-determination. Let us express our desire for neutrality so that
we may be able to follow active nonaligned independent foreign policies. Let us express our desire for peace and a
nuclear-free zone so we may be able to pursue a healthy and tranquil existence, to have peace that is autonomous and
not imposed.  353

xxxx

SPEECH OF COMMISSIONER TADEO 354

Para sa magbubukid, ano ha ang kahulugan ng U.S. military bases? Para sa magbubukid, ang kahulugan nito ay
pagkaalipin. Para sa magbubukid, ang pananatili ng U.S. military bases ay tinik sa dibdib ng sambayanang
Pilipinong patuloy na nakabaon. Para sa sambayanang magbubukid, ang ibig sabihin ng U.S. military bases ay batong
pabigat na patuloy na pinapasan ng sambayanang Pilipino. Para sa sambayanang magbubukid, ang pananatili
ng U.S. military bases ay isang nagdudumilat na katotohanan ng patuloy na paggahasa ng imperyalistang Estados
Unidos sa ating Inang Bayan - economically, politically and culturally. Para sa sambayanang magbubukid ang U.S.
military bases ay kasingkahulugan ng nuclear weapon - ang kahulugan ay magneto ng isang nuclear war. Para sa
sambayanang magbubukid, ang kahulugan ng U.S. military bases ay isang salot. 355

SPEECH OF COMMISSIONER QUESADA 356

xxxx

The drift in the voting on issues related to freeing ourselves from the instruments of domination and
subservience has clearly been defined these past weeks.

xxxx

So for the record, Mr. Presiding Officer, I would like to declare my support for the committee's position to enshrine in the
Constitution a fundamental principle forbidding foreign military bases, troops or facilities in any part of the Philippine
territory as a clear and concrete manifestation of our inherent right to national self-determination, independence
and sovereignty.

Mr. Presiding Officer, I would like to relate now these attributes of genuine nationhood to the social cost of allowing foreign
countries to maintain military bases in our country. Previous speakers have dwelt on this subject, either to highlight its
importance in relation to the other issues or to gloss over its significance and !llake this a part of future negotiations.
357

xxxx

Mr. Presiding Officer, I feel that banning foreign military bases is one of the solutions and is the response of the Filipino
people against this condition and other conditions that have already been clearly and emphatically discussed in past
deliberations. The deletion, therefore, of Section 3 in the Constitution we are drafting will have the following implications:

First, the failure of the Constitutional Commission to decisively respond to the continuing violation of our territorial
integrity via the military bases agreement which permits the retention of U.S. facilities within the Philippine soil
over which our authorities have no exclusive jurisdiction contrary to the accepted definition of the exercise of
sovereignty.

Second, consent by this forum, this Constitutional Commission, to an exception in the application of a provision in the
Bill of Rights that we have just drafted regarding equal application of the laws of the land to all inhabitants, permanent or
otherwise, within its territorial boundaries.

Third, the continued exercise by the United States of extraterritoriality despite the condemnations of such practice by
the world community of nations in the light of overwhelming international approval of eradicating all vestiges of
colonialism.358

xxxx

Sixth, the deification of a new concept called pragmatic sovereignty, in the hope that such can be wielded to force the
United States government to concede to better terms and conditions concerning the military bases agreement, including
the transfer of complete control to the Philippine government of the U.S. facilities, while in the meantime we have to
suffer all existing indignities and disrespect towards our rights as a sovereign nation.

xxxx

Eighth, the utter failure of this forum to view the issue of foreign military bases as essentially a question of
sovereignty which does not require in-depth studies or analyses and which this forum has, as a constituent assembly
drafting a constitution, the expertise and capacity to decide on except that it lacks the political will that brought it to
existence and now engages in an elaborate scheme of buck-passing.

xxxx

Without any doubt we can establish a new social order in our country, if we reclaim, restore, uphold and defend our
national sovereignty. National sovereignty is what the military bases issue is all about. It is only the sovereign people
exercising their national sovereignty who can design an independent course and take full control of their national destiny. 359

SPEECH OF COMMISSIONER P ADILLA 360

xxxx

Mr. Presiding Officer, in advocating the majority committee report, specifically Sections 3 and 4 on neutrality, nuclear and
bases-free country, some views stress sovereignty of the Republic and even invoke survival of the Filipino nation
and people. 361

REBUTTAL OF COMMISSIONER NOLLEDO 362

xxxx

The anachronistic and ephemeral arguments against the provisions of the committee report to dismantle the American
bases after 1991 only show the urgent need to free our country from the entangling alliance with any power bloc. 363

xxxx

xx x Mr. Presiding Officer, it is not necessary for us to possess expertise to know that the so-called RP-US Bases
Agreement will expire in 1991, that it infringes on our sovereignty and jurisdiction as well as national dignity and
honor, that it goes against the UN policy of disarmament and that it constitutes unjust intervention in our internal
affairs.  (Emphases Supplied)
364

The Constitutional Commission eventually agreed to allow foreign military bases, troops, or facilities, subject to the
provisions of Section 25. It is thus important to read its discussions carefully. From these discussions, we can deduce
three legal standards that were articulated by the Constitutional Commission Members. These are characteristics of any
agreement that the country, and by extension this Court, must ensure are observed. We can thereby determine whether a
military base or facility in the Philippines, which houses or is accessed by foreign military troops, is foreign or remains a
Philippine military base or facility. The legal standards we find applicable are: independence from foreign control,
sovereignty and applicable law, and national security and territorial integrity.

i. First standard: independence from foreign control

Very clearly, much of the opposition to the U.S. bases at the time of the Constitution's drafting was aimed at asserting
Philippine independence from the U.S., as well as control over our country's territory and military.

Under the Civil Code, there are several aspects of control exercised over property.

Property is classified as private or public.  It is public if "intended for public use, such as roads, canals, rivers, torrents,
365

ports and bridges constructed by the State, banks, shores, roadsteads, and others of similar character[,]" or "[t]hose which
belong to the State, without being for public use, and are intended for some public service or for the development of the
national wealth. " 366

Quite clearly, the Agreed Locations are contained within a property for public use, be it within a government military camp
or property that belongs to the Philippines. 1avvphi1

Once ownership is established, then the rights of ownership flow freely. Article 428 of the Civil Code provides that "[t]he
owner has the right to enjoy and dispose of a thing, without other limitations than those established by law." Moreover, the
owner "has also a right of action against the holder and possessor of the thing in order to recover it."

Philippine civil law therefore accords very strong rights to the owner of property, even against those who hold the property.
Possession, after all, merely raises a disputable presumption of ownership, which can be contested through normal
judicial processes. 367

In this case, EDCA explicitly provides that ownership of the Agreed Locations remains with the Philippine
govemment.  What U.S. personnel have a right to, pending mutual agreement, is access to and use of these locations.
368 369

The right of the owner of the property to allow access and use is consistent with the Civil Code, since the owner may
dispose of the property in whatever way deemed fit, subject to the limits of the law. So long as the right of ownership itself
is not transferred, then whatever rights are transmitted by agreement does not completely divest the owner of the rights
over the property, but may only limit them in accordance with law.
Hence, even control over the property is something that an owner may transmit freely. This act does not translate into the
full transfer of ownership, but only of certain rights. In Roman Catholic Apostolic Administrator of Davao, Inc. v. Land
Registration Commission, we stated that the constitutional proscription on property ownership is not violated despite the
foreign national's control over the property.370

EDCA, in respect of its provisions on Agreed Locations, is essentially a contract of use and access. Under its pertinent
provisions, it is the Designated Authority of the Philippines that shall, when requested, assist in facilitating transit or access
to public land and facilities.  The activities carried out within these locations are subject to agreement as authorized by the
371

Philippine govemment.  Granting the U.S. operational control over these locations is likewise subject to EDCA' s security
372

mechanisms, which are bilateral procedures involving Philippine consent and cooperation.  Finally, the Philippine
373

Designated Authority or a duly designated representative is given access to the Agreed Locations. 374

To our mind, these provisions do not raise the spectre of U.S. control, which was so feared by the Constitutional
Commission. In fact, they seem to have been the product of deliberate negotiation from the point of view of the Philippine
government, which balanced constitutional restrictions on foreign military bases and facilities against the security needs of
the country. In the 1947 MBA, the U.S. forces had "the right, power and authority x x x to construct (including dredging and
filling), operate, maintain, utilize, occupy, garrison and control the bases."  No similarly explicit provision is present in
375

EDCA.

Nevertheless, the threshold for allowing the presence of foreign military facilities and bases has been raised by the
present Constitution. Section 25 is explicit that foreign military bases, troops, or facilities shall not be allowed in the
Philippines, except under a treaty duly concurred in by the Senate. Merely stating that the Philippines would retain
ownership would do violence to the constitutional requirement if the Agreed Locations were simply to become a less
obvious manifestation of the U.S. bases that were rejected in 1991.

When debates took place over the military provisions of the Constitution, the committee rejected a specific provision
proposed by Commissioner Sarmiento. The discussion illuminates and provides context to the 1986 Constitutional
Commission's vision of control and independence from the U.S., to wit:

MR. SARMIENTO: Madam President, my proposed amendment reads as follows: "THE STATE SHALL ESTABLISH AND
MAINTAIN AN INDEPENDENT AND SELF-RELIANT ARMED FORCES OF THE PHILIPPINES." Allow me to briefly
explain, Madam President. The Armed Forces of the Philippines is a vital component of Philippine society depending upon
its training, orientation and support. It will either be the people's protector or a staunch supporter of a usurper or tyrant,
local and foreign interest. The Armed Forces of the Philippines' past and recent experience shows it has never been
independent and self-reliant. Facts, data and statistics will show that it has been substantially dependent upon a foreign
power. In March 1968, Congressman Barbero, himself a member of the Armed Forces of the Philippines, revealed top
secret documents showing what he described as U.S. dictation over the affairs of the Armed Forces of the Philippines. He
showed that under existing arrangements, the United States unilaterally determines not only the types and
quantity of arms and equipments that our armed forces would have, but also the time when these items are to be
made available to us. It is clear, as he pointed out, that the composition, capability and schedule of development
of the Armed Forces of the Philippines is under the effective control of the U.S. government.  (Emphases supplied)
376

Commissioner Sarmiento proposed a motherhood statement in the 1987 Constitution that would assert "independent" and
"self-reliant" armed forces. This proposal was rejected by the committee, however. As Commissioner De Castro
asserted, the involvement of the Philippine military with the U.S. did not, by itself, rob the Philippines of its real
independence. He made reference to the context of the times: that the limited resources of the Philippines and the
current insurgency at that time necessitated a strong military relationship with the U.S. He said that the U.S. would not in
any way control the Philippine military despite this relationship and the fact that the former would furnish military hardware
or extend military assistance and training to our military. Rather, he claimed that the proposal was in compliance with the
treaties between the two states.

MR. DE CASTRO: If the Commissioner will take note of my speech on U.S. military bases on 12 September 1986, I spoke
on the selfreliance policy of the armed forces. However, due to very limited resources, the only thing we could do is
manufacture small arms ammunition. We cannot blame the armed forces. We have to blame the whole Republic of the
Philippines for failure to provide the necessary funds to make the Philippine Armed Forces self-reliant. Indeed that is a
beautiful dream. And I would like it that way. But as of this time, fighting an insurgency case, a rebellion in our country -
insurgency - and with very limited funds and very limited number of men, it will be quite impossible for the Philippines to
appropriate the necessary funds therefor. However, if we say that the U.S. government is furnishing us the military
hardware, it is not control of our armed forces or of our government. It is in compliance with the Mutual Defense
Treaty. It is under the military assistance program that it becomes the responsibility of the United States to furnish us the
necessary hardware in connection with the military bases agreement. Please be informed that there are three (3) treaties
connected with the military bases agreement; namely: the RP-US Military Bases Agreement, the Mutual Defense Treaty
and the Military Assistance Program.

My dear Commissioner, when we enter into a treaty and we are furnished the military hardware pursuant to that
treaty, it is not in control of our armed forces nor control of our government. True indeed, we have military officers
trained in the U.S. armed forces school. This is part of our Military Assistance Program, but it does not mean that the
minds of our military officers are for the U.S. government, no. I am one of those who took four courses in the United States
schools, but I assure you, my mind is for the Filipino people. Also, while we are sending military officers to train or to study
in U.S. military schools, we are also sending our officers to study in other military schools such as in Australia, England
and in Paris. So, it does not mean that when we send military officers to United States schools or to other military schools,
we will be under the control of that country. We also have foreign officers in our schools, we in the Command and General
Staff College in Fort Bonifacio and in our National Defense College, also in Fort Bonifacio.  (Emphases supplied)
377

This logic was accepted in Tañada v. Angara, in which the Court ruled that independence does not mean the absence of
foreign participation:

Furthermore, the constitutional policy of a "self-reliant and independent national economy" does not necessarily rule out
the entry of foreign investments, goods and services. It contemplates neither "economic seclusion" nor "mendicancy
in the international community." As explained by Constitutional Commissioner Bernardo Villegas, sponsor of this
constitutional policy:

Economic self reliance is a primary objective of a developing country that is keenly aware of overdependence on external
assistance for even its most basic needs. It does not mean autarky or economic seclusion; rather, it means avoiding
mendicancy in the international community. Independence refers to the freedom from undue foreign control of the
national economy, especially in such strategic industries as in the development of natural resources and public
utilities.  (Emphases supplied)
378

The heart of the constitutional restriction on foreign military facilities and bases is therefore the assertion of independence
from the U.S. and other foreign powers, as independence is exhibited by the degree of foreign control exerted over these
areas.  The essence of that independence is self-governance and self-control.  Independence itself is "[t]he state or
1âwphi1
379

condition of being free from dependence, subjection, or control. " 380

Petitioners assert that EDCA provides the U.S. extensive control and authority over Philippine facilities and locations, such
that the agreement effectively violates Section 25 of the 1987 Constitution. 381

Under Article VI(3) of EDCA, U.S. forces are authorized to act as necessary for "operational control and defense." The
term "operational control" has led petitioners to regard U.S. control over the Agreed Locations as unqualified and,
therefore, total.  Petitioners contend that the word "their" refers to the subject "Agreed Locations."
382

This argument misreads the text, which is quoted below:

United States forces are authorized to exercise all rights and authorities within Agreed Locations that are necessary for
their operational control or defense, including taking appropriate measure to protect United States forces and United
States contractors. The United States should coordinate such measures with appropriate authorities of the Philippines.

A basic textual construction would show that the word "their," as understood above, is a possessive pronoun for the
subject "they," a third-person personal pronoun in plural form. Thus, "their" cannot be used for a non-personal subject
such as "Agreed Locations." The simple grammatical conclusion is that "their" refers to the previous third-person plural
noun, which is "United States forces." This conclusion is in line with the definition of operational control.

a. U.S. operational control as the exercise of authority over U.S. personnel, and not over the Agreed Locations

Operational control, as cited by both petitioner and respondents, is a military term referring to

[t]he authority to perform those functions of command over subordinate forces involving organizing and employing
commands and forces, assigning tasks, designating objective, and giving authoritative direction necessary to accomplish
the mission. 383

At times, though, operational control can mean something slightly different. In JUSMAG Philippines v. National Labor
Relations Commission, the Memorandum of Agreement between the AFP and JUSMAG Philippines defined the term as
follows: 384

The term "Operational Control" includes, but is not limited to, all personnel administrative actions, such as: hiring
recommendations; firing recommendations; position classification; discipline; nomination and approval of incentive awards;
and payroll computation.

Clearly, traditional standards define "operational control" as personnel control. Philippine law, for instance, deems
operational control as one exercised by police officers and civilian authorities over their subordinates and is distinct from
the administrative control that they also exercise over police subordinates.  Similarly, a municipal mayor exercises
385

operational control over the police within the municipal government,  just as city mayor possesses the same power over
386

the police within the city government.387

Thus, the legal concept of operational control involves authority over personnel in a commander-subordinate relationship
and does not include control over the Agreed Locations in this particular case. Though not necessarily stated in EDCA
provisions, this interpretation is readily implied by the reference to the taking of "appropriate measures to protect United
States forces and United States contractors."

It is but logical, even necessary, for the U.S. to have operational control over its own forces, in much the same way that
the Philippines exercises operational control over its own units.
For actual operations, EDCA is clear that any activity must be planned and pre-approved by the MDB-SEB.  This388

provision evinces the partnership aspect of EDCA, such that both stakeholders have a say on how its provisions should be
put into effect.

b. Operational control vis-à-vis effective command and control

Petitioners assert that beyond the concept of operational control over personnel, qualifying access to the Agreed
Locations by the Philippine Designated Authority with the phrase "consistent with operational safety and security
requirements in accordance with agreed procedures developed by the Parties" leads to the conclusion that the U.S.
exercises effective control over the Agreed Locations.  They claim that if the Philippines exercises possession of and
389

control over a given area, its representative should not have to be authorized by a special provision.
390

For these reasons, petitioners argue that the "operational control" in EDCA is the "effective command and control" in the
1947 MBA.  In their Memorandum, they distinguish effective command and control from operational control in U.S.
391

parlance.  Citing the Doctrine for the Armed Forces of the United States, Joint Publication 1, "command and control (C2)"
392

is defined as "the exercise of authority and direction by a properly designated commander over assigned and attached
forces in the accomplishment of the mission x x x."  Operational control, on the other hand, refers to "[t]hose functions of
393

command over assigned forces involving the composition of subordinate forces, the assignment of tasks, the designation
of objectives, the overall control of assigned resources, and the full authoritative direction necessary to accomplish the
mission."394

Two things demonstrate the errors in petitioners' line of argument.

Firstly, the phrase "consistent with operational safety and security requirements in accordance with agreed procedures
developed by the Parties" does not add any qualification beyond that which is already imposed by existing treaties. To
recall, EDCA is based upon prior treaties, namely the VFA and the MDT.  Treaties are in themselves contracts from
395

which rights and obligations may be claimed or waived.  In this particular case, the Philippines has already agreed to
396

abide by the security mechanisms that have long been in place between the U.S. and the Philippines based on the
implementation of their treaty relations. 397

Secondly, the full document cited by petitioners contradicts the equation of "operational control" with "effective command
and control," since it defines the terms quite differently, viz: 398

Command and control encompasses the exercise of authority, responsibility, and direction by a commander over assigned
and attached forces to accomplish the mission. Command at all levels is the art of motivating and directing people and
organizations into action to accomplish missions. Control is inherent in command. To control is to manage and direct
forces and functions consistent with a commander's command authority. Control of forces and functions helps
commanders and staffs compute requirements, allocate means, and integrate efforts. Mission command is the preferred
method of exercising C2. A complete discussion of tenets, organization, and processes for effective C2 is provided in
Section B, "Command and Control of Joint Forces," of Chapter V "Joint Command and Control."

Operational control is defined thus: 399

OPCON is able to be delegated from a lesser authority than COCOM. It is the authority to perform those functions of
command over subordinate forces involving organizing and employing commands and forces, assigning tasks, designating
objectives, and giving authoritative direction over all aspects of military operations and joint training necessary to
accomplish the mission. It should be delegated to and exercised by the commanders of subordinate organizations;
normally, this authority is exercised through subordinate JFCs, Service, and/or functional component commanders.
OPCON provides authority to organize and employ commands and forces as the commander considers necessary to
accomplish assigned missions. It does not include authoritative direction for logistics or matters of administration,
discipline, internal organization, or unit training. These elements of COCOM must be specifically delegated by the CCDR.
OPCON does include the authority to delineate functional responsibilities and operational areas of subordinate JFCs.

Operational control is therefore the delegable aspect of combatant command, while command and control is the overall
power and responsibility exercised by the commander with reference to a mission. Operational control is a narrower power
and must be given, while command and control is plenary and vested in a commander. Operational control does not
include the planning, programming, budgeting, and execution process input; the assignment of subordinate commanders;
the building of relationships with Department of Defense agencies; or the directive authority for logistics, whereas these
factors are included in the concept of command and control. 400

This distinction, found in the same document cited by petitioners, destroys the very foundation of the arguments they have
built: that EDCA is the same as the MBA.

c. Limited operational control over the Agreed Locations only for construction activitites

As petitioners assert, EDCA indeed contains a specific provision that gives to the U.S. operational control within the
Agreed Locations during construction activities.  This exercise of operational control is premised upon the approval by the
401

MDB and the SEB of the construction activity through consultation and mutual agreement on the requirements and
standards of the construction, alteration, or improvement. 402
Despite this grant of operational control to the U.S., it must be emphasized that the grant is only for construction activities.
The narrow and limited instance wherein the U.S. is given operational control within an Agreed Location cannot be
equated with foreign military control, which is so abhorred by the Constitution.

The clear import of the provision is that in the absence of construction activities, operational control over the Agreed
Location is vested in the Philippine authorities. This meaning is implicit in the specific grant of operational control only
during construction activities. The principle of constitutional construction, "expressio unius est exclusio alterius," means
the failure to mention the thing becomes the ground for inferring that it was deliberately excluded.  Following this
403

construction, since EDCA mentions the existence of U.S. operational control over the Agreed Locations for construction
activities, then it is quite logical to conclude that it is not exercised over other activities.

Limited control does not violate the Constitution. The fear of the commissioners was total control, to the point that the
foreign military forces might dictate the terms of their acts within the Philippines.  More important, limited control does not
404

mean an abdication or derogation of Philippine sovereignty and legal jurisdiction over the Agreed Locations. It is more akin
to the extension of diplomatic courtesies and rights to diplomatic agents,  which is a waiver of control on a limited scale
405

and subject to the terms of the treaty.

This point leads us to the second standard envisioned by the framers of the Constitution: that the Philippines must retain
sovereignty and jurisdiction over its territory.

ii. Second standard: Philippine sovereignty and applicable law

EDCA states in its Preamble the "understanding for the United States not to establish a permanent military presence or
base in the territory of the Philippines." Further on, it likewise states the recognition that "all United States access to and
use of facilities and areas will be at the invitation of the Philippines and with full respect for the Philippine Constitution and
Philippine laws."

The sensitivity of EDCA provisions to the laws of the Philippines must be seen in light of Philippine sovereignty and
jurisdiction over the Agreed Locations.

Sovereignty is the possession of sovereign power,  while jurisdiction is the conferment by law of power and authority to
406

apply the law.  Article I of the 1987 Constitution states:


407

The national territory comprises the Philippine archipelago, with all the islands and waters embraced therein, and all other
territories over which the Philippines has sovereignty or jurisdiction, consisting of its terrestrial, fluvial, and aerial
domains, including its territorial sea, the seabed, the subsoil, the insular shelves, and other submarine areas. The waters
around, between, and connecting the islands of the archipelago, regardless of their breadth and dimensions, form part of
the internal waters of the Philippines. (Emphasis supplied)

From the text of EDCA itself, Agreed Locations are territories of the Philippines that the U.S. forces are allowed to access
and use.  By withholding ownership of these areas and retaining unrestricted access to them, the government asserts
408

sovereignty over its territory. That sovereignty exists so long as the Filipino people exist. 409

Significantly, the Philippines retains primary responsibility for security with respect to the Agreed Locations.  Hence,
410

Philippine law remains in force therein, and it cannot be said that jurisdiction has been transferred to the U.S. Even the
previously discussed necessary measures for operational control and defense over U.S. forces must be coordinated with
Philippine authorities.411

Jurisprudence bears out the fact that even under the former legal regime of the MBA, Philippine laws continue to be in
force within the bases.  The difference between then and now is that EDCA retains the primary jurisdiction of the
412

Philippines over the security of the Agreed Locations, an important provision that gives it actual control over those
locations. Previously, it was the provost marshal of the U.S. who kept the peace and enforced Philippine law in the bases.
In this instance, Philippine forces act as peace officers, in stark contrast to the 1947 MBA provisions on jurisdiction. 413

iii. Third standard: must respect national security and territorial integrity

The last standard this Court must set is that the EDCA provisions on the Agreed Locations must not impair or threaten the
national security and territorial integrity of the Philippines.

This Court acknowledged in Bayan v. Zamora that the evolution of technology has essentially rendered the prior notion of
permanent military bases obsolete.

Moreover, military bases established within the territory of another state is no longer viable because of the alternatives
offered by new means and weapons of warfare such as nuclear weapons, guided missiles as well as huge sea vessels
that can stay afloat in the sea even for months and years without returning to their home country. These military warships
are actually used as substitutes for a land-home base not only of military aircraft but also of military personnel and
facilities. Besides, vessels are mobile as compared to a land-based military headquarters. 414

The VFA serves as the basis for the entry of U.S. troops in a limited scope. It does not allow, for instance, the re-
establishment of the Subic military base or the Clark Air Field as U.S. military reservations. In this context, therefore, this
Court has interpreted the restrictions on foreign bases, troops, or facilities as three independent restrictions. In accord with
this interpretation, each restriction must have its own qualification.

Petitioners quote from the website http://en.wikipedia.org to define what a military base is.  While the source is not
415

authoritative, petitioners make the point that the Agreed Locations, by granting access and use to U.S. forces and
contractors, are U.S. bases under a different name.  More important, they claim that the Agreed Locations invite
416

instances of attack on the Philippines from enemies of the U.S. 417

We believe that the raised fear of an attack on the Philippines is not in the realm of law, but of politics and policy. At the
very least, we can say that under international law, EDCA does not provide a legal basis for a justified attack on the
Philippines.

In the first place, international law disallows any attack on the Agreed Locations simply because of the presence of U.S.
personnel. Article 2(4) of the United Nations Charter states that "All Members shall refrain in their international relations
from the threat or use of force against the territorial integrity or political independence of any state, or in any other manner
inconsistent with the Purposes of the United Nations."  Any unlawful attack on the Philippines breaches the treaty, and
418

triggers Article 51 of the same charter, which guarantees the inherent right of individual or collective self-defence.

Moreover, even if the lawfulness of the attack were not in question, international humanitarian law standards prevent
participants in an armed conflict from targeting non-participants. International humanitarian law, which is the branch of
international law applicable to armed conflict, expressly limits allowable military conduct exhibited by forces of a participant
in an armed conflict.  Under this legal regime, participants to an armed conflict are held to specific standards of conduct
419

that require them to distinguish between combatants and non-combatants,  as embodied by the Geneva Conventions and
420

their Additional Protocols. 421

Corollary to this point, Professor John Woodcliffe, professor of international law at the University of Leicester, noted that
there is no legal consensus for what constitutes a base, as opposed to other terms such as "facilities" or "installation."  In 422

strategic literature, "base" is defined as an installation "over which the user State has a right to exclusive control in an
extraterritorial sense."  Since this definition would exclude most foreign military installations, a more important distinction
423

must be made.

For Woodcliffe, a type of installation excluded from the definition of "base" is one that does not fulfill a combat role. He
cites an example of the use of the territory of a state for training purposes, such as to obtain experience in local geography
and climactic conditions or to carry out joint exercises.  Another example given is an advanced communications
424

technology installation for purposes of information gathering and communication.  Unsurprisingly, he deems these non-
425

combat uses as borderline situations that would be excluded from the functional understanding of military bases and
installations.426

By virtue of this ambiguity, the laws of war dictate that the status of a building or person is presumed to be protected,
unless proven otherwise.  Moreover, the principle of distinction requires combatants in an armed conflict to distinguish
427

between lawful targets  and protected targets.  In an actual armed conflict between the U.S. and a third state, the Agreed
428 429

Locations cannot be considered U.S. territory, since ownership of territory even in times of armed conflict does not
change. 430

Hence, any armed attack by forces of a third state against an Agreed Location can only be legitimate under international
humanitarian law if it is against a bona fide U.S. military base, facility, or installation that directly contributes to the military
effort of the U.S. Moreover, the third state's forces must take all measures to ensure that they have complied with the
principle of distinction (between combatants and non-combatants).

There is, then, ample legal protection for the Philippines under international law that would ensure its territorial integrity
and national security in the event an Agreed Location is subjected to attack. As EDCA stands, it does not create the
situation so feared by petitioners - one in which the Philippines, while not participating in an armed conflict, would
be legitimately targeted by an enemy of the U.S. 431

In the second place, this is a policy question about the wisdom of allowing the presence of U.S. personnel within our
territory and is therefore outside the scope of judicial review.

Evidently, the concept of giving foreign troops access to "agreed" locations, areas, or facilities within the military base of
another sovereign state is nothing new on the international plane. In fact, this arrangement has been used as the
framework for several defense cooperation agreements, such as in the following:

1. 2006 U.S.-Bulgaria Defense Cooperation Agreement 432

2. 2009 U.S.-Colombia Defense Cooperation Agreement 433

3. 2009 U.S.-Poland Status of Forces Agreement 434

4. 2014 U.S.-Australia Force Posture Agreement 435

5. 2014 U.S.-Afghanistan Security and Defense Cooperation Agreement 436


In all of these arrangements, the host state grants U.S. forces access to their military bases.  That access is without
437

rental or similar costs to the U.S.  Further, U.S. forces are allowed to undertake construction activities in, and make
438

alterations and improvements to, the agreed locations, facilities, or areas.  As in EDCA, the host states retain ownership
439

and jurisdiction over the said bases. 440

In fact, some of the host states in these agreements give specific military-related rights to the U.S. For example, under
Article IV(l) of the US.-Bulgaria Defense Cooperation Agreement, "the United States forces x x x are authorized access to
and may use agreed facilities and areas x x x for staging and deploying of forces and materiel, with the purpose of
conducting x x x contingency operations and other missions, including those undertaken in the framework of the North
Atlantic Treaty." In some of these agreements, host countries allow U.S. forces to construct facilities for the latter’s
exclusive use. 441

Troop billeting, including construction of temporary structures, is nothing new. In Lim v. Executive Secretary, the Court
already upheld the Terms of Reference of Balikatan 02-1, which authorized U.S. forces to set up "[t]emporary structures
such as those for troop billeting, classroom instruction and messing x x x during the Exercise." Similar provisions are also
in the Mutual Logistics Support Agreement of 2002 and 2007, which are essentially executive agreements that implement
the VFA, the MDT, and the 1953 Military Assistance Agreement. These executive agreements similarly tackle the
"reciprocal provision of logistic support, supplies, and services,"  which include "[b ]illeting, x x x operations support (and
442

construction and use of temporary structures incident to operations support), training services, x x x storage services, x x x
during an approved activity."  These logistic supplies, support, and services include temporary use of "nonlethal items of
443

military equipment which are not designated as significant military equipment on the U.S. Munitions List, during an
approved activity."  The first Mutual Logistics Support Agreement has lapsed, while the second one has been extended
444

until 2017 without any formal objection before this Court from the Senate or any of its members.

The provisions in EDCA dealing with Agreed Locations are analogous to those in the aforementioned executive
agreements. Instead of authorizing the building of temporary structures as previous agreements have done, EDCA
authorizes the U.S. to build permanent structures or alter or improve existing ones for, and to be owned by, the
Philippines.  EDCA is clear that the Philippines retains ownership of altered or improved facilities and newly constructed
445

permanent or non-relocatable structures.  Under EDCA, U.S. forces will also be allowed to use facilities and areas for
446

"training; x x x; support and related activities; x x x; temporary accommodation of personnel; communications" and agreed
activities.
447

Concerns on national security problems that arise from foreign military equipment being present in the Philippines must
likewise be contextualized. Most significantly, the VFA already authorizes the presence of U.S. military equipment in
the country. Article VII of the VFA already authorizes the U.S. to import into or acquire in the Philippines "equipment,
materials, supplies, and other property" that will be used "in connection with activities" contemplated therein. The same
section also recognizes that "[t]itle to such property shall remain" with the US and that they have the discretion to "remove
such property from the Philippines at any time."

There is nothing novel, either, in the EDCA provision on the prepositioning and storing of "defense equipment, supplies,
and materiel,"  since these are sanctioned in the VFA. In fact, the two countries have already entered into various
448

implementing agreements in the past that are comparable to the present one. The Balikatan 02-1 Terms of Reference
mentioned in Lim v. Executive Secretary specifically recognizes that Philippine and U.S. forces "may share x x x in the use
of their resources, equipment and other assets." Both the 2002 and 2007 Mutual Logistics Support Agreements speak of
the provision of support and services, including the "construction and use of temporary structures incident to operations
support" and "storage services" during approved activities.  These logistic supplies, support, and services include the
449

"temporary use of x x x nonlethal items of military equipment which are not designated as significant military equipment on
the U.S. Munitions List, during an approved activity."  Those activities include "combined exercises and training,
450

operations and other deployments" and "cooperative efforts, such as humanitarian assistance, disaster relief and rescue
operations, and maritime anti-pollution operations" within or outside Philippine territory.  Under EDCA, the equipment,
451

supplies, and materiel that will be prepositioned at Agreed Locations include "humanitarian assistance and disaster relief
equipment, supplies, and materiel. "  Nuclear weapons are specifically excluded from the materiel that will be
452

prepositioned.

Therefore, there is no basis to invalidate EDCA on fears that it increases the threat to our national security. If anything,
EDCA increases the likelihood that, in an event requiring a defensive response, the Philippines will be prepared alongside
the U.S. to defend its islands and insure its territorial integrity pursuant to a relationship built on the MDT and VFA.

8. Others issues and concerns raised

A point was raised during the oral arguments that the language of the MDT only refers to mutual help and defense in the
Pacific area.  We believe that any discussion of the activities to be undertaken under EDCA vis-a-vis the defense of areas
453

beyond the Pacific is premature. We note that a proper petition on that issue must be filed before we rule thereon. We also
note that none of the petitions or memoranda has attempted to discuss this issue, except only to theorize that the U.S. will
not come to our aid in the event of an attack outside of the Pacific. This is a matter of policy and is beyond the scope of
this judicial review.

In reference to the issue on telecommunications, suffice it to say that the initial impression of the facility adverted to does
appear to be one of those that require a public franchise by way of congressional action under Section 11, Article XII of the
Constitution. As respondents submit, however, the system referred to in the agreement does not provide
telecommunications services to the public for compensation.  It is clear from Article VIl(2) of EDCA that the
454
telecommunication system is solely for the use of the U.S. and not the public in general, and that this system will not
interfere with that which local operators use. Consequently, a public franchise is no longer necessary.

Additionally, the charge that EDCA allows nuclear weapons within Philippine territory is entirely speculative. It is
noteworthy that the agreement in fact specifies that the prepositioned materiel shall not include nuclear
weapons.  Petitioners argue that only prepositioned nuclear weapons are prohibited by EDCA; and that, therefore, the
455

U.S. would insidiously bring nuclear weapons to Philippine territory.  The general prohibition on nuclear weapons,
456

whether prepositioned or not, is already expressed in the 1987 Constitution.  It would be unnecessary or superfluous to
457

include all prohibitions already in the Constitution or in the law through a document like EDCA.

Finally, petitioners allege that EDCA creates a tax exemption, which under the law must originate from Congress. This
allegation ignores jurisprudence on the government's assumption of tax liability. EDCA simply states that the taxes on the
use of water, electricity, and public utilities are for the account of the Philippine Government.  This provision creates a
458

situation in which a contracting party assumes the tax liability of the other.  In National Power Corporation v. Province of
459

Quezon, we distinguished between enforceable and unenforceable stipulations on the assumption of tax liability.
Afterwards, we concluded that an enforceable assumption of tax liability requires the party assuming the liability to have
actual interest in the property taxed.  This rule applies to EDCA, since the Philippine Government stands to benefit not
460

only from the structures to be built thereon or improved, but also from the joint training with U.S. forces, disaster
preparation, and the preferential use of Philippine suppliers.  Hence, the provision on the assumption of tax liability does
461

not constitute a tax exemption as petitioners have posited.

Additional issues were raised by petitioners, all relating principally to provisions already sufficiently addressed above. This
Court takes this occasion to emphasize that the agreement has been construed herein as to absolutely disauthorize the
violation of the Constitution or any applicable statute. On the contrary, the applicability of Philippine law is explicit in
EDCA.

EPILOGUE

The fear that EDCA is a reincarnation of the U.S. bases so zealously protested by noted personalities in Philippine history
arises not so much from xenophobia, but from a genuine desire for self-determination, nationalism, and above all a
commitment to ensure the independence of the Philippine Republic from any foreign domination.

Mere fears, however, cannot curtail the exercise by the President of the Philippines of his Constitutional prerogatives in
respect of foreign affairs. They cannot cripple him when he deems that additional security measures are made necessary
by the times. As it stands, the Philippines through the Department of Foreign Affairs has filed several diplomatic protests
against the actions of the People's Republic of China in the West Philippine Sea;  initiated arbitration against that country
462

under the United Nations Convention on the Law of the Sea;  is in the process of negotiations with the Moro Islamic
463

Liberation Front for peace in Southern Philippines,  which is the subject of a current case before this Court; and faces
464

increasing incidents of kidnappings of Filipinos and foreigners allegedly by the Abu Sayyaf or the New People's
Army.  The Philippine military is conducting reforms that seek to ensure the security and safety of the nation in the years
465

to come.  In the future, the Philippines must navigate a world in which armed forces fight with increasing sophistication in
466

both strategy and technology, while employing asymmetric warfare and remote weapons.

Additionally, our country is fighting a most terrifying enemy: the backlash of Mother Nature. The Philippines is one of the
countries most directly affected and damaged by climate change. It is no coincidence that the record-setting tropical
cyclone Yolanda (internationally named Haiyan), one of the most devastating forces of nature the world has ever seen hit
the Philippines on 8 November 2013 and killed at least 6,000 people.  This necessitated a massive rehabilitation
467

project.  In the aftermath, the U.S. military was among the first to extend help and support to the Philippines.
468

That calamity brought out the best in the Filipinos as thousands upon thousands volunteered their help, their wealth, and
their prayers to those affected. It also brought to the fore the value of having friends in the international community.

In order to keep the peace in its archipelago in this region of the world, and to sustain itself at the same time against the
destructive forces of nature, the Philippines will need friends. Who they are, and what form the friendships will take, are for
the President to decide. The only restriction is what the Constitution itself expressly prohibits. It appears that this
overarching concern for balancing constitutional requirements against the dictates of necessity was what led to EDCA.

As it is, EDCA is not constitutionally infirm. As an executive agreement, it remains consistent with existing laws and
treaties that it purports to implement.

WHEREFORE, we hereby DISMISS the petitions.

SO ORDERED

-Mitsubishi Corp. v BIR, GR 175772, Jun 5, 2017 (Exchange of notes)

Assailed in this petition for review on certiorari   are the Decision     . dated May 24, 2006 and the Resolution dated
1 2 3

December 4, 2006 of the Court of Tax Appeals (CTA) En Banc in C.T.A. EB No. 5, reversing the CTA Division's ruling   in 4

CTA Case No. 6139 which granted the claim for refund of erroneously paid income tax and branch profit remittance tax
(BPRT; collectively, subject taxes) filed by petitioner Mitsubishi Corporation - Manila Branch (petitioner) for the fiscal year
that ended on March 31, 1998.

The Facts

On June 11, 1987, the governments of Japan and the Philippines executed an Exchange of Notes,   whereby the former
5

agreed to extend a loan amounting to Forty Billion Four Hundred Million Japanese Yen (¥40,400,000,000) to the latter
through the then Overseas Economic Cooperation Fund (OECF, now Japan Bank for International Cooperation) for the
implementation of the Calaca II Coal-Fired Thermal Power Plant Project (Project).   In Paragraph 5 (2) of the Exchange of
6

Notes, the Philippine Government, by itself or through its executing agency, undertook to assume all taxes imposed by the
Philippines on Japanese contractors engaged in the Project:

(2) The Government of the Republic of the Philippines will, itself or through its executing agencies or instrumentalities,
assume all fiscal levies or taxes imposed in the Republic of the Philippines on Japanese firms and nationals operating as
suppliers, contractors or consultants on and/or in connection with any income that may accrue from the supply of products
of Japan and services of Japanese nationals to be provided under the Loan.   (Emphases, underscoring, and italics
7

supplied)

Consequently, the OECF and the Philippine Government entered into Loan Agreement No. PH-P76   dated September 25, 8

1987 for Forty Billion Four Hundred Million Japanese Yen (¥40,400,000,000). Due to the need for additional funding for
the Project, they also executed Loan Agreement No. PH-P141   dated December 20, 1994 for Five Billion Five Hundred
9

Thirteen Million Japanese Yen (¥5,513,000,000).  10

Meanwhile, on June 21, 1991, the National Power Corporation (NPC), as the executing government agency, entered into
a contract with Mitsubishi Corporation (i.e., petitioner's head office in Japan) for the engineering, supply, construction,
installation, testing, and commissioning of a steam generator, auxiliaries, and associated civil works for the Project
(Contract).   The Contract's foreign currency portion was funded by the OECF loans.   In line with the Exchange of Notes,
11 12

Article VIII (B) (1) of the Contract indicated NPC' s undertaking to pay any and all forms of taxes that are directly
imposable under the Contract:

Article VIII (B) (1)

B. FOR ONSHORE PORTION.

1.) [The] CORPORATION (NPC) shall, subject to the provisions under the Contract [Document] on Taxes, pay any and all
forms of taxes which are directly imposable under the Contract including VAT, that may be imposed by the Philippine
Government, or any of its agencies and political subdivisions.   (Emphases supplied)
13

Petitioner completed the project on December 2, 1995, but it was only accepted by NPC on January 31, 1998 through a
Certificate of Completion and Final Acceptance.  14

On July 15, 1998, petitioner filed its Income Tax Return for the fiscal year that ended on March 31, 1998 with the Bureau
of Internal Revenue (BIR). Petitioner included in its income tax due   the amount of ₱44,288,712.00, representing income
15

from the OECF-funded portion of the Project.   On the same day, petitioner also filed its Monthly Remittance Return of
16

Income Taxes Withheld and remitted ₱8,324,100.00 as BPRT for branch profits remitted to its head office in Japan out of
its income for the fiscal year that ended on March 31, 1998 . 17

On June 30, 2000, petitioner filed with the respondent Commissioner on Internal Revenue (CIR) an administrative claim
for refund of Fifty Two Million Six Hundred Twelve Thousand, Eight Hundred Twelve Pesos (P52,612,812.00),
representing the erroneously paid amounts of P44,288,712.00 as income tax and ₱8,324,100.00 as BPRT corresponding
to the OECF-funded portion of the Project.   To suspend the running of the two-year period to file a judicial claim for
18

refund, petitioner filed on July 13, 2000 a petition for review   before the CTA pursuant to Section 229 of the National
19

Internal Revenue Code (NIRC), which was docketed as C.T.A. Case No. 6139.   Petitioner anchored its claim for refund
20

on BIR Ruling No. DA-407-98 dated September 7, 1998,   which interpreted paragraph 5 (2) of the Exchange of Notes, to
21

wit:

In reply, please be informed that the aforequoted provisions of Notes-NAIA and Notes-Calaca are not grants of direct tax
exemption privilege to Japanese firms, Mitsubishi in this case, and Japanese nationals operating as suppliers, contractors
or consultants involved in either of the two projects because the said provisions state that it is the Government of the
Republic of the Philippines that is obligated to pay whatever fiscal levies or taxes they may be liable to. Thus, there is no
tax exemption to speak of because the said taxes shall be assumed by the Philippine Government; hence, the said
provision is not violative of the Constitutional prohibition against the grants of tax exemption without the concurrence of the
majority of the members of Congress. (Citation omitted)

In view thereof, x x x, this office is of the opinion and hereby holds that Mitsubishi has no liability for income tax and other
taxes and fiscal levies, including VAT, on the 75% of the NAIA II Project and on the 100% of the foreign currency portion
of the Calaca II Project since the said taxes were assumed by the Philippine Government.22 (Emphases and underscoring
supplied)

In a Decision   dated December 17, 2003, the CTA Division granted the petition and ordered the CIR to refund to
23

petitioner the amounts it erroneously paid as income tax and BPRT.   It held that based on the Exchange of Notes, the
24
Philippine Government, through the NPC as its executing agency, bound itself to assume or shoulder petitioner's tax
obligations. Therefore, petitioner's payments of income tax and BPR T to the CIR, when such payments should have been
made by the NPC, undoubtedly constitute erroneous payments under Section 229 of the NIRC.  25

The CTA Division acknowledged that based on Revenue Memorandum Circular (RMC) No. 42-99 dated June 2, 1999,
amending RMC No. 32-99, the proper remedy for a Japanese contractor who previously paid the taxes directly to the BIR
is to recover or obtain a refund from the government executing agency - the NPC in this case. It held, however, that RMC
No. 42-99 does not apply to petitioner as it filed its ITR on July 15, 1998 or almost a year before the issuance of the same.
It added that RMC No. 42-99 cannot be given retroactive effect as it would be unfair • •   to petitioner.
26

The CIR moved for reconsideration27 but was denied in a Resolution   dated April 23, 2004; thus, the CIR elevated the
28

matter to the CTA En Banc.  29

The CTA En Bane's Ruling

In a Decision   dated May 24, 2006, the CTA En Banc reversed the CTA Division's rulings and declared that petitioner is
30

not entitled to a refund of the taxes it paid to the CIR. It held that, first, petitioner failed to establish that its tax payments
were "erroneous" under the law to justify the refund, adding that the CIR has no power to grant a refund under Section 229
of the NIRC absent any tax exemption. It further observed that by its clear terms, the Exchange of Notes granted no tax
exemption to petitioner.  Second, the Exchange of Notes cannot be read as a treaty validly granting tax exemption
31

considering the lack of Senate concurrence as required under Article VII, Section 21 of the Constitution.  Third, RMC No.
32

42-99, which was already in effect when petitioner filed its administrative claim for refund on June 30, 2000, specifies
petitioner's proper remedy - that is, to recover the subject taxes from NPC, and not from the CIR. 33

Petitioner sought reconsideration,   but the CTA En Banc denied the motion in a Resolution   dated December 4, 2006;
34 35

hence, this petition.

The Issues Before the Court

The issues before the Court are two-fold: (a) whether petitioner is entitled to a refund; and (b) if in the affirmative, from
which government entity should the refund be claimed.

The Court's Ruling

The petition is meritorious.

I.

Sections 204 (C) of the NIRC grants the CIR the authority to credit or refund taxes which are erroneously collected by the
government:  36

SEC.204. Authority of the Commissioner to Compromise, Abate, and Refund or Credit Taxes. The Commissioner may –

x x xx

(C) Credit or refund taxes erroneously or illegally received or penalties imposed without authority, refund the value of
internal revenue stamps when they are returned in good condition by the purchaser, and, in his discretion, redeem or
change unused stamps that have been rendered unfit for use and refund their value upon proof of destruction. No credit or
refund of taxes or penalties shall be allowed unless the taxpayer files in writing with the Commissioner a claim for credit or
refund within two (2) years after the payment of the tax or penalty: Provided, however, That a return filed showing an
overpayment shall be considered as a written claim for credit or refund.

x x xx (Emphases and underscoring supplied)

The authority of the CIR to refund erroneously collected taxes 1s likewise reflected in Section 229 of the NIRC, which
reads: SEC. 229. Recovery of Tax Erroneously or Illegally Collected. - No suit or proceeding shall be maintained in any
court for the recovery of any national internal revenue tax hereafter alleged to have been erroneously or illegally assessed
or collected, or of any penalty claimed to have been collected without authority, or of any sum alleged to have been
excessively or in any manner wrongfully collected, until a claim for refund or credit has been duly filed with the
Commissioner; but such suit or proceeding may be maintained, whether or not such tax, penalty, or sum has been paid
under protest or duress."

x x x x (Emphases and underscoring supplied)

In this case, it is fairly apparent that the subject taxes in the amount of ₱52,612,812.00 was erroneously collected from
petitioner, considering that the obligation to pay the same had already been assumed by the Philippine Government by
virtue of its Exchange of Notes with the Japanese Government. Case law explains that an exchange of notes is
considered as an executive agreement, which is binding on the State even without Senate concurrence. In Abaya v.
Ebdane: 37
An "exchange of notes" is a record of a routine agreement that has many similarities with the private law contract. The
agreement consists of the exchange of two documents, each of the parties being in the possession of the one signed by
the representative of the other. Under the usual procedure, the accepting State repeats the text of the offering State to
record its assent. The signatories of the letters may be government Ministers, diplomats or departmental heads. The
technique of exchange of notes is frequently resorted to, either because of its speedy procedure, or, sometimes, to avoid
the process of legislative approval.

It is stated that "treaties, agreements, conventions, charters, protocols, declarations, memoranda of understanding, modus
vivendi and exchange of notes" all refer to "international instruments binding at international law."

xxxx

Significantly, an exchange of notes is considered a form of an executive agreement, which becomes binding through
executive action without the need of a vote by the Senate or Congress.  38

Paragraph 5 (2) of the Exchange of Notes provides for a tax assumption provision whereby:

(2) The Government of the Republic of the Philippines will, itself or through its executing agencies or instrumentalities,
assume all fiscal levies or taxes imposed in the Republic of the Philippines on Japanese firms and nationals operating as
suppliers, contractors or consultants on and/or in connection with any income that may accrue from the supply of products
of Japan and services of Japanese nationals to be provided under the Loan. (Emphases and underscoring supplied)

To "assume" means "[t]o take on, become bound as another is bound, or put oneself in place of another as to an
obligation or liability."  This means that the obligation or liability remains, although the same is merely passed on to a
39

different person. In this light, the concept of an assumption is therefore different from an exemption, the latter being the
"[f]reedom from a duty, liability or other requirement" or "[a] privilege given to a judgment debtor by law, allowing the
debtor to retain [a] certain property without liability."  Thus, contrary to the CTA En Bane's opinion, the constitutional
40

provisions on tax exemptions would not apply.

As explicitly worded, the Philippine Government, through its executing agencies (i.e., NPC in this case) particularly
assumed "all fiscal levies or taxes imposed in the Republic of the Philippines on Japanese firms and nationals operating
as suppliers, contractors or consultants on and/or in connection with any income that may accrue from the supply of
products of Japan and services of Japanese nationals to be provided under the [OECF] Loan." The Philippine
Government's assumption of "all fiscal levies and taxes," which includes the subject taxes, is clearly a form of concession
given to Japanese suppliers, contractors or consultants in consideration of the OECF Loan, which proceeds were used for
the implementation of the Project. As part of this, NPC entered into the June 21, 1991 Contract with Mitsubishi
Corporation (i.e., petitioner's head office in Japan) for the engineering, supply, construction, installation, testing, and
commissioning of a steam generator, auxiliaries, and associated civil works for the Project,  which foreign currency portion
41

was funded by the OECF loans.   Thus, in line with the tax assumption provision under the Exchange of Notes, Article VIII
42

(B) (1) of the Contract states that NPC shall pay any and all forms of taxes that are directly imposable under the Contract:

Article VIII (B) (1)

B. FOR ONSHORE PORTION.

1.) [The] CORPORATION (NPC) shall, subject to the provisions under the Contract [Document] on Taxes, pay any and all
forms of taxes which are directly imposable under the Contract including VAT, that may be imposed by the Philippine
Government, or any of its agencies and political subdivisions.  (Emphases supplied)
43

This notwithstanding, petitioner included in its income tax due the amount of ₱44,288,712.00, representing income from
the OECF-funded portion of the Project, and further remitted ₱8,324,100.00 as BPRT for branch profits remitted to its
head office in Japan out of its income for the fiscal year that ended on March 31, 1998.  These taxes clearly fall within the
45

ambit of the tax assumption provision under the Exchange of Notes, which was further fleshed out in the Contract. Hence,
it is the Philippine Government, through the NPC, which should shoulder the payment of the same.

It bears stressing that the CIR had already acknowledged, through its administrative issuances, that Japanese contractors
involved in the Project are not liable for the subject taxes. In RMC No. 42-99, the CIR interpreted the effect of the tax
assumption clause in the Exchange of Notes on petitioner's tax liability, to wit:

The foregoing provisions of the Exchange of Notes mean that the Japanese contractors or nationals engaged in EOCF-
funded projects in the Philippines shall not be required to shoulder all fiscal levies or taxes associated with the project. x x
xx

x x xx

x x x Since the executing government agencies are mandated to assume the payment of [income taxes] under the
Exchange of Notes, the said Japanese firms or nationals need not pay taxes due thereunder.  (Emphases and
46

underscoring supplied)

The CIR subsequently affirmed petitioner's non-liability for taxes and entitlement to tax refunds by issuing Revenue
Memorandum Order (RMO) No. 24-2005  addressed to specified BIR offices. The RMO provides:
47
Pursuant to the provisions of [RMC] No. 32-99 as amended by RMC No. 42-99, Japanese contractors and nationals
engaged in OECF funded projects in the Philippines shall not be required to shoulder the fiscal levies or taxes associated
with the project. Thus, the concerned Japanese contractors are entitled to claim for the refund of all taxes paid and
shouldered by them relative to the conduct of the Project.

You are, therefore, directed to expedite/ prioritize the processing of the claims for refund of Japanese contractors and
nationals so [as] not to delay and jeopardize the release of the funds for OECF funded projects.  (Emphases and
48

underscoring supplied)

Therefore, considering that petitioner paid the subject taxes in the aggregate amount of ₱52,612,812.00, which it was not
required to pay, the BIR erroneously collected such amount. Accordingly, petitioner is entitled to its refund.

As above-stated, the NIRC vests upon the CIR, being the head of the BIR, the authority to credit or refund taxes which are
erroneously collected by the government. This specific statutory mandate cannot be overridden by averse interpretations
made through mere administrative issuances, such as RMC No. 42-99, which - as argued by the CIR - shifts to the
executing agencies (particularly, NPC in this case) the power to refund the subject taxes: 49

B) INCOME TAX

1. Japanese firms or nationals operating as suppliers, contractors or consultants on and/or in connection with any income
that accrue from the supply of products and/or services to be provided under the Project Loan, shall file the prescribed
income tax returns. Since the executing government agencies are mandated to assume the payment thereof under the
Exchange of Notes, the said Japanese firms or nationals need not pay taxes thereunder.

2. The concerned Revenue District Officer shall, in turn, collect the said income taxes from the concerned executing
government agencies. 1âwphi1

3. In cases where income taxes were previously paid directly by the Japanese contractors or nationals, the corresponding
cash refund shall be recovered from the government executing agencies upon the presentation of proof of payment by the
Japanese contractors or nationals." (Emphasis and underscoring supplied)

3. In cases where income taxes were previously paid directly by the Japanese contractors or nationals, the corresponding
cash refund shall be recovered from the government executing agencies upon the presentation of proof of payment by the
Japanese contractors or nationals.   (Emphasis and underscoring supplied)
50

A revenue memorandum circular is an administrative ruling issued by the CIR to interpret tax laws. It is widely accepted
that an interpretation by the executive officers, whose duty is to enforce the law, is entitled to great respect from the
courts. However, such interpretation is not conclusive and will be disregarded if judicially found to be incorrect.  Verily,
51

courts will not tolerate administrative issuances that override, instead of remaining consistent and in harmony with, the law
they seek to implement,  as in this case. Thus, Item B (3) of RMC No. 42-99, an administrative issuance directing
52

petitioner to claim the refund from NPC, cannot prevail over Sections 204 and 229 of the NIRC, which provide that claims
for refund of erroneously collected taxes must be filed with the CIR.

All told, petitioner correctly filed its claim for tax refund under Sections 204 and 229 of the NIRC to recover the erroneously
paid taxes amounting to ₱44,288,712.00 as income tax and ₱8,324,100.00 as BPRT from the BIR. To reiterate,
petitioner's entitlement to the refund is based on the tax assumption provision in the Exchange of Notes. Given that this is
a case of tax assumption and not an exemption, the BIR is, therefore, not without recourse; it can properly collect the
subject taxes from the NPC   as the proper party that assumed petitioner's tax liability.
53

WHEREFORE, the petition is GRANTED. The Decision dated May 24, 2006 and the Resolution dated December 4, 2006
of the Court of Tax Appeals (CTA) En Banc in C.T.A. EB No. 5 are hereby REVERSED and SET ASIDE. The Decision
dated December 17, 2003 of the CTA in C.T.A. Case No. 6139 is REINSTATED.

SO ORDERED.

Social justice, Article II, Section 10, Article XII, Sections 1 (2)
-Central Azucarera de Bais v Heirs of Apostol, GR 215314, Mar 14, 2018

Time and again, the Court has put emphasis on the right of an employer to exercise its management
prerogative in dealing with its company Is affairs, including the right to dismiss erring employees. It is a
general principle of labor law to discourage interference with an employer's judgment in the conduct of his
business. Even as the law is solicitous of the welfare of the employees, it also recognizes employers
exercise of management prerogatives. As long as the company's exercise of judgment is in good faith to
advance its interest and not for the purpose of defeating or circumventing the rights of employees under
the laws or valid agreements, such exercise will be upheld. 1

The Case

Challenged before the Court via this Petition for Review on Certiorari under Rule 45 of the Rules of Court is
the Decision2 of the Court of Appeals (CA) in CA G.R. SP No. 06906, promulgated on May 22, 2013, which
affirmed the Decision3 and Resolution4 of the National Labor Relations Commission (NLRC) in NLRC Case
No. V-000451-2002, dated October 28, 2011 and February 27, 2012, respectively. Likewise challenged is
the subsequent Resolution5 of the CA promulgated on October 29, 2014, which upheld the earlier decision.

The Antecedent Facts

The respondent Zuelo Apostol, now deceased and represented herein by his heirs, commenced his 20
years of employment with petitioner Central Azucarera de Bais (CAB) on March 1, 1982 when he was hired
as the latter's Motor Pool Over-All Repairs Supervisor. 6 According to the petitioners, the respondent, as a
supervisor, was in charge of repairing company vehicles, which necessarily included the responsibilities of
(a) assigning the personnel and equipment for each and every repair job, and (b) taking custody of all
repair equipment and materials owned by CAB.7 Likewise, as a supervisor, one of the pre-requisites
accorded to the respondent was the enjoyment of a company house where the respondent could live so
long as he remains as a CAB employee.

On February 2, 2002, the parties' harmonious working relationship was disturbed when, during the
inspection of Tomasito A. Rosel (Rosel), one of CAB's security guards, it was discovered that the
respondent "was using his company house, as well as other company equipment to repair privately owned
vehicles."8 As reported by Rosel, he saw:

That the right side of the house was brightly lighted (sic) and the light came from an electrical line (trouble
light with a 100W bulb) extension coming from the house. The lighting connection was hanging some
distance from the house to the left side of the LANCER car, color white, which was parked after a pick-up
vehicle, color black. The LANCER CAR was undergoing repairs on its left side. That Mr. Francisco Sabanal
whom 1 personally know to be one of the regular workers of C.A.B. MOTOR POOL DEPARTMENT, hired as
automotive mechanic, was the one actually doing the repair work on the LANCER CAR mentioned above.
During the twenty minutes that I stayed in the premises of the house assigned to Mr. Apostol, I saw Mr.
Sabanal cutting with scissors metal sheets from the sheets that were there at the place, to repair the
LANCER CAR. He had with him on site, flattening tools and there was also an oxygen-acetylene outfit,
which he also used.9
This then triggered the CAB management, through its resident manager, Roberty Y. Dela Rosa, to issue a
memorandum addressed to the respondent for violating Rule 9 of CAB's Rules of Discipline, viz:

You will submit to this Office within 24 hours from receipt hereof your explanation in writing (to be placed
on the space indicated at the bottom of the enclosed duplicate hereof) why you should not be subjected to
our Rules of Discipline for the following acts:

For violating Rule 9 of the Rules of Discipline — for Utilizing material or equipment of the Company,
including power for doing private work without permission. Inspection by Security has disclosed that you
were having repairs done in CAB housing unit area assigned to you in Paper Village one car and one pick-
up for body repairs using oxygen and acetylene tanks with cutting accessories as well as steel plates for
the repairs, all of which are assumed to be company property there being no clearance or permit obtained
form the Company to bring in personal equipment to undertake repairs in CAB village.

Bais Central, February 4, 2002

Note: While giving you a chance to explain your side, within 24 hours from receipt hereof, you are put on
preventive suspension effective immediately.

(Sgd.)
ROBERTO Y. DELA ROSA
Resident Manager10
In response, the respondent submitted a handwritten explanation in the local dialect, which when
translated reads:

Dear Nonoy Steven,

First of all, I am asking for a thousand apologies because I undertook the repair of my personal vehicle
without securing your permission.

Noy, I did not use electric welding, compressor and grinder. What I used was a trouble light and my
personal acetylene and oxygen.

Noy, I am reiterating my asking for apology and excuse from you and I am really sorry that I have
violated your rules.
Sincerely yours,
Sgd. Zuelo Apostol11
On February 9, 2002, the respondent received a copy of the termination letter dated February 8, 2002,
which was signed by CAB's president, herein petitioner Antonio Steven L. Tan.

Thereafter, the respondent vacated the company house assigned to him, and on February 12, 2002, filed a
Complaint before the Sub-Regional Arbitration Branch No. VII of Dumaguete City against the petitioners
for constructive dismissal, illegal suspension, unfair labor practice, underpayment of overtime pay,
premium pay for holiday, separation pay, holiday pay, service incentive leave, vacation/sick leave,
recovery of actual, moral, and exemplary damages, and attorney's fees.

The Ruling of the Labor Arbiter

On May 30, 2002, the Labor Arbiter dismissed the respondent's submissions on the following
ratiocinations: (1) the allegations of unfair labor practice was not discussed in the respondent's position
paper, let alone substantiated; (2) CAB was well within its rights to impose preventive suspension upon
the respondent; (3) on the substantive aspect, CAB has reasonably shown that the complainant violated
company rules for utilizing company-owned materials and equipment; and (4) on the procedural aspect,
CAB complied with the twin requirements of notice. 12 Thus, the fallo of the decision states:

WHEREFORE, the complaint dated February 12, 2002 is dismissed for lack of merit.

SO ORDERED.13
The Ruling of the National Labor Relations Commission

Aggrieved, the respondent appealed the Labor Arbiter decision to the NLRC, which, after proper
consideration, reversed the same. The NLRC ruled that: (1) the respondent should have been given the
opportunity to be heard and to defend himself through a hearing; 14 (2) the respondent did not commit
serious misconduct because his "contrite and remorseful explanation belies any willfulness and wrongful
intent to violate the rules;"15 and (3) while the respondent did indeed violate the company rules, the
ultimate penalty of dismissal should not have been meted out to him. 16

The dispositive portion of the NLRC decision reads:

WHEREFORE, PREMISES CONSIDERED, the decision of the Labor Arbiter is, hereby, SET
ASIDE and VACATED and a new one entered finding [herein respondent] to have been illegally dismissed. [Herein
petitioner] Central Azucarera de Bais is, hereby, ordered to pay complainant the following:

Backwages P323,784.95  
Separation Pay P230,345.00  
TOTAL P554,129.00  

SO ORDERED.17
The Ruling of the Court of Appeals

From the NLRC's reversal of the Labor Arbiter's decision, the petitioners elevated the case to the CA, which
later on denied the petition and affirmed the NLRC decision. The CA averred that, while CAB was compliant
with the twin notice requirement, the respondent's violation "cannot be considered as so grave as to be
characterized either as serious misconduct or could lead to a loss of trust and confidence." 18 Thus, the CA
concluded:

WHEREFORE, in view of the foregoing premises, the Petition for Certiorari is DENIED. The NLRC's
Decision dated October 28. 2011 and its Resolution dated February 27, 2012, respectively, are
hereby AFFIRMED. Costs on petitioners.

SO ORDERED.19
The Issues

After the CA's denial of the petitioners' motion for reconsideration, the latter now comes before the Court
seeking the reversal of the assailed CA decision and resolution on the following grounds:

I. CONTRARY TO LAW AND JURISPRUDENCE, THE [CA] SERIOUSLY ERRED IN FINDING CAB GUILTY
OF ILLEGAL DISMISSAL BECAUSE SUBSTANTIVE AND PROCEDURAL DUE PROCESS REQUIREMENTS
WERE DULY COMPLIED WHEN MR. APOSTOL WAS TERMINATED.
II. CONTRARY TO LAW AND JURISPRUDENCE, THE [CA] USURPED PETITIONERS' MANAGEMENT
PREROGATIVE TO DETERMINE THE PENALTY COMMENSURATE TO THE OFFENSE COMMITTED,
WHICH HAD BEEN THE SUBJECT OF PRIOR NOTICE TO MR. APOSTOL, WHO KNEW THE
CONSEQUENCES OF HIS VIOLATION.
III. SINCE MR. APOSTOL WAS DISMISSED FOR JUST CAUSE AND IN COMPLIANCE WITH THE
REQUIREMENTS OF PROCEDURAL DUE PROCESS HE IS NOT ENTITLED TO BACKWAGES AND
SEPARATION PAY. IN ANY CASE, JURISPRUDENCE PROVIDES THAT IN A WRONGFUL TERMINATION,
GOOD FAITH MAY MITIGATE OR ABSOLVE THE PAYMENT OF BACKWAGES. 20

In sum, the petitioners put forth the following issues for the resolution of the Court: (1) whether or not
procedural and substantive due process was observed in the termination of the respondent's employment
with CAB; (2) whether or not the penalty meted out was commensurate to the violation; and
consequently, (3) whether or not the respondent is entitled to the payment of backwages and separation
pay.

The Court's Ruling

After a careful perusal of the arguments presented and the evidence submitted, the Court finds merit in
the petition.

The general rule is that only questions of law are revievvable by the Court. This is because it is not a trier
of facts;21 it is not duty-bound to analyze, review, and weigh the evidence all over again in the absence of
any showing of any arbitrariness, capriciousness, or palpable error. 22 Thus, factual findings of
administrative or quasi-judicial bodies, including labor tribunals, are accorded much respect by the Court
as they are specialized to rule on matters falling within their jurisdiction especially when these are
supported by substantial evidence.23 In labor cases, this doctrine applies with greater force as questions of
fact presented therein are for the labor tribunals to resolve. 24

The Court, however, permitted a relaxation of this rule whenever any of the following circumstances is
present:

(1) [W]hen the findings are grounded entirely on speculations, surmises or conjectures;

(2) when the inference made is manifestly mistaken, absurd or impossible;

(3) when there is grave abuse of discretion;

(4) when the judgment is based on a misapprehension of facts;

(5) when the findings of fact are conflicting;

(6) when in making its findings, the Court of Appeals went beyond the issues of the case, or its findings
are contrary to the admissions of both the appellant and the appellee;

(7) when the findings are contrary to that of the trial court;

(8) when the findings are conclusions without citation of specific evidence on which they are based;

(9) when the facts set forth in the petition, as well as in the petitioner's main and reply briefs, are not
disputed by the respondent;

(10) when the findings of fact are premised on the supposed absence of evidence and contradicted by the
evidence on record; or

(11) when the Court of Appeals manifestly overlooked certain relevant facts not disputed by the parties,
which, if properly considered, would justify a different conclusion. 25
Thus, in instances when the Labor Arbiter, the NLRC, and the CA made conflicting findings of fact, the
Court is justified—nay, the Court is compelled—to issue its own determination.

The case at hand calls for the resolution of several issues concerning the factual determination of the
court a quo.

First, on the matter of procedural due process, the Labor Arbiter and the CA were one in asseverating that
CAB complied with the procedure required of it by the Labor Code, its implementing rules and regulations,
and relevant jurisprudence. According to the Labor Arbiter,

[T]he documents which are admitted by both parties clearly show that CAB complied with the twin
requirements of due process by furnishing the [respondent] two written notices: first, a notice
apprising the complainant of the particular acts for which his dismissal is sought xxx and second, a
subsequent notice informing the complainant of the decision to dismiss him. 26 (Emphasis and underscoring
supplied)
Likewise, the CA was categorical when it asserted that CAB complied with the twin notice requirement. It
said:

Here, the twin notice requirement was substantially complied with by the petitioners.  It is
undisputed that Apostol received two notices. The first notice informed him of his violation and required
him to submit his written explanation on the matter. Thereafter, he received another notice
communicating to him that his employment with CAB was being severed by the company due to his
violation of its company's Rules of Discipline. 27 (Emphasis and underscoring supplied)
On the other hand, and contrary to the findings of both the Labor Arbiter and the CA, the NLRC found that
procedural due process was not properly observed when CAB terminated the respondent. In ruling thus,
the NLRC emphasized that, while there were actually two notices sent to the respondent, the lack of actual
hearing on the violations of the latter prior to his termination constituted a ground by which the dismissal
should be reversed. Thus,

[W]hile as a general rule a hearing is not required to satisfy the demands of procedural due process,  we
feel that the circumstances of this case required that a hearing should have been conducted to
determine the ownership of the materials and equipment used. That to us is vital in determining
the gravity of [respondent's] violation. That would have been more in accord with the employer's duty "to
afford the worker ample opportunity to be heard and defend himself with the assistance of his
representative if he so desires, in accordance with company rules and regulations promulgated pursuant to
guidelines set by the Department of Labor and Employment." 28 (Emphasis and underscoring supplied)
In the backdrop of this contradiction among the decisions, the Court is of the opinion that the Labor
Arbiter and the CA's findings are more in accord with established jurisprudence. The rights of the
respondent to procedural due process was observed by CAB.

As early as 2009, in the case of Perez vs. Philippine Telegraph and Telephone Company,29 the Court has
already laid down the guidelines in complying with the proper procedure in instances when termination of
employees is called for. In reconciling the Labor Code and its Implementing Rules and Regulations, and in
concluding that actual or formal hearing is not an absolute requirement, the Court interpreted and directed
that:

The test for the fair procedure guaranteed under Article 277(b) [now, Article 292(b)] cannot be whether
there has been a formal pretermination confrontation between the employer and the employee. The
"ample opportunity to be heard" standard is neither synonymous nor similar to a formal hearing. To
confine the employee's right to be heard to a solitary form narrows down that right. It deprives him of
other equally effective forms of adducing evidence in his defense. Certainly, such an exclusivist and
absolutist interpretation is overly restrictive. The "very nature of due process negates any concept of
inflexible procedures universally applicable to every imaginable situation."

x x x x

An employee's right to be heard in termination cases under Article 277(b) [now, Article 292(b)] as
implemented by Section 2(d), Rule I of the Implementing Rules of Book VI of the Labor Code should be
interpreted in broad strokes. It is satisfied not only by a formal face to face confrontation but by
any meaningful opportunity to controvert the charges against him and to submit evidence in
support thereof.30 (Emphasis and underscoring supplied)
Thus, in Perez, the Court formulated the following guiding principles in connection with the hearing
requirement in dismissal cases:

(a) "ample opportunity to be heard" means any meaningful opportunity (verbal or written)


given to the employee to answer the charges against him and submit evidence in support of his
defense, whether in a hearing, conference or some other fair, just and reasonable way.

(b) a formal hearing or conference becomes mandatory only when requested by the employee in writing or
substantial evidentiary disputes exist or a company rule or practice requires it, or when similar
circumstances justify it.

(c) the "ample opportunity to be heard" standard in the Labor Code prevails over the "hearing or
conference" requirement in the implementing rules and regulations. 31 (Emphasis and underscoring
supplied)
In the present case, the petitioners furnished the respondent with two notices: one, the memorandum
dated February 4, 2002 issued by CAB's resident manager 32 which informed the respondent of the charges
against him; and two, the letter of termination which, this time, notified the respondent of CAB's decision
to dismiss him.33 In the interim, CAB, through the memorandum issued by its resident manager, sought
the respondent's explanation on the incident.
The confluence of these facts, in the Court's opinion, sufficiently complies with the respondent's right to be
accorded ample opportunity to be heard.

Second, on the matter of substantive due process, the Court accedes to the uniform findings of the Labor
Arbiter, NLRC, and CA that the respondent did indeed violate company rules and regulations when he used
company equipment and materials for his personal vehicles. According to the records of this case, this
much is undisputed.

In ruling this way, the Labor Arbiter averred that "'the [respondent] violated CAB's company rules for
utilizing material or equipment of the company as well as the housing unit assigned to him in an improper
manner, i.e., for the repair of privately owned vehicles to the expense and damage of the company." 34 The
NLRC itself affirmed this finding by categorically saying that "it is not disputed that the complainant did
violate the company rules."35 More, interspersed in the CA decision are statements revealing this violation
by the respondent. Hence, the certainty by which the Labor Arbiter, NLRC, and CA pronounced this fact
requires no further disturbance—not even by the Court.

What is disputed, however, which the Court must rule upon, concerns the crux of the current controversy:
whether or not the respondent's act, which is violative of CAB's rules and regulations, warrants the
imposition of the ultimate penalty of dismissal. In this regard, the Court scoured once again the records of
the case, and after a judicious study thereof, favors the submission of the petitioners.

Article 297(c) [formerly Article 282(c)] of the Labor Code provides that an employer may terminate the
services of an employee for fraud or willful breach of the trust reposed in him. 36 According to the case
of Top Form Mfg. Co., Inc. vs. NLRC,37 an employer has a distinct prerogative to dismiss an employee if
the former has ample reason to distrust the latter or if there is sufficient evidence to show that the
employee has been guilty of breach of trust. This authority of the employer to dismiss an employee cannot
be denied whenever acts of violation are noted by the employer. 38

In ruling that employers have a right to impose a penalty of dismissal on supervisors or personnel
occupying positions of responsibility on the basis of loss of trust and confidence, the case of Moya vs. First
Solid Rubber Industries, Inc.39 stated thus:

Following the ruling in The Coca-Cola Export Corporation v. Gacayan, the employers have a right to
impose a penalty of dismissal on employees by reason of loss of trust and confidence.  More so, in the
case of supervisors or personnel occupying positions of responsibility, loss of trust, justifies
termination of employment. Loss of confidence as a just cause for termination of employment is
premised on the fact that an employee concerned holds a position of trust and confidence. This situation
holds where a person is entrusted with confidence on delicate matters, such as the custody,
handling, or care and protection of the employer's property. 40 (Emphasis and underscoring supplied,
citations omitted)
This discourse is further clarified in the recent case of Alaska Milk Corporation, and the Estate of Wilfred
Uytengsu vs. Ernesto L. Ponce 41 where the Court ruled that, in order to invoke this cause, certain
requirements must be complied with, namely: (1) the employee concerned must be holding a position of
trust and confidence; and (2) there must be an act that would justify the loss of trust and confidence. 42 In
addition to these, the case of Juliet B. Sta. Ana vs. Manila Jockey Club, Inc. 43 included, as a requirement,
that such loss of trust relates to the employee's performance of duties.

In the case at hand, a perusal of the entirety of the records would reveal that all the requirements for the
valid dismissal of the respondent exist.

To begin with, there is no doubt that the respondent, as CAB's motor pool over-all repairs supervisor, is in
a position of trust and confidence. He was in charge of repairing company vehicles, and was designated
with the responsibility of (a) assigning the personnel and equipment for each and every repair job, and (b)
taking custody of all repair equipment and materials owned by CAB. 44 In the language of Moya, the
respondent herein occupies a position of responsibility, where he is entrusted with confidence on delicate
matters, such as the custody, handling, or care and protection of CAB's properties.

Secondly, the respondent's violation of CAB's rules and regulations relating to the use of company
property for personal purposes was consistently held and upheld not only by the Labor Arbiter and the
NLRC, respectively, but also by the CA itself. That the respondent committed this act could not be denied.
What's more is that the respondent himself admitted to it. 45

Finally, the respondent's action was successfully conducted precisely because of his position in the
company. As CAB's motor pool over-all repairs supervisor, he was in the position to effect the repairs of
his personal property in the company house which was assigned to him. It could not be emphasized
further that this violation of company rules—from a supervisor no less—carries with it an impact to the
operations and management of a company, and a company's decision to terminate an employee for these
purposes is a decision that should be respected.
To be sure, the petitioners herein validly dismissed their erring employee.

Having thus ruled on the validity of the dismissal of the respondent, then it necessarily follows that he is
not entitled to both backwages and separation pay.

The Court has reiterated that the policy of social justice is not intended to countenance wrongdoing simply
because it is committed by the underprivileged. At best it may mitigate the penalty but it certainly will not
condone the offense. Compassion for the poor is an imperative of every humane society but only when the
recipient is not a rascal claiming an undeserved privilege. Social justice cannot be permitted to be refuge
of scoundrels any more than can equity be an impediment to the punishment of the guilty. Those who
invoke social justice may do so only if their hands are clean and their motives blameless and not simply
because they happen to be poor. This great policy of our Constitution is not meant for the protection of
those who have proved they are not worthy of it, like the workers who have tainted the cause of labor with
the blemishes of their own character. 46

WHEREFORE, premises considered, the Decision of the Court of Appeals in CA G.R. SP No. 06906, dated
May 22, 2013 and the subsequent Resolution dated October 29, 2014, as well as the Decision and
Resolution of the National Labor Relations Commission in NLRC Case No. V-000451-2002, dated October
28, 2011 and February 27, 2012 respectively, are hereby REVERSED and SET ASIDE. The Decision of
the Labor Arbiter dated May 30, 2002 in SUB-RAB- VII-02-003 9-2002-D is hereby  REINSTATED.

SO ORDERED.

RA 9262 – Anti Violence Against Women and their Children (2004)


-Garcia v Drilon, 699 SCRA 352 (2013)
Hailed as the bastion of Christianity in Asia, the Philippines boasts of 86.8 million Filipinos -or 93 percent
of a total population of 93.3 million - adhering to the teachings of Jesus Christ. 1 Yet, the admonition
for husbands to love their wives as their own bodies just as Christ loved the church and gave himself up
for her2 failed to prevent, or even to curb, the pervasiveness of violence against Filipino women. The
National Commission on the Role of Filipino Women (NCRFW) reported that, for the years 2000-2003,
"female violence comprised more than 90% of all forms of abuse and violence and more than 90% of
these reported cases were committed by the women's intimate partners such as their husbands and live-in
partners.”3

Thus, on March 8, 2004, after nine (9) years of spirited advocacy by women's groups, Congress enacted
Republic Act (R.A.) No. 9262, entitled “An Act Defining Violence Against Women and Their Children,
Providing for Protective Measures for Victims, Prescribing Penalties Therefor, and for Other Purposes.” It
took effect on March 27, 2004.4

R.A. 9262 is a landmark legislation that defines and criminalizes acts of violence against women and their
children (VAWC) perpetrated by women's intimate partners, i.e, husband; former husband; or any person
who has or had a sexual or dating relationship, or with whom the woman has a common child.5 The law
provides for protection orders from the barangay and the courts to prevent the commission of further acts
of VAWC; and outlines the duties and responsibilities of barangay officials, law enforcers, prosecutors and
court personnel, social workers, health care providers, and other local government officials in responding
to complaints of VAWC or requests for assistance.

A husband is now before the Court assailing the constitutionality of R.A. 9262 as being violative of the
equal protection and due process clauses, and an undue delegation of judicial power to barangay officials.

The Factual Antecedents

On March 23, 2006, Rosalie Jaype-Garcia (private respondent) filed, for herself and in behalf of her minor
children, a verified petition6 (Civil Case No. 06-797) before the Regional Trial Court (RTC) of Bacolod City
for the issuance of a Temporary Protection Order (TPO) against her husband, Jesus C. Garcia (petitioner),
pursuant to R.A. 9262. She claimed to be a victim of physical abuse; emotional, psychological, and
economic violence as a result of marital infidelity on the part of petitioner, with threats of deprivation of
custody of her children and of financial support. 7

Private respondent's claims

Private respondent married petitioner in 2002 when she was 34 years old and the former was eleven years
her senior. They have three (3) children, namely: Jo-Ann J. Garcia, 17 years old, who is the natural child
of petitioner but whom private respondent adopted; Jessie Anthone J. Garcia, 6 years old; and Joseph
Eduard J. Garcia, 3 years old.8
Private respondent described herself as a dutiful and faithful wife, whose life revolved around her husband.
On the other hand, petitioner, who is of Filipino-Chinese descent, is dominant, controlling, and demands
absolute obedience from his wife and children. He forbade private respondent to pray, and deliberately
isolated her from her friends. When she took up law, and even when she was already working part time at
a law office, petitioner trivialized her ambitions and prevailed upon her to just stay at home. He was often
jealous of the fact that his attractive wife still catches the eye of some men, at one point threatening that
he would have any man eyeing her killed.9

Things turned for the worse when petitioner took up an affair with a bank manager of Robinson's Bank,
Bacolod City, who is the godmother of one of their sons. Petitioner admitted to the affair when private
respondent confronted him about it in 2004. He even boasted to the household help about his sexual
relations with said bank manager. Petitioner told private respondent, though, that he was just using the
woman because of their accounts with the bank. 10

Petitioner's infidelity spawned a series of fights that left private respondent physically and emotionally
wounded. In one of their quarrels, petitioner grabbed private respondent on both arms and shook her with
such force that caused bruises and hematoma. At another time, petitioner hit private respondent forcefully
on the lips that caused some bleeding. Petitioner sometimes turned his ire on their daughter, Jo-Ann, who
had seen the text messages he sent to his paramour and whom he blamed for squealing on him. He beat
Jo-Ann on the chest and slapped her many times. When private respondent decided to leave petitioner, Jo-
Ann begged her mother to stay for fear that if the latter leaves, petitioner would beat her up. Even the
small boys are aware of private respondent's sufferings. Their 6- year-old son said that when he grows up,
he would beat up his father because of his cruelty to private respondent. 11

All the emotional and psychological turmoil drove private respondent to the brink of despair. On December
17, 2005, while at home, she attempted suicide by cutting her wrist. She was found by her son bleeding
on the floor. Petitioner simply fled the house instead of taking her to the hospital. Private respondent was
hospitalized for about seven (7) days in which time petitioner never bothered to visit, nor apologized or
showed pity on her. Since then, private respondent has been undergoing therapy almost every week and
is taking anti-depressant medications.12

When private respondent informed the management of Robinson's Bank that she intends to file charges
against the bank manager, petitioner got angry with her for jeopardizing the manager's job. He then
packed his things and told private respondent that he was leaving her for good. He even told private
respondent's mother, who lives with them in the family home, that private respondent should just accept
his extramarital affair since he is not cohabiting with his paramour and has not sired a child with her. 13

Private respondent is determined to separate from petitioner but she is afraid that he would take her
children from her and deprive her of financial support. Petitioner had previously warned her that if she
goes on a legal battle with him, she would not get a single centavo. 14

Petitioner controls the family businesses involving mostly the construction of deep wells. He is the
President of three corporations – 326 Realty Holdings, Inc., Negros Rotadrill Corporation, and J-Bros
Trading Corporation – of which he and private respondent are both stockholders. In contrast to the
absolute control of petitioner over said corporations, private respondent merely draws a monthly salary of
P20,000.00 from one corporation only, the Negros Rotadrill Corporation. Household expenses amounting
to not less than P200,000.00 a month are paid for by private respondent through the use of credit cards,
which, in turn, are paid by the same corporation together with the bills for utilities. 15

On the other hand, petitioner receives a monthly salary of P60,000.00 from Negros Rotadrill Corporation,
and enjoys unlimited cash advances and other benefits in hundreds of thousands of pesos from the
corporations.16  After private respondent confronted him about the affair, petitioner forbade her to hold
office at JBTC Building, Mandalagan, where all the businesses of the corporations are conducted, thereby
depriving her of access to full information about said businesses. Until the filing of the petition a
quo, petitioner has not given private respondent an accounting of the businesses the value of which she
had helped raise to millions of pesos.17

Action of the RTC of Bacolod City

Finding reasonable ground to believe that an imminent danger of violence against the private respondent
and her children exists or is about to recur, the RTC issued a TPO 18 on March 24, 2006 effective for thirty
(30) days, which is quoted hereunder: cralavvonlinelawlibrary

Respondent (petitioner herein), Jesus Chua Garcia, is hereby: cralavvonlinelawlibrary

a) Ordered to remove all his personal belongings from the conjugal dwelling or family home within 24
hours from receipt of the Temporary Restraining Order and if he refuses, ordering that he be removed by
police officers from the conjugal dwelling; this order is enforceable notwithstanding that the house is under
the name of 236 Realty Holdings Inc. (Republic Act No. 9262 states “regardless of ownership”), this is to
allow the Petitioner (private respondent herein) to enter the conjugal dwelling without any danger from
the Respondent.

After the Respondent leaves or is removed from the conjugal dwelling, or anytime the Petitioner decides to
return to the conjugal dwelling to remove things, the Petitioner shall be assisted by police officers when
re-entering the family home.

The Chief of Police shall also give the Petitioner police assistance on Sunday, 26 March 2006 because of
the danger that the Respondent will attempt to take her children from her when he arrives from Manila
and finds out about this suit.

b) To stay away from the petitioner and her children, mother and all her household help and driver from a
distance of 1,000 meters, and shall not enter the gate of the subdivision where the Petitioner may be
temporarily residing.

c) Not to harass, annoy, telephone, contact or otherwise communicate with the Petitioner, directly or
indirectly, or through other persons, or contact directly or indirectly her children, mother and household
help, nor send gifts, cards, flowers, letters and the like. Visitation rights to the children may be subject of
a modified TPO in the future.

d) To surrender all his firearms including a .9MM caliber firearm and a Walther PPK and ordering the
Philippine National Police Firearms and Explosives Unit and the Provincial Director of the PNP to cancel all
the Respondent's firearm licenses. He should also be ordered to surrender any unlicensed firearms in his
possession or control.

e) To pay full financial support for the Petitioner and the children, including rental of a house for them, and
educational and medical expenses.

f) Not to dissipate the conjugal business.

g) To render an accounting of all advances, benefits, bonuses and other cash he received from all the
corporations from 1 January 2006 up to 31 March 2006, which himself and as President of the corporations
and his Comptroller, must submit to the Court not later than 2 April 2006. Thereafter, an accounting of all
these funds shall be reported to the court by the Comptroller, copy furnished to the Petitioner, every 15
days of the month, under pain of Indirect Contempt of Court.

h) To ensure compliance especially with the order granting support pendente lite, and considering the
financial resources of the Respondent and his threat that if the Petitioner sues she will not get a single
centavo, the Respondent is ordered to put up a BOND TO KEEP THE PEACE in the amount of FIVE MILLION
PESOS, in two sufficient sureties.

On April 24, 2006, upon motion19 of private respondent, the trial court issued an amended TPO, 20 effective
for thirty (30) days, which included the following additional provisions:cralavvonlinelawlibrary

i) The petitioners (private respondents herein) are given the continued use of the Nissan Patrol and the
Starex Van which they are using in Negros Occidental.

j) The petitioners are given the continued use and occupation of the house in Parañaque, the continued
use of the Starex van in Metro Manila, whenever they go to Manila.

k) Respondent is ordered to immediately post a bond to keep the peace, in two sufficient sureties.

l) To give monthly support to the petitioner provisionally fixed in the sum of One Hundred Fifty Thousand
Pesos (Php 150,000.00) per month plus rental expenses of Fifty Thousand Pesos (Php 50,000.00) per
month until the matter of support could be finally resolved.

Two days later, or on April 26, 2006, petitioner filed an Opposition to the Urgent Ex-Parte Motion for
Renewal of the TPO21 seeking the denial of the renewal of the TPO on the grounds that it did not (1)
comply with the three-day notice rule, and (2) contain a notice of hearing. He further asked that the TPO
be modified by (1) removing one vehicle used by private respondent and returning the same to its rightful
owner, the J-Bros Trading Corporation, and (2) cancelling or reducing the amount of the bond from
P5,000,000.00 to a more manageable level at P100,000.00.

Subsequently, on May 23, 2006, petitioner moved 22 for the modification of the TPO to allow him visitation
rights to his children.
On May 24, 2006, the TPO was renewed and extended yet again, but subject only to the following
modifications prayed for by private respondent: cralavvonlinelawlibrary

a) That respondent (petitioner herein) return the clothes and other personal belongings of Rosalie and her
children to Judge Jesus Ramos, co-counsel for Petitioner, within 24 hours from receipt of the Temporary
Protection Order by his counsel, otherwise be declared in Indirect Contempt of Court; chanroblesvirtualawlibrary

b) Respondent shall make an accounting or list of furniture and equipment in the conjugal house in Pitimini
St., Capitolville Subdivision, Bacolod City within 24 hours from receipt of the Temporary Protection Order
by his counsel; chanroblesvirtualawlibrary

c) Ordering the Chief of the Women's Desk of the Bacolod City Police Headquarters to remove Respondent
from the conjugal dwelling within eight (8) hours from receipt of the Temporary Protection Order by his
counsel, and that he cannot return until 48 hours after the petitioners have left, so that the petitioner
Rosalie and her representatives can remove things from the conjugal home and make an inventory of the
household furniture, equipment and other things in the conjugal home, which shall be submitted to the
Court.

d) Deliver full financial support of Php200,000.00 and Php50,000.00 for rental and Php25,000.00 for
clothes of the three petitioners (sic) children within 24 hours from receipt of the Temporary Protection
Order by his counsel, otherwise be declared in indirect contempt of Court; chanroblesvirtualawlibrary

e) That respondent surrender his two firearms and all unlicensed firearms to the Clerk of Court within 24
hours from receipt of the Temporary Protection Order by his counsel; chanroblesvirtualawlibrary

f) That respondent shall pay petitioner educational expenses of the children upon presentation of proof of
payment of such expenses.23

Claiming that petitioner continued to deprive them of financial support; failed to faithfully comply with the
TPO; and committed new acts of harassment against her and their children, private respondent filed
another application24 for the issuance of a TPO ex parte. She alleged inter alia that petitioner contrived a
replevin suit against himself by J-Bros Trading, Inc., of which the latter was purportedly no longer
president, with the end in view of recovering the Nissan Patrol and Starex Van used by private respondent
and the children. A writ of replevin was served upon private respondent by a group of six or seven
policemen with long firearms that scared the two small boys, Jessie Anthone and Joseph Eduard. 25

While Joseph Eduard, then three years old, was driven to school, two men allegedly attempted to kidnap
him, which incident traumatized the boy resulting in his refusal to go back to school. On another occasion,
petitioner allegedly grabbed their daughter, Jo-Ann, by the arm and threatened her. 26  The incident was
reported to the police, and Jo-Ann subsequently filed a criminal complaint against her father for violation
of R.A. 7610, also known as the “Special Protection of Children Against Child Abuse, Exploitation and
Discrimination Act.”

Aside from the replevin suit, petitioner's lawyers initiated the filing by the housemaids working at the
conjugal home of a complaint for kidnapping and illegal detention against private respondent. This came
about after private respondent, armed with a TPO, went to said home to get her and her children's
belongings. Finding some of her things inside a housemaid's (Sheryl Jamola) bag in the maids' room,
private respondent filed a case for qualified theft against Jamola. 27

On August 23, 2006, the RTC issued a TPO,28 effective for thirty (30) days, which reads as follows: cralavvonlinelawlibrary

Respondent (petitioner herein), Jesus Chua Garcia, is hereby: cralavvonlinelawlibrary

1) Prohibited from threatening to commit or committing, personally or through another, acts of violence
against the offended party; chanroblesvirtualawlibrary

2) Prohibited from harassing, annoying, telephoning, contacting or otherwise communicating in any form
with the offended party, either directly or indirectly; chanroblesvirtualawlibrary

3) Required to stay away, personally or through his friends, relatives, employees or agents, from all the
Petitioners Rosalie J. Garcia and her children, Rosalie J. Garcia's three brothers, her mother Primitiva
Jaype, cook Novelita Caranzo, driver Romeo Hontiveros, laundrywoman Mercedita Bornales, security guard
Darwin Gayona and the petitioner's other household helpers from a distance of 1,000 meters, and shall not
enter the gate of the subdivision where the Petitioners are temporarily residing, as well as from the
schools of the three children; Furthermore, that respondent shall not contact the schools of the children
directly or indirectly in any manner including, ostensibly to pay for their tuition or other fees directly,
otherwise he will have access to the children through the schools and the TPO will be rendered
nugatory; chanroblesvirtualawlibrary

4) Directed to surrender all his firearms including .9MM caliber firearm and a Walther PPK to the Court; chanroblesvirtualawlibrary

5) Directed to deliver in full financial support of Php200,000.00 a month and Php50,000.00 for rental for
the period from August 6 to September 6, 2006; and support in arrears from March 2006 to August 2006
the total amount of Php1,312,000.00; chanroblesvirtualawlibrary

6) Directed to deliver educational expenses for 2006-2007 the amount of Php75,000.00 and
Php25,000.00; chanroblesvirtualawlibrary

7) Directed to allow the continued use of a Nissan Patrol with Plate No. FEW 508 and a Starex van with
Plate No. FFD 991 and should the respondent fail to deliver said vehicles, respondent is ordered to provide
the petitioner another vehicle which is the one taken by J Bros Tading; chanroblesvirtualawlibrary

8) Ordered not to dissipate, encumber, alienate, sell, lease or otherwise dispose of the conjugal assets, or
those real properties in the name of Jesus Chua Garcia only and those in which the conjugal partnership of
gains of the Petitioner Rosalie J. Garcia and respondent have an interest in, especially the conjugal home
located in No. 14, Pitimini St., Capitolville Subdivision, Bacolod City, and other properties which are
conjugal assets or those in which the conjugal partnership of gains of Petitioner Rosalie J. Garcia and the
respondent have an interest in and listed in Annexes “I,” “I-1,” and “I-2,” including properties covered by
TCT Nos. T-186325 and T-168814; chanroblesvirtualawlibrary

9) Ordered that the Register of Deeds of Bacolod City and E.B. Magalona shall be served a copy of this
TEMPORARY PROTECTION ORDER and are ordered not to allow the transfer, sale, encumbrance or
disposition of these above-cited properties to any person, entity or corporation without the personal
presence of petitioner Rosalie J. Garcia, who shall affix her signature in the presence of the Register of
Deeds, due to the fear of petitioner Rosalie that her signature will be forged in order to effect the
encumbrance or sale of these properties to defraud her or the conjugal partnership of gains.

In its Order29 dated September 26, 2006, the trial court extended the aforequoted TPO for another ten
(10) days, and gave petitioner a period of five (5) days within which to show cause why the TPO should
not be renewed, extended, or modified. Upon petitioner's manifestation, 30 however, that he has not
received a copy of private respondent's motion to modify/renew the TPO, the trial court directed in its
Order31 dated October 6, 2006 that petitioner be furnished a copy of said motion. Nonetheless, an
Order32 dated a day earlier, October 5, had already been issued renewing the TPO dated August 23, 2006.
The pertinent portion is quoted hereunder: cralavvonlinelawlibrary

x x x x

x x x it appearing further that the hearing could not yet be finally terminated, the Temporary Protection
Order issued on August 23, 2006 is hereby renewed and extended for thirty (30) days and continuously
extended and renewed for thirty (30) days, after each expiration, until further orders, and subject to such
modifications as may be ordered by the court.

After having received a copy of the foregoing Order, petitioner no longer submitted the required comment
to private respondent's motion for renewal of the TPO arguing that it would only be an “exercise in
futility.”33

Proceedings before the CA

During the pendency of Civil Case No. 06-797, petitioner filed before the Court of Appeals (CA) a
petition34 for prohibition (CA-G.R. CEB- SP. No. 01698), with prayer for injunction and temporary
restraining order, challenging (1) the constitutionality of R.A. 9262 for being violative of the due process
and the equal protection clauses, and (2) the validity of the modified TPO issued in the civil case for being
“an unwanted product of an invalid law.”

On May 26, 2006, the appellate court issued a 60-day Temporary Restraining Order 35 (TRO) against the
enforcement of the TPO, the amended TPOs and other orders pursuant thereto.

Subsequently, however, on January 24, 2007, the appellate court dismissed 36 the petition for failure of
petitioner to raise the constitutional issue in his pleadings before the trial court in the civil case, which is
clothed with jurisdiction to resolve the same. Secondly, the challenge to the validity of R.A. 9262 through
a petition for prohibition seeking to annul the protection orders issued by the trial court constituted a
collateral attack on said law.

His motion for reconsideration of the foregoing Decision having been denied in the Resolution 37 dated
August 14, 2007, petitioner is now before us alleging that –

The Issues

I.

THE COURT OF APPEALS ERRED IN DISMISSING THE PETITION ON THE THEORY THAT THE ISSUE OF
CONSTITUTIONALITY WAS NOT RAISED AT THE EARLIEST OPPORTUNITY AND THAT, THE PETITION
CONSTITUTES A COLLATERAL ATTACK ON THE VALIDITY OF THE LAW.

II.

THE COURT OF APPEALS COMMITTED SERIOUS ERROR IN FAILING TO CONCLUDE THAT R.A. 9262 IS
DISCRIMINATORY, UNJUST, AND VIOLATIVE OF THE EQUAL PROTECTION CLAUSE.

III.

THE COURT OF APPEALS COMMITTED GRAVE MISTAKE IN NOT FINDING THAT R.A. 9262 RUNS COUNTER
TO THE DUE PROCESS CLAUSE OF THE CONSTITUTION.

IV.

THE COURT OF APPEALS ERRED IN NOT FINDING THAT THE LAW DOES VIOLENCE TO THE POLICY OF THE
STATE TO PROTECT THE FAMILY AS A BASIC SOCIAL INSTITUTION.

V.

THE COURT OF APPEALS SERIOUSLY ERRED IN NOT DECLARING R.A. No. 9262 AS INVALID AND
UNCONSTITUTIONAL BECAUSE IT ALLOWS AN UNDUE DELEGATION OF JUDICIAL POWER TO THE
BARANGAY OFFICIALS.38 nadcralavvonlinelawlibrary

The Ruling of the Court

Before delving into the arguments propounded by petitioner against the constitutionality of R.A. 9262, we
shall first tackle the propriety of the dismissal by the appellate court of the petition for prohibition (CA-
G.R. CEB-SP. No. 01698) filed by petitioner.

As a general rule, the question of constitutionality must be raised at the earliest opportunity so that if not
raised in the pleadings, ordinarily it may not be raised in the trial, and if not raised in the trial court, it will
not be considered on appeal.39 Courts will not anticipate a question of constitutional law in advance of the
necessity of deciding it.40

In defending his failure to attack the constitutionality of R.A. 9262 before the RTC of Bacolod City,
petitioner argues that the Family Court has limited authority and jurisdiction that is “inadequate to tackle
the complex issue of constitutionality.” 41

We disagree.

Family Courts have authority and jurisdiction


to consider the constitutionality of a statute.

At the outset, it must be stressed that Family Courts are special courts, of the same level as Regional Trial
Courts. Under R.A. 8369, otherwise known as the “Family Courts Act of 1997,” family courts have
exclusive original jurisdiction to hear and decide cases of domestic violence against women and
children.42 In accordance with said law, the Supreme Court designated from among the branches of the
Regional Trial Courts at least one Family Court in each of several key cities identified. 43 To achieve
harmony with the first mentioned law, Section 7 of R.A. 9262 now provides that Regional Trial Courts
designated as Family Courts shall have original and exclusive jurisdiction over cases of VAWC defined
under the latter law, viz: cralavvonlinelawlibrary

SEC. 7. Venue. – The Regional Trial Court designated as a Family Court shall have original and
exclusive jurisdiction over cases of violence against women and their children under this law. In the
absence of such court in the place where the offense was committed, the case shall be filed in the Regional
Trial Court where the crime or any of its elements was committed at the option of the complainant.
(Emphasis supplied)
Inspite of its designation as a family court, the RTC of Bacolod City remains possessed of authority as a
court of general original jurisdiction to pass upon all kinds of cases whether civil, criminal, special
proceedings, land registration, guardianship, naturalization, admiralty or insolvency. 44 It is settled that
RTCs have jurisdiction to resolve the constitutionality of a statute, 45 “this authority being embraced in the
general definition of the judicial power to determine what are the valid and binding laws by the criterion of
their conformity to the fundamental law.”46 The Constitution vests the power of judicial review or the
power to declare the constitutionality or validity of a law, treaty, international or executive agreement,
presidential decree, order, instruction, ordinance, or regulation not only in this Court, but in all RTCs. 47 We
said in J.M. Tuason and Co., Inc. v. CA48 that, “[p]lainly the Constitution contemplates that the inferior
courts should have jurisdiction in cases involving constitutionality of any treaty or law, for it speaks of
appellate review of final judgments of inferior courts in cases where such constitutionality happens to be in
issue.” Section 5, Article VIII of the 1987 Constitution reads in part as follows: cralavvonlinelawlibrary

SEC. 5. The Supreme Court shall have the following powers: cralavvonlinelawlibrary

x x x

2. Review, revise, reverse, modify, or affirm on appeal or certiorari, as the law or the Rules of Court may
provide, final judgments and orders of lower courts in: cralavvonlinelawlibrary

a. All cases in which the constitutionality or validity of any treaty, international or executive agreement,
law, presidential decree, proclamation, order, instruction, ordinance, or regulation is in question.

xxxx

Thus, contrary to the posturing of petitioner, the issue of constitutionality of R.A. 9262 could have been
raised at the earliest opportunity in his Opposition to the petition for protection order before the RTC of
Bacolod City, which had jurisdiction to determine the same, subject to the review of this Court.

Section 20 of A.M. No. 04-10-11-SC, the Rule on Violence Against Women and Their Children, lays down a
new kind of procedure requiring the respondent to file an opposition to the petition and not an answer. 49 
Thus: cralavvonlinelawlibrary

SEC. 20. Opposition to petition. – (a) The respondent may file an opposition to the petition which he
himself shall verify. It must be accompanied by the affidavits of witnesses and shall show cause why a
temporary or permanent protection order should not be issued.

(b) Respondent shall not include in the opposition any counterclaim, cross-claim or third-party
complaint, but any cause of action which could be the subject thereof may be litigated in a separate civil
action. (Emphasis supplied)

We cannot subscribe to the theory espoused by petitioner that, since a counterclaim, cross-claim and
third-party complaint are to be excluded from the opposition, the issue of constitutionality cannot likewise
be raised therein. A counterclaim is defined as any claim for money or other relief which a defending
party may have against an opposing party.50 A cross- claim, on the other hand, is any claim by one party
against a co-party arising out of the transaction or occurrence that is the subject matter either of the
original action or of a counterclaim therein. 51 Finally, a third-party complaint is a claim that a defending
party may, with leave of court, file against a person not a party to the action for contribution, indemnity,
subrogation or any other relief, in respect of his opponent's claim. 52 As pointed out by Justice Teresita J.
Leonardo-De Castro, the unconstitutionality of a statute is not a cause of action that could be the subject
of a counterclaim, cross-claim or a third-party complaint. Therefore, it is not prohibited from being raised
in the opposition in view of the familiar maxim expressio unius est exclusio alterius.

Moreover, it cannot be denied that this issue affects the resolution of the case a quo because the right of
private respondent to a protection order is founded solely on the very statute the validity of which is being
attacked53 by petitioner who has sustained, or will sustain, direct injury as a result of its enforcement. The
alleged unconstitutionality of R.A. 9262 is, for all intents and purposes, a valid cause for the non-issuance
of a protection order.

That the proceedings in Civil Case No. 06-797 are summary in nature should not have deterred petitioner
from raising the same in his Opposition. The question relative to the constitutionality of a statute is one of
law which does not need to be supported by evidence. 54 Be that as it may, Section 25 of A.M. No. 04-10-
11-SC nonetheless allows the conduct of a hearing to determine legal issues, among others, viz: cralavvonlinelawlibrary
SEC. 25. Order for further hearing. - In case the court determines the need for further hearing, it may
issue an order containing the following: cralavvonlinelawlibrary

(a) Facts undisputed and admitted; chanroblesvirtualawlibrary

(b) Factual and legal issues to be resolved; chanroblesvirtualawlibrary

(c) Evidence, including objects and documents that have been marked and will be presented; chanroblesvirtualawlibrary

(d) Names of witnesses who will be ordered to present their direct testimonies in the form of affidavits;
and

(e) Schedule of the presentation of evidence by both parties which shall be done in one day, to the extent
possible, within the 30-day period of the effectivity of the temporary protection order issued. (Emphasis
supplied)

To obviate potential dangers that may arise concomitant to the conduct of a hearing when necessary,
Section 26 (b) of A.M. No. 04-10-11- SC provides that if a temporary protection order issued is due to
expire, the trial court may extend or renew the said order for a period of thirty (30) days each time until
final judgment is rendered. It may likewise modify the extended or renewed temporary protection order as
may be necessary to meet the needs of the parties. With the private respondent given ample protection,
petitioner could proceed to litigate the constitutional issues, without necessarily running afoul of the very
purpose for the adoption of the rules on summary procedure.

In view of all the foregoing, the appellate court correctly dismissed the petition for prohibition with prayer
for injunction and temporary restraining order (CA-G.R. CEB - SP. No. 01698). Petitioner may have
proceeded upon an honest belief that if he finds succor in a superior court, he could be granted an
injunctive relief. However, Section 22(j) of A.M. No. 04-10-11-SC expressly disallows the filing of a
petition for certiorari, mandamus or prohibition against any interlocutory order issued by the trial
court. Hence, the 60-day TRO issued by the appellate court in this case against the enforcement of the
TPO, the amended TPOs and other orders pursuant thereto was improper, and it effectively hindered the
case from taking its normal course in an expeditious and summary manner.

As the rules stand, a review of the case by appeal or certiorari before judgment is prohibited. Moreover, if
the appeal of a judgment granting permanent protection shall not stay its enforcement, 55 with more
reason that a TPO, which is valid only for thirty (30) days at a time, 56 should not be enjoined.

The mere fact that a statute is alleged to be unconstitutional or invalid, does not of itself entitle a litigant
to have the same enjoined.57 In Younger v. Harris, Jr.,58 the Supreme Court of the United States declared,
thus: cralavvonlinelawlibrary

Federal injunctions against state criminal statutes, either in their entirety or with respect to their separate
and distinct prohibitions, are not to be granted as a matter of course, even if such statutes are
unconstitutional. No citizen or member of the community is immune from prosecution, in good faith, for
his alleged criminal acts. The imminence of such a prosecution even though alleged to be unauthorized
and, hence, unlawful is not alone ground for relief in equity which exerts its extraordinary powers only to
prevent irreparable injury to the plaintiff who seeks its aid. (Citations omitted)

The sole objective of injunctions is to preserve the status quo until the trial court hears fully the merits of
the case. It bears stressing, however, that protection orders are granted ex parte so as to protect women
and their children from acts of violence. To issue an injunction against such orders will defeat the very
purpose of the law against VAWC.

Notwithstanding all these procedural flaws, we shall not shirk from our obligation to determine novel
issues, or issues of first impression, with far-reaching implications. We have, time and again, discharged
our solemn duty as final arbiter of constitutional issues, and with more reason now, in view of private
respondent's plea in her Comment59 to the instant Petition that we should put the challenge to the
constitutionality of R.A. 9262 to rest. And so we shall.

Intent of Congress in enacting R.A. 9262. 

Petitioner claims that since R.A. 9262 is intended to prevent and criminalize spousal and child abuse,
which could very well be committed by either the husband or the wife, gender alone is not enough basis to
deprive the husband/father of the remedies under the law. 60

A perusal of the deliberations of Congress on Senate Bill No. 2723, 61 which became R.A. 9262, reveals that
while the sponsor, Senator Luisa Pimentel-Ejercito (better known as Senator Loi Estrada), had originally
proposed what she called a “synthesized measure”62 – an amalgamation of two measures, namely, the
“Anti-Domestic Violence Act” and the “Anti- Abuse of Women in Intimate Relationships Act” 63 – providing
protection to “all family members, leaving no one in isolation” but at the same time giving special attention
to women as the “usual victims” of violence and abuse, 64 nonetheless, it was eventually agreed that men
be denied protection under the same measure. We quote pertinent portions of the deliberations: cralavvonlinelawlibrary

Wednesday, December 10, 2003

Senator Pangilinan. I just wanted to place this on record, Mr. President. Some women's groups have
expressed concerns and relayed these concerns to me that if we are to include domestic violence apart
from against women as well as other members of the household, including children or the husband, they
fear that this would weaken the efforts to address domestic violence of which the main victims or the bulk
of the victims really are the wives, the spouses or the female partners in a relationship. We would like to
place that on record. How does the good Senator respond to this kind of observation?

Senator Estrada. Yes, Mr. President, there is this group of women who call themselves “WIIR” Women in
Intimate Relationship. They do not want to include men in this domestic violence. But plenty of men are
also being abused by women. I am playing safe so I placed here members of the family, prescribing
penalties therefor and providing protective measures for victims. This includes the men, children, live-in,
common-law wives, and those related with the family. 65

x x x x

Wednesday, January 14, 2004

x x x x

The President Pro Tempore. x x x

Also, may the Chair remind the group that there was the discussion whether to limit this to women and
not to families which was the issue of the AWIR group. The understanding that I have is that we would be
having a broader scope rather than just women, if I remember correctly, Madam sponsor.

Senator Estrada. Yes, Mr. President.

As a matter of fact, that was brought up by Senator Pangilinan during the interpellation period.

I think Senator Sotto has something to say to that.

Senator Legarda. Mr. President, the reason I am in support of the measure. Do not get me wrong.
However, I believe that there is a need to protect women's rights especially in the domestic environment.

As I said earlier, there are nameless, countless, voiceless women who have not had the opportunity to file
a case against their spouses, their live-in partners after years, if not decade, of battery and abuse. If we
broaden the scope to include even the men, assuming they can at all be abused by the women or their
spouses, then it would not equalize the already difficult situation for women, Mr. President.

I think that the sponsor, based on our earlier conversations, concurs with this position. I am sure that the
men in this Chamber who love their women in their lives so dearly will agree with this representation.
Whether we like it or not, it is an unequal world. Whether we like it or not, no matter how empowered the
women are, we are not given equal opportunities especially in the domestic environment where the macho
Filipino man would always feel that he is stronger, more superior to the Filipino woman.

x x x x

The President Pro Tempore. What does the sponsor say?

Senator Estrada. Mr. President, before accepting this, the committee came up with this bill because the
family members have been included in this proposed measure since the other members of the family other
than women are also possible victims of violence. While women are most likely the intended victims, one
reason incidentally why the measure focuses on women, the fact remains that in some relatively few
cases, men also stand to be victimized and that children are almost always the helpless victims of
violence. I am worried that there may not be enough protection extended to other family members
particularly children who are excluded. Although Republic Act No. 7610, for instance, more or less,
addresses the special needs of abused children. The same law is inadequate. Protection orders for one are
not available in said law.

I am aware that some groups are apprehensive about granting the same protection to men, fearing that
they may use this law to justify their abusive behavior against women. However, we should also recognize
that there are established procedures and standards in our courts which give credence to evidentiary
support and cannot just arbitrarily and whimsically entertain baseless complaints.

Mr. President, this measure is intended to harmonize family relations and to protect the family as the basic
social institution. Though I recognize the unequal power relations between men and women in our society,
I believe we have an obligation to uphold inherent rights and dignity of both husband and wife and their
immediate family members, particularly children.

While I prefer to focus mainly on women, I was compelled to include other family members as a critical
input arrived at after a series of consultations/meetings with various NGOs, experts, sports groups and
other affected sectors, Mr. President.

Senator Sotto. Mr. President.

The President Pro Tempore. Yes, with the permission of the other senators.

Senator Sotto. Yes, with the permission of the two ladies on the Floor. The President Pro Tempore. Yes,
Sen. Vicente C. Sotto III is recognized. Senator Sotto. I presume that the effect of the proposed
amendment of

Senator Legarda would be removing the “men and children” in this particular bill and focus specifically on
women alone. That will be the net effect of that proposed amendment. Hearing the rationale mentioned by
the distinguished sponsor, Sen. Luisa “Loi” Ejercito Estrada, I am not sure now whether she is inclined to
accept the proposed amendment of Senator Legarda.

I am willing to wait whether she is accepting this or not because if she is going to accept this, I will
propose an amendment to the amendment rather than object to the amendment, Mr. President.

x x x x

Senator Estrada. The amendment is accepted, Mr. President. The President Pro Tempore. Is there any
objection?

x x x x

Senator Sotto. x x x May I propose an amendment to the amendment. The President Pro Tempore. Before
we act on the amendment? Senator Sotto. Yes, Mr. President.

The President Pro Tempore. Yes, please proceed.

Senator Sotto. Mr. President, I am inclined to believe the rationale used by the distinguished proponent of
the amendment. As a matter of fact, I tend to agree. Kung may maaabuso, mas malamang iyong babae
kaysa sa lalake. At saka iyong mga lalake, puwede na talagang magulpi iyan. Okey lang iyan. But I cannot
agree that we remove the children from this particular measure.

So, if I may propose an amendment –

The President Pro Tempore. To the amendment.

Senator Sotto. – more than the women, the children are very much abused. As a matter of fact, it is not
limited to minors. The abuse is not limited to seven, six, 5-year-old children. I have seen 14, 15-year-old
children being abused by their fathers, even by their mothers. And it breaks my heart to find out about
these things.

Because of the inadequate existing law on abuse of children, this particular measure will update that. It
will enhance and hopefully prevent the abuse of children and not only women.

SOTTO-LEGARDA AMENDMENTS

Therefore, may I propose an amendment that, yes, we remove the aspect of the men in the bill but not
the children.

Senator Legarda. I agree, Mr. President, with the Minority Leader.

The President Pro Tempore. Effectively then, it will be women AND CHILDREN.

Senator Sotto. Yes, Mr. President.

Senator Estrada. It is accepted, Mr. President.


The President Pro Tempore. Is there any objection? [Silence] There being none, the amendment, as
amended, is approved.66

It is settled that courts are not concerned with the wisdom, justice, policy, or expediency of a
statute.67 Hence, we dare not venture into the real motivations and wisdom of the members of Congress in
limiting the protection against violence and abuse under R.A. 9262 to women and children only. No proper
challenge on said grounds may be entertained in this proceeding. Congress has made its choice and it is
not our prerogative to supplant this judgment. The choice may be perceived as erroneous but even then,
the remedy against it is to seek its amendment or repeal by the legislative. By the principle of separation
of powers, it is the legislative that determines the necessity, adequacy, wisdom and expediency of any
law.68

We only step in when there is a violation of the Constitution. However, none was sufficiently shown in this
case.

R.A. 9262 does not violate the


guaranty of equal protection
of the laws.

Equal protection simply requires that all persons or things similarly situated should be treated alike, both
as to rights conferred and responsibilities imposed. The oft-repeated disquisition in the early case
of Victoriano v. Elizalde Rope Workers' Union69 is instructive:cralavvonlinelawlibrary

The guaranty of equal protection of the laws is not a guaranty of equality in the application of the laws
upon all citizens of the state. It is not, therefore, a requirement, in order to avoid the constitutional
prohibition against inequality, that every man, woman and child should be affected alike by a statute.
Equality of operation of statutes does not mean indiscriminate operation on persons merely as such, but
on persons according to the circumstances surrounding them. It guarantees equality, not identity of rights.
The Constitution does not require that things which are different in fact be treated in law as though they
were the same. The equal protection clause does not forbid discrimination as to things that are different. It
does not prohibit legislation which is limited either in the object to which it is directed or by the territory
within which it is to operate.

The equal protection of the laws clause of the Constitution allows classification. Classification in law, as in
the other departments of knowledge or practice, is the grouping of things in speculation or practice
because they agree with one another in certain particulars. A law is not invalid because of simple
inequality. The very idea of classification is that of inequality, so that it goes without saying that the mere
fact of inequality in no manner determines the matter of constitutionality. All that is required of a valid
classification is that it be reasonable, which means that the classification should be based on substantial
distinctions which make for real differences; that it must be germane to the purpose of the law; that it
must not be limited to existing conditions only; and that it must apply equally to each member of
the class. This Court has held that the standard is satisfied if the classification or distinction is based on a
reasonable foundation or rational basis and is not palpably arbitrary. (Emphasis supplied)

Measured against the foregoing jurisprudential yardstick, we find that R.A. 9262 is based on a valid
classification as shall hereinafter be discussed and, as such, did not violate the equal protection clause by
favoring women over men as victims of violence and abuse to whom the State extends its protection.

I. R.A. 9262 rests on substantial distinctions.

The unequal power relationship between women and men; the fact that women are more likely than men
to be victims of violence; and the widespread gender bias and prejudice against women all make for real
differences justifying the classification under the law. As Justice McIntyre succinctly states, “the
accommodation of differences ... is the essence of true equality.” 70

A. Unequal power relationship between men and women 

According to the Philippine Commission on Women (the National Machinery for Gender Equality and
Women's Empowerment), violence against women (VAW) is deemed to be closely linked with the unequal
power relationship between women and men otherwise known as “gender-based violence”. Societal
norms and traditions dictate people to think men are the leaders, pursuers, providers, and take on
dominant roles in society while women are nurturers, men's companions and supporters, and take on
subordinate roles in society. This perception leads to men gaining more power over women. With power
comes the need to control to retain that power. And VAW is a form of men's expression of controlling
women to retain power.71

The United Nations, which has long recognized VAW as a human rights issue, passed its Resolution 48/104
on the Declaration on Elimination of Violence Against Women on December 20, 1993 stating that “violence
against women is a manifestation of historically unequal power relations between men and women,
which have led to domination over and discrimination against women by men and to the prevention of the
full advancement of women, and that violence against women is one of the crucial social mechanisms by
which women are forced into subordinate positions, compared with men.” 72

Then Chief Justice Reynato S. Puno traced the historical and social context of gender-based violence and
developments in advocacies to eradicate VAW, in his remarks delivered during the Joint Launching of R.A.
9262 and its Implementing Rules last October 27, 2004, the pertinent portions of which are quoted
hereunder: cralavvonlinelawlibrary

History reveals that most societies sanctioned the use of violence against women. The patriarch of a family
was accorded the right to use force on members of the family under his control. I quote the early
studies:cralavvonlinelawlibrary

Traditions subordinating women have a long history rooted in patriarchy – the institutional rule of men.
Women were seen in virtually all societies to be naturally inferior both physically and intellectually. In
ancient Western societies, women whether slave, concubine or wife, were under the authority of men. In
law, they were treated as property.

The Roman concept of patria potestas  allowed the husband to beat, or even kill, his wife if she endangered
his property right over her. Judaism, Christianity and other religions oriented towards the patriarchal
family strengthened the male dominated structure of society.

English feudal law reinforced the tradition of male control over women. Even the eminent Blackstone has
been quoted in his commentaries as saying husband and wife were one and that one was the husband.
However, in the late 1500s and through the entire 1600s, English common law began to limit the right of
husbands to chastise their wives. Thus, common law developed the rule of thumb, which allowed husbands
to beat their wives with a rod or stick no thicker than their thumb.

In the later part of the 19 th century, legal recognition of these rights to chastise wives or inflict corporeal
punishment ceased. Even then, the preservation of the family was given more importance than preventing
violence to women.

The metamorphosis of the law on violence in the United States followed that of the English common law.
In 1871, the Supreme Court of Alabama became the first appellate court to strike down the common law
right of a husband to beat his wife: cralavvonlinelawlibrary

The privilege, ancient though it may be, to beat one's wife with a stick, to pull her hair, choke her, spit in
her face or kick her about the floor, or to inflict upon her like indignities, is not now acknowledged by our
law... In person, the wife is entitled to the same protection of the law that the husband can invoke for
himself.

As time marched on, the women's advocacy movement became more organized. The temperance leagues
initiated it. These leagues had a simple focus. They considered the evils of alcoholism as the root cause of
wife abuse. Hence, they demonstrated and picketed saloons, bars and their husbands' other watering
holes. Soon, however, their crusade was joined by suffragette movements, expanding the liberation
movement's agenda. They fought for women's right to vote, to own property, and more. Since then, the
feminist movement was on the roll.

The feminist movement exposed the private invisibility of the domestic violence to the public gaze. They
succeeded in transforming the issue into an important public concern. No less than the United States
Supreme Court, in 1992 case Planned Parenthood v. Casey, noted: cralavvonlinelawlibrary

In an average 12-month period in this country, approximately two million women are the victims of severe
assaults by their male partners. In a 1985 survey, women reported that nearly one of every eight
husbands had assaulted their wives during the past year. The [American Medical Association] views these
figures as “marked underestimates,” because the nature of these incidents discourages women from
reporting them, and because surveys typically exclude the very poor, those who do not speak English well,
and women who are homeless or in institutions or hospitals when the survey is conducted. According to
the AMA, “researchers on family violence agree that the true incidence of partner violence is probably
double the above estimates; or four million severely assaulted women per year.”

Studies on prevalence suggest that from one-fifth to one-third of all women will be physically assaulted by
a partner or ex-partner during their lifetime... Thus on an average day in the United States, nearly 11,000
women are severely assaulted by their male partners. Many of these incidents involve sexual assault... In
families where wife beating takes place, moreover, child abuse is often present as well.
Other studies fill in the rest of this troubling picture. Physical violence is only the most visible form of
abuse. Psychological abuse, particularly forced social and economic isolation of women, is also common.

Many victims of domestic violence remain with their abusers, perhaps because they perceive no superior
alternative...Many abused women who find temporary refuge in shelters return to their husbands, in large
part because they have no other source of income... Returning to one's abuser can be dangerous. Recent
Federal Bureau of Investigation statistics disclose that 8.8 percent of all homicide victims in the United
States are killed by their spouses...Thirty percent of female homicide victims are killed by their male
partners.

Finally in 1994, the United States Congress enacted the Violence Against Women Act.

In the International front, the women's struggle for equality was no less successful. The United States
Charter and the Universal Declaration of Human Rights affirmed the equality of all human beings. In 1979,
the UN General Assembly adopted the landmark Convention on the Elimination of all Forms of
Discrimination Against Women (CEDAW). In 1993, the UN General Assembly also adopted the Declaration
on the Elimination of Violence Against Women. World conferences on the role and rights of women have
been regularly held in Mexico City, Copenhagen, Nairobi and Beijing. The UN itself established a
Commission on the Status of Women.

The Philippines has been in cadence with the half – and full – steps of all these women's movements. No
less than Section 14, Article II of our 1987 Constitution mandates the State to recognize the role of
women in nation building and to ensure the fundamental equality before the law of women and men. Our
Senate has ratified the CEDAW as well as the Convention on the Rights of the Child and its two protocols.
To cap it all, Congress, on March 8, 2004, enacted Rep. Act No. 9262, entitled “An Act Defining Violence
Against Women and Their Children, Providing for Protective Measures for Victims, Prescribing Penalties
therefor and for other Purposes.” (Citations omitted)

B. Women are the “usual” and “most likely”


victims of violence.

At the time of the presentation of Senate Bill No. 2723, official statistics on violence against women and
children show that –

x x x physical injuries had the highest number of cases at 5,058 in 2002 representing 55.63% of total
cases reported (9,903). And for the first semester of 2003, there were 2,381 reported cases out of 4,354
cases which represent 54.31%. xxx (T)he total number of women in especially difficult circumstances
served by the Department of Social Welfare and Development (DSWD) for the year 2002, there are 1,417
physically abused/maltreated cases out of the total of 5,608 cases. xxx (T)here are 1,091 DSWD cases out
of a total number of 3,471 cases for the first semester of 2003. Female violence comprised more than
90% of all forms of abuse and violence and more than 90% of these reported cases were committed by
the women's intimate partners such as their husbands and live-in partners. 73

Recently, the Philippine Commission on Women presented comparative statistics on violence against
women across an eight-year period from 2004 to August of 2011 with violations under R.A. 9262 ranking
first among the different VAW categories since its implementation in 2004, 74 thus: cralavvonlinelawlibrary

Table 1. Annual Comparative Statistics on Violence Against Women, 2004 - 2011*

Reported Cases 2004 2005 2006 2007 2008 2009 2010 2011
Rape 997 927 659 837 811 770 1,042 832
Incestuous Rape 38 46 26 22 28 27 19 23
Attempted Rape 194 148 185 147 204 167 268 201
Acts of Lasciviousness 580 536 382 358 445 485 745 625
Physical Injuries 3,553 2,335 1,892 1,505 1,307 1,498 2,018 1,588
Sexual Harassment 53 37 38 46 18 54 83 63
RA 9262 218 924 1,269 2,387 3,599 5,285 9,974 9,021
Threats 319 223 199 182 220 208 374 213
Seduction 62 19 29 30 19 19 25 15
Concubinage 121 102 93 109 109 99 158 128
RA 9208 17 11 16 24 34 152 190 62
Abduction 29 16 34 23 28 18 25 22
/Kidnapping
Unjust Vexation 90 50 59 59 83 703 183 155
Total 6,271 5,374 4,881 5,729 6,905 9,485 15,104 12,948

*2011 report covers only from January to August


Source: Philippine National Police – Women and Children Protection Center (WCPC)

On the other hand, no reliable estimates may be obtained on domestic abuse and violence against men in
the Philippines because incidents thereof are relatively low and, perhaps, because many men will not even
attempt to report the situation. In the United Kingdom, 32% of women who had ever experienced
domestic violence did so four or five (or more) times, compared with 11% of the smaller number of men
who had ever experienced domestic violence; and women constituted 89% of all those who had
experienced 4 or more incidents of domestic violence. 75 Statistics in Canada show that spousal violence by
a woman against a man is less likely to cause injury than the other way around (18 percent versus 44
percent). Men, who experience violence from their spouses are much less likely to live in fear of violence
at the hands of their spouses, and much less likely to experience sexual assault. In fact, many cases of
physical violence by a woman against a spouse are in self-defense or the result of many years of physical
or emotional abuse.76

While there are, indeed, relatively few cases of violence and abuse perpetrated against men in the
Philippines, the same cannot render R.A. 9262 invalid.

In a 1960 case involving the violation of a city ordinance requiring drivers of animal-drawn vehicles to pick
up, gather and deposit in receptacles the manure emitted or discharged by their vehicle-drawing animals
in any public highways, streets, plazas, parks or alleys, said ordinance was challenged as violative of the
guaranty of equal protection of laws as its application is limited to owners and drivers of vehicle-drawing
animals and not to those animals, although not utilized, but similarly pass through the same streets.

The ordinance was upheld as a valid classification for the reason that, while there may be non-vehicle-
drawing animals that also traverse the city roads, “but their number must be negligible and their
appearance therein merely occasional, compared to the rig-drawing ones, as not to constitute a
menace to the health of the community.” 77 The mere fact that the legislative classification may result in
actual inequality is not violative of the right to equal protection, for every classification of persons or
things for regulation by law produces inequality in some degree, but the law is not thereby rendered
invalid.78

C. Gender bias and prejudices

From the initial report to the police through prosecution, trial, and sentencing, crimes against women are
often treated differently and less seriously than other crimes. This was argued by then United States
Senator Joseph R. Biden, Jr., now Vice President, chief sponsor of the Violence Against Women Act
(VAWA), in defending the civil rights remedy as a valid exercise of the U.S. Congress' authority under the
Commerce and Equal Protection Clauses. He stressed that the widespread gender bias in the U.S. has
institutionalized historic prejudices against victims of rape or domestic violence, subjecting them to
“double victimization” – first at the hands of the offender and then of the legal system. 79

Our own Senator Loi Estrada lamented in her Sponsorship Speech for Senate Bill No. 2723 that
“(w)henever violence occurs in the family, the police treat it as a private matter and advise the parties to
settle the conflict themselves. Once the complainant brings the case to the prosecutor, the latter is
hesitant to file the complaint for fear that it might later be withdrawn. This lack of response or reluctance
to be involved by the police and prosecution reinforces the escalating, recurring and often serious nature
of domestic violence.”80

Sadly, our own courts, as well, have exhibited prejudices and biases against our women.

In a recent case resolved on March 9, 2011, we fined RTC Judge Venancio J. Amila for Conduct
Unbecoming of a Judge. He used derogatory and irreverent language in reference to the complainant in a
petition for TPO and PPO under R.A. 9262, calling her as “only a live-in partner” and presenting her as an
“opportunist” and a “mistress” in an “illegitimate relationship.” Judge Amila even called her a “prostitute,”
and accused her of being motivated by “insatiable greed” and of absconding with the contested
property.81 Such remarks betrayed Judge Amila's prejudices and lack of gender sensitivity.

The enactment of R.A. 9262 aims to address the discrimination brought about by biases and prejudices
against women. As emphasized by the CEDAW Committee on the Elimination of Discrimination against
Women, addressing or correcting discrimination through specific measures focused on women
does not discriminate against men.82 Petitioner's contention,83 therefore, that R.A. 9262 is discriminatory
and that it is an “anti-male,” “husband-bashing,” and “hate-men” law deserves scant consideration. As a
State Party to the CEDAW, the Philippines bound itself to take all appropriate measures “to modify the
social and cultural patterns of conduct of men and women, with a view to achieving the elimination of
prejudices and customary and all other practices which are based on the idea of the inferiority or the
superiority of either of the sexes or on stereotyped roles for men and women.” 84 Justice Puno correctly
pointed out that “(t)he paradigm shift changing the character of domestic violence from a private affair to
a public offense will require the development of a distinct mindset on the part of the police, the
prosecution and the judges.”85

II. The classification is germane to the purpose of the law.

The distinction between men and women is germane to the purpose of R.A. 9262, which is to address
violence committed against women and children, spelled out in its Declaration of Policy, as follows: cralavvonlinelawlibrary

SEC. 2. Declaration of Policy. – It is hereby declared that the State values the dignity of women and
children and guarantees full respect for human rights. The State also recognizes the need to protect the
family and its members particularly women and children, from violence and threats to their personal safety
and security.

Towards this end, the State shall exert efforts to address violence committed against women and children
in keeping with the fundamental freedoms guaranteed under the Constitution and the provisions of the
Universal Declaration of Human Rights, the Convention on the Elimination of All Forms of Discrimination
Against Women, Convention on the Rights of the Child and other international human rights instruments of
which the Philippines is a party.

In 1979, the U.N. General Assembly adopted the CEDAW, which the Philippines ratified on August 5, 1981.
Subsequently, the Optional Protocol to the CEDAW was also ratified by the Philippines on October 6,
2003.86  This Convention mandates that State parties shall accord to women equality with men before the
law87 and shall take all appropriate measures to eliminate discrimination against women in all matters
relating to marriage and family relations on the basis of equality of men and women. 88  The Philippines
likewise ratified the Convention on the Rights of the Child and its two protocols. 89 It is, thus, bound by said
Conventions and their respective protocols.

III. The classification is not limited to existing


conditions only, and apply equally to all members

Moreover, the application of R.A. 9262 is not limited to the existing conditions when it was promulgated,
but to future conditions as well, for as long as the safety and security of women and their children are
threatened by violence and abuse.

R.A. 9262 applies equally to all women and children who suffer violence and abuse. Section 3 thereof
defines VAWC as: cralavvonlinelawlibrary

x x x any act or a series of acts committed by any person against a woman who is his wife, former wife, or
against a woman with whom the person has or had a sexual or dating relationship, or with whom he has a
common child, or against her child whether legitimate or illegitimate, within or without the family abode,
which result in or is likely to result in physical, sexual, psychological harm or suffering, or economic abuse
including threats of such acts, battery, assault, coercion, harassment or arbitrary deprivation of liberty. It
includes, but is not limited to, the following acts: cralavvonlinelawlibrary

A. "Physical Violence" refers to acts that include bodily or physical harm; chanroblesvirtualawlibrary

B. "Sexual violence" refers to an act which is sexual in nature, committed against a woman or her child. It
includes, but is not limited to: cralavvonlinelawlibrary

a) rape, sexual harassment, acts of lasciviousness, treating a woman or her child as a sex object, making
demeaning and sexually suggestive remarks, physically attacking the sexual parts of the victim's body,
forcing her/him to watch obscene publications and indecent shows or forcing the woman or her child to do
indecent acts and/or make films thereof, forcing the wife and mistress/lover to live in the conjugal home
or sleep together in the same room with the abuser; chanroblesvirtualawlibrary

b) acts causing or attempting to cause the victim to engage in any sexual activity by force, threat of force,
physical or other harm or threat of physical or other harm or coercion; chanroblesvirtualawlibrary

c) Prostituting the woman or child.

C. "Psychological violence" refers to acts or omissions causing or likely to cause mental or emotional
suffering of the victim such as but not limited to intimidation, harassment, stalking, damage to property,
public ridicule or humiliation, repeated verbal abuse and mental infidelity. It includes causing or allowing
the victim to witness the physical, sexual or psychological abuse of a member of the family to which the
victim belongs, or to witness pornography in any form or to witness abusive injury to pets or to unlawful
or unwanted deprivation of the right to custody and/or visitation of common children.

D. "Economic abuse" refers to acts that make or attempt to make a woman financially dependent which
includes, but is not limited to the following:

1. withdrawal of financial support or preventing the victim from engaging in any legitimate profession,
occupation, business or activity, except in cases wherein the other spouse/partner objects on valid, serious
and moral grounds as defined in Article 73 of the Family Code; chanroblesvirtualawlibrary

2. deprivation or threat of deprivation of financial resources and the right to the use and enjoyment of the
conjugal, community or property owned in common; chanroblesvirtualawlibrary

3. destroying household property; chanroblesvirtualawlibrary

4. controlling the victims' own money or properties or solely controlling the conjugal money or properties.

It should be stressed that the acts enumerated in the aforequoted provision are attributable to research
that has exposed the dimensions and

dynamics of battery. The acts described here are also found in the U.N. Declaration on the Elimination of
Violence Against Women.90 Hence, the argument advanced by petitioner that the definition of what
constitutes abuse removes the difference between violent action and simple marital tiffs is tenuous.

There is nothing in the definition of VAWC that is vague and ambiguous that will confuse petitioner in his
defense. The acts enumerated above are easily understood and provide adequate contrast between the
innocent and the prohibited acts. They are worded with sufficient definiteness that persons of ordinary
intelligence can understand what conduct is prohibited, and need not guess at its meaning nor differ in its
application.91 Yet, petitioner insists92 that phrases like “depriving or threatening to deprive the woman or
her child of a legal right,” “solely controlling the conjugal or common money or properties,” “marital
infidelity,” and “causing mental or emotional anguish” are so vague that they make every quarrel a case of
spousal abuse. However, we have stressed that the “vagueness” doctrine merely requires a reasonable
degree of certainty for the statute to be upheld – not absolute precision or mathematical exactitude, as
petitioner seems to suggest. Flexibility, rather than meticulous specificity, is permissible as long as the
metes and bounds of the statute are clearly delineated. An act will not be held invalid merely because it
might have been more explicit in its wordings or detailed in its provisions. 93

There is likewise no merit to the contention that R.A. 9262 singles out the husband or father as the culprit.
As defined above, VAWC may likewise be committed “against a woman with whom the person has or had a
sexual or dating relationship.” Clearly, the use of the gender-neutral word “person” who has or had a
sexual or dating relationship with the woman encompasses even lesbian relationships. Moreover, while the
law provides that the offender be related or connected to the victim by marriage, former marriage, or a
sexual or dating relationship, it does not preclude the application of the principle of conspiracy under
the Revised Penal Code (RPC). Thus, in the case of Go-Tan v. Spouses Tan,94 the parents-in-law of Sharica
Mari L. Go- Tan, the victim, were held to be proper respondents in the case filed by the latter upon the
allegation that they and their son (Go-Tan's husband) had community of design and purpose in tormenting
her by giving her insufficient financial support; harassing and pressuring her to be ejected from the family
home; and in repeatedly abusing her verbally, emotionally, mentally and physically.

R.A. 9262 is not violative of the due


process clause of the Constitution. 

Petitioner bewails the disregard of R.A. 9262, specifically in the issuance of POs, of all protections afforded
by the due process clause of the Constitution. Says he: “On the basis of unsubstantiated allegations, and
practically no opportunity to respond, the husband is stripped of family, property, guns, money, children,
job, future employment and reputation, all in a matter of seconds, without an inkling of what happened.” 95

A protection order is an order issued to prevent further acts of violence against women and their
children, their family or household members, and to grant other necessary reliefs. Its purpose is to
safeguard the offended parties from further harm, minimize any disruption in their daily life and facilitate
the opportunity and ability to regain control of their life. 96

“The scope of reliefs in protection orders is broadened to ensure that the victim or offended party is
afforded all the remedies necessary to curtail access by a perpetrator to the victim. This serves to
safeguard the victim from greater risk of violence; to accord the victim and any designated family or
household member safety in the family residence, and to prevent the perpetrator from committing acts
that jeopardize the employment and support of the victim. It also enables the court to award temporary
custody of minor children to protect the children from violence, to prevent their abduction by the
perpetrator and to ensure their financial support.” 97

The rules require that petitions for protection order be in writing, signed and verified by the
petitioner98 thereby undertaking full responsibility, criminal or civil, for every allegation therein. Since
“time is of the essence in cases of VAWC if further violence is to be prevented,” 99 the court is authorized to
issue ex parte a TPO after raffle but before notice and hearing when the life, limb or property of the victim
is in jeopardy and there is reasonable ground to believe that the order is necessary to protect the victim
from the immediate and imminent danger of VAWC or to prevent such violence, which is about to recur. 100

There need not be any fear that the judge may have no rational basis to issue an ex parte order. The
victim is required not only to verify the allegations in the petition, but also to attach her witnesses'
affidavits to the petition.101

The grant of a TPO ex parte cannot, therefore, be challenged as violative of the right to due process. Just
like a writ of preliminary attachment which is issued without notice and hearing because the time in which
the hearing will take could be enough to enable the defendant to abscond or dispose of his property, 102 in
the same way, the victim of VAWC may already have suffered harrowing experiences in the hands of her
tormentor, and possibly even death, if notice and hearing were required before such acts could be
prevented. It is a constitutional commonplace that the ordinary requirements of procedural due process
must yield to the necessities of protecting vital public interests, 103 among which is protection of women
and children from violence and threats to their personal safety and security.

It should be pointed out that when the TPO is issued ex parte, the court shall likewise order that notice be
immediately given to the respondent directing him to file an opposition within five (5) days from service.
Moreover, the court shall order that notice, copies of the petition and TPO be served immediately on the
respondent by the court sheriffs. The TPOs are initially effective for thirty (30) days from service on the
respondent.104

Where no TPO is issued ex parte, the court will nonetheless order the immediate issuance and service of
the notice upon the respondent requiring him to file an opposition to the petition within five (5) days from
service. The date of the preliminary conference and hearing on the merits shall likewise be indicated on
the notice.105

The opposition to the petition which the respondent himself shall verify, must be accompanied by the
affidavits of witnesses and shall show cause why a temporary or permanent protection order should not be
issued.106

It is clear from the foregoing rules that the respondent of a petition for protection order should be apprised
of the charges imputed to him and afforded an opportunity to present his side. Thus, the fear of petitioner
of being “stripped of family, property, guns, money, children, job, future employment and reputation, all in
a matter of seconds, without an inkling of what happened” is a mere product of an overactive imagination.
The essence of due process is to be found in the reasonable opportunity to be heard and submit any
evidence one may have in support of one's defense. "To be heard" does not only mean verbal arguments
in court; one may be heard also through pleadings. Where opportunity to be heard, either through oral
arguments or pleadings, is accorded, there is no denial of procedural due process. 107

It should be recalled that petitioner filed on April 26, 2006 an Opposition to the Urgent Ex-Parte Motion for
Renewal of the TPO that was granted only two days earlier on April 24, 2006. Likewise, on May 23, 2006,
petitioner filed a motion for the modification of the TPO to allow him visitation rights to his children. Still,
the trial court in its Order dated September 26, 2006, gave him five days (5) within which to show cause
why the TPO should not be renewed or extended. Yet, he chose not to file the required comment arguing
that it would just be an “exercise in futility,” conveniently forgetting that the renewal of the questioned
TPO was only for a limited period (30 days) each time, and that he could prevent the continued renewal of
said order if he can show sufficient cause therefor. Having failed to do so, petitioner may not now be heard
to complain that he was denied due process of law.

Petitioner next laments that the removal and exclusion of the respondent in the VAWC case from the
residence of the victim, regardless of ownership of the residence, is virtually a “blank check” issued to the
wife to claim any property as her conjugal home.108

The wording of the pertinent rule, however, does not by any stretch of the imagination suggest that this is
so. It states:
cralavvonlinelawlibrary

SEC. 11. Reliefs available to the offended party. -- The protection order shall include any, some or all of
the following reliefs: cralavvonlinelawlibrary
x x x x

(c) Removing and excluding the respondent from the residence of the offended party, regardless of
ownership of the residence, either temporarily for the purpose of protecting the offended party, or
permanently where no property rights are violated. If the respondent must remove personal effects from
the residence, the court shall direct a law enforcement agent to accompany the respondent to the
residence, remain there until the respondent has gathered his things and escort him from the residence; chanroblesvirtualawlibrary

xxxx

Indubitably, petitioner may be removed and excluded from private respondent's residence, regardless of
ownership, only temporarily for the purpose of protecting the latter. Such removal and exclusion may be
permanent only where no property rights are violated. How then can the private respondent just claim any
property and appropriate it for herself, as petitioner seems to suggest?

The non-referral of a VAWC case to a mediator is justified.

Petitioner argues that “by criminalizing run-of-the-mill arguments, instead of encouraging mediation and
counseling, the law has done violence to the avowed policy of the State to “protect and strengthen the
family as a basic autonomous social institution.” 109

Under Section 23(c) of A.M. No. 04-10-11-SC, the court shall not refer the case or any issue thereof to a
mediator. The reason behind this provision is well-explained by the Commentary on Section 311 of the
Model Code on Domestic and Family Violence as follows: 110

This section prohibits a court from ordering or referring parties to mediation in a proceeding for an order
for protection. Mediation is a process by which parties in equivalent bargaining positions voluntarily reach
consensual agreement about the issue at hand. Violence, however, is not a subject for compromise.
A process which involves parties mediating the issue of violence implies that the victim is somehow at
fault. In addition, mediation of issues in a proceeding for an order of protection is problematic because the
petitioner is frequently unable to participate equally with the person against whom the protection order
has been sought. (Emphasis supplied)

There is no undue delegation of judicial


power to barangay officials. 

Petitioner contends that protection orders involve the exercise of judicial power which, under the
Constitution, is placed upon the “Supreme Court and such other lower courts as may be established by
law” and, thus, protests the delegation of power to barangay officials to issue protection orders. 111 The
pertinent provision reads, as follows: cralavvonlinelawlibrary

SEC. 14. Barangay Protection Orders (BPOs); Who May Issue and How.
– Barangay Protection Orders (BPOs) refer to the protection order issued by the Punong Barangay ordering
the perpetrator to desist from committing acts under Section 5 (a) and (b) of this Act. A  Punong
Barangay who receives applications for a BPO shall issue the protection order to the applicant on the date
of filing after ex parte determination of the basis of the application. If the Punong Barangay is unavailable
to act on the application for a BPO, the application shall be acted upon by any available Barangay
Kagawad. If the BPO is issued by a Barangay Kagawad, the order must be accompanied by an attestation
by the Barangay Kagawad that the Punong Barangay was unavailable at the time of the issuance of the
BPO. BPOs shall be effective for fifteen (15) days. Immediately after the issuance of an  ex parte BPO,
the Punong Barangay or Barangay Kagawad shall personally serve a copy of the same on the respondent,
or direct any barangay official to effect its personal service.

The parties may be accompanied by a non-lawyer advocate in any proceeding before the  Punong
Barangay.

Judicial power includes the duty of the courts of justice to settle actual controversies involving rights which
are legally demandable and enforceable, and to determine whether or not there has been a grave abuse of
discretion amounting to lack or excess of jurisdiction on the part of any branch or instrumentality of the
Government.112 On the other hand, executive power "is generally defined as the power to enforce and
administer the laws. It is the power of carrying the laws into practical operation and enforcing their due
observance."113

As clearly delimited by the aforequoted provision, the BPO issued by the Punong Barangay or, in his
unavailability, by any available Barangay Kagawad, merely orders the perpetrator to desist from (a)
causing physical harm to the woman or her child; and (2) threatening to cause the woman or her child
physical harm. Such function of the Punong Barangay is, thus, purely executive in nature, in pursuance of
his duty under the Local Government Code to “enforce all laws and ordinances,” and to “maintain public
order in the barangay.”114

We have held that “(t)he mere fact that an officer is required by law to inquire into the existence of certain
facts and to apply the law thereto in order to determine what his official conduct shall be and the fact that
these acts may affect private rights do not constitute an exercise of judicial powers.” 115

In the same manner as the public prosecutor ascertains through a preliminary inquiry or proceeding
“whether there is reasonable ground to believe that an offense has been committed and the accused is
probably guilty thereof,” the Punong Barangay must determine reasonable ground to believe that an
imminent danger of violence against the woman and her children exists or is about to recur that would
necessitate the issuance of a BPO. The preliminary investigation conducted by the prosecutor is,
concededly, an executive, not a judicial, function. The same holds true with the issuance of a BPO.

We need not even belabor the issue raised by petitioner that since barangay officials and other law
enforcement agencies are required to extend assistance to victims of violence and abuse, it would be very
unlikely that they would remain objective and impartial, and that the chances of acquittal are nil. As
already stated, assistance by barangay officials and other law enforcement agencies is consistent with
their duty to enforce the law and to maintain peace and order.

Conclusion

Before a statute or its provisions duly challenged are voided, an unequivocal breach otor a clear conflict
with the Constitution, not merely a doubtful or argumentative one, must be demonstrated in such a
manner as to leave no doubt in the mind of the Court. In other words, the grounds for nullity must be
beyond reasonable doubt. 116 In the instant case, however, no concrete evidence and convincing
arguments were presented by petitioner to warrant a declaration of the unconstitutionality of R.A. 9262,
which is an act of Congress and signed into law by the highest officer of the co-equal executive
department. As we said in Estrada v. Sandiganbayan,117 courts must assume that the legislature is ever
conscious of the borders and edges of its plenary powers, and passed laws with full knowledge of the facts
and for the purpose of promoting what is right and advancing the welfare of the majority.

We reiterate here Justice Puno's observation that "the history of the women's movement against domestic
violence shows that one of its most difficult struggles was the fight against the violence oflaw itself. If we
keep that in mind, law will not again be a hindrance to the struggle of women for equality but will be its
fulfillment."118 Accordingly, the constitutionality of R.A. 9262 is, as it should be, sustained.

WHEREFORE, the instant petition for review on certiorari is hereby DENIED for lack of merit.

SO ORDERED

- Racho v Tanaka, GR 199515, Jun 25, 2018

Judicial recognition of a foreign divorce requires that the national law of the foreign spouse and the divorce
decree be pleaded and proved as a fact before the Regional Trial Court. The Filipino spouse may be
granted the capacity to remarry once our courts find that the foreign divorce was validly obtained by the
foreign spouse according to his or her national law, and that the foreign spouse's national law considers
the dissolution of the marital relationship to be absolute.

This is a Petition for Review on Certiorari 1 assailing the June 2, 2011 Decision 2 and October 3, 2011
Order3 of Branch 254, Regional Trial Court, Las Piñas City, which denied Rhodora Ilumin Racho's (Racho)
Petition for Judicial Determination and Declaration of Capacity to Marry. 4 The denial was on the ground
that a Certificate of Divorce issued by the Japanese Embassy was insufficient to prove the existence of a
divorce decree.

Racho and Seiichi Tanaka (Tanaka) were married on April 20, 2001 in Las Piñas City, Metro Manila. They
lived together for nine (9) years in Saitama Prefecture, Japan and did not have any children. 5

Racho alleged that on December 16, 2009, Tanaka filed for divorce and the divorce was granted. She
secured a Divorce Certificate6 issued by Consul Kenichiro Takayama (Consul Takayama) of the Japanese
Consulate in the Philippines and had it authenticated 7 by an authentication officer of the Department of
Foreign Affairs.8

She filed the Divorce Certificate with the Philippine Consulate General in Tokyo, Japan, where she was
informed that by reason of certain administrative changes, she was required to return to the Philippines to
report the documents for registration and to file the appropriate case for judicial recognition of divorce. 9

She tried to have the Divorce Certificate registered with the Civil Registry of Manila but was refused by the
City Registrar since there was no court order recognizing it. When she went to the Department of Foreign
Affairs to renew her passport, she was likewise told that she needed the proper court order. She was also
informed by the National Statistics Office that her divorce could only be annotated in the Certificate of
Marriage if there was a court order capacitating her to remarry. 10

She went to the Japanese Embassy, as advised by her lawyer, and secured a Japanese Law English
Version of the Civil Code of Japan, 2000 Edition. 11

On May 19, 2010, she filed a Petition for Judicial Determination and Declaration of Capacity to Marry 12 with
the Regional Trial Court, Las Piñas City.

On June 2, 2011, Branch 254, Regional Trial Court, Las Piñas City rendered a Decision, 13 finding that
Racho failed to prove that Tanaka legally obtained a divorce. It stated that while she was able to prove
Tanaka's national law, the Divorce Certificate was not competent evidence since it was not the divorce
decree itself.14

Racho filed a Motion for Reconsideration, 15 arguing that under Japanese law, a divorce by agreement
becomes effective by oral notification, or by a document signed by both parties and by two (2) or more
witnesses.16

In an Order17 dated October 3, 2011, the Regional Trial Court denied the Motion, finding that Racho failed
to present the notification of divorce and its acceptance. 18

On December 19, 2011, Racho filed a Petition for Review on Certiorari 19 with this Court. In its January 18,
2012 Resolution, this Court deferred action on her Petition pending her submission of a duly authenticated
acceptance certificate of the notification of divorce. 20

Petitioner initially submitted a Manifestation, 21 stating that a duly-authenticated acceptance certificate was
not among the documents presented at the Regional Trial Court because of its unavailability to petitioner
during trial. She also pointed out that the Divorce Certificate issued by ,the Consulate General of the
Japanese Embassy was sufficient proof of the fact of divorce. 22 She also manifested that Tanaka had
secured a marriage license on the basis of the same Divorce Certificate and had already remarried another
Filipino. Nevertheless, she has endeavored to secure the document as directed by this Court. 23

On March 16, 2012, petitioner submitted her Compliance, 24 attaching a duly authenticated Certificate of
Acceptance of the Report of Divorce that she obtained in Japan. 25 The Office of the Solicitor General
thereafter submitted its Comment26 on the Petition, to which petitioner submitted her Reply. 27

Petitioner argues that under the Civil Code of Japan, a divorce by agreement becomes effective upon
notification, whether oral or written, by both parties and by two (2) or more witnesses. She contends that
the Divorce Certificate stating "Acceptance Certification of Notification of Divorce issued by the Mayor of
Fukaya City, Saitama Pref., Japan on December 16, 2009" is sufficient to prove that she and her husband
have divorced by agreement and have already effected notification of the divorce. 28

She avers further that under Japanese law, the manner of proving a divorce by agreement is by record of
its notification and by the fact of its acceptance, both of which were stated in the Divorce Certificate. She
maintains that the Divorce Certificate is signed by Consul Takayama, whom the Department of Foreign
Affairs certified as duly appointed and qualified to sign the document. She also states that the Divorce
Certificate has already been filed and recorded with the Civil Registry Office of Manila. 29

She insists that she is now legally capacitated to marry since Article 728 of the Civil Code of Japan states
that a matrimonial relationship is terminated by divorce. 30

On the other hand, the Office of the Solicitor General posits that the Certificate of Divorce has no probative
value since it was not properly authenticated under Rule 132, Section 24 31 of the Rules of Court. However,
it states that it has no objection to the admission of the Certificate of Acceptance of the Report of Divorce
submitted by petitioner in compliance with this Court's January 18, 2012 Resolution. 32

It likewise points out that petitioner never mentioned that she and her husband obtained a divorce by
agreement and only mentioned it in her motion for reconsideration before the Regional Trial Court. Thus,
petitioner failed to prove that she is now capacitated to marry since her divorce was not obtained by the
alien spouse. She also failed to point to a specific provision in the Civil Code of Japan that allows persons
who obtained a divorce by agreement the capacity to remarry. In any case, a divorce by agreement is not
the divorce contemplated in Article 26 of the Family Code. 33

In rebuttal, petitioner insists that all her evidence, including the Divorce Certificate, was formally offered
and held to be admissible as evidence by the Regional Trial Court. 34 She also argues that the Office of the
Solicitor General should not have concluded that the law does not contemplate divorce by agreement or
consensual divorce since a discriminatory situation will arise if this type of divorce is not recognized. 35

The issue in this case, initially, was whether or not the Regional Trial Court erred in dismissing the Petition
for Declaration of Capacity to Marry for insufficiency of evidence. After the submission of Comment,
however, the issue has evolved to whether or not the Certificate of Acceptance of the Report of Divorce is
sufficient to prove the fact that a divorce between petitioner Rhodora Ilumin Racho and respondent Seiichi
Tanaka was validly obtained by the latter according to his national law.

Under Article 26 of the Family Code, a divorce between a foreigner and a Filipino may be recognized in the
Philippines as long as it was validly obtained according to the foreign spouse's national law, thus:

Article 26. All marriages solemnized outside the Philippines in accordance with the laws in force in the
country where they were solemnized, and valid there as such, shall also be valid in this country, except
those prohibited under Articles 35 (1), (4), (5) and (6), 36, 37 and 38.

Where a marriage between a Filipino citizen and a foreigner is validly celebrated and a divorce is
thereafter validly obtained abroad by the alien spouse capacitating him or her to remarry, the Filipino
spouse shall have capacity to remarry under Philippine law.36 (Emphasis supplied)
The second paragraph was included to avoid an absurd situation where a Filipino spouse remains married
to the foreign spouse even after a validly obtained divorce abroad. 37 The addition of the second paragraph
gives the Filipino spouse a substantive right to have the marriage considered as dissolved, and ultimately,
to grant him or her the capacity to remarry.38

Article 26 of the Family Code is applicable only in issues on the validity of remarriage. It cannot be the
basis for any other liability, whether civil or criminal, that the Filipino spouse may incur due to remarriage.

Mere presentation of the divorce decree before a trial court is insufficient. 39 In Garcia v. Recio,40 this Court
established the principle that before a foreign divorce decree is recognized in this jurisdiction, a separate
action must be instituted for that purpose. Courts do not take judicial notice of foreign laws and foreign
judgments; thus, our laws require that the divorce decree and the national law of the foreign spouse must
be pleaded and proved like any other fact before trial courts. 41 Hence, in Corpuz v. Sto. Tomas:42

The starting point in any recognition of a foreign divorce judgment is the acknowledgment that our courts
do not take judicial notice of foreign judgments and laws. Justice Herrera explained that, as a rule, "no
sovereign is bound to give effect within its dominion to a judgment rendered by a tribunal of another
country." This means that the foreign judgment and its authenticity must be proven as facts under our
rules on evidence, together with the alien's applicable national law to show the effect of the judgment on
the alien himself or herself. The recognition may be made in an action instituted specifically for the
purpose or in another action where a party invokes the foreign decree as an integral aspect of his claim or
defense.43
II

Respondent's national law was duly admitted by the Regional Trial Court. Petitioner presented "a copy [of]
the English Version of the Civil Code of Japan (Exh. "K") translated under the authorization of the Ministry
of Justice and the Code of Translation Committee." 44 Article 728(1) of the Civil Code of Japan reads:

Article 728. 1. The matrimonial relationship is terminated by divorce. 45


To prove the fact of divorce, petitioner presented the Divorce Certificate issued by Consul Takayama of
Japan on January 18, 2010, which stated in part:

This is to certify that the above statement has been made on the basis of the Acceptance Certification of
Notification of Divorce issued by the Mayor of Fukaya City, Saitama Pref., Japan on December 16, 2009. 46
This Certificate only certified that the divorce decree, or the Acceptance Certification of Notification of
Divorce, exists. It is not the divorce decree itself. The Regional Trial Court further clarified:

[T]he Civil Law of Japan recognizes two (2) types of divorce, namely: (1) judicial divorce and (2) divorce
by agreement.

Under the same law, the divorce by agreement becomes effective by notification, orally or in a document
signed by both parties and two or more witnesses of full age, in accordance with the provisions of Family
Registration Law of Japan.47
Thus, while respondent's national law was duly admitted, petitioner failed to present sufficient evidence
before the Regional Trial Court that a divorce was validly obtained according to the national law of her
foreign spouse. The Regional Trial Court would not have erred in dismissing her Petition.
III

Upon appeal to this Court, however, petitioner submitted a Certificate of Acceptance of the Report of
Divorce,48 certifying that the divorce issued by Susumu Kojima, Mayor of Fukaya City, Saitama Prefecture,
has been accepted on December 16, 2009. The seal on the document was authenticated by Kazutoyo
Oyabe, Consular Service Division, Ministry of Foreign Affairs, Japan. 49

The probative value of the Certificate of Acceptance of the Report of Divorce is a question of fact that
would not ordinarily be within this Court's ambit to resolve. Issues in a petition for review on certiorari
under Rule 45 of the Rules of Court50 are limited to questions of law.

In Garcia and Corpuz, this Court remanded the cases to the Regional Trial Courts for the reception of
evidence and for further proceedings.51 More recently in Medina v. Koike,52 this Court remanded the case
to the Court of Appeals to determine the national law of the foreign spouse:

Well entrenched is the rule that this Court is not a trier of facts. The resolution of factual issues is the
function of the lower courts, whose findings on these matters are received with respect and are in fact
binding subject to certain exceptions. In this regard, it is settled that appeals taken from judgments or
final orders rendered by RTC in the exercise of its original jurisdiction raising questions of fact or mixed
questions of fact and law should be brought to the Court of Appeals (CA) in accordance with Rule 41 of the
Rules of Court.

Nonetheless, despite the procedural restrictions on Rule 45 appeals as above-adverted, the Court may
refer the case to the CA under paragraph 2, Section 6 of Rule 56 of the Rules of Court, which provides:
SEC. 6. Disposition of improper appeal. - . . .

An appeal by certiorari taken to the Supreme Court from the Regional Trial Court submitting issues of fact
may be referred to the Court of Appeals for decision or appropriate action. The determination of the
Supreme Court on whether or not issues of fact are involved shall be final. 53
The court records, however, are already sufficient to fully resolve the factual issues. 54 Additionally, the
Office of the Solicitor General neither posed any objection to the admission of the Certificate of Acceptance
of the Report of Divorce55 nor argued that the Petition presented questions of fact. In the interest of
judicial economy and efficiency, this Court shall resolve this case on its merits.

IV

Under Rule 132, Section 24 of the Rules of Court, the admissibility of official records that are kept in a
foreign country requires that it must be accompanied by a certificate from a secretary of an embassy or
legation, consul general, consul, vice consul, consular agent or any officer of the foreign service of the
Philippines stationed in that foreign country:

Section 24. Proof of official record. - The record of public documents referred to in paragraph (a) of
Section 19, when admissible for any purpose, may be evidenced by an official publication thereof or by a
copy attested by the officer having the legal custody of the record, or by his deputy, and accompanied, if
the record is not kept in the Philippines, with a certificate that such officer has the custody. If the office in
which the record is kept is in a foreign country, the certificate may be made by a secretary of the embassy
or legation, consul general, consul, vice consul, or consular agent or by any officer in the foreign service of
the Philippines stationed in the foreign country in which the record is kept, and authenticated by the seal
of his office.
The Certificate of Acceptance of the Report of Divorce was accompanied by an Authentication 56 issued by
Consul Bryan Dexter B. Lao of the Embassy of the Philippines in Tokyo, Japan, certifying that Kazutoyo
Oyabe, Consular Service Division, Ministry of Foreign Affairs, Japan was an official in and for Japan. The
Authentication further certified that he was authorized to sign the Certificate of Acceptance of the Report
of Divorce and that his signature in it was genuine. Applying Rule 132, Section 24, the Certificate of
Acceptance of the Report of Divorce is admissible as evidence of the fact of divorce between petitioner and
respondent.

The Regional Trial Court established that according to the national law of Japan, a divorce by agreement
"becomes effective by notification." 57 Considering that the Certificate of Acceptance of the Report of
Divorce was duly authenticated, the divorce between petitioner and respondent was validly obtained
according to respondent's national law.

V
The Office of the Solicitor General, however, posits that divorce by agreement is not the divorce
contemplated in Article 26 of the Family Code, which provides:

Article 26. All marriages solemnized outside the Philippines in accordance with the laws in force in the
country where they were solemnized, and valid there as such, shall also be valid in this country, except
those prohibited under Articles 35 (1), (4), (5) and (6), 36, 37 and 38.

Where a marriage between a Filipino citizen and a foreigner is validly celebrated and a divorce is
thereafter validly obtained abroad by the alien spouse capacitating him or her to remarry, the Filipino
spouse shall have capacity to remarry under Philippine law.58 (Emphasis supplied)
Considering that Article 26 states that divorce must be "validly obtained abroad by the alien spouse," the
Office of the Solicitor General posits that only the foreign spouse may initiate divorce proceedings.

In a study on foreign marriages in 2007 conducted by the Philippine Statistics Authority, it was found that
"marriages between Filipino brides and foreign grooms comprised 5,537 or 66.7 percent while those
between Filipino grooms and foreign brides numbered 152 or 1.8 percent of the total marriages outside
the country."59 It also found that "[a]bout four in every ten interracial marriages (2,916 or 35.1%) were
between Filipino brides and Japanese grooms." Statistics for foreign marriages in 2016 shows that there
were 1,129 marriages between Filipino men and foreign women but 8,314 marriages between Filipina
women and foreign men.60 Thus, empirical data demonstrates that Filipino women are more likely to enter
into mixed marriages than Filipino men. Under Philippine laws relating to mixed marriages, Filipino women
are twice marginalized.

In this particular instance, it is the Filipina spouse who bears the burden of this narrow interpretation,
which may be unconstitutional. Article II, Section 14 of our Constitution provides:

Section 14. The State recognizes the role of women in nation-building, and shall ensure the fundamental
equality before the law of women and men.
This constitutional provision provides a more active application than the passive orientation of Article III,
Section 1 of the Constitution does, which simply states that no person shall "be denied the equal
protection of the laws." Equal protection, within the context of Article III, Section 1 only provides that any
legal burden or benefit that is given to men must also be given to women. It does not require the State to
actively pursue "affirmative ways and means to battle the patriarchy-that complex of political, cultural,
and economic factors that ensure women's disempowerment." 61

In 1980, our country became a signatory to the Convention on the Elimination of All Forms of
Discrimination Against Women (CEDAW). 62 Under Articles 2(f) and S(a) of the treaty, the Philippines as a
state party, is required:

Article 2

. . . .

(f) to take all appropriate measures, including legislation, to modify or abolish existing laws, regulations,
customs and practices which constitute discrimination against women;

. . . .

Article 5

. . . .

(a) To modify the social and cultural patterns of conduct of men and women, with a view to achieving the
elimination of prejudices and customary and all other practices which are based on the idea of the
inferiority or the superiority of either of the sexes or on stereotyped roles for men and women[.]
By enacting the Constitution and signing on the CEDAW, the State has committed to ensure and to
promote gender equality.

In 2009, Congress enacted Republic Act No. 9710 or the Magna Carta for Women, which provides that the
State "shall take all appropriate measures to eliminate discrimination against women in all matters relating
to marriage and family relations."63 This necessarily includes the second paragraph of Article 26 of the
Family Code. Thus, Article 26 should be interpreted to mean that it is irrelevant for courts to determine if it
is the foreign spouse that procures the divorce abroad. Once a divorce decree is issued, the divorce
becomes "validly obtained" and capacitates the foreign spouse to marry. The same status should be given
to the Filipino spouse.

The national law of Japan does not prohibit the Filipino spouse from initiating or participating in the divorce
proceedings. It would be inherently unjust for a Filipino woman to be prohibited by her own national laws
from something that a foreign law may allow. Parenthetically, the prohibition on Filipinos from participating
in divorce proceedings will not be protecting our own nationals.

The Solicitor General's narrow interpretation of Article 26 disregards any agency on the part of the Filipino
spouse. It presumes that the Filipino spouse is incapable of agreeing to the dissolution of the marital bond.
It perpetuates the notion that all divorce proceedings are protracted litigations fraught with bitterness and
drama. Some marriages can end amicably, without the parties harboring any ill will against each other.
The parties could forgo costly court proceedings and opt for, if the national law of the foreign spouse
allows it, a more convenient out-of-court divorce process. This ensures amity between the former spouses,
a friendly atmosphere for the children and extended families, and less financial burden for the family.

Absolute divorce was prohibited in our jurisdiction only in the mid-20 th century. The Philippines had divorce
laws in the past. In 1917, Act No. 271064 was enacted which allowed a wife to file for divorce in cases of
concubinage or a husband to file in cases of adultery.65

Executive Order No. 141, or the New Divorce Law, which was enacted during the Japanese occupation,
provided for 11 grounds for divorce, including "intentional or unjustified desertion continuously for at least
one year prior to the filing of [a petition for divorce]" and "slander by deed or gross insult by one spouse
against the other to such an extent as to make further living together impracticable." 66

At the end of World War II, Executive Order No. 141 was declared void and Act No. 2710 again took
effect.67 It was only until the enactment of the Civil Code in 1950 that absolute divorce was prohibited in
our jurisdiction.

It is unfortunate that legislation from the past appears to be more progressive than current enactments.
Our laws should never be intended to put Filipinos at a disadvantage. Considering that the Constitution
guarantees fundamental equality, this Court should not tolerate an unfeeling and callous interpretation of
laws. To rule that the foreign spouse may remarry, while the Filipino may not, only contributes to the
patriarchy. This interpretation encourages unequal partnerships and perpetuates abuse m intimate
relationships.68

In any case, the Solicitor General's argument has already been resolved in Republic v. Manalo,69 where
this Court held:

Paragraph 2 of Article 26 speaks of "a divorce . . . validly obtained abroad by the alien spouse capacitating
him or her to remarry." Based on a clear and plain reading of the provision, it only requires that there be a
divorce validly obtained abroad. The letter of the law does not demand that the alien spouse should be the
one who initiated the proceeding wherein the divorce decree was granted. It does not distinguish whether
the Filipino spouse is the petitioner or the respondent in the foreign divorce proceeding. The Court is
bound by the words of the statute; neither can We put words in the mouths of the lawmakers. "The
legislature is presumed to know the meaning of the words, to have used words advisedly, and to have
expressed its intent by the use of such words as are found in the statute.  Verba legis non est recedendum,
or from the words of a statute there should be no departure."

Assuming, for the sake of argument, that the word "obtained" should be interpreted to mean that the
divorce proceeding must be actually initiated by the alien spouse, still, the Court will not follow the letter
of the statute when to do so would depart from the true intent of the legislature or would otherwise yield
conclusions inconsistent with the general purpose of the act. Laws have ends to achieve, and statutes
should be so construed as not to defeat but to carry out such ends and purposes. As held in  League of
Cities of the Phils., et al. v. COMELEC, et al.:
The legislative intent is not at all times accurately reflected in the manner in which the resulting law is
couched. Thus, applying a verba legis or strictly literal interpretation of a statute may render it
meaningless and lead to inconvenience, an absurd situation or injustice. To obviate this aberration, and
bearing in mind the principle that the intent or the spirit of the law is the law itself, resort should be to the
rule that the spirit of the law controls its letter.
To reiterate, the purpose of Paragraph 2 of Article 26 is to avoid the absurd situation where the Filipino
spouse remains married to the alien spouse who, after a foreign divorce decree that is effective in the
country where it was rendered, is no longer married to the Filipino spouse. The provision is a corrective
measure to address an anomaly where the Filipino spouse is tied to the marriage while the foreign spouse
is free to marry under the laws of his or her country. Whether the Filipino spouse initiated the foreign
divorce proceeding or not, a favorable decree dissolving the marriage bond and capacitating his or her
alien spouse to remarry will have the same result: the Filipino spouse will effectively be without a husband
or wife. A Filipino who initiated a foreign divorce proceeding is in the same place and in like circumstance
as a Filipino who is at the receiving end of an alien initiated proceeding. Therefore, the subject provision
should not make a distinction. In both instance, it is extended as a means to recognize the residual effect
of the foreign divorce decree on Filipinos whose marital ties to their alien spouses are severed by operation
of the latter's national law.70 (Emphasis in the original)
Recent jurisprudence, therefore, holds that a foreign divorce may be recognized in this jurisdiction as long
as it is validly obtained, regardless of who among the spouses initiated the divorce proceedings.

The question in this case, therefore, is not who among the spouses initiated the proceedings but rather if
the divorce obtained by petitioner and respondent was valid.

The Regional Trial Court found that there were two (2) kinds of divorce in Japan: judicial divorce and
divorce by agreement. Petitioner and respondent's divorce was considered as a divorce by agreement,
which is a valid divorce according to Japan's national law. 71

The Office of the Solicitor General likewise posits that while petitioner was able to prove that the national
law of Japan allows absolute divorce, she was unable to "point to a specific provision of the Japan[ese]
Civil Code which states that both judicial divorce and divorce by agreement will allow the spouses to
remarry."72

To prove its argument, the Office of the Solicitor General cites Republic v. Orbecido III,73 where this Court
stated:

[R]espondent must also show that the divorce decree allows his former wife to remarry as specifically
required in Article 26. Otherwise, there would be no evidence sufficient to declare that he is capacitated to
enter into another marriage.

Nevertheless, we are unanimous in our holding that Paragraph 2 of Article 26 of the Family Code (E.O. No.
209, as amended by E.O. No. 227), should be interpreted to allow a Filipino citizen, who has been divorced
by a spouse who had acquired foreign citizenship and remarried, also to remarry. However, considering
that in the present petition there is no sufficient evidence submitted and on record, we are unable to
declare, based on respondent's bare allegations that his wife, who was naturalized as an American citizen,
had obtained a divorce decree and had remarried an American, that respondent is now capacitated to
remarry. Such declaration could only be made properly upon respondent's submission of the aforecited
evidence in his favor.74
The Office of the Solicitor General pointedly ignores that in Orbecido III, the respondent in that case
neither pleaded and proved that his wife had been naturalized as an American citizen, nor presented any
evidence of the national law of his alleged foreign spouse that would allow absolute divorce.

In this case, respondent's nationality was not questioned. The Regional Trial Court duly admitted
petitioner's presentation of respondent's national law. Article 728 of the Civil Code of Japan as quoted by
the Office of the Solicitor General states:

Article 728 of the Japan Civil Code reads:

1. The matrimonial relationship is terminated by divorce.

2. The same shall apply also if after the death of either husband or wife, the surviving spouse declares his
or her intention to terminate the matrimonial relationship. 75
The wording of the provision is absolute. The provision contains no other qualifications that could limit
either spouse's capacity to remarry.

In Garcia v. Recio,76 this Court reversed the Regional Trial Court's finding of the Filipino spouse's capacity
to remarry since the national law of the foreign spouse stated certain conditions before the divorce could
be considered absolute:

In its strict legal sense, divorce means the legal dissolution of a lawful union for a cause arising after
marriage. But divorces are of different types. The two basic ones are (1) absolute divorce or a  vinculo
matrimonii and (2) limited divorce or a mensa et thoro. The first kind terminates the marriage, while the
second suspends it and leaves the bond in full force. There is no showing in the case at bar which type of
divorce was procured by respondent.

Respondent presented a decree nisi or an interlocutory decree - a conditional or provisional judgment of


divorce. It is in effect the same as a separation from bed and board, although an absolute divorce may
follow after the lapse of the prescribed period during which no reconciliation is effected.

Even after the divorce becomes absolute, the court may under some foreign statutes and practices, still
restrict remarriage. Under some other jurisdictions, remarriage may be limited by statute; thus, the guilty
party in a divorce which was granted on the ground of adultery may be prohibited from marrying again.
The court may allow a remarriage only after proof of good behavior.

On its face, the herein Australian divorce decree contains a restriction that reads:
"1. A party to a marriage who marries again before this decree becomes absolute (unless the other party
has died) commits the offence of bigamy."
This quotation bolsters our contention that the divorce obtained by respondent may have been restricted.
It did not absolutely establish his legal capacity to remarry according to his national law. Hence, we find no
basis for the ruling of the trial court, which erroneously assumed that the Australian divorce ipso
facto restored respondent's capacity to remarry despite the paucity of evidence on this matter. 77
Here, the national law of the foreign spouse states that the matrimonial relationship is terminated by
divorce. The Certificate of Acceptance of the Report of Divorce does not state any qualifications that would
restrict the remarriage of any of the parties. There can be no other interpretation than that the divorce
procured by petitioner and respondent is absolute and completely terminates their marital tie.

Even under our laws, the effect of the absolute dissolution of the marital tie is to grant both parties the
legal capacity to remarry. Thus, Article 40 of the Family Code provides:

Article 40. The absolute nullity of a previous marriage may be invoked for purposes of remarriage on the
basis solely of a final judgment declaring such previous marriage void.
Petitioner alleges that respondent has since remarried, the National Statistics Office having found no
impediment to the registration of his Marriage Certificate. 78 The validity of respondent's subsequent
marriage is irrelevant for the resolution of the issues in this case. The existence of respondent's Marriage
Certificate, however, only serves to highlight the absurd situation sought to be prevented in the 1985 case
of Van Dorn v. Romillo, Jr.:79

It is true that owing to the nationality principle embodied in Article 15 of the Civil Code, only Philippine
nationals are covered by the policy against absolute divorces the same being considered contrary to our
concept of public policy and morality. However, aliens may obtain divorces abroad, which may be
recognized in the Philippines, provided they are valid according to their national law. In this case, the
divorce in Nevada released private respondent from the marriage from the standards of American law,
under which divorce dissolves the marriage. . . .

. . . .

Thus, pursuant to his national law, private respondent is no longer the husband of petitioner. He would
have no standing to sue in the case below as petitioner's husband entitled to exercise control over
conjugal assets. As he is bound by the Decision of his own country's Court, which validly exercised
jurisdiction over him, and whose decision he does not repudiate, he is estopped by his own representation
before said Court from asserting his right over the alleged conjugal property.

To maintain, as private respondent does, that, under our laws, petitioner has to be considered still married
to private respondent and still subject to a wife's obligations under Article 109, et. seq. of the Civil Code
cannot be just. Petitioner should not be obliged to live together with, observe respect and fidelity, and
render support to private respondent. The latter should not continue to be one of her heirs with possible
rights to conjugal property. She should not be discriminated against in her own country if the ends of
justice are to be served.80
The ruling in Van Dorn was eventually codified in the second paragraph of Article 26 of the Family Code
through the issuance of Executive Order No. 227 in 1987. The grant of substantive equal rights to the
Filipino spouse was broad enough that this Court, in the 1985 case of Quita v. Court of Appeals,81 "hinted,
by way of obiter dictum"82 that it could be applied to Filipinos who have since been naturalized as foreign
citizens.

In Republic v. Orbecido III,83 this Court noted the obiter in Quita and stated outright that Filipino citizens
who later become naturalized as foreign citizens may validly obtain a divorce from their Filipino spouses:

Thus, taking into consideration the legislative intent and applying the rule of reason, we hold that
Paragraph 2 of Article 26 should be interpreted to include cases involving parties who, at the time of the
celebration of the marriage were Filipino citizens, but later on, one of them becomes naturalized as a
foreign citizen and obtains a divorce decree. The Filipino spouse should likewise be allowed to remarry as if
the other party were a foreigner at the time of the solemnization of the marriage. To rule otherwise would
be to sanction absurdity and injustice. Where the interpretation of a statute according to its exact and
literal import would lead to mischievous results or contravene the clear purpose of the legislature, it should
be construed according to its spirit and reason, disregarding as far as necessary the letter of the law. A
statute may therefore be extended to cases not within the literal meaning of its terms, so long as they
come within its spirit or intent. 84
To insist, as the Office of the Solicitor General does, that under our laws, petitioner is still married to
respondent despite the latter's newfound companionship with another cannot be just. 85 Justice is better
served if she is not discriminated against in her own country. 86 As much as petitioner is free to seek
fulfillment in the love and devotion of another, so should she be free to pledge her commitment within the
institution of marriage.

WHEREFORE, the Petition is GRANTED. The Regional Trial Court June 2, 2011 Decision and October 3,
2011 Order in SP. Proc. No. 10-0032 are REVERSED and SET ASIDE. By virtue of Article 26, second
paragraph of the Family Code and the Certificate of Acceptance of the Report of Divorce dated December
16, 2009, petitioner Rhodora Ilumin Racho is declared capacitated to remarry.

SO ORDERED.

Protection to labor, Article II, Section 18; Article XIII, Section 3


-Hubilla v HSY Marketing, Ltd., GR 207254, Jan 10, 2018

When the evidence in labor cases is in equipoise, doubt is resolved in favor of the employee.

This is a Petition for Review on Certiorari  assailing the February 25, 2013 Decision  and May 30, 2013 Resolution  of the
1 2 3

Court of Appeals in CA-G.R. SP No. 126522, which upheld the Labor Arbiter's finding that the employees voluntarily
terminated their employment. The assailed judgments also set aside the National Labor Relations Commission's
application of the principle of equipoise on the ground that the employees failed to present any evidence in their favor.

HSY Marketing Ltd., Co., Wantofree Oriental Trading, Inc., Coen Fashion House and General Merchandise, Asia
Consumer Value Trading, Inc., Fabulous Jeans & Shirt & General Merchandise, LSG Manufacturing Corporation, Unite
General Merchandise, Rosario Q. Co, Lucia Pun Lin Yeung, and Alexander Arqueza (respondents) are engaged in
manufacturing and selling goods under the brand Novo Jeans & Shirt & General Merchandise (Novo Jeans). 4

Sometime in May 2010 and June 2010, several Novo Jeans employees  went to Raffy Tulfo's radio program to air their
5

grievances against their employers for alleged labor violations. They were referred to the Department of Labor and
Employment Camanava Regional Office.  6

These employees claimed that on June 7, 2010, they were not allowed to enter the Novo Jeans branches they were
employed in. They further averred that while Novo Jeans sent them a show cause letter the next day, they were in truth
already dismissed from employment. They sent a demand letter on July 19, 2010 to amicably settle the case before the
Department of Labor and Employment but no settlement was reached. They alleged that upon learning that the
Department of Labor and Employment was not the proper forum to address their grievances, they decided to file a notice
of withdrawal and file their complaint with the Labor Arbiter. 
7

On the other hand, Novo Jeans claimed that these employees voluntarily severed their employment but that they filed
complaints later with the Department of Labor and Employment. They alleged that the employees' notice of withdrawal
was not actually granted by the Department of Labor and Employment but that the employees nonetheless filed their
complaints before the Labor Arbiter.  8

On May 31, 2011, Labor Arbiter Arden S. Anni rendered a Decision  dismissing the complaints. He found that other than
9

the employees' bare allegations that they were dismissed from June 6 to 9, 2010, they did not present any other evidence
showing that their employment was terminated or that they were prevented from reporting for work.   The Labor Arbiter
10

likewise ruled that the employees voluntarily severed their employment since the airing of their grievances on Raffy Tulfo's
radio program "[was] enough reason for them not to report for work, simply because of a possible disciplinary action by
[Novo Jeans]."   The dispositive portion of the Labor Arbiter Decision read:
11

WHEREFORE, PREMISES CONSIDERED, judgment is hereby rendered DISMISSING the above-captioned consolidated
cases for utter lack of merit and for forum-shopping.

SO ORDERED. 12

The employees appealed to the National Labor Relations Commission. 13

On June 25, 2012, the National Labor Relations Commission rendered a Decision   reversing that of the Labor Arbiter and
14

finding that the employees were illegally dismissed. It ruled that the allegations of both parties "were unsubstantiated and
thus [were] equipoised" and that "if doubt exists between the evidence presented by the employer and that by the
employee, the scales of justice must be tilted in favor of the latter."  The dispositive portion of the National Labor Relations
15

Commission Decision read:

WHEREFORE, premises considered, judgment is hereby rendered finding the appeal meritorious with respect to the issue
of illegal dismissal. Complainants-appellants' respective employers are hereby found liable, jointly and severally, to pay
complainants-appellants their backwages and separation pay plus ten percent thereof as attorney's fees. Accordingly, the
decision of the Labor Arbiter dated May 31, 2011 is hereby MODIFIED. All other dispositions STANDS (sic) undisturbed.

The computation of the aforesaid awards is as follows:

....

TOTAL AWARD Php30,969,426.00

SO ORDERED. 16
Novo Jeans moved for partial reconsideration  but was denied by the National Labor Relations Commission in its August
17

24, 2012 Resolution.  Thus, it filed a Petition for Certiorari   with the Court of Appeals.
18 19

On February 25, 2013, the Court of Appeals rendered a Decision  reversing the Decision of the National Labor Relations
20

Commission and reinstating the Labor Arbiter Decision. The Court of Appeals found that Novo Jeans' counsel, as the
affiant, substantially complied with the verification requirement even if his personal knowledge was based on facts relayed
to him by his clients and on authentic records since he was not privy to the antecedents of the case. 21

The Court of Appeals stated that while the employees merely alleged that they were no longer allowed to report to work on
a particular day, Novo Jeans was able to present the First Notice of Termination of Employment sent to them, asking them
to explain their sudden absence from work without proper authorization. It likewise found that the Notices of Termination of
Employment (Notices) did not indicate that the employees were dismissed or that they were prevented from entering the
stores. 
22

According to the Court of Appeals, the equipoise rule was inapplicable in this case since it only applied when the evidence
between the parties was equally balanced. Considering that only Novo Jeans was able to present proof of its claims, the
Court of Appeals was inclined to rule in its favor.   Thus, the Court of Appeals concluded that the case involved voluntary
23

termination of employment, not illegal dismissal.  The dispositive portion of its Decision read:
24

WHEREFORE, in view of the foregoing, the instant Petition is hereby GRANTED. The assailed Decision dated June 25,
2012 and Resolution dated August 24, 2012 rendered by the National Labor Relations Commission in NLRC LAC No. 07-
001930-11/NLRC NCR Cases No. 08-10645-10, 08-10649-10, 08-10655-10, 08-10660-10, 08- 10662-10, 08-10666-10
and 08-10670-10 are hereby REVERSED and SET ASIDE. Corollarily, the Decision dated May 31, 2011 rendered by the
Labor Arbiter is hereby REINSTATED.

SO ORDERED. 25

The employees filed a Motion for Reconsideration  but it was denied in the Court of Appeals May 30, 2013
26

Resolution.  Hence, this Petition  was filed before this Court.


27 28

Petitioners point out that the Court of Appeals erred in not finding grave abuse of discretion, considering that the petition
filed before it was a special civil action for certiorari. They aver that the Court of Appeals should not have used the special
remedy of certiorari merely to re-evaluate the findings of a quasi-judicial body absent any finding of grave abuse of
discretion. 29

Petitioners likewise argue that respondents were unable to substantially comply with the verification requirement before
the Court of Appeals. They submit that respondents' counsel would have been privy to the antecedents of the case so as
to have personal knowledge and not merely knowledge as relayed by his clients.   They add that respondents
30

"deliberately withheld the Annexes of the Position Paper of the Petitioners submitted to the Labor Arbiter[;] hence, said
Position Paper cannot be considered authentic." 31

Petitioners assert that the Court of Appeals had no factual basis to rule in respondents' favor since there was no evidence
to prove that the Notices were sent to petitioners at their last known addresses. The evidence on record merely showed
sample letters of the Notices.  Petitioners maintain that this is a situation where the employees allege that they were
32

prevented from entering their work place and the employer alleges otherwise. They insist that if doubt exists between the
evidence presented by the employer and the evidence presented by the employees, the doubt must be resolved in favor of
the employees, consistent with the Labor Code's policy to afford protection to labor.  33

On the other hand, respondents argue that a defect in the verification will not necessarily cause the dismissal of the
pleading and that they had sufficiently complied with the requirement when the affiant attested that the petition was based
on facts relayed by his clients and on authentic records.   They also point out that only relevant and pertinent documents
34

should be attached to their pleadings before the courts; thus, the annexes of petitioner, not being relevant or pertinent,
need not be attached to their pleadings.  35

Respondents contend that the Court of Appeals recognized that the issue in their Petition for Certiorari concerned the
alleged grave abuse of discretion of the National Labor Relations Commission and thoroughly discussed the issue in the
assailed judgment.   They likewise submit that the Court of Appeals may review factual findings of the National Labor
36

Relations Commission since the finding of grave abuse of discretion requires a re-examination of the sufficiency or
absence of evidence. 37

Respondents maintain that the receipt of the Notices was admitted and recognized by the parties before the Labor Arbiter
and was never brought as an issue until the National Labor Relations Commission made a finding that the Notices were
never received.  According to respondents, petitioners were estopped from questioning the receipt of the Notices when
38

they already admitted to their receipt before the Labor Arbiter.  They argue that the Labor Arbiter and the Court of Appeals
39

did not err in finding that the termination of employment was voluntary since petitioners failed to present evidence of the
fact of their dismissal.  40

The main issue before this Court is whether or not petitioners were illegally dismissed by respondents. However, there are
certain procedural issues that must first be addressed, in particular: (1) whether or not the Court of Appeals may, in a
petition for certiorari, review and re-assess the factual findings of the National Labor Relations Commission; and (2)
whether or not verification based on facts relayed to the affiant by his clients is valid.
I

Before discussing the merits of the case, this Court takes this opportunity to clarify certain doctrines regarding the review
of factual findings by the Court of Appeals.

Factual findings of labor officials exercising quasi-judicial functions are accorded great respect and even finality by the
courts when the findings are supported by substantial evidence.  Substantial evidence is "the amount of relevant evidence
41

which a reasonable mind might accept as adequate to support a conclusion. "  Thus, in labor cases, the issues in petitions
42

for certiorari before the Court of Appeals are limited only to whether the National Labor Relations Commission committed
grave abuse of discretion.

However, this does not mean that the Court of Appeals is conclusively bound by the findings of the National Labor
Relations Commission. If the findings are arrived at arbitrarily, without resort to any substantial evidence, the National
Labor Relations Commission is deemed to have gravely abused its discretion:

However, this does not mean that the Court of Appeals is conclusively bound by the findings of the National Labor
Relations Commission. If the findings are arrived at arbitrarily, without resort to any substantial evidence, the National
Labor Relations Commission is deemed to have gravely abused its discretion:

On this matter, the settled rule is that factual findings of labor officials, who are deemed to have acquired expertise in
matters within their jurisdiction, are generally accorded not only respect but even finality by the courts when supported by
substantial evidence, i.e., the amount of relevant evidence which a reasonable mind might accept as adequate to support
a conclusion. We emphasize, nonetheless, that these findings are not infallible. When there is a showing that they were
arrived at arbitrarily or in disregard of the evidence on record, they may be examined by the courts. The [Court of Appeals]
can then grant a petition for certiorari if it finds that the [National Labor Relations Commission], in its assailed decision or
resolution, has made a factual finding that is not supported by substantial evidence. It is within the jurisdiction of the [Court
of Appeals], whose jurisdiction over labor cases has been expanded to review the findings of the [National Labor Relations
Commission]. 43

The Court of Appeals may also review factual findings if quasi-judicial agencies' findings are contradictory to its own
findings.   Thus, it must re-examine the records to determine which tribunal's findings were supported by the evidence.
44

In this instance, the Labor Arbiter and the National Labor Relations Commission made contradictory factual findings. Thus,
it was incumbent on the Court of Appeals to re-examine their findings to resolve the issues before it. The Court of Appeals
also found that the findings of the National Labor Relations Commission were not supported by substantial evidence, and
therefore, were rendered in grave abuse of discretion.

Thus, in the determination of whether the National Labor Relations Commission committed grave abuse of discretion, the
Court of Appeals may re-examine facts and re-assess the evidence. However, its findings may still be subject to review by
this Court.

This Court notes that in cases when the Court of Appeals acts as an appellate court, it is still a trier of facts. Questions of
fact may still be raised by the parties. If the parties raise pure questions of law, they may directly file with this Court.
Moreover, contradictory factual findings between the National Labor Relations Commission and the Court of Appeals do
not automatically justify this Court's review of the factual findings. They merely present a  prima facie basis to pursue the
action before this Court. The need to review the Court of Appeals' factual findings must still be pleaded, proved, and
substantiated by the party alleging their inaccuracy. This Court likewise retains its full discretion to review the factual
findings.

II

All petitions for certiorari are required to be verified upon filing.   The contents of verification are stated under Rule 7,
45

Section 4 of the Rules of Court:

Section 4. Verification. Except when otherwise specifically required by law or rule, pleadings need not be under oath,
verified or accompanied by affidavit.

A pleading is verified by an affidavit that the affiant has read the pleading and that the allegations therein are true and
correct of his personal knowledge or based on authentic records.

A pleading required to be verified which contains a verification based on "information and belief'', or upon "knowledge,
information and belief," or lacks a proper verification, shall be treated as an unsigned pleading.

Thus, for a pleading to be verified, the affiant must attest that he or she has read the pleading and that the allegations are
true and correct based on his or her personal knowledge or on authentic records. Otherwise, the pleading is treated as an
unsigned pleading.

Shipside Incorporation v. Court of Appeals required that the assurance should "not [be] the product of the imagination or a
46

matter of speculation, and that the pleading is filed in good faith."  However, verification is merely a formal, not
47

jurisdictional, requirement. It will not result in the outright dismissal of the case since courts may simply order the
correction of a defective verification.
48
Petitioners argue that respondents' verification was invalid since it was not based on authentic records, alleging that
respondents' failure to attach petitioners' position paper annexes to their Petition for Certiorari before the Court of Appeals
made their records inauthentic.  49

A pleading may be verified by attesting that the allegations are based either on personal knowledge  and on authentic
records, or on personal knowledge or on authentic records. The use of either, however, is not subject to the affiant's whim
but rather on the nature of the allegations being attested to. Circumstances may require that the affiant attest that the
allegations are based only on personal knowledge or only on authentic records. Certainly, there can be situations where
the affiant must attest to the allegations being based on both personal knowledge and on authentic records, thus:

A reading of the above-quoted Section 4 of Rule 7 indicates that a pleading may be verified under either of the two given
modes or under both. The veracity of the allegations in a pleading may be affirmed based on either one's own personal
knowledge or on authentic records, or both, as warranted. The use of the [conjunction] "or" connotes that either source
qualifies as a sufficient basis for verification and, needless to state, the concurrence of both sources is more than
sufficient. Bearing both a disjunctive and conjunctive sense, this parallel legal signification avoids a construction that will
exclude the combination of the alternatives or bar the efficacy of any one of the alternatives standing alone.

Contrary to petitioner's position, the range of permutation is not left to the pleader's liking, but is dependent on the
surrounding nature of the allegations which may warrant that a verification be based either purely on personal knowledge,
or entirely on authentic records, or on both sources. 50

Authentic records may be the basis of verification if a substantial portion of the allegations in the pleading is based on prior
court proceedings.  Here, the annexes that respondents allegedly failed to attach are employee information, supporting
51

documents, and work-related documents proving that petitioners were employed by respondents.   The fact of petitioners'
52

employment, however, has not been disputed by respondents. These documents would not have been the "relevant and
pertinent"  documents contemplated by the rules.
53

Petitioners likewise contend that respondents' Petition for Certiorari  before the Court of Appeals should not have been
54

given due course since the verification  signed by respondents' counsel, Atty. Eller Roel I. Daclan (Atty. Daclan), attested
55

that:

2. I caused the preparation of the foregoing petition and attest that, based upon facts relayed to me by my clients and
upon authentic records made available, all the allegations contained therein are true and correct[.]  56

Thus, the issue on verification centers on whether the phrase "based upon facts relayed to me by my clients" may be
considered sufficient compliance. To resolve this issue, this Court must first address whether respondents' counsel may
sign the verification on their behalf.

The rules on compliance with the requirement of the verification and certification of non-forum shopping were already
sufficiently outlined in Altres v. Empleo,   where this Court stated:
57

For the guidance of the bench and bar, the Court restates in capsule form the jurisprudential pronouncements already
reflected above respecting non-compliance with the requirements on, or submission of defective, verification and
certification against forum shopping:

1) A distinction must be made between non-compliance with the requirement on or submission of defective verification,
and noncompliance with the requirement on or submission of defective certification against forum shopping.

2) As to verification, non-compliance therewith or a defect therein does not necessarily render the pleading fatally
defective. The court may order its submission or correction or act on the pleading if the attending circumstances are such
that strict compliance with the Rule may be dispensed with in order that the ends of justice may be served thereby.

3) Verification is deemed substantially complied with when one who has ample knowledge to swear to the truth of the
allegations in the complaint or petition signs the verification, and when matters alleged in the petition have been made in
good faith or are true and correct.

4) As to certification against forum shopping, non-compliance therewith or a defect therein, unlike in verification, is
generally not curable by its subsequent submission or correction thereof, unless there is a need to relax the Rule on the
ground of "substantial compliance" or presence of "special circumstances or compelling reasons".

5) The certification against forum shopping must be signed by all the plaintiffs or petitioners in a case; otherwise, those
who did not sign will be dropped as parties to the case. Under reasonable or justifiable circumstances, however, as when
all the plaintiffs or petitioners share a common interest and invoke a common cause of action or defense, the signature of
only one of them in the certification against forum shopping substantially complies with the Rule.

6) Finally, the certification against forum shopping must be executed by the party-pleader, not by his counsel. If, however,
for reasonable or justifiable reasons, the party-pleader is unable to sign, he must execute a Special Power of Attorney
designating his counsel of record to sign on his behalf.  58
The policy behind the requirement of verification is to guard against the filing of fraudulent pleadings. Litigants run the risk
of perjury  if they sign the verification despite knowledge that the stated allegations are not true or are products of mere
59

speculation:

Verification is not an empty ritual or a meaningless formality. Its import must never be sacrificed in the name of mere
expedience or sheer caprice. For what is at stake is the matter of verity attested by the sanctity of an oath to secure an
assurance that the allegations in the pleading have been made in good faith, or are true and correct and not merely
speculative.  60

Thus, for verification to be valid, the affiant must have "ample knowledge to swear to the truth of the allegations in the
complaint or petition."  Facts relayed to the counsel by the client would be insufficient for counsel to swear to the truth of
61

the allegations in a pleading. Otherwise, counsel would be able to disclaim liability for any misrepresentation by the simple
expediency of stating that he or she was merely relaying facts with which he or she had no competency to attest to. For
this reason, the Rules of Court require no less than personal knowledge of the facts to sufficiently verify a pleading.

Respondents' counsel, not having sufficient personal knowledge to attest to the allegations of the pleading, was not able
to validly verify the facts as stated. Therefore, respondents' Petition for Certiorari before the Court of Appeals should have
been considered as an unsigned pleading.

Respondents' certification of non-forum shopping is likewise defective. The certification of non-forum shopping must be
signed by the litigant, not his or her counsel. The litigant may, for justifiable reasons, execute a special power of attorney
to authorize his or her counsel to sign on his or her behalf.   In this instance, the verification and certification against forum
62

shopping  was contained in one ( 1) document and was signed by respondents' counsel, Atty. Daclan.
63

Corporations, not being natural persons, may authorize their lawyers through a Secretary's Certificate to execute physical
acts. Among these acts is the signing of documents, such as the certification against forum shopping. A corporation's
inability to perform physical acts is considered as a justifiable reason to allow a person other than the litigant to sign the
certification against forum shopping.  By the same reasoning, partnerships, being artificial entities, may also authorize an
64

agent to sign the certification on their behalf.

Respondents include three (3) corporations, one (1) partnership, and three (3) sole proprietorships. Respondents LSG
Manufacturing Corporation, Asia Consumer Value Trading, Inc., and Wantofree Oriental Trading, Inc. submitted
Secretary's Certificates  authorizing Atty. Daclan to sign on their behalf. On the other hand, respondent HSY Marketing
65

Ltd., Co. submitted a Partnership Certification.  Meanwhile, respondents Alexander Arqueza (Arqueza), proprietor of
66

Fabulous Jeans and Shirt and General Merchandise, Rosario Q. Co (Co), proprietor of Unite General Merchandise, and
Lucia Pun Ling Yeung (Yeung), proprietor of Coen Fashion House & General Merchandise, submitted Special Powers of
Attorney  on their behalf.
67

However, sole proprietorships, unlike corporations, have no separate legal personality from their proprietors.  They cannot
68

claim the inability to do physical acts as a justifiable circumstance to authorize their counsel to sign on their behalf. Since
there was no other reason given for authorizing their counsel to sign on their behalf, respondents Arqueza, Co, and
Yeung's certification against forum shopping is invalid.

While courts may simply order the resubmission of the verification or its subsequent correction,  a defect in the
69

certification of non-forum shopping is not curable   unless there are substantial merits to the case.
70 71

However, respondents' Petition for Certiorari before the Court of Appeals was unmeritorious. Thus, its defective
verification and certification of non-forum shopping should have merited its outright dismissal.

III

When the evidence of the employer and the employee are in equipoise, doubts are resolved in favor of labor.  This is in 72

line with the policy of the State to afford greater protection to labor. 
73

Petitioners allege that they were illegally dismissed from service when they were prevented from entering their work
premises a day after airing their grievance in a radio show. On the other hand, respondents deny this allegation and state
that petitioners were never dismissed from employment.

In illegal dismissal cases, the burden of proof is on the employer to prove that the employee was dismissed for a valid
cause and that the employee was afforded due process prior to the dismissal.  74

Respondents allege that there was no dismissal since they sent petitioners a First Notice of Termination of Employment,
asking them to show cause why they should not be dismissed for their continued absence from work. However, petitioners
argue that this evidence should not be given weight since there is no proof that they received this Notice.

Indeed, no evidence has been presented proving that each and every petitioner received a copy of the First Notice of
Termination of Employment.  There are no receiving copies or acknowledgement receipts. What respondents presented
1âwphi1

were "Sample Letters of Respondents"  and not the actual Notices that were allegedly sent out.
75
While petitioners admitted that the Notices may have been sent, they have never actually admitted to receiving any of
them. In their Position Paper before the Labor Arbiter and in their Memorandum of Appeal before the National Labor
Relations Commission:

On June 7, 2010, all employees who went to complain against the respondent[ s] were not allowed to enter the stores of
respondent[s]. The next day, respondent[s] sent letter[s] to the employees purporting to be a show cause letter but the
truth of the matter is that all employees who went to the office of Tulfo to complain against the respondent[ s] were already
terminated[.]76

The lack of evidence of petitioners' receipts suggests that the Notices were an afterthought, designed to free respondents
from any liability without having to validly dismiss petitioners.

There is likewise no proof that petitioners abandoned their employment. To constitute abandonment, the employer must
prove that "first, the employee must have failed to report for work or must have been absent without valid or justifiable
reason; and second, [that] there must have been a clear intention on the part of the employee to sever the employer-
employee relationship manifested by some overt act." 77

Abandonment is essentially a matter of intent. It cannot be presumed from the occurrence of certain equivocal
acts.   There must be a positive and overt act signifying an employee's deliberate intent to sever his or her employment.
78

Thus, mere absence from work, even after a notice to return, is insufficient to prove abandonment.    The employer must
79

show that the employee unjustifiably refused to report for work and that the employee deliberately intended to sever the
employer-employee relation. Furthermore, there must be a concurrence of these two (2) elements.  Absent this 80

concurrence, there can be no abandonment.

Respondents have not presented any proof that petitioners intended to abandon their employment. They merely alleged
that petitioners have already voluntarily terminated their employment due to their continued refusal to report for work.
However, this is insufficient to prove abandonment.

Where both parties in a labor case have not presented substantial evidence to prove their allegations, the evidence is
considered to be in equipoise. In such a case, the scales of justice are tilted in favor of labor. Thus, petitioners are hereby
considered to have been illegally dismissed.

This Court notes that had petitioners been able to substantially prove their dismissal, it would have been rendered invalid
not only for having been made without just cause  but also for being in violation of their constitutional rights. A laborer
81

does not lose his or her right to freedom of expression upon employment.  This is "[a] political [right] essential to man's
82

enjoyment of his [or her] life, to his [or her] happiness, and to his [or her] full and complete fulfillment."  While the
83

Constitution and the courts recognize that employers have property rights that must also be protected, the human rights of
laborers are given primacy over these rights. Property rights may prescribe. Human rights do not.  84

When laborers air out their grievances regarding their employment in a public forum, they do so in the exercise of their
right to free expression. They are "fighting for their very survival, utilizing only the weapons afforded them by the
Constitution-the untrammelled enjoyment of their basic human rights."  Freedom and social justice afford them these
85

rights and it is the courts' duty to uphold and protect their free exercise. Thus, dismissing employees merely on the basis
that they complained about their employer in a radio show is not only invalid, it is unconstitutional.

However, there not being sufficient proof that the dismissal was meant to suppress petitioners' constitutional rights, this
Court is constrained to limit its conclusions to that of illegal dismissal under the Labor Code.

Petitioners were not dismissed under any of the causes mentioned in Article 279 [282]  of the Labor Code. They were not
86

validly informed of the causes of their dismissal. Thus, their dismissal was illegal.

An employee who is found to have been illegally dismissed is entitled to reinstatement without loss of seniority rights and
other privileges.   If reinstatement proves to be impossible due to the strained relations between the parties, the illegally
87

dismissed employee is entitled instead to separation pay. 88

WHEREFORE, the Petition is GRANTED. The February 25, 2013 Decision and May 30, 2013 Resolution of the Court of
Appeals in CA-G.R. SP No. 126522 are SET ASIDE. Respondents are DIRECTED to reinstate petitioners to their former
positions without loss of seniority rights or other privileges. SO ORDERED.

Promotion of health and ecology, Article II, Sections 15 and 16; Article XIII, Section 11
-Oposa v Factoran, GR 101083 (1993)

In a broader sense, this petition bears upon the right of Filipinos to a balanced and healthful ecology which the petitioners
dramatically associate with the twin concepts of "inter-generational responsibility" and "inter-generational justice."
Specifically, it touches on the issue of whether the said petitioners have a cause of action to "prevent the misappropriation
or impairment" of Philippine rainforests and "arrest the unabated hemorrhage of the country's vital life support systems
and continued rape of Mother Earth."

The controversy has its genesis in Civil Case No. 90-77 which was filed before Branch 66 (Makati, Metro Manila) of the
Regional Trial Court (RTC), National Capital Judicial Region. The principal plaintiffs therein, now the principal petitioners,
are all minors duly represented and joined by their respective parents. Impleaded as an additional plaintiff is the Philippine
Ecological Network, Inc. (PENI), a domestic, non-stock and non-profit corporation organized for the purpose of, inter alia,
engaging in concerted action geared for the protection of our environment and natural resources. The original defendant
was the Honorable Fulgencio S. Factoran, Jr., then Secretary of the Department of Environment and Natural Resources
(DENR). His substitution in this petition by the new Secretary, the Honorable Angel C. Alcala, was subsequently ordered
upon proper motion by the petitioners.  The complaint  was instituted as a taxpayers' class suit  and alleges that the
1 2 3

plaintiffs "are all citizens of the Republic of the Philippines, taxpayers, and entitled to the full benefit, use and enjoyment of
the natural resource treasure that is the country's virgin tropical forests." The same was filed for themselves and others
who are equally concerned about the preservation of said resource but are "so numerous that it is impracticable to bring
them all before the Court." The minors further asseverate that they "represent their generation as well as generations yet
unborn."  Consequently, it is prayed for that judgment be rendered:
4

. . . ordering defendant, his agents, representatives and other persons acting in his behalf to —

(1) Cancel all existing timber license agreements in the country;

(2) Cease and desist from receiving, accepting, processing, renewing or approving new timber license
agreements.

and granting the plaintiffs ". . . such other reliefs just and equitable under the premises." 5

The complaint starts off with the general averments that the Philippine archipelago of 7,100 islands has a land area of
thirty million (30,000,000) hectares and is endowed with rich, lush and verdant rainforests in which varied, rare and unique
species of flora and fauna may be found; these rainforests contain a genetic, biological and chemical pool which is
irreplaceable; they are also the habitat of indigenous Philippine cultures which have existed, endured and flourished since
time immemorial; scientific evidence reveals that in order to maintain a balanced and healthful ecology, the country's land
area should be utilized on the basis of a ratio of fifty-four per cent (54%) for forest cover and forty-six per cent (46%) for
agricultural, residential, industrial, commercial and other uses; the distortion and disturbance of this balance as a
consequence of deforestation have resulted in a host of environmental tragedies, such as (a) water shortages resulting
from drying up of the water table, otherwise known as the "aquifer," as well as of rivers, brooks and streams, (b)
salinization of the water table as a result of the intrusion therein of salt water, incontrovertible examples of which may be
found in the island of Cebu and the Municipality of Bacoor, Cavite, (c) massive erosion and the consequential loss of soil
fertility and agricultural productivity, with the volume of soil eroded estimated at one billion (1,000,000,000) cubic meters
per annum — approximately the size of the entire island of Catanduanes, (d) the endangering and extinction of the
country's unique, rare and varied flora and fauna, (e) the disturbance and dislocation of cultural communities, including the
disappearance of the Filipino's indigenous cultures, (f) the siltation of rivers and seabeds and consequential destruction of
corals and other aquatic life leading to a critical reduction in marine resource productivity, (g) recurrent spells of drought as
is presently experienced by the entire country, (h) increasing velocity of typhoon winds which result from the absence of
windbreakers, (i) the floodings of lowlands and agricultural plains arising from the absence of the absorbent mechanism of
forests, (j) the siltation and shortening of the lifespan of multi-billion peso dams constructed and operated for the purpose
of supplying water for domestic uses, irrigation and the generation of electric power, and (k) the reduction of the earth's
capacity to process carbon dioxide gases which has led to perplexing and catastrophic climatic changes such as the
phenomenon of global warming, otherwise known as the "greenhouse effect."

Plaintiffs further assert that the adverse and detrimental consequences of continued and deforestation are so capable of
unquestionable demonstration that the same may be submitted as a matter of judicial notice. This notwithstanding, they
expressed their intention to present expert witnesses as well as documentary, photographic and film evidence in the
course of the trial.

As their cause of action, they specifically allege that:

CAUSE OF ACTION

7. Plaintiffs replead by reference the foregoing allegations.

8. Twenty-five (25) years ago, the Philippines had some sixteen (16) million hectares of rainforests
constituting roughly 53% of the country's land mass.

9. Satellite images taken in 1987 reveal that there remained no more than 1.2 million hectares of said
rainforests or four per cent (4.0%) of the country's land area.

10. More recent surveys reveal that a mere 850,000 hectares of virgin old-growth rainforests are left,
barely 2.8% of the entire land mass of the Philippine archipelago and about 3.0 million hectares of
immature and uneconomical secondary growth forests.

11. Public records reveal that the defendant's, predecessors have granted timber license agreements
('TLA's') to various corporations to cut the aggregate area of 3.89 million hectares for commercial logging
purposes.

A copy of the TLA holders and the corresponding areas covered is hereto attached as Annex "A".
12. At the present rate of deforestation, i.e. about 200,000 hectares per annum or 25 hectares per hour —
nighttime, Saturdays, Sundays and holidays included — the Philippines will be bereft of forest resources
after the end of this ensuing decade, if not earlier.

13. The adverse effects, disastrous consequences, serious injury and irreparable damage of this continued
trend of deforestation to the plaintiff minor's generation and to generations yet unborn are evident and
incontrovertible. As a matter of fact, the environmental damages enumerated in paragraph 6 hereof are
already being felt, experienced and suffered by the generation of plaintiff adults.

14. The continued allowance by defendant of TLA holders to cut and deforest the remaining forest stands
will work great damage and irreparable injury to plaintiffs — especially plaintiff minors and their successors
— who may never see, use, benefit from and enjoy this rare and unique natural resource treasure.

This act of defendant constitutes a misappropriation and/or impairment of the natural resource property he
holds in trust for the benefit of plaintiff minors and succeeding generations.

15. Plaintiffs have a clear and constitutional right to a balanced and healthful ecology and are entitled to
protection by the State in its capacity as the parens patriae.

16. Plaintiff have exhausted all administrative remedies with the defendant's office. On March 2, 1990,
plaintiffs served upon defendant a final demand to cancel all logging permits in the country.

A copy of the plaintiffs' letter dated March 1, 1990 is hereto attached as Annex "B".

17. Defendant, however, fails and refuses to cancel the existing TLA's to the continuing serious damage
and extreme prejudice of plaintiffs.

18. The continued failure and refusal by defendant to cancel the TLA's is an act violative of the rights of
plaintiffs, especially plaintiff minors who may be left with a country that is desertified (sic), bare, barren and
devoid of the wonderful flora, fauna and indigenous cultures which the Philippines had been abundantly
blessed with.

19. Defendant's refusal to cancel the aforementioned TLA's is manifestly contrary to the public policy
enunciated in the Philippine Environmental Policy which, in pertinent part, states that it is the policy of the
State —

(a) to create, develop, maintain and improve conditions under which man and nature can thrive in
productive and enjoyable harmony with each other;

(b) to fulfill the social, economic and other requirements of present and future generations of Filipinos and;

(c) to ensure the attainment of an environmental quality that is conductive to a life of dignity and well-being.
(P.D. 1151, 6 June 1977)

20. Furthermore, defendant's continued refusal to cancel the aforementioned TLA's is contradictory to the
Constitutional policy of the State to —

a. effect "a more equitable distribution of opportunities, income and wealth" and "make full and efficient use
of natural resources (sic)." (Section 1, Article XII of the Constitution);

b. "protect the nation's marine wealth." (Section 2, ibid);

c. "conserve and promote the nation's cultural heritage and resources (sic)" (Section 14, Article XIV, id.);

d. "protect and advance the right of the people to a balanced and healthful ecology in accord with the
rhythm and harmony of nature." (Section 16, Article II, id.)

21. Finally, defendant's act is contrary to the highest law of humankind — the natural law — and violative
of plaintiffs' right to self-preservation and perpetuation.

22. There is no other plain, speedy and adequate remedy in law other than the instant action to arrest the
unabated hemorrhage of the country's vital life support systems and continued rape of Mother Earth.  6

On 22 June 1990, the original defendant, Secretary Factoran, Jr., filed a Motion to Dismiss the complaint based on two (2)
grounds, namely: (1) the plaintiffs have no cause of action against him and (2) the issue raised by the plaintiffs is a political
question which properly pertains to the legislative or executive branches of Government. In their 12 July 1990 Opposition
to the Motion, the petitioners maintain that (1) the complaint shows a clear and unmistakable cause of action, (2) the
motion is dilatory and (3) the action presents a justiciable question as it involves the defendant's abuse of discretion.
On 18 July 1991, respondent Judge issued an order granting the aforementioned motion to dismiss.  In the said order, not
7

only was the defendant's claim — that the complaint states no cause of action against him and that it raises a political
question — sustained, the respondent Judge further ruled that the granting of the relief prayed for would result in the
impairment of contracts which is prohibited by the fundamental law of the land.

Plaintiffs thus filed the instant special civil action for certiorari under Rule 65 of the Revised Rules of Court and ask this
Court to rescind and set aside the dismissal order on the ground that the respondent Judge gravely abused his discretion
in dismissing the action. Again, the parents of the plaintiffs-minors not only represent their children, but have also joined
the latter in this case.
8

On 14 May 1992, We resolved to give due course to the petition and required the parties to submit their respective
Memoranda after the Office of the Solicitor General (OSG) filed a Comment in behalf of the respondents and the
petitioners filed a reply thereto.

Petitioners contend that the complaint clearly and unmistakably states a cause of action as it contains sufficient allegations
concerning their right to a sound environment based on Articles 19, 20 and 21 of the Civil Code (Human Relations),
Section 4 of Executive Order (E.O.) No. 192 creating the DENR, Section 3 of Presidential Decree (P.D.) No. 1151
(Philippine Environmental Policy), Section 16, Article II of the 1987 Constitution recognizing the right of the people to a
balanced and healthful ecology, the concept of generational genocide in Criminal Law and the concept of man's
inalienable right to self-preservation and self-perpetuation embodied in natural law. Petitioners likewise rely on the
respondent's correlative obligation per Section 4 of E.O. No. 192, to safeguard the people's right to a healthful
environment.

It is further claimed that the issue of the respondent Secretary's alleged grave abuse of discretion in granting Timber
License Agreements (TLAs) to cover more areas for logging than what is available involves a judicial question.

Anent the invocation by the respondent Judge of the Constitution's non-impairment clause, petitioners maintain that the
same does not apply in this case because TLAs are not contracts. They likewise submit that even if TLAs may be
considered protected by the said clause, it is well settled that they may still be revoked by the State when the public
interest so requires.

On the other hand, the respondents aver that the petitioners failed to allege in their complaint a specific legal right violated
by the respondent Secretary for which any relief is provided by law. They see nothing in the complaint but vague and
nebulous allegations concerning an "environmental right" which supposedly entitles the petitioners to the "protection by the
state in its capacity as parens patriae." Such allegations, according to them, do not reveal a valid cause of action. They
then reiterate the theory that the question of whether logging should be permitted in the country is a political question
which should be properly addressed to the executive or legislative branches of Government. They therefore assert that the
petitioners' resources is not to file an action to court, but to lobby before Congress for the passage of a bill that would ban
logging totally.

As to the matter of the cancellation of the TLAs, respondents submit that the same cannot be done by the State without
due process of law. Once issued, a TLA remains effective for a certain period of time — usually for twenty-five (25) years.
During its effectivity, the same can neither be revised nor cancelled unless the holder has been found, after due notice and
hearing, to have violated the terms of the agreement or other forestry laws and regulations. Petitioners' proposition to have
all the TLAs indiscriminately cancelled without the requisite hearing would be violative of the requirements of due process.

Before going any further, We must first focus on some procedural matters. Petitioners instituted Civil Case No. 90-777 as
a class suit. The original defendant and the present respondents did not take issue with this matter. Nevertheless, We
hereby rule that the said civil case is indeed a class suit. The subject matter of the complaint is of common and general
interest not just to several, but to all citizens of the Philippines. Consequently, since the parties are so numerous, it,
becomes impracticable, if not totally impossible, to bring all of them before the court. We likewise declare that the plaintiffs
therein are numerous and representative enough to ensure the full protection of all concerned interests. Hence, all the
requisites for the filing of a valid class suit under Section 12, Rule 3 of the Revised Rules of Court are present both in the
said civil case and in the instant petition, the latter being but an incident to the former.

This case, however, has a special and novel element. Petitioners minors assert that they represent their generation as
well as generations yet unborn. We find no difficulty in ruling that they can, for themselves, for others of their generation
and for the succeeding generations, file a class suit. Their personality to sue in behalf of the succeeding generations can
only be based on the concept of intergenerational responsibility insofar as the right to a balanced and healthful ecology is
concerned. Such a right, as hereinafter expounded, considers
the "rhythm and harmony of nature." Nature means the created world in its entirety.  Such rhythm and harmony
9

indispensably include, inter alia, the judicious disposition, utilization, management, renewal and conservation of the
country's forest, mineral, land, waters, fisheries, wildlife, off-shore areas and other natural resources to the end that their
exploration, development and utilization be equitably accessible to the present as well as future generations.   Needless to
10

say, every generation has a responsibility to the next to preserve that rhythm and harmony for the full enjoyment of a
balanced and healthful ecology. Put a little differently, the minors' assertion of their right to a sound environment
constitutes, at the same time, the performance of their obligation to ensure the protection of that right for the generations
to come.

The locus standi of the petitioners having thus been addressed, We shall now proceed to the merits of the petition.
After a careful perusal of the complaint in question and a meticulous consideration and evaluation of the issues raised and
arguments adduced by the parties, We do not hesitate to find for the petitioners and rule against the respondent Judge's
challenged order for having been issued with grave abuse of discretion amounting to lack of jurisdiction. The pertinent
portions of the said order reads as follows:

xxx xxx xxx

After a careful and circumspect evaluation of the Complaint, the Court cannot help but agree with the
defendant. For although we believe that plaintiffs have but the noblest of all intentions, it (sic) fell short of
alleging, with sufficient definiteness, a specific legal right they are seeking to enforce and protect, or a
specific legal wrong they are seeking to prevent and redress (Sec. 1, Rule 2, RRC). Furthermore, the Court
notes that the Complaint is replete with vague assumptions and vague conclusions based on unverified
data. In fine, plaintiffs fail to state a cause of action in its Complaint against the herein defendant.

Furthermore, the Court firmly believes that the matter before it, being impressed with political color and
involving a matter of public policy, may not be taken cognizance of by this Court without doing violence to
the sacred principle of "Separation of Powers" of the three (3) co-equal branches of the Government.

The Court is likewise of the impression that it cannot, no matter how we stretch our jurisdiction, grant the
reliefs prayed for by the plaintiffs, i.e., to cancel all existing timber license agreements in the country and to
cease and desist from receiving, accepting, processing, renewing or approving new timber license
agreements. For to do otherwise would amount to "impairment of contracts" abhored (sic) by the
fundamental law.  11

We do not agree with the trial court's conclusions that the plaintiffs failed to allege with sufficient definiteness a specific
legal right involved or a specific legal wrong committed, and that the complaint is replete with vague assumptions and
conclusions based on unverified data. A reading of the complaint itself belies these conclusions.

The complaint focuses on one specific fundamental legal right — the right to a balanced and healthful ecology which, for
the first time in our nation's constitutional history, is solemnly incorporated in the fundamental law. Section 16, Article II of
the 1987 Constitution explicitly provides:

Sec. 16. The State shall protect and advance the right of the people to a balanced and healthful ecology in
accord with the rhythm and harmony of nature.

This right unites with the right to health which is provided for in the preceding section of the same article:

Sec. 15. The State shall protect and promote the right to health of the people and instill health
consciousness among them.

While the right to a balanced and healthful ecology is to be found under the Declaration of Principles and State Policies
and not under the Bill of Rights, it does not follow that it is less important than any of the civil and political rights
enumerated in the latter. Such a right belongs to a different category of rights altogether for it concerns nothing less than
self-preservation and self-perpetuation — aptly and fittingly stressed by the petitioners — the advancement of which may
even be said to predate all governments and constitutions. As a matter of fact, these basic rights need not even be written
in the Constitution for they are assumed to exist from the inception of humankind. If they are now explicitly mentioned in
the fundamental charter, it is because of the well-founded fear of its framers that unless the rights to a balanced and
healthful ecology and to health are mandated as state policies by the Constitution itself, thereby highlighting their
continuing importance and imposing upon the state a solemn obligation to preserve the first and protect and advance the
second, the day would not be too far when all else would be lost not only for the present generation, but also for those to
come — generations which stand to inherit nothing but parched earth incapable of sustaining life.

The right to a balanced and healthful ecology carries with it the correlative duty to refrain from impairing the environment.
During the debates on this right in one of the plenary sessions of the 1986 Constitutional Commission, the following
exchange transpired between Commissioner Wilfrido Villacorta and Commissioner Adolfo Azcuna who sponsored the
section in question:

MR. VILLACORTA:

Does this section mandate the State to provide sanctions against all forms of pollution —
air, water and noise pollution?

MR. AZCUNA:

Yes, Madam President. The right to healthful (sic) environment necessarily carries with it
the correlative duty of not impairing the same and, therefore, sanctions may be provided for
impairment of environmental balance.  12

The said right implies, among many other things, the judicious management and conservation of the country's forests.

Without such forests, the ecological or environmental balance would be irreversiby disrupted.
Conformably with the enunciated right to a balanced and healthful ecology and the right to health, as well as the other
related provisions of the Constitution concerning the conservation, development and utilization of the country's natural
resources,   then President Corazon C. Aquino promulgated on 10 June 1987 E.O. No. 192,   Section 4 of which
13 14

expressly mandates that the Department of Environment and Natural Resources "shall be the primary government agency
responsible for the conservation, management, development and proper use of the country's environment and natural
resources, specifically forest and grazing lands, mineral, resources, including those in reservation and watershed areas,
and lands of the public domain, as well as the licensing and regulation of all natural resources as may be provided for by
law in order to ensure equitable sharing of the benefits derived therefrom for the welfare of the present and future
generations of Filipinos." Section 3 thereof makes the following statement of policy:

Sec. 3. Declaration of Policy. — It is hereby declared the policy of the State to ensure the sustainable use,
development, management, renewal, and conservation of the country's forest, mineral, land, off-shore
areas and other natural resources, including the protection and enhancement of the quality of the
environment, and equitable access of the different segments of the population to the development and the
use of the country's natural resources, not only for the present generation but for future generations as
well. It is also the policy of the state to recognize and apply a true value system including social and
environmental cost implications relative to their utilization, development and conservation of our natural
resources.

This policy declaration is substantially re-stated it Title XIV, Book IV of the Administrative Code of 1987,  specifically in
15

Section 1 thereof which reads:

Sec. 1. Declaration of Policy. — (1) The State shall ensure, for the benefit of the Filipino people, the full
exploration and development as well as the judicious disposition, utilization, management, renewal and
conservation of the country's forest, mineral, land, waters, fisheries, wildlife, off-shore areas and other
natural resources, consistent with the necessity of maintaining a sound ecological balance and protecting
and enhancing the quality of the environment and the objective of making the exploration, development
and utilization of such natural resources equitably accessible to the different segments of the present as
well as future generations.

(2) The State shall likewise recognize and apply a true value system that takes into account social and
environmental cost implications relative to the utilization, development and conservation of our natural
resources.

The above provision stresses "the necessity of maintaining a sound ecological balance and protecting and enhancing the
quality of the environment." Section 2 of the same Title, on the other hand, specifically speaks of the mandate of the
DENR; however, it makes particular reference to the fact of the agency's being subject to law and higher authority. Said
section provides:

Sec. 2. Mandate. — (1) The Department of Environment and Natural Resources shall be primarily
responsible for the implementation of the foregoing policy.

(2) It shall, subject to law and higher authority, be in charge of carrying out the State's constitutional
mandate to control and supervise the exploration, development, utilization, and conservation of the
country's natural resources.

Both E.O. NO. 192 and the Administrative Code of 1987 have set the objectives which will serve as the bases for policy
formulation, and have defined the powers and functions of the DENR.

It may, however, be recalled that even before the ratification of the 1987 Constitution, specific statutes already paid special
attention to the "environmental right" of the present and future generations. On 6 June 1977, P.D. No. 1151 (Philippine
Environmental Policy) and P.D. No. 1152 (Philippine Environment Code) were issued. The former "declared a continuing
policy of the State (a) to create, develop, maintain and improve conditions under which man and nature can thrive in
productive and enjoyable harmony with each other, (b) to fulfill the social, economic and other requirements of present and
future generations of Filipinos, and (c) to insure the attainment of an environmental quality that is conducive to a life of
dignity and well-being."   As its goal, it speaks of the "responsibilities of each generation as trustee and guardian of the
16

environment for succeeding generations."   The latter statute, on the other hand, gave flesh to the said policy.
17

Thus, the right of the petitioners (and all those they represent) to a balanced and healthful ecology is as clear as the
DENR's duty — under its mandate and by virtue of its powers and functions under E.O. No. 192 and the Administrative
Code of 1987 — to protect and advance the said right.

A denial or violation of that right by the other who has the corelative duty or obligation to respect or protect the same gives
rise to a cause of action. Petitioners maintain that the granting of the TLAs, which they claim was done with grave abuse
of discretion, violated their right to a balanced and healthful ecology; hence, the full protection thereof requires that no
further TLAs should be renewed or granted.

A cause of action is defined as:


. . . an act or omission of one party in violation of the legal right or rights of the other; and its essential
elements are legal right of the plaintiff, correlative obligation of the defendant, and act or omission of the
defendant in violation of said legal right. 
18

It is settled in this jurisdiction that in a motion to dismiss based on the ground that the complaint fails to state a cause of
action,   the question submitted to the court for resolution involves the sufficiency of the facts alleged in the complaint
19

itself. No other matter should be considered; furthermore, the truth of falsity of the said allegations is beside the point for
the truth thereof is deemed hypothetically admitted. The only issue to be resolved in such a case is: admitting such alleged
facts to be true, may the court render a valid judgment in accordance with the prayer in the complaint?   In Militante vs.
20

Edrosolano,   this Court laid down the rule that the judiciary should "exercise the utmost care and circumspection in
21

passing upon a motion to dismiss on the ground of the absence thereof [cause of action] lest, by its failure to manifest a
correct appreciation of the facts alleged and deemed hypothetically admitted, what the law grants or recognizes is
effectively nullified. If that happens, there is a blot on the legal order. The law itself stands in disrepute."

After careful examination of the petitioners' complaint, We find the statements under the introductory affirmative
allegations, as well as the specific averments under the sub-heading CAUSE OF ACTION, to be adequate enough to
show, prima facie, the claimed violation of their rights. On the basis thereof, they may thus be granted, wholly or partly, the
reliefs prayed for. It bears stressing, however, that insofar as the cancellation of the TLAs is concerned, there is the need
to implead, as party defendants, the grantees thereof for they are indispensable parties.

The foregoing considered, Civil Case No. 90-777 be said to raise a political question. Policy formulation or determination
by the executive or legislative branches of Government is not squarely put in issue. What is principally involved is the
enforcement of a right vis-a-vis policies already formulated and expressed in legislation. It must, nonetheless, be
emphasized that the political question doctrine is no longer, the insurmountable obstacle to the exercise of judicial power
or the impenetrable shield that protects executive and legislative actions from judicial inquiry or review. The second
paragraph of section 1, Article VIII of the Constitution states that:

Judicial power includes the duty of the courts of justice to settle actual controversies involving rights which
are legally demandable and enforceable, and to determine whether or not there has been a grave abuse of
discretion amounting to lack or excess of jurisdiction on the part of any branch or instrumentality of the
Government.

Commenting on this provision in his book, Philippine Political Law,   Mr. Justice Isagani A. Cruz, a distinguished member
22

of this Court, says:

The first part of the authority represents the traditional concept of judicial power, involving the settlement of
conflicting rights as conferred as law. The second part of the authority represents a broadening of judicial
power to enable the courts of justice to review what was before forbidden territory, to wit, the discretion of
the political departments of the government.

As worded, the new provision vests in the judiciary, and particularly the Supreme Court, the power to rule
upon even the wisdom of the decisions of the executive and the legislature and to declare their acts invalid
for lack or excess of jurisdiction because tainted with grave abuse of discretion. The catch, of course, is the
meaning of "grave abuse of discretion," which is a very elastic phrase that can expand or contract
according to the disposition of the judiciary.

In Daza vs. Singson,   Mr. Justice Cruz, now speaking for this Court, noted:
23

In the case now before us, the jurisdictional objection becomes even less tenable and decisive. The
reason is that, even if we were to assume that the issue presented before us was political in nature, we
would still not be precluded from revolving it under the expanded jurisdiction conferred upon us that now
covers, in proper cases, even the political question. Article VII, Section 1, of the Constitution clearly
provides: . . .

The last ground invoked by the trial court in dismissing the complaint is the non-impairment of contracts clause found in
the Constitution. The court a quo declared that:

The Court is likewise of the impression that it cannot, no matter how we stretch our jurisdiction, grant the
reliefs prayed for by the plaintiffs, i.e., to cancel all existing timber license agreements in the country and to
cease and desist from receiving, accepting, processing, renewing or approving new timber license
agreements. For to do otherwise would amount to "impairment of contracts" abhored (sic) by the
fundamental law.  24

We are not persuaded at all; on the contrary, We are amazed, if not shocked, by such a sweeping pronouncement. In the
first place, the respondent Secretary did not, for obvious reasons, even invoke in his motion to dismiss the non-impairment
clause. If he had done so, he would have acted with utmost infidelity to the Government by providing undue and
unwarranted benefits and advantages to the timber license holders because he would have forever bound the
Government to strictly respect the said licenses according to their terms and conditions regardless of changes in policy
and the demands of public interest and welfare. He was aware that as correctly pointed out by the petitioners, into every
timber license must be read Section 20 of the Forestry Reform Code (P.D. No. 705) which provides:
. . . Provided, That when the national interest so requires, the President may amend, modify, replace or
rescind any contract, concession, permit, licenses or any other form of privilege granted herein . . .

Needless to say, all licenses may thus be revoked or rescinded by executive action. It is not a contract, property or
a property right protested by the due process clause of the Constitution. In Tan vs. Director of Forestry,   this Court
25

held:

. . . A timber license is an instrument by which the State regulates the utilization and disposition of forest
resources to the end that public welfare is promoted. A timber license is not a contract within the purview of
the due process clause; it is only a license or privilege, which can be validly withdrawn whenever dictated
by public interest or public welfare as in this case.

A license is merely a permit or privilege to do what otherwise would be unlawful, and is not a contract
between the authority, federal, state, or municipal, granting it and the person to whom it is granted; neither
is it property or a property right, nor does it create a vested right; nor is it taxation (37 C.J. 168). Thus, this
Court held that the granting of license does not create irrevocable rights, neither is it property or property
rights (People vs. Ong Tin, 54 O.G. 7576).

We reiterated this pronouncement in Felipe Ysmael, Jr. & Co., Inc. vs. Deputy Executive Secretary:  26

. . . Timber licenses, permits and license agreements are the principal instruments by which the State
regulates the utilization and disposition of forest resources to the end that public welfare is promoted. And
it can hardly be gainsaid that they merely evidence a privilege granted by the State to qualified entities,
and do not vest in the latter a permanent or irrevocable right to the particular concession area and the
forest products therein. They may be validly amended, modified, replaced or rescinded by the Chief
Executive when national interests so require. Thus, they are not deemed contracts within the purview of
the due process of law clause [See Sections 3(ee) and 20 of Pres. Decree No. 705, as amended. Also,
Tan v. Director of Forestry, G.R. No. L-24548, October 27, 1983, 125 SCRA 302].

Since timber licenses are not contracts, the non-impairment clause, which reads:

Sec. 10. No law impairing, the obligation of contracts shall be passed.  27

cannot be invoked.

In the second place, even if it is to be assumed that the same are contracts, the instant case does not involve a law or
even an executive issuance declaring the cancellation or modification of existing timber licenses. Hence, the non-
impairment clause cannot as yet be invoked. Nevertheless, granting further that a law has actually been passed
mandating cancellations or modifications, the same cannot still be stigmatized as a violation of the non-impairment clause.
This is because by its very nature and purpose, such as law could have only been passed in the exercise of the police
power of the state for the purpose of advancing the right of the people to a balanced and healthful ecology, promoting their
health and enhancing the general welfare. In Abe vs. Foster Wheeler
Corp.   this Court stated:
28

The freedom of contract, under our system of government, is not meant to be absolute. The same is
understood to be subject to reasonable legislative regulation aimed at the promotion of public health,
moral, safety and welfare. In other words, the constitutional guaranty of non-impairment of obligations of
contract is limited by the exercise of the police power of the State, in the interest of public health, safety,
moral and general welfare.

The reason for this is emphatically set forth in Nebia vs. New York,   quoted in Philippine American Life Insurance Co. vs.
29

Auditor General,  to wit:


30

Under our form of government the use of property and the making of contracts are normally matters of
private and not of public concern. The general rule is that both shall be free of governmental interference.
But neither property rights nor contract rights are absolute; for government cannot exist if the citizen may at
will use his property to the detriment of his fellows, or exercise his freedom of contract to work them harm.
Equally fundamental with the private right is that of the public to regulate it in the common interest.

In short, the non-impairment clause must yield to the police power of the state.  31

Finally, it is difficult to imagine, as the trial court did, how the non-impairment clause could apply with respect to the prayer
to enjoin the respondent Secretary from receiving, accepting, processing, renewing or approving new timber licenses for,
save in cases of renewal, no contract would have as of yet existed in the other instances. Moreover, with respect to
renewal, the holder is not entitled to it as a matter of right.

WHEREFORE, being impressed with merit, the instant Petition is hereby GRANTED, and the challenged Order of
respondent Judge of 18 July 1991 dismissing Civil Case No. 90-777 is hereby set aside. The petitioners may therefore
amend their complaint to implead as defendants the holders or grantees of the questioned timber license agreements.

No pronouncement as to costs.
SO ORDERED.

Family as a basic autonomous social institution


-Republic v Albios, 707 SCRA 584 (2013)

This is a petition for review on certiorari under Rule 45 of the Rules t of Court assailing the September 29, 2011
Decision1 of the Court of Appeals (CA), in CA-G.R. CV No. 95414, which affirmed the April 25, 2008Decision 2 of the
Regional Trial Court, Imus, Cavite (RTC). declaring the marriage of Daniel Lee Fringer (Fringer) and respondent Liberty
Albios (A/bios) as void from the beginning.

The facts

On October 22, 2004, Fringer, an American citizen, and Albios were married before Judge Ofelia I. Calo of the
Metropolitan Trial Court, Branch59, Mandaluyong City (MeTC), as evidenced by a Certificate of Marriage with Register No.
2004-1588.3

On December 6, 2006, Albios filed with the RTC a petition for declaration of nullity 4 of her marriage with Fringer. She
alleged that immediately after their marriage, they separated and never lived as husband and wife because they never
really had any intention of entering into a married state or complying with any of their essential marital obligations. She
described their marriage as one made in jest and, therefore, null and void ab initio .

Summons was served on Fringer but he did not file his answer. On September 13, 2007, Albios filed a motion to set case
for pre-trial and to admit her pre-trial brief. The RTC ordered the Assistant Provincial Prosecutor to conduct an
investigation and determine the existence of a collusion. On October 2, 2007, the Assistant Prosecutor complied and
reported that she could not make a determination for failure of both parties to appear at the scheduled investigation.

At the pre-trial, only Albios, her counsel and the prosecutor appeared. Fringer did not attend the hearing despite being
duly notified of the schedule. After the pre-trial, hearing on the merits ensued.

Ruling of the RTC

In its April 25, 2008 Decision,5 the RTC declared the marriage void ab initio, the dispositive portion of which reads:

WHEREFORE, premises considered, judgment is hereby rendered declaring the marriage of Liberty Albios and Daniel
Lee Fringer as void from the very beginning. As a necessary consequence of this pronouncement, petitioner shall cease
using the surname of respondent as she never acquired any right over it and so as to avoid a misimpression that she
remains the wife of respondent.

xxxx

SO ORDERED.6

The RTC was of the view that the parties married each other for convenience only. Giving credence to the testimony of
Albios, it stated that she contracted Fringer to enter into a marriage to enable her to acquire American citizenship; that in
consideration thereof, she agreed to pay him the sum of $2,000.00; that after the ceremony, the parties went their
separate ways; that Fringer returned to the United States and never again communicated with her; and that, in turn, she
did not pay him the $2,000.00 because he never processed her petition for citizenship. The RTC, thus, ruled that when
marriage was entered into for a purpose other than the establishment of a conjugal and family life, such was a farce and
should not be recognized from its inception.

Petitioner Republic of the Philippines, represented by the Office of the Solicitor General (OSG), filed a motion for
reconsideration. The RTC issued the Order, 7 dated February 5, 2009, denying the motion for want of merit. It explained
that the marriage was declared void because the parties failed to freely give their consent to the marriage as they had no
intention to be legally bound by it and used it only as a means to acquire American citizenship in consideration of
$2,000.00.

Not in conformity, the OSG filed an appeal before the CA.

Ruling of the CA

In its assailed decision, dated September 29, 2011, the CA affirmed the RTC ruling which found that the essential requisite
of consent was lacking. The CA stated that the parties clearly did not understand the nature and consequence of getting
married and that their case was similar to a marriage in jest. It further explained that the parties never intended to enter
into the marriage contract and never intended to live as husband and wife or build a family. It concluded that their purpose
was primarily for personal gain, that is, for Albios to obtain foreign citizenship, and for Fringer, the consideration of
$2,000.00.

Hence, this petition.

Assignment of Error
THE COURT OF APPEALS ERRED ON A QUESTION OF LAWWHEN IT HELD THAT A MARRIAGE CONTRACTED
FOR THEPURPOSE OF OBTAINING FOREIGN CITIZENSHIP WAS DONEIN JEST, HENCE, LACKING IN THE
ESSENTIAL ELEMENT OFCONSENT.8

The OSG argues that albeit the intention was for Albios to acquire American citizenship and for Fringer to be paid
$2,000.00, both parties freely gave their consent to the marriage, as they knowingly and willingly entered into that
marriage and knew the benefits and consequences of being bound by it. According to the OSG, consent should be
distinguished from motive, the latter being inconsequential to the validity of marriage.

The OSG also argues that the present case does not fall within the concept of a marriage in jest. The parties here
intentionally consented to enter into a real and valid marriage, for if it were otherwise, the purpose of Albios to acquire
American citizenship would be rendered futile.

On October 29, 2012, Albios filed her Comment 9 to the petition, reiterating her stand that her marriage was similar to a
marriage by way of jest and, therefore, void from the beginning.

On March 22, 2013, the OSG filed its Reply 10 reiterating its arguments in its petition for review on certiorari.

Ruling of the Court

The resolution of this case hinges on this sole question of law: Is a marriage, contracted for the sole purpose of acquiring
American citizenship in consideration of $2,000.00, void ab initio on the ground of lack of consent?

The Court resolves in the negative.

Before the Court delves into its ruling, It shall first examine the phenomenon of marriage fraud for the purposes of
immigration.

Marriage Fraud in Immigration

The institution of marriage carries with it concomitant benefits. This has led to the development of marriage fraud for the
sole purpose of availing of particular benefits. In the United States, marriages where a couple marries only to achieve a
particular purpose or acquire specific benefits, have been referred to as "limited purpose" marriages. 11 A common limited
purpose marriage is one entered into solely for the legitimization of a child. 12 Another, which is the subject of the present
case, is for immigration purposes. Immigration law is usually concerned with the intention of the couple at the time of their
marriage,13 and it attempts to filter out those who use marriage solely to achieve immigration status. 14

In 1975, the seminal case of Bark v. Immigration and Naturalization Service, 15 established the principal test for determining
the presence of marriage fraud in immigration cases. It ruled that a "marriage is a sham if the bride and groom did not
intend to establish a life together at the time they were married. "This standard was modified with the passage of the
Immigration Marriage Fraud Amendment of 1986 (IMFA), which now requires the couple to instead demonstrate that the
marriage was not "entered into for the purpose of evading the immigration laws of the United States." The focus, thus,
shifted from determining the intention to establish a life together, to determining the intention of evading immigration
laws.16 It must be noted, however, that this standard is used purely for immigration purposes and, therefore, does not
purport to rule on the legal validity or existence of a marriage.

The question that then arises is whether a marriage declared as a sham or fraudulent for the limited purpose of
immigration is also legally void and in existent. The early cases on limited purpose marriages in the United States made
no definitive ruling. In 1946, the notable case of

United States v. Rubenstein17 was promulgated, wherein in order to allow an alien to stay in the country, the parties had
agreed to marry but not to live together and to obtain a divorce within six months. The Court, through Judge Learned
Hand, ruled that a marriage to convert temporary into permanent permission to stay in the country was not a marriage,
there being no consent, to wit:

x x x But, that aside, Spitz and Sandler were never married at all. Mutual consent is necessary to every contract; and no
matter what forms or ceremonies the parties may go through indicating the contrary, they do not contract if they do not in
fact assent, which may always be proved. x x x Marriage is no exception to this rule: a marriage in jest is not a marriage at
all. x x x It is quite true that a marriage without subsequent consummation will be valid; but if the spouses agree to a
marriage only for the sake of representing it as such to the outside world and with the understanding that they will put an
end to it as soon as it has served its purpose to deceive, they have never really agreed to be married at all. They must
assent to enter into the relation as it is ordinarily understood, and it is not ordinarily understood as merely a pretence, or
cover, to deceive others.18

(Italics supplied)

On the other end of the spectrum is the 1969 case of Mpiliris v. Hellenic Lines, 19 which declared as valid a marriage
entered into solely for the husband to gain entry to the United States, stating that a valid marriage could not be avoided
"merely because the marriage was entered into for a limited purpose." 20 The 1980 immigration case of Matter of
McKee,21 further recognized that a fraudulent or sham marriage was intrinsically different from a non subsisting one.
Nullifying these limited purpose marriages for lack of consent has, therefore, been recognized as problematic. The
problem being that in order to obtain an immigration benefit, a legal marriage is first necessary. 22 At present, United States
courts have generally denied annulments involving" limited purpose" marriages where a couple married only to achieve a
particular purpose, and have upheld such marriages as valid. 23

The Court now turns to the case at hand.

Respondent’s marriage not void

In declaring the respondent’s marriage void, the RTC ruled that when a marriage was entered into for a purpose other
than the establishment of a conjugal and family life, such was a farce and should not be recognized from its inception. In
its resolution denying the OSG’s motion for reconsideration, the RTC went on to explain that the marriage was declared
void because the parties failed to freely give their consent to the marriage as they had no intention to be legally bound by it
and used it only as a means for the respondent to acquire American citizenship. Agreeing with the RTC, the CA ruled that
the essential requisite of consent was lacking. It held that the parties clearly did not understand the nature and
consequence of getting married. As in the Rubenstein case, the CA found the marriage to be similar to a marriage in jest
considering that the parties only entered into the marriage for the acquisition of American citizenship in exchange of
$2,000.00. They never intended to enter into a marriage contract and never intended to live as husband and wife or build a
family.

The CA’s assailed decision was, therefore, grounded on the parties’ supposed lack of consent. Under Article 2 of the
Family Code, consent is an essential requisite of marriage. Article 4 of the same Code provides that the absence of any
essential requisite shall render a marriage void ab initio.

Under said Article 2, for consent to be valid, it must be (1) freely given and (2) made in the presence of a solemnizing
officer. A "freely given" consent requires that the contracting parties willingly and deliberately enter into the marriage.
Consent must be real in the sense that it is not vitiated nor rendered defective by any of the vices of consent under
Articles45 and 46 of the Family Code, such as fraud, force, intimidation, and undue influence. 24 Consent must also be
conscious or intelligent, in that the parties must be capable of intelligently understanding the nature of, and both the
beneficial or unfavorable consequences of their act.25 Their understanding should not be affected by insanity, intoxication,
drugs, or hypnotism.26

Based on the above, consent was not lacking between Albios and Fringer. In fact, there was real consent because it was
not vitiated nor rendered defective by any vice of consent. Their consent was also conscious and intelligent as they
understood the nature and the beneficial and inconvenient consequences of their marriage, as nothing impaired their
ability to do so. That their consent was freely given is best evidenced by their conscious purpose of acquiring American
citizenship through marriage. Such plainly demonstrates that they willingly and deliberately contracted the marriage. There
was a clear intention to enter into a real and valid marriage so as to fully comply with the requirements of an application for
citizenship. There was a full and complete understanding of the legal tie that would be created between them, since it was
that precise legal tie which was necessary to accomplish their goal.

In ruling that Albios’ marriage was void for lack of consent, the CA characterized such as akin to a marriage by way of jest.
A marriage in jest is a pretended marriage, legal in form but entered into as a joke, with no real intention of entering into
the actual marriage status, and with a clear understanding that the parties would not be bound. The ceremony is not
followed by any conduct indicating a purpose to enter into such a relation. 27 It is a pretended marriage not intended to be
real and with no intention to create any legal ties whatsoever, hence, the absence of any genuine consent. Marriages in
jest are void ab initio, not for vitiated, defective, or unintelligent consent, but for a complete absence of consent. There is
no genuine consent because the parties have absolutely no intention of being bound in any way or for any purpose.

The respondent’s marriage is not at all analogous to a marriage in jest.  Albios and Fringer had an undeniable intention to
1âwphi1

be bound in order to create the very bond necessary to allow the respondent to acquire American citizenship. Only a
genuine consent to be married would allow them to further their objective, considering that only a valid marriage can
properly support an application for citizenship. There was, thus, an apparent intention to enter into the actual marriage
status and to create a legal tie, albeit for a limited purpose. Genuine consent was, therefore, clearly present.

The avowed purpose of marriage under Article 1 of the Family Code is for the couple to establish a conjugal and family
life. The possibility that the parties in a marriage might have no real intention to establish a life together is, however,
insufficient to nullify a marriage freely entered into in accordance with law. The same Article 1 provides that the nature,
consequences, and incidents of marriage are governed by law and not subject to stipulation. A marriage may, thus, only
be declared void or voidable under the grounds provided by law. There is no law that declares a marriage void if it is
entered into for purposes other than what the Constitution or law declares, such as the acquisition of foreign citizenship.
Therefore, so long as all the essential and formal requisites prescribed by law are present, and it is not void or voidable
under the grounds provided by law, it shall be declared valid. 28

Motives for entering into a marriage are varied and complex. The State does not and cannot dictate on the kind of life that
a couple chooses to lead. Any attempt to regulate their lifestyle would go into the realm of their right to privacy and would
raise serious constitutional questions.29 The right to marital privacy allows married couples to structure their marriages in
almost any way they see fit, to live together or live apart, to have children or no children, to love one another or not, and so
on.30 Thus, marriages entered into for other purposes, limited or otherwise, such as convenience, companionship, money,
status, and title, provided that they comply with all the legal requisites, 31 are equally valid. Love, though the ideal
consideration in a marriage contract, is not the only valid cause for marriage. Other considerations, not precluded by law,
may validly support a marriage.
Although the Court views with disdain the respondent’s attempt to utilize marriage for dishonest purposes, It cannot
declare the marriage void. Hence, though the respondent’s marriage may be considered a sham or fraudulent for the
purposes of immigration, it is not void ab initio and continues to be valid and subsisting.

Neither can their marriage be considered voidable on the ground of fraud under Article 45 (3) of the Family Code. Only the
circumstances listed under Article 46 of the same Code may constitute fraud, namely, (1) non- disclosure of a previous
conv1ctwn involving moral turpitude; (2) concealment by the wife of a pregnancy by another man; (3) concealment of a
sexually transmitted disease; and (4) concealment of drug addiction, alcoholism, or homosexuality. No other
misrepresentation or deceit shall constitute fraud as a ground for an action to annul a marriage. Entering into a marriage
for the sole purpose of evading immigration laws does not qualify under any of the listed circumstances. Furthermore,
under Article 47 (3), the ground of fraud may only be brought by the injured or innocent party. In the present case, there is
no injured party because Albios and Fringer both conspired to enter into the sham marriage.

Albios has indeed made a mockery of the sacred institution of marriage. Allowing her marriage with Fringer to be declared
void would only further trivialize this inviolable institution. The Court cannot declare such a marriage void in the event the
parties fail to qualify for immigration benefits, after they have availed of its benefits, or simply have no further use for it.
These unscrupulous individuals cannot be allowed to use the courts as instruments in their fraudulent schemes. Albios
already misused a judicial institution to enter into a marriage of convenience; she should not be allowed to again abuse it
to get herself out of an inconvenient situation.

No less than our Constitution declares that marriage, as an in violable social institution, is the foundation of the family and
shall be protected by the State.32 It must, therefore, be safeguarded from the whims and caprices of the contracting
parties. This Court cannot leave the impression that marriage may easily be entered into when it suits the needs of the
parties, and just as easily nullified when no longer needed.

WHEREFORE, the petition is GRANTED. The September 29, 2011 Decision of the Court of Appeals in CA-G.R. CV No.
95414 is ANNULLED, and Civil Case No. 1134-06 is DISMISSED for utter lack of merit.

SO ORDERED.

Protection of the mother and the unborn


-Imbong v Ochoa, 721 SCRA 146 (2014)

Freedom of religion was accorded preferred status by the framers of our fundamental law. And this Court has consistently
affirmed this preferred status, well aware that it is "designed to protect the broadest possible liberty of conscience, to allow
each man to believe as his conscience directs, to profess his beliefs , and to live as he believes he ought to live,
consistent with the liberty of others and with the common good." 1

To this day, poverty is still a major stumbling block to the nation's emergence as a developed country, leaving our people
beleaguered in a state of hunger, illiteracy and unemployment. While governmental policies have been geared towards the
revitalization of the economy, the bludgeoning dearth in social services remains to be a problem that concerns not only the
poor, but every member of society. The government continues to tread on a trying path to the realization of its very
purpose, that is, the general welfare of the Filipino people and the development of the country as a whole. The legislative
branch, as the main facet of a representative government, endeavors to enact laws and policies that aim to remedy
looming societal woes, while the executive is closed set to fully implement these measures and bring concrete and
substantial solutions within the reach of Juan dela Cruz. Seemingly distant is the judicial branch, oftentimes regarded as
an inert governmental body that merely casts its watchful eyes on clashing stakeholders until it is called upon to
adjudicate. Passive, yet reflexive when called into action, the Judiciary then willingly embarks on its solemn duty to
interpret legislation vis-a-vis the most vital and enduring principle that holds Philippine society together - the supremacy of
the Philippine Constitution.

Nothing has polarized the nation more in recent years than the issues of population growth control, abortion and
contraception. As in every democratic society, diametrically opposed views on the subjects and their perceived
consequences freely circulate in various media. From television debates  to sticker campaigns,  from rallies by socio-
2 3

political activists to mass gatherings organized by members of the clergy  - the clash between the seemingly antithetical
4

ideologies of the religious conservatives and progressive liberals has caused a deep division in every level of the society.
Despite calls to withhold support thereto, however, Republic Act (R.A.) No. 10354, otherwise known as the Responsible
Parenthood and Reproductive Health Act of 2012 (RH Law), was enacted by Congress on December 21, 2012.

Shortly after the President placed his imprimatur on the said law, challengers from various sectors of society came
knocking on the doors of the Court, beckoning it to wield the sword that strikes down constitutional disobedience. Aware of
the profound and lasting impact that its decision may produce, the Court now faces the iuris controversy, as presented in
fourteen (14) petitions and two (2) petitions- in-intervention, to wit:

(1) Petition for Certiorari and Prohibition,  filed by spouses Attys. James M. Imbong and Lovely Ann C. Imbong, in
5

their personal capacities as citizens, lawyers and taxpayers and on behalf of their minor children; and the
Magnificat Child Leaming Center, Inc., a domestic, privately-owned educational institution (Jmbong);
(2) Petition for Prohibition,  filed by the Alliance for the Family Foundation Philippines, Inc., through its president,
6

Atty. Maria Concepcion S. Noche  and several others  in their personal capacities as citizens and on behalf of the
7 8

generations unborn (ALFI);

(3) Petition for Certiorari,  filed by the Task Force for Family and Life Visayas, Inc., and Valeriano S. Avila, in their
9

capacities as citizens and taxpayers (Task Force Family);

(4) Petition for Certiorari and Prohibition,  filed by Serve Life Cagayan De Oro City, Inc.,  Rosevale Foundation,
10 11

Inc.,  a domestic, privately-owned educational institution, and several others,  in their capacities as citizens (Serve
12 13

Life);

(5) Petition,  filed by Expedito A. Bugarin, Jr. in his capacity as a citizen (Bugarin);
14

(6) Petition for Certiorari and Prohibition,  filed by Eduardo Olaguer and the Catholic Xybrspace Apostolate of the
15

Philippines,  in their capacities as a citizens and taxpayers (Olaguer);


16

(7) Petition for Certiorari and Prohibition,  filed by the Philippine Alliance of Xseminarians Inc.,  and several
17 18

others  in their capacities as citizens and taxpayers (PAX);


19

(8) Petition,  filed by Reynaldo J. Echavez, M.D. and several others,  in their capacities as citizens and taxpayers
20 21

(Echavez);

(9) Petition for Certiorari and Prohibition,  filed by spouses Francisco and Maria Fenny C. Tatad and Atty. Alan F.
22

Paguia, in their capacities as citizens, taxpayers and on behalf of those yet unborn. Atty. Alan F. Paguia is also
proceeding in his capacity as a member of the Bar (Tatad);

(10) Petition for Certiorari and Prohibition,  filed by Pro-Life Philippines Foundation Inc.  and several others,  in
23 24 25

their capacities as citizens and taxpayers and on behalf of its associates who are members of the Bar (Pro-Life);

(11) Petition for Prohibition,  filed by Millennium Saint Foundation, Inc.,  Attys. Ramon Pedrosa, Cita Borromeo-
26 27

Garcia, Stella Acedera, and Berteni Catalufia Causing, in their capacities as citizens, taxpayers and members of
the Bar (MSF);

(12) Petition for Certiorari and Prohibition,  filed by John Walter B. Juat and several others,  in their capacities as
28 29

citizens (Juat) ;

(13) Petition for Certiorari and Prohibition,  filed by Couples for Christ Foundation, Inc. and several others,  in their
30 31

capacities as citizens (CFC);

(14) Petition for Prohibition  filed by Almarim Centi Tillah and Abdulhussein M. Kashim in their capacities as
32

citizens and taxpayers (Tillah); and

(15) Petition-In-Intervention,  filed by Atty. Samson S. Alcantara in his capacity as a citizen and a taxpayer
33

(Alcantara); and

(16) Petition-In-Intervention,  filed by Buhay Hayaang Yumabong (B UHAY) , an accredited political party.
34

A perusal of the foregoing petitions shows that the petitioners are assailing the constitutionality of RH Law on the following
GROUNDS:

• The RH Law violates the right to life of the unborn. According to the petitioners, notwithstanding its declared
policy against abortion, the implementation of the RH Law would authorize the purchase of hormonal
contraceptives, intra-uterine devices and injectables which are abortives, in violation of Section 12, Article II of the
Constitution which guarantees protection of both the life of the mother and the life of the unborn from conception. 35

• The RH Law violates the right to health and the right to protection against hazardous products. The petitioners
posit that the RH Law provides universal access to contraceptives which are hazardous to one's health, as it
causes cancer and other health problems. 36

• The RH Law violates the right to religious freedom. The petitioners contend that the RH Law violates the
constitutional guarantee respecting religion as it authorizes the use of public funds for the procurement of
contraceptives. For the petitioners, the use of public funds for purposes that are believed to be contrary to their
beliefs is included in the constitutional mandate ensuring religious freedom. 37

It is also contended that the RH Law threatens conscientious objectors of criminal prosecution, imprisonment and other
forms of punishment, as it compels medical practitioners 1] to refer patients who seek advice on reproductive health
programs to other doctors; and 2] to provide full and correct information on reproductive health programs and service,
although it is against their religious beliefs and convictions. 38
In this connection, Section 5 .23 of the Implementing Rules and Regulations of the RH Law (RH-IRR),  provides that
39

skilled health professionals who are public officers such as, but not limited to, Provincial, City, or Municipal Health Officers,
medical officers, medical specialists, rural health physicians, hospital staff nurses, public health nurses, or rural health
midwives, who are specifically charged with the duty to implement these Rules, cannot be considered as conscientious
objectors. 40

It is also argued that the RH Law providing for the formulation of mandatory sex education in schools should not be
allowed as it is an affront to their religious beliefs. 41

While the petit10ners recognize that the guarantee of religious freedom is not absolute, they argue that the RH Law fails to
satisfy the "clear and present danger test" and the "compelling state interest test" to justify the regulation of the right to
free exercise of religion and the right to free speech. 42

• The RH Law violates the constitutional provision on involuntary servitude. According to the petitioners, the RH
Law subjects medical practitioners to involuntary servitude because, to be accredited under the PhilHealth
program, they are compelled to provide forty-eight (48) hours of pro bona services for indigent women, under
threat of criminal prosecution, imprisonment and other forms of punishment. 43

The petitioners explain that since a majority of patients are covered by PhilHealth, a medical practitioner would effectively
be forced to render reproductive health services since the lack of PhilHealth accreditation would mean that the majority of
the public would no longer be able to avail of the practitioners services. 44

• The RH Law violates the right to equal protection of the law. It is claimed that the RH Law discriminates against
the poor as it makes them the primary target of the government program that promotes contraceptive use. The
petitioners argue that, rather than promoting reproductive health among the poor, the RH Law seeks to introduce
contraceptives that would effectively reduce the number of the poor. 45

• The RH Law is "void-for-vagueness" in violation of the due process clause of the Constitution. In imposing the
penalty of imprisonment and/or fine for "any violation," it is vague because it does not define the type of conduct to
be treated as "violation" of the RH Law. 46

In this connection, it is claimed that "Section 7 of the RH Law violates the right to due process by removing from them (the
people) the right to manage their own affairs and to decide what kind of health facility they shall be and what kind of
services they shall offer."  It ignores the management prerogative inherent in corporations for employers to conduct their
47

affairs in accordance with their own discretion and judgment.

• The RH Law violates the right to free speech. To compel a person to explain a full range of family planning
methods is plainly to curtail his right to expound only his own preferred way of family planning. The petitioners note
that although exemption is granted to institutions owned and operated by religious groups, they are still forced to
refer their patients to another healthcare facility willing to perform the service or procedure.
48

• The RH Law intrudes into the zone of privacy of one's family protected by the Constitution. It is contended that
the RH Law providing for mandatory reproductive health education intrudes upon their constitutional right to raise
their children in accordance with their beliefs. 49

It is claimed that, by giving absolute authority to the person who will undergo reproductive health procedure, the RH Law
forsakes any real dialogue between the spouses and impedes the right of spouses to mutually decide on matters
pertaining to the overall well-being of their family. In the same breath, it is also claimed that the parents of a child who has
suffered a miscarriage are deprived of parental authority to determine whether their child should use contraceptives. 50

• The RH Law violates the constitutional principle of non-delegation of legislative authority. The petitioners
question the delegation by Congress to the FDA of the power to determine whether a product is non-abortifacient
and to be included in the Emergency Drugs List (EDL). 51

• The RH Law violates the one subject/one bill rule provision under Section 26( 1 ), Article VI of the Constitution. 52

• The RH Law violates Natural Law. 53

• The RH Law violates the principle of Autonomy of Local Government Units (LGUs) and the Autonomous Region
of Muslim Mindanao {ARMM). It is contended that the RH Law, providing for reproductive health measures at the
local government level and the ARMM, infringes upon the powers devolved to LGUs and the ARMM under the
Local Government Code and R.A . No. 9054. 54

Various parties also sought and were granted leave to file their respective comments-in-intervention in defense of the
constitutionality of the RH Law. Aside from the Office of the Solicitor General (OSG) which commented on the petitions in
behalf of the respondents,  Congressman Edcel C. Lagman,  former officials of the Department of Health Dr. Esperanza I.
55 56

Cabral, Jamie Galvez-Tan, and Dr. Alberto G. Romualdez,  the Filipino Catholic Voices for Reproductive Health
57

(C4RH),  Ana Theresa "Risa" Hontiveros,  and Atty. Joan De Venecia  also filed their respective Comments-in-
58 59 60

Intervention in conjunction with several others. On June 4, 2013, Senator Pia Juliana S. Cayetano was also granted leave
to intervene. 61
The respondents, aside from traversing the substantive arguments of the petitioners, pray for the dismissal of the petitions
for the principal reasons that 1] there is no actual case or controversy and, therefore, the issues are not yet ripe for judicial
determination.; 2] some petitioners lack standing to question the RH Law; and 3] the petitions are essentially petitions for
declaratory relief over which the Court has no original jurisdiction.

Meanwhile, on March 15, 2013, the RH-IRR for the enforcement of the assailed legislation took effect.

On March 19, 2013, after considering the issues and arguments raised, the Court issued the Status Quo Ante Order
(SQAO), enjoining the effects and implementation of the assailed legislation for a period of one hundred and twenty (120)
days, or until July 17, 2013. 62

On May 30, 2013, the Court held a preliminary conference with the counsels of the parties to determine and/or identify the
pertinent issues raised by the parties and the sequence by which these issues were to be discussed in the oral arguments.
On July 9 and 23, 2013, and on August 6, 13, and 27, 2013, the cases were heard on oral argument. On July 16, 2013,
the SQAO was ordered extended until further orders of the Court. 63

Thereafter, the Court directed the parties to submit their respective memoranda within sixty (60) days and, at the same
time posed several questions for their clarification on some contentions of the parties. 64

The Status Quo Ante

(Population, Contraceptive and Reproductive Health Laws

Prior to the RH Law

Long before the incipience of the RH Law, the country has allowed the sale, dispensation and distribution of contraceptive
drugs and devices. As far back as June 18, 1966, the country enacted R.A. No. 4729 entitled "An Act to Regu,late the
Sale, Dispensation, and/or Distribution of Contraceptive Drugs and Devices." Although contraceptive drugs and devices
were allowed, they could not be sold, dispensed or distributed "unless such sale, dispensation and distribution is by a duly
licensed drug store or pharmaceutical company and with the prescription of a qualified medical practitioner." 65

In addition, R.A. No. 5921,  approved on June 21, 1969, contained provisions relative to "dispensing of abortifacients or
66

anti-conceptional substances and devices." Under Section 37 thereof, it was provided that "no drug or chemical product or
device capable of provoking abortion or preventing conception as classified by the Food and Drug Administration shall be
delivered or sold to any person without a proper prescription by a duly licensed physician."

On December 11, 1967, the Philippines, adhering to the UN Declaration on Population, which recognized that the
population problem should be considered as the principal element for long-term economic development, enacted
measures that promoted male vasectomy and tubal ligation to mitigate population growth.  Among these measures
67

included R.A. No. 6365, approved on August 16, 1971, entitled "An Act Establishing a National Policy on Population,
Creating the Commission on Population and for Other Purposes. " The law envisioned that "family planning will be made
part of a broad educational program; safe and effective means will be provided to couples desiring to space or limit family
size; mortality and morbidity rates will be further reduced."

To further strengthen R.A. No. 6365, then President Ferdinand E . Marcos issued Presidential Decree. (P.D.) No.
79,  dated December 8, 1972, which, among others, made "family planning a part of a broad educational program,"
68

provided "family planning services as a part of over-all health care," and made "available all acceptable methods of
contraception, except abortion, to all Filipino citizens desirous of spacing, limiting or preventing pregnancies."

Through the years, however, the use of contraceptives and family planning methods evolved from being a component of
demographic management, to one centered on the promotion of public health, particularly, reproductive health.  Under 69

that policy, the country gave priority to one's right to freely choose the method of family planning to be adopted, in
conformity with its adherence to the commitments made in the International Conference on Population and
Development.  Thus, on August 14, 2009, the country enacted R.A. No. 9710 or "The Magna Carta for Women, " which,
70

among others, mandated the State to provide for comprehensive health services and programs for women, including
family planning and sex education. 71

The RH Law

Despite the foregoing legislative measures, the population of the country kept on galloping at an uncontrollable pace.
From a paltry number of just over 27 million Filipinos in 1960, the population of the country reached over 76 million in the
year 2000 and over 92 million in 2010.  The executive and the legislative, thus, felt that the measures were still not
72

adequate. To rein in the problem, the RH Law was enacted to provide Filipinos, especially the poor and the marginalized,
access and information to the full range of modem family planning methods, and to ensure that its objective to provide for
the peoples' right to reproductive health be achieved. To make it more effective, the RH Law made it mandatory for health
providers to provide information on the full range of modem family planning methods, supplies and services, and for
schools to provide reproductive health education. To put teeth to it, the RH Law criminalizes certain acts of refusals to
carry out its mandates.

Stated differently, the RH Law is an enhancement measure to fortify and make effective the current laws on contraception,
women's health and population control.
Prayer of the Petitioners - Maintain the Status Quo

The petitioners are one in praying that the entire RH Law be declared unconstitutional. Petitioner ALFI, in particular,
argues that the government sponsored contraception program, the very essence of the RH Law, violates the right to health
of women and the sanctity of life, which the State is mandated to protect and promote. Thus, ALFI prays that "the status
quo ante - the situation prior to the passage of the RH Law - must be maintained."  It explains:
73

x x x. The instant Petition does not question contraception and contraceptives per se. As provided under Republic Act No.
5921 and Republic Act No. 4729, the sale and distribution of contraceptives are prohibited unless dispensed by a
prescription duly licensed by a physician. What the Petitioners find deplorable and repugnant under the RH Law is the role
that the State and its agencies - the entire bureaucracy, from the cabinet secretaries down to the barangay officials in the
remotest areas of the country - is made to play in the implementation of the contraception program to the fullest extent
possible using taxpayers' money. The State then will be the funder and provider of all forms of family planning methods
and the implementer of the program by ensuring the widespread dissemination of, and universal access to, a full range of
family planning methods, devices and supplies. 74

ISSUES

After a scrutiny of the various arguments and contentions of the parties, the Court has synthesized and refined them to the
following principal issues:

I. PROCEDURAL: Whether the Court may exercise its power of judicial review over the controversy.

1] Power of Judicial Review

2] Actual Case or Controversy

3] Facial Challenge

4] Locus Standi

5] Declaratory Relief

6] One Subject/One Title Rule

II. SUBSTANTIVE: Whether the RH law is unconstitutional:

1] Right to Life

2] Right to Health

3] Freedom of Religion and the Right to Free Speech

4] The Family

5] Freedom of Expression and Academic Freedom

6] Due Process

7] Equal Protection

8] Involuntary Servitude

9] Delegation of Authority to the FDA

10] Autonomy of Local Govemments/ARMM

DISCUSSION

Before delving into the constitutionality of the RH Law and its implementing rules, it behooves the Court to resolve some
procedural impediments.

I. PROCEDURAL ISSUE: Whether the Court can exercise its power of judicial review over the controversy.

The Power of Judicial Review

In its attempt to persuade the Court to stay its judicial hand, the OSG asserts that it should submit to the legislative and
political wisdom of Congress and respect the compromises made in the crafting of the RH Law, it being "a product of a
majoritarian democratic process"  and "characterized by an inordinate amount of transparency."  The OSG posits that the
75 76
authority of the Court to review social legislation like the RH Law by certiorari is "weak," since the Constitution vests the
discretion to implement the constitutional policies and positive norms with the political departments, in particular, with
Congress.  It further asserts that in view of the Court's ruling in Southern Hemisphere v. Anti-Terrorism Council,  the
77 78

remedies of certiorari and prohibition utilized by the petitioners are improper to assail the validity of the acts of the
legislature.79

Moreover, the OSG submits that as an "as applied challenge," it cannot prosper considering that the assailed law has yet
to be enforced and applied to the petitioners, and that the government has yet to distribute reproductive health devices
that are abortive. It claims that the RH Law cannot be challenged "on its face" as it is not a speech-regulating measure. 80

In many cases involving the determination of the constitutionality of the actions of the Executive and the Legislature, it is
often sought that the Court temper its exercise of judicial power and accord due respect to the wisdom of its co-equal
branch on the basis of the principle of separation of powers. To be clear, the separation of powers is a fundamental
principle in our system of government, which obtains not through express provision but by actual division in our
Constitution. Each department of the government has exclusive cognizance of matters within its jurisdiction and is
supreme within its own sphere. 81

Thus, the 1987 Constitution provides that: (a) the legislative power shall be vested in the Congress of the Philippines;  (b)82

the executive power shall be vested in the President of the Philippines;  and (c) the judicial power shall be vested in one
83

Supreme Court and in such lower courts as may be established by law.  The Constitution has truly blocked out with deft
84

strokes and in bold lines, the allotment of powers among the three branches of government. 85

In its relationship with its co-equals, the Judiciary recognizes the doctrine of separation of powers which imposes upon the
courts proper restraint, born of the nature of their functions and of their respect for the other branches of government, in
striking down the acts of the Executive or the Legislature as unconstitutional. Verily, the policy is a harmonious blend of
courtesy and caution. 86

It has also long been observed, however, that in times of social disquietude or political instability, the great landmarks of
the Constitution are apt to be forgotten or marred, if not entirely obliterated.  In order to address this, the Constitution
87

impresses upon the Court to respect the acts performed by a co-equal branch done within its sphere of competence and
authority, but at the same time, allows it to cross the line of separation - but only at a very limited and specific point - to
determine whether the acts of the executive and the legislative branches are null because they were undertaken with
grave abuse of discretion.  Thus, while the Court may not pass upon questions of wisdom, justice or expediency of the RH
88

Law, it may do so where an attendant unconstitutionality or grave abuse of discretion results.  The Court must
89

demonstrate its unflinching commitment to protect those cherished rights and principles embodied in the Constitution.

In this connection, it bears adding that while the scope of judicial power of review may be limited, the Constitution makes
no distinction as to the kind of legislation that may be subject to judicial scrutiny, be it in the form of social legislation or
otherwise. The reason is simple and goes back to the earlier point. The Court may pass upon the constitutionality of acts
of the legislative and the executive branches, since its duty is not to review their collective wisdom but, rather, to make
sure that they have acted in consonance with their respective authorities and rights as mandated of them by the
Constitution. If after said review, the Court finds no constitutional violations of any sort, then, it has no more authority of
proscribing the actions under review.  This is in line with Article VIII, Section 1 of the Constitution which expressly
90

provides:

Section 1. The judicial power shall be vested in one Supreme Court and in such lower courts as may be established by
law.

Judicial power includes the duty of the courts of justice to settle actual controversies involving rights which are legally
demandable and enforceable, and to determine whether or not there has been a grave abuse of discretion amounting to
lack or excess of jurisdiction on the part of any branch or instrumentality of the Government. [Emphases supplied]

As far back as Tanada v. Angara,  the Court has unequivocally declared that certiorari, prohibition and mandamus are
91

appropriate remedies to raise constitutional issues and to review and/or prohibit/nullify, when proper, acts of legislative
and executive officials, as there is no other plain, speedy or adequate remedy in the ordinary course of law. This ruling
was later on applied in Macalintal v. COMELEC,  Aldaba v. COMELEC,  Magallona v. Ermita,  and countless others. In
92 93 94

Tanada, the Court wrote:

In seeking to nullify an act of the Philippine Senate on the ground that it contravenes the Constitution, the petition no doubt
raises a justiciable controversy. Where an action of the legislative branch is seriously alleged to have infringed the
Constitution, it becomes not only the right but in fact the duty of the judiciary to settle the dispute. "The question thus
posed is judicial rather than political. The duty (to adjudicate) remains to assure that the supremacy of the Constitution is
upheld. " Once a "controversy as to the application or interpretation of constitutional provision is raised before this Court
(as in the instant case), it becomes a legal issue which the Court is bound by constitutional mandate to decide. [Emphasis
supplied]

In the scholarly estimation of former Supreme Court Justice Florentino Feliciano, "judicial review is essential for the
maintenance and enforcement of the separation of powers and the balancing of powers among the three great
departments of government through the definition and maintenance of the boundaries of authority and control between
them. To him, judicial review is the chief, indeed the only, medium of participation - or instrument of intervention - of the
judiciary in that balancing operation. 95
Lest it be misunderstood, it bears emphasizing that the Court does not have the unbridled authority to rule on just any and
every claim of constitutional violation. Jurisprudence is replete with the rule that the power of judicial review is limited by
four exacting requisites, viz : (a) there must be an actual case or controversy; (b) the petitioners must possess locus
standi; (c) the question of constitutionality must be raised at the earliest opportunity; and (d) the issue of constitutionality
must be the lis mota of the case. 96

Actual Case or Controversy

Proponents of the RH Law submit that the subj ect petitions do not present any actual case or controversy because the
RH Law has yet to be implemented.  They claim that the questions raised by the petitions are not yet concrete and ripe for
97

adjudication since no one has been charged with violating any of its provisions and that there is no showing that any of the
petitioners' rights has been adversely affected by its operation.  In short, it is contended that judicial review of the RH Law
98

is premature.

An actual case or controversy means an existing case or controversy that is appropriate or ripe for determination, not
conjectural or anticipatory, lest the decision of the court would amount to an advisory opinion.  The rule is that courts do
99

not sit to adjudicate mere academic questions to satisfy scholarly interest, however intellectually challenging. The
controversy must be justiciable-definite and concrete, touching on the legal relations of parties having adverse legal
interests. In other words, the pleadings must show an active antagonistic assertion of a legal right, on the one hand, and a
denial thereof, on the other; that is, it must concern a real, tangible and not merely a theoretical question or issue. There
ought to be an actual and substantial controversy admitting of specific relief through a decree conclusive in nature, as
distinguished from an opinion advising what the law would be upon a hypothetical state of facts. 100

Corollary to the requirement of an actual case or controversy is the requirement of ripeness.  A question is ripe for
101

adjudication when the act being challenged has had a direct adverse effect on the individual challenging it. For a case to
be considered ripe for adjudication, it is a prerequisite that something has then been accomplished or performed by either
branch before a court may come into the picture, and the petitioner must allege the existence of an immediate or
threatened injury to himself as a result of the challenged action. He must show that he has sustained or is immediately in
danger of sustaining some direct injury as a result of the act complained of 102

In The Province of North Cotabato v. The Government of the Republic of the Philippines,  where the constitutionality of an
103

unimplemented Memorandum of Agreement on the Ancestral Domain (MOA-AD) was put in question, it was argued that
the Court has no authority to pass upon the issues raised as there was yet no concrete act performed that could possibly
violate the petitioners' and the intervenors' rights. Citing precedents, the Court ruled that the fact of the law or act in
question being not yet effective does not negate ripeness. Concrete acts under a law are not necessary to render the
controversy ripe. Even a singular violation of the Constitution and/or the law is enough to awaken judicial duty.

In this case, the Court is of the view that an actual case or controversy exists and that the same is ripe for judicial
determination. Considering that the RH Law and its implementing rules have already taken effect and that budgetary
measures to carry out the law have already been passed, it is evident that the subject petitions present a justiciable
controversy. As stated earlier, when an action of the legislative branch is seriously alleged to have infringed the
Constitution, it not only becomes a right, but also a duty of the Judiciary to settle the dispute. 104

Moreover, the petitioners have shown that the case is so because medical practitioners or medical providers are in danger
of being criminally prosecuted under the RH Law for vague violations thereof, particularly public health officers who are
threatened to be dismissed from the service with forfeiture of retirement and other benefits. They must, at least, be heard
on the matter NOW.

Facial Challenge

The OSG also assails the propriety of the facial challenge lodged by the subject petitions, contending that the RH Law
cannot be challenged "on its face" as it is not a speech regulating measure. 105

The Court is not persuaded.

In United States (US) constitutional law, a facial challenge, also known as a First Amendment Challenge, is one that is
launched to assail the validity of statutes concerning not only protected speech, but also all other rights in the First
Amendment.  These include religious freedom, freedom of the press, and the right of the people to peaceably assemble,
106

and to petition the Government for a redress of grievances.  After all, the fundamental right to religious freedom, freedom
107

of the press and peaceful assembly are but component rights of the right to one's freedom of expression, as they are
modes which one's thoughts are externalized.

In this jurisdiction, the application of doctrines originating from the U.S. has been generally maintained, albeit with some
modifications. While this Court has withheld the application of facial challenges to strictly penal statues,  it has expanded
108

its scope to cover statutes not only regulating free speech, but also those involving religious freedom, and other
fundamental rights.  The underlying reason for this modification is simple. For unlike its counterpart in the U.S., this Court,
109

under its expanded jurisdiction, is mandated by the Fundamental Law not only to settle actual controversies involving
rights which are legally demandable and enforceable, but also to determine whether or not there has been a grave abuse
of discretion amounting to lack or excess of jurisdiction on the part of any branch or instrumentality of the
Government.  Verily, the framers of Our Constitution envisioned a proactive Judiciary, ever vigilant with its duty to
110

maintain the supremacy of the Constitution.


Consequently, considering that the foregoing petitions have seriously alleged that the constitutional human rights to life,
speech and religion and other fundamental rights mentioned above have been violated by the assailed legislation, the
Court has authority to take cognizance of these kindred petitions and to determine if the RH Law can indeed pass
constitutional scrutiny. To dismiss these petitions on the simple expedient that there exist no actual case or controversy,
would diminish this Court as a reactive branch of government, acting only when the Fundamental Law has been
transgressed, to the detriment of the Filipino people.

Locus Standi

The OSG also attacks the legal personality of the petitioners to file their respective petitions. It contends that the "as
applied challenge" lodged by the petitioners cannot prosper as the assailed law has yet to be enforced and applied against
them,  and the government has yet to distribute reproductive health devices that are abortive.
111 112

The petitioners, for their part, invariably invoke the "transcendental importance" doctrine and their status as citizens and
taxpayers in establishing the requisite locus standi.

Locus standi or legal standing is defined as a personal and substantial interest in a case such that the party has sustained
or will sustain direct injury as a result of the challenged governmental act.  It requires a personal stake in the outcome of
113

the controversy as to assure the concrete adverseness which sharpens the presentation of issues upon which the court so
largely depends for illumination of difficult constitutional questions.
114

In relation to locus standi, the "as applied challenge" embodies the rule that one can challenge the constitutionality of a
statute only if he asserts a violation of his own rights. The rule prohibits one from challenging the constitutionality of the
statute grounded on a violation of the rights of third persons not before the court. This rule is also known as the prohibition
against third-party standing. 115

Transcendental Importance

Notwithstanding, the Court leans on the doctrine that "the rule on standing is a matter of procedure, hence, can be relaxed
for non-traditional plaintiffs like ordinary citizens, taxpayers, and legislators when the public interest so requires, such as
when the matter is of transcendental importance, of overreaching significance to society, or of paramount public interest." 116

In Coconut Oil Refiners Association, Inc. v. Torres,  the Court held that in cases of paramount importance where serious
117

constitutional questions are involved, the standing requirement may be relaxed and a suit may be allowed to prosper even
where there is no direct injury to the party claiming the right of judicial review. In the first Emergency Powers
Cases,  ordinary citizens and taxpayers were allowed to question the constitutionality of several executive orders
118

although they had only an indirect and general interest shared in common with the public.

With these said, even if the constitutionality of the RH Law may not be assailed through an "as-applied challenge, still, the
Court has time and again acted liberally on the locus s tandi requirement. It has accorded certain individuals standing to
sue, not otherwise directly injured or with material interest affected by a Government act, provided a constitutional issue of
transcendental importance is invoked. The rule on locus standi is, after all, a procedural technicality which the Court has,
on more than one occasion, waived or relaxed, thus allowing non-traditional plaintiffs, such as concerned citizens,
taxpayers, voters or legislators, to sue in the public interest, albeit they may not have been directly injured by the operation
of a law or any other government act. As held in Jaworski v. PAGCOR: 119

Granting arguendo that the present action cannot be properly treated as a petition for prohibition, the transcendental
importance of the issues involved in this case warrants that we set aside the technical defects and take primary jurisdiction
over the petition at bar. One cannot deny that the issues raised herein have potentially pervasive influence on the social
and moral well being of this nation, specially the youth; hence, their proper and just determination is an imperative need.
This is in accordance with the well-entrenched principle that rules of procedure are not inflexible tools designed to hinder
or delay, but to facilitate and promote the administration of justice. Their strict and rigid application, which would result in
technicalities that tend to frustrate, rather than promote substantial justice, must always be eschewed. (Emphasis
supplied)

In view of the seriousness, novelty and weight as precedents, not only to the public, but also to the bench and bar, the
issues raised must be resolved for the guidance of all. After all, the RH Law drastically affects the constitutional provisions
on the right to life and health, the freedom of religion and expression and other constitutional rights. Mindful of all these
and the fact that the issues of contraception and reproductive health have already caused deep division among a broad
spectrum of society, the Court entertains no doubt that the petitions raise issues of transcendental importance warranting
immediate court adjudication. More importantly, considering that it is the right to life of the mother and the unborn which is
primarily at issue, the Court need not wait for a life to be taken away before taking action.

The Court cannot, and should not, exercise judicial restraint at this time when rights enshrined in the Constitution are
being imperilled to be violated. To do so, when the life of either the mother or her child is at stake, would lead to
irreparable consequences.

Declaratory Relief

The respondents also assail the petitions because they are essentially petitions for declaratory relief over which the Court
has no original jurisdiction.  Suffice it to state that most of the petitions are praying for injunctive reliefs and so the Court
120
would just consider them as petitions for prohibition under Rule 65, over which it has original jurisdiction. Where the case
has far-reaching implications and prays for injunctive reliefs, the Court may consider them as petitions for prohibition under
Rule 65.121

One Subject-One Title

The petitioners also question the constitutionality of the RH Law, claiming that it violates Section 26(1 ), Article VI of the
Constitution,  prescribing the one subject-one title rule. According to them, being one for reproductive health with
122

responsible parenthood, the assailed legislation violates the constitutional standards of due process by concealing its true
intent - to act as a population control measure. 123

To belittle the challenge, the respondents insist that the RH Law is not a birth or population control measure,  and that the
124

concepts of "responsible parenthood" and "reproductive health" are both interrelated as they are inseparable. 125

Despite efforts to push the RH Law as a reproductive health law, the Court sees it as principally a population control
measure. The corpus of the RH Law is geared towards the reduction of the country's population. While it claims to save
lives and keep our women and children healthy, it also promotes pregnancy-preventing products. As stated earlier, the RH
Law emphasizes the need to provide Filipinos, especially the poor and the marginalized, with access to information on the
full range of modem family planning products and methods. These family planning methods, natural or modem, however,
are clearly geared towards the prevention of pregnancy.

For said reason, the manifest underlying objective of the RH Law is to reduce the number of births in the country.

It cannot be denied that the measure also seeks to provide pre-natal and post-natal care as well. A large portion of the
law, however, covers the dissemination of information and provisions on access to medically-safe, non-abortifacient,
effective, legal, affordable, and quality reproductive health care services, methods, devices, and supplies, which are all
intended to prevent pregnancy.

The Court, thus, agrees with the petitioners' contention that the whole idea of contraception pervades the entire RH Law. It
is, in fact, the central idea of the RH Law.  Indeed, remove the provisions that refer to contraception or are related to it
126

and the RH Law loses its very foundation.  As earlier explained, "the other positive provisions such as skilled birth
127

attendance, maternal care including pre-and post-natal services, prevention and management of reproductive tract
infections including HIV/AIDS are already provided for in the Magna Carta for Women." 128

Be that as it may, the RH Law does not violate the one subject/one bill rule. In Benjamin E. Cawaling, Jr. v. The
Commission on Elections and Rep. Francis Joseph G Escudero, it was written:

It is well-settled that the "one title-one subject" rule does not require the Congress to employ in the title of the enactment
language of such precision as to mirror, fully index or catalogue all the contents and the minute details therein. The rule is
sufficiently complied with if the title is comprehensive enough as to include the general object which the statute seeks to
effect, and where, as here, the persons interested are informed of the nature, scope and consequences of the proposed
law and its operation. Moreover, this Court has invariably adopted a liberal rather than technical construction of the rule
"so as not to cripple or impede legislation." [Emphases supplied]

In this case, a textual analysis of the various provisions of the law shows that both "reproductive health" and "responsible
parenthood" are interrelated and germane to the overriding objective to control the population growth. As expressed in the
first paragraph of Section 2 of the RH Law:

SEC. 2. Declaration of Policy. - The State recognizes and guarantees the human rights of all persons including their right
to equality and nondiscrimination of these rights, the right to sustainable human development, the right to health which
includes reproductive health, the right to education and information, and the right to choose and make decisions for
themselves in accordance with their religious convictions, ethics, cultural beliefs, and the demands of responsible
parenthood.

The one subject/one title rule expresses the principle that the title of a law must not be "so uncertain that the average
person reading it would not be informed of the purpose of the enactment or put on inquiry as to its contents, or which is
misleading, either in referring to or indicating one subject where another or different one is really embraced in the act, or in
omitting any expression or indication of the real subject or scope of the act."129

Considering the close intimacy between "reproductive health" and "responsible parenthood" which bears to the attainment
of the goal of achieving "sustainable human development" as stated under its terms, the Court finds no reason to believe
that Congress intentionally sought to deceive the public as to the contents of the assailed legislation.

II - SUBSTANTIVE ISSUES:

1-The Right to Life


Position of the Petitioners

The petitioners assail the RH Law because it violates the right to life and health of the unborn child under Section 12,
Article II of the Constitution. The assailed legislation allowing access to abortifacients/abortives effectively sanctions
abortion.130
According to the petitioners, despite its express terms prohibiting abortion, Section 4(a) of the RH Law considers
contraceptives that prevent the fertilized ovum to reach and be implanted in the mother's womb as an abortifacient; thus,
sanctioning contraceptives that take effect after fertilization and prior to implantation, contrary to the intent of the Framers
of the Constitution to afford protection to the fertilized ovum which already has life.

They argue that even if Section 9 of the RH Law allows only "non-abortifacient" hormonal contraceptives, intrauterine
devices, injectables and other safe, legal, non-abortifacient and effective family planning products and supplies, medical
research shows that contraceptives use results in abortion as they operate to kill the fertilized ovum which already has
life.
131

As it opposes the initiation of life, which is a fundamental human good, the petitioners assert that the State sanction of
contraceptive use contravenes natural law and is an affront to the dignity of man. 132

Finally, it is contended that since Section 9 of the RH Law requires the Food and Drug Administration (FDA) to certify that
the product or supply is not to be used as an abortifacient, the assailed legislation effectively confirms that abortifacients
are not prohibited. Also considering that the FDA is not the agency that will actually supervise or administer the use of
these products and supplies to prospective patients, there is no way it can truthfully make a certification that it shall not be
used for abortifacient purposes. 133

Position of the Respondents

For their part, the defenders of the RH Law point out that the intent of the Framers of the Constitution was simply the
prohibition of abortion. They contend that the RH Law does not violate the Constitution since the said law emphasizes that
only "non-abortifacient" reproductive health care services, methods, devices products and supplies shall be made
accessible to the public. 134

According to the OSG, Congress has made a legislative determination that contraceptives are not abortifacients by
enacting the RH Law. As the RH Law was enacted with due consideration to various studies and consultations with the
World Health Organization (WHO) and other experts in the medical field, it is asserted that the Court afford deference and
respect to such a determination and pass judgment only when a particular drug or device is later on determined as an
abortive.135

For his part, respondent Lagman argues that the constitutional protection of one's right to life is not violated considering
that various studies of the WHO show that life begins from the implantation of the fertilized ovum. Consequently, he
argues that the RH Law is constitutional since the law specifically provides that only contraceptives that do not prevent the
implantation of the fertilized ovum are allowed. 136

The Court's Position

It is a universally accepted principle that every human being enjoys the right to life. 137

Even if not formally established, the right to life, being grounded on natural law, is inherent and, therefore, not a creation
of, or dependent upon a particular law, custom, or belief. It precedes and transcends any authority or the laws of men.

In this jurisdiction, the right to life is given more than ample protection. Section 1, Article III of the Constitution provides:

Section 1. No person shall be deprived of life, liberty, or property without due process of law, nor shall any person be
denied the equal protection of the laws.

As expounded earlier, the use of contraceptives and family planning methods in the Philippines is not of recent vintage.
From the enactment of R.A. No. 4729, entitled "An Act To Regulate The Sale, Dispensation, and/or Distribution of
Contraceptive Drugs and Devices "on June 18, 1966, prescribing rules on contraceptive drugs and devices which prevent
fertilization,  to the promotion of male vasectomy and tubal ligation,  and the ratification of numerous international
138 139

agreements, the country has long recognized the need to promote population control through the use of contraceptives in
order to achieve long-term economic development. Through the years, however, the use of contraceptives and other
family planning methods evolved from being a component of demographic management, to one centered on the promotion
of public health, particularly, reproductive health. 140

This has resulted in the enactment of various measures promoting women's rights and health and the overall promotion of
the family's well-being. Thus, aside from R.A. No. 4729, R.A. No. 6365 or "The Population Act of the Philippines" and R.A.
No. 9710, otherwise known as the "The Magna Carta of Women" were legislated. Notwithstanding this paradigm shift, the
Philippine national population program has always been grounded two cornerstone principles: "principle of no-abortion"
and the "principle of non-coercion."  As will be discussed later, these principles are not merely grounded on administrative
141

policy, but rather, originates from the constitutional protection expressly provided to afford protection to life and guarantee
religious freedom.

When Life Begins*

Majority of the Members of the Court are of the position that the question of when life begins is a scientific and medical
issue that should not be decided, at this stage, without proper hearing and evidence. During the deliberation, however, it
was agreed upon that the individual members of the Court could express their own views on this matter.
In this regard, the ponente, is of the strong view that life begins at fertilization.

In answering the question of when life begins, focus should be made on the particular phrase of Section 12 which reads:

Section 12. The State recognizes the sanctity of family life and shall protect and strengthen the family as a basic
autonomous social institution. It shall equally protect the life of the mother and the life of the unborn from conception. The
natural and primary right and duty of parents in the rearing of the youth for civic efficiency and the development of moral
character shall receive the support of the Government.

Textually, the Constitution affords protection to the unborn from conception. This is undisputable because before
conception, there is no unborn to speak of. For said reason, it is no surprise that the Constitution is mute as to any
proscription prior to conception or when life begins. The problem has arisen because, amazingly, there are quarters who
have conveniently disregarded the scientific fact that conception is reckoned from fertilization. They are waving the view
that life begins at implantation. Hence, the issue of when life begins.

In a nutshell, those opposing the RH Law contend that conception is synonymous with "fertilization" of the female ovum by
the male sperm.  On the other side of the spectrum are those who assert that conception refers to the "implantation" of
142

the fertilized ovum in the uterus. 143

Plain and Legal Meaning

It is a canon in statutory construction that the words of the Constitution should be interpreted in their plain and ordinary
meaning. As held in the recent case of Chavez v. Judicial Bar Council: 144

One of the primary and basic rules in statutory construction is that where the words of a statute are clear, plain, and free
from ambiguity, it must be given its literal meaning and applied without attempted interpretation. It is a well-settled principle
of constitutional construction that the language employed in the Constitution must be given their ordinary meaning except
where technical terms are employed. As much as possible, the words of the Constitution should be understood in the
sense they have in common use. What it says according to the text of the provision to be construed compels acceptance
and negates the power of the courts to alter it, based on the postulate that the framers and the people mean what they
say. Verba legis non est recedendum - from the words of a statute there should be no departure.

The raison d' etre for the rule is essentially two-fold: First, because it is assumed that the words in which constitutional
provisions are couched express the objective sought to be attained; and second, because the Constitution is not primarily
a lawyer's document but essentially that of the people, in whose consciousness it should ever be present as an important
condition for the rule of law to prevail.

In conformity with the above principle, the traditional meaning of the word "conception" which, as described and defined by
all reliable and reputable sources, means that life begins at fertilization.

Webster's Third New International Dictionary describes it as the act of becoming pregnant, formation of a viable zygote;
the fertilization that results in a new entity capable of developing into a being like its parents.145

Black's Law Dictionary gives legal meaning to the term "conception" as the fecundation of the female ovum by the male
spermatozoon resulting in human life capable of survival and maturation under normal conditions. 146

Even in jurisprudence, an unborn child has already a legal personality. In Continental Steel Manufacturing Corporation v.
Hon. Accredited Voluntary Arbitrator Allan S. Montano,  it was written:
147

Life is not synonymous with civil personality. One need not acquire civil personality first before he/she could die. Even a
child inside the womb already has life. No less than the Constitution recognizes the life of the unborn from conception, that
the State must protect equally with the life of the mother. If the unborn already has life, then the cessation thereof even
prior to the child being delivered, qualifies as death. [Emphases in the original]

In Gonzales v. Carhart,  Justice Anthony Kennedy, writing for the US Supreme Court, said that the State "has respect for
148

human life at all stages in the pregnancy" and "a legitimate and substantial interest in preserving and promoting fetal life."
Invariably, in the decision, the fetus was referred to, or cited, as a baby or a child. 149

Intent of the Framers

Records of the Constitutional Convention also shed light on the intention of the Framers regarding the term "conception"
used in Section 12, Article II of the Constitution. From their deliberations, it clearly refers to the moment of "fertilization."
The records reflect the following:

Rev. Rigos: In Section 9, page 3, there is a sentence which reads:

"The State shall equally protect the life of the mother and the life of the unborn from the moment of conception."

When is the moment of conception?


xxx

Mr. Villegas: As I explained in the sponsorship speech, it is when the ovum is fertilized by the sperm that there is human
life. x x x.
150

xxx

As to why conception is reckoned from fertilization and, as such, the beginning of human life, it was explained:

Mr. Villegas: I propose to review this issue in a biological manner. The first question that needs to be answered is: Is the
fertilized ovum alive? Biologically categorically says yes, the fertilized ovum is alive. First of all, like all living organisms, it
takes in nutrients which it processes by itself. It begins doing this upon fertilization. Secondly, as it takes in these nutrients,
it grows from within. Thirdly, it multiplies itself at a geometric rate in the continuous process of cell division. All these
processes are vital signs of life. Therefore, there is no question that biologically the fertilized ovum has life.

The second question: Is it human? Genetics gives an equally categorical "yes." At the moment of conception, the nuclei of
the ovum and the sperm rupture. As this happens 23 chromosomes from the ovum combine with 23 chromosomes of the
sperm to form a total of 46 chromosomes. A chromosome count of 46 is found only - and I repeat, only in human cells.
Therefore, the fertilized ovum is human.

Since these questions have been answered affirmatively, we must conclude that if the fertilized ovum is both alive and
human, then, as night follows day, it must be human life. Its nature is human. 151

Why the Constitution used the phrase "from the moment of conception" and not "from the moment of fertilization" was not
because of doubt when human life begins, but rather, because:

Mr. Tingson: x x x x the phrase from the moment of conception" was described by us here before with the scientific phrase
"fertilized ovum" may be beyond the comprehension of some people; we want to use the simpler phrase "from the moment
of conception." 152

Thus, in order to ensure that the fertilized ovum is given ample protection under the Constitution, it was discussed:

Rev. Rigos: Yes, we think that the word "unborn" is sufficient for the purpose of writing a Constitution, without specifying
"from the moment of conception."

Mr. Davide: I would not subscribe to that particular view because according to the Commissioner's own admission, he
would leave it to Congress to define when life begins. So, Congress can define life to begin from six months after
fertilization; and that would really be very, very, dangerous. It is now determined by science that life begins from the
moment of conception. There can be no doubt about it. So we should not give any doubt to Congress, too. 153

Upon further inquiry, it was asked:

Mr. Gascon: Mr. Presiding Officer, I would like to ask a question on that point. Actually, that is one of the questions I was
going to raise during the period of interpellations but it has been expressed already. The provision, as proposed right now
states:

The State shall equally protect the life of the mother and the life of the unborn from the moment of conception.

When it speaks of "from the moment of conception," does this mean when the egg meets the sperm?

Mr. Villegas: Yes, the ovum is fertilized by the sperm.

Mr. Gascon: Therefore that does not leave to Congress the right to determine whether certain contraceptives that we know
today are abortifacient or not because it is a fact that some of the so-called contraceptives deter the rooting of the ovum in
the uterus. If fertilization has already occurred, the next process is for the fertilized ovum to travel towards the uterus and
to take root. What happens with some contraceptives is that they stop the opportunity for the fertilized ovum to reach the
uterus. Therefore, if we take the provision as it is proposed, these so called contraceptives should be banned.

Mr. Villegas: Yes, if that physical fact is established, then that is what is called abortifacient and, therefore, would be
unconstitutional and should be banned under this provision.

Mr. Gascon: Yes. So my point is that I do not think it is up to Congress to state whether or not these certain contraceptives
are abortifacient. Scientifically and based on the provision as it is now proposed, they are already considered
abortifacient. 154

From the deliberations above-quoted, it is apparent that the Framers of the Constitution emphasized that the State shall
provide equal protection to both the mother and the unborn child from the earliest opportunity of life, that is, upon
fertilization or upon the union of the male sperm and the female ovum. It is also apparent is that the Framers of the
Constitution intended that to prohibit Congress from enacting measures that would allow it determine when life begins.
Equally apparent, however, is that the Framers of the Constitution did not intend to ban all contraceptives for being
unconstitutional. In fact, Commissioner Bernardo Villegas, spearheading the need to have a constitutional provision on the
right to life, recognized that the determination of whether a contraceptive device is an abortifacient is a question of fact
which should be left to the courts to decide on based on established evidence. 155

From the discussions above, contraceptives that kill or destroy the fertilized ovum should be deemed an abortive and thus
prohibited. Conversely, contraceptives that actually prevent the union of the male sperm and the female ovum, and those
that similarly take action prior to fertilization should be deemed non-abortive, and thus, constitutionally permissible.

As emphasized by the Framers of the Constitution:

x x x           x x x          x x x

Mr. Gascon: xx xx. As I mentioned in my speech on the US bases, I am pro-life, to the point that I would like not only to
protect the life of the unborn, but also the lives of the millions of people in the world by fighting for a nuclear-free world. I
would just like to be assured of the legal and pragmatic implications of the term "protection of the life of the unborn from
the moment of conception." I raised some of these implications this afternoon when I interjected in the interpellation of
Commissioner Regalado. I would like to ask that question again for a categorical answer.

I mentioned that if we institutionalize the term "the life of the unborn from the moment of conception" we are also actually
saying "no," not "maybe," to certain contraceptives which are already being encouraged at this point in time. Is that the
sense of the committee or does it disagree with me?

Mr. Azcuna: No, Mr. Presiding Officer, because contraceptives would be preventive. There is no unborn yet. That is yet
unshaped.

Mr. Gascon: Yes, Mr. Presiding Officer, but I was speaking more about some contraceptives, such as the intra-uterine
device which actually stops the egg which has already been fertilized from taking route to the uterus. So if we say "from
the moment of conception," what really occurs is that some of these contraceptives will have to be unconstitutionalized.

Mr. Azcuna: Yes, to the extent that it is after the fertilization.

Mr. Gascon: Thank you, Mr. Presiding Officer. 156

The fact that not all contraceptives are prohibited by the 1987 Constitution is even admitted by petitioners during the oral
arguments. There it was conceded that tubal ligation, vasectomy, even condoms are not classified as abortifacients. 157

Atty. Noche:

Before the union of the eggs, egg and the sperm, there is no life yet.

Justice Bersamin:

There is no life.

Atty. Noche:

So, there is no life to be protected.

Justice Bersamin:

To be protected.

Atty. Noche:

Under Section 12, yes.

Justice Bersamin:

So you have no objection to condoms?

Atty. Noche:

Not under Section 12, Article II.

Justice Bersamin:

Even if there is already information that condoms sometimes have porosity?


Atty. Noche:

Well, yes, Your Honor, there are scientific findings to that effect, Your Honor, but I am discussing here Section 12, Article
II, Your Honor, yes.

Justice Bersamin:

Alright.

Atty. Noche:

And it's not, I have to admit it's not an abortifacient, Your Honor. 158

Medical Meaning

That conception begins at fertilization is not bereft of medical foundation. Mosby s Medical, Nursing, and Allied Health
Dictionary defines conception as "the beginning of pregnancy usually taken to be the instant a spermatozoon enters an
ovum and forms a viable zygote." 159

It describes fertilization as "the union of male and female gametes to form a zygote from which the embryo develops." 160

The Textbook of Obstetrics (Physiological & Pathological Obstetrics),  used by medical schools in the Philippines, also
161

concludes that human life (human person) begins at the moment of fertilization with the union of the egg and the sperm
resulting in the formation of a new individual, with a unique genetic composition that dictates all developmental stages that
ensue.

Similarly, recent medical research on the matter also reveals that: "Human development begins after the union of male
and female gametes or germ cells during a process known as fertilization (conception). Fertilization is a sequence of
events that begins with the contact of a sperm (spermatozoon) with a secondary oocyte (ovum) and ends with the fusion
of their pronuclei (the haploid nuclei of the sperm and ovum) and the mingling of their chromosomes to form a new cell.
This fertilized ovum, known as a zygote, is a large diploid cell that is the beginning, or primordium, of a human being." 162

The authors of Human Embryology & Teratology  mirror the same position. They wrote: "Although life is a continuous
163

process, fertilization is a critical landmark because, under ordinary circumstances, a new, genetically distinct human
organism is thereby formed.... The combination of 23 chromosomes present in each pronucleus results in 46
chromosomes in the zygote. Thus the diploid number is restored and the embryonic genome is formed. The embryo now
exists as a genetic unity."

In support of the RH Bill, The Philippine Medical Association came out with a "Paper on the Reproductive Health Bill
(Responsible Parenthood Bill)" and therein concluded that:

CONCLUSION

The PMA throws its full weight in supporting the RH Bill at the same time that PMA maintains its strong position that
fertilization is sacred because it is at this stage that conception, and thus human life, begins. Human lives are sacred from
the moment of conception, and that destroying those new lives is never licit, no matter what the purported good outcome
would be. In terms of biology and human embryology, a human being begins immediately at fertilization and after that,
there is no point along the continuous line of human embryogenesis where only a "potential" human being can be posited.
Any philosophical, legal, or political conclusion cannot escape this objective scientific fact.

The scientific evidence supports the conclusion that a zygote is a human organism and that the life of a new human being
commences at a scientifically well defined "moment of conception." This conclusion is objective, consistent with the factual
evidence, and independent of any specific ethical, moral, political, or religious view of human life or of human embryos. 164

Conclusion: The Moment of Conception is Reckoned from


Fertilization

In all, whether it be taken from a plain meaning, or understood under medical parlance, and more importantly, following
the intention of the Framers of the Constitution, the undeniable conclusion is that a zygote is a human organism and that
the life of a new human being commences at a scientifically well-defined moment of conception, that is, upon fertilization.

For the above reasons, the Court cannot subscribe to the theory advocated by Hon. Lagman that life begins at
implantation.  According to him, "fertilization and conception are two distinct and successive stages in the reproductive
165

process. They are not identical and synonymous."  Citing a letter of the WHO, he wrote that "medical authorities confirm
166

that the implantation of the fertilized ovum is the commencement of conception and it is only after implantation that
pregnancy can be medically detected." 167

This theory of implantation as the beginning of life is devoid of any legal or scientific mooring. It does not pertain to the
beginning of life but to the viability of the fetus. The fertilized ovum/zygote is not an inanimate object - it is a living human
being complete with DNA and 46 chromosomes.  Implantation has been conceptualized only for convenience by those
168
who had population control in mind. To adopt it would constitute textual infidelity not only to the RH Law but also to the
Constitution.

Not surprisingly, even the OSG does not support this position.

If such theory would be accepted, it would unnervingly legitimize the utilization of any drug or device that would prevent
the implantation of the fetus at the uterine wall. It would be provocative and further aggravate religious-based divisiveness.

It would legally permit what the Constitution proscribes - abortion and abortifacients.

The RH Law and Abortion

The clear and unequivocal intent of the Framers of the 1987 Constitution in protecting the life of the unborn from
conception was to prevent the Legislature from enacting a measure legalizing abortion. It was so clear that even the Court
cannot interpret it otherwise. This intent of the Framers was captured in the record of the proceedings of the 1986
Constitutional Commission. Commissioner Bernardo Villegas, the principal proponent of the protection of the unborn from
conception, explained:

The intention .. .is to make sure that there would be no pro-abortion laws ever passed by Congress or any pro-abortion
decision passed by the Supreme Court. 169

A reading of the RH Law would show that it is in line with this intent and actually proscribes abortion. While the Court has
opted not to make any determination, at this stage, when life begins, it finds that the RH Law itself clearly mandates that
protection be afforded from the moment of fertilization. As pointed out by Justice Carpio, the RH Law is replete with
provisions that embody the policy of the law to protect to the fertilized ovum and that it should be afforded safe travel to
the uterus for implantation.170

Moreover, the RH Law recognizes that abortion is a crime under Article 256 of the Revised Penal Code, which penalizes
the destruction or expulsion of the fertilized ovum. Thus:

1] xx x.

Section 4. Definition of Terms. - For the purpose of this Act, the following terms shall be defined as follows:

xxx.

(q) Reproductive health care refers to the access to a full range of methods, facilities, services and supplies that contribute
to reproductive health and well-being by addressing reproductive health-related problems. It also includes sexual health,
the purpose of which is the enhancement of life and personal relations. The elements of reproductive health care include
the following:

xxx.

(3) Proscription of abortion and management of abortion complications;

xxx.

2] xx x.

Section 4. x x x.

(s) Reproductive health rights refers to the rights of individuals and couples, to decide freely and responsibly whether or
not to have children; the number, spacing and timing of their children; to make other decisions concerning reproduction,
free of discrimination, coercion and violence; to have the information and means to do so; and to attain the highest
standard of sexual health and reproductive health: Provided, however, That reproductive health rights do not include
abortion, and access to abortifacients.

3] xx x.

SEC. 29. Repealing Clause. - Except for prevailing laws against abortion, any law, presidential decree or issuance,
executive order, letter of instruction, administrative order, rule or regulation contrary to or is inconsistent with the
provisions of this Act including Republic Act No. 7392, otherwise known as the Midwifery Act, is hereby repealed, modified
or amended accordingly.

The RH Law and Abortifacients

In carrying out its declared policy, the RH Law is consistent in prohibiting abortifacients. To be clear, Section 4(a) of the
RH Law defines an abortifacient as:

Section 4. Definition of Terms - x x x x


(a) Abortifacient refers to any drug or device that induces abortion or the destruction of a fetus inside the mother's womb
or the prevention of the fertilized ovum to reach and be implanted in the mother's womb upon determination of the FDA.

As stated above, the RH Law mandates that protection must be afforded from the moment of fertilization. By using the
word " or," the RH Law prohibits not only drugs or devices that prevent implantation, but also those that induce abortion
and those that induce the destruction of a fetus inside the mother's womb. Thus, an abortifacient is any drug or device that
either:

(a) Induces abortion; or

(b) Induces the destruction of a fetus inside the mother's womb; or

(c) Prevents the fertilized ovum to reach and be implanted in the mother's womb, upon determination of the FDA.

Contrary to the assertions made by the petitioners, the Court finds that the RH Law, consistent with the Constitution,
recognizes that the fertilized ovum already has life and that the State has a bounden duty to protect it. The conclusion
becomes clear because the RH Law, first, prohibits any drug or device that induces abortion (first kind), which, as
discussed exhaustively above, refers to that which induces the killing or the destruction of the fertilized ovum, and,
second, prohibits any drug or device the fertilized ovum to reach and be implanted in the mother's womb (third kind).

By expressly declaring that any drug or device that prevents the fertilized ovum to reach and be implanted in the mother's
womb is an abortifacient (third kind), the RH Law does not intend to mean at all that life only begins only at implantation,
as Hon. Lagman suggests. It also does not declare either that protection will only be given upon implantation, as the
petitioners likewise suggest. Rather, it recognizes that: one, there is a need to protect the fertilized ovum which already
has life, and two, the fertilized ovum must be protected the moment it becomes existent - all the way until it reaches and
implants in the mother's womb. After all, if life is only recognized and afforded protection from the moment the fertilized
ovum implants - there is nothing to prevent any drug or device from killing or destroying the fertilized ovum prior to
implantation.

From the foregoing, the Court finds that inasmuch as it affords protection to the fertilized ovum, the RH Law does not
sanction abortion. To repeat, it is the Court's position that life begins at fertilization, not at implantation. When a fertilized
ovum is implanted in the uterine wall , its viability is sustained but that instance of implantation is not the point of beginning
of life. It started earlier. And as defined by the RH Law, any drug or device that induces abortion, that is, which kills or
destroys the fertilized ovum or prevents the fertilized ovum to reach and be implanted in the mother's womb, is an
abortifacient.

Proviso Under Section 9 of the RH Law

This notwithstanding, the Court finds that the proviso under Section 9 of the law that "any product or supply included or to
be included in the EDL must have a certification from the FDA that said product and supply is made available on the
condition that it is not to be used as an abortifacient" as empty as it is absurd. The FDA, with all its expertise, cannot fully
attest that a drug or device will not all be used as an abortifacient, since the agency cannot be present in every instance
when the contraceptive product or supply will be used. 171

Pursuant to its declared policy of providing access only to safe, legal and non-abortifacient contraceptives, however, the
Court finds that the proviso of Section 9, as worded, should bend to the legislative intent and mean that "any product or
supply included or to be included in the EDL must have a certification from the FDA that said product and supply is made
available on the condition that it cannot be used as abortifacient." Such a construction is consistent with the proviso under
the second paragraph of the same section that provides:

Provided, further, That the foregoing offices shall not purchase or acquire by any means emergency contraceptive pills,
postcoital pills, abortifacients that will be used for such purpose and their other forms or equivalent.

Abortifacients under the RH-IRR

At this juncture, the Court agrees with ALFI that the authors of the RH-IRR gravely abused their office when they redefined
the meaning of abortifacient. The RH Law defines "abortifacient" as follows:

SEC. 4. Definition of Terms. - For the purpose of this Act, the following terms shall be defined as follows:

(a) Abortifacient refers to any drug or device that induces abortion or the destruction of a fetus inside the mother's womb
or the prevention of the fertilized ovum to reach and be implanted in the mother's womb upon determination of the FDA.

Section 3.0l (a) of the IRR, however, redefines "abortifacient" as:

Section 3.01 For purposes of these Rules, the terms shall be defined as follows:

a) Abortifacient refers to any drug or device that primarily induces abortion or the destruction of a fetus inside the mother's
womb or the prevention of the fertilized ovum to reach and be implanted in the mother's womb upon determination of the
Food and Drug Administration (FDA). [Emphasis supplied]
Again in Section 3.0lG) of the RH-IRR, "contraceptive," is redefined, viz:

j) Contraceptive refers to any safe, legal, effective and scientifically proven modern family planning method, device, or
health product, whether natural or artificial, that prevents pregnancy but does not primarily destroy a fertilized ovum or
prevent a fertilized ovum from being implanted in the mother's womb in doses of its approved indication as determined by
the Food and Drug Administration (FDA).

The above-mentioned section of the RH-IRR allows "contraceptives" and recognizes as "abortifacient" only those that
primarily induce abortion or the destruction of a fetus inside the mother's womb or the prevention of the fertilized ovum to
reach and be implanted in the mother's womb. 172

This cannot be done.

In this regard, the observations of Justice Brion and Justice Del Castillo are well taken. As they pointed out, with the
insertion of the word "primarily," Section 3.0l(a) and G) of the RH-IRR  must be struck down for being ultra vires.
173

Evidently, with the addition of the word "primarily," in Section 3.0l(a) and G) of the RH-IRR is indeed ultra vires. It
contravenes Section 4(a) of the RH Law and should, therefore, be declared invalid. There is danger that the insertion of
the qualifier "primarily" will pave the way for the approval of contraceptives which may harm or destroy the life of the
unborn from conception/fertilization in violation of Article II, Section 12 of the Constitution. With such qualification in the
RH-IRR, it appears to insinuate that a contraceptive will only be considered as an "abortifacient" if its sole known effect is
abortion or, as pertinent here, the prevention of the implantation of the fertilized ovum.

For the same reason, this definition of "contraceptive" would permit the approval of contraceptives which are actually
abortifacients because of their fail-safe mechanism. 174

Also, as discussed earlier, Section 9 calls for the certification by the FDA that these contraceptives cannot act as abortive.
With this, together with the definition of an abortifacient under Section 4 (a) of the RH Law and its declared policy against
abortion, the undeniable conclusion is that contraceptives to be included in the PNDFS and the EDL will not only be those
contraceptives that do not have the primary action of causing abortion or the destruction of a fetus inside the mother's
womb or the prevention of the fertilized ovum to reach and be implanted in the mother's womb, but also those that do not
have the secondary action of acting the same way.

Indeed, consistent with the constitutional policy prohibiting abortion, and in line with the principle that laws should be
construed in a manner that its constitutionality is sustained, the RH Law and its implementing rules must be consistent
with each other in prohibiting abortion. Thus, the word " primarily" in Section 3.0l(a) and G) of the RH-IRR should be
declared void. To uphold the validity of Section 3.0l(a) and G) of the RH-IRR and prohibit only those contraceptives that
have the primary effect of being an abortive would effectively "open the floodgates to the approval of contraceptives which
may harm or destroy the life of the unborn from conception/fertilization in violation of Article II, Section 12 of the
Constitution."175

To repeat and emphasize, in all cases, the "principle of no abortion" embodied in the constitutional protection of life must
be upheld.

2-The Right to Health

The petitioners claim that the RH Law violates the right to health because it requires the inclusion of hormonal
contraceptives, intrauterine devices, injectables and family products and supplies in the National Drug Formulary and the
inclusion of the same in the regular purchase of essential medicines and supplies of all national hospitals.  Citing various
176

studies on the matter, the petitioners posit that the risk of developing breast and cervical cancer is greatly increased in
women who use oral contraceptives as compared to women who never use them. They point out that the risk is decreased
when the use of contraceptives is discontinued. Further, it is contended that the use of combined oral contraceptive pills is
associated with a threefold increased risk of venous thromboembolism, a twofold increased risk of ischematic stroke, and
an indeterminate effect on risk of myocardial infarction.  Given the definition of "reproductive health" and "sexual health"
177

under Sections 4(p)  and (w)  of the RH Law, the petitioners assert that the assailed legislation only seeks to ensure that
178 179

women have pleasurable and satisfying sex lives. 180

The OSG, however, points out that Section 15, Article II of the Constitution is not self-executory, it being a mere statement
of the administration's principle and policy. Even if it were self-executory, the OSG posits that medical authorities refute
the claim that contraceptive pose a danger to the health of women. 181

The Court's Position

A component to the right to life is the constitutional right to health. In this regard, the Constitution is replete with provisions
protecting and promoting the right to health. Section 15, Article II of the Constitution provides:

Section 15. The State shall protect and promote the right to health of the people and instill health consciousness among
them.

A portion of Article XIII also specifically provides for the States' duty to provide for the health of the people, viz:
HEALTH

Section 11. The State shall adopt an integrated and comprehensive approach to health development which shall endeavor
to make essential goods, health and other social services available to all the people at affordable cost. There shall be
priority for the needs of the underprivileged, sick, elderly, disabled, women, and children. The State shall endeavor to
provide free medical care to paupers.

Section 12. The State shall establish and maintain an effective food and drug regulatory system and undertake
appropriate health, manpower development, and research, responsive to the country's health needs and problems.

Section 13. The State shall establish a special agency for disabled person for their rehabilitation, self-development, and
self-reliance, and their integration into the mainstream of society.

Finally, Section 9, Article XVI provides:

Section 9. The State shall protect consumers from trade malpractices and from substandard or hazardous products.

Contrary to the respondent's notion, however, these provisions are self-executing. Unless the provisions clearly express
the contrary, the provisions of the Constitution should be considered self-executory. There is no need for legislation to
implement these self-executing provisions.  In Manila Prince Hotel v. GSIS,  it was stated:
182 183

x x x Hence, unless it is expressly provided that a legislative act is necessary to enforce a constitutional mandate, the
presumption now is that all provisions of the constitution are self-executing. If the constitutional provisions are treated as
requiring legislation instead of self-executing, the legislature would have the power to ignore and practically nullify the
mandate of the fundamental law. This can be cataclysmic. That is why the prevailing view is, as it has always been, that –

... in case of doubt, the Constitution should be considered self-executing rather than non-self-executing. . . . Unless the
contrary is clearly intended, the provisions of the Constitution should be considered self-executing, as a contrary rule
would give the legislature discretion to determine when, or whether, they shall be effective. These provisions would be
subordinated to the will of the lawmaking body, which could make them entirely meaningless by simply refusing to pass
the needed implementing statute. (Emphases supplied)

This notwithstanding, it bears mentioning that the petitioners, particularly ALFI, do not question contraception and
contraceptives per se.  In fact, ALFI prays that the status quo - under R.A. No. 5921 and R.A. No. 4729, the sale and
184

distribution of contraceptives are not prohibited when they are dispensed by a prescription of a duly licensed by a
physician - be maintained. 185

The legislative intent in the enactment of the RH Law in this regard is to leave intact the provisions of R.A. No. 4729.
There is no intention at all to do away with it. It is still a good law and its requirements are still in to be complied with. Thus,
the Court agrees with the observation of respondent Lagman that the effectivity of the RH Law will not lead to the
unmitigated proliferation of contraceptives since the sale, distribution and dispensation of contraceptive drugs and devices
will still require the prescription of a licensed physician. With R.A. No. 4729 in place, there exists adequate safeguards to
ensure the public that only contraceptives that are safe are made available to the public. As aptly explained by respondent
Lagman:

D. Contraceptives cannot be
dispensed and used without
prescription

108. As an added protection to voluntary users of contraceptives, the same cannot be dispensed and used without
prescription.

109. Republic Act No. 4729 or "An Act to Regulate the Sale, Dispensation, and/ or Distribution of Contraceptive Drugs and
Devices" and Republic Act No. 5921 or "An Act Regulating the Practice of Pharmacy and Setting Standards of
Pharmaceutical Education in the Philippines and for Other Purposes" are not repealed by the RH Law and the provisions
of said Acts are not inconsistent with the RH Law.

110. Consequently, the sale, distribution and dispensation of contraceptive drugs and devices are particularly governed by
RA No. 4729 which provides in full:

"Section 1. It shall be unlawful for any person, partnership, or corporation, to sell, dispense or otherwise distribute whether
for or without consideration, any contraceptive drug or device, unless such sale, dispensation or distribution is by a duly
licensed drug store or pharmaceutical company and with the prescription of a qualified medical practitioner.

"Sec. 2 . For the purpose of this Act:

"(a) "Contraceptive drug" is any medicine, drug, chemical, or portion which is used exclusively for the purpose of
preventing fertilization of the female ovum: and
"(b) "Contraceptive device" is any instrument, device, material, or agent introduced into the female reproductive
system for the primary purpose of preventing conception.

"Sec. 3 Any person, partnership, or corporation, violating the provisions of this Act shall be punished with a fine of not
more than five hundred pesos or an imprisonment of not less than six months or more than one year or both in the
discretion of the Court.

"This Act shall take effect upon its approval.

"Approved: June 18, 1966"

111. Of the same import, but in a general manner, Section 25 of RA No. 5921 provides:

"Section 25. Sale of medicine, pharmaceuticals, drugs and devices. No medicine, pharmaceutical, or drug of whatever
nature and kind or device shall be compounded, dispensed, sold or resold, or otherwise be made available to the
consuming public except through a prescription drugstore or hospital pharmacy, duly established in accordance with the
provisions of this Act.

112. With all of the foregoing safeguards, as provided for in the RH Law and other relevant statutes, the pretension of the
petitioners that the RH Law will lead to the unmitigated proliferation of contraceptives, whether harmful or not, is
completely unwarranted and baseless.  [Emphases in the Original. Underlining supplied.]
186

In Re: Section 10 of the RH Law:

The foregoing safeguards should be read in connection with Section 10 of the RH Law which provides:

SEC. 10. Procurement and Distribution of Family Planning Supplies. - The DOH shall procure, distribute to LGUs and
monitor the usage of family planning supplies for the whole country. The DOH shall coordinate with all appropriate local
government bodies to plan and implement this procurement and distribution program. The supply and budget allotments
shall be based on, among others, the current levels and projections of the following:

(a) Number of women of reproductive age and couples who want to space or limit their children;

(b) Contraceptive prevalence rate, by type of method used; and

(c) Cost of family planning supplies.

Provided, That LGUs may implement its own procurement, distribution and monitoring program consistent with the overall
provisions of this Act and the guidelines of the DOH.

Thus, in the distribution by the DOH of contraceptive drugs and devices, it must consider the provisions of R.A. No. 4729,
which is still in effect, and ensure that the contraceptives that it will procure shall be from a duly licensed drug store or
pharmaceutical company and that the actual dispensation of these contraceptive drugs and devices will done following a
prescription of a qualified medical practitioner. The distribution of contraceptive drugs and devices must not be
indiscriminately done. The public health must be protected by all possible means. As pointed out by Justice De Castro, a
heavy responsibility and burden are assumed by the government in supplying contraceptive drugs and devices, for it may
be held accountable for any injury, illness or loss of life resulting from or incidental to their use.
187

At any rate, it bears pointing out that not a single contraceptive has yet been submitted to the FDA pursuant to the RH
Law. It behooves the Court to await its determination which drugs or devices are declared by the FDA as safe, it being the
agency tasked to ensure that food and medicines available to the public are safe for public consumption. Consequently,
the Court finds that, at this point, the attack on the RH Law on this ground is premature. Indeed, the various kinds of
contraceptives must first be measured up to the constitutional yardstick as expounded herein, to be determined as the
case presents itself.

At this point, the Court is of the strong view that Congress cannot legislate that hormonal contraceptives and intra-uterine
devices are safe and non-abortifacient. The first sentence of Section 9 that ordains their inclusion by the National Drug
Formulary in the EDL by using the mandatory "shall" is to be construed as operative only after they have been tested,
evaluated, and approved by the FDA. The FDA, not Congress, has the expertise to determine whether a particular
hormonal contraceptive or intrauterine device is safe and non-abortifacient. The provision of the third sentence concerning
the requirements for the inclusion or removal of a particular family planning supply from the EDL supports this
construction.

Stated differently, the provision in Section 9 covering the inclusion of hormonal contraceptives, intra-uterine devices,
injectables, and other safe, legal, non-abortifacient and effective family planning products and supplies by the National
Drug Formulary in the EDL is not mandatory. There must first be a determination by the FDA that they are in fact safe,
legal, non-abortifacient and effective family planning products and supplies. There can be no predetermination by
Congress that the gamut of contraceptives are "safe, legal, non-abortifacient and effective" without the proper scientific
examination.
3 -Freedom of Religion
and the Right to Free Speech

Position of the Petitioners:

1. On Contraception

While contraceptives and procedures like vasectomy and tubal ligation are not covered by the constitutional proscription,
there are those who, because of their religious education and background, sincerely believe that contraceptives, whether
abortifacient or not, are evil. Some of these are medical practitioners who essentially claim that their beliefs prohibit not
only the use of contraceptives but also the willing participation and cooperation in all things dealing with contraceptive use.
Petitioner PAX explained that "contraception is gravely opposed to marital chastity, it is contrary to the good of the
transmission of life, and to the reciprocal self-giving of the spouses; it harms true love and denies the sovereign rule of
God in the transmission of Human life." 188

The petitioners question the State-sponsored procurement of contraceptives, arguing that the expenditure of their taxes on
contraceptives violates the guarantee of religious freedom since contraceptives contravene their religious beliefs. 189

2. On Religious Accommodation and


The Duty to Refer

Petitioners Imbong and Luat note that while the RH Law attempts to address religious sentiments by making provisions for
a conscientious objector, the constitutional guarantee is nonetheless violated because the law also imposes upon the
conscientious objector the duty to refer the patient seeking reproductive health services to another medical practitioner
who would be able to provide for the patient's needs. For the petitioners, this amounts to requiring the conscientious
objector to cooperate with the very thing he refuses to do without violating his/her religious beliefs. 190

They further argue that even if the conscientious objector's duty to refer is recognized, the recognition is unduly limited,
because although it allows a conscientious objector in Section 23 (a)(3) the option to refer a patient seeking reproductive
health services and information - no escape is afforded the conscientious objector in Section 23 (a)(l) and (2), i.e. against
a patient seeking reproductive health procedures. They claim that the right of other individuals to conscientiously object,
such as: a) those working in public health facilities referred to in Section 7; b) public officers involved in the implementation
of the law referred to in Section 23(b ); and c) teachers in public schools referred to in Section 14 of the RH Law, are also
not recognize. 191

Petitioner Echavez and the other medical practitioners meanwhile, contend that the requirement to refer the matter to
another health care service provider is still considered a compulsion on those objecting healthcare service providers. They
add that compelling them to do the act against their will violates the Doctrine of Benevolent Neutrality. Sections 9, 14 and
1 7 of the law are too secular that they tend to disregard the religion of Filipinos. Authorizing the use of contraceptives with
abortive effects, mandatory sex education, mandatory pro-bono reproductive health services to indigents encroach upon
the religious freedom of those upon whom they are required. 192

Petitioner CFC also argues that the requirement for a conscientious objector to refer the person seeking reproductive
health care services to another provider infringes on one's freedom of religion as it forces the objector to become an
unwilling participant in the commission of a serious sin under Catholic teachings. While the right to act on one's belief may
be regulated by the State, the acts prohibited by the RH Law are passive acts which produce neither harm nor injury to the
public.
193

Petitioner CFC adds that the RH Law does not show compelling state interest to justify regulation of religious freedom
because it mentions no emergency, risk or threat that endangers state interests. It does not explain how the rights of the
people (to equality, non-discrimination of rights, sustainable human development, health, education, information, choice
and to make decisions according to religious convictions, ethics, cultural beliefs and the demands of responsible
parenthood) are being threatened or are not being met as to justify the impairment of religious freedom. 194

Finally, the petitioners also question Section 15 of the RH Law requiring would-be couples to attend family planning and
responsible parenthood seminars and to obtain a certificate of compliance. They claim that the provision forces individuals
to participate in the implementation of the RH Law even if it contravenes their religious beliefs.  As the assailed law
195

dangles the threat of penalty of fine and/or imprisonment in case of non-compliance with its provisions, the petitioners
claim that the RH Law forcing them to provide, support and facilitate access and information to contraception against their
beliefs must be struck down as it runs afoul to the constitutional guarantee of religious freedom.

The Respondents' Positions

The respondents, on the other hand, contend that the RH Law does not provide that a specific mode or type of
contraceptives be used, be it natural or artificial. It neither imposes nor sanctions any religion or belief.  They point out
196

that the RH Law only seeks to serve the public interest by providing accessible, effective and quality reproductive health
services to ensure maternal and child health, in line with the State's duty to bring to reality the social justice health
guarantees of the Constitution,  and that what the law only prohibits are those acts or practices, which deprive others of
197

their right to reproductive health.  They assert that the assailed law only seeks to guarantee informed choice, which is an
198

assurance that no one will be compelled to violate his religion against his free will.199
The respondents add that by asserting that only natural family planning should be allowed, the petitioners are effectively
going against the constitutional right to religious freedom, the same right they invoked to assail the constitutionality of the
RH Law.  In other words, by seeking the declaration that the RH Law is unconstitutional, the petitioners are asking that
200

the Court recognize only the Catholic Church's sanctioned natural family planning methods and impose this on the entire
citizenry.
201

With respect to the duty to refer, the respondents insist that the same does not violate the constitutional guarantee of
religious freedom, it being a carefully balanced compromise between the interests of the religious objector, on one hand,
who is allowed to keep silent but is required to refer -and that of the citizen who needs access to information and who has
the right to expect that the health care professional in front of her will act professionally. For the respondents, the
concession given by the State under Section 7 and 23(a)(3) is sufficient accommodation to the right to freely exercise
one's religion without unnecessarily infringing on the rights of others.202

Whatever burden is placed on the petitioner's religious freedom is minimal as the duty to refer is limited in duration,
location and impact. 203

Regarding mandatory family planning seminars under Section 15 , the respondents claim that it is a reasonable regulation
providing an opportunity for would-be couples to have access to information regarding parenthood, family planning,
breastfeeding and infant nutrition. It is argued that those who object to any information received on account of their
attendance in the required seminars are not compelled to accept information given to them. They are completely free to
reject any information they do not agree with and retain the freedom to decide on matters of family life without intervention
of the State. 204

For their part, respondents De Venecia et al., dispute the notion that natural family planning is the only method acceptable
to Catholics and the Catholic hierarchy. Citing various studies and surveys on the matter, they highlight the changing
stand of the Catholic Church on contraception throughout the years and note the general acceptance of the benefits of
contraceptives by its followers in planning their families.

The Church and The State

At the outset, it cannot be denied that we all live in a heterogeneous society. It is made up of people of diverse ethnic,
cultural and religious beliefs and backgrounds. History has shown us that our government, in law and in practice, has
allowed these various religious, cultural, social and racial groups to thrive in a single society together. It has embraced
minority groups and is tolerant towards all - the religious people of different sects and the non-believers. The undisputed
fact is that our people generally believe in a deity, whatever they conceived Him to be, and to whom they call for guidance
and enlightenment in crafting our fundamental law. Thus, the preamble of the present Constitution reads:

We, the sovereign Filipino people, imploring the aid of Almighty God, in order to build a just and humane society, and
establish a Government that shall embody our ideals and aspirations, promote the common good, conserve and develop
our patrimony, and secure to ourselves and our posterity, the blessings of independence and democracy under the rule of
law and a regime of truth, justice, freedom, love, equality, and peace, do ordain and promulgate this Constitution.

The Filipino people in "imploring the aid of Almighty God " manifested their spirituality innate in our nature and
consciousness as a people, shaped by tradition and historical experience. As this is embodied in the preamble, it means
that the State recognizes with respect the influence of religion in so far as it instills into the mind the purest principles of
morality.  Moreover, in recognition of the contributions of religion to society, the 1935, 1973 and 1987 constitutions
205

contain benevolent and accommodating provisions towards religions such as tax exemption of church property, salary of
religious officers in government institutions, and optional religious instructions in public schools.

The Framers, however, felt the need to put up a strong barrier so that the State would not encroach into the affairs of the
church, and vice-versa. The principle of separation of Church and State was, thus, enshrined in Article II, Section 6 of the
1987 Constitution, viz:

Section 6. The separation of Church and State shall be inviolable.

Verily, the principle of separation of Church and State is based on mutual respect.  Generally, the State cannot meddle in
1âwphi1

the internal affairs of the church, much less question its faith and dogmas or dictate upon it. It cannot favor one religion
and discriminate against another. On the other hand, the church cannot impose its beliefs and convictions on the State
and the rest of the citizenry. It cannot demand that the nation follow its beliefs, even if it sincerely believes that they are
good for the country.

Consistent with the principle that not any one religion should ever be preferred over another, the Constitution in the above-
cited provision utilizes the term "church" in its generic sense, which refers to a temple, a mosque, an iglesia, or any other
house of God which metaphorically symbolizes a religious organization. Thus, the "Church" means the religious
congregations collectively.

Balancing the benefits that religion affords and the need to provide an ample barrier to protect the State from the pursuit of
its secular objectives, the Constitution lays down the following mandate in Article III, Section 5 and Article VI, Section 29
(2), of the 1987 Constitution:
Section. 5. No law shall be made respecting an establishment of religion, or prohibiting the free exercise thereof. The free
exercise and enjoyment of religious profession and worship, without discrimination or preference, shall forever be allowed.
No religious test shall be required for the exercise of civil or political rights.

Section 29.

xxx.

No public money or property shall be appropriated, applied, paid, or employed, directly or indirectly, for the use, benefit, or
support of any sect, church, denomination, sectarian institution, or system of religion, or of any priest, preacher, minister,
other religious teacher, or dignitary as such, except when such priest, preacher, minister, or dignitary is assigned to the
armed forces, or to any penal institution, or government orphanage or leprosarium.

In short, the constitutional assurance of religious freedom provides two guarantees: the Establishment Clause and the
Free Exercise Clause.

The establishment clause "principally prohibits the State from sponsoring any religion or favoring any religion as against
other religions. It mandates a strict neutrality in affairs among religious groups."  Essentially, it prohibits the establishment
206

of a state religion and the use of public resources for the support or prohibition of a religion.

On the other hand, the basis of the free exercise clause is the respect for the inviolability of the human
conscience.  Under this part of religious freedom guarantee, the State is prohibited from unduly interfering with the
207

outside manifestations of one's belief and faith.  Explaining the concept of religious freedom, the Court, in Victoriano v.
208

Elizalde Rope Workers Union  wrote:


209

The constitutional provisions not only prohibits legislation for the support of any religious tenets or the modes of worship of
any sect, thus forestalling compulsion by law of the acceptance of any creed or the practice of any form of worship (U.S.
Ballard, 322 U.S. 78, 88 L. ed. 1148, 1153), but also assures the free exercise of one's chosen form of religion within limits
of utmost amplitude. It has been said that the religion clauses of the Constitution are all designed to protect the broadest
possible liberty of conscience, to allow each man to believe as his conscience directs, to profess his beliefs, and to live as
he believes he ought to live, consistent with the liberty of others and with the common good. Any legislation whose effect
or purpose is to impede the observance of one or all religions, or to discriminate invidiously between the religions, is
invalid, even though the burden may be characterized as being only indirect. (Sherbert v. Verner, 374 U.S. 398, 10
L.ed.2d 965, 83 S. Ct. 1970) But if the state regulates conduct by enacting, within its power, a general law which has for its
purpose and effect to advance the state's secular goals, the statute is valid despite its indirect burden on religious
observance, unless the state can accomplish its purpose without imposing such burden. (Braunfeld v. Brown, 366 U.S.
599, 6 Led. 2d. 563, 81 S. Ct. 144; McGowan v. Maryland, 366 U.S. 420, 444-5 and 449).

As expounded in Escritor,

The establishment and free exercise clauses were not designed to serve contradictory purposes. They have a single goal-
to promote freedom of individual religious beliefs and practices. In simplest terms, the free exercise clause prohibits
government from inhibiting religious beliefs with penalties for religious beliefs and practice, while the establishment clause
prohibits government from inhibiting religious belief with rewards for religious beliefs and practices. In other words, the two
religion clauses were intended to deny government the power to use either the carrot or the stick to influence individual
religious beliefs and practices. 210

Corollary to the guarantee of free exercise of one's religion is the principle that the guarantee of religious freedom is
comprised of two parts: the freedom to believe, and the freedom to act on one's belief. The first part is absolute. As
explained in Gerona v. Secretary of Education: 211

The realm of belief and creed is infinite and limitless bounded only by one's imagination and thought. So is the freedom of
belief, including religious belief, limitless and without bounds. One may believe in most anything, however strange, bizarre
and unreasonable the same may appear to others, even heretical when weighed in the scales of orthodoxy or doctrinal
standards. But between the freedom of belief and the exercise of said belief, there is quite a stretch of road to travel. 212

The second part however, is limited and subject to the awesome power of the State and can be enjoyed only with proper
regard to the rights of others. It is "subject to regulation where the belief is translated into external acts that affect the
public welfare." 213

Legislative Acts and the

Free Exercise Clause

Thus, in case of conflict between the free exercise clause and the State, the Court adheres to the doctrine of benevolent
neutrality. This has been clearly decided by the Court in Estrada v. Escritor, (Escritor)  where it was stated "that
214

benevolent neutrality-accommodation, whether mandatory or permissive, is the spirit, intent and framework underlying the
Philippine Constitution."  In the same case, it was further explained that"
215

The benevolent neutrality theory believes that with respect to these governmental actions, accommodation of religion may
be allowed, not to promote the government's favored form of religion, but to allow individuals and groups to exercise their
religion without hindrance. "The purpose of accommodation is to remove a burden on, or facilitate the exercise of, a
person's or institution's religion."  "What is sought under the theory of accommodation is not a declaration of
216

unconstitutionality of a facially neutral law, but an exemption from its application or its 'burdensome effect,' whether by the
legislature or the courts."
217

In ascertaining the limits of the exercise of religious freedom, the compelling state interest test is proper.  Underlying the
218

compelling state interest test is the notion that free exercise is a fundamental right and that laws burdening it should be
subject to strict scrutiny.  In Escritor, it was written:
219

Philippine jurisprudence articulates several tests to determine these limits. Beginning with the first case on the Free
Exercise Clause, American Bible Society, the Court mentioned the "clear and present danger" test but did not employ it.
Nevertheless, this test continued to be cited in subsequent cases on religious liberty. The Gerona case then pronounced
that the test of permissibility of religious freedom is whether it violates the established institutions of society and law. The
Victoriano case mentioned the "immediate and grave danger" test as well as the doctrine that a law of general applicability
may burden religious exercise provided the law is the least restrictive means to accomplish the goal of the law. The case
also used, albeit inappropriately, the "compelling state interest" test. After Victoriano , German went back to the Gerona
rule. Ebralinag then employed the "grave and immediate danger" test and overruled the Gerona test. The fairly recent
case of Iglesia ni Cristo went back to the " clear and present danger" test in the maiden case of A merican Bible Society.
Not surprisingly, all the cases which employed the "clear and present danger" or "grave and immediate danger" test
involved, in one form or another, religious speech as this test is often used in cases on freedom of expression. On the
other hand, the Gerona and German cases set the rule that religious freedom will not prevail over established institutions
of society and law. Gerona, however, which was the authority cited by German has been overruled by Ebralinag which
employed the "grave and immediate danger" test . Victoriano was the only case that employed the "compelling state
interest" test, but as explained previously, the use of the test was inappropriate to the facts of the case.

The case at bar does not involve speech as in A merican Bible Society, Ebralinag and Iglesia ni Cristo where the "clear
and present danger" and "grave and immediate danger" tests were appropriate as speech has easily discernible or
immediate effects. The Gerona and German doctrine, aside from having been overruled, is not congruent with the
benevolent neutrality approach, thus not appropriate in this jurisdiction. Similar to Victoriano, the present case involves
purely conduct arising from religious belief. The "compelling state interest" test is proper where conduct is involved for the
whole gamut of human conduct has different effects on the state's interests: some effects may be immediate and short-
term while others delayed and far-reaching. A test that would protect the interests of the state in preventing a substantive
evil, whether immediate or delayed, is therefore necessary. However, not any interest of the state would suffice to prevail
over the right to religious freedom as this is a fundamental right that enjoys a preferred position in the hierarchy of rights -
"the most inalienable and sacred of all human rights", in the words of Jefferson. This right is sacred for an invocation of the
Free Exercise Clause is an appeal to a higher sovereignty. The entire constitutional order of limited government is
premised upon an acknowledgment of such higher sovereignty, thus the Filipinos implore the "aid of Almighty God in order
to build a just and humane society and establish a government." As held in Sherbert, only the gravest abuses,
endangering paramount interests can limit this fundamental right. A mere balancing of interests which balances a right
with just a colorable state interest is therefore not appropriate. Instead, only a compelling interest of the state can prevail
over the fundamental right to religious liberty. The test requires the state to carry a heavy burden, a compelling one, for to
do otherwise would allow the state to batter religion, especially the less powerful ones until they are destroyed. In
determining which shall prevail between the state's interest and religious liberty, reasonableness shall be the guide. The
"compelling state interest" serves the purpose of revering religious liberty while at the same time affording protection to the
paramount interests of the state. This was the test used in Sherbert which involved conduct, i.e. refusal to work on
Saturdays. In the end, the "compelling state interest" test, by upholding the paramount interests of the state, seeks to
protect the very state, without which, religious liberty will not be preserved. [Emphases in the original. Underlining
supplied.]

The Court's Position

In the case at bench, it is not within the province of the Court to determine whether the use of contraceptives or one's
participation in the support of modem reproductive health measures is moral from a religious standpoint or whether the
same is right or wrong according to one's dogma or belief. For the Court has declared that matters dealing with "faith,
practice, doctrine, form of worship, ecclesiastical law, custom and rule of a church ... are unquestionably ecclesiastical
matters which are outside the province of the civil courts."  The jurisdiction of the Court extends only to public and secular
220

morality. Whatever pronouncement the Court makes in the case at bench should be understood only in this realm where it
has authority. Stated otherwise, while the Court stands without authority to rule on ecclesiastical matters, as vanguard of
the Constitution, it does have authority to determine whether the RH Law contravenes the guarantee of religious freedom.

At first blush, it appears that the RH Law recognizes and respects religion and religious beliefs and convictions. It is
replete with assurances the no one can be compelled to violate the tenets of his religion or defy his religious convictions
against his free will. Provisions in the RH Law respecting religious freedom are the following:

1. The State recognizes and guarantees the human rights of all persons including their right to equality and
nondiscrimination of these rights, the right to sustainable human development, the right to health which includes
reproductive health, the right to education and information, and the right to choose and make decisions for themselves in
accordance with their religious convictions, ethics, cultural beliefs, and the demands of responsible parenthood. [Section
2, Declaration of Policy]

2 . The State recognizes marriage as an inviolable social institution and the foundation of the family which in turn is the
foundation of the nation. Pursuant thereto, the State shall defend:
(a) The right of spouses to found a family in accordance with their religious convictions and the demands of responsible
parenthood." [Section 2, Declaration of Policy]

3. The State shall promote and provide information and access, without bias, to all methods of family planning, including
effective natural and modern methods which have been proven medically safe, legal, non-abortifacient, and effective in
accordance with scientific and evidence-based medical research standards such as those registered and approved by the
FDA for the poor and marginalized as identified through the NHTS-PR and other government measures of identifying
marginalization: Provided, That the State shall also provide funding support to promote modern natural methods of family
planning, especially the Billings Ovulation Method, consistent with the needs of acceptors and their religious convictions.
[Section 3(e), Declaration of Policy]

4. The State shall promote programs that: (1) enable individuals and couples to have the number of children they desire
with due consideration to the health, particularly of women, and the resources available and affordable to them and in
accordance with existing laws, public morals and their religious convictions. [Section 3CDJ

5. The State shall respect individuals' preferences and choice of family planning methods that are in accordance with their
religious convictions and cultural beliefs, taking into consideration the State's obligations under various human rights
instruments. [Section 3(h)]

6. Active participation by nongovernment organizations (NGOs) , women's and people's organizations, civil society, faith-
based organizations, the religious sector and communities is crucial to ensure that reproductive health and population and
development policies, plans, and programs will address the priority needs of women, the poor, and the marginalized.
[Section 3(i)]

7. Responsible parenthood refers to the will and ability of a parent to respond to the needs and aspirations of the family
and children. It is likewise a shared responsibility between parents to determine and achieve the desired number of
children, spacing and timing of their children according to their own family life aspirations, taking into account
psychological preparedness, health status, sociocultural and economic concerns consistent with their religious convictions.
[Section 4(v)] (Emphases supplied)

While the Constitution prohibits abortion, laws were enacted allowing the use of contraceptives. To some medical
practitioners, however, the whole idea of using contraceptives is an anathema. Consistent with the principle of benevolent
neutrality, their beliefs should be respected.

The Establishment Clause

and Contraceptives

In the same breath that the establishment clause restricts what the government can do with religion, it also limits what
religious sects can or cannot do with the government. They can neither cause the government to adopt their particular
doctrines as policy for everyone, nor can they not cause the government to restrict other groups. To do so, in simple
terms, would cause the State to adhere to a particular religion and, thus, establishing a state religion.

Consequently, the petitioners are misguided in their supposition that the State cannot enhance its population control
program through the RH Law simply because the promotion of contraceptive use is contrary to their religious beliefs.
Indeed, the State is not precluded to pursue its legitimate secular objectives without being dictated upon by the policies of
any one religion. One cannot refuse to pay his taxes simply because it will cloud his conscience. The demarcation line
between Church and State demands that one render unto Caesar the things that are Caesar's and unto God the things
that are God's.221

The Free Exercise Clause and the Duty to Refer

While the RH Law, in espousing state policy to promote reproductive health manifestly respects diverse religious beliefs in
line with the Non-Establishment Clause, the same conclusion cannot be reached with respect to Sections 7, 23 and 24
thereof. The said provisions commonly mandate that a hospital or a medical practitioner to immediately refer a person
seeking health care and services under the law to another accessible healthcare provider despite their conscientious
objections based on religious or ethical beliefs.

In a situation where the free exercise of religion is allegedly burdened by government legislation or practice, the
compelling state interest test in line with the Court's espousal of the Doctrine of Benevolent Neutrality in Escritor, finds
application. In this case, the conscientious objector's claim to religious freedom would warrant an exemption from
obligations under the RH Law, unless the government succeeds in demonstrating a more compelling state interest in the
accomplishment of an important secular objective. Necessarily so, the plea of conscientious objectors for exemption from
the RH Law deserves no less than strict scrutiny.

In applying the test, the first inquiry is whether a conscientious objector's right to religious freedom has been burdened. As
in Escritor, there is no doubt that an intense tug-of-war plagues a conscientious objector. One side coaxes him into
obedience to the law and the abandonment of his religious beliefs, while the other entices him to a clean conscience yet
under the pain of penalty. The scenario is an illustration of the predicament of medical practitioners whose religious beliefs
are incongruent with what the RH Law promotes.
The Court is of the view that the obligation to refer imposed by the RH Law violates the religious belief and conviction of a
conscientious objector. Once the medical practitioner, against his will, refers a patient seeking information on modem
reproductive health products, services, procedures and methods, his conscience is immediately burdened as he has been
compelled to perform an act against his beliefs. As Commissioner Joaquin A. Bernas (Commissioner Bernas) has written,
"at the basis of the free exercise clause is the respect for the inviolability of the human conscience. 222

Though it has been said that the act of referral is an opt-out clause, it is, however, a false compromise because it makes
pro-life health providers complicit in the performance of an act that they find morally repugnant or offensive. They cannot,
in conscience, do indirectly what they cannot do directly. One may not be the principal, but he is equally guilty if he abets
the offensive act by indirect participation.

Moreover, the guarantee of religious freedom is necessarily intertwined with the right to free speech, it being an
externalization of one's thought and conscience. This in turn includes the right to be silent. With the constitutional
guarantee of religious freedom follows the protection that should be afforded to individuals in communicating their beliefs
to others as well as the protection for simply being silent. The Bill of Rights guarantees the liberty of the individual to utter
what is in his mind and the liberty not to utter what is not in his mind.  While the RH Law seeks to provide freedom of
223

choice through informed consent, freedom of choice guarantees the liberty of the religious conscience and prohibits any
degree of compulsion or burden, whether direct or indirect, in the practice of one's religion. 224

In case of conflict between the religious beliefs and moral convictions of individuals, on one hand, and the interest of the
State, on the other, to provide access and information on reproductive health products, services, procedures and methods
to enable the people to determine the timing, number and spacing of the birth of their children, the Court is of the strong
view that the religious freedom of health providers, whether public or private, should be accorded primacy. Accordingly, a
conscientious objector should be exempt from compliance with the mandates of the RH Law. If he would be compelled to
act contrary to his religious belief and conviction, it would be violative of "the principle of non-coercion" enshrined in the
constitutional right to free exercise of religion.

Interestingly, on April 24, 2013, Scotland's Inner House of the Court of Session, found in the case of Doogan and Wood v.
NHS Greater Glasgow and Clyde Health Board,  that the midwives claiming to be conscientious objectors under the
225

provisions of Scotland's Abortion Act of 1967, could not be required to delegate, supervise or support staff on their labor
ward who were involved in abortions.  The Inner House stated "that if 'participation' were defined according to whether the
226

person was taking part 'directly' or ' indirectly' this would actually mean more complexity and uncertainty." 227

While the said case did not cover the act of referral, the applicable principle was the same - they could not be forced to
assist abortions if it would be against their conscience or will.

Institutional Health Providers

The same holds true with respect to non-maternity specialty hospitals and hospitals owned and operated by a religious
group and health care service providers. Considering that Section 24 of the RH Law penalizes such institutions should
they fail or refuse to comply with their duty to refer under Section 7 and Section 23(a)(3), the Court deems that it must be
struck down for being violative of the freedom of religion. The same applies to Section 23(a)(l) and (a)(2) in relation to
Section 24, considering that in the dissemination of information regarding programs and services and in the performance
of reproductive health procedures, the religious freedom of health care service providers should be respected.

In the case of Islamic Da'wah Council of the Philippines, Inc. v. Office of the Executive Secretary  it was stressed:
228

Freedom of religion was accorded preferred status by the framers of our fundamental law. And this Court has consistently
affirmed this preferred status, well aware that it is "designed to protect the broadest possible liberty of conscience, to allow
each man to believe as his conscience directs, to profess his beliefs, and to live as he believes he ought to live, consistent
with the liberty of others and with the common good." 10

The Court is not oblivious to the view that penalties provided by law endeavour to ensure compliance. Without set
consequences for either an active violation or mere inaction, a law tends to be toothless and ineffectual. Nonetheless,
when what is bartered for an effective implementation of a law is a constitutionally-protected right the Court firmly chooses
to stamp its disapproval. The punishment of a healthcare service provider, who fails and/or refuses to refer a patient to
another, or who declines to perform reproductive health procedure on a patient because incompatible religious beliefs, is a
clear inhibition of a constitutional guarantee which the Court cannot allow.

The Implementing Rules and Regulation (RH-IRR)

The last paragraph of Section 5.24 of the RH-IRR reads:

Provided, That skilled health professional such as provincial, city or municipal health officers, chiefs of hospital, head
nurses, supervising midwives, among others, who by virtue of their office are specifically charged with the duty to
implement the provisions of the RPRH Act and these Rules, cannot be considered as conscientious objectors.

This is discriminatory and violative of the equal protection clause. The conscientious objection clause should be equally
protective of the religious belief of public health officers. There is no perceptible distinction why they should not be
considered exempt from the mandates of the law. The protection accorded to other conscientious objectors should equally
apply to all medical practitioners without distinction whether they belong to the public or private sector. After all, the
freedom to believe is intrinsic in every individual and the protective robe that guarantees its free exercise is not taken off
even if one acquires employment in the government.

It should be stressed that intellectual liberty occupies a place inferior to none in the hierarchy of human values. The mind
must be free to think what it wills, whether in the secular or religious sphere, to give expression to its beliefs by oral
discourse or through the media and, thus, seek other candid views in occasions or gatherings or in more permanent
aggrupation. Embraced in such concept then are freedom of religion, freedom of speech, of the press, assembly and
petition, and freedom of association. 229

The discriminatory provision is void not only because no such exception is stated in the RH Law itself but also because it
is violative of the equal protection clause in the Constitution. Quoting respondent Lagman, if there is any conflict between
the RH-IRR and the RH Law, the law must prevail.

Justice Mendoza:

I'll go to another point. The RH law .. .in your Comment- in-Intervention on page 52, you mentioned RH Law is replete with
provisions in upholding the freedom of religion and respecting religious convictions. Earlier, you affirmed this with
qualifications. Now, you have read, I presumed you have read the IRR-Implementing Rules and Regulations of the RH
Bill?

Congressman Lagman:

Yes, Your Honor, I have read but I have to admit, it's a long IRR and I have not thoroughly dissected the nuances of the
provisions.

Justice Mendoza:

I will read to you one provision. It's Section 5.24. This I cannot find in the RH Law. But in the IRR it says: " .... skilled health
professionals such as provincial, city or municipal health officers, chief of hospitals, head nurses, supervising midwives,
among others, who by virtue of their office are specifically charged with the duty to implement the provisions of the RPRH
Act and these Rules, cannot be considered as conscientious objectors." Do you agree with this?

Congressman Lagman:

I will have to go over again the provisions, Your Honor.

Justice Mendoza:

In other words, public health officers in contrast to the private practitioners who can be conscientious objectors, skilled
health professionals cannot be considered conscientious objectors. Do you agree with this? Is this not against the
constitutional right to the religious belief?

Congressman Lagman:

Your Honor, if there is any conflict between the IRR and the law, the law must prevail. 230

Compelling State Interest

The foregoing discussion then begets the question on whether the respondents, in defense of the subject provisions, were
able to: 1] demonstrate a more compelling state interest to restrain conscientious objectors in their choice of services to
render; and 2] discharge the burden of proof that the obligatory character of the law is the least intrusive means to achieve
the objectives of the law.

Unfortunately, a deep scrutiny of the respondents' submissions proved to be in vain. The OSG was curiously silent in the
establishment of a more compelling state interest that would rationalize the curbing of a conscientious objector's right not
to adhere to an action contrary to his religious convictions. During the oral arguments, the OSG maintained the same
silence and evasion. The Transcripts of the Stenographic Notes disclose the following:

Justice De Castro:

Let's go back to the duty of the conscientious objector to refer. ..

Senior State Solicitor Hilbay:

Yes, Justice.

Justice De Castro:

... which you are discussing awhile ago with Justice Abad. What is the compelling State interest in imposing this duty to
refer to a conscientious objector which refuses to do so because of his religious belief?
Senior State Solicitor Hilbay:

Ahh, Your Honor, ..

Justice De Castro:

What is the compelling State interest to impose this burden?

Senior State Solicitor Hilbay:

In the first place, Your Honor, I don't believe that the standard is a compelling State interest, this is an ordinary health
legislation involving professionals. This is not a free speech matter or a pure free exercise matter. This is a regulation by
the State of the relationship between medical doctors and their patients. 231

Resultantly, the Court finds no compelling state interest which would limit the free exercise clause of the conscientious
objectors, however few in number. Only the prevention of an immediate and grave danger to the security and welfare of
the community can justify the infringement of religious freedom. If the government fails to show the seriousness and
immediacy of the threat, State intrusion is constitutionally unacceptable.232

Freedom of religion means more than just the freedom to believe. It also means the freedom to act or not to act according
to what one believes. And this freedom is violated when one is compelled to act against one's belief or is prevented from
acting according to one's belief.233

Apparently, in these cases, there is no immediate danger to the life or health of an individual in the perceived scenario of
the subject provisions. After all, a couple who plans the timing, number and spacing of the birth of their children refers to a
future event that is contingent on whether or not the mother decides to adopt or use the information, product, method or
supply given to her or whether she even decides to become pregnant at all. On the other hand, the burden placed upon
those who object to contraceptive use is immediate and occurs the moment a patient seeks consultation on reproductive
health matters.

Moreover, granting that a compelling interest exists to justify the infringement of the conscientious objector's religious
freedom, the respondents have failed to demonstrate "the gravest abuses, endangering paramount interests" which could
limit or override a person's fundamental right to religious freedom. Also, the respondents have not presented any
government effort exerted to show that the means it takes to achieve its legitimate state objective is the least intrusive
means.  Other than the assertion that the act of referring would only be momentary, considering that the act of referral by
234

a conscientious objector is the very action being contested as violative of religious freedom, it behooves the respondents
to demonstrate that no other means can be undertaken by the State to achieve its objective without violating the rights of
the conscientious objector. The health concerns of women may still be addressed by other practitioners who may perform
reproductive health-related procedures with open willingness and motivation. Suffice it to say, a person who is forced to
perform an act in utter reluctance deserves the protection of the Court as the last vanguard of constitutional freedoms.

At any rate, there are other secular steps already taken by the Legislature to ensure that the right to health is protected.
Considering other legislations as they stand now, R.A . No. 4 729 or the Contraceptive Act, R.A. No. 6365 or "The
Population Act of the Philippines" and R.A. No. 9710, otherwise known as "The Magna Carta of Women," amply cater to
the needs of women in relation to health services and programs. The pertinent provision of Magna Carta on
comprehensive health services and programs for women, in fact, reads:

Section 17. Women's Right to Health. - (a) Comprehensive Health Services. - The State shall, at all times, provide for a
comprehensive, culture-sensitive, and gender-responsive health services and programs covering all stages of a woman's
life cycle and which addresses the major causes of women's mortality and morbidity: Provided, That in the provision for
comprehensive health services, due respect shall be accorded to women's religious convictions, the rights of the spouses
to found a family in accordance with their religious convictions, and the demands of responsible parenthood, and the right
of women to protection from hazardous drugs, devices, interventions, and substances.

Access to the following services shall be ensured:

(1) Maternal care to include pre- and post-natal services to address pregnancy and infant health and nutrition;

(2) Promotion of breastfeeding;

(3) Responsible, ethical, legal, safe, and effective methods of family planning;

(4) Family and State collaboration in youth sexuality education and health services without prejudice to the primary
right and duty of parents to educate their children;

(5) Prevention and management of reproductive tract infections, including sexually transmitted diseases, HIV, and
AIDS;

(6) Prevention and management of reproductive tract cancers like breast and cervical cancers, and other
gynecological conditions and disorders;
(7) Prevention of abortion and management of pregnancy-related complications;

(8) In cases of violence against women and children, women and children victims and survivors shall be provided
with comprehensive health services that include psychosocial, therapeutic, medical, and legal interventions and
assistance towards healing, recovery, and empowerment;

(9) Prevention and management of infertility and sexual dysfunction pursuant to ethical norms and medical
standards;

(10) Care of the elderly women beyond their child-bearing years; and

(11) Management, treatment, and intervention of mental health problems of women and girls. In addition, healthy
lifestyle activities are encouraged and promoted through programs and projects as strategies in the prevention of
diseases.

(b) Comprehensive Health Information and Education. - The State shall provide women in all sectors with appropriate,
timely, complete, and accurate information and education on all the above-stated aspects of women's health in
government education and training programs, with due regard to the following:

(1) The natural and primary right and duty of parents in the rearing of the youth and the development of moral
character and the right of children to be brought up in an atmosphere of morality and rectitude for the enrichment
and strengthening of character;

(2) The formation of a person's sexuality that affirms human dignity; and

(3) Ethical, legal, safe, and effective family planning methods including fertility awareness.

As an afterthought, Asst. Solicitor General Hilbay eventually replied that the compelling state interest was "Fifteen
maternal deaths per day, hundreds of thousands of unintended pregnancies, lives changed, x x x."  He, however, failed to
235

substantiate this point by concrete facts and figures from reputable sources.

The undisputed fact, however, is that the World Health Organization reported that the Filipino maternal mortality rate
dropped to 48 percent from 1990 to 2008,   although there was still no RH Law at that time. Despite such revelation, the
236

proponents still insist that such number of maternal deaths constitute a compelling state interest.

Granting that there are still deficiencies and flaws in the delivery of social healthcare programs for Filipino women, they
could not be solved by a measure that puts an unwarrantable stranglehold on religious beliefs in exchange for blind
conformity.

Exception: Life Threatening Cases

All this notwithstanding, the Court properly recognizes a valid exception set forth in the law. While generally healthcare
service providers cannot be forced to render reproductive health care procedures if doing it would contravene their
religious beliefs, an exception must be made in life-threatening cases that require the performance of emergency
procedures. In these situations, the right to life of the mother should be given preference, considering that a referral by a
medical practitioner would amount to a denial of service, resulting to unnecessarily placing the life of a mother in grave
danger. Thus, during the oral arguments, Atty. Liban, representing CFC, manifested: "the forced referral clause that we
are objecting on grounds of violation of freedom of religion does not contemplate an emergency." 237

In a conflict situation between the life of the mother and the life of a child, the doctor is morally obliged always to try to
save both lives. If, however, it is impossible, the resulting death to one should not be deliberate. Atty. Noche explained:

Principle of Double-Effect. - May we please remind the principal author of the RH Bill in the House of Representatives of
the principle of double-effect wherein intentional harm on the life of either the mother of the child is never justified to bring
about a "good" effect. In a conflict situation between the life of the child and the life of the mother, the doctor is morally
obliged always to try to save both lives. However, he can act in favor of one (not necessarily the mother) when it is
medically impossible to save both, provided that no direct harm is intended to the other. If the above principles are
observed, the loss of the child's life or the mother's life is not intentional and, therefore, unavoidable. Hence, the doctor
would not be guilty of abortion or murder. The mother is never pitted against the child because both their lives are equally
valuable.238

Accordingly, if it is necessary to save the life of a mother, procedures endangering the life of the child may be resorted to
even if is against the religious sentiments of the medical practitioner. As quoted above, whatever burden imposed upon a
medical practitioner in this case would have been more than justified considering the life he would be able to save.

Family Planning Seminars

Anent the requirement imposed under Section 15  as a condition for the issuance of a marriage license, the Court finds
239

the same to be a reasonable exercise of police power by the government. A cursory reading of the assailed provision
bares that the religious freedom of the petitioners is not at all violated. All the law requires is for would-be spouses to
attend a seminar on parenthood, family planning breastfeeding and infant nutrition. It does not even mandate the type of
family planning methods to be included in the seminar, whether they be natural or artificial. As correctly noted by the OSG,
those who receive any information during their attendance in the required seminars are not compelled to accept the
information given to them, are completely free to reject the information they find unacceptable, and retain the freedom to
decide on matters of family life without the intervention of the State.

4-The Family and the Right to Privacy

Petitioner CFC assails the RH Law because Section 23(a) (2) (i) thereof violates the provisions of the Constitution by
intruding into marital privacy and autonomy. It argues that it cultivates disunity and fosters animosity in the family rather
than promote its solidarity and total development. 240

The Court cannot but agree.

The 1987 Constitution is replete with provisions strengthening the family as it is the basic social institution. In fact, one
article, Article XV, is devoted entirely to the family.

ARTICLE XV
THE FAMILY

Section 1. The State recognizes the Filipino family as the foundation of the nation. Accordingly, it shall strengthen its
solidarity and actively promote its total development.

Section 2. Marriage, as an inviolable social institution, is the foundation of the family and shall be protected by the State.

Section 3. The State shall defend:

The right of spouses to found a family in accordance with their religious convictions and the demands of responsible
parenthood;

The right of children to assistance, including proper care and nutrition, and special protection from all forms of neglect,
abuse, cruelty, exploitation and other conditions prejudicial to their development;

The right of the family to a family living wage and income; and

The right of families or family assoc1at1ons to participate in the planning and implementation of policies and programs that
affect them.

In this case, the RH Law, in its not-so-hidden desire to control population growth, contains provisions which tend to wreck
the family as a solid social institution. It bars the husband and/or the father from participating in the decision making
process regarding their common future progeny. It likewise deprives the parents of their authority over their minor
daughter simply because she is already a parent or had suffered a miscarriage.

The Family and Spousal Consent

Section 23(a) (2) (i) of the RH Law states:

The following acts are prohibited:

(a) Any health care service provider, whether public or private, who shall: ...

(2) refuse to perform legal and medically-safe reproductive health procedures on any person of legal age on the ground of
lack of consent or authorization of the following persons in the following instances:

(i) Spousal consent in case of married persons: provided, That in case of disagreement, the decision of the one
undergoing the procedures shall prevail. [Emphasis supplied]

The above provision refers to reproductive health procedures like tubal litigation and vasectomy which, by their very
nature, should require mutual consent and decision between the husband and the wife as they affect issues intimately
related to the founding of a family. Section 3, Art. XV of the Constitution espouses that the State shall defend the "right of
the spouses to found a family." One person cannot found a family. The right, therefore, is shared by both spouses. In the
same Section 3, their right "to participate in the planning and implementation of policies and programs that affect them " is
equally recognized.

The RH Law cannot be allowed to infringe upon this mutual decision-making. By giving absolute authority to the spouse
who would undergo a procedure, and barring the other spouse from participating in the decision would drive a wedge
between the husband and wife, possibly result in bitter animosity, and endanger the marriage and the family, all for the
sake of reducing the population. This would be a marked departure from the policy of the State to protect marriage as an
inviolable social institution.
241
Decision-making involving a reproductive health procedure is a private matter which belongs to the couple, not just one of
them. Any decision they would reach would affect their future as a family because the size of the family or the number of
their children significantly matters. The decision whether or not to undergo the procedure belongs exclusively to, and
shared by, both spouses as one cohesive unit as they chart their own destiny. It is a constitutionally guaranteed private
right. Unless it prejudices the State, which has not shown any compelling interest, the State should see to it that they chart
their destiny together as one family.

As highlighted by Justice Leonardo-De Castro, Section 19( c) of R.A. No. 9710, otherwise known as the "Magna Carta for
Women," provides that women shall have equal rights in all matters relating to marriage and family relations, including the
joint decision on the number and spacing of their children. Indeed, responsible parenthood, as Section 3(v) of the RH Law
states, is a shared responsibility between parents. Section 23(a)(2)(i) of the RH Law should not be allowed to betray the
constitutional mandate to protect and strengthen the family by giving to only one spouse the absolute authority to decide
whether to undergo reproductive health procedure. 242

The right to chart their own destiny together falls within the protected zone of marital privacy and such state intervention
would encroach into the zones of spousal privacy guaranteed by the Constitution. In our jurisdiction, the right to privacy
was first recognized in Marje v. Mutuc,  where the Court, speaking through Chief Justice Fernando, held that "the right to
243

privacy as such is accorded recognition independently of its identification with liberty; in itself, it is fully deserving of
constitutional protection."  Marje adopted the ruling of the US Supreme Court in Griswold v. Connecticut,  where Justice
244 245

William O. Douglas wrote:

We deal with a right of privacy older than the Bill of Rights -older than our political parties, older than our school system.
Marriage is a coming together for better or for worse, hopefully enduring, and intimate to the degree of being sacred. It is
an association that promotes a way of life, not causes; a harmony in living, not political faiths; a bilateral loyalty, not
commercial or social projects. Yet it is an association for as noble a purpose as any involved in our prior decisions.

Ironically, Griswold invalidated a Connecticut statute which made the use of contraceptives a criminal offense on the
ground of its amounting to an unconstitutional invasion of the right to privacy of married persons. Nevertheless, it
recognized the zone of privacy rightfully enjoyed by couples. Justice Douglas in Grisworld wrote that "specific guarantees
in the Bill of Rights have penumbras, formed by emanations from those guarantees that help give them life and substance.
Various guarantees create zones of privacy." 246

At any rate, in case of conflict between the couple, the courts will decide.

The Family and Parental Consent

Equally deplorable is the debarment of parental consent in cases where the minor, who will be undergoing a procedure, is
already a parent or has had a miscarriage. Section 7 of the RH law provides:

SEC. 7. Access to Family Planning. – x x x.

No person shall be denied information and access to family planning services, whether natural or artificial: Provided, That
minors will not be allowed access to modern methods of family planning without written consent from their parents or
guardian/s except when the minor is already a parent or has had a miscarriage.

There can be no other interpretation of this provision except that when a minor is already a parent or has had a
miscarriage, the parents are excluded from the decision making process of the minor with regard to family planning. Even
if she is not yet emancipated, the parental authority is already cut off just because there is a need to tame population
growth.

It is precisely in such situations when a minor parent needs the comfort, care, advice, and guidance of her own parents.
The State cannot replace her natural mother and father when it comes to providing her needs and comfort. To say that
their consent is no longer relevant is clearly anti-family. It does not promote unity in the family. It is an affront to the
constitutional mandate to protect and strengthen the family as an inviolable social institution.

More alarmingly, it disregards and disobeys the constitutional mandate that "the natural and primary right and duty of
parents in the rearing of the youth for civic efficiency and the development of moral character shall receive the support of
the Government."  In this regard, Commissioner Bernas wrote:
247

The 1987 provision has added the adjective "primary" to modify the right of parents. It imports the assertion that the right
of parents is superior to that of the State.  [Emphases supplied]
248

To insist on a rule that interferes with the right of parents to exercise parental control over their minor-child or the right of
the spouses to mutually decide on matters which very well affect the very purpose of marriage, that is, the establishment
of conjugal and family life, would result in the violation of one's privacy with respect to his family. It would be dismissive of
the unique and strongly-held Filipino tradition of maintaining close family ties and violative of the recognition that the State
affords couples entering into the special contract of marriage to as one unit in forming the foundation of the family and
society.
The State cannot, without a compelling state interest, take over the role of parents in the care and custody of a minor
child, whether or not the latter is already a parent or has had a miscarriage. Only a compelling state interest can justify a
state substitution of their parental authority.

First Exception: Access to Information

Whether with respect to the minor referred to under the exception provided in the second paragraph of Section 7 or with
respect to the consenting spouse under Section 23(a)(2)(i), a distinction must be made. There must be a differentiation
between access to information about family planning services, on one hand, and access to the reproductive health
procedures and modern family planning methods themselves, on the other. Insofar as access to information is concerned,
the Court finds no constitutional objection to the acquisition of information by the minor referred to under the exception in
the second paragraph of Section 7 that would enable her to take proper care of her own body and that of her unborn child.
After all, Section 12, Article II of the Constitution mandates the State to protect both the life of the mother as that of the
unborn child. Considering that information to enable a person to make informed decisions is essential in the protection and
maintenance of ones' health, access to such information with respect to reproductive health must be allowed. In this
situation, the fear that parents might be deprived of their parental control is unfounded because they are not prohibited to
exercise parental guidance and control over their minor child and assist her in deciding whether to accept or reject the
information received.

Second Exception: Life Threatening Cases

As in the case of the conscientious objector, an exception must be made in life-threatening cases that require the
performance of emergency procedures. In such cases, the life of the minor who has already suffered a miscarriage and
that of the spouse should not be put at grave risk simply for lack of consent. It should be emphasized that no person
should be denied the appropriate medical care urgently needed to preserve the primordial right, that is, the right to life.

In this connection, the second sentence of Section 23(a)(2)(ii)  should be struck down. By effectively limiting the
249

requirement of parental consent to "only in elective surgical procedures," it denies the parents their right of parental
authority in cases where what is involved are "non-surgical procedures." Save for the two exceptions discussed above,
and in the case of an abused child as provided in the first sentence of Section 23(a)(2)(ii), the parents should not be
deprived of their constitutional right of parental authority. To deny them of this right would be an affront to the constitutional
mandate to protect and strengthen the family.

5 - Academic Freedom

It is asserted that Section 14 of the RH Law, in relation to Section 24 thereof, mandating the teaching of Age-and
Development-Appropriate Reproductive Health Education under threat of fine and/or imprisonment violates the principle of
academic freedom . According to the petitioners, these provisions effectively force educational institutions to teach
reproductive health education even if they believe that the same is not suitable to be taught to their students.  Citing
250

various studies conducted in the United States and statistical data gathered in the country, the petitioners aver that the
prevalence of contraceptives has led to an increase of out-of-wedlock births; divorce and breakdown of families; the
acceptance of abortion and euthanasia; the "feminization of poverty"; the aging of society; and promotion of promiscuity
among the youth. 251

At this point, suffice it to state that any attack on the validity of Section 14 of the RH Law is premature because the
Department of Education, Culture and Sports has yet to formulate a curriculum on age-appropriate reproductive health
education. One can only speculate on the content, manner and medium of instruction that will be used to educate the
adolescents and whether they will contradict the religious beliefs of the petitioners and validate their apprehensions. Thus,
considering the premature nature of this particular issue, the Court declines to rule on its constitutionality or validity.

At any rate, Section 12, Article II of the 1987 Constitution provides that the natural and primary right and duty of parents in
the rearing of the youth for civic efficiency and development of moral character shall receive the support of the
Government. Like the 1973 Constitution and the 1935 Constitution, the 1987 Constitution affirms the State recognition of
the invaluable role of parents in preparing the youth to become productive members of society. Notably, it places more
importance on the role of parents in the development of their children by recognizing that said role shall be "primary," that
is, that the right of parents in upbringing the youth is superior to that of the State.
252

It is also the inherent right of the State to act as parens patriae to aid parents in the moral development of the youth.
Indeed, the Constitution makes mention of the importance of developing the youth and their important role in nation
building.  Considering that Section 14 provides not only for the age-appropriate-reproductive health education, but also
253

for values formation; the development of knowledge and skills in self-protection against discrimination; sexual abuse and
violence against women and children and other forms of gender based violence and teen pregnancy; physical, social and
emotional changes in adolescents; women's rights and children's rights; responsible teenage behavior; gender and
development; and responsible parenthood, and that Rule 10, Section 11.01 of the RH-IRR and Section 4(t) of the RH Law
itself provides for the teaching of responsible teenage behavior, gender sensitivity and physical and emotional changes
among adolescents - the Court finds that the legal mandate provided under the assailed provision supplements, rather
than supplants, the rights and duties of the parents in the moral development of their children.

Furthermore, as Section 14 also mandates that the mandatory reproductive health education program shall be developed
in conjunction with parent-teacher-community associations, school officials and other interest groups, it could very well be
said that it will be in line with the religious beliefs of the petitioners. By imposing such a condition, it becomes apparent that
the petitioners' contention that Section 14 violates Article XV, Section 3(1) of the Constitution is without merit. 254
While the Court notes the possibility that educators might raise their objection to their participation in the reproductive
health education program provided under Section 14 of the RH Law on the ground that the same violates their religious
beliefs, the Court reserves its judgment should an actual case be filed before it.

6 - Due Process

The petitioners contend that the RH Law suffers from vagueness and, thus violates the due process clause of the
Constitution. According to them, Section 23 (a)(l) mentions a "private health service provider" among those who may be
held punishable but does not define who is a "private health care service provider." They argue that confusion further
results since Section 7 only makes reference to a "private health care institution."

The petitioners also point out that Section 7 of the assailed legislation exempts hospitals operated by religious groups from
rendering reproductive health service and modern family planning methods. It is unclear, however, if these institutions are
also exempt from giving reproductive health information under Section 23(a)(l), or from rendering reproductive health
procedures under Section 23(a)(2).

Finally, it is averred that the RH Law punishes the withholding, restricting and providing of incorrect information, but at the
same time fails to define "incorrect information."

The arguments fail to persuade.

A statute or act suffers from the defect of vagueness when it lacks comprehensible standards that men of common
intelligence must necessarily guess its meaning and differ as to its application. It is repugnant to the Constitution in two
respects: (1) it violates due process for failure to accord persons, especially the parties targeted by it, fair notice of the
conduct to avoid; and (2) it leaves law enforcers unbridled discretion in carrying out its provisions and becomes an
arbitrary flexing of the Government muscle.  Moreover, in determining whether the words used in a statute are vague,
255

words must not only be taken in accordance with their plain meaning alone, but also in relation to other parts of the
statute. It is a rule that every part of the statute must be interpreted with reference to the context, that is, every part of it
must be construed together with the other parts and kept subservient to the general intent of the whole enactment. 256

As correctly noted by the OSG, in determining the definition of "private health care service provider," reference must be
made to Section 4(n) of the RH Law which defines a "public health service provider," viz:

(n) Public health care service provider refers to: (1) public health care institution, which is duly licensed and accredited and
devoted primarily to the maintenance and operation of facilities for health promotion, disease prevention, diagnosis,
treatment and care of individuals suffering from illness, disease, injury, disability or deformity, or in need of obstetrical or
other medical and nursing care; (2) public health care professional, who is a doctor of medicine, a nurse or a midvvife; (3)
public health worker engaged in the delivery of health care services; or (4) barangay health worker who has undergone
training programs under any accredited government and NGO and who voluntarily renders primarily health care services
in the community after having been accredited to function as such by the local health board in accordance with the
guidelines promulgated by the Department of Health (DOH) .

Further, the use of the term "private health care institution" in Section 7 of the law, instead of "private health care service
provider," should not be a cause of confusion for the obvious reason that they are used synonymously.

The Court need not belabor the issue of whether the right to be exempt from being obligated to render reproductive health
service and modem family planning methods, includes exemption from being obligated to give reproductive health
information and to render reproductive health procedures. Clearly, subject to the qualifications and exemptions earlier
discussed, the right to be exempt from being obligated to render reproductive health service and modem family planning
methods, necessarily includes exemption from being obligated to give reproductive health information and to render
reproductive health procedures. The terms "service" and "methods" are broad enough to include the providing of
information and the rendering of medical procedures.

The same can be said with respect to the contention that the RH Law punishes health care service providers who
intentionally withhold, restrict and provide incorrect information regarding reproductive health programs and services. For
ready reference, the assailed provision is hereby quoted as follows:

SEC. 23. Prohibited Acts. - The following acts are prohibited:

(a) Any health care service provider, whether public or private, who shall:

(1) Knowingly withhold information or restrict the dissemination thereof, and/ or intentionally provide incorrect information
regarding programs and services on reproductive health including the right to informed choice and access to a full range of
legal, medically-safe, non-abortifacient and effective family planning methods;

From its plain meaning, the word "incorrect" here denotes failing to agree with a copy or model or with established rules;
inaccurate, faulty; failing to agree with the requirements of duty, morality or propriety; and failing to coincide with the
truth.   On the other hand, the word "knowingly" means with awareness or deliberateness that is intentional.  Used
257 258

together in relation to Section 23(a)(l), they connote a sense of malice and ill motive to mislead or misrepresent the public
as to the nature and effect of programs and services on reproductive health. Public health and safety demand that health
care service providers give their honest and correct medical information in accordance with what is acceptable in medical
practice. While health care service providers are not barred from expressing their own personal opinions regarding the
programs and services on reproductive health, their right must be tempered with the need to provide public health and
safety. The public deserves no less.

7-Egual Protection

The petitioners also claim that the RH Law violates the equal protection clause under the Constitution as it discriminates
against the poor because it makes them the primary target of the government program that promotes contraceptive use .
They argue that, rather than promoting reproductive health among the poor, the RH Law introduces contraceptives that
would effectively reduce the number of the poor. Their bases are the various provisions in the RH Law dealing with the
poor, especially those mentioned in the guiding principles  and definition of terms  of the law.
259 260

They add that the exclusion of private educational institutions from the mandatory reproductive health education program
imposed by the RH Law renders it unconstitutional.

In Biraogo v. Philippine Truth Commission,  the Court had the occasion to expound on the concept of equal protection.
261

Thus:

One of the basic principles on which this government was founded is that of the equality of right which is embodied in
Section 1, Article III of the 1987 Constitution. The equal protection of the laws is embraced in the concept of due process,
as every unfair discrimination offends the requirements of justice and fair play. It has been embodied in a separate clause,
however, to provide for a more specific guaranty against any form of undue favoritism or hostility from the government.
Arbitrariness in general may be challenged on the basis of the due process clause. But if the particular act assailed
partakes of an unwarranted partiality or prejudice, the sharper weapon to cut it down is the equal protection clause.

"According to a long line of decisions, equal protection simply requires that all persons or things similarly situated should
be treated alike, both as to rights conferred and responsibilities imposed." It "requires public bodies and inst itutions to
treat similarly situated individuals in a similar manner." "The purpose of the equal protection clause is to secure every
person within a state's jurisdiction against intentional and arbitrary discrimination, whether occasioned by the express
terms of a statue or by its improper execution through the state's duly constituted authorities." "In other words, the concept
of equal justice under the law requires the state to govern impartially, and it may not draw distinctions between individuals
solely on differences that are irrelevant to a legitimate governmental objective."

The equal protection clause is aimed at all official state actions, not just those of the legislature. Its inhibitions cover all the
departments of the government including the political and executive departments, and extend to all actions of a state
denying equal protection of the laws, through whatever agency or whatever guise is taken.

It, however, does not require the universal application of the laws to all persons or things without distinction. What it simply
requires is equality among equals as determined according to a valid classification. Indeed, the equal protection clause
permits classification. Such classification, however, to be valid must pass the test of reasonableness. The test has four
requisites: (1) The classification rests on substantial distinctions; (2) It is germane to the purpose of the law; (3) It is not
limited to existing conditions only; and (4) It applies equally to all members of the same class. "Superficial differences do
not make for a valid classification."

For a classification to meet the requirements of constitutionality, it must include or embrace all persons who naturally
belong to the class. "The classification will be regarded as invalid if all the members of the class are not similarly treated,
both as to rights conferred and obligations imposed. It is not necessary that the classification be made with absolute
symmetry, in the sense that the members of the class should possess the same characteristics in equal degree.
Substantial similarity will suffice; and as long as this is achieved, all those covered by the classification are to be treated
equally. The mere fact that an individual belonging to a class differs from the other members, as long as that class is
substantially distinguishable from all others, does not justify the non-application of the law to him."

The classification must not be based on existing circumstances only, or so constituted as to preclude addition to the
number included in the class. It must be of such a nature as to embrace all those who may thereafter be in similar
circumstances and conditions. It must not leave out or "underinclude" those that should otherwise fall into a certain
classification. [Emphases supplied; citations excluded]

To provide that the poor are to be given priority in the government's reproductive health care program is not a violation of
the equal protection clause. In fact, it is pursuant to Section 11, Article XIII of the Constitution which recognizes the distinct
necessity to address the needs of the underprivileged by providing that they be given priority in addressing the health
development of the people. Thus:

Section 11. The State shall adopt an integrated and comprehensive approach to health development which shall endeavor
to make essential goods, health and other social services available to all the people at affordable cost. There shall be
priority for the needs of the underprivileged, sick, elderly, disabled, women, and children. The State shall endeavor to
provide free medical care to paupers.

It should be noted that Section 7 of the RH Law prioritizes poor and marginalized couples who are suffering from fertility
issues and desire to have children. There is, therefore, no merit to the contention that the RH Law only seeks to target the
poor to reduce their number. While the RH Law admits the use of contraceptives, it does not, as elucidated above,
sanction abortion. As Section 3(1) explains, the "promotion and/or stabilization of the population growth rate is incidental
to the advancement of reproductive health."

Moreover, the RH Law does not prescribe the number of children a couple may have and does not impose conditions
upon couples who intend to have children. While the petitioners surmise that the assailed law seeks to charge couples
with the duty to have children only if they would raise them in a truly humane way, a deeper look into its provisions shows
that what the law seeks to do is to simply provide priority to the poor in the implementation of government programs to
promote basic reproductive health care.

With respect to the exclusion of private educational institutions from the mandatory reproductive health education program
under Section 14, suffice it to state that the mere fact that the children of those who are less fortunate attend public
educational institutions does not amount to substantial distinction sufficient to annul the assailed provision. On the other
hand, substantial distinction rests between public educational institutions and private educational institutions, particularly
because there is a need to recognize the academic freedom of private educational institutions especially with respect to
religious instruction and to consider their sensitivity towards the teaching of reproductive health education.

8-Involuntary Servitude

The petitioners also aver that the RH Law is constitutionally infirm as it violates the constitutional prohibition against
involuntary servitude. They posit that Section 17 of the assailed legislation requiring private and non-government health
care service providers to render forty-eight (48) hours of pro bono reproductive health services, actually amounts to
involuntary servitude because it requires medical practitioners to perform acts against their will.262

The OSG counters that the rendition of pro bono services envisioned in Section 17 can hardly be considered as forced
labor analogous to slavery, as reproductive health care service providers have the discretion as to the manner and time of
giving pro bono services. Moreover, the OSG points out that the imposition is within the powers of the government, the
accreditation of medical practitioners with PhilHealth being a privilege and not a right.

The point of the OSG is well-taken.

It should first be mentioned that the practice of medicine is undeniably imbued with public interest that it is both a power
and a duty of the State to control and regulate it in order to protect and promote the public welfare. Like the legal
profession, the practice of medicine is not a right but a privileged burdened with conditions as it directly involves the very
lives of the people. A fortiori, this power includes the power of Congress  to prescribe the qualifications for the practice of
263

professions or trades which affect the public welfare, the public health, the public morals, and the public safety; and to
regulate or control such professions or trades, even to the point of revoking such right altogether. 264

Moreover, as some petitioners put it, the notion of involuntary servitude connotes the presence of force, threats,
intimidation or other similar means of coercion and compulsion.  A reading of the assailed provision, however, reveals
265

that it only encourages private and non- government reproductive healthcare service providers to render pro bono service.
Other than non-accreditation with PhilHealth, no penalty is imposed should they choose to do otherwise. Private and non-
government reproductive healthcare service providers also enjoy the liberty to choose which kind of health service they
wish to provide, when, where and how to provide it or whether to provide it all. Clearly, therefore, no compulsion, force or
threat is made upon them to render pro bono service against their will. While the rendering of such service was made a
prerequisite to accreditation with PhilHealth, the Court does not consider the same to be an unreasonable burden, but
rather, a necessary incentive imposed by Congress in the furtherance of a perceived legitimate state interest.

Consistent with what the Court had earlier discussed, however, it should be emphasized that conscientious objectors are
exempt from this provision as long as their religious beliefs and convictions do not allow them to render reproductive
health service, pro bona or otherwise.

9-Delegation of Authority to the FDA

The petitioners likewise question the delegation by Congress to the FDA of the power to determine whether or not a
supply or product is to be included in the Essential Drugs List (EDL). 266

The Court finds nothing wrong with the delegation. The FDA does not only have the power but also the competency to
evaluate, register and cover health services and methods. It is the only government entity empowered to render such
services and highly proficient to do so. It should be understood that health services and methods fall under the gamut of
terms that are associated with what is ordinarily understood as "health products."

In this connection, Section 4 of R.A. No. 3 720, as amended by R.A. No. 9711 reads:

SEC. 4. To carry out the provisions of this Act, there is hereby created an office to be called the Food and Drug
Administration (FDA) in the Department of Health (DOH). Said Administration shall be under the Office of the Secretary
and shall have the following functions, powers and duties:

"(a) To administer the effective implementation of this Act and of the rules and regulations issued pursuant to the
same;

"(b) To assume primary jurisdiction in the collection of samples of health products;


"(c) To analyze and inspect health products in connection with the implementation of this Act;

"(d) To establish analytical data to serve as basis for the preparation of health products standards, and to
recommend standards of identity, purity, safety, efficacy, quality and fill of container;

"(e) To issue certificates of compliance with technical requirements to serve as basis for the issuance of
appropriate authorization and spot-check for compliance with regulations regarding operation of manufacturers,
importers, exporters, distributors, wholesalers, drug outlets, and other establishments and facilities of health
products, as determined by the FDA;

"x x x

"(h) To conduct appropriate tests on all applicable health products prior to the issuance of appropriate
authorizations to ensure safety, efficacy, purity, and quality;

"(i) To require all manufacturers, traders, distributors, importers, exporters, wholesalers, retailers, consumers, and
non-consumer users of health products to report to the FDA any incident that reasonably indicates that said
product has caused or contributed to the death, serious illness or serious injury to a consumer, a patient, or any
person;

"(j) To issue cease and desist orders motu propio or upon verified complaint for health products, whether or not
registered with the FDA Provided, That for registered health products, the cease and desist order is valid for thirty
(30) days and may be extended for sixty ( 60) days only after due process has been observed;

"(k) After due process, to order the ban, recall, and/or withdrawal of any health product found to have caused
death, serious illness or serious injury to a consumer or patient, or is found to be imminently injurious, unsafe,
dangerous, or grossly deceptive, and to require all concerned to implement the risk management plan which is a
requirement for the issuance of the appropriate authorization;

x x x.

As can be gleaned from the above, the functions, powers and duties of the FDA are specific to enable the agency to carry
out the mandates of the law. Being the country's premiere and sole agency that ensures the safety of food and medicines
available to the public, the FDA was equipped with the necessary powers and functions to make it effective. Pursuant to
the principle of necessary implication, the mandate by Congress to the FDA to ensure public health and safety by
permitting only food and medicines that are safe includes "service" and "methods." From the declared policy of the RH
Law, it is clear that Congress intended that the public be given only those medicines that are proven medically safe, legal,
non-abortifacient, and effective in accordance with scientific and evidence-based medical research standards. The
philosophy behind the permitted delegation was explained in Echagaray v. Secretary of Justice,  as follows:
267

The reason is the increasing complexity of the task of the government and the growing inability of the legislature to cope
directly with the many problems demanding its attention. The growth of society has ramified its activities and created
peculiar and sophisticated problems that the legislature cannot be expected reasonably to comprehend. Specialization
even in legislation has become necessary. To many of the problems attendant upon present day undertakings, the
legislature may not have the competence, let alone the interest and the time, to provide the required direct and efficacious,
not to say specific solutions.

10- Autonomy of Local Governments and the Autonomous Region

of Muslim Mindanao (ARMM)

As for the autonomy of local governments, the petitioners claim that the RH Law infringes upon the powers devolved to
local government units (LGUs) under Section 17 of the Local Government Code. Said Section 17 vested upon the LGUs
the duties and functions pertaining to the delivery of basic services and facilities, as follows:

SECTION 17. Basic Services and Facilities. –

(a) Local government units shall endeavor to be self-reliant and shall continue exercising the powers and
discharging the duties and functions currently vested upon them. They shall also discharge the functions and
responsibilities of national agencies and offices devolved to them pursuant to this Code. Local government units
shall likewise exercise such other powers and discharge such other functions and responsibilities as are
necessary, appropriate, or incidental to efficient and effective provision of the basic services and facilities
enumerated herein.

(b) Such basic services and facilities include, but are not limited to, x x x.

While the aforementioned provision charges the LGUs to take on the functions and responsibilities that have
already been devolved upon them from the national agencies on the aspect of providing for basic services and
facilities in their respective jurisdictions, paragraph (c) of the same provision provides a categorical exception of
cases involving nationally-funded projects, facilities, programs and services.  Thus:
268
(c) Notwithstanding the provisions of subsection (b) hereof, public works and infrastructure projects and other
facilities, programs and services funded by the National Government under the annual General Appropriations Act,
other special laws, pertinent executive orders, and those wholly or partially funded from foreign sources, are not
covered under this Section, except in those cases where the local government unit concerned is duly designated
as the implementing agency for such projects, facilities, programs and services. [Emphases supplied]

The essence of this express reservation of power by the national government is that, unless an LGU is particularly
designated as the implementing agency, it has no power over a program for which funding has been provided by the
national government under the annual general appropriations act, even if the program involves the delivery of basic
services within the jurisdiction of the LGU.  A complete relinquishment of central government powers on the matter of
269

providing basic facilities and services cannot be implied as the Local Government Code itself weighs against it. 270

In this case, a reading of the RH Law clearly shows that whether it pertains to the establishment of health care
facilities,  the hiring of skilled health professionals,  or the training of barangay health workers,  it will be the national
271 272 273

government that will provide for the funding of its implementation. Local autonomy is not absolute. The national
government still has the say when it comes to national priority programs which the local government is called upon to
implement like the RH Law.

Moreover, from the use of the word "endeavor," the LG Us are merely encouraged to provide these services. There is
nothing in the wording of the law which can be construed as making the availability of these services mandatory for the
LGUs. For said reason, it cannot be said that the RH Law amounts to an undue encroachment by the national government
upon the autonomy enjoyed by the local governments.

The ARMM

The fact that the RH Law does not intrude in the autonomy of local governments can be equally applied to the ARMM. The
RH Law does not infringe upon its autonomy. Moreover, Article III, Sections 6, 10 and 11 of R.A. No. 9054, or the organic
act of the ARMM, alluded to by petitioner Tillah to justify the exemption of the operation of the RH Law in the autonomous
region, refer to the policy statements for the guidance of the regional government. These provisions relied upon by the
petitioners simply delineate the powers that may be exercised by the regional government, which can, in no manner, be
characterized as an abdication by the State of its power to enact legislation that would benefit the general welfare. After
all, despite the veritable autonomy granted the ARMM, the Constitution and the supporting jurisprudence, as they now
stand, reject the notion of imperium et imperio in the relationship between the national and the regional
governments.  Except for the express and implied limitations imposed on it by the Constitution, Congress cannot be
274

restricted to exercise its inherent and plenary power to legislate on all subjects which extends to all matters of general
concern or common interest. 275

11 - Natural Law

With respect to the argument that the RH Law violates natural law,  suffice it to say that the Court does not duly recognize
276

it as a legal basis for upholding or invalidating a law. Our only guidepost is the Constitution. While every law enacted by
man emanated from what is perceived as natural law, the Court is not obliged to see if a statute, executive issuance or
ordinance is in conformity to it. To begin with, it is not enacted by an acceptable legitimate body. Moreover, natural laws
are mere thoughts and notions on inherent rights espoused by theorists, philosophers and theologists. The jurists of the
philosophical school are interested in the law as an abstraction, rather than in the actual law of the past or
present.  Unless, a natural right has been transformed into a written law, it cannot serve as a basis to strike down a law.
277

In Republic v. Sandiganbayan,  the very case cited by the petitioners, it was explained that the Court is not duty-bound to
278

examine every law or action and whether it conforms with both the Constitution and natural law. Rather, natural law is to
be used sparingly only in the most peculiar of circumstances involving rights inherent to man where no law is applicable. 279

At any rate, as earlier expounded, the RH Law does not sanction the taking away of life. It does not allow abortion in any
shape or form. It only seeks to enhance the population control program of the government by providing information and
making non-abortifacient contraceptives more readily available to the public, especially to the poor.

Facts and Fallacies

and the Wisdom of the Law

In general, the Court does not find the RH Law as unconstitutional insofar as it seeks to provide access to medically-safe,
non-abortifacient, effective, legal, affordable, and quality reproductive healthcare services, methods, devices, and
supplies. As earlier pointed out, however, the religious freedom of some sectors of society cannot be trampled upon in
pursuit of what the law hopes to achieve. After all, the Constitutional safeguard to religious freedom is a recognition that
man stands accountable to an authority higher than the State.

In conformity with the principle of separation of Church and State, one religious group cannot be allowed to impose its
beliefs on the rest of the society. Philippine modem society leaves enough room for diversity and pluralism. As such,
everyone should be tolerant and open-minded so that peace and harmony may continue to reign as we exist alongside
each other.

As healthful as the intention of the RH Law may be, the idea does not escape the Court that what it seeks to address is
the problem of rising poverty and unemployment in the country. Let it be said that the cause of these perennial issues is
not the large population but the unequal distribution of wealth. Even if population growth is controlled, poverty will remain
as long as the country's wealth remains in the hands of the very few.

At any rate, population control may not be beneficial for the country in the long run. The European and Asian countries,
which embarked on such a program generations ago , are now burdened with ageing populations. The number of their
young workers is dwindling with adverse effects on their economy. These young workers represent a significant human
capital which could have helped them invigorate, innovate and fuel their economy. These countries are now trying to
reverse their programs, but they are still struggling. For one, Singapore, even with incentives, is failing.

And in this country, the economy is being propped up by remittances from our Overseas Filipino Workers. This is because
we have an ample supply of young able-bodied workers. What would happen if the country would be weighed down by an
ageing population and the fewer younger generation would not be able to support them? This would be the situation when
our total fertility rate would go down below the replacement level of two (2) children per woman. 280

Indeed, at the present, the country has a population problem, but the State should not use coercive measures (like the
penal provisions of the RH Law against conscientious objectors) to solve it. Nonetheless, the policy of the Court is non-
interference in the wisdom of a law.

x x x. But this Court cannot go beyond what the legislature has laid down. Its duty is to say what the law is as enacted by
the lawmaking body. That is not the same as saying what the law should be or what is the correct rule in a given set of
circumstances. It is not the province of the judiciary to look into the wisdom of the law nor to question the policies adopted
by the legislative branch. Nor is it the business of this Tribunal to remedy every unjust situation that may arise from the
application of a particular law. It is for the legislature to enact remedial legislation if that would be necessary in the
premises. But as always, with apt judicial caution and cold neutrality, the Court must carry out the delicate function of
interpreting the law, guided by the Constitution and existing legislation and mindful of settled jurisprudence. The Court's
function is therefore limited, and accordingly, must confine itself to the judicial task of saying what the law is, as enacted
by the lawmaking body. 281

Be that as it may, it bears reiterating that the RH Law is a mere compilation and enhancement of the prior existing
contraceptive and reproductive health laws, but with coercive measures. Even if the Court decrees the RH Law as entirely
unconstitutional, there will still be the Population Act (R.A. No. 6365), the Contraceptive Act (R.A. No. 4729) and the
reproductive health for women or The Magna Carta of Women (R.A. No. 9710), sans the coercive provisions of the
assailed legislation. All the same, the principle of "no-abortion" and "non-coercion" in the adoption of any family planning
method should be maintained.

WHEREFORE, the petitions are PARTIALLY GRANTED. Accordingly, the Court declares R.A. No. 10354 as NOT
UNCONSTITUTIONAL except with respect to the following provisions which are declared UNCONSTITUTIONAL:

1) Section 7 and the corresponding provision in the RH-IRR insofar as they: a) require private health facilities and
non-maternity specialty hospitals and hospitals owned and operated by a religious group to refer patients, not in an
emergency or life-threatening case, as defined under Republic Act No. 8344, to another health facility which is
conveniently accessible; and b) allow minor-parents or minors who have suffered a miscarriage access to modem
methods of family planning without written consent from their parents or guardian/s;

2) Section 23(a)(l) and the corresponding provision in the RH-IRR, particularly Section 5 .24 thereof, insofar as
they punish any healthcare service provider who fails and or refuses to disseminate information regarding
programs and services on reproductive health regardless of his or her religious beliefs.

3) Section 23(a)(2)(i) and the corresponding provision in the RH-IRR insofar as they allow a married individual, not
in an emergency or life-threatening case, as defined under Republic Act No. 8344, to undergo reproductive health
procedures without the consent of the spouse;

4) Section 23(a)(2)(ii) and the corresponding provision in the RH-IRR insofar as they limit the requirement of
parental consent only to elective surgical procedures.

5) Section 23(a)(3) and the corresponding provision in the RH-IRR, particularly Section 5.24 thereof, insofar as
they punish any healthcare service provider who fails and/or refuses to refer a patient not in an emergency or life-
threatening case, as defined under Republic Act No. 8344, to another health care service provider within the same
facility or one which is conveniently accessible regardless of his or her religious beliefs;

6) Section 23(b) and the corresponding provision in the RH-IRR, particularly Section 5 .24 thereof, insofar as they
punish any public officer who refuses to support reproductive health programs or shall do any act that hinders the
full implementation of a reproductive health program, regardless of his or her religious beliefs;

7) Section 17 and the corresponding prov1s10n in the RH-IRR regarding the rendering of pro bona reproductive
health service in so far as they affect the conscientious objector in securing PhilHealth accreditation; and

8) Section 3.0l(a) and Section 3.01 G) of the RH-IRR, which added the qualifier "primarily" in defining
abortifacients and contraceptives, as they are ultra vires and, therefore, null and void for contravening Section 4(a)
of the RH Law and violating Section 12, Article II of the Constitution.
The Status Quo Ante Order issued by the Court on March 19, 2013 as extended by its Order, dated July 16, 2013 , is
hereby LIFTED, insofar as the provisions of R.A. No. 10354 which have been herein declared as constitutional.

SO ORDERED.

Rights of indigenous cultural communities, Article II , Section 22; Article VI, Section 5 (2); Article XII, Section 5;
Article XIII, Section 6; Article XIV, Section 17; Article XVI, Sec 12
-Province of North Cotabato v GRP Peace Panel on Ancestral Domain, 568 SCRA 402 (2008), (within the framework
of national unity) [SEE PREVIOUS PAGE 1]

Honest public service, Article II, Sections 27-28


-Ang Ladlad v Comelec, 618 SCRA 32 (2010)

... [F]reedom to differ is not limited to things that do not matter much. That would be a mere shadow of freedom. The test
of its substance is the right to differ as to things that touch the heart of the existing order.

Justice Robert A. Jackson

West Virginia State Board of Education v. Barnette 1

One unavoidable consequence of everyone having the freedom to choose is that others may make different choices –
choices we would not make for ourselves, choices we may disapprove of, even choices that may shock or offend or anger
us. However, choices are not to be legally prohibited merely because they are different, and the right to disagree and
debate about important questions of public policy is a core value protected by our Bill of Rights. Indeed, our democracy is
built on genuine recognition of, and respect for, diversity and difference in opinion.

Since ancient times, society has grappled with deep disagreements about the definitions and demands of morality. In
many cases, where moral convictions are concerned, harmony among those theoretically opposed is an insurmountable
goal. Yet herein lies the paradox – philosophical justifications about what is moral are indispensable and yet at the same
time powerless to create agreement. This Court recognizes, however, that practical solutions are preferable to ideological
stalemates; accommodation is better than intransigence; reason more worthy than rhetoric. This will allow persons of
diverse viewpoints to live together, if not harmoniously, then, at least, civilly.

Factual Background

This is a Petition for Certiorari under Rule 65 of the Rules of Court, with an application for a writ of preliminary mandatory
injunction, filed by Ang Ladlad LGBT Party (Ang Ladlad) against the Resolutions of the Commission on Elections
(COMELEC) dated November 11, 20092 (the First Assailed Resolution) and December 16, 20093 (the Second Assailed
Resolution) in SPP No. 09-228 (PL) (collectively, the Assailed Resolutions). The case has its roots in the COMELEC’s
refusal to accredit Ang Ladlad as a party-list organization under Republic Act (RA) No. 7941, otherwise known as the
Party-List System Act.4

Ang Ladlad is an organization composed of men and women who identify themselves as lesbians, gays, bisexuals, or
trans-gendered individuals (LGBTs). Incorporated in 2003, Ang Ladlad first applied for registration with the COMELEC in
2006. The application for accreditation was denied on the ground that the organization had no substantial membership
base. On August 17, 2009, Ang Ladlad again filed a Petition5 for registration with the COMELEC.

Before the COMELEC, petitioner argued that the LGBT community is a marginalized and under-represented sector that is
particularly disadvantaged because of their sexual orientation and gender identity; that LGBTs are victims of exclusion,
discrimination, and violence; that because of negative societal attitudes, LGBTs are constrained to hide their sexual
orientation; and that Ang Ladlad complied with the 8-point guidelines enunciated by this Court in Ang Bagong Bayani-
OFW Labor Party v. Commission on Elections.6 Ang Ladlad laid out its national membership base consisting of individual
members and organizational supporters, and outlined its platform of governance. 7

On November 11, 2009, after admitting the petitioner’s evidence, the COMELEC (Second Division) dismissed the Petition
on moral grounds, stating that:

x x x This Petition is dismissible on moral grounds. Petitioner defines the Filipino Lesbian, Gay, Bisexual and Transgender
(LGBT) Community, thus:

x x x a marginalized and under-represented sector that is particularly disadvantaged because of their sexual orientation
and gender identity.

and proceeded to define sexual orientation as that which:

x x x refers to a person’s capacity for profound emotional, affectional and sexual attraction to, and intimate and sexual
relations with, individuals of a different gender, of the same gender, or more than one gender."
This definition of the LGBT sector makes it crystal clear that petitioner tolerates immorality which offends religious beliefs.
In Romans 1:26, 27, Paul wrote:

For this cause God gave them up into vile affections, for even their women did change the natural use into that which is
against nature: And likewise also the men, leaving the natural use of the woman, burned in their lust one toward another;
men with men working that which is unseemly, and receiving in themselves that recompense of their error which was
meet.

In the Koran, the hereunder verses are pertinent:

For ye practice your lusts on men in preference to women "ye are indeed a people transgressing beyond bounds." (7.81)
"And we rained down on them a shower (of brimstone): Then see what was the end of those who indulged in sin and
crime!" (7:84) "He said: "O my Lord! Help Thou me against people who do mischief" (29:30).

As correctly pointed out by the Law Department in its Comment dated October 2, 2008:

The ANG LADLAD apparently advocates sexual immorality as indicated in the Petition’s par. 6F: ‘Consensual partnerships
or relationships by gays and lesbians who are already of age’. It is further indicated in par. 24 of the Petition which waves
for the record: ‘In 2007, Men Having Sex with Men or MSMs in the Philippines were estimated as 670,000 (Genesis 19 is
the history of Sodom and Gomorrah).

Laws are deemed incorporated in every contract, permit, license, relationship, or accreditation. Hence, pertinent provisions
of the Civil Code and the Revised Penal Code are deemed part of the requirement to be complied with for accreditation.

ANG LADLAD collides with Article 695 of the Civil Code which defines nuisance as ‘Any act, omission, establishment,
business, condition of property, or anything else which x x x (3) shocks, defies; or disregards decency or morality x x x

It also collides with Article 1306 of the Civil Code: ‘The contracting parties may establish such stipulations, clauses, terms
and conditions as they may deem convenient, provided they are not contrary to law, morals, good customs, public order or
public policy. Art 1409 of the Civil Code provides that ‘Contracts whose cause, object or purpose is contrary to
law, morals, good customs, public order or public policy’ are inexistent and void from the beginning.

Finally to safeguard the morality of the Filipino community, the Revised Penal Code, as amended, penalizes ‘Immoral
doctrines, obscene publications and exhibitions and indecent shows’ as follows:

Art. 201. Immoral doctrines, obscene publications and exhibitions, and indecent shows. — The penalty of prision mayor or
a fine ranging from six thousand to twelve thousand pesos, or both such imprisonment and fine, shall be imposed upon:

1. Those who shall publicly expound or proclaim doctrines openly contrary to public morals;

2. (a) The authors of obscene literature, published with their knowledge in any form; the editors publishing such
literature; and the owners/operators of the establishment selling the same;

(b) Those who, in theaters, fairs, cinematographs or any other place, exhibit indecent or immoral plays,
scenes, acts or shows, it being understood that the obscene literature or indecent or immoral plays,
scenes, acts or shows, whether live or in film, which are prescribed by virtue hereof, shall include those
which: (1) glorify criminals or condone crimes; (2) serve no other purpose but to satisfy the market for
violence, lust or pornography; (3) offend any race or religion; (4) tend to abet traffic in and use of prohibited
drugs; and (5) are contrary to law, public order, morals, good customs, established policies, lawful orders,
decrees and edicts.

3. Those who shall sell, give away or exhibit films, prints, engravings, sculpture or literature which are offensive to
morals.

Petitioner should likewise be denied accreditation not only for advocating immoral doctrines but likewise for not being
truthful when it said that it "or any of its nominees/party-list representatives have not violated or failed to comply with laws,
rules, or regulations relating to the elections."

Furthermore, should this Commission grant the petition, we will be exposing our youth to an environment that does not
conform to the teachings of our faith. Lehman Strauss, a famous bible teacher and writer in the U.S.A. said in one article
that "older practicing homosexuals are a threat to the youth." As an agency of the government, ours too is the State’s
avowed duty under Section 13, Article II of the Constitution to protect our youth from moral and spiritual degradation. 8

When Ang Ladlad sought reconsideration,9 three commissioners voted to overturn the First Assailed Resolution
(Commissioners Gregorio Y. Larrazabal, Rene V. Sarmiento, and Armando Velasco), while three commissioners voted to
deny Ang Ladlad’s Motion for Reconsideration (Commissioners Nicodemo T. Ferrer, Lucenito N. Tagle, and Elias R.
Yusoph). The COMELEC Chairman, breaking the tie and speaking for the majority in his Separate Opinion, upheld the
First Assailed Resolution, stating that:

I. The Spirit of Republic Act No. 7941


Ladlad is applying for accreditation as a sectoral party in the party-list system. Even assuming that it has properly proven
its under-representation and marginalization, it cannot be said that Ladlad’s expressed sexual orientations per se would
benefit the nation as a whole.

Section 2 of the party-list law unequivocally states that the purpose of the party-list system of electing congressional
representatives is to enable Filipino citizens belonging to marginalized and under-represented sectors, organizations and
parties, and who lack well-defined political constituencies but who could contribute to the formulation and enactment of
appropriate legislation that will benefit the nation as a whole, to become members of the House of Representatives.

If entry into the party-list system would depend only on the ability of an organization to represent its constituencies, then all
representative organizations would have found themselves into the party-list race. But that is not the intention of the
framers of the law. The party-list system is not a tool to advocate tolerance and acceptance of misunderstood persons or
groups of persons. Rather, the party-list system is a tool for the realization of aspirations of marginalized individuals whose
interests are also the nation’s – only that their interests have not been brought to the attention of the nation because of
their under representation. Until the time comes when Ladlad is able to justify that having mixed sexual orientations and
transgender identities is beneficial to the nation, its application for accreditation under the party-list system will remain just
that.

II. No substantial differentiation

In the United States, whose equal protection doctrine pervades Philippine jurisprudence, courts do not recognize lesbians,
gays, homosexuals, and bisexuals (LGBT) as a "special class" of individuals. x x x Significantly, it has also been held that
homosexuality is not a constitutionally protected fundamental right, and that "nothing in the U.S. Constitution discloses a
comparable intent to protect or promote the social or legal equality of homosexual relations," as in the case of race or
religion or belief.

xxxx

Thus, even if society’s understanding, tolerance, and acceptance of LGBT’s is elevated, there can be no denying that
Ladlad constituencies are still males and females, and they will remain either male or female protected by the same Bill of
Rights that applies to all citizens alike.

xxxx

IV. Public Morals

x x x There is no question about not imposing on Ladlad Christian or Muslim religious practices. Neither is there any
attempt to any particular religious group’s moral rules on Ladlad. Rather, what are being adopted as moral parameters and
precepts are generally accepted public morals. They are possibly religious-based, but as a society, the Philippines cannot
ignore its more than 500 years of Muslim and Christian upbringing, such that some moral precepts espoused by said
religions have sipped [sic] into society and these are not publicly accepted moral norms.

V. Legal Provisions

But above morality and social norms, they have become part of the law of the land. Article 201 of the Revised Penal Code
imposes the penalty of prision mayor upon "Those who shall publicly expound or proclaim doctrines openly contrary to
public morals." It penalizes "immoral doctrines, obscene publications and exhibition and indecent shows." "Ang Ladlad"
apparently falls under these legal provisions. This is clear from its Petition’s paragraph 6F: "Consensual partnerships or
relationships by gays and lesbians who are already of age’ It is further indicated in par. 24 of the Petition which waves for
the record: ‘In 2007, Men Having Sex with Men or MSMs in the Philippines were estimated as 670,000. Moreoever, Article
694 of the Civil Code defines "nuisance" as any act, omission x x x or anything else x x x which shocks, defies or
disregards decency or morality x x x." These are all unlawful. 10

On January 4, 2010, Ang Ladlad filed this Petition, praying that the Court annul the Assailed Resolutions and direct the
COMELEC to grant Ang Ladlad’s application for accreditation. Ang Ladlad also sought the issuance ex parte of a
preliminary mandatory injunction against the COMELEC, which had previously announced that it would begin printing the
final ballots for the May 2010 elections by January 25, 2010.

On January 6, 2010, we ordered the Office of the Solicitor General (OSG) to file its Comment on behalf of COMELEC not
later than 12:00 noon of January 11, 2010. 11 Instead of filing a Comment, however, the OSG filed a Motion for Extension,
requesting that it be given until January 16, 2010 to Comment. 12 Somewhat surprisingly, the OSG later filed a Comment in
support of petitioner’s application. 13 Thus, in order to give COMELEC the opportunity to fully ventilate its position, we
required it to file its own comment. 14 The COMELEC, through its Law Department, filed its Comment on February 2,
2010.15

In the meantime, due to the urgency of the petition, we issued a temporary restraining order on January 12, 2010, effective
immediately and continuing until further orders from this Court, directing the COMELEC to cease and desist from
implementing the Assailed Resolutions.16

Also, on January 13, 2010, the Commission on Human Rights (CHR) filed a Motion to Intervene or to Appear as Amicus
Curiae, attaching thereto its Comment-in-Intervention. 17 The CHR opined that the denial of Ang Ladlad’s petition on moral
grounds violated the standards and principles of the Constitution, the Universal Declaration of Human Rights (UDHR), and
the International Covenant on Civil and Political Rights (ICCPR). On January 19, 2010, we granted the CHR’s motion to
intervene.

On January 26, 2010, Epifanio D. Salonga, Jr. filed his Motion to Intervene 18 which motion was granted on February 2,
2010.19

The Parties’ Arguments

Ang Ladlad argued that the denial of accreditation, insofar as it justified the exclusion by using religious dogma, violated
the constitutional guarantees against the establishment of religion. Petitioner also claimed that the Assailed Resolutions
contravened its constitutional rights to privacy, freedom of speech and assembly, and equal protection of laws, as well as
constituted violations of the Philippines’ international obligations against discrimination based on sexual orientation.

The OSG concurred with Ang Ladlad’s petition and argued that the COMELEC erred in denying petitioner’s application for
registration since there was no basis for COMELEC’s allegations of immorality. It also opined that LGBTs have their own
special interests and concerns which should have been recognized by the COMELEC as a separate classification.
However, insofar as the purported violations of petitioner’s freedom of speech, expression, and assembly were concerned,
the OSG maintained that there had been no restrictions on these rights.

In its Comment, the COMELEC reiterated that petitioner does not have a concrete and genuine national political agenda to
benefit the nation and that the petition was validly dismissed on moral grounds. It also argued for the first time that the
LGBT sector is not among the sectors enumerated by the Constitution and RA 7941, and that petitioner made untruthful
statements in its petition when it alleged its national existence contrary to actual verification reports by COMELEC’s field
personnel.

Our Ruling

We grant the petition.

Compliance with the Requirements of the Constitution and Republic Act No. 7941

The COMELEC denied Ang Ladlad’s application for registration on the ground that the LGBT sector is neither enumerated
in the Constitution and RA 7941, nor is it associated with or related to any of the sectors in the enumeration.

Respondent mistakenly opines that our ruling in Ang Bagong Bayani stands for the proposition that only those sectors
specifically enumerated in the law or related to said sectors (labor, peasant, fisherfolk, urban poor, indigenous cultural
communities, elderly, handicapped, women, youth, veterans, overseas workers, and professionals) may be registered
under the party-list system. As we explicitly ruled in Ang Bagong Bayani-OFW Labor Party v. Commission on
Elections,20 "the enumeration of marginalized and under-represented sectors is not exclusive". The crucial element is not
whether a sector is specifically enumerated, but whether a particular organization complies with the requirements of the
Constitution and RA 7941.

Respondent also argues that Ang Ladlad made untruthful statements in its petition when it alleged that it had nationwide
existence through its members and affiliate organizations. The COMELEC claims that upon verification by its field
personnel, it was shown that "save for a few isolated places in the country, petitioner does not exist in almost all provinces
in the country."21

This argument that "petitioner made untruthful statements in its petition when it alleged its national existence" is a new
one; previously, the COMELEC claimed that petitioner was "not being truthful when it said that it or any of its
nominees/party-list representatives have not violated or failed to comply with laws, rules, or regulations relating to the
elections." Nowhere was this ground for denial of petitioner’s accreditation mentioned or even alluded to in the Assailed
Resolutions. This, in itself, is quite curious, considering that the reports of petitioner’s alleged non-existence were already
available to the COMELEC prior to the issuance of the First Assailed Resolution. At best, this is irregular procedure; at
worst, a belated afterthought, a change in respondent’s theory, and a serious violation of petitioner’s right to procedural
due process.

Nonetheless, we find that there has been no misrepresentation. A cursory perusal of Ang Ladlad’s initial petition shows
that it never claimed to exist in each province of the Philippines. Rather, petitioner alleged that the LGBT community in the
Philippines was estimated to constitute at least 670,000 persons; that it had 16,100 affiliates and members around the
country, and 4,044 members in its electronic discussion group. 22 Ang Ladlad also represented itself to be "a national LGBT
umbrella organization with affiliates around the Philippines composed of the following LGBT networks:"

§ Abra Gay Association

§ Aklan Butterfly Brigade (ABB) – Aklan

§ Albay Gay Association

§ Arts Center of Cabanatuan City – Nueva Ecija


§ Boys Legion – Metro Manila

§ Cagayan de Oro People Like Us (CDO PLUS)

§ Can’t Live in the Closet, Inc. (CLIC) – Metro Manila

§ Cebu Pride – Cebu City

§ Circle of Friends

§ Dipolog Gay Association – Zamboanga del Norte

§ Gay, Bisexual, & Transgender Youth Association (GABAY)

§ Gay and Lesbian Activists Network for Gender Equality (GALANG) – Metro Manila

§ Gay Men’s Support Group (GMSG) – Metro Manila

§ Gay United for Peace and Solidarity (GUPS) – Lanao del Norte

§ Iloilo City Gay Association – Iloilo City

§ Kabulig Writer’s Group – Camarines Sur

§ Lesbian Advocates Philippines, Inc. (LEAP)

§ LUMINA – Baguio City

§ Marikina Gay Association – Metro Manila

§ Metropolitan Community Church (MCC) – Metro Manila

§ Naga City Gay Association – Naga City

§ ONE BACARDI

§ Order of St. Aelred (OSAe) – Metro Manila

§ PUP LAKAN

§ RADAR PRIDEWEAR

§ Rainbow Rights Project (R-Rights), Inc. – Metro Manila

§ San Jose del Monte Gay Association – Bulacan

§ Sining Kayumanggi Royal Family – Rizal

§ Society of Transexual Women of the Philippines (STRAP) – Metro Manila

§ Soul Jive – Antipolo, Rizal

§ The Link – Davao City

§ Tayabas Gay Association – Quezon

§ Women’s Bisexual Network – Metro Manila

§ Zamboanga Gay Association – Zamboanga City23

Since the COMELEC only searched for the names ANG LADLAD LGBT or LADLAD LGBT, it is no surprise that they
found that petitioner had no presence in any of these regions. In fact, if COMELEC’s findings are to be believed, petitioner
does not even exist in Quezon City, which is registered as Ang Ladlad’s principal place of business.

Against this backdrop, we find that Ang Ladlad has sufficiently demonstrated its compliance with the legal requirements for
accreditation. Indeed, aside from COMELEC’s moral objection and the belated allegation of non-existence, nowhere in the
records has the respondent ever found/ruled that Ang Ladlad is not qualified to register as a party-list organization under
any of the requisites under RA 7941 or the guidelines in Ang Bagong Bayani. The difference, COMELEC claims, lies
in Ang Ladlad’s morality, or lack thereof.

Religion as the Basis for Refusal to Accept Ang Ladlad’s Petition for Registration

Our Constitution provides in Article III, Section 5 that "[n]o law shall be made respecting an establishment of religion, or
prohibiting the free exercise thereof." At bottom, what our non-establishment clause calls for is "government neutrality in
religious matters."24 Clearly, "governmental reliance on religious justification is inconsistent with this policy of
neutrality."25 We thus find that it was grave violation of the non-establishment clause for the COMELEC to utilize the Bible
and the Koran to justify the exclusion of Ang Ladlad.

Rather than relying on religious belief, the legitimacy of the Assailed Resolutions should depend, instead, on whether the
COMELEC is able to advance some justification for its rulings beyond mere conformity to religious doctrine. Otherwise
stated, government must act for secular purposes and in ways that have primarily secular effects. As we held in Estrada v.
Escritor:26

x x x The morality referred to in the law is public and necessarily secular, not religious as the dissent of Mr. Justice Carpio
holds. "Religious teachings as expressed in public debate may influence the civil public order but public moral disputes
may be resolved only on grounds articulable in secular terms." Otherwise, if government relies upon religious beliefs in
formulating public policies and morals, the resulting policies and morals would require conformity to what some might
regard as religious programs or agenda. The non-believers would therefore be compelled to conform to a standard of
conduct buttressed by a religious belief, i.e., to a "compelled religion," anathema to religious freedom. Likewise, if
government based its actions upon religious beliefs, it would tacitly approve or endorse that belief and thereby also tacitly
disapprove contrary religious or non-religious views that would not support the policy. As a result, government will not
provide full religious freedom for all its citizens, or even make it appear that those whose beliefs are disapproved are
second-class citizens. 1avvphi1

In other words, government action, including its proscription of immorality as expressed in criminal law like concubinage,
must have a secular purpose. That is, the government proscribes this conduct because it is "detrimental (or dangerous) to
those conditions upon which depend the existence and progress of human society" and not because the conduct is
proscribed by the beliefs of one religion or the other. Although admittedly, moral judgments based on religion might have a
compelling influence on those engaged in public deliberations over what actions would be considered a moral
disapprobation punishable by law. After all, they might also be adherents of a religion and thus have religious opinions and
moral codes with a compelling influence on them; the human mind endeavors to regulate the temporal and spiritual
institutions of society in a uniform manner, harmonizing earth with heaven. Succinctly put, a law could be religious or
Kantian or Aquinian or utilitarian in its deepest roots, but it must have an articulable and discernible secular purpose and
justification to pass scrutiny of the religion clauses. x x x Recognizing the religious nature of the Filipinos and the elevating
influence of religion in society, however, the Philippine constitution's religion clauses prescribe not a strict but a benevolent
neutrality. Benevolent neutrality recognizes that government must pursue its secular goals and interests but at the same
time strive to uphold religious liberty to the greatest extent possible within flexible constitutional limits. Thus, although the
morality contemplated by laws is secular, benevolent neutrality could allow for accommodation of morality based on
religion, provided it does not offend compelling state interests.27

Public Morals as a Ground to Deny Ang Ladlad’s Petition for Registration

Respondent suggests that although the moral condemnation of homosexuality and homosexual conduct may be religion-
based, it has long been transplanted into generally accepted public morals. The COMELEC argues:

Petitioner’s accreditation was denied not necessarily because their group consists of LGBTs but because of the danger it
poses to the people especially the youth. Once it is recognized by the government, a sector which believes that there is
nothing wrong in having sexual relations with individuals of the same gender is a bad example. It will bring down the
standard of morals we cherish in our civilized society. Any society without a set of moral precepts is in danger of losing its
own existence.28

We are not blind to the fact that, through the years, homosexual conduct, and perhaps homosexuals themselves, have
borne the brunt of societal disapproval. It is not difficult to imagine the reasons behind this censure – religious beliefs,
convictions about the preservation of marriage, family, and procreation, even dislike or distrust of homosexuals
themselves and their perceived lifestyle. Nonetheless, we recall that the Philippines has not seen fit to criminalize
homosexual conduct. Evidently, therefore, these "generally accepted public morals" have not been convincingly
transplanted into the realm of law.29

The Assailed Resolutions have not identified any specific overt immoral act performed by Ang Ladlad. Even the OSG
agrees that "there should have been a finding by the COMELEC that the group’s members have committed or are
committing immoral acts."30 The OSG argues:

x x x A person may be sexually attracted to a person of the same gender, of a different gender, or more than one gender,
but mere attraction does not translate to immoral acts. There is a great divide between thought and action. Reduction ad
absurdum. If immoral thoughts could be penalized, COMELEC would have its hands full of disqualification cases against
both the "straights" and the gays." Certainly this is not the intendment of the law. 31
Respondent has failed to explain what societal ills are sought to be prevented, or why special protection is required for the
youth. Neither has the COMELEC condescended to justify its position that petitioner’s admission into the party-list system
would be so harmful as to irreparably damage the moral fabric of society. We, of course, do not suggest that the state is
wholly without authority to regulate matters concerning morality, sexuality, and sexual relations, and we recognize that the
government will and should continue to restrict behavior considered detrimental to society. Nonetheless, we cannot
countenance advocates who, undoubtedly with the loftiest of intentions, situate morality on one end of an argument or
another, without bothering to go through the rigors of legal reasoning and explanation. In this, the notion of morality is
robbed of all value. Clearly then, the bare invocation of morality will not remove an issue from our scrutiny.

We also find the COMELEC’s reference to purported violations of our penal and civil laws flimsy, at best; disingenuous, at
worst. Article 694 of the Civil Code defines a nuisance as "any act, omission, establishment, condition of property, or
anything else which shocks, defies, or disregards decency or morality," the remedies for which are a prosecution under
the Revised Penal Code or any local ordinance, a civil action, or abatement without judicial proceedings. 32 A violation of
Article 201 of the Revised Penal Code, on the other hand, requires proof beyond reasonable doubt to support a criminal
conviction. It hardly needs to be emphasized that mere allegation of violation of laws is not proof, and a mere blanket
invocation of public morals cannot replace the institution of civil or criminal proceedings and a judicial determination of
liability or culpability.

As such, we hold that moral disapproval, without more, is not a sufficient governmental interest to justify exclusion of
homosexuals from participation in the party-list system. The denial of Ang Ladlad’s registration on purely moral grounds
amounts more to a statement of dislike and disapproval of homosexuals, rather than a tool to further any substantial public
interest. Respondent’s blanket justifications give rise to the inevitable conclusion that the COMELEC targets homosexuals
themselves as a class, not because of any particular morally reprehensible act. It is this selective targeting that implicates
our equal protection clause.

Equal Protection

Despite the absolutism of Article III, Section 1 of our Constitution, which provides "nor shall any person be denied equal
protection of the laws," courts have never interpreted the provision as an absolute prohibition on classification. "Equality,"
said Aristotle, "consists in the same treatment of similar persons." 33 The equal protection clause guarantees that no person
or class of persons shall be deprived of the same protection of laws which is enjoyed by other persons or other classes in
the same place and in like circumstances.34

Recent jurisprudence has affirmed that if a law neither burdens a fundamental right nor targets a suspect class, we will
uphold the classification as long as it bears a rational relationship to some legitimate government end. 35 In Central Bank
Employees Association, Inc. v. Banko Sentral ng Pilipinas, 36 we declared that "[i]n our jurisdiction, the standard of analysis
of equal protection challenges x x x have followed the ‘rational basis’ test, coupled with a deferential attitude to legislative
classifications and a reluctance to invalidate a law unless there is a showing of a clear and unequivocal breach of the
Constitution."37

The COMELEC posits that the majority of the Philippine population considers homosexual conduct as immoral and
unacceptable, and this constitutes sufficient reason to disqualify the petitioner. Unfortunately for the respondent, the
Philippine electorate has expressed no such belief. No law exists to criminalize homosexual behavior or expressions or
parties about homosexual behavior. Indeed, even if we were to assume that public opinion is as the COMELEC describes
it, the asserted state interest here – that is, moral disapproval of an unpopular minority – is not a legitimate state interest
that is sufficient to satisfy rational basis review under the equal protection clause. The COMELEC’s differentiation, and its
unsubstantiated claim that Ang Ladlad cannot contribute to the formulation of legislation that would benefit the nation,
furthers no legitimate state interest other than disapproval of or dislike for a disfavored group.

From the standpoint of the political process, the lesbian, gay, bisexual, and transgender have the same interest in
participating in the party-list system on the same basis as other political parties similarly situated. State intrusion in this
case is equally burdensome. Hence, laws of general application should apply with equal force to LGBTs, and they deserve
to participate in the party-list system on the same basis as other marginalized and under-represented sectors.

It bears stressing that our finding that COMELEC’s act of differentiating LGBTs from heterosexuals insofar as the party-list
system is concerned does not imply that any other law distinguishing between heterosexuals and homosexuals under
different circumstances would similarly fail. We disagree with the OSG’s position that homosexuals are a class in
themselves for the purposes of the equal protection clause. 38 We are not prepared to single out homosexuals as a
separate class meriting special or differentiated treatment. We have not received sufficient evidence to this effect, and it is
simply unnecessary to make such a ruling today. Petitioner itself has merely demanded that it be recognized under the
same basis as all other groups similarly situated, and that the COMELEC made "an unwarranted and impermissible
classification not justified by the circumstances of the case."

Freedom of Expression and Association

Under our system of laws, every group has the right to promote its agenda and attempt to persuade society of the validity
of its position through normal democratic means.39 It is in the public square that deeply held convictions and differing
opinions should be distilled and deliberated upon. As we held in Estrada v. Escritor: 40

In a democracy, this common agreement on political and moral ideas is distilled in the public square. Where citizens are
free, every opinion, every prejudice, every aspiration, and every moral discernment has access to the public square where
people deliberate the order of their life together. Citizens are the bearers of opinion, including opinion shaped by, or
espousing religious belief, and these citizens have equal access to the public square. In this representative democracy,
the state is prohibited from determining which convictions and moral judgments may be proposed for public deliberation.
Through a constitutionally designed process, the people deliberate and decide. Majority rule is a necessary principle in this
democratic governance. Thus, when public deliberation on moral judgments is finally crystallized into law, the laws will
largely reflect the beliefs and preferences of the majority, i.e., the mainstream or median groups.  Nevertheless, in the very
act of adopting and accepting a constitution and the limits it specifies – including protection of religious freedom "not only
for a minority, however small – not only for a majority, however large – but for each of us" – the majority imposes upon
itself a self-denying ordinance. It promises not to do what it otherwise could do: to ride roughshod over the dissenting
minorities.

Freedom of expression constitutes one of the essential foundations of a democratic society, and this freedom applies not
only to those that are favorably received but also to those that offend, shock, or disturb. Any restriction imposed in this
sphere must be proportionate to the legitimate aim pursued. Absent any compelling state interest, it is not for the
COMELEC or this Court to impose its views on the populace. Otherwise stated, the COMELEC is certainly not free to
interfere with speech for no better reason than promoting an approved message or discouraging a disfavored one.

This position gains even more force if one considers that homosexual conduct is not illegal in this country. It follows that
both expressions concerning one’s homosexuality and the activity of forming a political association that supports LGBT
individuals are protected as well.

Other jurisdictions have gone so far as to categorically rule that even overwhelming public perception that homosexual
conduct violates public morality does not justify criminalizing same-sex conduct. 41 European and United Nations judicial
decisions have ruled in favor of gay rights claimants on both privacy and equality grounds, citing general privacy and equal
protection provisions in foreign and international texts.42 To the extent that there is much to learn from other jurisdictions
that have reflected on the issues we face here, such jurisprudence is certainly illuminating. These foreign authorities, while
not formally binding on Philippine courts, may nevertheless have persuasive influence on the Court’s analysis.

In the area of freedom of expression, for instance, United States courts have ruled that existing free speech doctrines
protect gay and lesbian rights to expressive conduct. In order to justify the prohibition of a particular expression of opinion,
public institutions must show that their actions were caused by "something more than a mere desire to avoid the
discomfort and unpleasantness that always accompany an unpopular viewpoint." 43

With respect to freedom of association for the advancement of ideas and beliefs, in Europe, with its vibrant human rights
tradition, the European Court of Human Rights (ECHR) has repeatedly stated that a political party may campaign for a
change in the law or the constitutional structures of a state if it uses legal and democratic means and the changes it
proposes are consistent with democratic principles. The ECHR has emphasized that political ideas that challenge the
existing order and whose realization is advocated by peaceful means must be afforded a proper opportunity of expression
through the exercise of the right of association, even if such ideas may seem shocking or unacceptable to the authorities
or the majority of the population. 44 A political group should not be hindered solely because it seeks to publicly debate
controversial political issues in order to find solutions capable of satisfying everyone concerned. 45 Only if a political party
incites violence or puts forward policies that are incompatible with democracy does it fall outside the protection of the
freedom of association guarantee.46

We do not doubt that a number of our citizens may believe that homosexual conduct is distasteful, offensive, or even
defiant. They are entitled to hold and express that view. On the other hand, LGBTs and their supporters, in all likelihood,
believe with equal fervor that relationships between individuals of the same sex are morally equivalent to heterosexual
relationships. They, too, are entitled to hold and express that view. However, as far as this Court is concerned, our
democracy precludes using the religious or moral views of one part of the community to exclude from consideration the
values of other members of the community.

Of course, none of this suggests the impending arrival of a golden age for gay rights litigants. It well may be that this
Decision will only serve to highlight the discrepancy between the rigid constitutional analysis of this Court and the more
complex moral sentiments of Filipinos. We do not suggest that public opinion, even at its most liberal, reflect a clear-cut
strong consensus favorable to gay rights claims and we neither attempt nor expect to affect individual perceptions of
homosexuality through this Decision.

The OSG argues that since there has been neither prior restraint nor subsequent punishment imposed on Ang Ladlad,
and its members have not been deprived of their right to voluntarily associate, then there has been no restriction on their
freedom of expression or association. The OSG argues that:

There was no utterance restricted, no publication censored, or any assembly denied. [COMELEC] simply exercised its
authority to review and verify the qualifications of petitioner as a sectoral party applying to participate in the party-list
system. This lawful exercise of duty cannot be said to be a transgression of Section 4, Article III of the Constitution.

xxxx

A denial of the petition for registration x x x does not deprive the members of the petitioner to freely take part in the
conduct of elections. Their right to vote will not be hampered by said denial. In fact, the right to vote is a constitutionally-
guaranteed right which cannot be limited.
As to its right to be elected in a genuine periodic election, petitioner contends that the denial of Ang Ladlad’s petition has
the clear and immediate effect of limiting, if not outrightly nullifying the capacity of its members to fully and equally
participate in public life through engagement in the party list elections.

This argument is puerile. The holding of a public office is not a right but a privilege subject to limitations imposed by law. x
x x47

The OSG fails to recall that petitioner has, in fact, established its qualifications to participate in the party-list system, and –
as advanced by the OSG itself – the moral objection offered by the COMELEC was not a limitation imposed by law. To the
extent, therefore, that the petitioner has been precluded, because of COMELEC’s action, from publicly expressing its
views as a political party and participating on an equal basis in the political process with other equally-qualified party-list
candidates, we find that there has, indeed, been a transgression of petitioner’s fundamental rights.

Non-Discrimination and International Law

In an age that has seen international law evolve geometrically in scope and promise, international human rights law, in
particular, has grown dynamically in its attempt to bring about a more just and humane world order. For individuals and
groups struggling with inadequate structural and governmental support, international human rights norms are particularly
significant, and should be effectively enforced in domestic legal systems so that such norms may become actual, rather
than ideal, standards of conduct.

Our Decision today is fully in accord with our international obligations to protect and promote human rights. In particular,
we explicitly recognize the principle of non-discrimination as it relates to the right to electoral participation, enunciated in
the UDHR and the ICCPR.

The principle of non-discrimination is laid out in Article 26 of the ICCPR, as follows:

Article 26

All persons are equal before the law and are entitled without any discrimination to the equal protection of the law. In this
respect, the law shall prohibit any discrimination and guarantee to all persons equal and effective protection against
discrimination on any ground such as race, colour, sex, language, religion, political or other opinion, national or social
origin, property, birth or other status.

In this context, the principle of non-discrimination requires that laws of general application relating to elections be applied
equally to all persons, regardless of sexual orientation. Although sexual orientation is not specifically enumerated as a
status or ratio for discrimination in Article 26 of the ICCPR, the ICCPR Human Rights Committee has opined that the
reference to "sex" in Article 26 should be construed to include "sexual orientation." 48 Additionally, a variety of United
Nations bodies have declared discrimination on the basis of sexual orientation to be prohibited under various international
agreements.49

The UDHR provides:

Article 21.

(1) Everyone has the right to take part in the government of his country, directly or through freely chosen representatives.

Likewise, the ICCPR states:

Article 25

Every citizen shall have the right and the opportunity, without any of the distinctions mentioned in article 2 and without
unreasonable restrictions:

(a) To take part in the conduct of public affairs, directly or through freely chosen representatives;

(b) To vote and to be elected at genuine periodic elections which shall be by universal and equal suffrage and
shall be held by secret ballot, guaranteeing the free expression of the will of the electors;

(c) To have access, on general terms of equality, to public service in his country.

As stated by the CHR in its Comment-in-Intervention, the scope of the right to electoral participation is elaborated by the
Human Rights Committee in its General Comment No. 25 (Participation in Public Affairs and the Right to Vote) as follows:

1. Article 25 of the Covenant recognizes and protects the right of every citizen to take part in the conduct of public affairs,
the right to vote and to be elected and the right to have access to public service. Whatever form of constitution or
government is in force, the Covenant requires States to adopt such legislative and other measures as may be necessary
to ensure that citizens have an effective opportunity to enjoy the rights it protects. Article 25 lies at the core of democratic
government based on the consent of the people and in conformity with the principles of the Covenant.
xxxx

15. The effective implementation of the right and the opportunity to stand for elective office ensures that persons entitled to
vote have a free choice of candidates. Any restrictions on the right to stand for election, such as minimum age, must be
justifiable on objective and reasonable criteria. Persons who are otherwise eligible to stand for election should not be
excluded by unreasonable or discriminatory requirements such as education, residence or descent, or by reason of
political affiliation. No person should suffer discrimination or disadvantage of any kind because of that person's candidacy.
States parties should indicate and explain the legislative provisions which exclude any group or category of persons from
elective office.50

We stress, however, that although this Court stands willing to assume the responsibility of giving effect to the Philippines’
international law obligations, the blanket invocation of international law is not the panacea for all social ills. We refer now
to the petitioner’s invocation of the Yogyakarta Principles (the Application of International Human Rights Law In Relation to
Sexual Orientation and Gender Identity),51 which petitioner declares to reflect binding principles of international law.

At this time, we are not prepared to declare that these Yogyakarta Principles contain norms that are obligatory on the
Philippines. There are declarations and obligations outlined in said Principles which are not reflective of the current state
of international law, and do not find basis in any of the sources of international law enumerated under Article 38(1) of the
Statute of the International Court of Justice.52 Petitioner has not undertaken any objective and rigorous analysis of these
alleged principles of international law to ascertain their true status.

We also hasten to add that not everything that society – or a certain segment of society – wants or demands is
automatically a human right. This is not an arbitrary human intervention that may be added to or subtracted from at will. It
is unfortunate that much of what passes for human rights today is a much broader context of needs that identifies many
social desires as rights in order to further claims that international law obliges states to sanction these innovations. This
has the effect of diluting real human rights, and is a result of the notion that if "wants" are couched in "rights" language,
then they are no longer controversial.1avvphi1

Using even the most liberal of lenses, these Yogyakarta Principles, consisting of a declaration formulated by various
international law professors, are – at best – de lege ferenda – and do not constitute binding obligations on the Philippines.
Indeed, so much of contemporary international law is characterized by the "soft law" nomenclature, i.e., international law is
full of principles that promote international cooperation, harmony, and respect for human rights, most of which amount to
no more than well-meaning desires, without the support of either State practice or opinio juris. 53

As a final note, we cannot help but observe that the social issues presented by this case are emotionally charged, societal
attitudes are in flux, even the psychiatric and religious communities are divided in opinion. This Court’s role is not to
impose its own view of acceptable behavior. Rather, it is to apply the Constitution and laws as best as it can, uninfluenced
by public opinion, and confident in the knowledge that our democracy is resilient enough to withstand vigorous debate.

WHEREFORE, the Petition is hereby GRANTED. The Resolutions of the Commission on Elections dated November 11,
2009 and December 16, 2009 in SPP No. 09-228 (PL) are hereby SET ASIDE. The Commission on Elections is directed
to GRANT petitioner’s application for party-list accreditation.

SO ORDERED.

Kinds of Autonomy: Administrative Autonomy v Political Autonomy


-Limbona v Conte Mangelin, 170 SCRA 786

The acts of the Sangguniang Pampook of Region XII are assailed in this petition. The antecedent facts are as follows:

1. On September 24, 1986, petitioner Sultan Alimbusar Limbona was appointed as a member of the
Sangguniang Pampook, Regional Autonomous Government, Region XII, representing Lanao del Sur.

2. On March 12, 1987 petitioner was elected Speaker of the Regional Legislative Assembly or Batasang
Pampook of Central Mindanao (Assembly for brevity).

3. Said Assembly is composed of eighteen (18) members. Two of said members, respondents Acmad
Tomawis and Pakil Dagalangit, filed on March 23, 1987 with the Commission on Elections their respective
certificates of candidacy in the May 11, 1987 congressional elections for the district of Lanao del Sur but
they later withdrew from the aforesaid election and thereafter resumed again their positions as members of
the Assembly.

4. On October 21, 1987 Congressman Datu Guimid Matalam, Chairman of the Committee on Muslim
Affairs of the House of Representatives, invited Mr. Xavier Razul, Pampook Speaker of Region XI,
Zamboanga City and the petitioner in his capacity as Speaker of the Assembly, Region XII, in a letter
which reads:

The Committee on Muslim Affairs well undertake consultations and dialogues with local
government officials, civic, religious organizations and traditional leaders on the recent and
present political developments and other issues affecting Regions IX and XII.
The result of the conference, consultations and dialogues would hopefully chart the
autonomous governments of the two regions as envisioned and may prod the President to
constitute immediately the Regional Consultative Commission as mandated by the
Commission.

You are requested to invite some members of the Pampook Assembly of your respective
assembly on November 1 to 15, 1987, with venue at the Congress of the Philippines. Your
presence, unstinted support and cooperation is (sic) indispensable.

5. Consistent with the said invitation, petitioner sent a telegram to Acting Secretary Johnny Alimbuyao of
the Assembly to wire all Assemblymen that there shall be no session in November as "our presence in the
house committee hearing of Congress take (sic) precedence over any pending business in batasang
pampook ... ."

6. In compliance with the aforesaid instruction of the petitioner, Acting Secretary Alimbuyao sent to the
members of the Assembly the following telegram:

TRANSMITTING FOR YOUR INFORMATION AND GUIDANCE TELEGRAM RECEIVED


FROM SPEAKER LIMBONA QUOTE CONGRESSMAN JIMMY MATALAM CHAIRMAN
OF THE HOUSE COMMITTEE ON MUSLIM AFFAIRS REQUESTED ME TO ASSIST
SAID COMMITTEE IN THE DISCUSSION OF THE PROPOSED AUTONOMY ORGANIC
NOV. 1ST TO 15. HENCE WERE ALL ASSEMBLYMEN THAT THERE SHALL BE NO
SESSION IN NOVEMBER AS OUR PRESENCE IN THE HOUSE COMMITTEE HEARING
OF CONGRESS TAKE PRECEDENCE OVER ANY PENDING BUSINESS IN BATASANG
PAMPOOK OF MATALAM FOLLOWS UNQUOTE REGARDS.

7. On November 2, 1987, the Assembly held session in defiance of petitioner's advice, with the following
assemblymen present:

1. Sali, Salic

2. Conding, Pilipinas (sic)

3. Dagalangit, Rakil

4. Dela Fuente, Antonio

5. Mangelen, Conte

6. Ortiz, Jesus

7. Palomares, Diego

8. Sinsuat, Bimbo

9. Tomawis, Acmad

10. Tomawis, Jerry

After declaring the presence of a quorum, the Speaker Pro-Tempore was authorized to preside in the
session. On Motion to declare the seat of the Speaker vacant, all Assemblymen in attendance voted in the
affirmative, hence, the chair declared said seat of the Speaker vacant. 8. On November 5, 1987, the
session of the Assembly resumed with the following Assemblymen present:

1. Mangelen Conte-Presiding Officer

2. Ali Salic

3. Ali Salindatu

4. Aratuc, Malik

5. Cajelo, Rene

6. Conding, Pilipinas (sic)

7. Dagalangit, Rakil

8. Dela Fuente, Antonio


9. Ortiz, Jesus

10 Palomares, Diego

11. Quijano, Jesus

12. Sinsuat, Bimbo

13. Tomawis, Acmad

14. Tomawis, Jerry

An excerpt from the debates and proceeding of said session reads:

HON. DAGALANGIT: Mr. Speaker, Honorable Members of the House, with the presence of our colleagues
who have come to attend the session today, I move to call the names of the new comers in order for them
to cast their votes on the previous motion to declare the position of the Speaker vacant. But before doing
so, I move also that the designation of the Speaker Pro Tempore as the Presiding Officer and Mr. Johnny
Evangelists as Acting Secretary in the session last November 2, 1987 be reconfirmed in today's session.

HON. SALIC ALI: I second the motions.

PRESIDING OFFICER: Any comment or objections on the two motions presented? Me chair hears none
and the said motions are approved. ...

Twelve (12) members voted in favor of the motion to declare the seat of the Speaker vacant; one
abstained and none voted against.  1

Accordingly, the petitioner prays for judgment as follows:

WHEREFORE, petitioner respectfully prays that-

(a) This Petition be given due course;

(b) Pending hearing, a restraining order or writ of preliminary injunction be issued enjoining respondents
from proceeding with their session to be held on November 5, 1987, and on any day thereafter;

(c) After hearing, judgment be rendered declaring the proceedings held by respondents of their session on
November 2, 1987 as null and void;

(d) Holding the election of petitioner as Speaker of said Legislative Assembly or Batasan Pampook, Region
XII held on March 12, 1987 valid and subsisting, and

(e) Making the injunction permanent.

Petitioner likewise prays for such other relief as may be just and equitable. 
2

Pending further proceedings, this Court, on January 19, 1988, received a resolution filed by the Sangguniang Pampook,
"EXPECTING ALIMBUSAR P. LIMBONA FROM MEMBERSHIP OF THE SANGGUNIANG PAMPOOK AUTONOMOUS
REGION XII,"   on the grounds, among other things, that the petitioner "had caused to be prepared and signed by him
3

paying [sic] the salaries and emoluments of Odin Abdula, who was considered resigned after filing his Certificate of
Candidacy for Congressmen for the First District of Maguindanao in the last May 11, elections. . . and nothing in the record
of the Assembly will show that any request for reinstatement by Abdula was ever made . . ."   and that "such action of Mr.
4

Lim bona in paying Abdula his salaries and emoluments without authority from the Assembly . . . constituted a usurpation
of the power of the Assembly,"   that the petitioner "had recently caused withdrawal of so much amount of cash from the
5

Assembly resulting to the non-payment of the salaries and emoluments of some Assembly [sic],"   and that he had "filed a
6

case before the Supreme Court against some members of the Assembly on question which should have been resolved
within the confines of the Assembly,"   for which the respondents now submit that the petition had become "moot and
7

academic".  8

The first question, evidently, is whether or not the expulsion of the petitioner (pending litigation) has made the case moot
and academic.

We do not agree that the case has been rendered moot and academic by reason simply of the expulsion resolution so
issued. For, if the petitioner's expulsion was done purposely to make this petition moot and academic, and to preempt the
Court, it will not make it academic.

On the ground of the immutable principle of due process alone, we hold that the expulsion in question is of no force and
effect. In the first place, there is no showing that the Sanggunian had conducted an investigation, and whether or not the
petitioner had been heard in his defense, assuming that there was an investigation, or otherwise given the opportunity to
do so. On the other hand, what appears in the records is an admission by the Assembly (at least, the respondents) that
"since November, 1987 up to this writing, the petitioner has not set foot at the Sangguniang Pampook."   "To be sure, the
9

private respondents aver that "[t]he Assemblymen, in a conciliatory gesture, wanted him to come to Cotabato City,"   but 10

that was "so that their differences could be threshed out and settled."   Certainly, that avowed wanting or desire to thresh
11

out and settle, no matter how conciliatory it may be cannot be a substitute for the notice and hearing contemplated by law.

While we have held that due process, as the term is known in administrative law, does not absolutely require notice and
that a party need only be given the opportunity to be heard,   it does not appear herein that the petitioner had, to begin
12

with, been made aware that he had in fact stood charged of graft and corruption before his collegues. It cannot be said
therefore that he was accorded any opportunity to rebut their accusations. As it stands, then, the charges now levelled
amount to mere accusations that cannot warrant expulsion.

In the second place, (the resolution) appears strongly to be a bare act of vendetta by the other Assemblymen against the
petitioner arising from what the former perceive to be abduracy on the part of the latter. Indeed, it (the resolution) speaks
of "a case [having been filed] [by the petitioner] before the Supreme Court . . . on question which should have been
resolved within the confines of the Assemblyman act which some members claimed unnecessarily and unduly assails their
integrity and character as representative of the people"   an act that cannot possibly justify expulsion. Access to judicial
13

remedies is guaranteed by the Constitution,   and, unless the recourse amounts to malicious prosecution, no one may be
14

punished for seeking redress in the courts.

We therefore order reinstatement, with the caution that should the past acts of the petitioner indeed warrant his removal,
the Assembly is enjoined, should it still be so minded, to commence proper proceedings therefor in line with the most
elementary requirements of due process. And while it is within the discretion of the members of the Sanggunian to punish
their erring colleagues, their acts are nonetheless subject to the moderating band of this Court in the event that such
discretion is exercised with grave abuse.

It is, to be sure, said that precisely because the Sangguniang Pampook(s) are "autonomous," the courts may not rightfully
intervene in their affairs, much less strike down their acts. We come, therefore, to the second issue: Are the so-called
autonomous governments of Mindanao, as they are now constituted, subject to the jurisdiction of the national courts? In
other words, what is the extent of self-government given to the two autonomous governments of Region IX and XII?

The autonomous governments of Mindanao were organized in Regions IX and XII by Presidential Decree No.
1618   promulgated on July 25, 1979. Among other things, the Decree established "internal autonomy"   in the two regions
15 16

"[w]ithin the framework of the national sovereignty and territorial integrity of the Republic of the Philippines and its
Constitution,"   with legislative and executive machinery to exercise the powers and responsibilities   specified therein.
17 18

It requires the autonomous regional governments to "undertake all internal administrative matters for the respective
regions,"   except to "act on matters which are within the jurisdiction and competence of the National
19

Government,"   "which include, but are not limited to, the following:
20

(1) National defense and security;

(2) Foreign relations;

(3) Foreign trade;

(4) Currency, monetary affairs, foreign exchange, banking and quasi-banking, and external borrowing,

(5) Disposition, exploration, development, exploitation or utilization of all natural resources;

(6) Air and sea transport

(7) Postal matters and telecommunications;

(8) Customs and quarantine;

(9) Immigration and deportation;

(10) Citizenship and naturalization;

(11) National economic, social and educational planning; and

(12) General auditing.  21

In relation to the central government, it provides that "[t]he President shall have the power of general supervision and
control over the Autonomous Regions ..."  22

Now, autonomy is either decentralization of administration or decentralization of power. There is decentralization of


administration when the central government delegates administrative powers to political subdivisions in order to broaden
the base of government power and in the process to make local governments "more responsive and accountable,"   "and 23
ensure their fullest development as self-reliant communities and make them more effective partners in the pursuit of
national development and social progress."   At the same time, it relieves the central government of the burden of
24

managing local affairs and enables it to concentrate on national concerns. The President exercises "general
supervision"   over them, but only to "ensure that local affairs are administered according to law."   He has no control over
25 26

their acts in the sense that he can substitute their judgments with his own.  27

Decentralization of power, on the other hand, involves an abdication of political power in the favor of local governments
units declare to be autonomous . In that case, the autonomous government is free to chart its own destiny and shape its
future with minimum intervention from central authorities. According to a constitutional author, decentralization of power
amounts to "self-immolation," since in that event, the autonomous government becomes accountable not to the central
authorities but to its constituency.  28

But the question of whether or not the grant of autonomy Muslim Mindanao under the 1987 Constitution involves, truly, an
effort to decentralize power rather than mere administration is a question foreign to this petition, since what is involved
herein is a local government unit constituted prior to the ratification of the present Constitution. Hence, the Court will not
resolve that controversy now, in this case, since no controversy in fact exists. We will resolve it at the proper time and in
the proper case.

Under the 1987 Constitution, local government units enjoy autonomy in these two senses, thus:

Section 1. The territorial and political subdivisions of the Republic of the Philippines are the provinces,
cities, municipalities, and barangays. Here shall be autonomous regions in Muslim Mindanao ,and the
Cordilleras as hereinafter provided. 29

Sec. 2. The territorial and political subdivisions shall enjoy local autonomy. 
30

xxx xxx xxx

See. 15. Mere shall be created autonomous regions in Muslim Mindanao and in the Cordilleras consisting
of provinces, cities, municipalities, and geographical areas sharing common and distinctive historical and
cultural heritage, economic and social structures, and other relevant characteristics within the framework of
this Constitution and the national sovereignty as well as territorial integrity of the Republic of the
Philippines.  31

An autonomous government that enjoys autonomy of the latter category [CONST. (1987), art. X, sec. 15.] is subject alone
to the decree of the organic act creating it and accepted principles on the effects and limits of "autonomy." On the other
hand, an autonomous government of the former class is, as we noted, under the supervision of the national government
acting through the President (and the Department of Local Government).   If the Sangguniang Pampook (of Region XII),
32

then, is autonomous in the latter sense, its acts are, debatably beyond the domain of this Court in perhaps the same way
that the internal acts, say, of the Congress of the Philippines are beyond our jurisdiction. But if it is autonomous in the
former category only, it comes unarguably under our jurisdiction. An examination of the very Presidential Decree creating
the autonomous governments of Mindanao persuades us that they were never meant to exercise autonomy in the second
sense, that is, in which the central government commits an act of self-immolation. Presidential Decree No. 1618, in the
first place, mandates that "[t]he President shall have the power of general supervision and control over Autonomous
Regions."  In the second place, the Sangguniang Pampook, their legislative arm, is made to discharge chiefly
33

administrative services, thus:

SEC. 7. Powers of the Sangguniang Pampook. The Sangguniang Pampook shall exercise local legislative
powers over regional affairs within the framework of national development plans, policies and goals, in the
following areas:

(1) Organization of regional administrative system;

(2) Economic, social and cultural development of the Autonomous Region;

(3) Agricultural, commercial and industrial programs for the Autonomous Region;

(4) Infrastructure development for the Autonomous Region;

(5) Urban and rural planning for the Autonomous Region;

(6) Taxation and other revenue-raising measures as provided for in this Decree;

(7) Maintenance, operation and administration of schools established by the Autonomous Region;

(8) Establishment, operation and maintenance of health, welfare and other social services, programs and
facilities;

(9) Preservation and development of customs, traditions, languages and culture indigenous to the
Autonomous Region; and
(10) Such other matters as may be authorized by law,including the enactment of such measures as may
be necessary for the promotion of the general welfare of the people in the Autonomous Region.

The President shall exercise such powers as may be necessary to assure that enactment and acts of the
Sangguniang Pampook and the Lupong Tagapagpaganap ng Pook are in compliance with this Decree,
national legislation, policies, plans and programs.

The Sangguniang Pampook shall maintain liaison with the Batasang Pambansa.  34

Hence, we assume jurisdiction. And if we can make an inquiry in the validity of the expulsion in question, with more reason
can we review the petitioner's removal as Speaker.

Briefly, the petitioner assails the legality of his ouster as Speaker on the grounds that: (1) the Sanggunian, in convening on
November 2 and 5, 1987 (for the sole purpose of declaring the office of the Speaker vacant), did so in violation of the
Rules of the Sangguniang Pampook since the Assembly was then on recess; and (2) assuming that it was valid, his ouster
was ineffective nevertheless for lack of quorum.

Upon the facts presented, we hold that the November 2 and 5, 1987 sessions were invalid. It is true that under Section 31
of the Region XII Sanggunian Rules, "[s]essions shall not be suspended or adjourned except by direction of the
Sangguniang Pampook,"   but it provides likewise that "the Speaker may, on [sic] his discretion, declare a recess of "short
35

intervals."   Of course, there is disagreement between the protagonists as to whether or not the recess called by the
36

petitioner effective November 1 through 15, 1987 is the "recess of short intervals" referred to; the petitioner says that it is
while the respondents insist that, to all intents and purposes, it was an adjournment and that "recess" as used by their
Rules only refers to "a recess when arguments get heated up so that protagonists in a debate can talk things out
informally and obviate dissenssion [sic] and disunity.   The Court agrees with the respondents on this regard, since
37

clearly, the Rules speak of "short intervals." Secondly, the Court likewise agrees that the Speaker could not have validly
called a recess since the Assembly had yet to convene on November 1, the date session opens under the same
Rules.   Hence, there can be no recess to speak of that could possibly interrupt any session. But while this opinion is in
38

accord with the respondents' own, we still invalidate the twin sessions in question, since at the time the petitioner called
the "recess," it was not a settled matter whether or not he could. do so. In the second place, the invitation tendered by the
Committee on Muslim Affairs of the House of Representatives provided a plausible reason for the intermission sought.
Thirdly, assuming that a valid recess could not be called, it does not appear that the respondents called his attention to
this mistake. What appears is that instead, they opened the sessions themselves behind his back in an apparent act of
mutiny. Under the circumstances, we find equity on his side. For this reason, we uphold the "recess" called on the ground
of good faith.

It does not appear to us, moreover, that the petitioner had resorted to the aforesaid "recess" in order to forestall the
Assembly from bringing about his ouster. This is not apparent from the pleadings before us. We are convinced that the
invitation was what precipitated it.

In holding that the "recess" in question is valid, we are not to be taken as establishing a precedent, since, as we said, a
recess can not be validly declared without a session having been first opened. In upholding the petitioner herein, we are
not giving him a carte blanche to order recesses in the future in violation of the Rules, or otherwise to prevent the lawful
meetings thereof.

Neither are we, by this disposition, discouraging the Sanggunian from reorganizing itself pursuant to its lawful
prerogatives. Certainly, it can do so at the proper time. In the event that be petitioner should initiate obstructive moves, the
Court is certain that it is armed with enough coercive remedies to thwart them.  39

In view hereof, we find no need in dwelling on the issue of quorum.

WHEREFORE, premises considered, the petition is GRANTED. The Sangguniang Pampook, Region XII, is ENJOINED to
(1) REINSTATE the petitioner as Member, Sangguniang Pampook, Region XII; and (2) REINSTATE him as Speaker
thereof. No costs.

SO ORDERED.

Judicial naturalization; CA 63, CA 473, RA 530


-So v Republic of the Philippines, 513 SCRA 267 (2007)

Assailed in this Petition for Review on Certiorari is the Decision 1 of the Court of Appeals (CA) in CA-G.R. CV No. 80437
which reversed the Decision2 of the Regional Trial Court (RTC) of Manila, Branch 8, in Naturalization Case No. 02-102984.
Likewise assailed is the appellate court’s Resolution denying the Motion for Reconsideration of its Decision.

Antecedents

On February 28, 2002, petitioner Edison So filed before the RTC a Petition for Naturalization 3 under Commonwealth Act
(C.A.) No. 473, otherwise known as the Revised Naturalization Law, as amended. He alleged the following in his petition:
He was born on February 17, 1982, in Manila; he is a Chinese citizen who has lived in No. 528 Lavezares St., Binondo,
Manila, since birth; as an employee, he derives an average annual income of around P100,000.00 with free board and
lodging and other benefits; he is single, able to speak and write English, Chinese and Tagalog; he is exempt from the filing
of Declaration of Intention to become a citizen of the Philippines pursuant to Section 6 of Commonwealth Act (C.A.) No.
473, as amended, because he was born in the Philippines, and studied in a school recognized by the Government where
Philippine history, government and culture are taught; he is a person of good moral character; he believes in the principles
underlying the Philippine constitution; he has conducted himself in a proper and irreproachable manner during the entire
period of his residence in the Philippines in his relation with the constituted government as well as with the community in
which he is living; he has mingled socially with the Filipinos and has evinced a sincere desire to learn and embrace the
customs, traditions and ideals of the Filipino people; he has all the qualifications provided under Section 2 and none of the
disqualifications under Section 4 of C.A. No. 473, as amended; he is not opposed to organized government or affiliated
with any association or group of persons who uphold and teach doctrines opposing all organized governments; he is not
defending or teaching the necessity or propriety of violence, personal assault or assassination for the success or
predominance of men’s ideas; he is not a polygamist or a believer in the practice of polygamy; he has not been convicted
of any crime involving moral turpitude; he is not suffering from any incurable contagious diseases or from mental
alienation; the nation of which he is a citizen is not at war with the Philippines; it is his intention in good faith to become a
citizen of the Philippines and to renounce absolutely and forever all allegiance and fidelity to any foreign prince, potentate,
state or sovereignty, and particularly to China; and he will reside continuously in the Philippines from the time of the filing
of the petition up to the time of his admission as citizen of the Philippines. The petition was docketed as Naturalization
Case No. 02-102984.

Attached to the petition were the Joint Affidavit4 of Atty. Artemio Adasa, Jr. and Mark B. Salcedo; and petitioner’s
Certificate of Live Birth,5 Alien Certificate of Registration, 6 and Immigrant Certificate of Residence.7

On March 22, 2002, the RTC issued an Order8 setting the petition for hearing at 8:30 a.m. of December 12 and 17, 2002
during which all persons concerned were enjoined to show cause, if any, why the petition should not be granted. The
entire petition and its annexes, including the order, were ordered published once a week for three consecutive weeks in
the Official Gazette and also in a newspaper of general circulation in the City of Manila. The RTC likewise ordered that
copies of the petition and notice be posted in public and conspicuous places in the Manila City Hall Building. 9

Petitioner thus caused the publication of the above order, as well as the entire petition and its annexes, in the Official
Gazette on May 20, 200210 and May 27, 2002,11 and in Today, a newspaper of general circulation in the City of Manila, on
May 25, 2002 and June 1, 2002.

No one opposed the petition. During the hearing, petitioner presented Atty. Adasa, Jr. who testified that he came to know
petitioner in 1991 as the legal consultant and adviser of the So family’s business. He would usually attend parties and
other social functions hosted by petitioner’s family. He knew petitioner to be obedient, hardworking, and possessed of
good moral character, including all the qualifications mandated by law. Atty. Adasa, Jr. further testified that petitioner was
gainfully employed and presently resides at No. 528 Lavezares Street, Binondo, Manila; petitioner had been practicing
Philippine tradition and those embodied in the Constitution; petitioner had been socially active, mingled with some of his
neighbors and had conducted himself in a proper and irreproachable manner during his entire stay in the Philippines; and
petitioner and his family observed Christmas and New Year and some occasions such as fiestas. According to the
witness, petitioner was not disqualified under C.A. No. 473 to become a Filipino citizen: he is not opposed to organized
government or believes in the use of force; he is not a polygamist and has not been convicted of a crime involving moral
turpitude; neither is he suffering from any mental alienation or any incurable disease. 12

Another witness for petitioner, Mark Salcedo, testified that he has known petitioner for ten (10) years; they first met at a
birthday party in 1991. He and petitioner were classmates at the University of Santo Tomas (UST) where they took up
Pharmacy. Petitioner was a member of some school organizations and mingled well with friends. 13 Salcedo further testified
that he saw petitioner twice a week, and during fiestas and special occasions when he would go to petitioner’s house. He
has known petitioner to have resided in Manila since birth. Petitioner is intelligent, a person of good moral character, and
believes in the principles of the Philippine Constitution. Petitioner has a gainful occupation, has conducted himself in a
proper and irreproachable manner and has all the qualifications to become a Filipino citizen.

Petitioner also testified and attempted to prove that he has all the qualifications and none of the disqualifications to
become a citizen of the Philippines.

At the conclusion of his testimonial evidence, petitioner offered in evidence the following documents: (1) Certificate of Live
Birth;14 (2) Alien Certificate of Registration; 15 (3) Immigrant Certificate of Residence; 16 (4) Elementary Pupil’s17 and High
School Student’s18 Permanent Record issued by Chang Kai Shek College; (5) Transcript of Record issued by the
University of Santo Tomas;19 (6) Certification of Part-Time Employment dated November 20, 2002; 20 (7) Income Tax
Returns and Certificate of Withholding Tax for the year 2001; 21 (8) Certification from Metrobank that petitioner is a
depositor;22 (9) Clearances that he has not been charged or convicted of any crime involving moral turpitude; 23 and (10)
Medical Certificates and Psychiatric Evaluation issued by the Philippine General Hospital. 24 The RTC admitted all these in
evidence.

The RTC granted the petition on June 4, 2003.25 The fallo of the decision reads:

WHEREFORE, judgment is hereby rendered GRANTING the petition and declaring that petitioner EDISON SO has all the
qualifications and none of the disqualifications to become a Filipino citizen and he is hereby admitted as citizen of the
Philippines, after taking the necessary oath of allegiance, as soon as this decision becomes final, subject to payment of
cost of P30,000.00.
SO ORDERED.26

The trial court ruled that the witnesses for petitioner had known him for the period required by law, and they had affirmed
that petitioner had all the qualifications and none of the disqualifications to become a Filipino citizen. Thus, the court
concluded that petitioner had satisfactorily supported his petition with evidence.

Respondent Republic of the Philippines, through the Office of the Solicitor General (OSG), appealed the decision to the
CA on the following grounds:

I.

THE LOWER COURT ERRED IN GRANTING THE PETITION FOR NATURALIZATION DESPITE THE FACT THAT THE
TWO (2) CHARACTER WITNESSES, NAMELY: ARTEMIO ADASA, JR. AND MARK SALCEDO WERE NOT QUALIFIED
CHARACTER WITNESSES.

II.

PETITIONER IS NOT QUALIFIED TO BE ADMITTED AS CITIZEN OF THE PHILIPPINES. 27

Respondent contended that based on the evidence on record, appellee failed to prove that he possesses all the
qualifications under Section 2 and none of the disqualifications under Section 4 of C.A. No. 473. It insisted that his two (2)
character witnesses did not know him well enough to vouch for his fitness to become a Filipino citizen; they merely made
general statements without giving specific details about his character and moral conduct. 28 The witnesses did not even
reside in the same place as petitioner.29 Respondent likewise argued that petitioner himself failed to prove that he is
qualified to become a Filipino citizen because he did not give any explanation or specific answers to the questions
propounded by his lawyer. He merely answered "yes" or "no" or gave general statements in answer to his counsel’s
questions. Thus, petitioner was unable to prove that he had all the qualifications and none of the disqualifications required
by law to be a naturalized Filipino citizen.30

On the other hand, petitioner averred that he graduated cum laude from the UST with the degree of Bachelor of Science in
Pharmacy. He is now on his second year as a medical student at the UST Medicine and Surgery. He avers that the
requirements for naturalization under C.A. No. 473, as amended by LOI 270, in relation to Presidential Decree Nos. 836
and 1379, had been relaxed after the Philippine government entered into diplomatic relations with the People’s Republic of
China; the requirements were further relaxed when Republic Act (R.A.) No. 9139 was signed into law. 31 Petitioner pointed
out that the petition, with all its annexes, was published in the official gazette and a newspaper of general circulation;
notices were likewise sent to the National Bureau of Investigation, Department of Justice, Department of Foreign Affairs,
and the OSG. But none from these offices came forward to oppose the petition before the lower court. 32 Petitioner insisted
that he has all the qualifications and none of the disqualifications to become Filipino. This was clearly established by his
witnesses.

In its Reply Brief, respondent alleged that R.A. No. 9139 applies to administrative naturalization filed with the Special
Committee on Naturalization. It insisted that even in the absence of any opposition, a petition for naturalization may be
dismissed.

In its Decision33 dated August 4, 2005, the CA set aside the ruling of the RTC and dismissed the petition for naturalization
without prejudice.34 According to the CA, petitioner’s two (2) witnesses were not credible because they failed to mention
specific details of petitioner’s life or character to show how well they knew him; they merely "parroted" the provisions of the
Naturalization Act without clearly explaining their applicability to petitioner’s case. 35 The appellate court likewise ruled that
petitioner failed to comply with the requirement of the law that the applicant must not be less than 21 years of age on the
day of the hearing of the petition; during the first hearing on December 12, 2002, petitioner was only twenty (20) years,
nine (9) months, and twenty five (25) days old, falling short of the requirement. 36 The CA stated, however, that it was not its
intention to forever close the door to any future application for naturalization which petitioner would file, and that it believes
that he would make a good Filipino citizen in due time, a decided asset to this country. 37

Petitioner’s motion for reconsideration38 was denied in a Resolution39 dated November 24, 2005; hence, the present
petition grounded on the sole issue:

WHETHER OR NOT THE HONORABLE COURT OF APPEALS COMMITTED REVERSIBLE ERROR WHEN IT
REVERSED THE DECISION OF THE REGIONAL TRIAL COURT OF MANILA. 40

In support of his petition, petitioner reiterates the arguments he set forth in the Brief filed before the CA.

In its Comment41 on the petition, respondent countered that R.A. No. 9139 (which took effect on August 8, 2001 and where
the applicant’s age requirement was lowered to eighteen (18) years old), refers only to administrative naturalization filed
with the Special Committee on Naturalization; it does not apply to judicial naturalization before the court, as in the present
case.42 Respondent, through the OSG, avers that its failure to oppose the petition before the court a quo does not preclude
it from appealing the decision of the RTC to the CA; it is even authorized to question an already final decision by filing a
petition for cancellation of citizenship. 43 Lastly, respondent reiterates its argument that petitioner’s character witnesses are
not qualified to prove the former’s qualifications.
In determining whether or not an applicant for naturalization is entitled to become a Filipino citizen, it is necessary to
resolve the following issues: (1) whether or not R.A. No. 9139 applies to petitions for naturalization by judicial act; and (2)
whether or not the witnesses presented by petitioner are "credible" in accordance with the jurisprudence and the definition
and guidelines set forth in C.A. No. 473.

The petition is denied for lack of merit.

Naturalization signifies the act of formally adopting a foreigner into the political body of a nation by clothing him or her with
the privileges of a citizen.44 Under current and existing laws, there are three ways by which an alien may become a citizen
by naturalization: (a) administrative naturalization pursuant to R.A. No. 9139; (b) judicial naturalization pursuant to C.A.
No. 473, as amended; and (c) legislative naturalization in the form of a law enacted by Congress bestowing Philippine
citizenship to an alien.45

Petitioner’s contention that the qualifications an applicant for naturalization should possess are those provided for in R.A.
No. 9139 and not those set forth in C.A. No. 473 is barren of merit. The qualifications and disqualifications of an applicant
for naturalization by judicial act are set forth in Sections 246 and 447 of C.A. No. 473. On the other hand, Sections 348 and
449 of R.A. No. 9139 provide for the qualifications and disqualifications of an applicant for naturalization by administrative
act.

Indeed, R.A. No. 9139 was enacted as a remedial measure intended to make the process of acquiring Philippine
citizenship less tedious, less technical and more encouraging. 50 It likewise addresses the concerns of degree holders who,
by reason of lack of citizenship requirement, cannot practice their profession, thus promoting "brain gain" for the
Philippines.51 These however, do not justify petitioner’s contention that the qualifications set forth in said law apply even to
applications for naturalization by judicial act.

First. C.A. No. 473 and R.A. No. 9139 are separate and distinct laws – the former covers all aliens regardless of class
while the latter covers native-born aliens who lived here in the Philippines all their lives, who never saw any other country
and all along thought that they were Filipinos; who have demonstrated love and loyalty to the Philippines and affinity to the
customs and traditions.52 To reiterate, the intention of the legislature in enacting R.A. No. 9139 was to make the process of
acquiring Philippine citizenship less tedious, less technical and more encouraging which is administrative rather than
judicial in nature. Thus, although the legislature believes that there is a need to liberalize the naturalization law of the
Philippines, there is nothing from which it can be inferred that C.A. No. 473 was intended to be amended or repealed by
R.A. No. 9139. What the legislature had in mind was merely to prescribe another mode of acquiring Philippine citizenship
which may be availed of by native born aliens. The only implication is that, a native born alien has the choice to apply for
judicial or administrative naturalization, subject to the prescribed qualifications and disqualifications.

In the instant case, petitioner applied for naturalization by judicial act, though at the time of the filing of his petition,
administrative naturalization under R.A. No. 9139 was already available. Consequently, his application should be
governed by C.A. No. 473.

Second. If the qualifications prescribed in R.A. No. 9139 would be made applicable even to judicial naturalization, the
coverage of the law would be broadened since it would then apply even to aliens who are not native born. It must be
stressed that R.A. No. 9139 applies only to aliens who were born in the Philippines and have been residing here.

Third. Applying the provisions of R.A. No. 9139 to judicial naturalization is contrary to the intention of the legislature to
liberalize the naturalization procedure in the country. One of the qualifications set forth in R.A. No. 9139 is that the
applicant was born in the Philippines and should have been residing herein since birth. Thus, one who was born here but
left the country, though resided for more than ten (10) years from the filing of the application is also disqualified. On the
other hand, if we maintain the distinct qualifications under each of the two laws, an alien who is not qualified under R.A.
No. 9139 may still be naturalized under C.A. No. 473.

Thus, absent a specific provision expressly amending C.A. No. 473, the law stands and the qualifications and
disqualifications set forth therein are maintained.

In any event, petitioner failed to prove that the witnesses he presented were competent to vouch for his good moral
character, and are themselves possessed of good moral character. It must be stressed that character witnesses in
naturalization proceedings stand as insurers of the applicant’s conduct and character. Thus, they ought to testify on
specific facts and events justifying the inference that the applicant possesses all the qualifications and none of the
disqualifications provided by law.53

Petitioner’s witnesses, Atty. Adasa and Salcedo, did not testify on his specific acts; they did not elaborate on his traits.
Their testimonies do not convince the Court that they personally know petitioner well and are therefore in a position to
vouch for his qualifications. As correctly found by the CA, the witnesses’ testimonies consisted mainly of general
statements in answer to the leading questions propounded by his counsel. What they conveniently did was to enumerate
the qualifications as set forth in the law without giving specific details. The pertinent portion of Atty. Adasa’s testimony
follows:

q Do you know the petitioner Edison So?

a Yes, Sir.
q Will you please tell us how did you come to know him?

a Well I came to know him[,] the petitioner[,] when I was the legal consultant and adviser of their family business and I
used to ah (sic) me[e]t him during my visit to their place way back in 1991 to 1992.

q From that day of 1991 up to the present, is your relationship with the petitioner more or less contin[u]ous?

a Yes, sir, because aside from the usual professional visit that I did to their family some social function was sponsored
normally and I am (sic) invited and I used to attend.

q During the birthday party of the petitioner, did you usually attend petitioner’s birthday?

a On several occasions I attend the birthday.

q Will you please tell us where the petitioner resides at present?

a At present the petitioner resides at No. 528 Lavezares Street, Binondo, Manila.

q Do you know for how long the petitioner resides in the Philippines?

a As far as I personally known (sic) Your Honor is that since birth.

q During all the times that you have know[n] the petitioner, what is your impression of his conduct?

a Well ah (sic) I have personally known him to be obedient and hard working individual and ah (sic) he has a good moral
character and he has been ah (sic) no adverse report concerning the character of the petitioner.

q In your opinion does the petitioner has the qualifications necessary to become [a] citizen of the Philippines?

a Yes.

q Can you tell us why do you say so?

a I would say Your Honor that petitioner has posses (sic) all the qualifications mandated by law and presently he is more
than 21 years old and he has resided in the Philippines particularly in the City of Manila contin[u]ously for more than ten
(10) years and that since his birth; and that he has good moral character and I have observed that ah (sic) he has been
practicing Philippine traditions and ah (sic) those embodied in the Philippine constitution and he has been socially active
and meddle (sic) some of his neighbors and ah (sic) I am sure he has desire to embrace and learn the customs and ideas
and traditions in the Philippine[s] and as I earlier mentioned that he conducted himself in proper and approachable (sic)
manner during his entire residence in our country and he has a gainful occupation.

q Will you please tell us what are these customs which the petitioner embraced?

a Well I have observed that ah (sic) together with his family they used to ah observed (sic) the usual Filipino celebration
during Christmas and new year and some occasions such as fiestas.

q And do you know whether petitioner is not disqualified under Commonwealth Act to become Filipino citizen of the
Philippines (sic)?

a Ah there has been no incident or occasion which I learned that would disqualify of coming (sic) the citizen of the
Republic of the Philippines. I have noticed that ah (sic) he is qualified under Commonwealth Act 473 as amended because
he is not opposed to ah (sic) organized government. His family and himself does not believed (sic) in the use of force in
the success of his ideas and ah (sic) he is not a poligamist (sic) or believer in the practice of illegal and he has not been
convicted in any crime involving him in any crime (sic). and he is not suffering from any mental alienation or any incurable
contidious (sic) disease. as provided for.

q Will you please tell us why you know all these stage?

a Because of ah (sic) the personal attachment with his family we have continuously having ah (sic) the usual contact with
his family.54

It can thus be inferred that Atty. Adasa is close to petitioner’s family, but not specifically to petitioner. Atty. Adasa’s
statements refer to his observations on the family’s practices and not to petitioner in particular. Nothing in his testimony
suggests that he was close to petitioner and knew him well enough to vouch for his qualifications.

Salcedo, on the other hand, testified thus:

q Now do you know the petitioner in this case Edison So?


a Yes, Sir.

q Are you personally acquainted with him?

a Yes, Sir.

q How long have you known the petitioner?

a I have known him for about ten (10) years, Sir.

q Will you please inform the Honorable court under what circumstances did you come to know the petitioner?

a I met him in a birthday party in 1991, Sir.

q And from 1991 up to the present is your relationship with the petitioner more or less contin[u]ous?

a Yes, Sir.

q How often did you see the petitioner?

a I see him twice a week, Sir.

q And during this time that you met the petitioner, what did you usually do?

a We play some games, Sir. We play Patentero (sic).

q Do you go to church together?

a Yes, Sir.

q During fiestas in your place, did the petitioner go?

a Yes, Sir.

q How about during fiestas in the place where the petitioner reside[s], did you also go during fiestas?

a Yes, Sir.

q During occasion in the house of the petitioner, are you invited?

a Yes, Sir.

q How many time[s] did you go to his (sic) residence of the petitioner?

a Twice a week, sir.

q Will you please tell us where the petitioner resides?

a The petitioner resides at 528 Lavezares Street, Tondo, Manila, Sir.

q For how long does the petitioner reside in that address?

a Since birth, Sir.

q During all the times that you have known the petitioner, will you please tell us your impression of his conduct?

a He is a person of good moral, sir, and he believed in the principles of the Philippines (sic) Constitution.

q Will you please cite one or two of these principles underlined the principles (sic) of the Philippines (sic) Constitution?

a Ah the Philippines is a Republican of the (sic) state, sovereignty preside (sic) over the people and the government
authority emanate from within; and the other one is the civilian government is not supreme over the military.

q Now in your opinion does the petitioner have all the qualifications necessary to become a citizen of the Philippines?

a Yes, Sir.

q What are these qualifications?


a He is at least 21 years old, he is a person of good moral and has been residing in the Philippines since birth.

q What else?

a He must be a Filipino and ah must practice the traditions and customs, Sir.

q Do you know whether the petitioner conducted himself in a proper and appraochable (sic) manner during the period of
his residence in the Philippines?

a Yes, Sir.

q Do you know if the petitioner has a gainful occupation?

a Yes, Sir.

q What is the occupation of the petitioner?

a Ah (sic) he is the secretary in a wood factory in Commonwealth, Sir.

q And aside from being the secretary, what else did the petitioner do?

a He help (sic) in the factory cargo, Sir.

q Is the petitioner still a student?

a Yes, Sir.

q Where is he studying?

a In UST, Sir.

q Is he your classmate?

a Yes, Sir.

q What was his course?

a Pharmacy, Sir.

q So when you said he was the secretary he only works as part time secretary?

a Yes, Sir.

q You said the petitioner meddle (sic) socially with the Filipinos?

a Yes, Sir.

q Will you please name at least one of those Filipinos the petitioner meddle (sic) with?

a Samuel Falmera, Sir, Marlon Kahocom, Sir.

q Who else?

a Elmer Ramos, Sir.

q Who else?

a Sharmaine Santos, Sir.

q You said the petitioner is of good moral character?

a Yes, Sir.

q Why do you know that?

a As a classmate I can see him I go with him and ah (sic) I can see that he has ah better approached (sic) with other
people and I can see that he mixed very well with friends.
q So during school days you see him everyday?

a Yes, Sir.

q When there are no classes during the vacation you see the petitioner twice a week?

a Yes, Sir.

q Does the petitioner (sic), do you think the petitioner is not disqualified to become the citizen of the Republic of the
Philippines?

a Yes, Sir, he is not disqualified, Sir.

q Why do you say that he is not disqualified?

a Because he abide [by] any law in the government, sir, ah (sic) he is not polygamus and he is not convicted of any crime,
Sir.

q Do you know ever the petitioner oppose to any organized government?

a No, Sir.

q Do you know whether he believe[s] in the use of force in any such ideas?

a No, Sir.

q Do you know if the petitioner is a believer in the practice of polygamy?

a No, Sir.

q Do you know whether the petitioner suffer[s] from mental alienation or incurable disease illnesses?

a No, Sir.

q Why do you know?

a I know him personally, sir, I have been with him as my classmate, sir and ah (sic) he is a very intelligent person, Sir.

q Is the petitioner a member also of any organization or association in your school?

a Yes, Sir.

q What organization?

a He is a member of Wishten and a member of starget, Sir.

q What does starget means?

a Starget is an organization of Chinese community in UST, Sir.

q How about the other one which you mentioned?

a Ah (sic) these are twisting, sir he represents the ah the (sic) school intercollegiate, Sir. 55

Again, Salcedo did not give specific details on petitioner’s qualifications.

In sum, petitioner’s witnesses clearly did not personally know him well enough; their testimonies do not satisfactorily
establish that petitioner has all the qualifications and none of the disqualifications prescribed by law.

In naturalization proceedings, it is the burden of the applicant to prove not only his own good moral character but also the
good moral character of his/her witnesses, who must be credible persons. 56 Within the purview of the naturalization law, a
"credible person" is not only an individual who has not been previously convicted of a crime; who is not a police character
and has no police record; who has not perjured in the past; or whose affidavit or testimony is not incredible. What must be
credible is not the declaration made but the person making it. This implies that such person must have a good standing in
the community; that he is known to be honest and upright; that he is reputed to be trustworthy and reliable; and that his
word may be taken on its face value, as a good warranty of the applicant’s worthiness. 57

The records likewise do not show that the character witnesses of petitioner are persons of good standing in the
community; that they are honest and upright, or reputed to be trustworthy and reliable. The most that was established was
the educational attainment of the witnesses; however, this cannot be equated with their credibility. In fine, petitioner
focused on presenting evidence tending to build his own good moral character and neglected to establish the credibility
and good moral character of his witnesses.58

We do not agree with petitioner’s argument that respondent is precluded from questioning the RTC decision because of its
failure to oppose the petition. A naturalization proceeding is not a judicial adversary proceeding, and the decision rendered
therein does not constitute res judicata. A certificate of naturalization may be cancelled if it is subsequently discovered that
the applicant obtained it by misleading the court upon any material fact. Law and jurisprudence even authorize the
cancellation of a certificate of naturalization upon grounds or conditions arising subsequent to the granting of the
certificate.59 If the government can challenge a final grant of citizenship, with more reason can it appeal the decision of the
RTC within the reglementary period despite its failure to oppose the petition before the lower court.

Thus, petitioner failed to show full and complete compliance with the requirements of naturalization law. For this reason,
we affirm the decision of the CA denying the petition for naturalization without prejudice.

It must be stressed that admission to citizenship is one of the highest privileges that the Republic of the Philippines can
confer upon an alien. It is a privilege that should not be conferred except upon persons fully qualified for it, and upon strict
compliance with the law.60

IN LIGHT OF ALL THE FOREGOING, the petition is DENIED for lack of merit.

SO ORDERED.

- Republic v Go Pei Hung, GR212785, Apr 4, 2018

A Petition for Naturalization must be denied when full and complete compliance with the requirements of
Commonwealth Act. No. 473 (CA 473), or the Revised Naturalization Law, is not shown.

This Petition for Review on Certiorari 1 seeks to set aside (1) the February 28, 2014 Decision 2 of the Court
of Appeals (CA) in CA-G.R. CV No. 97542 affirming the July 21, 2010 Decision 3 of the Regional Trial Court
(RTC) of Manila City, Branch 16 in Naturalization Case No. 07-118391, as well as (2) the CA's June 5,
2014 Resolution4 denying petitioner's Motion for Reconsideration.

Factual Antecedents

On December 3, 2007, respondent Go Pei Hung - a British subject and Hong Kong resident - filed a
Petition for Naturalization5 seeking Philippine citizenship. The case was lodged before the RTC of Manila,
Branch 16 and docketed as Naturalization Case No. 07-118391.

After trial, the RTC issued its July 21, 2010 Decision granting the respondent's petition for naturalization.
The RTC declared, thus:

The issue to be resolve [sic] here is whether or not the petitioner deserves to become a Filipino citizen.

In Commonwealth Act No. 473, approved June 17, 1939, provided [sic] that persons having certain
specified qualifications may become a citizen [sic] of the Philippines by naturalization.

Section 2. Qualifications. - Subject to Section 4 of this Act, any person having the following qualifications
may become a citizen of the Philippines by naturalization:
First. He must be not less than twenty-one years of age on the day of the hearing of the petition;

Second.  He must have resided in the Philippines for a continuous period of not less than ten years;

Third. He must be of good moral character and believes in the principles underlying the Philippine
Constitution, and must have conducted himself in a proper and irreproachable manner during the entire
period of his residence in the Philippines in his relation wife the constituted government as well as with the
community in which he is living.

Fourth. He must own real estate in the Philippines worth not less than five thousand pesos, Philippine
currency, or must have some known lucrative trade, profession, or lawful occupation:

Fifth. He must be able to speak and write English or Spanish and any one of the principal Philippine
languages; and

Sixth. He must have enrolled his minor children of school age, in any of the public schools or private
schools recognized by the Office of Private Education of the Philippines, where the Philippine history,
government and civics are taught or prescribed as part of the school curriculum, during the entire period
of the residence in the Philippines required of him prior to the hearing of his petition for naturalization as
Philippine citizen.
The Court, upon reviewing the records of this case, the pieces of documentary evidence and the
testimonies of the petitioner and his two (2) character witnesses, x x x finds that petitioner Go Pei Hung,
has complied with all the qualifications stated in Section 2 of Commonwealth Act 473.

It appeared that there is no impediment to the Court's nod of approval to petitioner's supplication[, H]e
had presented at least two (2) credible persons, stating that they are citizens of the Philippines and
personally know the petitioner to be a resident of the Philippines for the period of time required (Section 7
of CA 473).

As held in Lim versus Republic 17 SCRA 424, 427, (1996[)] citing Vy Tain vs. Republic, L-19918, July 30,
1965.

'As construed by case law, they must have personal knowledge of the petitioner's conduct during the
entire period of his residence in the Philippines.'

Also in [the] case of Edison So vs. Republic, G.R. No. 170603, January 29, 2007 and Republic vs.
Hong, G.R. No. 168877, March 24, 2006[:]

"In naturalization proceedings, the applicant has the onus  to prove not only his own good moral character
but also the good moral character of his/her witnesses, who must, be credible persons."

Both witnesses presented by petitioner made common declarations that they came to know him [in] 1995
and became good friends with petitioner. Verily, given the birth of petitioner in 1961, the testimony of his
two (2) witnesses, Mr. La To Sy Lai and So An Ui Henry Co Sy, that they came to know the petitioner
sometime in 1995, [revealed] x x x that they had personal cognition of petitioner's demeanor during the
petitioner's residence in the Philippines. Certainly, they see and observe the applicant continuously, every
day and every week in order to be competent to testify on his reputation and conduct.

WHEREFORE, premises considered, the Petition, for Naturalization filed by petitioner Go Pei Hung is hereby
GRANTED.

Let [a] copy of this Decision be sent to the following concerned government agencies:

1. Bureau of Immigration
2. Department of Foreign Affairs
3. Office of the Solicitor General
4. National Bureau of Investigation

Under Republic Act 530, this decision granting the application for naturalization shall not become final and
executory until after two (2) years from the promulgation of the decision and after another hearing is
conducted to determine whether or not the applicant has complied with the requirements of Section 1 of
said law with the attendance of the Solicitor General or his authorized representative x x x, and so finds
[that] during the intervening time the applicant:
(1) [has] not left the Philippines;
(2) has dedicated himself continuously to a lawful calling or profession;
(3) has not been convicted of any offense or violation of Government promulgated rate; and
(4) or committed any act prejudicial to the interest of the nation or contrary to any Government
announced policies.

Set hearing on August 30, 2012 at 8:30 o'clock in the morning.


SO ORDERED.6 (Emphasis in the original; citations omitted)

Ruling of the Court of Appeals

Petitioner interposed an appeal with the CA, which was docketed as CA-G.R. CV No. 97542. On February
28, 2014, the CA issued the assailed Decision, pronouncing thus:

x x x [T]he Republic of the Philippines, through the OSG, filed the present appeal, alleging that:
'I.
THE TRIAL COURT ERRED IN GRANTING THE PETITION DESPITE PETITIONER-APPELLEE'S FAILURE TO
FILE A DECLARATION OF INTENTION, AS REQUIRED BY SECTION 5 OF COMMONWEALTH ACT (C.A.) NO.
473;

II.
THE TRIAL COURT ERRED IN GRANTING THE PETITION DESPITE PETITIONER-APPELLEE'S FAILURE TO
ATTACH A CERTIFICATE OF HIS ARRIVAL IN THE PHILIPPINES, AS MANDATED BY SECTION 7 OF
COMMONWEALTH ACT X X X NO. 473:

III.
THE TRIAL COURT ERRED IN GRANTING THE PETITION DESPITE PETITIONER-APPELLEE'S FAILURE TO
SHOW BY CLEAR AND CONVINCING EVIDENCE THAT HE HAS A LUCRATIVE TRADE, PROFESSION OR
OCCUPATION, AS REQUIRED BY PARAGRAPH 4, SECTION 2 OF CA. NO. 473; and

IV.
THE TRIAL COURT ERRED IN GRANTING THE PETITION DESPITE PETITIONER-APPELLEE'S FAILURE TO
PRESENT DURING THE HEARING OF THE PRESENT  CASE AT LEAST TWO CREDIBLE PERSONS AS
PROVIDED BY SECTION 7 OF CA. NO. 473.'

Petitioner-appellee opposes the appeal and claims that he has all the qualifications and none of the
disqualifications to be a naturalized Philippine citizen.

The sole issue in this appeal is whether x x x the court a quo committed a reversible error in granting the
petition for naturalization.

After [a] careful consideration of the arguments and the evidence on record, this Court rules to dismiss the
appeal.

Anent the first assigned error, the Republic claims that the petitioner failed to file with the OSG a
Declaration of Intention as required under Section 5 of Commonwealth Act (CA) No. 473, as amended,
which provides that:

'Sec. 5. Declaration of Mention. - One year prior to the filing of his petition for admission to
Philippine citizenship, the applicant for Philippine citizenship shall file with the Bureau of
Justice, a declaration under oath that it is bona fide his intention to become a citizen of the
Philippines. x x x'

As the foregoing Section 5 of CA No. 473, as amended, provides, the declaration shall be filed with the
Bureau of Justice, now the OSG, at least one year before the filing of the petition, and shall set forth the
following:
(a) name, age, occupation, personal description, place of birth, last foreign residence and allegiance, the
date of arrival, the name of the vessel or aircraft in which he came to the Philippines, and the place of
residence in the Philippines at the time of making the declaration;

(b) a certificate showing the date, place and manner of his arrival;

(c) a statement that he has enrolled his minor children, if any, in any of the public schools or private
schools recognized by the Office of Private Education of the Philippines, now the Department of Education,
where Philippine history, government, and civics are taught or prescribed as part of the school curriculum,
during the entire period of the residence in the Philippines required of him prior to the hearing of his
petition for naturalization as Philippine citizen; and

(d) two photographs of himself.


Petitioner-appellee does not deny that he failed to file with the OSG the required declaration of intention,
but he claims that he is exempted from filing the same pursuant to Section 6 of CA 473, as amended,
which provides that:
'Sec. 6. Persons exempt from requirement to wake a declaration of intention. - Persons born in the
Philippines and have received their primary and secondary education in public schools or those recognized
by the Government and not limited to any race or nationality, and those who have resided
continuously in the Philippines for a period of thirty years or more before filing their
application, may be naturalized without having to make a declaration of intention upon
complying with the other requirements of this Act. To such requirements shall be added that which
establishes that the applicant has given primary and secondary education to all his children in the public
schools or in private schools recognized by the Government and not limited to any race or nationality. The
same shall be understood to be applicable with respect to the widow and minor children of an alien who
has declared his intention to become a citizen of the Philippines, and dies before he is actually naturalized.'

According to petitioner-appellee, he has been continuously residing in the Philippines since 1973, during
which he resided at 2277-B Luna Street, Pasay City. Also, he studied [at the] Philippine Pasay Chinese
School in 1974 and later graduated [from] Grade VI in 1976. Thus, petitioner-appellee claims that,
counted from 1973 to 2007 when he filed the petition for naturalization, he [had] been continuously
residing in the Philippines for a period of thirty-four (34) years.

As to why petitioner-appellee stated in his petition that he continuously resided in the Philippines starting
in 1989 only, he explained that it was [in] that year that he was officially issued a Certificate of Permanent
Residence by the Bureau of Immigration. But, to be entitled to that status, he had to [have] resided in the
Philippines for a longer period of time.

This Court is convinced that petitioner-appellee has been residing in the Philippines earlier than 1989. As
narrated in the petition, he commenced his residence in the Philippines in 1973 at 2277-B Luna Street
Pasay City. A year later, he enrolled at the Philippine Pasay Chinese School, where he later graduated
[from] Grade VI in 1976. That he had been living in the Philippines in 1973 was also established by
petioner-appellee during his direct examination, thus:

x x x x

It bears stressing that this testimony was not contradicted or refuted by the Republic which was
represented by the City Prosecutor of Manila.

Thus, counted from 1973 to 2007 when he filed the petition for naturalization, petitioner-appellee had
been continuously residing in the Philippines for more than thirty (30) years, or a period of thirty-four (34)
years to be exact. Pursuant to Section 6 of CA 473, as amended, petitioner-appellee is exempted from
filing the aforesaid declaration of intention.

Relatedly, considering that petitioner-appellee is exempted from filing the declaration of intention,
petitioner-appellee is also exempted from filing the certificate of arrival which is, after all, just a
component of the declaration of intention as provided under Section 5 of CA No. 473, as amended.

It is also not amiss to mention that all the information needed to be stated in the declaration of intention
were stated also in the petition for naturalization and were proven during the presentation of evidence. So,
while petitioner-appellee is exempted from filing the said declaration, he, nevertheless, provided and
proved the facts needed to support his petition for naturalization.

As for the third, assigned error, the Republic claims that the petitioner-appellee does not have a lucrative
trade, profession or occupation within the meaning of the Naturalization Law, and that while petitioner-
appellee alleged in his petition that he derived an annual income of $165,000.00 as a businessman, he
failed to present any evidence to support his supposed business.

The Court is not persuaded.

According to Section 1 of CA No. 473, as amended, one of the qualifications of a person applying to be a
naturalized Philippine citizen is that he must either own real estate in the Philippines worth not less than
five thousand pesos, Philippine currency, or have some known lucrative trade, profession, or lawful
occupation. Petitioner-appellee sought to establish that he is a businessman, [from] which he derives an
average annual income of P165,000.00 During the trial, he marked and offered in evidence his Annual
Income Tax Returns for the years 2007, 2008 and 2009. He also testified that he was helping in the
business, which was put up by his wife, called the Excel Parts Sales Center, located at 1161 R. Hidalgo
Street, Quiapo, Manila. This was affirmed by petitioner-appellee's witness, Lato Sy Lai, who told the court
that petitioner-appellee's business is the sale of automobile parts.

Thus, contrary to the claim of the Republic, petitioner-appellee was able to prove that he has a lucrative
trade, profession or occupation, which is the sale of automobile parts, one which has not been rebutted by
the Republic nor has been shown to be illegal, immoral or against public policy.

As for the fourth and last assigned error, the Republic claims that the petitioner-appellee failed to present
credible persons as character witnesses, and that the two persons who testified for the petitioner-appellee
resorted to mere generalizations.

Again, the Court is not persuaded.

Petitioner-appellee, presented two character witnesses: Lato Sy Lai and So An Ui Henry Sy. Both witnesses
testified in court and were cross-examined by the City Prosecutor of Manila on such matters as how they
met petitioner-appellee, how the petitioner-appellee related to Filipinos and how petitioner-appellee has
adapted to Filipino culture, customs and traditions. We have reviewed the testimonies of these, witnesses
and we find no error on the part of the trial court when it found these witnesses credible. As held in  People
vs. dela Cruz, the matter of evaluating the credibility of witnesses depends largely on the assessment of
the trial court, and appellate courts rely heavily on the weight given by the trial court on the credibility of a
witness as it had a first-hand opportunity to hear and see the witness testify.

It must be stressed again, that despite its opportunity to do so, the Republic failed to present any evidence
or witness, to oppose the testimonial evidence presented by the petitioner-appellee.

In fine, the Republic has failed to show that the court a quo committed reversible error in granting
petitioner-appellee's petition for naturalization.
WHEREFORE, the instant appeal is DISMISSED  and the Decision dated July 21, 2010 of the Regional
Trial Court of Manila, Branch 16, in Naturalization Case No. 07-118391 is AFFIRMED.

SO ORDERED.7 (Emphasis in the original: citations omitted)

Petitioner moved for reconsideration, but in its June 5, 2014 Resolution, the appellate court held its
ground.

Issues

In the present Petition, it is argued that —

The petition for naturalization should not [have been] granted because: i) respondent did not
file his declaration of intention with the OSG; ii) respondent did not state the details of his
arrival in the Philippines in his petition and the certificate of arrival was not attached to the
petition.; iii) respondent is not engaged in a lucrative profession, trade or occupation; and iv)
respondent failed to present during hearing qualified character witnesses as required under CA
No. 473.8 (Emphasis in the original)

Petitioner's Arguments

In its Petition and Reply9 seeking reversal of the CA dispositions and denial of respondent's Petition for
Naturalization in Naturalization Case No. 07-118391, petitioner contends that naturalization should be
denied due to the failure of respondent to attach a Declaration of Intention and Certificate of Arrival to his
Petition for Naturalization, as required under CA No. 473; that contrary to the CA's pronouncement,
respondent is not exempt from filing the required Declaration of Intention as he was neither born in the
Philippines, nor had he resided therein for a period of 30 years or more, as the record showed that he was
born in Hong Kong and became a permanent Philippine resident only in 1989 - or for a period less than the
required 30-year residency counted from the filing of his Petition for Naturalization in 2007; that the
Certificate of Arrival - which is lacking - is equally important as it prevents aliens who have surreptitiously
entered the country without the proper document or certificate of entry from acquiring citizenship by
naturalization, and the absence of such document renders the Petition for Naturalization null and void; that
the Petition for Naturalization was not validly published in its entirety; that respondent was not. engaged
in a lucrative trade, profession or occupation as he only had an average annual, income of P165,000.00 in
2007 - when he filed the Petition for Naturalization - or a monthly income of only P13,750.00, which was
insufficient for the support of his wife and three minor children, much less for his sole sustenance: that the
two witnesses presented in respondent's favor were not credible character witnesses as they resorted to
mere generalizations in their testimonies and did not delve into specific details - and they did not actually
know respondent well since they both came to know him only in 1995.

Regarding procedural matters, petitioner argues that, while it did not attach the annexes to the instant
Petition to the copy sent to respondent, these documents were nonetheless known to the latter and he had
them in his possession all throughout these proceedings.

Respondent's Arguments

In his Comment,10 respondent argues that the instant Petition should be denied as it violated Section 4 of
Rule 45 of the Rules of Court11 as petitioner did not attach the annexes to the copy of its Petition sent to
respondent; besides the Petition is without merit. In particular, respondent argues that he is exempt from
filing a Declaration of Intention and submitting a Certificate of Arrival, as he has been a resident of the
Philippines for more than 30 years, having arrived in the country in 1973 and residing therein since; that
the petitioner's computation of respondent's residency from 1989 reckoned from the issuance of his
certificate of permanent residence, was incorrect; that the Certificate of Arrival is a mere "component part,
in the filing of the Declaration of Intention" 12 - which is thus no longer required since respondent is exempt
from filing the said Declaration of Intention; that the Petition for Naturalization was validly published in
accordance with the requirements of law; that respondent was engaged in a lucrative trade, as in fact
since January 2010, he was already earning a monthly income of P50,000.00 as a commission sales
executive; and that the witnesses for respondent gave credible testimonies on the latter's character and
behavior.

Our Ruling

The Court grants the Petition.


In Republic v. Huang Te Fu,13 a case decided by this ponente, the following pronouncement was made:

In Republic v. Hong, it was held in essence that an applicant for naturalization must show full and
complete compliance with the requirements of the naturalization law; otherwise, his petition for
naturalization will be denied. This ponente has likewise held that "[t]he courts must always be mindful that
naturalization proceedings are imbued with the highest public interest. Naturalization laws should he
rigidly enforced and strictly construed in favor of the government and against the applicant. The burden of
proof rests upon the applicant to show full and complete compliance with the requirements of
law."14 (Citations omitted)

Section 7 of the Revised Naturalization Law or CA 473 requires, among others, that an applicant for
naturalization must attach a Certificate of Arrival to the Petition for Naturalization:

Section 7. Petition for citizenship. – Any person desiring to acquire Philippine citizenship shall file with the
competent court, a petition in triplicate, accompanied by two photographs of the petitioner, setting forth
his name and surname; his present and former places of residence; his occupation; the place and date of
his birth; whether single or married and the father of children, the name, age, birthplace and residence of
the wife and of the children; the approximate date of his or her arrival in the Philippines, the name
of the port of debarkation, and, if he remembers it, the name of the ship on which he came;  a
declaration that he has the qualifications required by this Act, specifying the same, and that he is not
disqualified for naturalization under the provisions of this Act; that he has compiled with the requirements
of section five of this Act; and that he will reside continuously in the Philippines from the date of the filing
of the petition up to the time of his admission to Philippine citizenship. The petition must be signed by the
applicant in his own handwriting and be supported by the affidavit of at least two credible persons, stating
that they are citizens of the Philippines and personally know the petitioner to be a resident of the
Philippines for the period of time required by this Act and a person of good repute and morally
irreproachable, and that said petitioner has in their opinion all the qualifications necessary to become a
citizen of the Philippines and is not in any way disqualified under the provisions of this Act. The petition
shall also set forth the names and post-office addresses of such witnesses as the petitioner may desire to
introduce at the hearing of the case. The certificate of arrival, and the declaration of intention must
be made part of the petition. (Emphasis supplied)

Respondent came to the country sometime in 1973; thus, he should have attached a Certificate of Arrival
to his Petition for Naturalization. This is mandatory as respondent must prove that he entered the country
legally and not by unlawful means or any other manner that is not sanctioned by law. Because if he
entered the country illegally, this would render his stay in the country unwarranted from the start, and no
number of years' stay here will validate his unlawful entry. The spring cannot rise higher than its source,
so to speak.

In Republic v. Judge De la Rosa,15 this Court held that the failure to attach a copy of the applicant's
certificate of arrival to the petition as required by Section 7 of CA 473 is fatal to an applicant's petition for
naturalization. the ruling in said case proceeds from pronouncements in the past, to wit:

Finally, petitioner-appellant failed to attach in his petition a certificate of arrival as required by Sec. 7 of
Com. Act No. 473, as amended, which omission likewise nullifies his petition. The reason for the
requirement that the certificate of arrival should form part of the petition is to prevent aliens, who illegally
entered the Philippines, from acquiring citizenship by naturalization. If, as he pretends, his certificate was
taken back by the Bureau of Immigration and in lieu thereof he was issued an immigrant's certificate of
residence, he could have submitted the same or a certified true copy thereof. 16

Naturalization granted without the filing of a certificate of arrival as required by the statute, the same
being a matter of substance, is illegally procured. (U.S, vs. Ness, 62 L. Ed. 321). 17 (Citations omitted)

x x x Again in the above quoted Section 7 of the law, the certificate of arrival must be made a part of the
petition. This provision is mandatory and it has been enacted for the purpose of preventing aliens, who
have surreptitiously come into the islands without the proper document or certificate of entry, from
acquiring citizenship by naturalization, unless the said provision is complied with. This Court cannot grant
the petition as the said grant would be a clear violation of the express mandate of the law. 18

The Certificate of Arrival should prove that respondent's entry to the country is lawful. Without it, his
Petition for Naturalization is incomplete and must be denied outright.
Even if respondent acquired permanent resident status, this does not do away with the requirement of said
certificate of arrival. An application to become a naturalized Philippine citizen involves requirements
different and separate from that for permanent residency here.

Respondent likewise argues that the required certificate of arrival is a "mere component part in the filing
of the Declaration of Intention"19 and thus unnecessary since he is exempt from submitting the latter
document. This is not correct. The Declaration of Intention is entirely different from the Certificate of
Arrival; the latter is just as important because it proves that the applicant's entry to the country was not
illegal - that he was a documented alien whose arrival and presence in the country is in good faith and
with evident intention to submit to and abide by the laws of the Republic. Certainly, an illegal and
surreptitious entry into the country by aliens whose undocumented arrival constitutes a threat to national
security and the safety of its citizens may not be rewarded later on with citizenship by naturalization or
otherwise; to repeat, a spring will not rise higher than its source.

On the issue of petitioner's alleged failure to attach the required annexes to the copy of the instant Petition
that was sent to respondent, this is rendered insignificant and moot by the fact that respondent's
application for naturalization - which is patently defective for failure to attach the required certificate of
arrival - involves the national interest, as well as the security and safety of the country and its citizens.
Any procedural infirmities in this case are superseded by the national interest. "[T]echnicalities take a
backseat against substantive rights, and not the other way around." 20

To repeat, strict compliance with all statutory requirements is necessary before an applicant may acquire
Philippine citizenship by naturalization. The absence of even a single requirement is fatal to an application
for naturalization.

In naturalization proceedings, the burden of proof is upon the applicant to show full and complete
compliance with the requirements of the law. The opportunity of a foreigner to become a citizen by
naturalization is a mere matter of grace, favor or privilege extended to him by the State; the applicant
does not possess any natural, inherent, existing or vested right to be admitted to Philippine citizenship.
The only right that a foreigner has, to be given the chance to become a Filipino citizen, is that which the
statute confers upon him; and to acquire such right, he must strictly comply with ail the statutory
conditions and requirements. The absence of one jurisdictional requirement is fatal to the petition as this
necessarily results in the dismissal or severance of the naturalization process.

Hence, all other issues need not be discussed further as respondent failed to strictly follow the requirement
mandated by the statute.

It should be emphasized that 'a naturalization proceeding is so infused with public interest that it has been
differently categorized and given special treatment, x x x Unlike in ordinary judicial contest, the granting
of a petition for naturalization does not preclude the reopening of that case and giving the government
another opportunity to present new evidence. A decision or order granting citizenship will not even
constitute res judicata to any matter or reason supporting a subsequent judgment cancelling the
certification of naturalization already granted, on the ground that it had been illegally or fraudulently
procured. For the same reason, issues even if not raised in the lower court may be entertained on appeal.
As the matters brought to the attention of this Court x x x involve facts contained in the disputed decision
of the lower court and admitted by the parties in their pleadings, the present proceeding may be
considered adequate for the purpose of determining the correctness or incorrectness of said decision, in
the light of the law and extant jurisprudence.'

Ultimately, respondent failed to prove full and complete compliance with the requirements of the
Naturalization Law. As such, his petition for naturalization must be denied without prejudice to his right to
re-file his application.21

Having disposed of the case in the foregoing manner, this Court finds no need to resolve the other issues
raised by the parties. With the finding that respondent's Petition for Naturalization did not include the
Certificate of Arrival as required by CA 473, as amended, the said Petition should have been dismissed
outright on that, sole ground.

WHEREFORE, the Petition is GRANTED. The February 28, 2014 Decision and June 5, 2014 Resolution of
the Court of Appeals in CA-G.R. CV No. 97542 are REVERSED AND SET ASIDE. The respondent's Petition
for Naturalization in Naturalization Case No. 07-118391 before the Regional Trial Court of Manila City,
Branch 16 is DISMISSED.

SO ORDERED.

Effect of acquisition of foreign citizenship before RA 9225


Effect of acquisition of foreign citizenship under RA 9225
- Maquiling v Comelec, 696 SCRA 420 (2013) & 700 SCRA 367 (2013)

THE CASE

This is a Petition for Certiorari ender Rule 64 in conjunction with Rule 65 of the Rules of Court to review the Resolutions of
the Commission on Elections (COMELEC). The Resolution 1 in SPA No. 10-1 09(DC) of the COMELEC First Division dated
5 October 201 0 is being assailed for applying Section 44 of the Local Government Code while the Resolution 2 of the
COMELEC En Banc dated 2 February 2011 is being questioned for finding that respondent Rommel Arnado y Cagoco
(respondent Arnado/Arnado) is solely a Filipino citizen qualified to run for public office despite his continued use of a U.S.
passport.

FACTS

Respondent Arnado is a natural born Filipino citizen. 3 However, as a consequence of his subsequent naturalization as a
citizen of the United States of America, he lost his Filipino citizenship. Arnado applied for repatriation under Republic Act
(R.A.) No. 9225 before the Consulate General of the Philippines in San Franciso, USA and took the Oath of Allegiance to
the Republic of the Philippines on 10 July 2008. 4 On the same day an Order of Approval of his Citizenship Retention and
Re-acquisition was issued in his favor.5

The aforementioned Oath of Allegiance states:

I, Rommel Cagoco Arnado, solemnly swear that I will support and defend the Constitution of the Republic of the
Philippines and obey the laws and legal orders promulgated by the duly constituted authorities of the Philippines and I
hereby declare that I recognize and accept the supreme authority of the Philippines and will maintain true faith and
allegiance thereto; and that I impose this obligation upon myself voluntarily without mental reservation or purpose of
evasion.6

On 3 April 2009 Arnado again took his Oath of Allegiance to the Republic and executed an Affidavit of Renunciation of his
foreign citizenship, which states:

I, Rommel Cagoco Arnado, do solemnly swear that I absolutely and perpetually renounce all allegiance and fidelity to the
UNITED STATES OF AMERICA of which I am a citizen, and I divest myself of full employment of all civil and political
rights and privileges of the United States of America.

I solemnly swear that all the foregoing statement is true and correct to the best of my knowledge and belief. 7

On 30 November 2009, Arnado filed his Certificate of Candidacy for Mayor of Kauswagan, Lanao del Norte, which
contains, among others, the following statements:

I am a natural born Filipino citizen / naturalized Filipino citizen.

I am not a permanent resident of, or immigrant to, a foreign country.

I am eligible for the office I seek to be elected to.

I will support and defend the Constitution of the Republic of the Philippines and will maintain true faith and allegiance
thereto. I will obey the laws, legal orders and decrees promulgated by the duly constituted authorities.

I impose this obligation upon myself voluntarily without mental reservation or purpose of evasion. 8

On 28 April 2010, respondent Linog C. Balua (Balua), another mayoralty candidate, filed a petition to disqualify Arnado
and/or to cancel his certificate of candidacy for municipal mayor of Kauswagan, Lanao del Norte in connection with the 10
May 2010 local and national elections.9

Respondent Balua contended that Arnado is not a resident of Kauswagan, Lanao del Norte and that he is a foreigner,
attaching thereto a certification issued by the Bureau of Immigration dated 23 April 2010 indicating the nationality of
Arnado as "USA-American."10To further bolster his claim of Arnado’s US citizenship, Balua presented in his Memorandum
a computer-generated travel record11 dated 03 December 2009 indicating that Arnado has been using his US Passport No.
057782700 in entering and departing the Philippines. The said record shows that Arnado left the country on 14 April 2009
and returned on 25 June 2009, and again departed on 29 July 2009, arriving back in the Philippines on 24 November
2009.

Balua likewise presented a certification from the Bureau of Immigration dated 23 April 2010, certifying that the name
"Arnado, Rommel Cagoco" appears in the available Computer Database/Passenger manifest/IBM listing on file as of 21
April 2010, with the following pertinent travel records:

DATE OF Arrival : 01/12/2010

NATIONALITY : USA-AMERICAN
PASSPORT : 057782700

DATE OF Arrival : 03/23/2010

NATIONALITY : USA-AMERICAN

PASSPORT : 05778270012

On 30 April 2010, the COMELEC (First Division) issued an Order 13 requiring the respondent to personally file his answer
and memorandum within three (3) days from receipt thereof.

After Arnado failed to answer the petition, Balua moved to declare him in default and to present evidence ex-parte.

Neither motion was acted upon, having been overtaken by the 2010 elections where Arnado garnered the highest number
of votes and was subsequently proclaimed as the winning candidate for Mayor of Kauswagan, Lanao del Norte.

It was only after his proclamation that Arnado filed his verified answer, submitting the following documents as evidence: 14

1. Affidavit of Renunciation and Oath of Allegiance to the Republic of the Philippines dated 03 April 2009;

2. Joint-Affidavit dated 31 May 2010 of Engr. Virgil Seno, Virginia Branzuela, Leoncio Daligdig, and Jessy Corpin,
all neighbors of Arnado, attesting that Arnado is a long-time resident of Kauswagan and that he has been
conspicuously and continuously residing in his family’s ancestral house in Kauswagan;

3. Certification from the Punong Barangay of Poblacion, Kauswagan, Lanao del Norte dated 03 June 2010 stating
that Arnado is a bona fide resident of his barangay and that Arnado went to the United States in 1985 to work and
returned to the Philippines in 2009;

4. Certification dated 31 May 2010 from the Municipal Local Government Operations Office of Kauswagan stating
that Dr. Maximo P. Arnado, Sr. served as Mayor of Kauswagan, from January 1964 to June 1974 and from 15
February 1979 to 15 April 1986; and

5. Voter Certification issued by the Election Officer of Kauswagan certifying that Arnado has been a registered
voter of Kauswagan since 03 April 2009.

THE RULING OF THE COMELEC FIRST DIVISION

Instead of treating the Petition as an action for the cancellation of a certificate of candidacy based on
misrepresentation,15 the COMELEC First Division considered it as one for disqualification. Balua’s contention that Arnado
is a resident of the United States was dismissed upon the finding that "Balua failed to present any evidence to support his
contention,"16 whereas the First Division still could "not conclude that Arnado failed to meet the one-year residency
requirement under the Local Government Code." 17

In the matter of the issue of citizenship, however, the First Division disagreed with Arnado’s claim that he is a Filipino
citizen.18

We find that although Arnado appears to have substantially complied with the requirements of R.A. No. 9225, Arnado’s act
of consistently using his US passport after renouncing his US citizenship on 03 April 2009 effectively negated his Affidavit
of Renunciation.

xxxx

Arnado’s continued use of his US passport is a strong indication that Arnado had no real intention to renounce his US
citizenship and that he only executed an Affidavit of Renunciation to enable him to run for office. We cannot turn a blind
eye to the glaring inconsistency between Arnado’s unexplained use of a US passport six times and his claim that he re-
acquired his Philippine citizenship and renounced his US citizenship. As noted by the Supreme Court in the Yu case, "a
passport is defined as an official document of identity and nationality issued to a person intending to travel or sojourn in
foreign countries." Surely, one who truly divested himself of US citizenship would not continue to avail of privileges
reserved solely for US nationals.19

The dispositive portion of the Resolution rendered by the COMELEC

First Division reads:

WHEREFORE, in view of the foregoing, the petition for disqualification and/or to cancel the certificate of candidacy of
Rommel C. Arnado is hereby GRANTED. Rommel C. Arnado’s proclamation as the winning candidate for Municipal Mayor
of Kauswagan, Lanao del Nore is hereby ANNULLED. Let the order of succession under Section 44 of the Local
Government Code of 1991 take effect.20
The Motion for Reconsideration and
the Motion for Intervention

Arnado sought reconsideration of the resolution before the COMELEC En Banc on the ground that "the evidence is
insufficient to justify the Resolution and that the said Resolution is contrary to law." 21 He raised the following contentions:22

1. The finding that he is not a Filipino citizen is not supported by the evidence consisting of his Oath of Allegiance
and the Affidavit of Renunciation, which show that he has substantially complied with the requirements of R.A. No.
9225;

2. The use of his US passport subsequent to his renunciation of his American citizenship is not tantamount to a
repudiation of his Filipino citizenship, as he did not perform any act to swear allegiance to a country other than the
Philippines;

3. He used his US passport only because he was not informed of the issuance of his Philippine passport, and that
he used his Philippine passport after he obtained it;

4. Balua’s petition to cancel the certificate of candidacy of Arnado was filed out of time, and the First Division’s
treatment of the petition as one for disqualification constitutes grave abuse of discretion amounting to excess of
jurisdiction;23

5. He is undoubtedly the people’s choice as indicated by his winning the elections;

6. His proclamation as the winning candidate ousted the COMELEC from jurisdiction over the case; and

7. The proper remedy to question his citizenship is through a petition for quo warranto, which should have been
filed within ten days from his proclamation.

Petitioner Casan Macode Maquiling (Maquiling), another candidate for mayor of Kauswagan, and who garnered the
second highest number of votes in the 2010 elections, intervened in the case and filed before the COMELEC En Banc a
Motion for Reconsideration together with an Opposition to Arnado’s Amended Motion for Reconsideration. Maquiling
argued that while the First Division correctly disqualified Arnado, the order of succession under Section 44 of the Local
Government Code is not applicable in this case. Consequently, he claimed that the cancellation of Arnado’s candidacy
and the nullification of his proclamation, Maquiling, as the legitimate candidate who obtained the highest number of lawful
votes, should be proclaimed as the winner.

Maquiling simultaneously filed his Memorandum with his Motion for Intervention and his Motion for Reconsideration.
Arnado opposed all motions filed by Maquiling, claiming that intervention is prohibited after a decision has already been
rendered, and that as a second-placer, Maquiling undoubtedly lost the elections and thus does not stand to be prejudiced
or benefitted by the final adjudication of the case.

RULING OF THE COMELEC EN BANC

In its Resolution of 02 February 2011, the COMELEC En Banc held that under Section 6 of Republic Act No. 6646, the
Commission "shall continue with the trial and hearing of the action, inquiry or protest even after the proclamation of the
candidate whose qualifications for office is questioned."

As to Maquiling’s intervention, the COMELEC En Banc also cited Section 6 of R.A. No. 6646 which allows intervention in
proceedings for disqualification even after elections if no final judgment has been rendered, but went on further to say that
Maquiling, as the second placer, would not be prejudiced by the outcome of the case as it agrees with the dispositive
portion of the Resolution of the First Division allowing the order of succession under Section 44 of the Local Government
Code to take effect.

The COMELEC En Banc agreed with the treatment by the First Division of the petition as one for disqualification, and
ruled that the petition was filed well within the period prescribed by law, 24 having been filed on 28 April 2010, which is not
later than 11 May 2010, the date of proclamation.

However, the COMELEC En Banc reversed and set aside the ruling of the First Division and granted Arnado’s Motion for
Reconsideration, on the following premises:

First:

By renouncing his US citizenship as imposed by R.A. No. 9225, the respondent embraced his Philippine citizenship as
though he never became a citizen of another country. It was at that time, April 3, 2009, that the respondent became a pure
Philippine Citizen again.

xxxx

The use of a US passport … does not operate to revert back his status as a dual citizen prior to his renunciation as there
is no law saying such. More succinctly, the use of a US passport does not operate to "un-renounce" what he has earlier on
renounced. The First Division’s reliance in the case of In Re: Petition for Habeas Corpus of Willy Yu v. Defensor-Santiago,
et al. is misplaced. The petitioner in the said case is a naturalized citizen who, after taking his oath as a naturalized
Filipino, applied for the renewal of his Portuguese passport. Strict policy is maintained in the conduct of citizens who are
not natural born, who acquire their citizenship by choice, thus discarding their original citizenship. The Philippine State
expects strict conduct of allegiance to those who choose to be its citizens. In the present case, respondent is not a
naturalized citizen but a natural born citizen who chose greener pastures by working abroad and then decided to repatriate
to supposedly help in the progress of Kauswagan. He did not apply for a US passport after his renunciation. Thus the
mentioned case is not on all fours with the case at bar.

xxxx

The respondent presented a plausible explanation as to the use of his US passport. Although he applied for a Philippine
passport, the passport was only issued on June 18, 2009. However, he was not notified of the issuance of his Philippine
passport so that he was actually able to get it about three (3) months later. Yet as soon as he was in possession of his
Philippine passport, the respondent already used the same in his subsequent travels abroad. This fact is proven by the
respondent’s submission of a certified true copy of his passport showing that he used the same for his travels on the
following dates: January 31, 2010, April 16, 2010, May 20, 2010, January 12, 2010, March 31, 2010 and June 4, 2010.
This then shows that the use of the US passport was because to his knowledge, his Philippine passport was not yet
issued to him for his use. As probably pressing needs might be undertaken, the respondent used whatever is within his
control during that time.25

In his Separate Concurring Opinion, COMELEC Chairman Sixto Brillantes cited that the use of foreign passport is not one
of the grounds provided for under Section 1 of Commonwealth Act No. 63 through which Philippine citizenship may be
lost.

"The application of the more assimilative principle of continuity of citizenship is more appropriate in this case. Under said
principle, once a person becomes a citizen, either by birth or naturalization, it is assumed that he desires to continue to be
a citizen, and this assumption stands until he voluntarily denationalizes or expatriates himself. Thus, in the instant case
respondent after reacquiring his Philippine citizenship should be presumed to have remained a Filipino despite his use of
his American passport in the absence of clear, unequivocal and competent proof of expatriation. Accordingly, all doubts
should be resolved in favor of retention of citizenship." 26

On the other hand, Commissioner Rene V. Sarmiento dissented, thus:

Respondent evidently failed to prove that he truly and wholeheartedly abandoned his allegiance to the United States. The
latter’s continued use of his US passport and enjoyment of all the privileges of a US citizen despite his previous
renunciation of the afore-mentioned citizenship runs contrary to his declaration that he chose to retain only his Philippine
citizenship. Respondent’s submission with the twin requirements was obviously only for the purpose of complying with the
requirements for running for the mayoralty post in connection with the May 10, 2010 Automated National and Local
Elections.

Qualifications for elective office, such as citizenship, are continuing requirements; once any of them is lost during his
incumbency, title to the office itself is deemed forfeited. If a candidate is not a citizen at the time he ran for office or if he
lost his citizenship after his election to office, he is disqualified to serve as such. Neither does the fact that respondent
obtained the plurality of votes for the mayoralty post cure the latter’s failure to comply with the qualification requirements
regarding his citizenship.

Since a disqualified candidate is no candidate at all in the eyes of the law, his having received the highest number of votes
does not validate his election. It has been held that where a petition for disqualification was filed before election against a
candidate but was adversely resolved against him after election, his having obtained the highest number of votes did not
make his election valid. His ouster from office does not violate the principle of vox populi suprema est lex because the
application of the constitutional and statutory provisions on disqualification is not a matter of popularity. To apply it is to
breath[e] life to the sovereign will of the people who expressed it when they ratified the Constitution and when they elected
their representatives who enacted the law.27

THE PETITION BEFORE THE COURT

Maquiling filed the instant petition questioning the propriety of declaring Arnado qualified to run for public office despite his
continued use of a US passport, and praying that Maquiling be proclaimed as the winner in the 2010 mayoralty race in
Kauswagan, Lanao del Norte.

Ascribing both grave abuse of discretion and reversible error on the part of the COMELEC En Banc for ruling that Arnado
is a Filipino citizen despite his continued use of a US passport, Maquiling now seeks to reverse the finding of the
COMELEC En Banc that Arnado is qualified to run for public office.

Corollary to his plea to reverse the ruling of the COMELEC En Banc or to affirm the First Division’s disqualification of
Arnado, Maquiling also seeks the review of the applicability of Section 44 of the Local Government Code, claiming that the
COMELEC committed reversible error in ruling that "the succession of the vice mayor in case the respondent is
disqualified is in order."
There are three questions posed by the parties before this Court which will be addressed seriatim as the subsequent
questions hinge on the result of the first.

The first question is whether or not intervention is allowed in a disqualification case.

The second question is whether or not the use of a foreign passport after renouncing foreign citizenship amounts to
undoing a renunciation earlier made.

A better framing of the question though should be whether or not the use of a foreign passport after renouncing foreign
citizenship affects one’s qualifications to run for public office.

The third question is whether or not the rule on succession in the Local Government Code is applicable to this case.

OUR RULING

Intervention of a rival candidate in a


disqualification case is proper when
there has not yet been any
proclamation of the winner.

Petitioner Casan Macode Maquiling intervened at the stage when respondent Arnado filed a Motion for Reconsideration of
the First Division Resolution before the COMELEC En Banc. As the candidate who garnered the second highest number
of votes, Maquiling contends that he has an interest in the disqualification case filed against Arnado, considering that in
the event the latter is disqualified, the votes cast for him should be considered stray and the second-placer should be
proclaimed as the winner in the elections.

It must be emphasized that while the original petition before the COMELEC is one for cancellation of the certificate of
candidacy and / or disqualification, the COMELEC First Division and the COMELEC En Banc correctly treated the petition
as one for disqualification.

The effect of a disqualification case is enunciated in Section 6 of R.A. No. 6646:

Sec. 6. Effect of Disqualification Case. - Any candidate who has been declared by final judgment to be disqualified shall
not be voted for, and the votes cast for him shall not be counted. If for any reason a candidate is not declared by final
judgment before an election to be disqualified and he is voted for and receives the winning number of votes in such
election, the Court or Commission shall continue with the trial and hearing of the action, inquiry, or protest and, upon
motion of the complainant or any intervenor, may during the pendency thereof order the suspension of the proclamation of
such candidate whenever the evidence of his guilt is strong.

Mercado v. Manzano28

clarified the right of intervention in a disqualification case. In that case, the Court said:

That petitioner had a right to intervene at that stage of the proceedings for the disqualification against private respondent
is clear from Section 6 of R.A. No. 6646, otherwise known as the Electoral Reforms Law of 1987, which provides: Any
candidate who has been declared by final judgment to be disqualified shall not be voted for, and the votes cast for him
shall not be counted. If for any reason a candidate is not declared by final judgment before an election to be disqualified
and he is voted for and receives the winning number of votes in such election, the Court or Commission shall continue
with the trial and hearing of the action, inquiry, or protest and, upon motion of the complainant or any intervenor, may
during the pendency thereof order the suspension of the proclamation of such candidate whenever the evidence of guilt is
strong. Under this provision, intervention may be allowed in proceedings for disqualification even after election if there has
yet been no final judgment rendered.29

Clearly then, Maquiling has the right to intervene in the case. The fact that the COMELEC En Banc has already ruled that
Maquiling has not shown that the requisites for the exemption to the second-placer rule set forth in Sinsuat v.
COMELEC30 are present and therefore would not be prejudiced by the outcome of the case, does not deprive Maquiling of
the right to elevate the matter before this Court.

Arnado’s claim that the main case has attained finality as the original petitioner and respondents therein have not
appealed the decision of the COMELEC En Banc, cannot be sustained. The elevation of the case by the intervenor
prevents it from attaining finality. It is only after this Court has ruled upon the issues raised in this instant petition that the
disqualification case originally filed by Balua against Arnado will attain finality.

The use of foreign passport after renouncing one’s foreign citizenship is a positive and voluntary act of
representation as to one’s nationality and citizenship; it does not divest Filipino citizenship regained by
repatriation but it recants the Oath of Renunciation required to qualify one to run for an elective position.

Section 5(2) of The Citizenship Retention and Re-acquisition Act of 2003 provides:
Those who retain or re-acquire Philippine citizenship under this Act shall enjoy full civil and political rights and be subject
to all attendant liabilities and responsibilities under existing laws of the Philippines and the following conditions:

xxxx

(2)Those seeking elective public in the Philippines shall meet the qualification for holding such public office as required by
the Constitution and existing laws and, at the time of the filing of the certificate of candidacy, make a personal and sworn
renunciation of any and all foreign before any public officer authorized to administer an oath.

x x x31

Rommel Arnado took all the necessary steps to qualify to run for a public office. He took the Oath of Allegiance and
renounced his foreign citizenship. There is no question that after performing these twin requirements required under
Section 5(2) of R.A. No. 9225 or the Citizenship Retention and Re-acquisition Act of 2003, he became eligible to run for
public office.

Indeed, Arnado took the Oath of Allegiance not just only once but twice: first, on 10 July 2008 when he applied for
repatriation before the Consulate General of the Philippines in San Francisco, USA, and again on 03 April 2009
simultaneous with the execution of his Affidavit of Renunciation. By taking the Oath of Allegiance to the Republic, Arnado
re-acquired his Philippine citizenship. At the time, however, he likewise possessed American citizenship. Arnado had
therefore become a dual citizen.

After reacquiring his Philippine citizenship, Arnado renounced his American citizenship by executing an Affidavit of
Renunciation, thus completing the requirements for eligibility to run for public office.

By renouncing his foreign citizenship, he was deemed to be solely a Filipino citizen, regardless of the effect of such
renunciation under the laws of the foreign country. 32

However, this legal presumption does not operate permanently and is open to attack when, after renouncing the foreign
citizenship, the citizen performs positive acts showing his continued possession of a foreign citizenship. 33

Arnado himself subjected the issue of his citizenship to attack when, after renouncing his foreign citizenship, he continued
to use his US passport to travel in and out of the country before filing his certificate of candidacy on 30 November 2009.
The pivotal question to determine is whether he was solely and exclusively a Filipino citizen at the time he filed his
certificate of candidacy, thereby rendering him eligible to run for public office.

Between 03 April 2009, the date he renounced his foreign citizenship, and 30 November 2009, the date he filed his COC,
he used his US passport four times, actions that run counter to the affidavit of renunciation he had earlier executed. By
using his foreign passport, Arnado positively and voluntarily represented himself as an American, in effect declaring before
immigration authorities of both countries that he is an American citizen, with all attendant rights and privileges granted by
the United States of America.

The renunciation of foreign citizenship is not a hollow oath that can simply be professed at any time, only to be violated the
next day. It requires an absolute and perpetual renunciation of the foreign citizenship and a full divestment of all civil and
political rights granted by the foreign country which granted the citizenship.

Mercado v. Manzano34 already hinted at this situation when the Court declared:

His declarations will be taken upon the faith that he will fulfill his undertaking made under oath. Should he betray that trust,
there are enough sanctions for declaring the loss of his Philippine citizenship through expatriation in appropriate
proceedings. In Yu v. Defensor-Santiago, we sustained the denial of entry into the country of petitioner on the ground that,
after taking his oath as a naturalized citizen, he applied for the renewal of his Portuguese passport and declared in
commercial documents executed abroad that he was a Portuguese national. A similar sanction can be taken against
anyone who, in electing Philippine citizenship, renounces his foreign nationality, but subsequently does some act
constituting renunciation of his Philippine citizenship.

While the act of using a foreign passport is not one of the acts enumerated in Commonwealth Act No. 63 constituting
renunciation and loss of Philippine citizenship, 35 it is nevertheless an act which repudiates the very oath of renunciation
required for a former Filipino citizen who is also a citizen of another country to be qualified to run for a local elective
position.

When Arnado used his US passport on 14 April 2009, or just eleven days after he renounced his American citizenship, he
recanted his Oath of Renunciation36 that he "absolutely and perpetually renounce(s) all allegiance and fidelity to the
UNITED STATES OF AMERICA"37 and that he "divest(s) himself of full employment of all civil and political rights and
privileges of the United States of America." 38

We agree with the COMELEC En Banc that such act of using a foreign passport does not divest Arnado of his Filipino
citizenship, which he acquired by repatriation. However, by representing himself as an American citizen, Arnado
voluntarily and effectively reverted to his earlier status as a dual citizen. Such reversion was not retroactive; it took place
the instant Arnado represented himself as an American citizen by using his US passport.
This act of using a foreign passport after renouncing one’s foreign citizenship is fatal to Arnado’s bid for public office, as it
effectively imposed on him a disqualification to run for an elective local position.

Arnado’s category of dual citizenship is that by which foreign citizenship is acquired through a positive act of applying for
naturalization. This is distinct from those considered dual citizens by virtue of birth, who are not required by law to take the
oath of renunciation as the mere filing of the certificate of candidacy already carries with it an implied renunciation of
foreign citizenship.39 Dual citizens by naturalization, on the other hand, are required to take not only the Oath of Allegiance
to the Republic of the Philippines but also to personally renounce foreign citizenship in order to qualify as a candidate for
public office.

By the time he filed his certificate of candidacy on 30 November 2009, Arnado was a dual citizen enjoying the rights and
privileges of Filipino and American citizenship. He was qualified to vote, but by the express disqualification under Section
40(d) of the Local Government Code,40 he was not qualified to run for a local elective position.

In effect, Arnado was solely and exclusively a Filipino citizen only for a period of eleven days, or from 3 April 2009 until 14
April 2009, on which date he first used his American passport after renouncing his American citizenship.

This Court has previously ruled that:

Qualifications for public office are continuing requirements and must be possessed not only at the time of appointment or
election or assumption of office but during the officer's entire tenure. Once any of the required qualifications is lost, his title
may be seasonably challenged. x x x.41

The citizenship requirement for elective public office is a continuing one. It must be possessed not just at the time of the
renunciation of the foreign citizenship but continuously. Any act which violates the oath of renunciation opens the
citizenship issue to attack.

We agree with the pronouncement of the COMELEC First Division that "Arnado’s act of consistently using his US passport
effectively negated his "Affidavit of Renunciation." 42 This does not mean, that he failed to comply with the twin
requirements under R.A. No. 9225, for he in fact did.

It was after complying with the requirements that he performed positive acts which effectively disqualified him from running
for an elective public office pursuant to Section 40(d) of the Local Government Code of 1991.

The purpose of the Local Government Code in disqualifying dual citizens from running for any elective public office would
be thwarted if we were to allow a person who has earlier renounced his foreign citizenship, but who subsequently
represents himself as a foreign citizen, to hold any public office.

Arnado justifies the continued use of his US passport with the explanation that he was not notified of the issuance of his
Philippine passport on 18 June 2009, as a result of which he was only able to obtain his Philippine passport three (3)
months later.43

The COMELEC En Banc differentiated Arnado from Willy Yu, the Portuguese national who sought naturalization as a
Filipino citizen and later applied for the renewal of his Portuguese passport. That Arnado did not apply for a US passport
after his renunciation does not make his use of a US passport less of an act that violated the Oath of Renunciation he
took. It was still a positive act of representation as a US citizen before the immigration officials of this country.

The COMELEC, in ruling favorably for Arnado, stated "Yet, as soon as he was in possession of his Philippine passport,
the respondent already used the same in his subsequent travels abroad." 44 We cannot agree with the COMELEC. Three
months from June is September. If indeed, Arnado used his Philippine passport as soon as he was in possession of it, he
would not have used his US passport on 24 November 2009.

Besides, Arnado’s subsequent use of his Philippine passport does not correct the fact that after he renounced his foreign
citizenship and prior to filing his certificate of candidacy, he used his US passport. In the same way that the use of his
foreign passport does not undo his Oath of Renunciation, his subsequent use of his Philippine passport does not undo his
earlier use of his US passport.

Citizenship is not a matter of convenience. It is a badge of identity that comes with attendant civil and political rights
accorded by the state to its citizens. It likewise demands the concomitant duty to maintain allegiance to one’s flag and
country. While those who acquire dual citizenship by choice are afforded the right of suffrage, those who seek election or
appointment to public office are required to renounce their foreign citizenship to be deserving of the public trust. Holding
public office demands full and undivided allegiance to the Republic and to no other.

We therefore hold that Arnado, by using his US passport after renouncing his American citizenship, has recanted the
same Oath of Renunciation he took. Section 40(d) of the Local Government Code applies to his situation. He is
disqualified not only from holding the public office but even from becoming a candidate in the May 2010 elections.

We now resolve the next issue.

Resolving the third issue necessitates revisiting Topacio v. Paredes 45 which is the jurisprudential spring of the principle that
a second-placer cannot be proclaimed as the winner in an election contest. This doctrine must be re-examined and its
soundness once again put to the test to address the ever-recurring issue that a second-placer who loses to an ineligible
candidate cannot be proclaimed as the winner in the elections.

The Facts of the case are as follows:

On June 4, 1912, a general election was held in the town of Imus, Province of Cavite, to fill the office of municipal
president. The petitioner, Felipe Topacio, and the respondent, Maximo Abad, were opposing candidates for that office.
Topacio received 430 votes, and Abad 281. Abad contested the election upon the sole ground that Topacio was ineligible
in that he was reelected the second time to the office of the municipal president on June 4, 1912, without the four years
required by Act No. 2045 having intervened. 46

Abad thus questioned the eligibility of To p a c i o on the basis of a statutory prohibition for seeking a second re-election
absent the four year interruption.

The often-quoted phrase in Topacio v. Paredes is that "the wreath of victory cannot be transferred from an ineligible
candidate to any other candidate when the sole question is the eligibility of the one receiving a plurality of the legally cast
ballots."47

This phrase is not even the ratio decidendi; it is a mere obiter dictum. The Court was comparing "the effect of a decision
that a candidate is not entitled to the office because of fraud or irregularities in the elections x x x with that produced by
declaring a person ineligible to hold such an office."

The complete sentence where the phrase is found is part of a comparison and contrast between the two situations, thus:

Again, the effect of a decision that a candidate is not entitled to the office because of fraud or irregularities in the elections
is quite different from that produced by declaring a person ineligible to hold such an office. In the former case the court,
after an examination of the ballots may find that some other person than the candidate declared to have received a
plurality by the board of canvassers actually received the greater number of votes, in which case the court issues its
mandamus to the board of canvassers to correct the returns accordingly; or it may find that the manner of holding the
election and the returns are so tainted with fraud or illegality that it cannot be determined who received a plurality of the
legally cast ballots. In the latter case, no question as to the correctness of the returns or the manner of casting and
counting the ballots is before the deciding power, and generally the only result can be that the election fails entirely. In the
former, we have a contest in the strict sense of the word, because of the opposing parties are striving for supremacy. If it
be found that the successful candidate (according to the board of canvassers) obtained a plurality in an illegal manner,
and that another candidate was the real victor, the former must retire in favor of the latter. In the other case, there is not,
strictly speaking, a contest, as the wreath of victory cannot be transferred from an ineligible candidate to any other
candidate when the sole question is the eligibility of the one receiving a plurality of the legally cast ballots. In the one case
the question is as to who received a plurality of the legally cast ballots; in the other, the question is confined to the
personal character and circumstances of a single individual. 48 (Emphasis supplied)

Note that the sentence where the phrase is found starts with "In the other case, there is not, strictly speaking, a contest" in
contrast to the earlier statement, "In the former, we have a contest in the strict sense of the word, because of the opposing
parties are striving for supremacy."

The Court in Topacio v. Paredes cannot be said to have held that "the wreath of victory cannot be transferred from an
ineligible candidate to any other candidate when the sole question is the eligibility of the one receiving a plurality of the
legally cast ballots."

A proper reading of the case reveals that the ruling therein is that since the Court of First Instance is without jurisdiction to
try a disqualification case based on the eligibility of the person who obtained the highest number of votes in the election,
its jurisdiction being confined "to determine which of the contestants has been duly elected" the judge exceeded his
jurisdiction when he "declared that no one had been legally elected president of the municipality of Imus at the general
election held in that town on 4 June 1912" where "the only question raised was whether or not Topacio was eligible to be
elected and to hold the office of municipal president."

The Court did not rule that Topacio was disqualified and that Abad as the second placer cannot be proclaimed in his
stead. The Court therein ruled:

For the foregoing reasons, we are of the opinion and so hold that the respondent judge exceeded his jurisdiction in
declaring in those proceedings that no one was elected municipal president of the municipality of Imus at the last general
election; and that said order and all subsequent proceedings based thereon are null and void and of no effect; and,
although this decision is rendered on respondents' answer to the order to show cause, unless respondents raised some
new and additional issues, let judgment be entered accordingly in 5 days, without costs. So ordered. 49

On closer scrutiny, the phrase relied upon by a host of decisions does not even have a legal basis to stand on. It was a
mere pronouncement of the Court comparing one process with another and explaining the effects thereof. As an
independent statement, it is even illogical.

Let us examine the statement:


"x x x the wreath of victory cannot be transferred from an ineligible candidate to any other candidate when the sole
question is the eligibility of the one receiving a plurality of the legally cast ballots."

What prevents the transfer of the wreath of victory from the ineligible candidate to another candidate?

When the issue being decided upon by the Court is the eligibility of the one receiving a plurality of the legally cast ballots
and ineligibility is thereafter established, what stops the Court from adjudging another eligible candidate who received the
next highest number of votes as the winner and bestowing upon him that "wreath?"

An ineligible candidate who receives the highest number of votes is a wrongful winner. By express legal mandate, he
could not even have been a candidate in the first place, but by virtue of the lack of material time or any other intervening
circumstances, his ineligibility might not have been passed upon prior to election date. Consequently, he may have had
the opportunity to hold himself out to the electorate as a legitimate and duly qualified candidate. However, notwithstanding
the outcome of the elections, his ineligibility as a candidate remains unchanged. Ineligibility does not only pertain to his
qualifications as a candidate but necessarily affects his right to hold public office. The number of ballots cast in his favor
cannot cure the defect of failure to qualify with the substantive legal requirements of eligibility to run for public office.

The popular vote does not cure the


ineligibility of a candidate.

The ballot cannot override the constitutional and statutory requirements for qualifications and disqualifications of
candidates. When the law requires certain qualifications to be possessed or that certain disqualifications be not possessed
by persons desiring to serve as elective public officials, those qualifications must be met before one even becomes a
candidate. When a person who is not qualified is voted for and eventually garners the highest number of votes, even the
will of the electorate expressed through the ballot cannot cure the defect in the qualifications of the candidate. To rule
otherwise is to trample upon and rent asunder the very law that sets forth the qualifications and disqualifications of
candidates. We might as well write off our election laws if the voice of the electorate is the sole determinant of who should
be proclaimed worthy to occupy elective positions in our republic.

This has been, in fact, already laid down by the Court in Frivaldo v. COMELEC 50 when we pronounced:

x x x. The fact that he was elected by the people of Sorsogon does not excuse this patent violation of the salutary rule
limiting public office and employment only to the citizens of this country. The qualifications prescribed for elective office
cannot be erased by the electorate alone.

The will of the people as expressed through the ballot cannot cure the vice of ineligibility, especially if they mistakenly
believed, as in this case, that the candidate was qualified. Obviously, this rule requires strict application when the
deficiency is lack of citizenship. If a person seeks to serve in the Republic of the Philippines, he must owe his total loyalty
to this country only, abjuring and renouncing all fealty and fidelity to any other state. 51 (Emphasis supplied)

This issue has also been jurisprudentially clarified in Velasco v. COMELEC 52 where the Court ruled that the ruling in
Quizon and Saya-ang cannot be interpreted without qualifications lest "Election victory x x x becomes a magic formula to
bypass election eligibility requirements." 53

We have ruled in the past that a candidate’s victory in the election may be considered a sufficient basis to rule in favor of
the candidate sought to be disqualified if the main issue involves defects in the candidate’s certificate of candidacy. We
said that while provisions relating to certificates of candidacy are mandatory in terms, it is an established rule of
interpretation as regards election laws, that mandatory provisions requiring certain steps before elections will be construed
as directory after the elections, to give effect to the will of the people. We so ruled in Quizon v. COMELEC and Saya-ang
v. COMELEC:

The present case perhaps presents the proper time and opportunity to fine-tune our above ruling. We say this with the
realization that a blanket and unqualified reading and application of this ruling can be fraught with dangerous significance
for the rule of law and the integrity of our elections. For one, such blanket/unqualified reading may provide a way around
the law that effectively negates election requirements aimed at providing the electorate with the basic information to make
an informed choice about a candidate’s eligibility and fitness for office.

The first requirement that may fall when an unqualified reading is made is Section 39 of the LGC which specifies the basic
qualifications of local government officials. Equally susceptive of being rendered toothless is Section 74 of the OEC that
sets out what should be stated in a COC. Section 78 may likewise be emasculated as mere delay in the resolution of the
petition to cancel or deny due course to a COC can render a Section 78 petition useless if a candidate with false COC
data wins. To state the obvious, candidates may risk falsifying their COC qualifications if they know that an election victory
will cure any defect that their COCs may have. Election victory then becomes a magic formula to bypass election eligibility
requirements. (Citations omitted)

What will stop an otherwise disqualified individual from filing a seemingly valid COC, concealing any disqualification, and
employing every strategy to delay any disqualification case filed against him so he can submit himself to the electorate
and win, if winning the election will guarantee a disregard of constitutional and statutory provisions on qualifications and
disqualifications of candidates?
It is imperative to safeguard the expression of the sovereign voice through the ballot by ensuring that its exercise respects
the rule of law. To allow the sovereign voice spoken through the ballot to trump constitutional and statutory provisions on
qualifications and disqualifications of candidates is not democracy or republicanism. It is electoral anarchy. When set rules
are disregarded and only the electorate’s voice spoken through the ballot is made to matter in the end, it precisely serves
as an open invitation for electoral anarchy to set in.
1âwphi1

Maquiling is not a second-placer as


he obtained the highest number of
votes from among the qualified
candidates.

With Arnado’s disqualification, Maquiling then becomes the winner in the election as he obtained the highest number of
votes from among the qualified candidates.

We have ruled in the recent cases of Aratea v. COMELEC 54 and Jalosjos v. COMELEC55 that a void COC cannot produce
any legal effect.

Thus, the votes cast in favor of the ineligible candidate are not considered at all in determining the winner of an election.

Even when the votes for the ineligible candidate are disregarded, the will of the electorate is still respected, and even more
so. The votes cast in favor of an ineligible candidate do not constitute the sole and total expression of the sovereign voice.
The votes cast in favor of eligible and legitimate candidates form part of that voice and must also be respected.

As in any contest, elections are governed by rules that determine the qualifications and disqualifications of those who are
allowed to participate as players. When there are participants who turn out to be ineligible, their victory is voided and the
laurel is awarded to the next in rank who does not possess any of the disqualifications nor lacks any of the qualifications
set in the rules to be eligible as candidates.

There is no need to apply the rule cited in Labo v. COMELEC 56 that when the voters are well aware within the realm of
notoriety of a candidate’s disqualification and still cast their votes in favor said candidate, then the eligible candidate
obtaining the next higher number of votes may be deemed elected. That rule is also a mere obiter that further complicated
the rules affecting qualified candidates who placed second to ineligible ones.

The electorate’s awareness of the candidate’s disqualification is not a prerequisite for the disqualification to attach to the
candidate. The very existence of a disqualifying circumstance makes the candidate ineligible. Knowledge by the electorate
of a candidate’s disqualification is not necessary before a qualified candidate who placed second to a disqualified one can
be proclaimed as the winner. The second-placer in the vote count is actually the first-placer among the qualified
candidates.

That the disqualified candidate has already been proclaimed and has assumed office is of no moment. The subsequent
disqualification based on a substantive ground that existed prior to the filing of the certificate of candidacy voids not only
the COC but also the proclamation.

Section 6 of R.A. No. 6646 provides:

Section 6. Effect of Disqualification Case. - Any candidate who has been declared by final judgment to be disqualified shall
not be voted for, and the votes cast for him shall not be counted. If for any reason a candidate is not declared by final
judgment before an election to be disqualified and he is voted for and receives the winning number of votes in such
election, the Court or Commission shall continue with the trial and hearing of the action, inquiry, or protest and, upon
motion of the complainant or any intervenor, may during the pendency thereof order the suspension of the proclamation of
such candidate whenever the evidence of his guilt is strong.

There was no chance for Arnado’s proclamation to be suspended under this rule because Arnado failed to file his answer
to the petition seeking his disqualification. Arnado only filed his Answer on 15 June 2010, long after the elections and after
he was already proclaimed as the winner.

The disqualifying circumstance surrounding Arnado’s candidacy involves his citizenship. It does not involve the
commission of election offenses as provided for in the first sentence of Section 68 of the Omnibus Election Code, the
effect of which is to disqualify the individual from continuing as a candidate, or if he has already been elected, from holding
the office.

The disqualifying circumstance affecting Arnado is his citizenship. As earlier discussed, Arnado was both a Filipino and an
American citizen when he filed his certificate of candidacy. He was a dual citizen disqualified to run for public office based
on Section 40(d) of the Local Government Code.

Section 40 starts with the statement "The following persons are disqualified from running for any elective local position."
The prohibition serves as a bar against the individuals who fall under any of the enumeration from participating as
candidates in the election.

With Arnado being barred from even becoming a candidate, his certificate of candidacy is thus rendered void from the
beginning. It could not have produced any other legal effect except that Arnado rendered it impossible to effect his
disqualification prior to the elections because he filed his answer to the petition when the elections were conducted
already and he was already proclaimed the winner.

To hold that such proclamation is valid is to negate the prohibitory character of the disqualification which Arnado
possessed even prior to the filing of the certificate of candidacy. The affirmation of Arnado's disqualification, although
made long after the elections, reaches back to the filing of the certificate of candidacy. Arnado is declared to be not a
candidate at all in the May 201 0 elections.

Arnado being a non-candidate, the votes cast in his favor should not have been counted. This leaves Maquiling as the
qualified candidate who obtained the highest number of votes. Therefore, the rule on succession under the Local
Government Code will not apply.

WHEREFORE, premises considered, the Petition is GRANTED. The Resolution of the COMELEC En Bane dated 2
February 2011 is hereby ANNULLED and SET ASIDE. Respondent ROMMEL ARNADO y CAGOCO is disqualified from
running for any local elective position. CASAN MACODE MAQUILING is hereby DECLARED the duly elected Mayor of
Kauswagan, Lanao del Norte in the 10 May 2010 elections.

This Decision is immediately executory.

Let a copy of this Decision be served personally upon the parties and the Commission on Elections.

No pronouncement as to costs.

SO ORDERED.

- David v Agbay, GR 199113, Mar 18, 2015

This is a petition for review under Rule 45 seeking to reverse the Order  dated October 8, 2011 of the Regional Trial Court
1

(RTC) of Pinamalayan, Oriental Mindoro, which denied the petition for certiorari filed by Renato(petitioner)M. David.
Petitioner assailed the Order  dated March 22, 2011 of the Municipal Trial Court (MTC) of Socorro, Oriental Mindoro
2

denying his motion for redetermination of probable cause.

The factual antecedents:

In 1974, petitioner migrated to Canada where he became a Canadian citizen by naturalization. Upon their retirement,
petitioner and his wife returned to the Philippines. Sometime in 2000, they purchased a 600-square meter lot along the
beach in Tambong, Gloria, Oriental Mindoro where they constructed a residential house. However, in the year 2004, they
came to know that the portion where they built their house is public land and part of the salvage zone.

On April 12, 2007, petitioner filed a Miscellaneous Lease Application  (MLA) over the subject land with the Department of
3

Environment and Natural Resources (DENR) at the Community Environment and Natural Resources Office (CENRO) in
Socorro. In the said application, petitioner indicated that he is a Filipino citizen.

Private respondent Editha A. Agbay opposed the application on the ground that petitioner, a Canadian citizen, is
disqualified to own land. She also filed a criminal complaint for falsification of public documents under Article 172 of
the Revised Penal Code (RPC) (I.S. No. 08-6463) against the petitioner.

Meanwhile, petitioner re-acquired his Filipino citizenship under the provisions of Republic Act No. 9225,  (R.A. 9225) as
4

evidenced by Identification Certificate No. 266-10-07  issued by the Consulate General of the Philippines (Toronto) on
5

October 11, 2007.

In his defense, petitioner averred that at the time he filed his application, he had intended to re-acquire Philippine
citizenship and that he had been assured by a CENRO officer that he could declare himself as a Filipino. He further
alleged that he bought the property from the Agbays who misrepresented to him that the subject property was titled land
and they have the right and authority to convey the same. The dispute had in fact led to the institution of civil and criminal
suits between him and private respondent’s family.

On January 8, 2008,  the Office of the Provincial Prosecutor issued its Resolution  finding probable cause to indict
6 7

petitioner for violation of Article 172 of the RPC and recommending the filing of the corresponding information in court.
Petitioner challenged the said resolution in a petition for review he filed before the Department of Justice (DOJ).

On June 3, 2008, the CENRO issued an order rejecting petitioner’s MLA. It ruled that petitioner’s subsequent re-
acquisition of Philippine citizenship did not cure the defect in his MLA which was void ab initio.8

In the meantime, on July 26, 2010, the petition for review filed by petitioner was denied by the DOJ which held that the
presence of the elements of the crime of falsification of public document suffices to warrant indictment of the petitioner
notwithstanding the absence of any proof that he gained or intended to injure a third person in committing the act of
falsification.  Consequently, an information for Falsification of Public Document was filed before the MTC (Criminal Case
9

No. 2012) and a warrant of arrest was issued against the petitioner.
On February 11, 2011, after the filing of the Information and before his arrest, petitioner filed an Urgent Motion for Re-
Determination of Probable Cause  in the MTC. Interpreting the provisions of the law relied upon by petitioner, the said
10

court denied the motion, holding that R.A. 9225 makes a distinction between those who became foreign citizens during its
effectivity, and those who lost their Philippine citizenship before its enactment when the governing law was
Commonwealth Act No. 63  (CA 63). Since the crime for which petitioner was charged was alleged and admitted to have
11

been committed on April 12, 2007 before he had re- acquired his Philippine citizenship, the MTC concluded that petitioner
was at that time still a Canadian citizen. Thus, the MTC ordered:

WHEREFORE, for lack of jurisdiction over the person of the accused, and for lack of merit, the motion is DENIED.

SO ORDERED. 12

In his motion for reconsideration,  petitioner questioned the foregoing order denying him relief on the ground of lack of
13

jurisdiction and insisted that the issue raised is purely legal. He argued that since his application had yet to receive final
evaluation and action by the DENR Region IV-B office in Manila, it is academic to ask the citizenship of the applicant
(petitioner) who had re-acquired Philippine citizenship six months after he applied for lease of public land. The MTC
denied the motion for reconsideration. 14

Dissatisfied, petitioner elevated the case to the RTC via a petition  for certiorari under Rule 65, alleging grave abuse of
15

discretion on the part of the MTC. He asserted that first, jurisdiction over the person of an accused cannot be a pre-
condition for the re-determination of probable cause by the court that issues a warrant of arrest; and second, the March
22, 2011 Order disregarded the legal fiction that once a natural-born Filipino citizen who had been naturalized in another
country re-acquires his citizenship under R.A. 9225, his Filipino citizenship is thus deemed not to have been lost on
account of said naturalization.

In his Comment and Opposition,  the prosecutor emphasized that the act of falsification was already consummated as
16

petitioner has not yet re-acquired his Philippine citizenship, and his subsequent oath to re-acquire Philippine citizenship
will only affect his citizenship status and not his criminal act which was long consummated prior to said oath of allegiance.

On October 8, 2011, the RTC issued the assailed Order denying the petition for certiorari after finding no grave abuse of
discretion committed by the lower court, thus:

ACCORDINGLY, the petition is hereby DENIED. At any rate petitioner is not left without any remedy or recourse because
he can proceed to trial where he can make use of his claim to be a Filipino citizen as his defense to be adjudicated in a full
blown trial, and in case of conviction, to appeal such conviction.

SO ORDERED. 17

Petitioner is now before us arguing that –

A. By supporting the prosecution of the petitioner for falsification, the lower court has disregarded the undisputed
fact that petitioner is a natural-born Filipino citizen, and that by re-acquiring the same status under R.A. No. 9225
he was by legal fiction "deemed not to have lost" it at the time of his naturalization in Canada and through the time
when he was said to have falsely claimed Philippine citizenship.

B. By compelling petitioner to first return from his legal residence in Canada and to surrender or allow himself to be
arrested under a warrant for his alleged false claim to Philippine citizenship, the lower court has pre-empted the
right of petitioner through his wife and counsel to question the validity of the said warrant of arrest against him
before the same is implemented, which is tantamount to a denial of due process. 18

In his Comment, the Solicitor General contends that petitioner’s argument regarding the retroactivity of R.A. 9225 is
without merit.  It is contended that this Court’s rulings in Frivaldo v. Commission on Elections  and Altarejos v.
1âwphi1
19

Commission on Elections  on the retroactivity of one’s re- acquisition of Philippine citizenship to the date of filing his
20

application therefor cannot be applied to the case of herein petitioner. Even assuming for the sake of argument that such
doctrine applies in the present situation, it will still not work for petitioner’s cause for the simple reason that he had not
alleged, much less proved, that he had already applied for reacquisition of Philippine citizenship before he made the
declaration in the Public Land Application that he is a Filipino. Moreover, it is stressed that in falsification of public
document, it is not necessary that the idea of gain or intent to injure a third person be present. As to petitioner’s defense of
good faith, such remains to be a defense which may be properly raised and proved in a full- blown trial.

On the issue of jurisdiction over the person of accused (petitioner), the Solicitor General opines that in seeking an
affirmative relief from the MTC when he filed his Urgent Motion for Re-determination of Probable Cause, petitioner is
deemed to have submitted his person to the said court’s jurisdiction by his voluntary appearance. Nonetheless, the RTC
correctly ruled that the lower court committed no grave abuse of discretion in denying the petitioner’s motion after a
judicious, thorough and personal evaluation of the parties’ arguments contained in their respective pleadings, and the
evidence submitted before the court.

In sum, the Court is asked to resolve whether (1) petitioner may be indicted for falsification for representing himself as a
Filipino in his Public Land Application despite his subsequent re-acquisition of Philippine citizenship under the provisions
of R.A. 9225; and (2) the MTC properly denied petitioner’s motion for re-determination of probable cause on the ground of
lack of jurisdiction over the person of the accused (petitioner).
R.A. 9225, otherwise known as the "Citizenship Retention and Re- acquisition Act of 2003," was signed into law by
President Gloria Macapagal-Arroyo on August 29, 2003. Sections 2 and 3 of said law read:

SEC. 2. Declaration of Policy.–It is hereby declared the policy of the State that all Philippine citizens who become citizens
of another country shall be deemed not to have lost their Philippine citizenship under the conditions of this Act.

SEC. 3. Retention of Philippine Citizenship.–Any provision of law to the contrary notwithstanding, natural-born citizens of
the Philippines who have lost their Philippine citizenship by reason of their naturalization as citizens of a foreign country
are hereby deemed to have reacquired Philippine citizenship upon taking the following oath of allegiance to the
Republic:

"I ______________________, solemnly swear (or affirm) that I will support and defend the Constitution of the Republic of
the Philippines and obey the laws and legal orders promulgated by the duly constituted authorities of the Philippines; and I
hereby declare that I recognize and accept the supreme authority of the Philippines and will maintain true faith and
allegiance thereto; and that I impose this obligation upon myself voluntarily without mental reservation or purpose of
evasion."

Natural-born citizens of the Philippines who, after the effectivity of this Act, become citizens of a foreign country
shall retain their Philippine citizenship upon taking the aforesaid oath. (Emphasis supplied)

While Section 2 declares the general policy that Filipinos who have become citizens of another country shall be deemed
"not to have lost their Philippine citizenship," such is qualified by the phrase "under the conditions of this Act." Section 3
lays down such conditions for two categories of natural-born Filipinos referred to in the first and second paragraphs. Under
the first paragraph are those natural-born Filipinos who have lost their citizenship by naturalization in a foreign country
who shall re-acquire their Philippine citizenship upon taking the oath of allegiance to the Republic of the Philippines. The
second paragraph covers those natural-born Filipinos who became foreign citizens after R.A. 9225 took effect, who
shall retain their Philippine citizenship upon taking the same oath. The taking of oath of allegiance is required for both
categories of natural-born Filipino citizens who became citizens of a foreign country, but the terminology used is different,
"re-acquired" for the first group, and "retain" for the second group.

The law thus makes a distinction between those natural-born Filipinos who became foreign citizens before and after the
effectivity of R.A. 9225. Although the heading of Section 3 is "Retention of Philippine Citizenship", the authors of the law
intentionally employed the terms "re-acquire" and "retain" to describe the legal effect of taking the oath of allegiance to the
Republic of the Philippines. This is also evident from the title of the law using both re-acquisition and retention.

In fine, for those who were naturalized in a foreign country, they shall be deemed to have re-acquired their Philippine
citizenship which was lost pursuant to CA 63, under which naturalization in a foreign country is one of the ways by which
Philippine citizenship may be lost. As its title declares, R.A. 9225 amends CA 63 by doing away with the provision in the
old law which takes away Philippine citizenship from natural-born Filipinos who become naturalized citizens of other
countries and allowing dual citizenship,  and also provides for the procedure for re-acquiring and retaining Philippine
21

citizenship. In the case of those who became foreign citizens after R.A. 9225 took effect, they shall retain Philippine
citizenship despite having acquired foreign citizenship provided they took the oath of allegiance under the new law.

Petitioner insists we should not distinguish between re-acquisition and retention in R.A. 9225. He asserts that in criminal
cases, that interpretation of the law which favors the accused is preferred because it is consistent with the constitutional
presumption of innocence, and in this case it becomes more relevant when a seemingly difficult question of law is
expected to have been understood by the accused, who is a non-lawyer, at the time of the commission of the alleged
offense. He further cites the letter-reply dated January 31, 2011  of the Bureau of Immigration (BI) to his query, stating
22

that his status as a natural-born Filipino will be governed by Section 2 of R.A. 9225.

These contentions have no merit.

That the law distinguishes between re-acquisition and retention of Philippine citizenship was made clear in the discussion
of the Bicameral Conference Committee on the Disagreeing Provisions of House Bill No. 4720 and Senate Bill No. 2130
held on August 18, 2003, where Senator Franklin Drilon was responding to the query of Representative Exequiel Javier:

REP. JAVIER. I have some questions in Section 3. Here, under Section 3 of the Senate version, "Any provision of law on
the contrary notwithstanding, natural-born citizens of the Philippines who, after the effectivity of this Act, shall… and so
forth, ano, shall retain their Philippine citizenship.

Now in the second paragraph, natural-born citizens who have lost their citizenship by reason of their naturalization after
the effectivity of this Act are deemed to have reacquired…

THE CHAIRMAN (SEN. DRILON). Prior to the effectivity.

REP. JAVIER. Well, you have two kinds of natural-born citizens here. Natural-born citizens who acquired foreign
citizenship after the effectivity of this act are considered to have retained their citizenship. But natural-born citizens who
lost their Filipino citizenship before the effectivity of this act are considered to have reacquired. May I know the distinction?
Do you mean to say that natural-born citizens who became, let’s say, American citizens after the effectivity of this act are
considered natural-born?
Now in the second paragraph are the natural-born citizens who lost their citizenship before the effectivity of this act are no
longer natural born citizens because they have just reacquired their citizenship. I just want to know this distinction, Mr.
Chairman.

THE CHAIRMAN (SEN. DRILON). The title of the Senate version is precisely retention and reacquisition. The
reacquisition will apply to those who lost their Philippine citizenship by virtue of Commonwealth Act 63. Upon the
effectivity -- assuming that we can agree on this, upon the effectivity of this new measure amending Commonwealth Act
63, the Filipinos who lost their citizenship is deemed to have reacquired their Philippine citizenship upon the effectivity of
the act.

The second aspect is the retention of Philippine citizenship applying to future instances. So that’s the distinction.

REP. JAVIER. Well, I’m just asking this question because we are here making distinctions between natural-born citizens.
Because this is very important for certain government positions, ‘no, because natural-born citizens are only qualified for a
specific…

THE CHAIRMAN (SEN. DRILON). That is correct.

REP. JAVIER. ...positions under the Constitution and under the law.

THE CHAIRMAN (SEN. DRILON). Yes. We can get to that later on. It’s one of the provisions, yes. But just for purposes of
the explanation, Congressman Javier, that is our conceptualization. Reacquired for those who previously lost
[Filipino citizenship] by virtue of Commonwealth Act 63, and retention for those in the future. (Emphasis supplied)

Considering that petitioner was naturalized as a Canadian citizen prior to the effectivity of R.A. 9225, he belongs to the
first category of natural- born Filipinos under the first paragraph of Section 3 who lost Philippine citizenship by
naturalization in a foreign country. As the new law allows dual citizenship, he was able to re-acquire his Philippine
citizenship by taking the required oath of allegiance.

For the purpose of determining the citizenship of petitioner at the time of filing his MLA, it is not necessary to discuss the
rulings in Frivaldo and Altarejos on the retroactivity of such reacquisition because R.A. 9225 itself treats those of his
category as having already lost Philippine citizenship, in contradistinction to those natural-born Filipinos who became
foreign citizens after R.A. 9225 came into force. In other words, Section 2 declaring the policy that considers Filipinos who
became foreign citizens as not to have lost their Philippine citizenship, should be read together with Section 3, the second
paragraph of which clarifies that such policy governs all cases after the new law’s effectivity.

As to the letter-reply of BI, it simply quoted Section 2 of R.A. 9225 without any reference to Section 3 on the particular
application of reacquisition and retention to Filipinos who became foreign citizens before and after the effectivity of R.A.
9225.

Petitioner’s plea to adopt the interpretation most favorable to the accused is likewise misplaced. Courts adopt an
interpretation more favorable to the accused following the time-honored principle that penal statutes are construed strictly
against the State and liberally in favor of the accused.  R.A. 9225, however, is not a penal law.
23

Falsification of documents under paragraph 1, Article 172  in relation to Article 171  of the RPC refers to falsification by a
24 25

private individual, or a public officer or employee who did not take advantage of his official position, of public, private, or
commercial documents. The elements of falsification of documents under paragraph 1, Article 172 of the RPC are:

(1)that the offender is a private individual or a public officer or employee who did not take advantage of his official
position;

(2)that he committed any of the acts of falsification enumerated in Article 171 of the RPC; and

(3)that the falsification was committed in a public, official or commercial document. 26

Petitioner made the untruthful statement in the MLA, a public document, that he is a Filipino citizen at the time of the filing
of said application, when in fact he was then still a Canadian citizen. Under CA 63, the governing law at the time he was
naturalized as Canadian citizen, naturalization in a foreign country was among those ways by which a natural-born citizen
loses his Philippine citizenship. While he re-acquired Philippine citizenship under R.A. 9225 six months later, the
falsification was already a consummated act, the said law having no retroactive effect insofar as his dual citizenship status
is concerned. The MTC therefore did not err in finding probable cause for falsification of public document under Article
172, paragraph 1.

The MTC further cited lack of jurisdiction over the person of petitioner accused as ground for denying petitioner’s motion
for re- determination of probable cause, as the motion was filed prior to his arrest. However, custody of the law is not
required for the adjudication of reliefs other than an application for bail.  In Miranda v. Tuliao,  which involved a motion to
27 28

quash warrant of arrest, this Court discussed the distinction between custody of the law and jurisdiction over the person,
and held that jurisdiction over the person of the accused is deemed waived when he files any pleading seeking an
affirmative relief, except in cases when he invokes the special jurisdiction of the court by impugning such jurisdiction over
his person. Thus:
In arguing, on the other hand, that jurisdiction over their person was already acquired by their filing of the above Urgent
Motion, petitioners invoke our pronouncement, through Justice Florenz D. Regalado, in Santiago v. Vasquez:

The voluntary appearance of the accused, whereby the court acquires jurisdiction over his person, is accomplished either
by his pleading to the merits (such as by filing a motion to quash or other pleadings requiring the exercise of the court’s
jurisdiction thereover, appearing for arraignment, entering trial) or by filing bail. On the matter of bail, since the same is
intended to obtain the provisional liberty of the accused, as a rule the same cannot be posted before custody of the
accused has been acquired by the judicial authorities either by his arrest or voluntary surrender.

Our pronouncement in Santiago shows a distinction between custody of the law and jurisdiction over the person. Custody
of the law is required before the court can act upon the application for bail, but is not required for the adjudication of other
reliefs sought by the defendant where the mere application therefor constitutes a waiver of the defense of lack of
jurisdiction over the person of the accused. Custody of the law is accomplished either by arrest or voluntary surrender,
while jurisdiction over the person of the accused is acquired upon his arrest or voluntary appearance. One can be under
the custody of the law but not yet subject to the jurisdiction of the court over his person, such as when a person arrested
by virtue of a warrant files a motion before arraignment to quash the warrant. On the other hand, one can be subject to the
jurisdiction of the court over his person, and yet not be in the custody of the law, such as when an accused escapes
custody after his trial has commenced. Being in the custody of the law signifies restraint on the person, who is thereby
deprived of his own will and liberty, binding him to become obedient to the will of the law. Custody of the law is literally
custody over the body of the accused. It includes, but is not limited to, detention.

xxxx

While we stand by our above pronouncement in Pico insofar as it concerns bail, we clarify that, as a general rule, one
who seeks an affirmative relief is deemed to have submitted to the jurisdiction of the court. As we held in the
aforecited case of Santiago, seeking an affirmative relief in court, whether in civil or criminal proceedings,
constitutes voluntary appearance.

xxxx

To recapitulate what we have discussed so far, in criminal cases, jurisdiction over the person of the accused is
deemed waived by the accused when he files any pleading seeking an affirmative relief, except in cases when he
invokes the special jurisdiction of the court by impugning such jurisdiction over his person. Therefore, in narrow
cases involving special appearances, an accused can invoke the processes of the court even though there is neither
jurisdiction over the person nor custody of the law. However, if a person invoking the special jurisdiction of the court
applies for bail, he must first submit himself to the custody of the law.  (Emphasis supplied)
29

Considering that petitioner sought affirmative relief in filing his motion for re-determination of probable cause, the MTC
clearly erred in stating that it lacked jurisdiction over his person. Notwithstanding such erroneous ground stated in the
MTC's order, the RTC correctly ruled that no grave abuse of discretion was committed by the MTC in denying the said
motion for lack of merit.

WHEREFORE, the petition is DENIED. The Order dated October 8, 2011 of the Regional Trial Court of Pinamalayan,
Oriental Mindoro in Civil Case No. SCA-07-11 (Criminal Case No. 2012) is hereby AFFIRMED and UPHELD.

With costs against the petitioner.

SO ORDERED.

Who are citizens of the Philippines? Article IV, Section 1


those who elect Philippine citizenship pursuant to the 1935 Constitution
-Republic v Sagun, 666 SCRA 321 (2012)

Before us is a petition for review on certiorari filed by the Solicitor General on behalf of the Republic of the Philippines,
seeking the reversal of the April 3, 2009 Decision of the Regional Trial Court (RTC), Branch 3, of Baguio City in Spcl. Pro.

Case No. 17-R. The RTC granted the petition filed by respondent Nora Fe Sagun entitled "In re: Judicial Declaration of

Election of Filipino Citizenship, Nora Fe Sagun v. The Local Civil Registrar of Baguio City."

The facts follow:

Respondent is the legitimate child of Albert S. Chan, a Chinese national, and Marta Borromeo, a Filipino citizen. She was
born on August 8, 1959 in Baguio City and did not elect Philippine citizenship upon reaching the age of majority. In 1992,

at the age of 33 and after getting married to Alex Sagun, she executed an Oath of Allegiance to the Republic of the

Philippines. Said document was notarized by Atty. Cristeta Leung on December 17, 1992, but was not recorded and
registered with the Local Civil Registrar of Baguio City.

Sometime in September 2005, respondent applied for a Philippine passport. Her application was denied due to the
citizenship of her father and there being no annotation on her birth certificate that she has elected Philippine citizenship.
Consequently, she sought a judicial declaration of her election of Philippine citizenship and prayed that the Local Civil
Registrar of Baguio City be ordered to annotate the same on her birth certificate.

In her petition, respondent averred that she was raised as a Filipino, speaks Ilocano and Tagalog fluently and attended
local schools in Baguio City, including Holy Family Academy and the Saint Louis University. Respondent claimed that
despite her part-Chinese ancestry, she always thought of herself as a Filipino. She is a registered voter of Precinct No.
0419A of Barangay Manuel A. Roxas in Baguio City and had voted in local and national elections as shown in the Voter
Certification issued by Atty. Maribelle Uminga of the Commission on Elections of Baguio City.

She asserted that by virtue of her positive acts, she has effectively elected Philippine citizenship and such fact should be
annotated on her record of birth so as to entitle her to the issuance of a Philippine passport.

On August 7, 2007, the Office of the Solicitor General (OSG) entered its appearance as counsel for the Republic of the
Philippines and authorized the City Prosecutor of Baguio City to appear in the above mentioned case. However, no

comment was filed by the City Prosecutor.

After conducting a hearing, the trial court rendered the assailed Decision on April 3, 2009 granting the petition and
declaring respondent a Filipino citizen. The fallo of the decision reads:

WHEREFORE, the instant petition is hereby GRANTED. Petitioner Nora Fe Sagun y Chan is hereby DECLARED [a]
FILIPINO CITIZEN, having chosen or elected Filipino citizenship.

Upon payment of the required fees, the Local Civil Registrar of Baguio City is hereby directed to annotate [on] her birth
certificate, this judicial declaration of Filipino citizenship of said petitioner.

IT IS SO ORDERED. 7

Contending that the lower court erred in so ruling, petitioner, through the OSG, directly filed the instant recourse via a
petition for review on certiorari before us. Petitioner raises the following issues:

Whether or not an action or proceeding for judicial declaration of Philippine citizenship is procedurally and
jurisdictionally permissible; and,

II

Whether or not an election of Philippine citizenship, made twelve (12) years after reaching the age of majority, is
considered to have been made "within a reasonable time" as interpreted by jurisprudence. 8

Petitioner argues that respondent’s petition before the RTC was improper on two counts: for one, law and jurisprudence
clearly contemplate no judicial action or proceeding for the declaration of Philippine citizenship; and for another, the
pleaded registration of the oath of allegiance with the local civil registry and its annotation on respondent’s birth certificate
are the ministerial duties of the registrar; hence, they require no court order. Petitioner asserts that respondent’s petition
before the trial court seeking a judicial declaration of her election of Philippine citizenship undeniably entails a
determination and consequent declaration of her status as a Filipino citizen which is not allowed under our legal system.
Petitioner also argues that if respondent’s intention in filing the petition is ultimately to have her oath of allegiance
registered with the local civil registry and annotated on her birth certificate, then she does not have to resort to court
proceedings.

Petitioner further argues that even assuming that respondent’s action is sanctioned, the trial court erred in finding
respondent as having duly elected Philippine citizenship since her purported election was not in accordance with the
procedure prescribed by law and was not made within a "reasonable time." Petitioner points out that while respondent
executed an oath of allegiance before a notary public, there was no affidavit of her election of Philippine citizenship.
Additionally, her oath of allegiance which was not registered with the nearest local civil registry was executed when she
was already 33 years old or 12 years after she reached the age of majority. Accordingly, it was made beyond the period
allowed by law.

In her Comment, respondent avers that notwithstanding her failure to formally elect Filipino citizenship upon reaching the

age of majority, she has in fact effectively elected Filipino citizenship by her performance of positive acts, among which is
the exercise of the right of suffrage. She claims that she had voted and participated in all local and national elections from
the time she was of legal age. She also insists that she is a Filipino citizen despite the fact that her "election" of Philippine
citizenship was delayed and unregistered.

In reply, petitioner argues that the special circumstances invoked by respondent, like her continuous and uninterrupted
10 

stay in the Philippines, her having been educated in schools in the country, her choice of staying here despite the
naturalization of her parents as American citizens, and her being a registered voter, cannot confer on her Philippine
citizenship as the law specifically provides the requirements for acquisition of Philippine citizenship by election.
Essentially, the issues for our resolution are: (1) whether respondent’s petition for declaration of election of Philippine
citizenship is sanctioned by the Rules of Court and jurisprudence; (2) whether respondent has effectively elected
Philippine citizenship in accordance with the procedure prescribed by law.

The petition is meritorious.

At the outset, it is necessary to stress that a direct recourse to this Court from the decisions, final resolutions and orders of
the RTC may be taken where only questions of law are raised or involved. There is a question of law when the doubt or
difference arises as to what the law is on a certain state of facts, which does not call for an examination of the probative
value of the evidence presented by the parties-litigants. On the other hand, there is a question of fact when the doubt or
controversy arises as to the truth or falsity of the alleged facts. Simply put, when there is no dispute as to fact, the question
of whether the conclusion drawn therefrom is correct or not, is a question of law. 11

In the present case, petitioner assails the propriety of the decision of the trial court declaring respondent a Filipino citizen
after finding that respondent was able to substantiate her election of Filipino citizenship. Petitioner contends that
respondent’s petition for judicial declaration of election of Philippine citizenship is procedurally and jurisdictionally
impermissible. Verily, petitioner has raised questions of law as the resolution of these issues rest solely on what the law
provides given the attendant circumstances.

In granting the petition, the trial court stated:

This Court believes that petitioner was able to fully substantiate her petition regarding her election of Filipino citizenship,
and the Local Civil Registrar of Baguio City should be ordered to annotate in her birth certificate her election of Filipino
citizenship. This Court adds that the petitioner’s election of Filipino citizenship should be welcomed by this country and
people because the petitioner has the choice to elect citizenship of powerful countries like the United States of America
and China, however, petitioner has chosen Filipino citizenship because she grew up in this country, and has learned to
love the Philippines. Her choice of electing Filipino citizenship is, in fact, a testimony that many of our people still wish to
live in the Philippines, and are very proud of our country.

WHEREFORE, the instant petition is hereby GRANTED. Petitioner Nora Fe Sagun y Chan is hereby DECLARED as
FILIPINO CITIZEN, having chosen or elected Filipino citizenship. 12

For sure, this Court has consistently ruled that there is no proceeding established by law, or the Rules for the judicial
declaration of the citizenship of an individual. There is no specific legislation authorizing the institution of a judicial
13 

proceeding to declare that a given person is part of our citizenry. This was our ruling in Yung Uan Chu v. Republic citing
14  15 

the early case of Tan v. Republic of the Philippines, where we clearly stated:


16 

Under our laws, there can be no action or proceeding for the judicial declaration of the citizenship of an individual. Courts
of justice exist for settlement of justiciable controversies, which imply a given right, legally demandable and enforceable,
an act or omission violative of said right, and a remedy, granted or sanctioned by law, for said breach of right. As an
incident only of the adjudication of the rights of the parties to a controversy, the court may pass upon, and make a
pronouncement relative to their status. Otherwise, such a pronouncement is beyond judicial power. x x x

Clearly, it was erroneous for the trial court to make a specific declaration of respondent’s Filipino citizenship as such
pronouncement was not within the court’s competence.

As to the propriety of respondent’s petition seeking a judicial declaration of election of Philippine citizenship, it is
imperative that we determine whether respondent is required under the law to make an election and if so, whether she has
complied with the procedural requirements in the election of Philippine citizenship.

When respondent was born on August 8, 1959, the governing charter was the 1935 Constitution, which declares as
citizens of the Philippines those whose mothers are citizens of the Philippines and elect Philippine citizenship upon
reaching the age of majority. Sec. 1, Art. IV of the 1935 Constitution reads:

Section 1. The following are citizens of the Philippines:

xxxx

(4) Those whose mothers are citizens of the Philippines and, upon reaching the age of majority, elect Philippine
citizenship.

Under Article IV, Section 1(4) of the 1935 Constitution, the citizenship of a legitimate child born of a Filipino mother and an
alien father followed the citizenship of the father, unless, upon reaching the age of majority, the child elected Philippine
citizenship. The right to elect Philippine citizenship was recognized in the 1973 Constitution when it provided that "[t]hose
who elect Philippine citizenship pursuant to the provisions of the Constitution of nineteen hundred and thirty-five" are
citizens of the Philippines. Likewise, this recognition by the 1973 Constitution was carried over to the 1987
17 

Constitution which states that "[t]hose born before January 17, 1973 of Filipino mothers, who elect Philippine citizenship
upon reaching the age of majority" are Philippine citizens. It should be noted, however, that the 1973 and 1987
18 

Constitutional provisions on the election of Philippine citizenship should not be understood as having a curative effect on
any irregularity in the acquisition of citizenship for those covered by the 1935 Constitution. If the citizenship of a person
was subject to challenge under the old charter, it remains subject to challenge under the new charter even if the judicial
challenge had not been commenced before the effectivity of the new Constitution. 19

Being a legitimate child, respondent’s citizenship followed that of her father who is Chinese, unless upon reaching the age
of majority, she elects Philippine citizenship. It is a settled rule that only legitimate children follow the citizenship of the
father and that illegitimate children are under the parental authority of the mother and follow her nationality. An illegitimate
20 

child of Filipina need not perform any act to confer upon him all the rights and privileges attached to citizens of the
Philippines; he automatically becomes a citizen himself. But in the case of respondent, for her to be considered a Filipino
21 

citizen, she must have validly elected Philippine citizenship upon reaching the age of majority.

Commonwealth Act (C.A.) No. 625, enacted pursuant to Section 1(4), Article IV of the 1935 Constitution, prescribes the
22 

procedure that should be followed in order to make a valid election of Philippine citizenship, to wit:

Section 1. The option to elect Philippine citizenship in accordance with subsection (4), [S]ection 1, Article IV, of the
Constitution shall be expressed in a statement to be signed and sworn to by the party concerned before any officer
authorized to administer oaths, and shall be filed with the nearest civil registry. The said party shall accompany the
aforesaid statement with the oath of allegiance to the Constitution and the Government of the Philippines.

Based on the foregoing, the statutory formalities of electing Philippine citizenship are: (1) a statement of election under
oath; (2) an oath of allegiance to the Constitution and Government of the Philippines; and (3) registration of the statement
of election and of the oath with the nearest civil registry.23

Furthermore, no election of Philippine citizenship shall be accepted for registration under C.A. No. 625 unless the party
exercising the right of election has complied with the requirements of the Alien Registration Act of 1950. In other words, he
should first be required to register as an alien. Pertinently, the person electing Philippine citizenship is required to file a
24 

petition with the Commission of Immigration and Deportation (now Bureau of Immigration) for the cancellation of his alien
certificate of registration based on his aforesaid election of Philippine citizenship and said Office will initially decide, based
on the evidence presented the validity or invalidity of said election. Afterwards, the same is elevated to the Ministry (now
25 

Department) of Justice for final determination and review. 26 


1âwphi1

It should be stressed that there is no specific statutory or procedural rule which authorizes the direct filing of a petition for
declaration of election of Philippine citizenship before the courts. The special proceeding provided under Section 2, Rule
108 of the Rules of Court on Cancellation or Correction of Entries in the Civil Registry, merely allows any interested party
to file an action for cancellation or correction of entry in the civil registry, i.e., election, loss and recovery of citizenship,
which is not the relief prayed for by the respondent.

Be that as it may, even if we set aside this procedural infirmity, still the trial court’s conclusion that respondent duly elected
Philippine citizenship is erroneous since the records undisputably show that respondent failed to comply with the legal
requirements for a valid election. Specifically, respondent had not executed a sworn statement of her election of Philippine
citizenship. The only documentary evidence submitted by respondent in support of her claim of alleged election was her
oath of allegiance, executed 12 years after she reached the age of majority, which was unregistered. As aptly pointed out
by the petitioner, even assuming arguendo that respondent’s oath of allegiance suffices, its execution was not within a
reasonable time after respondent attained the age of majority and was not registered with the nearest civil registry as
required under Section 1 of C.A. No. 625. The phrase "reasonable time" has been interpreted to mean that the election
should be made generally within three (3) years from reaching the age of majority. Moreover, there was no satisfactory
27 

explanation proffered by respondent for the delay and the failure to register with the nearest local civil registry.

Based on the foregoing circumstances, respondent clearly failed to comply with the procedural requirements for a valid
and effective election of Philippine citizenship. Respondent cannot assert that the exercise of suffrage and the
participation in election exercises constitutes a positive act of election of Philippine citizenship since the law specifically
lays down the requirements for acquisition of citizenship by election. The mere exercise of suffrage, continuous and
uninterrupted stay in the Philippines, and other similar acts showing exercise of Philippine citizenship cannot take the
place of election of Philippine citizenship. Hence, respondent cannot now be allowed to seek the intervention of the court
to confer upon her Philippine citizenship when clearly she has failed to validly elect Philippine citizenship. As we held
in Ching, the prescribed procedure in electing Philippine citizenship is certainly not a tedious and painstaking process. All
28 

that is required of the elector is to execute an affidavit of election of Philippine citizenship and, thereafter, file the same
with the nearest civil registry. Having failed to comply with the foregoing requirements, respondent’s petition before the
trial court must be denied.

WHEREFORE, the petition is GRANTED. The Decision dated April 3, 2009 of the Regional Trial Court, Branch 3 of
Baguio City in Spcl. Pro. Case No. 17-R is REVERSED and SET ASIDE. The petition for judicial declaration of election of
Philippine citizenship filed by respondent Nora Fe Sagun is hereby DISMISSED for lack of merit. No costs. SO
ORDERED.

Foundling
- David vs Poe-Llamanzares, GR 221538, Sep 20, 2016

The words of our most fundamental law cannot be read so as to callously exclude all foundlings from public
service.

When the names of the parents of a foundling cannot be discovered despite a diligent search, but sufficient
evidence is presented to sustain a reasonable inference that satisfies the quantum of proof required to
conclude that at least one or both of his or her parents is Filipino, then this should be sufficient to establish
that he or she is a natural-born citizen. When these inferences are made by the Senate Electoral Tribunal
in the exercise of its sole and exclusive prerogative to decide the qualifications of the members of the
Senate, then there is no grave abuse of discretion remediable by either Rule 65 of the Rules of Court or
Article VIII, Section I of the Constitution.

This case certainly does not decide with finality the citizenship of every single foundling as natural-born.
The circumstances of each case are unique, and substantial proof may exist to show that a foundling is not
natural-born. The nature of the Senate Electoral Tribunal and its place in the scheme of political powers,
as devised by the Constitution, are likewise different from the other ways to raise questions of citizenship.

Before this Court is a Petition for Certiorari 1 filed by petitioner Rizalito Y. David (David). He prays for the
nullification of the assailed November 17, 2015 Decision and December 3, 2015 Resolution of public
respondent Senate Electoral Tribunal in SET Case No. 001-15. 2 The assailed November 17, 2015
Decision3 dismissed the Petition for Quo Warranto filed by David, which sought to unseat private
respondent Mary Grace Poe-Llamanzares as a Senator for allegedly not being a natural-born citizen of the
Philippines and, therefore, not being qualified to hold such office under Article VI, Section 3 4 of the 1987
Constitution. The assailed December 3, 2015 Resolution 5 denied David's Motion for Reconsideration.

Senator Mary Grace Poe-Llamanzares (Senator Poe) is a foundling whose biological parents are unknown.
As an infant, she was abandoned at the Parish Church of Jaro, Iloilo. 6 Edgardo Militar found her outside the
church on September 3, 1968 at about 9:30 a.m. 7 He later turned her over to Mr. and Mrs. Emiliano
Militar.8 Emiliano Militar reported to the Office of the Local Civil Registrar that the infant was found on
September 6, 1968.9 She was given the name Mary Grace Natividad Contreras Militar. 10 Local Civil
Registrar issued a Certificate of Live Birth/Foundling Certificate stating: ChanRoblesVirtualawlibrary

Circumstances: THE SUBJECT CHILD WAS FOUND IN THE PARISH CHURCHD [sic] OF JARO, ON
SEPTEMBER 3, 1968 AT ABOUT 9:30 A.M. BY EDGARDO MILITAR AND THE SAID CHILD IS PRESENTLY IN
THE CUSTODY OF MR. AND MRS. EMILIANO MILITAR AT STA. ISABEL STREET, JARO . . . 11 chanroblesvirtuallawlibrary

On May 13, 1974, the Municipal Court of San Juan, Rizal promulgated the Decision granting the Petition for
Adoption of Senator Poe by Spouses Ronald Allan Poe (more popularly known as Fernando Poe, Jr.) and
Jesusa Sonora Poe (more popularly known as Susan Roces). 12 The Decision also ordered the change in
Senator Poe's name from Mary Grace Natividad Contreras Militar to Mary Grace Natividad Sonora
Poe.13 October 27, 2005, Clerk of Court III Eleanor A. Sorio certified that the Decision had become final in
a Certificate of Finality.14 chanrobleslaw

On April 11, 1980, the Office of Civil Registrar-Iloilo received the Decision of the San Juan Court Municipal
Court and noted on Senator Poe's foundling certificate that she was adopted by Spouses Ronald Allan and
Jesusa Poe.15 This hand-written notation appears on Senator Poe's foundling certificate: ChanRoblesVirtualawlibrary

NOTE: Adopted child by the Spouses Ronald Allan Poe and Jesusa Sonora Poe as per Court Order, Mun.
Court, San Juan, Rizal, by Hon. Judge Alfredo M. Gorgonio dated May 13, 1974, under Sp. Proc. No.
138.16chanroblesvirtuallawlibrary

Senator Poe became a registered voter in Greenhills, San Juan, Metro Manila when she turned 18 years
old.17 The Commission on Elections issued her a Voter's Identification Card for Precinct No. 196,
Greenhills, San Juan, Metro Manila on December 13, 1986. 18 chanrobleslaw

On April 4, 1988, the Department of Foreign Affairs issued her a Philippine passport. 19 Her passport was
renewed on April 5, 1993, May 19, 1998, October 13, 2009, December 19, 2013, and March 18,
2014.20 Having become Senator, she was also issued a Philippine diplomatic passport on December 19,
2013.21 chanrobleslaw

Senator Poe took Development Studies at the University of the Philippines, Manila, but eventually went to
the United States in 1988 to obtain her college degree. 22 In 1991, she earned a bachelor's degree in
Political Science from Boston College, Chestnut Hill, Massachusetts. 23 chanrobleslaw

On July 27, 1991, Senator Poe married Teodoro Misael Daniel V. Llamanzares, both an American and
Filipino national since birth.24 The marriage took place in Sanctuario de San Jose Parish, San Juan,
Manila.25  On July 29, 1991, Senator Poe returned to the United States with her husband. 26 For some time,
cralawred

she lived with her husband and children in the United States. 27 chanrobleslaw

Senator Poe and her husband had three (3) children: Brian Daniel (Brian), Hanna MacKenzie (Hanna), and
Jesusa Anika (Anika).28 Brian was born in the United States on April 16, 1992. Hanna was born on July 10,
1998, and Anika on June 5, 2004. Both Hanna and Anika were born in the Philippines. 29 chanrobleslaw

Senator Poe was naturalized and granted American citizenship on October 18, 2001. 30 She was
subsequently given a United States passport. 31 chanrobleslaw

Senator Poe's adoptive father, Fernando Poe, Jr., ran for President of the Republic of the Philippines in the
2004 National Elections.32 To support her father's candidacy, Senator Poe and her daughter Hanna
returned to the Philippines on April 8, 2004.33 After the Elections, she returned to the United States on July
8, 2004.34 It was during her stay in the Philippines that she gave birth to her youngest daughter, Anika. 35 chanrobleslaw

Fernando Poe, Jr. was hospitalized on December 11, 2004 and eventually "slipped into a coma." 36 Senator
Poe returned to the Philippines on December 13, 2004.37 On December 14, 2004, her father died.38 She
stayed in the country until February 3, 2005 to attend her father's funeral and to attend to the settling of
his estate.39
chanrobleslaw

In 2004, Senator Poe resigned from work in the United States. She never looked for work again in the
United States.40 chanrobleslaw

Senator Poe decided to return home in 2005.41 After consulting her children, they all agreed to return to
the Philippines to support the grieving Susan Roces. 42 In early 2005, they notified Brian and Hanna's
schools Virginia, United States that they would be transferring to the Philippines the following
semester.43 She came back on May 24, 2005.44 Her children also arrived in the first half of
2005.45 However, her husband stayed in the United States to "finish pending projects, and to arrange for
the sale of the family home there."46 chanrobleslaw

Following her return, Senator Poe was issued by the Bureau of Internal Revenue a Tax Identification
Number (TIN) on July 22, 2005.47 chanrobleslaw

On July 7, 2006, Senator Poe took the Oath of Allegiance to Republic of the Philippines: 48

I, Mary Grace Poe Llamanzares, solemnly swear that I will support and defend the Constitution of the
Republic of the Philippines and obey the laws and legal orders promulgated by the duly constituted
authorities of the Philippines; and I hereby declare that I recognize and accept the supreme authority of
the Philippines and will maintain true faith and allegiance thereto; and that I impose this obligation upon
myself voluntarily without mental reservation or purpose of evasion. 49 chanroblesvirtuallawlibrary

On July 10, 2006, Senator Poe filed a Petition for Retention and or Re-acquisition of Philippine Citizenship
through Republic Act No. 9225.50 She also "filed applications for derivative citizenship on behalf of her
three children who were all below eighteen (18) years of age at that time." 51 chanrobleslaw

The Petition was granted by the Bureau of Immigration and Deportation on July 18, 2006 through an
Order signed by Associate Commissioner Roy M. Almoro for Commissioner Alipio F. Fernandez, Jr: 52

A careful review of the documents submitted in support of the instant petition indicate that David was a
former citizen of the Republic of the Philippines being born to Filipino parents and is presumed to be a
natural born Philippine citizen; thereafter, became an American citizen and is now a holder of an American
passport; was issued an ACT and ICR and has taken her oath of allegiance to the Republic of the
Philippines on July 7, 2006 and so is thereby deemed to have re-acquired her Philippine
Citizenship.53 (Emphasis in the original)
In the same Order, Senator Poe's children were "deemed Citizens of the Philippines in accordance with
Section 4 of R[epublic] A[ct] No. 9225."54 Until now, the Order "has not been set aside by the Department
of Justice or any other agency of Government." 55 chanrobleslaw

On July 31, 2006, the Bureau of Immigration issued Identification Certificates in the name of Senator Poe
and her children.56 It stated that Senator Poe is a "citizen of the Philippines pursuant to the Citizenship
Retention and Re-acquisition Act of 2003 . . . in relation to Administrative Order No. 91, Series of 2004
and Memorandum Circular No. AFF-2-005 per Office Order No. AFF-06-9133 signed Associate
Commissioner Roy M. Almoro dated July 18, 2006."57 chanrobleslaw

Senator Poe became a registered voter of Barangay Santa Lucia, San Juan City on August 31, 2006. 58 chanrobleslaw

Senator Poe made several trips to the United States of America between 2006 and 2009 using her United
States Passport No. 170377935.59 She used her passport "after having taken her Oath of Allegiance to the
Republic on 07 July 2006, but not after she has formally renounced her American citizenship on 20
October 2010."60 The following are the flight records given by the Bureau of Immigration: ChanRoblesVirtualawlibrary

Departures Flight No.  


November 1, 2006 SQ071  
July 20, 2007 PR730  
October 31, 2007 PR300  
October 2, 2008 PR358  
April 20, 2009 PR104  
July 31, 2009 PR730  
October 19, 2009 PR102  
November 15, 2009 PR103  
December 27, 2009 PR112  
March 27, 2010 PR102  
   
Arrivals Flight No.  
November 4, 2006 SQ076  
July 23, 2007 PR731  
November 5, 2007 PR337  
May 8, 2008 PR103  
October 5, 2008 PR359  
May 21, 2009 PR105  
August 3, 2009 PR733  
November 15, 2009 PR10361  
On October 6, 2010, President Benigno Simeon Aquino III appointed Senator Poe as Chairperson of the
Movie and Television Review and Classification Board (MTRCB). 62 On October 20, 2010, Senator Poe
executed an Affidavit of Renunciation of Allegiance to the United States of America and Renunciation of
American Citizenship,63 stating:

I, MARY GRACE POE-LLAMANZARES, Filipino, of legal age, and presently residing at No. 107 Rodeo
chanRoblesvirtualLawlibrary

Drive, Corinthian Hills, Quezon City, Philippines, after having been duly sworn to in accordance with the
law, do hereby depose and state that with this affidavit, I hereby expressly and voluntarily renounce my
United States nationality/American citizenship, together with all rights and privileges and all duties and
allegiance and fidelity thereunto pertaining. I make this renunciation intentionally, voluntarily, and of my
own free will, free of any duress or undue influence. 64 (Emphasis in the original)

The affidavit was submitted to the Bureau of Immigration on October 21, 2010. 65 On October 21, 2010,
she took her Oath of Office as MTRCB Chairperson and assumed office on October 26, 2010. 66 Her oath of
office stated: ChanRoblesVirtualawlibrary

PANUNUMPA SA KATUNGKULAN

Ako, si MARY GRACE POE LLAMANZARES, na itinalaga sa katungkulan bilang Chairperson, Movie and
Television Review and Classification Board, ay taimtim na nanunumpa na tutuparin ko nang buong husay
at katapatan, sa abot ng aking kakayahan, ang mga tungkulin ng aking kasalukuyang katungkulan at ng
mga iba pang pagkaraan nito'y gagampanan ko sa ilalim ng Republika ng Pilipinas; na  aking itataguyod at
ipagtatanggol ang Saligan Batas ng Pilipinas; na tunay na mananalig at tatalima ako rito; na susundin ko
ang mga batas, mga kautusang lega, at mga dekretong pinaiiral ng mga sadyang itinakdang may
kapangyarihan ng Republika ng Pilipinas; at kusa kong babalikatin ang pananagutang ito, nang walang ano
mang pasubali o hangaring umiwas.

Kasihan nawa ako ng Diyos.

NILAGDAAN AT PINANUMPAAN sa harap ko ngayong ika-21 ng Oktubre 2010, Lungsod ng Maynila,


Pilipinas.67 (Emphasis in the original)
Senator Poe executed an Oath/Affirmation of Renunciation of Nationality of the United States 68 in the
presence of Vice-Consul Somer E. Bessire-Briers on July 12, 2011. 69 On this occasion, she also filled out
the Questionnaire Information for Determining Possible Loss of U.S. Citizenship. 70 On December 9, 2011,
Vice Consul Jason Galian executed a Certificate of Loss of Nationality for Senator Poe. 71 The certificate was
approved by the Overseas Citizen Service, Department of State, on February 3, 2012. 72 chanrobleslaw
Senator Poe decided to run as Senator in the 2013 Elections. 73 On September 27, 2012, she executed a
Certificate of Candidacy, which was submitted to the Commission on Elections on October 2, 2012. 74 She
won and was declared as Senator-elect on May 16, 2013. 75 chanrobleslaw

David, a losing candidate in the 2013 Senatorial Elections, filed before the Senate Electoral Tribunal a
Petition for Quo Warranto on August 6, 2015.76 He contested the election of Senator Poe for failing to
"comply with the citizenship and residency requirements mandated by the 1987 Constitution." 77 chanrobleslaw

Thereafter, the Senate Electoral Tribunal issued Resolution No. 15-01 requiring David "to correct the
formal defects of his petition."78 David filed his amended Petition on August 17, 2015. 79 chanrobleslaw

On August 18, 2015, Resolution No. 15-02 was issued by the Senate Electoral Tribunal, through its
Executive Committee, ordering the Secretary of the Senate Electoral Tribunal to summon Senator Poe to
file an answer to the amended Petition.80 chanrobleslaw

Pending the filing of Senator Poe's answer, David filed a Motion Subpoena the Record of Application of
Citizenship Re-acquisition and related documents from the Bureau of Immigration on August 25,
2015.81 The documents requested included Senator Poe's record of travels and NSO kept Birth
Certificate.82 On August 26, 2015, the Senate Electoral Tribunal issued Resolution No. 15-04 granting the
Motion.83 The same Resolution directed the Secretary of the Tribunal to issue a subpoena to the concerned
officials of the Bureau of Immigration and the National Statistics Office. 84 The subpoenas ordered the
officials to appear on September 1, 2015 at 10:00 a.m. before the Office of the Secretary of the Senate
bearing three (3) sets of the requested documents. 85 The subpoenas were complied with by both the
Bureau of Immigration and the National Statistics Office on September 1, 2015. 86 chanrobleslaw

On September 1, 2015, Senator Poe submitted her Verified Answer with (1) Prayer for Summary
Dismissal; (2) Motion for Preliminary Hearing on Grounds for Immediate Dismissal/Affirmative Defenses;
(3) Motion to Cite David for Direct Contempt of Court; and (4) Counterclaim for Indirect Contempt of
Court.87 chanrobleslaw

On September 2, 2015, the Senate Electoral Tribunal issued Resolution No. 15-05 requiring the parties to
file a preliminary conference brief on or before September 9, 2015. 88 The Resolution also set the
Preliminary Conference on September 11, 2015. 89 During the Preliminary Conference, the parties "agreed
to drop the issue of residency on the ground of prescription." 90 chanrobleslaw

Oral arguments were held by the Senate Electoral Tribunal on September 21, 2015. 91 The parties were
then "required to submit their respective [memoranda], without prejudice to the submission of DNA
evidence by [Senator Poe] within thirty (30) days from the said date." 92 chanrobleslaw

On October 21, 2015, Senator Poe moved to extend for 15 days the submission of DNA test results. 93 The
Senate Electoral Tribunal granted the Motion on October 27, 2015 through Resolution No. 15-08. 94 On
November 5, 2015, Senator Poe filed a Manifestation regarding the results of DNA Testing, 95 which stated
that "none of the tests that [Senator Poe] took provided results that would shed light to the real identity of
her biological parents."96 The Manifestation also stated that Senator Poe was to continue to find closure
regarding the issue and submit any development to the Senate Electoral Tribunal. Later, Senator Poe
submitted "the issue of her natural-born Filipino citizenship as a foundling for resolution upon the legal
arguments set forth in her submissions to the Tribunal." 97 On November 6, 2015, through Resolution No.
15-10, the Senate Electoral Tribunal "noted the [M]anifestation and considered the case submitted for
resolution."98 chanrobleslaw

On November 17, 2015, the Senate Electoral Tribunal promulgated its assailed Decision finding Senator
Poe to be a natural-born citizen and, therefore, qualified to hold office as Senator. 99 The Decision
stated:ChanRoblesVirtualawlibrary

We rule that Respondent is a natural-born citizen under the 1935 Constitution and continue to be a
natural-born citizen as defined under the 1987 Constitution, as she is a citizen of the Philippines from
birth, without having to perform any act to acquire or perfect (her) Philippine citizenship.

. . . .

In light of our earlier pronouncement that Respondent is a natural-born Filipino citizen, Respondent validly
reacquired her natural-born Filipino citizenship upon taking her Oath of Allegiance to the Republic of the
Philippines, as required under Section 3 of R.A. No. 9225.

Under Section 11 of B.I. Memorandum Circular No. AFF 05-002 (the Revised Rules Implementing R.A. No.
9225), the foregoing Oath of Allegiance is the "final act" to reacquire natural-born Philippine citizenship.

. . . .

To repeat, Respondent never used her USA passport from the moment she renounced her American
citizenship on 20 October 2010. She remained solely a natural-born Filipino citizen from that time on until
today.

WHEREFORE, in view of the foregoing, the petition for quo warranto is DISMISSED.

No pronouncement as to costs.

SO ORDERED.100 (Citations omitted)
On November 23, 2015, David moved for reconsideration. 101 The Senate Electoral Tribunal issued
Resolution No. 15-11 on November 24, 2015, giving Senator Poe five (5) days to comment on the Motion
for Reconsideration.102 chanrobleslaw

Senator Poe filed her Comment/Opposition to the Motion for Reconsideration on December 1,
2015.103 David's Motion for Reconsideration was denied by the Senate Electoral Tribunal on December 3,
2015:104

WHEREFORE, the Tribunal resolves to DENY the Verified Motion for Reconsideration (of the Decision
promulgated on 17 November 2015) of David Rizalito Y. David dated 23 November 2015.

The Tribunal further resolves to CONFIRM Resolution No. 15-11 dated 24 November 2015 issued by the
Executive Committee of the Tribunal; to NOTE the Comment/Opposition filed by counsel for Respondent
on 01 December 2015; to GRANT the motion for leave to appear and submit memorandum as amici
curiae filed by Dean Arturo de Castro [and to] NOTE the Memorandum (for Volunteer Amicus Curiae)
earlier submitted by Dean de Castro before the Commission on Elections in SPA No. 15-139 (DC), entitled
"Amado D. Valdez, Petitoner, versus Mary Grace Natividad Sonora Poe Llaman[z]ares, Respondent."

SO ORDERED.105 (Emphasis in the original)


On December 8, 2015, the Senate Electoral Tribunal's Resolution was received by David. 106 On December
9, 2015, David filed the pre Petition for Certiorari before this Court. 107 chanrobleslaw

On December 16, 2015, this Court required the Senate Electoral Tribunal and Senator Poe to comment on
the Petition "within a non-extendible period of fifteen (15) days from notice." 108 The Resolution also set
oral arguments on January 19, 2016.109 The Senate Electoral Tribunal, through the Office of the Solicitor
General, submitted its Comment on December 30, 2015. 110 Senator Poe submitted her Comment on
January 4, 2016.111 chanrobleslaw

This case was held in abeyance pending the resolution of the Commission on Elections case on the issue of
private respondent's citizenship.

For resolution is the sole issue of whether the Senate Electoral Tribunal committed grave abuse of
discretion amounting to lack or excess of jurisdiction in dismissing petitioner's Petition for Quo Warranto
based on its finding that private respondent is a natural-born Filipino citizen, qualified to hold a seat as
Senator under Article VI, Section 3 of the 1987 Constitution.

Petitioner comes to this Court invoking our power of judicial review through a petition for certiorari under
Rule 65 of the 1997 Rules of Civil Procedure. He seeks to annul the assailed Decision and Resolution of the
Senate Electoral Tribunal, which state its findings and conclusions on private respondent's citizenship.

Ruling on petitioner's plea for post-judgment relief calls for a consideration of two (2) factors: first, the
breadth of this Court's competence relative to that of the Senate Electoral Tribunal; and second, the
nature of the remedial vehicle—a petition for certiorari—through which one who is aggrieved by a
judgment of the Senate Electoral Tribunal may seek relief from this Court.

I. A

The Senate Electoral Tribunal, along with the House of Representatives Electoral Tribunal, is a creation of
Article VI, Section 17 of the 1987 Constitution: 112
ARTICLE VI
The Legislative Department

. . . .

SECTION 17. The Senate and the House of Representatives shall each have an Electoral Tribunal which
shall be the sole judge of all contests relating to the election, returns, and qualifications of their respective
Members. Each Electoral Tribunal shall be composed of nine Members, three of whom shall be Justices of
the Supreme Court to be designated by the Chief Justice, and the remaining six shall be Members of the
Senate or the House of Representatives, as the case may be, who shall be chosen on the basis of
proportional representation from the political parties and the parties or organizations registered under the
party-list system represented therein. The senior Justice in the Electoral Tribunal shall be its Chairman.
(Emphasis supplied)
Through Article VI, Section 17, the Constitution segregates from all other judicial and quasi-judicial bodies
(particularly, courts and the Commission on Elections 113) the power to rule on contests114 relating to the
election, returns, and qualifications of members of the Senate (as well as of the House of
Representatives). These powers are granted to a separate and distinct constitutional organ. There are two
(2) aspects to the exclusivity of the Senate Electoral Tribunal's power. The power to resolve such contests
is exclusive to any other body. The resolution of such contests is its only task; it performs no other
function.

The 1987 Constitution is not the first fundamental law to introduce into our legal system an "independent,
impartial and non-partisan body attached to the legislature and specially created for that singular
purpose."115 The 1935 Constitution similarly created an Electoral Commission, independent from the
National Assembly, to be the sole judge of all contests relating to members of the National
Assembly.116 This was a departure from the system introduced by prior organic acts enforced under
American colonial rule—namely: the Philippine Bill of 1902 and the Jones Law of 1916—which vested the
power to resolve such contests in the legislature itself. When the 1935 Constitution was amended to make
room for a bicameral legislature, a corresponding amendment was made for there to be separate electoral
tribunals for each chamber of Congress.117 The 1973 Constitution did away with these electoral tribunals,
but they have since been restored by the 1987 Constitution.

All constitutional provisions—under the 1935 and 1987 Constitutions—which provide for the creation of
electoral tribunals (or their predecessor, the Electoral Commission), have been unequivocal in their
language. The electoral tribunal shall be the "sole" judge.

In Lazatin v. House Electoral Tribunal:118

The use of the word "sole" emphasizes the exclusive character of the jurisdiction conferred. . . . The
exercise of the power by the Electoral Commission under the 1935 Constitution has been described as
"intended to be as complete and unimpaired as if it had remained originally in the legislature[.]" Earlier,
this grant of power to the legislature was characterized by Justice Malcohn as "full, clear and complete." . .
. Under the amended 1935 Constitution, the power was unqualifiedly reposed upon the Electoral
Tribunal . . . and it remained as full, clear and complete as that previously granted the legislature and the
Electoral Commission. . . . The same may be said with regard to the jurisdiction of the Electoral Tribunals
under the 1987 Constitution. 119chanroblesvirtuallawlibrary

Exclusive, original jurisdiction over contests relating to the election, returns, and qualifications of the
elective officials falling within the scope of their powers is, thus, vested in these electoral tribunals. It is
only before them that post-election challenges against the election, returns, and qualifications of Senators
and Representatives (as well as of the President and the Vice-President, in the case of the Presidential
Electoral Tribunal) may be initiated.

The judgments of these tribunals are not beyond the scope of any review. Article VI, Section 17's
stipulation of electoral tribunals' being the "sole" judge must be read in harmony with Article VIII, Section
1's express statement that "[j]udicial power includes the duty of the courts of justice . . . to determine
whether or not there has been a grave abuse of discretion amounting to lack or excess of jurisdiction on
the part of any branch or instrumentality of the Government." Judicial review is, therefore, still
possible. In Libanan v. House of Representatives Electoral Tribunal:120

The Court has stressed that ". . . so long as the Constitution grants the [House of Representatives
Electoral Tribunal] the power to be the sole judge of all contests relating to the election, returns and
qualifications of members of the House of Representatives, any final action taken by the [House of
Representatives Electoral Tribunal] on a matter within its jurisdiction shall, as a rule, not be reviewed by
this Court . . . the power granted to the Electoral Tribunal . . . excludes the exercise of any authority on
the part of this Court that would in any wise restrict it or curtail it or even affect the same."

The Court did recognize, of course, its power of judicial review in exceptional cases. In  Robles vs. [House
of Representatives Electoral Tribunal], the Court has explained that while the judgments of the Tribunal
are beyond judicial interference, the Court may do so, however, but only "in the exercise of this Court's
so-called extraordinary jurisdiction, . . . upon a determination that the Tribunal's decision or resolution
was rendered without or in excess of its jurisdiction, or with grave abuse of discretion or
paraphrasing Morrero, upon a clear showing of such arbitrary and improvident use by the Tribunal of its
power as constitutes a denial of due process of law, or upon a demonstration of a very clear unmitigated
error, manifestly constituting such grave abuse of discretion that there has to be a remedy for such
abuse."

In the old, but still relevant, case of Morrero vs. Bocar, the Court has ruled that the power of the Electoral
Commission "is beyond judicial interference except, in any event, upon a clear showing of such arbitrary
and improvident use of power as will constitute a denial of due process." The Court does not, to
paraphrase it in Co vs. [House of Representatives Electoral Tribunal], venture into the perilous area of
correcting perceived errors of independent branches of the Government; it comes in only when it has to
vindicate a denial of due process or correct an abuse of discretion so grave or glaring that no less than the
Constitution itself calls for remedial action. 121 (Emphasis supplied, citations omitted)
This Court reviews judgments of the House and Senate Electoral Tribunals not in the exercise of its
appellate jurisdiction. Our review is limited to a determination of whether there has been an error in
jurisdiction, not an error in judgment.

I. B

A party aggrieved by the rulings of the Senate or House Electoral Tribunal invokes the jurisdiction of this
Court through the vehicle of a petition for certiorari under Rule 65 of the 1997 Rules of Civil Procedure. An
appeal is a continuation of the proceedings in the tribunal from which the appeal is taken. A petition for
certiorari is allowed in Article VIII, Section 1 of the Constitution and described in the 1997 Rules of Civil
Procedure as an independent civil action. 122 The viability of such a petition is premised on an allegation of
"grave abuse of discretion."123chanrobleslaw

The term "grave abuse of discretion" has been generally held to refer to such arbitrary, capricious, or
whimsical exercise of judgment as is tantamount to lack of jurisdiction: ChanRoblesVirtualawlibrary

[T]he abuse of discretion must be patent and gross as to amount to an evasion of a positive duty or a
virtual refusal to perform a duty enjoined by law, or to act at all in contemplation of law, as where the
power is exercised in an arbitrary and despotic manner by reason of passion and hostility. Mere abuse of
discretion is not enough: it must be grave. 124chanroblesvirtuallawlibrary

There is grave abuse of discretion when a constitutional organ such as the Senate Electoral Tribunal or the
Commission on Elections, makes manifestly gross errors in its factual inferences such that critical pieces of
evidence, which have been nevertheless properly introduced by a party, or admitted, or which were the
subject of stipulation, are ignored or not accounted for.125 chanrobleslaw

A glaring misinterpretation of the constitutional text or of statutory provisions, as well as a misreading or


misapplication of the current state of jurisprudence, is also considered grave abuse of discretion. 126 The
arbitrariness consists in the disregard of the current state of our law.

Adjudication that fails to consider the facts and evidence or frivolously departs from settled principles
engenders a strong suspicion of partiality. This can be a badge of hostile intent against a party.

Writs of certiorari have, therefore, been issued: (a) where the tribunal's approach to an issue is premised
on wrong considerations and its conclusions founded on a gross misreading, if not misrepresentation, of
the evidence;127 (b) where a tribunal's assessment of a case is "far from reasonable[,] [and] based solely
on very personal and subjective assessment standards when the law is replete with standards that can be
used";128 "(c) where the tribunal's action on the appreciation and evaluation of evidence oversteps the
limits of its discretion to the point of being grossly unreasonable"; 129 and (d) where the tribunal invokes
erroneous or irrelevant considerations in resolving an issue. 130 chanrobleslaw

I. C

We find no basis for concluding that the Senate Electoral Tribunal acted without or in excess of jurisdiction,
or with grave abuse of discretion amounting to lack or excess of jurisdiction.

The Senate Electoral Tribunal's conclusions are in keeping with a faithful and exhaustive reading of the
Constitution, one that proceeds from an intent to give life to all the aspirations of all its provisions.

Ruling on the Petition for Quo Warranto initiated by petitioner, the Senate Electoral Tribunal was
confronted with a novel legal question: the citizenship status of children whose biological parents are
unknown, considering that the Constitution, in Article IV, Section 1(2) explicitly makes reference to one's
father or mother. It was compelled to exercise its original jurisdiction in the face of a constitutional
ambiguity that, at that point, was without judicial precedent.

Acting within this void, the Senate Electoral Tribunal was only asked to make a reasonable interpretation
of the law while needfully considering the established personal circumstances of private respondent. It
could not have asked the impossible of private respondent, sending her on a proverbial fool's errand to
establish her parentage, when the controversy before it arose because private respondent's parentage was
unknown and has remained so throughout her life.

The Senate Electoral Tribunal knew the limits of human capacity. It did not insist on burdening private
respondent with conclusively proving, within the course of the few short months, the one thing that she
has never been in a position to know throughout her lifetime. Instead, it conscientiously appreciated the
implications of all other facts known about her finding. Therefore, it arrived at conclusions in a manner in
keeping with the degree of proof required in proceedings before a quasi-judicial body: not absolute
certainty, not proof beyond reasonable doubt or preponderance of evidence, but "substantial evidence, or
that amount of relevant evidence which a reasonable mind might accept as adequate to justify a
conclusion."131chanrobleslaw

In the process, it avoided setting a damning precedent for all children with the misfortune of having been
abandoned by their biological parents. Far from reducing them to inferior, second-class citizens, the
Senate Electoral Tribunal did justice to the Constitution's aims of promoting and defending the well-being
of children, advancing human rights, and guaranteeing equal protection of the laws and equal access to
opportunities for public service.

II

Article VI, Section 3 of the 1987 Constitution spells out the requirement that "[n]o person shall be a
Senator unless he [or she] is a natural-born citizen of the Philippines."

Petitioner asserts that private respondent is not a natural-born citizen and, therefore, not qualified to sit as
Senator of the Republic, chiefly on two (2) grounds. First, he argues that as a foundling whose parents are
unknown, private respondent fails to satisfy the jus sanguinis principle: that is, that she failed to establish
her Filipino "blood line," which is supposedly the essence of the Constitution's determination of who are
natural-born citizens of the Philippines. Proceeding from this first assertion, petitioner insists that as
private respondent was never a natural-born citizen, she could never leave reverted to natural-born status
despite the performance of acts that ostensibly comply with Republic Act No. 9225, otherwise known as
the Citizenship Retention and Re-acquisition Act of 2003.

Petitioner's case hinges on the primacy he places over Article IV, Section 1 of the 1987 Constitution and its
enumeration of who are Filipino citizens, more specifically on Section 1(2), which identifies as citizens
"[t]hose whose fathers or mothers are citizens of the Philippines." Petitioner similarly claims that, as
private respondent's foundling status is settled, the burden to prove Filipino parentage was upon her. With
private respondent having supposedly failed to discharge this burden, the supposed inevitable conclusion
is that she is not a natural-born Filipino.

III

At the heart of this controversy is a constitutional ambiguity. Definitely, foundlings have biological parents,
either or both of whom can be Filipinos. Yet, by the nature of their being foundlings, they may, at critical
times, not know their parents. Thus, this controversy must consider possibilities where parentage may be
Filipino but, due to no fault of the foundling, remains unknown. 132 Resolving this controversy hinges on
constitutional interpretation.

Discerning constitutional meaning is an exercise in discovering the sovereign's purpose so as to identify


which among competing interpretations of the same text is the more contemporarily viable construction.
Primarily, the actual words—text—and how they are situated within the whole document—context—
govern. Secondarily, when discerning meaning from the plain text (i.e., verba legis) fails,
contemporaneous construction may settle what is more viable. Nevertheless, even when a reading of the
plain text is already sufficient, contemporaneous construction may still be resorted to as a means for
verifying or validating the clear textual or contextual meaning of the Constitution.

III. A

The entire exercise of interpreting a constitutional provision must necessarily begin with the text itself. The
language of the provision being interpreted is the principal source from which this Court determines
constitutional intent.133 chanrobleslaw
To the extent possible, words must be given their ordinary meaning; this is consistent with the basic
precept of verba legis.134 The Constitution is truly a public document in that it was ratified and approved
by a direct act of the People exercising their right of suffrage, they approved of it through a plebiscite. The
preeminent consideration in reading the Constitution, therefore, is the People's consciousness: that is,
popular, rather than technical-legal, understanding. Thus: ChanRoblesVirtualawlibrary

We look to the language of the document itself in our search for its meaning. We do not of course stop
there, but that is where we begin. It is to be assumed that the words in which constitutional provisions are
couched express the objective sought to be attained. They are to be given their ordinary meaning except
where technical terms are employed in which case the significance thus attached to them prevails. As the
Constitution is not primarily a lawyer's document, it being essential for the rule of law to obtain that it
should ever be present in the people's consciousness, its language as much as possible should be
understood in the sense they have in common use. What it says according to the text of the provision to
be construed compels acceptance and negates the power of the courts to alter it, based on the postulate
that the framers and the people mean what they say. Thus, these are the cases where the need for
construction is reduced to a minimum.135 (Emphasis supplied)
Reading a constitutional provision requires awareness of its relation with the whole of the Constitution. A
constitutional provision is but a constituent of a greater whole. It is the framework of the Constitution that
animates each of its components through the dynamism of these components' interrelations. What is
called into operation is the entire document, not simply a peripheral item. The Constitution should,
therefore, be appreciated and read as a singular, whole unit—ut magis valeat quam pereat.136 Each
provision must be understood and effected in a way that gives life to all that the Constitution contains,
from its foundational principles to its finest fixings. 137
chanrobleslaw

The words and phrases that establish its framework and its values color each provision at the heart of a
controversy in an actual case. In Civil Liberties Union v. Executive Secretary:138

It is a well-established rule in constitutional construction that no one provision of the Constitution is to be


separated from all the others, to be considered alone, but that all the provisions bearing upon a particular
subject are to be brought into view and to be so interpreted as to effectuate the great purposes of the
instrument. Sections bearing on a particular subject should be considered and interpreted together as to
effectuate the whole purpose of the Constitution and one section is not to be allowed to defeat another, if
by any reasonable construction, the two can be made to stand together.

In other words, the court must harmonize them, if practicable, and must lean in favor of construction
which will render every word operative, rather than one which may make the words idle and
nugatory.139 (Citations omitted)
Reading a certain text includes a consideration of jurisprudence that has previously considered that exact
same text, if any. Our legal system is founded on the basic principle that "judicial decisions applying or
interpreting the laws or the Constitution shall form part of [our] legal system." 140 Jurisprudence is not an
independent source of law. Nevertheless, judicial interpretation is deemed part of or written into the text
itself as of the date that it was originally passed. This is because judicial construction articulates the
contemporaneous intent that the text brings to effect.141 Nevertheless, one must not fall into the
temptation of considering prior interpretation as immutable.

Interpretation grounded on textual primacy likewise looks into how the text has evolved. Unless
completely novel, legal provisions are the result of the re-adoption—often with accompanying re-
calibration—of previously existing rules. Even when seemingly novel, provisions are often introduced as a
means of addressing the inadequacies and excesses of previously existing rules.

One may trace the historical development of text by comparing its current iteration with prior counterpart
provisions, keenly taking note of changes in syntax, along with accounting for more conspicuous
substantive changes such as the addition and deletion of provisos or items in enumerations, shifting
terminologies, the use of more emphatic or more moderate qualifiers, and the imposition of heavier
penalties. The tension between consistency and change galvanizes meaning.

Article IV, Section 1 of the 1987 Constitution, which enumerates who are citizens of the Philippines, may
be compared with counterpart provisions, not only in earlier Constitutions but even in organic laws 142 and
in similar mechanisms143 introduced by colonial rulers whose precepts nevertheless still resonate today.

Even as ordinary meaning is preeminent, a realistic appreciation of legal interpretation must grapple with
the truth that meaning is not always singular and uniform. In Social Weather Stations, Inc. v. Commission
on Elections,144 this Court explained the place of a holistic approach in legal interpretation: ChanRoblesVirtualawlibrary

Interestingly, both COMELEC and petitioners appeal to what they (respectively) construe to be plainly
evident from Section 5.2(a)'s text on the part of COMELEC, that the use of the words "paid for" evinces no
distinction between direct purchasers and those who purchase via subscription schemes; and, on the part
of petitioners, that Section 5.2(a)'s desistance from actually using the word "subscriber" means that
subscribers are beyond its contemplation. The variance in the parties' positions, considering that they are
both banking on what they claim to be the Fair Election Act's plain meaning, is the best evidence of an
extant ambiguity.

Second, statutory construction cannot lend itself to pedantic rigor that foments absurdity. The dangers of
inordinate insistence on literal interpretation are commonsensical and need not be belabored. These
dangers are by no means endemic to legal interpretation. Even in everyday conversations, misplaced
literal interpretations are fodder for humor. A fixation on technical rules of grammar is no less innocuous.
A pompously doctrinaire approach to text can stifle, rather than facilitate, the legislative wisdom that
unbridled textualism purports to bolster.

Third, the assumption that there is, in all cases, a universal plain language is erroneous. In reality,
universality and uniformity in meaning is a rarity. A contrary belief wrongly assumes that language is
static.

The more appropriate and more effective approach is, thus, holistic rather than parochial: to
consider context and the interplay of the historical, the contemporary, and even the
envisioned. Judicial interpretation entails the convergence of social realities and social ideals. The latter
are meant to be effected by the legal apparatus, chief of which is the bedrock of the prevailing legal order:
the Constitution. Indeed, the word in the vernacular that describes the Constitution — saligan —
demonstrates this imperative of constitutional primacy.

Thus, we refuse to read Section 5.2(a) of the Fair Election Act in isolation. Here, we consider not an
abstruse provision but a stipulation that is part of the whole, i.e., the statute of which it is a part, that is
aimed at realizing the ideal of fair elections. We consider not a cloistered provision but a norm that should
have a present authoritative effect to achieve the ideals of those who currently read, depend on, and
demand fealty from the Constitution. 145 (Emphasis supplied)
III. B

Contemporaneous construction and aids that are external to the text may be resorted to when the text is
capable of multiple, viable meanings. 146 It is only then that one can go beyond the strict boundaries of the
document. Nevertheless, even when meaning has already been ascertained from a reading of the plain
text, contemporaneous construction may serve to verify or validate the meaning yielded by such reading.

Limited resort to contemporaneous construction is justified by the realization that the business of
understanding the Constitution is not exclusive to this Court. The basic democratic foundation of our
constitutional order necessarily means that all organs of government, and even the People, read the
fundamental law and are guided by it. When competing viable interpretations arise, a justiciable
controversy may ensue requiring judicial intervention in order to arrive with finality at which interpretation
shall be sustained. To remain true to its democratic moorings, however, judicial involvement must remain
guided by a framework or deference and constitutional avoidance. This same principle underlies the basic
doctrine that courts are to refrain from issuing advisory opinions. Specifically as regards this Court, only
constitutional issues that are narrowly framed, sufficient to resolve an actual case, may be
entertained.147chanrobleslaw

When permissible then, one may consider analogous jurisprudence (that is, judicial decisions on similar,
but not the very same, matters or concerns), 148 as well as thematically similar statutes and international
norms that form part of our legal system. This includes discerning the purpose and aims of the text in light
of the specific facts under consideration. It is also only at this juncture—when external aids may be
consulted—that the supposedly underlying notions of the framers, as articulated through records of
deliberations and other similar accounts, can be illuminating.

III. C

In the hierarchy of the means for constitutional interpretation, inferring meaning from the supposed intent
of the framers or fathoming the original understanding of the individuals who adopted the basic document
is the weakest approach.

These methods leave the greatest room for subjective interpretation. Moreover, they allow for the greatest
errors. The alleged intent of the framers is not necessarily encompassed or exhaustively articulated in the
records of deliberations. Those that have been otherwise silent and have not actively engaged in
interpellation and debate may have voted for or against a proposition for reasons entirely their own and
not necessarily in complete agreement with those articulated by the more vocal. It is even possible that
the beliefs that motivated them were based on entirely erroneous premises. Fathoming original
understanding can also misrepresent history as it compels a comprehension of actions made within specific
historical episodes through detached, and not necessarily better-guided, modern lenses.
Moreover, the original intent of the framers of the Constitution is not always uniform with the original
understanding of the People who ratified it. In Civil Liberties Union: ChanRoblesVirtualawlibrary

While it is permissible in this jurisdiction to consult the debates and proceedings of the constitutional
convention in order to arrive at the reason and purpose of the resulting Constitution, resort thereto may
be had only when other guides fail as said proceedings are powerless to vary the terms of the Constitution
when the meaning is clear. Debates in the constitutional convention "are of value as showing the views of
the individual members, and as indicating the reasons for their votes, but they give us no light as to the
views of the large majority who did not talk, much less of the mass of our fellow citizens whose votes at
the polls gave the instrument the force of fundamental law. We think it safer to construe the constitution
from what appears upon its face." The proper interpretation therefore depends more on how it was
understood by the people adopting it than in the framer's understanding thereof. 149 (Emphasis supplied)
IV

Though her parents are unknown, private respondent is a Philippine citizen without the need for an
express statement in the Constitution making her so. Her status as such is but the logical consequence of
a reasonable reading of the Constitution within its plain text. The Constitution provides its own cues; there
is not even a need to delve into the deliberations of its framers and the implications of international legal
instruments. This reading proceeds from several levels.

On an initial level, a plain textual reading readily identifies the specific provision, which principally
governs: the Constitution's actual definition, in Article IV, Section 2, of "natural-born citizens." This
definition must be harmonized with Section 1's enumeration, which includes a reference to parentage.
These provisions must then be appreciated in relation to the factual milieu of this case. The pieces of
evidence before the Senate Electoral Tribunal, admitted facts, and uncontroverted circumstances
adequately justify the conclusion of private respondent's Filipino parentage.

On another level, the assumption should be that foundlings are natural-born unless there is substantial
evidence to the contrary. This is necessarily engendered by a complete consideration of the whole
Constitution, not just its provisions on citizenship. This includes its mandate of defending the well-being of
children, guaranteeing equal protection of the law, equal access to opportunities for public service, and
respecting human rights, as well as its reasons for requiring natural-born status for select public offices.
Moreover, this is a reading validated by contemporaneous construction that considers related legislative
enactments, executive and administrative actions, and international instruments.

Private respondent was a Filipino citizen at birth. This status' commencement from birth means that
private respondent never had to do anything to consummate this status. By definition, she is natural-born.
Though subsequently naturalized, she reacquired her natural-born status upon satisfying the requirement
of Republic Act No. 9225. Accordingly, she is qualified to hold office as Senator of the Republic.

V. A

Article IV, Section 1 of the 1987 Constitution enumerates who are citizens of the Philippines: ChanRoblesVirtualawlibrary

Section 1. The following are citizens of the Philippines:

chanRoblesvirtualLawlibrary
(1) Those who are citizens of the Philippines at the time of the adoption of this Constitution;
(2) Those whose fathers or mothers are citizens of the Philippines;
(3) Those born before January 17, 1973, of Filipino mothers, who elect Philippine citizenship upon reaching the age
of majority; and
(4) Those who are naturalized in accordance with law.150
Article IV, Section 2 identifies who are natural-born citizens: ChanRoblesVirtualawlibrary

Sec. 2. Natural-born citizens are those who are citizens of the Philippines from birth without having
to perform any act to acquire or perfect their Philippine citizenship. Those who elect Philippine
citizenship in accordance with paragraph (3), Section 1 hereof shall be deemed natural-born citizens.
(Emphasis supplied)
Section 2's significance is self-evident. It provides a definition of the term "natural-born citizens." This is
distinct from Section 1's enumeration of who are citizens. As against Section 1's generic listing, Section 2
specifically articulates those who may count themselves as natural-born.
The weight and implications of this categorical definition are better appreciated when supplemented with
an understanding of how our concepts of citizenship and natural-born citizenship have evolved. As will be
seen, the term "natural-born citizen" was a transplanted, but tardily defined, foreign concept.

V. B

Citizenship is a legal device denoting political affiliation. It is the "right to have rights." 151 It is one's
personal and . . . permanent membership in a political community. . . The core of citizenship is the
capacity to enjoy political rights, that is, the right to participate in government principally through the right
to vote, the right to hold public office[,] and the right to petition the government for redress of
grievance.152 chanrobleslaw

Citizenship also entails obligations to the political community of which one is part. 153 Citizenship, therefore,
is intimately tied with the notion that loyalty is owed to the state, considering the benefits and protection
provided by it. This is particularly so if these benefits and protection have been enjoyed from the moment
of the citizen's birth.

Tecson v. Commission on Elections154 reckoned with the historical development of our concept of


citizenship, beginning under Spanish colonial rule.155 Under the Spanish, the native inhabitants of the
Philippine Islands were identified not as citizens but as "Spanish subjects." 156 Church records show that
native inhabitants were referred to as "indios." The alternative identification of native inhabitants as
subjects or as indios demonstrated the colonial master's regard for native inhabitants as
inferior.157 Natives were, thus, reduced to subservience in their own land.

Under the Spanish Constitution of 1876, persons born within Spanish territory, not just peninsular Spain,
were considered Spaniards, classification, however, did not extend to the Philippine Islands, as Article 89
expressly mandated that the archipelago was to be governed by special laws. 158 It was only on December
18, 1889, upon the effectivity in this jurisdiction of the Civil Code of Spain, that there existed a categorical
enumeration of who were Spanish citizens,159 thus: ChanRoblesVirtualawlibrary

(a) Persons born in Spanish territory,


(b) Children of a Spanish father or mother, even if they were born outside of Spain,
(c) Foreigners who have obtained naturalization papers,
(d) Those who, without such papers, may have become domiciled inhabitants of any town of the Monarchy.160
1898 marked the end of Spanish colonial rule. The Philippine Islands were ceded by Spain to the United
States of America under the Treaty of Paris, which was entered into on December 10, 1898. The Treaty of
Paris did not automatically convert the native inhabitants to American citizens. 161 Instead, it left the
determination of the native inhabitants' status to the Congress of the United States: ChanRoblesVirtualawlibrary

Spanish subjects, natives of the Peninsula, residing in the territory over which Spain by the present treaty
relinquishes or cedes her sovereignty may remain in such territory or may remove therefrom. . . . In case
they remain in the territory they may preserve their allegiance to the Crown of Spain by making . . . a
declaration of their decision to preserve such allegiance; in default of which declaration they shall be held
to have renounced it and to have adopted the nationality of the territory in which they may reside.

Thus -

The civil rights and political status of the native inhabitants of the territories hereby ceded to the United
States shall be determined by Congress.162 chanroblesvirtuallawlibrary

Pending legislation by the United States Congress, the native inhabitants who had ceased to be Spanish
subjects were "issued passports describing them to be citizens of the Philippines entitled to the protection
of the United States."163 chanrobleslaw

The term "citizens of the Philippine Islands" first appeared in legislation in the Philippine Organic Act,
otherwise known as the Philippine Bill of 1902: 164

Section 4. That all inhabitants of the Philippine Islands continuing to reside therein, who were Spanish
subjects on the eleventh day of April, eighteen hundred and ninety-nine, and then resided in said Islands,
and their children born subsequent thereto, shall be deemed and held to be citizens of the Philippine
Islands and as such entitled to the protection of the United States, except such as shall have elected to
preserve their allegiance to the Crown of Spain in accordance with the provisions of the treaty of peace
between the United States and Spain signed at Paris December tenth, eighteen hundred and ninety-eight.
(Emphasis supplied)
The Philippine Bill of 1902 explicitly covered the status of children born in the Philippine Islands to its
inhabitants who were Spanish subjects as of April 11, 1899. However, it did not account for the status of
children born in the Islands to parents who were not Spanish subjects. A view was expressed that the
common law concept of jus soli (or citizenship by place of birth), which was operative in the United States,
applied to the Philippine Islands.165 chanrobleslaw

On March 23, 1912, the United States Congress amended Section 4 of the Philippine Bill of 1902. It was
made to include a proviso for the enactment by the legislature of a law on acquiring citizenship. This
proviso read: ChanRoblesVirtualawlibrary

Provided, That the Philippine Legislature, herein provided for, is hereby authorized to provide by law for
the acquisition of Philippine citizenship by those natives of the Philippine Islands who do not come within
the foregoing provisions, the natives of the insular possessions of the United States, and such other
persons residing in the Philippine Islands who are citizens of the United States, or who could become
citizens of the United States under the laws of the United States if residing therein. 166 chanroblesvirtuallawlibrary

In 1916, the Philippine Autonomy Act, otherwise known as the Jones Law of 1916, replaced the Philippine
Bill of 1902. It restated the citizenship provision of the Philippine Bill of 1902, as amended: 167

Section 2.—Philippine Citizenship and Naturalization

That all inhabitants of the Philippine Islands who were Spanish subjects on the eleventh day of April,
eighteen hundred and ninety-nine, and then resided in said Islands, and their children born subsequent
thereto, shall be deemed and held to be citizens of the Philippine Islands, except such as shall have
elected to preserve their allegiance to the Crown of Spain in accordance with the provisions of the treaty of
peace between the United States and Spain, signed at Paris December tenth, eighteen hundred and
ninety-eight, and except such others as have since become citizens of some other country: Provided, That
the Philippine Legislature, herein provided for, is hereby authorized to provide by law for the acquisition of
Philippine citizenship by those natives of the Philippine Islands who do not come within the foregoing
provisions, the natives of the insular possessions of the United States, and such other persons residing in
the Philippine Islands who are citizens of the United States, or who could become citizens of the United
States under the laws of the United States if residing therein.
The Jones Law of 1916 provided that a native-born inhabitant of the Philippine Islands was deemed to be a
citizen of the Philippines as of April 11, 1899 if he or she was "(1) a subject of Spain on April 11, 1899, (2)
residing in the Philippines on said date, and (3) since that date, not a citizen of some other country." 168 chanrobleslaw

There was previously the view that jus soli may apply as a mode of acquiring citizenship. It was the 1935
Constitution that made sole reference to parentage vis-a-vis the determination of citizenship. 169 Article III,
Section 1 of the 1935 Constitution provided: ChanRoblesVirtualawlibrary

SECTION 1. The following are citizens of the Philippines:

chanRoblesvirtualLawlibrary
(1) Those who are citizens of the Philippine Islands at the time of the adoption of this Constitution.
(2) Those born in the Philippines Islands of foreign parents who, before the adoption of this Constitution, had been
elected to public office in the Philippine Islands.
(3) Those whose fathers are citizens of the Philippines.
(4) Those whose mothers are citizens of the Philippines and upon reaching the age of majority, elect Philippine
citizenship.
(5) Those who are naturalized in accordance with law.
The term "natural-born citizen" first appeared in this jurisdiction in the 1935 Constitution's provision
stipulating the qualifications for President and Vice-President of the Philippines. Article VII, Section 3
read: ChanRoblesVirtualawlibrary

SECTION 3. No person may be elected to the office of President or Vice-President, unless he be a natural-
born citizen of the Philippines, a qualified voter, forty years of age or over, and has been a resident of the
Philippines for at least ten years immediately preceding the election.
While it used the term "natural-born citizen," the 1935 Constitution did not define the term.

Article II, Section 1(4) of the 1935 Constitution—read with the then civil law provisions that stipulated the
automatic loss of Filipino citizens lip by women who marry alien husbands—was discriminatory towards
women.170 The 1973 Constitution rectified this problematic situation: ChanRoblesVirtualawlibrary

SECTION 1. The following are citizens of the Philippines:

chanRoblesvirtualLawlibrary
(1) Those who are citizens of the Philippines at the time of the adoption of this Constitution.
(2) Those whose fathers or mothers are citizens of the Philippines.
(3) Those who elect Philippine citizenship pursuant to the provisions of the Constitution of nineteen hundred and
thirty-five.
(4) Those who are naturalized in accordance with law.

SECTION 2. A female citizen of the Philippines who marries an alien shall retain her Philippine citizenship,
unless by her act or omission she is deemed, under the law, to have renounced her citizenship. 171 chanroblesvirtuallawlibrary

The 1973 Constitution was the first instrument to actually define the term "natural-born citizen." Article
III, Section 4 of the 1973 Constitution provided: ChanRoblesVirtualawlibrary

SECTION 4. A natural-born citizen is one who is a citizen of the Philippines from birth without having to
perform any act to acquire or perfect his Philippine citizenship. 172 chanroblesvirtuallawlibrary

The present Constitution adopted most of the provisions of the 1973 Constitution on citizenship, "except
for subsection (3) thereof that aimed to correct the irregular situation generated by the questionable
proviso in the 1935 Constitution." 173 chanrobleslaw

Article IV, Section 1 of the 1987 Constitution now reads: ChanRoblesVirtualawlibrary

Section 1. The following are citizens of the Philippines:

chanRoblesvirtualLawlibrary
(1) Those who are citizens of the Philippines at the time of the adoption of this Constitution;
(2) Those whose fathers or mothers are citizens of the Philippines;
(3) Those born before January 17, 1973, of Filipino mothers, who elect Philippine citizenship upon reaching the age
of majority; and
(4) Those who are naturalized in accordance with law.174
Article IV, Section 2 also calibrated the 1973 Constitution's previous definition of natural-born citizens, as
follows: ChanRoblesVirtualawlibrary

Sec. 2. Natural-born citizens are those who are citizens of the Philippines from birth without having
to perform any act to acquire or perfect their Philippine citizenship. Those who elect Philippine
citizenship in accordance with paragraph (3), Section 1 hereof shall be deemed natural-born citizens.
(Emphasis supplied)
Ironically, the concept of "natural-born" citizenship is a "foreign" concept that was transplanted into this
jurisdiction as part of the 1935 Constitution's eligibility requirements for President and Vice-President of
the Philippines.

In the United States Constitution, from which this concept originated, the term "natural-born citizen"
appears in only a single instance: as an eligibility requirement for the presidency. 175 It is not defined in
that Constitution or in American laws. Its origins and rationale for inclusion as a requirement for the
presidency are not even found in the records of constitutional deliberations. 176 However, it has been
suggested that, as the United States was under British colonial rule before its independence, the
requirement of being natural-born was introduced as a safeguard against foreign infiltration in the
administration of national government: ChanRoblesVirtualawlibrary

It has been suggested, quite plausibly, that this language was inserted in response to a letter sent by John
Jay to George Washington, and probably to other delegates, on July 25, 1787, which stated: ChanRoblesVirtualawlibrary

Permit me to hint, whether it would be wise and seasonable to provide a strong check to the admission of
Foreigners into the administration of our national Government; and to declare expressly that the
Command in Chief of the American army shall not be given to nor devolve on, any but a
natural born Citizen.
Possibly this letter was motivated by distrust of Baron Von Steuben, who had served valiantly in the
Revolutionary forces, but whose subsequent loyalty was suspected by Jay. Another theory is that the Jay
letter, and the resulting constitutional provision, responded to rumors that the Convention was concocting
a monarchy to be ruled by a foreign monarch.177 chanroblesvirtuallawlibrary

In the United States, however, citizenship is based on jus soli, not jus sanguinis.

V. C

Today, there are only two (2) categories of Filipino citizens: natural-born and naturalized.
A natural-born citizen is defined in Article IV, Section 2 as one who is a citizen of the Philippines "from
birth without having to perform any act to acquire or perfect Philippine citizenship." By necessary
implication, a naturalized citizen is one who is not natural-born. Bengson v. House of Representatives
Electoral Tribunal178 articulates this definition by dichotomy:ChanRoblesVirtualawlibrary

[O]nly naturalized Filipinos are considered not natural-born citizens. It is apparent from the enumeration
of who are citizens under the present Constitution that there are only two classes of citizens: . . . A citizen
who is not a naturalized Filipino, i.e., did not have to undergo the process of naturalization to obtain
Philippine citizenship, necessarily is a natural-born Filipino. 179 chanroblesvirtuallawlibrary

Former Associate Justice Artemio Panganiban further shed light on the concept of naturalized citizens in his
Concurring Opinion in Bengson: naturalized citizens, he stated, are "former aliens or foreigners who had to
undergo a rigid procedure, in which they had to adduce sufficient evidence to prove that they possessed all
the qualifications and none of the disqualifications provided by law in order to become Filipino
citizens."180
chanrobleslaw

One who desires to acquire Filipino citizenship by naturalization is generally required to file a verified
petition.181 He or she must establish. among others, that he or she is of legal age, is of good moral
character, and has the capacity to adapt to Filipino culture, tradition, and principles, or otherwise has
resided in the Philippines for a significant period of time. 182 Further, the applicant must show that he or
she will not be a threat to the state, to the public, and to the Filipinos' core beliefs. 183 chanrobleslaw

V. D

Article IV, Section 1 of the 1987 Constitution merely gives an enumeration. Section 2 categorically defines
"natural-born citizens." This constitutional definition is further clarified in jurisprudence, which delineates
natural-born citizenship from naturalized citizenship. Consistent with Article 8 of the Civil Code, this
jurisprudential clarification is deemed written into the interpreted text, thus establishing its
contemporaneous intent.

Therefore, petitioner's restrictive reliance on Section 1 and the need to establish bloodline is misplaced. It
is inordinately selective and myopic. It divines Section 1's mere enumeration but blatantly turns a blind
eye to the succeeding Section's unequivocal definition.

Between Article IV, Section 1(2), which petitioner harps on, and Section 2, it is Section 2 that is on point.
To determine whether private respondent is a natural-born citizen, we must look into whether she had to
do anything to perfect her citizenship. In view of Bengson, this calls for an inquiry into whether she
underwent the naturalization process to become a Filipino.

She did not.

At no point has it been substantiated that private respondent went through the actual naturalization
process. There is no more straightforward and more effective way to terminate this inquiry than this
realization of total and utter lack of proof.

At most, there have been suggestions likening a preferential approach to foundlings, as well as compliance
with Republic Act No. 9225, with naturalization. These attempts at analogies are misplaced. The statutory
mechanisms for naturalization are clear, specific, and narrowly devised. The investiture of citizenship on
foundlings benefits children, individuals whose capacity to act is restricted. 184 It is a glaring mistake to
liken them to an adult filing before the relevant authorities a sworn petition seeking to become a Filipino,
the grant of which is contingent on evidence that he or she must himself or herself adduce. As shall later
be discussed, Republic Act No. 9225 is premised on the immutability of natural-born status. It privileges
natural-born citizens and proceeds from an entirely different premise from the restrictive process of
naturalization.

So too, the jurisprudential treatment of naturalization vis-a-vis natural-born status is clear. It should be
with the actual process of naturalization that natural-born status is to be contrasted, not against other
procedures relating to citizenship. Otherwise, the door may be thrown open for the unbridled diminution of
the status of citizens.

V. E

Natural-born citizenship is not concerned with being a human thoroughbred.

Section 2 defines "natural-born citizens." Section 1(2) stipulates that to be a citizen, either one's father or
one's mother must be a Filipino citizen.
That is all there is to Section 1(2). Physical features, genetics, pedigree, and ethnicity are not
determinative of citizenship.

Section 1(2) does not require one's parents to be natural-born Filipino citizens. It does not even require
them to conform to traditional conceptions of what is indigenously or ethnically Filipino. One or both
parents can, therefore, be ethnically foreign.

Section 1(2) requires nothing more than one ascendant degree: parentage. The citizenship of everyone
else in one's ancestry is irrelevant. There is no need, as petitioner insists, for a pure Filipino bloodline.

Section 1(2) requires citizenship, not identity. A conclusion of Filipino citizenship may be sustained by
evidence adduced in a proper proceeding, which substantially proves that either or both of one's parents is
a Filipino citizen.

V. F

Private respondent has done this. The evidence she adduced in these proceedings attests to how at least
one—if not both—of her biological parents were Filipino citizens.

Proving private respondent's biological parentage is now practically impossible. To begin with, she was
abandoned as a newborn infant. She was abandoned almost half a century ago. By now, there are only a
handful of those who, in 1968, were able-minded adults who can still lucidly render testimonies on the
circumstances of her birth and finding. Even the identification of individuals against whom DNA evidence
may be tested is improbable, and by sheer economic cost, prohibitive.

However, our evidentiary rules admit of alternative means for private respondent to establish her
parentage.

In lieu of direct evidence, facts may be proven through circumstantial evidence. In Suerte-Felipe v.
People:185

Direct evidence is that which proves the fact in dispute without the aid of any inference or presumption;
while circumstantial evidence is the proof of fact or facts from which, taken either singly or collectively, the
existence of a particular fact in dispute may be inferred as a necessary or probable consequence. 186 chanroblesvirtuallawlibrary

People v. Raganas187 further defines circumstantial evidence: ChanRoblesVirtualawlibrary

Circumstantial evidence is that which relates to a series of facts other than the fact in issue, which by
experience have been found so associated with such fact that in a relation of cause and effect, they lead
us to a satisfactory conclusion.188 (Citation omitted)
Rule 133, Section 4 of the Revised Rules on Evidence, for instance, stipulates when circumstantial
evidence is sufficient to justify a conviction in criminal proceedings: ChanRoblesVirtualawlibrary

Section 4. Circumstantial evidence, when sufficient. — Circumstantial evidence is sufficient for conviction
if:

chanRoblesvirtualLawlibrary (a) There is more than one circumstances;

(b) The facts from which the inferences are derived are proven; and  cralawlawlibrary

(c) The combination of all the circumstances is such as to produce a conviction beyond reasonable doubt.
Although the Revised Rules on Evidence's sole mention of circumstantial evidence is in reference to
criminal proceedings, this Court has nevertheless sustained the use of circumstantial evidence in other
proceedings.189 There is no rational basis for making the use of circumstantial evidence exclusive to
criminal proceedings and for not considering circumstantial facts as valid means for proof in civil and/or
administrative proceedings.

In criminal proceedings, circumstantial evidence suffices to sustain a conviction (which may result in
deprivation of life, liberty, and property) anchored on the highest standard or proof that our legal system
would require, i.e., proof beyond reasonable doubt. If circumstantial evidence suffices for such a high
standard, so too may it suffice to satisfy the less stringent standard of proof in administrative and quasi-
judicial proceedings such as those before the Senate Electoral Tribunal, i.e., substantial evidence. 190 chanrobleslaw

Private respondent was found as a newborn infant outside the Parish Church of Jaro, Iloilo on September
3, 1968.191 In 1968, Iloilo, as did most—if not all—Philippine provinces, had a predominantly Filipino
population.192 Private respondent is described as having "brown almond-shaped eyes, a low nasal bridge,
straight black hair and an oval-shaped face."193 She stands at 5 feet and 2 inches tall. 194 Further, in 1968,
there was no international airport in Jaro, Iloilo.
These circumstances are substantial evidence justifying an inference that her biological parents were
Filipino. Her abandonment at a Catholic Church is more or less consistent with how a Filipino who, in 1968,
lived in a predominantly religious and Catholic environment, would have behaved. The absence of an
international airport in Jaro, Iloilo precludes the possibility of a foreigner mother, along with a foreigner
father, swiftly and surreptitiously coming in and out of Jaro, Iloilo just to give birth and leave her offspring
there. Though proof of ethnicity is unnecessary, her physical features nonetheless attest to it.

In the other related case of Poe-Llamanzares v. Commission on Elections,195 the Solicitor General


underscored how it is statistically more probable that private respondent was born a Filipino citizen rather
than as a foreigner. He submitted the following table is support of his statistical inference: 196

NUMBER OF FOREIGN AND FILIPINO CHILDREN BORN IN THE PHILIPPINES: 1965-1975 and 2010-2014

FOREIGN CHILDREN BORN IN THE FILIPINO CHILDREN BORN IN THE


YEAR
PHILIPPINES PHILIPPINES
1965 1,479 795,415
1966 1,437 823,342
1967 1,440 840,302
1968 1,595 898,570
1969 1,728 946,753
1970 1,521 966,762
1971 1,401 963,749
1972 1,784 968,385
1973 1,212 1,045,290
1974 1,496 1,081,873
1975 1,493 1,223,837
2010 1,244 1,782,877
2011 1,140 1,746,685
2012 1,454 1,790,367
2013 1,315 1,751,523
2014 1,351 1,748,782

Source: Philippine Statistics Authority [illegible] 197


chanroblesvirtuallawlibrary

Thus, out of the 900,165 recorded births in the Philippines in 1968, only 1,595 or 0.18% newborns were
foreigners. This translates to roughly 99.8% probability that private respondent was born a Filipino citizen.

Given the sheer difficulty, if not outright impossibility, of identifying her parents after half a century, a
range of substantive proof is available to sustain a reasonable conclusion as to private respondent's
parentage.

VI

Before a discussion on how private respondent's natural-born status is sustained by a general assumption
on foundlings arising from a comprehensive reading and validated by a contemporaneous construction of
the Constitution, and considering that we have just discussed the evidence pertaining to the circumstances
of private respondent's birth, it is opportune to consider petitioner's allegations that private respondent
bore the burden of proving—through proof of her bloodline—her natural-born status.

Petitioner's claim that the burden of evidence shifted to private respondent upon a mere showing that she
is a foundling is a serious error.

Petitioner invites this Court to establish a jurisprudential presumption that all newborns who have been
abandoned in rural areas in the Philippines are not Filipinos. His emphasis on private respondent's
supposed burden to prove the circumstances of her birth places upon her an impossible condition. To
require proof from private respondent borders on the absurd when there is no dispute that the crux of the
controversy—the identity of her biological parents—is simply not known.
"Burden of proof is the duty of a party to present evidence on the facts in issue necessary to establish his
claim or defense by the amount of evidence required by law." Burden of proof lies on the party making the
allegations;198 that is, the party who "alleges the affirmative of the issue" 199 Burden of proof never shifts
from one party to another. What shifts is the burden of evidence. This shift happens when a party makes a
prima facie case in his or her favor. 200 The other party then bears the "burden of going forward" 201 with
the evidence considering that which has ostensibly been established against him or her.

In an action for quo warranto, the burden of proof necessarily falls on the party who brings the action and
who alleges that the respondent is ineligible for the office involved in the controversy. In proceedings
before quasi-judicial bodies such as the Senate Electoral Tribunal, the requisite quantum of proof is
substantial evidence.202 This burden was petitioner's to discharge. Once the petitioner makes a prima facie
case, the burden of evidence shifts to the respondent.

Private respondent's admitted status as a foundling does not establish a prima facie case in favor of
petitioner. While it does establish that the identities of private respondent's biological parents are not
known, it does not automatically mean that neither her father nor her mother is a Filipino.

The most that petitioner had in his favor was doubt. A taint of doubt, however, is by no means substantial
evidence establishing a prima facie case and shifting the burden of evidence to private respondent.

Isolating the fact of private respondent's being a foundling, petitioner trivializes other uncontroverted
circumstances that we have previously established as substantive evidence of private respondent's
parentage: ChanRoblesVirtualawlibrary

(1) Petitioner was found in front of a church in Jaro, Iloilo;

(2) She was only an infant when she was found, practically a newborn;

(3) She was-found sometime in September 1968;

(4) Immediately after she was found, private respondent was registered as a foundling;

(5) There was no international airport in Jaro, Iloilo; and

(6) Private respondent's physical features are consistent with those of typical Filipinos.
Petitioner's refusal to account for these facts demonstrates an imperceptive bias. As against petitioner's
suggested conclusions, the more reasonable inference from these facts is that at least one of private
respondent's parents is a Filipino.

VII

Apart from how private respondent is a natural-born Filipino citizen consistent with a reading that
harmonizes Article IV, Section 2's definition of natural-born citizens and Section 1(2)'s reference to
parentage, the Constitution sustains a presumption that all foundlings found in the Philippines are born to
at least either a Filipino father or a Filipino mother and are thus natural-born, unless there is substantial
proof otherwise. Consistent with Article IV, Section 1(2), any such countervailing proof must show that
both—not just one—of a foundling's biological parents are not Filipino citizens.

VII. A

Quoting heavily from Associate Justice Teresita Leonardo-De Castro's Dissenting Opinion to the assailed
November 17, 2015 Decision, petitioner intimates that no inference or presumption in favor of natural-
born citizenship may be indulged in resolving this case. 203 He insists that it is private respondent's duty to
present incontrovertible proof of her Filipino parentage.

Relying on presumptions is concededly less than ideal. Common sense dictates that actual proof is
preferable. Nevertheless, resolving citizenship issues based on presumptions is firmly established in
jurisprudence.
In 2004, this Court resolved Tecson on the basis of presumptions. Ruling on the allegations that former
presidential candidate Ronald Allan Poe (more popularly known as Fernando Poe, Jr.) was not a natural-
born Filipino citizen, this Court proceeded from the presumptions that: first, Fernando Poe Jr.'s
grandfather, Lorenzo Pou, was born sometime in 1870, while the country was still under Spanish colonial
rule;204 and second, that Lorenzo Pou's place of residence, as indicated in his dearth certificate, must have
also been his place of residence before death, which subjected him to the "en masse Filipinization," or
sweeping investiture of Filipino citizenship effected by the Philippine Bill of 1902. 205 This Court then noted
that Lorenzo Pou's citizenship would have extended to his son and Fernando Poe Jr.'s father, Allan F. Poe.
Based on these, Fernando Poe. Jr. would then have been a natural-born Filipino as he was born while the
1935 Constitution, which conferred Filipino citizenship to those born to Filipino fathers, was in effect: ChanRoblesVirtualawlibrary

In ascertaining, in G.R. No. 161824, whether grave abuse of discretion has been committed by the
COMELEC, it is necessary to take on the matter of whether or not respondent FPJ is a natural-born citizen,
which, in turn, depended on whether or not the father of respondent, Allan F. Poe, would have himself
been a Filipino citizen and, in the affirmative, whether or not the alleged illegitimacy of respondent
prevents him from taking after the Filipino citizenship of his putative father. Any conclusion on the Filipino
citizenship of Lorenzo Pou could only be drawn from the presumption that having died in 1954 at 84 years
old, when the Philippines was under Spanish rule, and that San Carlos, Pangasinan, his place of residence
upon his death in 1954, in the absence of any other evidence, could have well been his place of residence
before death, such that Lorenzo Pou would have benefited from the "en masse Filipinization" that the
Philippine Bill had effected in 1902. That citizenship (of Lorenzo Pou), if acquired, would thereby extend to
his son, Allan F. Poe, father of respondent FPJ. The 1935 Constitution, during which regime respondent FPJ
has seen first light, confers citizenship to all persons whose fathers are Filipino citizens regardless of
whether such children are legitimate or illegitimate. 206 chanroblesvirtuallawlibrary

It is true that there is jurisprudence—Paa v. Chan207 and Go v. Ramos208 (which merely cites Paa)—to the


effect that presumptions cannot be entertained in citizenship cases.

Paa, decided in 1967, stated: ChanRoblesVirtualawlibrary

It is incumbent upon the respondent, who claims Philippine citizenship, to prove to the satisfaction of the
court that he is really a Filipino. No presumption can be indulged in favor of the  claimant, of Philippine
citizenship, and any doubt regarding citizenship must be resolved in favor of the State. 209 (Emphasis
supplied)
These pronouncements are no longer controlling in light of this Court's more recent ruling in Tecson.

Moreover, what this Court stated in Paa was that "no presumption can be indulged in favor of the claimant
of Philippine citizenship." This reference to "the claimant" was preceded by a sentence specifically
referencing the duty of "the respondent." The syntax of this Court's pronouncement—using the definitive
article "the"—reveals that its conclusion was specific only to Chan and to his circumstances. Otherwise,
this Court would have used generic language. Instead of the definite article "the," it could have used the
indefinite article "a" in that same sentence: "no presumption can be indulged in favor of a claimant of
Philippine citizenship." In the alternative, it could have used other words that would show absolute or
sweeping application, for instance: "no presumption can be indulged in favor of any/every claimant of
Philippine citizenship;" or, "no presumption can be indulged in favor of all claimants of Philippine
citizenship."

The factual backdrop of Paa is markedly different from those of this case. Its statements, therefore, are
inappropriate precedents for this case. In Paa, clear evidence was adduced showing that respondent
Quintin Chan was registered as an alien with the Bureau of Immigration. His father was likewise registered
as an alien. These pieces of evidence already indubitably establish foreign citizenship and shut the door to
any presumption. In contrast, petitioner in this case presents no proof, direct or circumstantial, of private
respondent's or of both of her parents' foreign citizenship.

Go cited Paa, taking the same quoted portion but revising it to make it appear that the same
pronouncement was generally applicable: ChanRoblesVirtualawlibrary

It is incumbent upon one who claims Philippine citizenship to prove to the satisfaction of the court that he
is really a Filipino. No presumption can be indulged hi favor of the claimant of Philippine citizenship, and
any doubt regarding citizenship must be resolved in favor of the state. 210 (Emphasis supplied)
Thus, Paa's essential and pivotal nuance was lost in proverbial translation. In any case, Go was decided by
this Court sitting in Division. It cannot overturn Tecson, which was decided by this Court sitting En Banc.
Likewise, Go's factual and even procedural backdrops are different from those of this case. Go involved the
deportation of an allegedly illegal and undesirable alien, not an election controversy. In Go, copies of birth
certificates unequivocally showing the Chinese citizenship of Go and of his siblings were adduced.

VII. B

The presumption that all foundlings found in the Philippines are born to at least either a Filipino father or a
Filipino mother (and are thus natural-born, unless there is substantial proof otherwise) arises when one
reads the Constitution as a whole, so as to "effectuate [its] whole purpose." 211 chanrobleslaw

As much as we have previously harmonized Article IV, Section 2 with Article IV, Section 1(2),
constitutional provisions on citizenship must not be taken in isolation. They must be read in light of the
constitutional mandate to defend the well-being of children, to guarantee equal protection of the law and
equal access to opportunities for public service, and to respect human rights. They must also be read in
conjunction with the Constitution's reasons for requiring natural-born status for select public offices.
Further, this presumption is validated by contemporaneous construction that considers related legislative
enactments, executive and administrative actions, and international instruments.

Article II, Section 13 and Article XV, Section 3 of the 1987 Constitution require the state to enhance
children's well-being and to project them from conditions prejudicial to or that may undermine their
development. Fulfilling this mandate includes preventing discriminatory conditions and, especially,
dismantling mechanisms for discrimination that hide behind the veneer of the legal apparatus: ChanRoblesVirtualawlibrary

ARTICLE II

....

State Policies

. . . .

SECTION 13. The State recognizes the vital role of the youth in nation-building and shall promote and
protect their physical, moral, spiritual, intellectual, and social well-being. It shall inculcate in the
youth patriotism and nationalism, and encourage their involvement in public and civic affairs.

....

ARTICLE XV
The Family

. . . .

SECTION 3. The State shall defend:

chanRoblesvirtualLawlibrary . . . .

(2) The right of children to assistance, including proper care and nutrition, and special protection
from all forms of neglect, abuse, cruelty, exploitation, and other conditions prejudicial to their
development[.] (Emphasis supplied)
Certain crucial government offices are exclusive to natural-born citizens of the Philippines. The 1987
Constitution makes the following offices exclusive to natural-born citizens: ChanRoblesVirtualawlibrary

(1) President;212
(2) Vice-President;213
(3) Senator;214
(4) Member of the House of Representatives;215
(5) Member of the Supreme Court or any lower collegiate court;216
(6) Chairperson and Commissioners of the Civil Service Commission;217
(7) Chairperson and Commissioners of the Commission on Elections;218
(8) Chairperson and Commissioners of the Commission on Audit;219
(9) Ombudsman and his or her deputies;220
(10) Board of Governors of the Bangko Sentral ng Pilipinas;221 and
(11) Chairperson and Members of the Commission on Human Rights.222
Apart from these, other positions that are limited to natural-born citizens include, among others, city
fiscals,223 assistant city fiscals,224 Presiding Judges and Associate Judges of the Sandiganbayan, and other
public offices.225 Certain professions are also limited to natural-born citizens, 226 as are other legally
established benefits and incentives.227 chanrobleslaw

Concluding that foundlings are not natural-born Filipino citizens is tantamount to permanently
discriminating against our foundling citizens. They can then never be of service to the country in the
highest possible capacities. It is also tantamount to excluding them from certain means such as
professions and state scholarships, which will enable the actualization of their aspirations. These
consequences cannot be tolerated by the Constitution, not least of all through the present politically
charged proceedings, the direct objective of which is merely to exclude a singular politician from office.
Concluding that foundlings are not natural-born citizens creates an inferior class of citizens who are made
to suffer that inferiority through no fault of their own.

If that is not discrimination, we do not know what is.

The Constitution guarantees equal protection of the laws and equal access to opportunities for public
service:ChanRoblesVirtualawlibrary

ARTICLE II

....

State Policies

. . . .

SECTION 26. The State shall guarantee equal access to opportunities for public service, and prohibit
political dynasties as may be defined by law.

....

ARTICLE III
Bill of Rights

SECTION 1. No person shall be deprived of life, liberty, or property without due process of law, nor shall
any person be denied the equal protection of the laws.

....

ARTICLE XIII
Social Justice and Human Rights

SECTION 1. The Congress shall give highest priority to the enactment of measures that protect and
enhance the right of all the people to human dignity, reduce social, economic, and political
inequalities, and remove cultural inequities by equitably diffusing wealth and political power
for the common good. (Emphasis supplied)
The equal protection clause serves as a guarantee that "persons under like circumstances and falling
within the same class are treated alike, in terms of 'privileges conferred and liabilities enforced.' It is a
guarantee against 'undue favor and individual or class privilege, as well as hostile discrimination or
oppression of inequality.'" 228 chanrobleslaw

Other than the anonymity of their biological parents, no substantial distinction 229 differentiates foundlings
from children with known Filipino parents. They are both entitled to the full extent of the state's protection
from the moment of their birth. Foundlings' misfortune in failing to identify the parents who abandoned
them—an inability arising from no fault of their own—cannot be the foundation of a rule that reduces them
to statelessness or, at best, as inferior, second-class citizens who are not entitled to as much benefits and
protection from the state as those who know their parents. Sustaining this classification is not only
inequitable; it is dehumanizing. It condemns those who, from the very beginning of their lives, were
abandoned to a life of desolation and deprivation.

This Court does not exist in a vacuum. It is a constitutional organ, mandated to effect the Constitution's
dictum of defending and promoting the well-being and development of children. It is not our business to
reify discriminatory classes based on circumstances of birth.

Even more basic than their being citizens of the Philippines, foundlings are human persons whose dignity
we value and rights we, as a civilized nation, respect. Thus: ChanRoblesVirtualawlibrary

ARTICLE II

....

State Policies

. . . .
SECTION 11. The State values the dignity of every human person and guarantees  full respect for
human rights. (Emphasis supplied)
VII. C

Though the matter is settled by interpretation exclusively within the confines of constitutional text, the
presumption that foundlings are natural-born citizens of the Philippines (unless substantial evidence of the
foreign citizenship of both of the foundling's parents is presented) is validated by a parallel consideration
or contemporaneous construction of the Constitution with acts of Congress, international instruments in
force in the Philippines, as well as acts of executive organs such as the Bureau of Immigration, Civil
Registrars, and the President of the Philippines.

Congress has enacted statutes founded on the premise that foundlings are Filipino citizens at birth. It has
adopted mechanisms to effect the constitutional mandate to protect children. Likewise, the Senate has
ratified treaties that put this mandate into effect.

Republic Act No. 9344, otherwise known as the Juvenile Justice and Welfare Act of 2006, provides: ChanRoblesVirtualawlibrary

SEC. 2. Declaration of State Policy. - The following State policies shall be observed at all times:

chanRoblesvirtualLawlibrary . . . .

(b) The State shall protect the best interests of the child through measures that will ensure the
observance of international standards of child protection, especially those to which the
Philippines is a party. Proceedings before any authority shall be conducted in the best interest of the
child and in a manner which allows the child to participate and to express himself/herself freely. The
participation of children in the program and policy formulation and implementation related to juvenile
justice and welfare shall be ensured by the concerned government agency. (Emphasis supplied)
Section 4(b) of the Republic Act No. 9344 defines the "best interest of the child" as the "totality of the
circumstances and conditions which are most congenial to the survival, protection and feelings of security
of the child and most encouraging to the child's physical, psychological and emotional development."

Consistent with this statute is our ratification230 of the United Nations Convention on the Rights of the
Child. This specifically requires the states-parties' protection of: first, children's rights to immediate
registration and nationality after birth; second, against statelessness; and third, against discrimination on
account of their birth status.231 Pertinent portions of the Convention read: ChanRoblesVirtualawlibrary

Preamble

The State Parties to the present Convention,

Considering that, in accordance with the principles proclaimed in the Charter of the United Nations,
recognition of the inherent dignity and of the equal and inalienable rights of all members of the
human family is the foundation of freedom, justice and peace in the world,

Bearing in mind that the peoples of the United Nations have, in the Charter, reaffirmed their faith in
fundamental human rights and in the dignity and worth of the human person, and have
determined to promote social progress and better standards of life in larger freedom,

Recognizing that the United Nations has, in the Universal Declaration of Human Rights and in the
International Covenants on Human Rights, proclaimed and agreed that everyone is entitled to all the
rights and freedoms set forth therein, without distinction of any kind, such as race, colour, sex,
language, religion, political or other opinion, national or social origin, property, birth or other status,

Recalling that, in the Universal Declaration of Human Rights, the United Nations has proclaimed
that childhood is entitled to special care and assistance,

. . . .

Have agreed as follows:

chanRoblesvirtualLawlibrary . . . .

Article 2

1. State parties shall respect and ensure the rights set forth in the present Convention to each child
within their jurisdiction without discrimination of any kind, irrespective of the child's or his
or her parent's or legal guardian's race, colour, sex, language, religion, political or other opinion,
national, ethnic or social origin, property, disability, birth or other status.
2. States Parties shall take appropriate measures to ensure that the child is protected
against all forms of discrimination or punishment on the basis of the status, activities,
expressed opinions, or beliefs of the child's parents, legal guardians, or family members.

Article 3

1. In all actions concerning children, whether undertaken by public or private social welfare
institutions, courts of law, administrative authorities or legislative bodies, the best interests of
the child shall be a primary consideration.

2. States Parties undertake to ensure the child such protection and care as is necessary for
his or her well-being, taking into account the rights and duties of his or her parents, legal
guardians, or other individuals legally responsible for him or her, and, to this end, shall take all
appropriate legislative and administrative measures.

. . . .

Article 7

1. The child, shall be registered immediately after birth and shall have the right from birth to a
name, the right to acquire a nationality and as far as possible, the right to know and be cared
for by his or her parents.

2. States Parties shall ensure the implementation of these rights in accordance with their
national law and their obligations under the relevant international instruments in this field, in
particular where the child would otherwise be stateless. (Emphasis supplied)

The Philippines likewise ratified232 the 1966 International Covenant on Civil and Political Rights. As with the
Convention on the Rights of the Child, this treaty requires that children be allowed immediate registration
after birth and to acquire a nationality. It similarly defends them against discrimination: ChanRoblesVirtualawlibrary

Article 24. . . .

1. Every child shall have, without any discrimination as to race, colour, sex, language, religion,
national or social origin, property or birth, the right to such measures of protection as are required by his
status as a minor, on the part of his family, society and the State.

2. Every child shall be registered immediately after birth and shall have a name.

3. Every child has the right to acquire a nationality.

. . . .

Article 26. All persons are equal before the law and are entitled without any discrimination to the
equal protection of the law. In this respect, the law shall prohibit any discrimination and
guarantee to all persons equal and effective protection against discrimination on any
ground such as race, colour, sex, language, religion, political or other opinion, national or social origin,
property, birth or other status. (Emphasis supplied)
Treaties are "international agreements] concluded between state| in written form and governed by
international law, whether embodied in a single instrument or in two or more related instruments and
whatever its particular designation."233 Under Article VII, Section 21 of the 1987 Constitution, treaties
require concurrence by the Senate before they became binding: ChanRoblesVirtualawlibrary

SECTION 21. No treaty or international agreement shall be valid and effective unless concurred in by at
least two-thirds of all the Members of the Senate.
The Senate's ratification of a treaty makes it legally effective and binding by transformation. It then has
the force and effect of a statute enacted by Congress. In Pharmaceutical and Health Care Association of
the Philippines v. Duque III, et al.:234

Under the 1987 Constitution, international law can become part of the sphere of domestic law either by
transformation or incorporation. The transformation method requires that an international law be
transformed into a domestic law through a constitutional mechanism such as local legislation. The
incorporation method applies when, by mere constitutional declaration, international law is deemed to
have the force of domestic law.

Treaties become part of the law of the land through transformation pursuant to Article VII, Section 21 of
the Constitution which provides that "[n]o treaty or international agreement shall be valid and effective
unless concurred in by at least two-thirds of all the members of the Senate." Thus, treaties or
conventional international law must go through a process prescribed by the Constitution for it to be
transformed into municipal law that can be applied to domestic conflicts. 235 (Emphasis supplied)
Following ratification by the Senate, no further action, legislative or otherwise, is necessary. Thereafter,
the whole of government—including the judiciary—is duty-bound to abide by the treaty, consistent with
the maxim pacta sunt servanda.

Accordingly, by the Constitution and by statute, foundlings cannot be the object of discrimination. They
are vested with the rights to be registered and granted nationality upon birth. To deny them these rights,
deprive them of citizenship, and render them stateless is to unduly burden them, discriminate them, and
undermine their development.

Not only Republic Act No. 9344, the Convention on the Rights of the Child, and the International Covenant
on Civil and Political Rights effect the constitutional dictum of promoting the well-being of children and
protecting them from discrimination. Other legislative enactments demonstrate the intent to treat
foundlings as Filipino citizens from birth.

Republic Act No. 8552, though briefly referred to as the Domestic Adoption Act of 1998, is formally entitled
An Act Establishing the Rules and Policies on Domestic Adoption of Filipino Children and for Other
Purposes. It was enacted as a mechanism to "provide alternative protection and assistance through foster
care or adoption of every child who is neglected, orphaned, or abandoned." 236 chanrobleslaw

Foundlings are explicitly among the "Filipino children" covered by Republic Act No. 8552: 237

SECTION 5. Location of Unknown Parent(s). — It shall be the duty of the Department or the child-placing
or child-caring agency which has custody of the child to exert all efforts to locate his/her unknown
biological parent(s). If such efforts fail, the child shall be registered as a foundling and
subsequently be the subject of legal proceedings where he/she shall be declared
abandoned. (Emphasis supplied)
Similarly, Republic Act No. 8043, though briefly referred to as the Inter-Country Adoption Act of 1995, is
formally entitled An Act Establishing the Rules to Govern Inter-Country Adoption of Filipino Children,
and for Other Purposes. As with Republic Act No. 8552, it expressly includes foundlings among "Filipino
children" who may be adopted: ChanRoblesVirtualawlibrary

SECTION 8. Who May Be Adopted. — Only a legally free child may be the subject of inter-country
adoption, hi order that such child may be considered for placement, the following documents must be
submitted: to the Board:

chanRoblesvirtualLawlibrary     a) Child study;

    b) Birth certificate/foundling certificate;

    c) Deed of voluntary commitment/decree of abandonment/death certificate of parents;

    d) Medical evaluation/history;

    e) Psychological evaluation, as necessary; and  cralawlawlibrary

    f) Recent photo of the child. (Emphasis supplied)


In the case of foundlings, foundling certificates may be presented in lieu of authenticated birth certificates
to satisfy the requirement for the issuance of passports, which will then facilitate their adoption by
foreigners: ChanRoblesVirtualawlibrary

SECTION 5. If the applicant is an adopted person, he must present a certified true copy of the Court Order
of Adoption, certified true copy of his original and amended birth certificates as issued by the OCRG. If the
applicant is a minor, a Clearance from the DSWD shall be required. In case the applicant is for adoption by
foreign parents under R.A. No. 8043, the following, shall be required:

chanRoblesvirtualLawlibrary
a) Certified true copy of the Court Decree of Abandonment of Child, the Death Certificate of the child's parents, or
the Deed of Voluntary Commitment executed after the birth of the child.

b) Endorsement of child to the Intercountry Adoption Board by the DSWD.

c) Authenticated Birth or Foundling Certificate. 238 (Emphasis supplied)


Our statutes on adoption allow for the recognition of foundlings' Filipino citizenship on account of their
birth. They benefit from this without having to do any act to perfect their citizenship or without having to
complete the naturalization process. Thus, by definition, they are natural-born citizens.

Specifically regarding private respondent, several acts of executive organs have recognized her natural-
born status. This status was never questioned throughout her life; that is, until circumstances made it
appear that she was a viable candidate for President of the Philippines. Until this, as well as the
proceedings in the related case of Poe-Llamanzares, private respondent's natural-born status has been
affirmed and reaffirmed through various official public acts.

First, private respondent was issued a foundling certificate and benefitted from the domestic adoption
process. Second, on July 18, 2006, she was granted an order of reacquisition of natural-born citizenship
under Republic Act No. 9225 by the Bureau of Immigration. Third, on October 6, 2010, the President of
the Philippines appointed her as MTRCB Chairperson—an office that requires natural-born citizenship. 239 chanrobleslaw

VIII

As it is settled that private respondent's being a foundling is not a bar to natural-born citizenship,
petitioner's proposition as to her inability to benefit from Republic Act No. 9225 crumbles. Private
respondent, a natural-born Filipino citizen, re-acquired natural-born Filipino citizenship when, following her
naturalization as a citizen of the United States, she complied with the requisites of Republic Act No. 9225.

VIII. A

"Philippine citizenship may be lost or reacquired in the manner provided by law." 240 Commonwealth Act
No. 63, which was in effect when private respondent was naturalized an American citizen on October 18,
2001, provided in Section 1(1) that "[a] Filipino citizen may lose his citizenship . . . [b]y naturalization in a
foreign country." Thus, private respondent lost her Philippine citizenship when she was naturalized an
American citizen. However, on July 7, 2006, she took her Oath of Allegiance to the Republic of the
Philippines under Section 3 of Republic Act No. 9225. Three (3) days later, July 10, 2006, she filed before
the Bureau of Immigration and Deportation a Petition for Reacquisition of her Philippine citizenship. Shortly
after, this Petition was granted.241
chanrobleslaw

Republic Act No. 9225 superseded Commonwealth Act No. 63 242 and Republic Act No. 8171243 specifically
"to do away with the provision in Commonwealth Act No. 63 which takes away Philippine citizenship from
natural-born Filipinos who become naturalized citizens of other countries." 244 chanrobleslaw

The citizenship regime put in place by Republic Act No. 9225 is designed, in its own words, to ensure "that
all Philippine citizens who become citizens of another country shall be deemed not to have lost their
Philippine citizenship."245 This Court shed light on this in Calilung v. Commission on Elections:246 "[w]hat
Rep. Act No. 9225 does is allow dual citizenship to natural-born Filipino citizens who have lost Philippine
citizenship by reason of their naturalization as citizens of a foreign country." 247 chanrobleslaw

Republic Act No. 9225 made natural-born Filipinos' status permanent and immutable despite naturalization
as citizens of other countries. To effect this, Section 3 of Republic Act No. 9225 provides: ChanRoblesVirtualawlibrary

SEC. 3. Retention of Philippine Citizenship. — Any provision of law to the contrary notwithstanding,
natural-born citizens of the Philippines who have lost their Philippine citizenship by reason of their
naturalization as citizens of a foreign country are hereby deemed to have reacquired Philippine citizenship
upon taking the following oath of allegiance to the Republic: ChanRoblesVirtualawlibrary

"I _________________________, solemnly swear (or affirm) that I will support and defend the
Constitution of the Republic of the Philippines and obey the laws and legal orders promulgated by the duly
constituted authorities of the Philippines; and I hereby declare that I recognize and accept the supreme
authority of the Philippines and will maintain true faith and allegiance thereto; and that I impose this
obligation upon myself voluntarily without mental reservation or purpose of evasion."
Natural-born citizens of the Philippines who, after the effectivity of this Act, become citizens of a foreign
country shall retain their Philippine citizenship upon taking the aforesaid oath.
Section 3's implications are clear. Natural-born Philippine citizens who, after Republic Act 9225 took effect,
are naturalized in foreign countries "retain," that is, keep, their Philippine citizenship, although the
effectivity of this retention and the ability to exercise the rights and capacities attendant to this status are
subject to certain solemnities (i.e., oath of allegiance and other requirements for specific rights and/or
acts, as enumerated in Section 5). On the other hand, those who became citizens of another country
before the effectivity of Republic Act No. 9225 "reacquire" their Philippine citizenship and may exercise
attendant rights and capacities, also upon compliance with certain solemnities. Read in conjunction with
Section 2's declaration of a policy of immutability, this reacquisition is not a mere restoration that leaves a
vacuum in the intervening period. Rather, this reacquisition works to restore natural-born status as though
it was never lost at all.
VIII. B

Taking the Oath of Allegiance effects the retention or reacquisition of natural-born citizenship. It also
facilitates the enjoyment of civil and political rights, "subject to all attendant liabilities and
responsibilities."248 However, other conditions must be met for the exercise of other faculties: ChanRoblesVirtualawlibrary

Sec. 5. Civil and Political Rights and Liabilities. - Those who retain or re-acquire Philippine citizenship
under this Act shall enjoy full civil and political rights and be subject to all attendant liabilities and
responsibilities under existing laws of the Philippines and the following conditions:

chanRoblesvirtualLawlibrary
(1) Those intending to exercise their right of suffrage must meet the requirements under Section 1, Article V of
the Constitution, Republic Act No. 9189, otherwise known as "the Overseas Absentee Voting Act of 2003"
and other existing laws;

(2) Those seeking elective public office in the Philippines shall meet the qualifications for holding such public
office as required by the Constitution and existing laws and, at the time of the filing of the certificate of
candidacy, make a personal and sworn renunciation of any and all foreign citizenship before any public
officer authorized to administer an oath;

(3) Those appointed to any public office shall subscribe and swear to an oath of allegiance to the Republic of
the Philippines and its duly constituted authorities prior to their assumption of office; Provided, That they
renounce their oath of allegiance to the country where they took that oath;

(4) Those intending to practice their profession in the Philippines shall apply with the proper authority for a
license or permit to engage in such practice; and

(5) That the right to vote or be elected or appointed to any public office in the Philippines cannot be exercised by,
or extended to, those who:

a. are candidates for or are occupying any public office in the country of which they are naturalized citizens;
and/or

b. are in active service as commissioned or noncommissioned officers in the armed forces of the country which
they are naturalized citizens. (Emphasis supplied)
Thus, natural-born Filipinos who have been naturalized elsewhere and wish to run for elective public office
must comply with all of the following requirements:

First, taking the oath of allegiance to the Republic. This effects the retention or reacquisition of one's
chanRoblesvirtualLawlibrary

status as a natural-born Filipino.249 This also enables the enjoyment of full civil and political rights, subject
to all attendant liabilities and responsibilities under existing laws, provided the solemnities recited in
Section 5 of Republic Act No. 9225 are satisfied. 250 chanrobleslaw

Second, compliance with Article V, Section 1 of the 1987 Constitution, 251 Republic Act No. 9189, otherwise
known as the Overseas Absentee Voting Act of 2003, and other existing laws. This is to facilitate the
exercise of the right of suffrage; that is, to allow for voting in elections. 252 chanrobleslaw

Third, "mak[ing] a personal and sworn renunciation of any and all foreign citizenship before any public
officer authorized to administer an oath."253 This, along with satisfying the other qualification requirements
under relevant laws, makes one eligible for elective public office.

As explained in Sobejana-Condon v. Commission on Elections,254 this required sworn renunciation is


intended to complement Article XI, Section 18 of the Constitution in that "[p]ublic officers and employees
owe the State and this Constitution allegiance at all times and any public officer or employee who seeks to
change his citizenship or acquire the status of an immigrant of another country during his tenure shall be
dealt with by law."255 It is also in view of this that Section 5(5) similarly bars those who seek or occupy
public office elsewhere and/or who are serving in the armed forces of other countries from being appointed
or elected to public office in the Philippines.
VIII. C

Private respondent has complied with all of these requirements. First, on July 7, 2006, she took the Oath
of Allegiance to the Republic of the Philippines. 256 Second, on August 31, 2006, she became a registered
voter of Barangay Santa Lucia, San Juan.257 This evidences her compliance with Article V, Section 1 of the
1987 Constitution. Since she was to vote within the country, this dispensed with the need to comply with
the Overseas Absentee Voting Act of 2003. Lastly, on October 20, 2010, she executed an Affidavit of
Renunciation of Allegiance to the United States of America and Renunciation of American
Citizenship.258 This was complemented by her execution of an Oath/Affirmation of Renunciation of
Nationality of the United States259 before Vice-Consul Somer E. Bessire-Briers on July 12, 2011, 260 which
was, in turn, followed by Vice Consul Jason Galian's issuance of a Certificate of Loss of Nationality on
December 9, 2011261 and the approval of this certificate by the Overseas Citizen Service, Department of
State, on February 3, 2012.262 chanrobleslaw

Private respondent has, therefore, not only fully reacquired natural-born citizenship; she has also complied
with all of the other requirements for eligibility to elective public office, as stipulated in Republic Act No.
9225.

VIII. D

It is incorrect to intimate that private respondent's having had to comply with Republic Act No. 9225
shows that she is a naturalized, rather than a natural-born, Filipino citizen. It is wrong to postulate that
compliance with Republic Act No. 9225 signifies the performance of acts to perfect citizenship.

To do so is to completely disregard the unequivocal policy of permanence and immutability as articulated


in Section 2 of Republic Act No. 9225 and as illuminated in jurisprudence. It is to erroneously assume that
a natural-born Filipino citizen's naturalization elsewhere is an irreversible termination of his or her natural-
born status.

To belabor the point, those who take the Oath of Allegiance under Section 3 of Republic Act No. 9225
reacquire natural-born citizenship. The prefix "re" signifies reference to the preceding state of affairs. It is
to this status quo ante that one returns. "Re"-acquiring can only mean a reversion to "the way things
were." Had Republic Act No. 9225 intended to mean the investiture of an entirely new status, it should not
have used a word such as "reacquire." Republic Act No. 9225, therefore, does not operate to make new
citizens whose citizenship commences only from the moment of compliance with its requirements.

Bengson, speaking on the analogous situation of repatriation, ruled that repatriation involves the
restoration of former status or the recovery of one's original nationality: ChanRoblesVirtualawlibrary

Moreover, repatriation results in the recovery of the original nationality. This means that a naturalized
Filipino who lost his citizenship will be restored to his prior status as a naturalized Filipino citizen. On the
other hand, if he was originally a natural-born citizen before he lost his Philippine citizenship, he will be
restored to his former status as a natural-born Filipino.263 (Emphasis supplied)
Although Bengson was decided while Commonwealth Act No. 63 was in force, its ruling is in keeping with
Republic Act No. 9225 's policy of permanence and immutablity: "all Philippine citizens of another country
shall be deemed not to have lost their Philippine citizenship." 264 In Bengson's words, the once naturalized
citizen is "restored" or brought back to his or her natural-born status. There may have been an
interruption in the recognition of this status, as, in the interim, he or she was naturalized elsewhere, but
the restoration of natural-born status expurgates this intervening fact. Thus, he or she does not become a
Philippine citizen only from the point of restoration and moving forward. He or she is recognized, de jure,
as a Philippine citizen from birth, although the intervening fact may have consequences de facto.

Republic Act No. 9225 may involve extended processes not limited to taking the Oath of Allegiance and
requiring compliance with additional solemnities, but these are for facilitating the enjoyment of other
incidents to citizenship, not for effecting the reacquisition of natural-born citizenship itself. Therefore, it is
markedly different from naturalization as there is no singular, extended process with which the former
natural-born citizen must comply.

IX

To hold, as petitioner suggests, that private respondent is stateless 265 is not only to set a dangerous and
callous precedent. It is to make this Court an accomplice to injustice.

Equality, the recognition of the humanity of every individual, and social justice are the bedrocks of our
constitutional order. By the unfortunate fortuity of the inability or outright irresponsibility of those gave
them life, foundlings are compelled to begin their very existence at a disadvantage. Theirs is a continuing
destitution that can never be truly remedied by any economic relief.

If we are to make the motives of our Constitution true, then we an never tolerate an interpretation that
condemns foundlings to an even greater misfortune because of their being abandoned. The Constitution
cannot be rendered inert and meaningless for them by mechanical judicial fiat.

Dura lex sed lex is not a callous and unthinking maxim to be deployed against other reasonable
interpretations of our basic law. It does command us to consider legal text, but always with justice in
mind.

It is the empowering and ennobling interpretation of the Constitution that we must always sustain. Not
only will this manner of interpretation edify the less fortunate; it establishes us, as Filipinos, as a humane
and civilized people.

The Senate Electoral Tribunal acted well within the bounds of its constitutional competence when it ruled
that private respondent is a natural-born citizen qualified to sit as Senator of the Republic. Contrary to
petitioner's arguments, there is no basis for annulling its assailed Decision and Resolution.

WHEREFORE, the Petition for Certiorari is DISMISSED. Public respondent Senate Electoral Tribunal did
not act without or in excess of its jurisdiction or with grave abuse of discretion amounting to lack or excess
of jurisdiction in rendering its assailed November 17, 2015 Decision and December 3, 2015 Resolution.

Private respondent Mary Grace Poe-Llamanzares is a natural-born Filipino citizen qualified to hold office as
Senator of the Republic.

SO ORDERED

Qualifications and disqualifications of voters, Article V, Section 1


- Kabataan Party List v Comelec, GR 221318 (2010)

Rights beget responsibilities; progress begets change.

Before the Court is a petition for certiorari and prohibition1 filed by herein petitioners Kabataan Party-List,


represented by Representative James Mark Terry L. Ridon and National President Marjohara S. Tucay;
Sarah Jane I. Elago, President of the National Union of Students of the Philippines; Veneer Mari E.
Crisostomo and Einstein Z. Recedes, Chairperson and Deputy Secretary-General of Anakbayan,
respectively; Marc Lino J. Abila, National President of the College Editors Guild of the Philippines; Charisse
Bernadine I. Bañez, Chairperson of the League of Filipino Students; Arlene Clarisse Y. Julve, member
of Alyansa ng mga Grupong Haligi ngAgham at Teknolohiya para sa Mamamayan (AGHAM); and Sining
Maria Rosa L. Marfori (petitioners) assailing the constitutionality of Republic Act No. (RA) 10367, entitled
"An Act Providing for Mandatory Biometrics Voter Registration," 2 as well as respondent Commission on
Elections (COMELEC) Resolution Nos. 9721,3 9863,4 and 10013,5 all related thereto.

The Facts

On February 15, 2013, President Benigno S. Aquino III signed into law RA 10367, which is a consolidation
of House Bill No. 3469 and Senate Bill No. 1030, passed by the House of Representatives and the Senate
on December 11, 2012 and December 12, 2012,6 respectively. Essentially, RA 10367 mandates the
COMELEC to implement a mandatory biometrics registration system for new voters 7 in order to establish a
clean, complete, permanent, and updated list of voters through the adoption of biometric technology. 8 RA
10367 was duly published on February 22, 2013, 9 and took effect fifteen (15) days after.10

RA 10367 likewise directs that "[r]egistered voters whose biometrics have not been captured shall
submit themselves for validation."11 "Voters who fail to submit for validation on or before the last
day of filing of application for registration for purposes of the May 2016 [E]lections shall be
deactivated x x x."12 Nonetheless, voters may have their records reactivated after the May 2016
Elections, provided that they comply with the procedure found in Section 28 13 of RA 8189,14 also known as
"The Voter's Registration Act of 1996." 15

On June 26, 2013, the COMELEC issued Resolution No. 972116 which serves as the implementing rules
and regulations of RA 10367, thus, prescribing the procedure for validation, 17 deactivation,18 and
reactivation of voters' registration records (VRRs). 19 Among others, the said Resolution provides that: (a)
"[t]he registration records of voters without biometrics data who failed to submit for validation
on or before the last day of filing of applications for registration for the purpose of the May 9,
2016 National and Local Elections shall be deactivated in the last [Election Registration Board (ERB)]
hearing to be conducted prior to said elections"; 20 (b) "[t]he following registered voters shall have their
biometrics data validated: [(1)] Those who do not have BIOMETRICS data appearing in the Voter['s]
Registration System (VRS); and [(2)] Those who have incomplete BIOMETRICS data appearing in the
VRS";21 (c) "[d]eactivated voters shall not be allowed to vote";22 and (d) "[d]eactivation x x x shall
comply with the requirements on posting, ERB hearing and service of individual notices to the
deactivated voters."23 Resolution No. 9721 further states that, as of the last day of registration and
validation for the 2013 Elections on October 31, 2012, a total of 9,018,256 registered voters were without
biometrics data.24 Accordingly, all Election Officers (EOs) were directed to "conduct [an] information
campaign on the conduct of validation."25 cralawred

On July 1, 2013, the COMELEC, pursuant to the aforesaid Resolution, commenced the mandatory biometric
system of registration. To make biometric registration convenient and accessible to the voting public, aside
from the COMELEC offices in every local government unit, it likewise established satellite registration
offices in barangays and mails.26

On April 1, 2014, the COMELEC issued Resolution No. 986327 which amended certain portions28 of
Resolution No. 985329 dated February 19, 2014, by stating that ERBs shall deactivate the VRRs of those
who "failed to submit for validation despite notice on or before October 31, 2015," and that the
"[d]eactivation for cases falling under this ground shall be made during the November 16, 2015 Board
hearing."30

A month later, or in May 2014, the COMELEC launched the NoBio-NoBoto public information


campaign which ran concurrently with the period of continuing registration. 31

On November 3, 2015, the COMELEC issued Resolution No. 1001332 which provides for the "procedures
in the deactivation of [VRRs] who do not have biometrics data in the [VRS] after the October 31, 2015
deadline of registration and validation." 33 Among others, the said Resolution directed the EOs to: (a)
"[p]ost the lists of voters without biometrics data in the bulletin boards of the City/Municipal hall,
Office of the Election Officer and in the barangay hall along with the notice of ERB hearing;" and (b)
"[s]end individual notices to the affected voters included in the generated list of voters without
biometrics data."34 It also provides that "[a]ny opposition/objection to the deactivation of records shall be
filed not later than November 9, 2015 in accordance with the period prescribed in Section 4, 35 [Chapter I,]
Resolution No. 9853."36 During the ERB hearing, which proceedings are summary in nature, 37 "the ERBs
shall, based dn the list of voters without biometrics data, order the deactivation of registration records on
the ground of 'failure to validate.'"38 Thereafter, EOs were required to "[s]end individual notices to the
deactivated voters within five (5) days from the last day of ERB hearing." 39 Moreover, Resolution No.
10013 clarified that the "[Registration records of voters with incomplete biometrics data and those
corrupted data (biometrics) in the database shall not be deactivated and be allowed to vote in
the May 9, 2016 Synchronized National, Local and [Autonomous Region on Muslim Mindanao (ARMM)]
Regional Elections."40

On November 25, 2015, herein petitioners filed the instant petition with application for temporary
restraining order (TRO) and/or writ of preliminary mandatory injunction (WPI) assailing the
constitutionality of the biometrics validation requirement imposed under RA 10367, as well as COMELEC
Resolution Nos. 9721, 9863, and 10013, all related thereto. They contend that: (a) biometrics validation
rises to the level of an additional, substantial qualification where there is penalty of deactivation; 41 (b)
biometrics deactivation is not the disqualification by law contemplated by the 1987 Constitution; 42 (c)
biometrics validation gravely violates the Constitution, considering that, applying the strict scrutiny test, it
is not poised with a compelling reason for state regulation and hence, an unreasonable deprivation of the
right to suffrage;43 (d) voters to be deactivated are not afforded due process; 44 and (e) poor experience
with biometrics should serve as warning against exacting adherence to the system. 45 Albeit already subject
of a prior petition46 filed before this Court, petitioners also raise herein the argument that deactivation by
November 16, 2015 would result in the premature termination of the registration period contrary to
Section 847 of RA 8189.48 Ultimately, petitioners pray that this Court declare RA 10367, as well as
COMELEC Resolution Nos. 9721, 9863, and 10013, unconstitutional and that the COMELEC be commanded
to desist from deactivating registered voters without biometric information, to reinstate voters who are
compliant with the requisites of RA 8189 but have already been delisted, and to extend the system of
continuing registration and capture of biometric information of voters until January 8, 2016. 49

On December 1, 2015, the Court required the COMELEC to file its comment to the petition. Meanwhile, it
issued a TRO requiring the COMELEC to desist from deactivating the registration records of voters without
biometric information, pending resolution of the case at hand. 50

On December 7, 2015, COMELEC Chairman Juan Andres D. Bautista, through a letter 51 addressed to the
Court En Banc, urgently appealed for the immediate lifting of the above-mentioned TRO, stating that the
COMELEC is set to finalize the Project of Precincts (POP) on December 15, 2015, and that the TRO issued
in this case has the effect of including the 2.4 Million deactivated voters in the list of voters, which, in turn,
would require revisions to the POP and consequently, adversely affect the timelines of all other interrelated
preparatory activities to the prejudice of the successful implementation of the Automated Election System
(AES) for the 2016 Elections.52
On December 11, 2015, the COMELEC, through the Office of the Solicitor General, filed its comment 53 to
the instant petition. On even date, petitioners filed a manifestation 54 asking the Court to continue the TRO
against the deactivation of voters without biometric information. 55

With no further pleadings required of the parties, the case was submitted for resolution.

The Issue Before the Court

The core issue in this case is whether or not RA 10367, as well as COMELEC Resolution Nos. 9721, 9863,
and 10013, all related thereto, are unconstitutional.

The Ruling of the Court

The petition is bereft of merit.

I.

At the outset, the Court passes upon the procedural objections raised in this case. In particular, the
COMELEC claims that petitioners: (a) failed to implead the Congress, the Office of the President, and the
ERB which it purports are indispensable parties to the case; 56 (b) did not have the legal standing to
institute the instant petition;57 and (c) erroneously availed of certiorari and prohibition as a mode of
questioning the constitutionality of RA 10367 and the assailed COMELEC Resolutions. 58

The submissions do not hold.

Recognizing that the petition is hinged on an important constitutional issue pertaining to the right of
suffrage, the Court views the matter as one of transcendental public importance and of compelling
significance. Consequently, it deems it proper to brush aside the foregoing procedural barriers and instead,
resolve the case on its merits. As resonated in the case of Pabillo v. COMELEC,59 citing Capalla v.
COMELEC60 and Guingona, Jr. v. COMELEC:61
There can be no doubt that the coming 10 May 2010 [in this case, the May 2016] elections is a matter of
great public concern. On election day, the country's registered voters will come out to exercise the sacred
right of suffrage. Not only is it an exercise that ensures the preservation of our democracy, the coming
elections also embodies our people's last ounce of hope for a better future. It is the final opportunity,
patiently awaited by our people, for the peaceful transition of power to the next chosen leaders of our
country. If there is anything capable of directly affecting the lives of ordinary Filipinos so as to
come within the ambit of a public concern, it is the coming elections, [x x x.]
Thus, in view of the compelling significance and transcending public importance of the issues raised by
petitioners, the technicalities raised by respondents should not be allowed to stand in the way, if the ends
of justice would not be subserved by a rigid adherence to the rules of procedure. (Emphasis and
underscoring supplied)
Furthermore, the issue on whether or not the policy on biometrics validation, as provided under RA 10367
and fleshed out in the assailed COMELEC Resolutions, should be upheld is one that demands immediate
adjudication in view of the critical preparatory activities that are currently being undertaken by the
COMELEC with regard to the impending May 2016 Elections. Thus, it would best subserve the ends of
justice to settle this controversy not only in order to enlighten the citizenry, but also so as not to stymy
the operations of a co-constitutional body. As pronounced in Roque, Jr. v. COMELEC:62
[T]he bottom line is that the Court may except a particular case from the operations of its rules when the
demands of justice so require. Put a bit differently, rules of procedure are merely tools designed to
facilitate the attainment of justice. Accordingly, technicalities and procedural barriers should not be
allowed to stand in the way, if the ends of justice would not be subserved by a rigid adherence to the rules
of procedure.63ChanRoblesVirtualawlibrary

That being said, the Court now proceeds to resolve the substantive issues in this case.

II.

Essentially, the present petition is a constitutional challenge against the biometrics validation requirement
imposed under RA 10367, including COMELEC Resolution Nos. 9721, 9863, and 10013. As non-compliance
with the same results in the penalty of deactivation, petitioners posit that it has risen to the level of an
unconstitutional substantive requirement in the exercise of the right of suffrage. 64 They submit that the
statutory requirement of biometric validation is no different from the unconstitutional requirement of
literacy and property because mere non-validation already absolutely curtails the exercise of the right of
suffrage through deactivation.65 Further, they advance the argument that deactivation is not the
disqualification by law contemplated as a valid limitation to the exercise of suffrage under the 1987
Constitution.66

The contestation is untenable.

As early as the 1936 case of The People of the Philippine Islands v. Corral,67 it has been recognized that
"[t]he right to vote is not a natural right but is a right created by law. Suffrage is a privilege granted
by the State to such persons or classes as are most likely to exercise it for the public good. In
the early stages of the evolution of the representative system of government, the exercise of the right of
suffrage was limited to a small portion of the inhabitants. But with the spread of democratic ideas, the
enjoyment of the franchise in the modern states has come to embrace the mass of the audit classes of
persons are excluded from the franchise." 68

Section 1, Article V of the 1987 Constitution delineates the current parameters for the exercise of suffrage:
Section 1. Suffrage may be exercised by all citizens of the Philippines not otherwise disqualified by law,
who are at least eighteen years of age, and who shall have resided in the Philippines for at least one year
and in the place wherein they propose to vote for at least six months immediately preceding the election.
No literacy, property, or other substantive requirement shall be imposed on the exercise of suffrage.
Dissecting the provision, one must meet the following qualifications in order to exercise the right of
suffrage: first, he must be a Filipino citizen; second, he must not be disqualified by law; and third, he
must have resided in the Philippines for at least one (1) year and in the place wherein he proposes to vote
for at least six (6) months immediately preceding the election.

The second item more prominently reflects the franchised nature of the right of suffrage. The State may
therefore regulate said right by imposing statutory disqualifications, with the restriction, however, that the
same do not amount to, as per the second sentence of the provision, a "literacy, property or other
substantive requirement." Based on its genesis, it may be gleaned that the limitation is geared towards
the elimination of irrelevant standards that are purely based on socio-economic considerations that have
no bearing on the right of a citizen to intelligently cast his vote and to further the public good.

To contextualize, the first Philippine Election Law, Act No. 1582, which took effect on January 15, 1907,
mandated that only men who were at least twenty-three (23) years old and "comprised within one of the
following three classes" were allowed to vote: (a) those who prior to the 13 th of August, 1898, held the
office of municipal captain, governadorcillo, alcalde, lieutenant, cabeza de barangay, or member of
any ayuntamiento; (b) those who own real property to the value of P500.00, or who annually pay P30.00
or more of the established taxes; and (c) those, who speak, read, and write English or Spanish.

When the 1935 Constitution was adopted, the minimum voting age was lowered to twenty-one (21) and
the foregoing class qualification and property requirements were removed. 69 However, the literacy
requirement was retained and only men who were able to read and write were given the right to vote. 70 It
also made women's right to vote dependent on a plebiscite held for such purpose. 71

During the 1971 Constitutional Convention, the delegates decided to remove the literacy and property
requirements to broaden the political base and discontinue the exclusion of millions of citizens from the
political systems:72
Sponsorship Speech of Delegate Manglapus

DELEGATE MANGLAPUS: Mr. President, the draft proposal, the subject matter of Report No. 11 contains
amendments that are designed to improve Article V on suffrage and to broaden the electoral base of our
country. The three main points that are taken up in this draft which will be developed in the sponsorship
speeches that will follow might need explanatory remarks, x x x.
xxxx

(2) The present requirement, reading and writing, is eliminated and instead a provision is introduced which
says, "No literacy, property, or other substantive requirement shall be imposed on the exercise
of suffrage;"

xxxx
The draft before us is in keeping with the trend towards the broadening of the electoral base
already begun with the lowering of the voting age to 18, and it is in keeping further with the
Committee's desire to discontinue the alienation and exclusion of millions of citizens from the
political system and from participation in the political life of the country. The requirement of
literacy for voting is eliminated for it is noted that there are very few countries left in the world where
literacy remains a condition for voting. There is no Southeast Asian country that imposes this requirement.
The United States Supreme Court only a few months ago declared unconstitutional any state law that
would continue to impose this requirement for voting.

xxxx

It is to be noted that all those who testified before the Committee favoured the elimination of the
literacy requirement. It must be stressed that those witnesses represented all levels of society x x x.

Sponsorship Speech of Delegate Ordoñez

x x x in the process, as we evolve, many and more of our people were left to the sidelines because they
could no longer participate in the process of government simply because their ability to read and write had
become inadequate. This, however, did not mean that they were no longer responsive to the demands of
the times, that they were unsensible to what was happening among them. And so in the process as years
went on, conscious efforts were made to liberate, to free these persons who were formerly entitled in the
course of election by means of whittling away the requirements for the exercise of the right to vote. First
of all, was the property requirement. There were times in the English constitutional history that it was
common to say as an answer to a question, "Who are entitled to vote?" that the following cannot vote - -
criminals, paupers, members of the House of Lords. They were landed together at the same figurative
category.

Eventually, with the wisdom of the times, property requirement was eliminated but the last remaining
vestige which bound the members of the community to ignorance, which was the persistence of this
requirement of literacy remained. And this is again preserved in our Constitution, in our Election Code,
which provides that those who cannot prepare their ballots themselves shall not be qualified to vote.

xxxx

Unless you remove this literacy test, the cultural minorities, the underprivileged, the urban
guerrillas will forever be outcasts of our society, irresponsive of what is happening. And if this
condition were to continue, my friends, we cannot fully claim that we have representative
democracy. Let us reverse the cycle. Let us eliminate the social imbalance by granting to these persons
who are very responsible the right to participate in the choice of the persons who are to make their laws
for them. (Emphases supplied)
As clarified on interpellation, the phrase "other substantive requirement" carries the same tack as the
other standards alienating particular classes based on socio-economic considerations irrelevant to suffrage,
such as the payment of taxes. Moreover, as particularly noted and as will be later elaborated on, the
phrase did not contemplate any restriction on procedural requirements, such as that of registration:
DELEGATE DE LOS REYES: On page 2, Line 3, the following appears:
"For other substantive requirement, no literacy[,] property, or other substantive requirement shall be
imposed on the exercise of suffrage."
just what is contemplated in the phrase, "substantive requirement?"

DELEGATE OCCEÑA: I can answer that, but it belongs to the sphere of someone else in the Committee. We
use this term as distinguished from procedural requirements. For instance, the law cannot come in
and say that those who should be allowed to vote should have paid certain taxes. That would be
a substantial requirement in addition to what is provided for in the Constitution. But the law can step in
as far as certain procedural requirements are concerned like requiring registration, and also
step in as far as these classifications are concerned. 73 (Emphases supplied)
As it finally turned out, the imposition of literacy, property, or other substantive requirement was
proscribed and the following provision on suffrage was adopted 74 in the 1973 Constitution:
Section 1. Suffrage shall be exercised by citizens of the Philippines not otherwise disqualified by law, who
are eighteen years of age or over, and who shall have resided in the Philippines for at least one year and
in the place wherein they propose to vote for at least six months preceding the election. No literacy,
property, or other substantive requirement shall be imposed on the exercise of suffrage. The
Batasang Pambansa shall provide a system for the purpose of securing the secrecy and sanctity of the
vote. (Emphasis supplied)
After deliberating on and eventually, striking down a proposal to exclude literacy requirements from the
limitation,75 the exact provision prohibiting the imposition of "literacy, property, or other substantive
requirement[s]" in the 1973 Constitution was fully adopted in the 1987 Constitution.

Along the contours of this limitation then, Congress, pursuant to Section 118 of Batas Pambansa Bilang
881, or the Omnibus Election Code, among others, imposed the following legal disqualifications:
Section 118. Disqualifications. - The following shall be disqualified from voting:
(a) Any person who has been sentenced by final judgment to suffer imprisonment for not less than one
year, such disability not having been removed by plenary pardon or granted amnesty: Provided, however,
That any person disqualified to vote under this paragraph shall automatically reacquire the right to vote
upon expiration of five years after service of sentence.

(b) Any person who has been adjudged by final judgment by competent court or tribunal of having
committed any crime involving disloyalty to the duly constituted government such as rebellion, sedition,
violation of the anti-subversion and firearms laws, or any crime against national security, unless restored
to his full civil and political rights in accordance with law: Provided, That he shall regain his right to vote
automatically upon expiration of five years after service of sentence.

(c) Insane or incompetent persons as declared by competent authority.


A "qualification" is loosely defined as "the possession of qualities, properties (such as fitness or capacity)
inherently or legally necessary to make one eligible for a position or office, or to perform a public duty or
function."76

Properly speaking, the concept of a "qualification", at least insofar as the discourse on suffrage is
concerned, should be distinguished from the concept of "registration", which is jurisprudentially regarded
as only the means by which a person's qualifications to vote is determined. In Yra v.
Abaño,77 citing Meffert v. Brown,78 it was stated that "[t]he act of registering is only one step towards
voting, and it is not one of the elements that makes the citizen a qualified voter [and] one may be a
qualified voter without exercising the right to vote." 79 In said case, this Court definitively characterized
registration as a form of regulation and not as a qualification for the right of suffrage:
Registration regulates the exercise of the right of suffrage. It is not a qualification for such
right.80 (Emphasis supplied)
As a form of regulation, compliance with the registration procedure is dutifully enjoined. Section 115 of the
Omnibus Election Code provides:
Section 115. Necessity of Registration. - In order that a qualified elector may vote in any election,
plebiscite or referendum, he must be registered in the permanent list of voters for the city or
municipality in which he resides. (Emphasis supplied)
Thus, although one is deemed to be a "qualified elector," he must nonetheless still comply with the
registration procedure in order to vote.

As the deliberations on the 1973 Constitution made clear, registration is a mere procedural requirement
which does not fall under the limitation that "[n]o literacy, property, or other substantive requirement shall
be imposed on the exercise of suffrage." This was echoed in AKBAYAN-Youth v. COMELEC81 (AKBAYAN-
Youth), wherein the Court pronounced that the process of registration is a procedural limitation on the
right to vote. Albeit procedural, the right of a citizen to vote nevertheless remains conditioned upon it:
Needless to say, the exercise of the right of suffrage, as in the enjoyment of all other rights, is subject to
existing substantive and procedural requirements embodied in our Constitution, statute books and other
repositories of law. Thus, as to the substantive aspect, Section 1, Article V of the Constitution provides: chanRoblesvirtualLawlibrary

xxxx

As to the procedural limitation, the right of a citizen to vote is necessarily conditioned upon
certain procedural requirements he must undergo: among others, the process of
registration. Specifically, a citizen in order to be qualified to exercise his right to vote, in addition to the
minimum requirements set by the fundamental charter, is obliged by law to register, at present, under the
provisions of Republic Act No. 8189, otherwise known as the Voters Registration Act of 1996. 82 (Emphasis
and underscoring supplied)
RA 8189 primarily governs the process of registration. It defines "registration" as "the act of accomplishing
and filing of a sworn application for registration by a qualified voter before the election officer of the city or
municipality wherein he resides and including the same in the book of registered voters upon approval by
the [ERB]."83 As stated in Section 2 thereof, RA 8189 was passed in order "to systematize the present
method of registration in order to establish a clean, complete, permanent and updated list of voters."

To complement RA 8189 in light of the advances in modern technology, RA 10367, or the assailed
Biometrics Law, was signed into law in February 2013. It built on the policy considerations behind RA 8189
as it institutionalized biometrics validation as part of the registration process:
Section 1. Declaration of Policy. - It is the policy of the State to establish a clean, complete, permanent
and updated list of voters through the adoption of biometric technology.
"Biometrics refers to a quantitative analysis that provides a positive identification of an individual such as
voice, photograph, fingerprint, signature, iris, and/or such other identifiable features." 84

Sections 3 and 10 of RA 10367 respectively require registered and new voters to submit themselves for
biometrics validation:
Section 3. Who Shall Submit for Validation. - Registered voters whose biometrics have not been captured
shall submit themselves for validation.

Section 10. Mandatory Biometrics Registration. - The Commission shall implement a mandatory biometrics
registration system for new voters.
Under Section 2 (d) of RA 10367, "validation" is defined as "the process of taking the biometrics of
registered voters whose biometrics have not yet been captured."

The consequence of non-compliance is "deactivation" which "refers to the removal of the registration
record of the registered voter from the corresponding precinct book of voters for failure to comply with the
validation process as required by [RA 10367]." 85 Section 7 states:
Section 7. Deactivation. - Voters who fail to submit for validation on or before the last day of filing of
application for registration for purposes of the May 2016 elections shall be deactivated pursuant to this
Act. (Emphases supplied)
Notably, the penalty of deactivation, as well as the requirement of validation, neutrally applies to all
voters. Thus, petitioners' argument that the law creates artificial class of voters 86 is more imagined than
real. There is no favor accorded to an "obedient group." If anything, non-compliance by the "disobedient"
only rightfully results into prescribed consequences. Surely, this is beyond the intended mantle of the
equal protection of the laws, which only works "against undue favor and individual or class privilege, as
well as hostile discrimination or the oppression of inequality." 87

It should also be pointed out that deactivation is not novel to RA 10367. RA 8189 already provides for
certain grounds for deactivation, of which not only the disqualifications under the Constitution or the
Omnibus Election are listed.
Section 27. Deactivation of Registration. The board shall deactivate the registration and remove the
registration records of the following persons from the corresponding precinct book of voters and place the
same, properly marked and dated in indelible ink, in the inactive file after entering the cause or causes of
deactivation:chanRoblesvirtualLawlibrary

a) Any person who has been sentenced by final judgment to suffer imprisonment for not less than one (1)
year, such disability not having been removed by plenary pardon or amnesty: Provided, however, That any
person disqualified to vote under this paragraph shall automatically reacquire the right to vote upon
expiration of five (5) years after service of sentence as certified by the clerks of courts of the
Municipal/Municipal Circuit/Metropolitan/Regional Trial Courts and the Sandiganbayan;

b) Any person who has been adjudged by final judgment by a competent court or tribunal of having
caused/committed any crime involving disloyalty to the duly constituted government such as rebellion,
sedition, violation of the anti-subversion and firearms laws, or any crime against national security, unless
restored to his full civil and political rights in accordance with law; Provided, That he shall regain his right
to vote automatically upon expiration of five (5) years after service of sentence;

c) Any person declared by competent authority to be insane or incompetent unless such disqualification
has been subsequently removed by a declaration of a proper authority that such person is no longer
insane or incompetent;

d) Any person who did not vote in the two (2) successive preceding regular elections as shown by their
voting records. For this purpose, regular elections do not include the Sangguniang Kabataan (SK)
elections;

e) Any person whose registration has been ordered excluded by the Court; and

f) Any person who has lost his Filipino citizenship.

For this purpose, the clerks of court for the Municipal/Municipal ( Circuit/Metropolitan/Regional Trial Courts
and the Sandiganbayan shall furnish the Election Officer of the city or municipality concerned at the end of
each month a certified list of persons who are disqualified under paragraph (a) hereof, with their
addresses. The Commission may request a certified list of persons who have lost their Filipino Citizenship
or declared as insane or incompetent with their addresses from other government agencies.

The Election Officer shall post in the bulletin board of his office a certified list of those persons whose
registration were deactivated and the reasons therefor, and furnish copies thereof to the local heads of
political parties, the national central file, provincial file, and the voter concerned.
With these considerations in mind, petitioners' claim that biometrics validation imposed under RA 10367,
and implemented under COMELEC Resolution Nos. 9721, 9863, and 10013, must perforce fail. To
reiterate, this requirement is not a "qualification" to the exercise of the right of suffrage, but a mere
aspect of the registration procedure, of which the State has the right to reasonably regulate. It was
institutionalized conformant to the limitations of the 1987 Constitution and is a mere complement to the
existing Voter's Registration Act of 1996. Petitioners would do well to be reminded of this Court's
pronouncement in AKBAYAN-Youth, wherein it was held that:
[T]he act of registration is an indispensable precondition to the right of suffrage. For registration is part
and parcel of the right to vote and an indispensable element in the election process. Thus, contrary to
petitioners' argument, registration cannot and should not be denigrated to the lowly stature of a mere
statutory requirement. Proceeding from the significance of registration as a necessary requisite to
the right to vote, the State undoubtedly, in the exercise of its inherent police power, may then
enact laws to safeguard and regulate the act of voter's registration for the ultimate purpose of
conducting honest, orderly and peaceful election, to the incidental yet generally important end, that
even pre-election activities could be performed by the duly constituted authorities in a realistic and orderly
manner - one which is not indifferent, and so far removed from the pressing order of the day and the
prevalent circumstances of the times.88 (Emphasis and underscoring supplied)
Thus, unless it is shown that a registration requirement rises to the level of a literacy, property or other
substantive requirement as contemplated by the Framers of the Constitution - that is, one which
propagates a socio-economic standard which is bereft of any rational basis to a person's ability to
intelligently cast his vote and to further the public good - the same cannot be struck down as
unconstitutional, as in this case.

III.

For another, petitioners assert that biometrics validation gravely violates the Constitution, considering
that, applying the strict scrutiny test, it is not poised with a compelling reason for state regulation and
hence, an unreasonable deprivation of the right to suffrage. 89 They cite the case of White Light Corp. v.
City of Manila90 (White Light), wherein the Court stated that the scope of the strict scrutiny test covers the
protection of the right of suffrage.91
Contrary to petitioners' assertion, the regulation passes the strict scrutiny test.

In terms of judicial review of statutes or ordinances, strict scrutiny refers to the standard for determining
the quality and the amount of governmental interest brought to justify the regulation of fundamental
freedoms. Strict scrutiny is used today to test the validity of laws dealing with the regulation of speech,
gender, or race as well as other fundamental rights as expansion from its earlier applications to equal
protection.92 As pointed out by petitioners, the United States Supreme Court has expanded the scope of
strict scrutiny to protect fundamental rights such as suffrage, judicial access, and interstate travel. 93

Applying strict scrutiny, the focus is on the presence of compelling, rather than substantial,
governmental interest and on the absence of less restrictive means for achieving that
interest,94 and the burden befalls upon the State to prove the same. 95

In this case, respondents have shown that the biometrics validation requirement under RA 10367
advances a compelling state interest. It was precisely designed to facilitate the conduct of orderly, honest,
and credible elections by containing - if not eliminating, the perennial problem of having flying voters, as
well as dead and multiple registrants. According to the sponsorship speech of Senator Aquilino L. Pimentel
III, the objective of the law was to cleanse the national voter registry so as to eliminate electoral fraud
and ensure that the results of the elections were truly reflective of the genuine will of the people. 96 The
foregoing consideration is unquestionably a compelling state interest.

Also, it was shown that the regulation is the least restrictive means for achieving the above-said interest.
Section 697 of Resolution No. 9721 sets the procedure for biometrics validation, whereby the registered
voter is only required to: (a) personally appear before the Office of the Election Officer; (b) present a
competent evidence of identity; and (c) have his photo, signature, and fingerprints recorded. It is, in
effect, a manner of updating one's registration for those already registered under RA 8189, or a first-time
registration for new registrants. The re-registration process is amply justified by the fact that the
government is adopting a novel technology like biometrics in order to address the bane of electoral fraud
that has enduringly plagued the electoral exercises in this country. While registrants may be
inconvenienced by waiting in long lines or by not being accommodated on certain days due to heavy
volume of work, these are typical burdens of voting that are remedied by bureaucratic improvements to be
implemented by the COMELEC as an administrative institution. By and large, the COMELEC has not turned
a blind eye to these realities. It has tried to account for the exigencies by holding continuous registration
as early as May 6, 2014 until October 31, 2015, or for over a period of 18 months. To make the validation
process as convenient as possible, the COMELEC even went to the extent of setting up off-site and satellite
biometrics registration in shopping malls and conducted the same on Sundays. 98 Moreover, it deserves
mentioning that RA 10367 and Resolution No. 9721 did not mandate registered voters to submit
themselves to validation every time there is an election. In fact, it only required the voter to undergo the
validation process one (1) time, which shall remain effective in succeeding elections, provided that he
remains an active voter. To add, the failure to validate did not preclude deactivated voters from exercising
their right to vote in the succeeding elections. To rectify such status, they could still apply for
reactivation99 following the procedure laid down in Section 28 100 of RA 8189.

That being said, the assailed regulation on the right to suffrage was sufficiently justified as it was indeed
narrowly tailored to achieve the compelling state interest of establishing a clean, complete, permanent and
updated list of voters, and was demonstrably the least restrictive means in promoting that interest. 101

IV.

Petitioners further aver that RA 10367 and the COMELEC Resolution Nos. 9721, 9863, and 10013 violate
the tenets of procedural due process because of the short periods of time between hearings and notice,
and the summary nature of the deactivation proceedings. 102

Petitioners are mistaken.

At the outset, it should be pointed out that the COMELEC, through Resolution No. 10013, had directed EOs
to: (a) "[p]ost the lists of voters without biometrics data in the bulletin boards of the City/Municipal hall,
Office of the Election Officer and in the barangay hall along with the notice of ERB hearing;" and (b) [s]end
individual notices to the affected voters included in the generated list of voters without biometrics
data.103 The same Resolution also accords concerned individuals the opportunity to file their
opposition/objection to the deactivation of VRRs not later than November 9, 2015 in accordance with the
period prescribed in Section 4,104 Chapter I, Resolution No. 9853. Meanwhile, Resolution Nos. 9721 and
9863 respectively state that "[d]eactivation x x x shall comply with the requirements on posting, ERB
hearing and service of individual notices to the deactivated voters," 105 and that the "Reactivation for cases
falling under this ground shall be made during the November 16, 2015 Board hearing." 106 While the
proceedings are summary in nature, the urgency of finalizing the voters' list for the upcoming May 2016
Elections calls for swift and immediate action on the deactivation of VRRs of voters who fail to comply with
the mandate of RA 10367. After all, in the preparation for the May 2016 National and Local Elections, time
is of the essence. The summary nature of the proceedings does not depart from the fact that petitioners
were given the opportunity to be heard.

Relatedly, it deserves emphasis that the public has been sufficiently informed of the implementation of RA
10367 and its deactivation feature. RA 10367 was duly published as early as February 22, 2013, 107 and
took effect fifteen (15) days after. 108 Accordingly, dating to the day of its publications, all are bound to
know the terms of its provisions, including the consequences of non-compliance. As implemented, the
process of biometrics validation commenced on July 1, 2013, or approximately two and a half (2 1/2)
years before the October 31, 2015 deadline. To add, the COMELEC conducted a massive public information
campaign, i.e., NoBio-NoBoto, from May 2014 until October 31, 2015, or a period of eighteen (18)
months, whereby voters were reminded to update and validate their registration records. On top of that,
the COMELEC exerted efforts to make the validation process more convenient for the public as it enlisted
the assistance of malls across Metro Manila to serve as satellite registration centers and declared Sundays
as working days for COMELEC offices within the National Capital Region and in highly urbanized
cities.109 Considering these steps, the Court finds that the public has been sufficiently apprised of the
implementation of RA 10367, and its penalty of deactivation in case of failure to comply. Thus, there was
no violation of procedural due process.

V.

Petitioners aver that the poor experience of other countries - i.e., Guatemala, Britain, Cote d'lvoire,
Uganda, and Kenya - in implementing biometrics registration should serve as warning in adhering to the
system. They highlighted the inherent difficulties in launching the same such as environmental and
geographical challenges, lack of training and skills, mechanical breakdown, and the need for re-
registration. They even adrnitted that while biometrics may address electoral fraud caused by multiple
registrants, it does not, however, solve other election-related problems such as vote-buying and source-
code manipulation.110

Aside from treading on mere speculation, the insinuations are improper. Clearly, petitioners' submissions
principally assail the wisdom of the legislature in adopting the biometrics registration system in curbing
electoral fraud. In this relation, it is significant to point out that questions relating to the wisdom, morality,
or practicability of statutes are policy matters that should not be addressed to the judiciary. As elucidated
in the case of Fariñas v. The Executive Secretary:111
[P]olicy matters are not the concern of the Court. Government policy is within the exclusive dominion
of the political branches of the government. It is not for this Court to look into the wisdom or
propriety of legislative determination. Indeed, whether an enactment is wise or unwise, whether it is
based on sound economic theory, whether it is the best means to achieve the desired results, whether, in
short, the legislative discretion within its prescribed limits should be exercised in a particular manner are
matters for the judgment of the legislature, and the serious, conflict of opinions does not suffice to bring
them within the range of judicial cognizance. 112 (Emphases and underscoring supplied)
In the exercise of its legislative power, Congress has a wide latitude of discretion to enact laws, such as RA
10367, to combat electoral fraud which, in this case, was through the establishment of an updated voter
registry. In making such choices to achieve its desired result, Congress has necessarily sifted through the
policy's wisdom, which this Court has no authority to review, much less reverse. 113 Whether RA 10367 was
wise or unwise, or was the best means in curtailing electoral fraud is a question that does not present a
justiciable issue cognizable by the courts. Indeed, the reason behind the legislature's choice of adopting
biometrics registration notwithstanding the experience of foreign countries, the difficulties in its
implementation, or its concomitant failure to address equally pressing election problems, is essentially a
policy question and, hence, beyond the pale of judicial scrutiny.

VI.

Finally, petitioners' proffer that Resolution No. 9863 which fixed the deadline for validation on October 31,
2015 violates Section 8 of RA 8189 which states:
Section 8. System of Continuing Registration of Voters. - The personal filing of application of registration of
voters shall be conducted daily in the office of the Election Officer during regular office hours. No
registration shall, however, be conducted during the period starting one hundred twenty (120)
days before a regular election and ninety (90) days before a special election. (Emphasis added.)
The position is, once more, wrong.

Aside from committing forum shopping by raising this issue despite already being subject of a prior
petition filed before this Court, i.e., G.R. No. 220918,114 petitioners fail to consider that the 120- and 90-
day periods stated therein refer to the prohibitive period beyond which voter registration may no longer be
conducted. As already resolved in this Court's Resolution dated December 8, 2015 in G.R. No. 220918, the
subject provision does not mandate COMELEC to conduct voter registration up to such time; rather, it only
provides a period which may not be reduced, but may be extended depending on the administrative
necessities and other exigencies.115 Verily, as the constitutional body tasked to enforce and implement
election laws, the COMELEC has the power to promulgate the necessary rules and regulations to fulfil its
mandate.116 Perforce, this power includes the determination of the periods to accomplish certain pre-
election acts,117 such as voter registration.
At this conclusory juncture, this Court reiterates that voter registration does not begin and end with the
filing of applications which, in reality, is just the initial phase that must be followed by the approval of
applications by the ERB.118 Thereafter, the process of filing petitions for inclusion and exclusion follows.
These steps are necessary for the generation of the1 final list of voters which, in turn, is a pre-requisite for
the preparation and completion of the Project of Precincts (POP) that is vital for the actual elections. The
POP contains the number of registered voters in each precinct and clustered precinct, the names of the
barangays, municipalities, cities, provinces, legislative districts, and regions included in the precincts, and
the names and locations of polling centers where each precinct and clustered precinct are assigned. 119 The
POP is necessary to determine the total number of boards of election inspectors to be constituted, the
allocation of forms and supplies to be procured for the election day, the number of vote counting machines
and other paraphernalia to be deployed, and the budget needed. More importantly, the POP will be used as
the basis for the fmalization of the Election Management System (EMS) which generates the templates of
the official ballots and determines the voting jurisdiction of legislative districts, cities, municipalities, and
provinces.120 The EMS determines the configuration of the canvassing and consolidation system for each
voting jurisdiction. Accordingly, as the constitutional body specifically charged with the enforcement and
administration of all laws and regulations relative to the conduct of an election, plebiscite, initiative,
referendum, and recall,121 the COMELEC should be given sufficient leeway in accounting for the exigencies
of the upcoming elections. In fine, its measures therefor should be respected, unless it is clearly shown
that the same are devoid of any reasonable justification.

WHEREFORE, the petition is DISMISSED due to lack of merit. The temporary restraining order issued by
this Court on December 1, 2015 is consequently DISSOLVED. SO ORDERED. ch

House of Representatives, Article VI , Sections 5-8


a. District Representatives and Questions of Apportionment
- Aquino III v Comelec, 617 SCRA 623 (2010)

This case comes before this Court by way of a Petition for Certiorari and Prohibition under Rule 65 of the Rules of Court.
In this original action, petitioners Senator Benigno Simeon C. Aquino III and Mayor Jesse Robredo, as public officers,
taxpayers and citizens, seek the nullification as unconstitutional of Republic Act No. 9716, entitled "An Act Reapportioning
the Composition of the First (1st) and Second (2nd) Legislative Districts in the Province of Camarines Sur and Thereby
Creating a New Legislative District From Such Reapportionment." Petitioners consequently pray that the respondent
Commission on Elections be restrained from making any issuances and from taking any steps relative to the
implementation of Republic Act No. 9716.

Republic Act No. 9716 originated from House Bill No. 4264, and was signed into law by President Gloria Macapagal
Arroyo on 12 October 2009. It took effect on 31 October 2009, or fifteen (15) days following its publication in the Manila
Standard, a newspaper of general circulation. 1 In substance, the said law created an additional legislative district for the
Province of Camarines Sur by reconfiguring the existing first and second legislative districts of the province.

Prior to Republic Act No. 9716, the Province of Camarines Sur was estimated to have a population of
1,693,821,2 distributed among four (4) legislative districts in this wise:

District Municipalities/Cities Population


1st District Del Gallego Libmanan
Ragay Minalabac 417,304
Lupi Pamplona
Sipocot Pasacao
Cabusao San
Fernando
2nd District Gainza Canaman
Milaor Camaligan 474,899
Naga Magarao
Pili Bombon
Ocampo Calabanga
3rd District Caramoan Sangay
Garchitorena San Jose 372,548
Goa Tigaon
Lagonoy Tinamba
Presentacion Siruma
4th District Iriga Buhi
Baao Bula 429,070
Balatan Nabua
Bato

Following the enactment of Republic Act No. 9716, the first and second districts of Camarines Sur were reconfigured in
order to create an additional legislative district for the province. Hence, the first district municipalities of Libmanan,
Minalabac, Pamplona, Pasacao, and San Fernando were combined with the second district municipalities of Milaor and
Gainza to form a new second legislative district. The following table 3 illustrates the reapportionment made by Republic Act
No. 9716:

District Municipalities/Cities Population


1st District Del Gallego  
Ragay 176,383
Lupi
Sipocot
Cabusao
2nd District Libmanan San
Minalabac Fernando 276,777
Pamplona Gainza
Pasacao Milaor
3rd District (formerly 2nd Naga Camaligan 439,043
District) Pili Magarao
Ocampo Bombon
Canaman Calabanga
4th District (formerly 3rd District) Caramoan Sangay 372,548
Garchitorena San Jose
Goa Tigaon
Lagonoy Tinamba
Presentacion Siruma
5th District (formerly 4th District) Iriga Buhi 429,070
Baao Bula
Balatan Nabua
Bato

Republic Act No. 9716 is a well-milled legislation. The factual recitals by both parties of the origins of the bill that became
the law show that, from the filing of House Bill No. 4264 until its approval by the Senate on a vote of thirteen (13) in favor
and two (2) against, the process progressed step by step, marked by public hearings on the sentiments and position of the
local officials of Camarines Sur on the creation of a new congressional district, as well as argumentation and debate on
the issue, now before us, concerning the stand of the oppositors of the bill that a population of at least 250,000 is required
by the Constitution for such new district.4

Petitioner Aquino III was one of two senators who voted against the approval of the Bill by the Senate. His co-petitioner,
Robredo, is the Mayor of Naga City, which was a part of the former second district from which the municipalities of Gainza
and Milaor were taken for inclusion in the new second district. No other local executive joined the two; neither did the
representatives of the former third and fourth districts of the province.

Petitioners contend that the reapportionment introduced by Republic Act No. 9716, runs afoul of the explicit constitutional
standard that requires a minimum population of two hundred fifty thousand (250,000) for the creation of a legislative
district.5 The petitioners claim that the reconfiguration by Republic Act No. 9716 of the first and second districts of
Camarines Sur is unconstitutional, because the proposed first district will end up with a population of less than 250,000 or
only 176,383.

Petitioners rely on Section 5(3), Article VI of the 1987 Constitution as basis for the cited 250,000 minimum population
standard.6 The provision reads:

Article VI

Section 5. (1) x x x x

(2) x x x x

(3) Each legislative district shall comprise, as far as practicable, contiguous, compact, and adjacent territory. Each
city with a population of at least two hundred fifty thousand, or each province, shall have at least one
representative.

(4) x x x x (Emphasis supplied).

The petitioners posit that the 250,000 figure appearing in the above-cited provision is the minimum population requirement
for the creation of a legislative district.7 The petitioners theorize that, save in the case of a newly created province, each
legislative district created by Congress must be supported by a minimum population of at least 250,000 in order to be
valid.8 Under this view, existing legislative districts may be reapportioned and severed to form new districts, provided each
resulting district will represent a population of at least 250,000. On the other hand, if the reapportionment would result in
the creation of a legislative seat representing a populace of less than 250,000 inhabitants, the reapportionment must be
stricken down as invalid for non-compliance with the minimum population requirement.
In support of their theory, the petitioners point to what they claim is the intent of the framers of the 1987 Constitution to
adopt a population minimum of 250,000 in the creation of additional legislative seats. 9 The petitioners argue that when the
Constitutional Commission fixed the original number of district seats in the House of Representatives to two hundred
(200), they took into account the projected national population of fifty five million (55,000,000) for the year
1986.10 According to the petitioners, 55 million people represented by 200 district representatives translates to roughly
250,000 people for every one (1) representative. 11 Thus, the 250,000 population requirement found in Section 5(3), Article
VI of the 1987 Constitution is actually based on the population constant used by the Constitutional Commission in
distributing the initial 200 legislative seats.

Thus did the petitioners claim that in reapportioning legislative districts independently from the creation of a province,
Congress is bound to observe a 250,000 population threshold, in the same manner that the Constitutional Commission did
in the original apportionment.

Verbatim, the submission is that:

1. Republic Act 9716 is unconstitutional because the newly apportioned first district of Camarines Sur failed to
meet the population requirement for the creation of the legislative district as explicitly provided in Article VI, Section
5, Paragraphs (1) and (3) of the Constitution and Section 3 of the Ordinance appended thereto; and

2. Republic Act 9716 violates the principle of proportional representation as provided in Article VI, Section 5
paragraphs (1), (3) and (4) of the Constitution. 12

The provision subject of this case states:

Article VI

Section 5. (1) The House of Representatives shall be composed of not more than two hundred and fifty members, unless
otherwise fixed by law, who shall be elected from legislative districts apportioned among the provinces, cities and the
Metropolitan Manila area in accordance with the number of their respective inhabitants, and on the basis of a uniform and
progressive ratio, and those who, as provided by law, shall be elected through a party-list system of registered national,
regional and sectoral parties or organizations.

(2) x x x x

(3) Each legislative district shall comprise, as far as practicable, contiguous, compact, and adjacent territory. Each
city with a population of at least two hundred fifty thousand, or each province, shall have at least one
representative.

(4) Within three years following the return of every census, the Congress shall make a reapportionment of
legislative districts based on the standards provided in this section.

On the other hand, the respondents, through the Office of the Solicitor General, seek the dismissal of the present petition
based on procedural and substantive grounds.

On procedural matters, the respondents argue that the petitioners are guilty of two (2) fatal technical defects: first,
petitioners committed an error in choosing to assail the constitutionality of Republic Act No. 9716 via the remedy of
Certiorari and Prohibition under Rule 65 of the Rules of Court; and second, the petitioners have no locus standi to
question the constitutionality of Republic Act No. 9716.

On substantive matters, the respondents call attention to an apparent distinction between cities and provinces drawn by
Section 5(3), Article VI of the 1987 Constitution. The respondents concede the existence of a 250,000 population
condition, but argue that a plain and simple reading of the questioned provision will show that the same has no application
with respect to the creation of legislative districts in provinces.13 Rather, the 250,000 minimum population is only a
requirement for the creation of a legislative district in a city.

In sum, the respondents deny the existence of a fixed population requirement for the reapportionment of districts in
provinces. Therefore, Republic Act No. 9716, which only creates an additional legislative district within the province of
Camarines Sur, should be sustained as a perfectly valid reapportionment law.

We first pass upon the threshold issues.

The respondents assert that by choosing to avail themselves of the remedies of Certiorari and Prohibition, the petitioners
have committed a fatal procedural lapse. The respondents cite the following reasons:

1. The instant petition is bereft of any allegation that the respondents had acted without or in excess of jurisdiction,
or with grave abuse of discretion. 1avvphi1

2. The remedy of Certiorari and Prohibition must be directed against a tribunal, board, officer or person, whether
exercising judicial, quasi-judicial, or ministerial functions. Respondents maintain that in implementing Republic Act
No. 9716, they were not acting as a judicial or quasi-judicial body, nor were they engaging in the performance of a
ministerial act.

3. The petitioners could have availed themselves of another plain, speedy and adequate remedy in the ordinary
course of law. Considering that the main thrust of the instant petition is the declaration of unconstitutionality of
Republic Act No. 9716, the same could have been ventilated through a petition for declaratory relief, over which
the Supreme Court has only appellate, not original jurisdiction.

The respondents likewise allege that the petitioners had failed to show that they had sustained, or is in danger of
sustaining any substantial injury as a result of the implementation of Republic Act No. 9716. The respondents, therefore,
conclude that the petitioners lack the required legal standing to question the constitutionality of Republic Act No. 9716.

This Court has paved the way away from procedural debates when confronted with issues that, by reason of constitutional
importance, need a direct focus of the arguments on their content and substance.

The Supreme Court has, on more than one occasion, tempered the application of procedural rules, 14 as well as relaxed the
requirement of locus standi whenever confronted with an important issue of overreaching significance to society. 15

Hence, in Del Mar v. Philippine Amusement and Gaming Corporation (PAGCOR) 16 and Jaworski v. PAGCOR,17 this Court
sanctioned momentary deviation from the principle of the hierarchy of courts, and took original cognizance of cases raising
issues of paramount public importance. The Jaworski case ratiocinates:

Granting arguendo that the present action cannot be properly treated as a petition for prohibition, the transcendental
importance of the issues involved in this case warrants that we set aside the technical defects and take primary jurisdiction
over the petition at bar. One cannot deny that the issues raised herein have potentially pervasive influence on the social
and moral well being of this nation, specially the youth; hence, their proper and just determination is an imperative
need. This is in accordance with the well-entrenched principle that rules of procedure are not inflexible tools designed to
hinder or delay, but to facilitate and promote the administration of justice. Their strict and rigid application, which would
result in technicalities that tend to frustrate, rather than promote substantial justice, must always be eschewed. (Emphasis
supplied)

Anent the locus standi requirement, this Court has already uniformly ruled in Kilosbayan v. Guingona, 18 Tatad v. Executive
Secretary,19 Chavez v. Public Estates Authority20 and Bagong Alyansang Makabayan v. Zamora,21 just to name a few, that
absence of direct injury on the part of the party seeking judicial review may be excused when the latter is able to craft an
issue of transcendental importance. In Lim v. Executive Secretary,22 this Court held that in cases of transcendental
importance, the cases must be settled promptly and definitely, and so, the standing requirements may be relaxed. This
liberal stance has been echoed in the more recent decision on Chavez v. Gonzales. 23

Given the weight of the issue raised in the instant petition, the foregoing principles must apply. The beaten path must be
taken. We go directly to the determination of whether or not a population of 250,000 is an indispensable constitutional
requirement for the creation of a new legislative district in a province.

We deny the petition.

We start with the basics. Any law duly enacted by Congress carries with it the presumption of constitutionality. 24 Before a
law may be declared unconstitutional by this Court, there must be a clear showing that a specific provision of the
fundamental law has been violated or transgressed. When there is neither a violation of a specific provision of the
Constitution nor any proof showing that there is such a violation, the presumption of constitutionality will prevail and the
law must be upheld. To doubt is to sustain. 25

There is no specific provision in the Constitution that fixes a 250,000 minimum population that must compose a legislative
district.

As already mentioned, the petitioners rely on the second sentence of Section 5(3), Article VI of the 1987 Constitution,
coupled with what they perceive to be the intent of the framers of the Constitution to adopt a minimum population of
250,000 for each legislative district.

The second sentence of Section 5(3), Article VI of the Constitution, succinctly provides: "Each city with a population of at
least two hundred fifty thousand, or each province, shall have at least one representative."

The provision draws a plain and clear distinction between the entitlement of a city to a district on one hand, and the
entitlement of a province to a district on the other. For while a province is entitled to at least a representative, with nothing
mentioned about population, a city must first meet a population minimum of 250,000 in order to be similarly entitled.

The use by the subject provision of a comma to separate the phrase "each city with a population of at least two hundred
fifty thousand" from the phrase "or each province" point to no other conclusion than that the 250,000 minimum population
is only required for a city, but not for a province. 26

Plainly read, Section 5(3) of the Constitution requires a 250,000 minimum population only for a city to be entitled to a
representative, but not so for a province.
The 250,000 minimum population requirement for legislative districts in cities was, in turn, the subject of interpretation by
this Court in Mariano, Jr. v. COMELEC.27

In Mariano, the issue presented was the constitutionality of Republic Act No. 7854, which was the law that converted the
Municipality of Makati into a Highly Urbanized City. As it happened, Republic Act No. 7854 created an additional legislative
district for Makati, which at that time was a lone district. The petitioners in that case argued that the creation of an
additional district would violate Section 5(3), Article VI of the Constitution, because the resulting districts would be
supported by a population of less than 250,000, considering that Makati had a total population of only 450,000. The
Supreme Court sustained the constitutionality of the law and the validity of the newly created district, explaining the
operation of the Constitutional phrase "each city with a population of at least two hundred fifty thousand," to wit:

Petitioners cannot insist that the addition of another legislative district in Makati is not in accord with section 5(3), Article
VI of the Constitution for as of the latest survey (1990 census), the population of Makati stands at only four hundred fifty
thousand (450,000). Said section provides, inter alia, that a city with a population of at least two hundred fifty
thousand (250,000) shall have at least one representative. Even granting that the population of Makati as of the 1990
census stood at four hundred fifty thousand (450,000), its legislative district may still be increased since it has met the
minimum population requirement of two hundred fifty thousand (250,000). In fact, Section 3 of the Ordinance appended to
the Constitution provides that a city whose population has  increased to more than two hundred fifty thousand
(250,000)  shall be entitled to at least one congressional representative.28 (Emphasis supplied)

The Mariano case limited the application of the 250,000 minimum population requirement for cities only to its initial
legislative district. In other words, while Section 5(3), Article VI of the Constitution requires a city to have a minimum
population of 250,000 to be entitled to a representative, it does not have to increase its population by another 250,000 to
be entitled to an additional district.

There is no reason why the Mariano case, which involves the creation of an additional district within a city, should not be
applied to additional districts in provinces. Indeed, if an additional legislative district created within a city is not required to
represent a population of at least 250,000 in order to be valid, neither should such be needed for an additional district in a
province, considering moreover that a province is entitled to an initial seat by the mere fact of its creation and regardless
of its population.

Apropos for discussion is the provision of the Local Government Code on the creation of a province which, by virtue of and
upon creation, is entitled to at least a legislative district. Thus, Section 461 of the Local Government Code states:

Requisites for Creation. – (a) A province may be created if it has an average annual income, as certified by the
Department of Finance, of not less than Twenty million pesos (P20,000,000.00) based on 1991 constant prices and either
of the following requisites:

(i) a contiguous territory of at least two thousand (2,000) square kilometers, as certified by the Lands Management
Bureau; or

(ii) a population of not less than two hundred fifty thousand (250,000) inhabitants as certified by the National
Statistics Office.

Notably, the requirement of population is not an indispensable requirement, but is merely an alternative addition to the
indispensable income requirement.

Mariano, it would turn out, is but a reflection of the pertinent ideas that ran through the deliberations on the words and
meaning of Section 5 of Article VI.

The whats, whys, and wherefores of the population requirement of "at least two hundred fifty thousand" may be gleaned
from the records of the Constitutional Commission which, upon framing the provisions of Section 5 of Article VI, proceeded
to form an ordinance that would be appended to the final document. The Ordinance is captioned "APPORTIONING THE
SEATS OF THE HOUSE OF REPRESENTATIVES OF THE CONGRESS OF THE PHILIPPINES TO THE DIFFERENT
LEGISLATIVE DISTRICTS IN PROVINCES AND CITIES AND THE METROPOLITAN MANILA AREA." Such records
would show that the 250,000 population benchmark was used for the 1986 nationwide apportionment of legislative
districts among provinces, cities and Metropolitan Manila. Simply put, the population figure was used to determine how
many districts a province, city, or Metropolitan Manila should have. Simply discernible too is the fact that, for the purpose,
population had to be the determinant. Even then, the requirement of 250,000 inhabitants was not taken as an absolute
minimum for one legislative district. And, closer to the point herein at issue, in the determination of the precise district
within the province to which, through the use of the population benchmark, so many districts have been apportioned,
population as a factor was not the sole, though it was among, several determinants.

From its journal,29 we can see that the Constitutional Commission originally divided the entire country into two hundred
(200) districts, which corresponded to the original number of district representatives. The 200 seats were distributed by the
Constitutional Commission in this manner: first, one (1) seat each was given to the seventy-three (73) provinces and the
ten (10) cities with a population of at least 250,000; 30 second, the remaining seats were then redistributed among the
provinces, cities and the Metropolitan Area "in accordance with the number of their inhabitants on the basis of a uniform
and progressive ratio."31 Commissioner Davide, who later became a Member and then Chief Justice of the Court,
explained this in his sponsorship remark32 for the Ordinance to be appended to the 1987 Constitution:
Commissioner Davide: The ordinance fixes at 200 the number of legislative seats which are, in turn, apportioned among
provinces and cities with a population of at least 250, 000 and the Metropolitan Area in accordance with the number of
their respective inhabitants on the basis of a uniform and progressive ratio. The population is based on the 1986
projection, with the 1980 official enumeration as the point of reckoning. This projection indicates that our population is
more or less 56 million. Taking into account the mandate that each city with at least 250, 000 inhabitants and each
province shall have at least one representative, we first allotted one seat for each of the 73 provinces, and each one for all
cities with a population of at least 250, 000, which are the Cities of Manila, Quezon, Pasay, Caloocan, Cebu, Iloilo,
Bacolod, Cagayan de Oro, Davao and Zamboanga. Thereafter, we then proceed[ed] to increase whenever appropriate the
number of seats for the provinces and cities in accordance with the number of their inhabitants on the basis of a uniform
and progressive ratio. (Emphasis supplied).

Thus was the number of seats computed for each province and city. Differentiated from this, the determination of the
districts within the province had to consider "all protests and complaints formally received" which, the records show, dealt
with determinants other than population as already mentioned.

Palawan is a case in point. Journal No. 107 of the Constitutional Commission narrates:

INTERPELLATION OF MR. NOLLEDO:

Mr. Nolledo inquired on the reason for including Puerto Princesa in the northern towns when it was more affinity with the
southern town of Aborlan, Batarasa, Brooke’s Point, Narra, Quezon and Marcos. He stated that the First District has a
greater area than the Second District. He then queried whether population was the only factor considered by the
Committee in redistricting.

Replying thereto, Mr. Davide explained that the Committee took into account the standards set in Section 5 of the Article
on the Legislative Department, namely: 1) the legislative seats should be apportioned among the provinces and cities and
the Metropolitan Manila area in accordance with their inhabitants on the basis of a uniform and progressive ratio; and 2)
the legislative district must be compact, adjacent and contiguous.

Mr. Nolledo pointed out that the last factor was not met when Puerto Princesa was included with the northern towns. He
then inquired what is the distance between Puerto Princesa from San Vicente.

xxxx

Thereupon, Mr. Nolledo stated that Puerto Princesa has a population of 75,480 and based on the apportionment, its
inclusion with the northern towns would result in a combined population of 265,000 as against only 186,000 for the south.
He added that Cuyo and Coron are very important towns in the northern part of Palawan and, in fact, Cuyo was the capital
of Palawan before its transfer to Puerto Princesa. He also pointed out that there are more potential candidates in the north
and therefore if Puerto Princesa City and the towns of Cuyo and Coron are lumped together, there would be less
candidates in the south, most of whose inhabitants are not interested in politics. He then suggested that Puerto Princesa
be included in the south or the Second District.

Mr. Davide stated that the proposal would be considered during the period of amendments. He requested that the
COMELEC staff study said proposal.33

"PROPOSED AMENDMENT OF MR. NOLLEDO

On the districting of Palawan, Mr. Nolledo pointed out that it was explained in the interpellations that District I has a total
population of 265,358 including the City of Puerto Princesa, while the Second District has a total population of 186,733. He
proposed, however, that Puerto Princesa be included in the Second District in order to satisfy the contiguity requirement in
the Constitution considering that said City is nearer the southern towns comprising the Second District.

In reply to Mr. Monsod’s query, Mr. Nolledo explained that with the proposed transfer of Puerto Princesa City to the
Second District, the First District would only have a total population of 190,000 while the Second District would have
262,213, and there would be no substantial changes.

Mr. Davide accepted Mr. Nolledo’s proposal to insert Puerto Princesa City before the Municipality of Aborlan.

There being no objection on the part of the Members the same was approved by the Body.

APPROVAL OF THE APPORTIONMENT AND DISTRICTING OF PALAWAN

There being no other amendment, on motion of Mr. Davide, there being no objection, the apportionment and districting for
the province of Palawan was approved by the Body.34

The districting of Palawan disregarded the 250,000 population figure. It was decided by the importance of the towns and
the city that eventually composed the districts.

Benguet and Baguio are another reference point. The Journal further narrates:
At this juncture, Mr. Davide informed the Body that Mr. Regalado made a reservation with the Committee for the possible
reopening of the approval of Region I with respect to Benguet and Baguio City.

REMARKS OF MR. REGALADO

Mr. Regalado stated that in the formulation of the Committee, Baguio City and Tuba are placed in one district. He stated
that he was toying with the idea that, perhaps as a special consideration for Baguio because it is the summer capital of the
Philippines, Tuba could be divorced from Baguio City so that it could, by itself, have its own constituency and Tuba could
be transferred to the Second District together with Itogon. Mr. Davide, however, pointed out that the population of Baguio
City is only 141,149.

Mr. Regalado admitted that the regular population of Baguio may be lower during certain times of the year, but the
transient population would increase the population substantially and, therefore, for purposes of business and professional
transactions, it is beyond question that population-wise, Baguio would more than qualify, not to speak of the official
business matters, transactions and offices that are also there.

Mr. Davide adverted to Director de Lima’s statement that unless Tuba and Baguio City are united, Tuba will be isolated
from the rest of Benguet as the place can only be reached by passing through Baguio City. He stated that the Committee
would submit the matter to the Body.

Upon inquiry of the Chair whether he is insisting on his amendment, Mr. Regalado stated that the Body should have a say
on the matter and that the considerations he had given are not on the demographic aspects but on the fact that Baguio
City is the summer capital, the venue and situs of many government offices and functions.

On motion of Mr. Davide, there being no objection, the Body approved the reconsideration of the earlier approval of the
apportionment and districting of Region I, particularly Benguet.

Thereafter, on motion of Mr. Davide, there being no objection, the amendment of Mr. Regalado was put to a vote. With 14
Members voting in favor and none against, the amendment was approved by the Body.

Mr. Davide informed that in view of the approval of the amendment, Benguet with Baguio City will have two seats. The
First District shall comprise of the municipalities of Mankayan, Buguias, Bakun, Kabayan, Kibungan, Bokod, Atok,
Kapangan, Tublay, La Trinidad, Sablan, Itogon and Tuba. The Second District shall comprise of Baguio City alone.

There being no objection, the Body approved the apportionment and districting of Region I. 35

Quite emphatically, population was explicitly removed as a factor.

It may be additionally mentioned that the province of Cavite was divided into districts based on the distribution of its three
cities, with each district having a city: one district "supposed to be a fishing area; another a vegetable and fruit area; and
the third, a rice growing area," because such consideration "fosters common interests in line with the standard of
compactness."36 In the districting of Maguindanao, among the matters discussed were "political stability and common
interest among the people in the area" and the possibility of "chaos and disunity" considering the "accepted regional,
political, traditional and sectoral leaders." 37 For Laguna, it was mentioned that municipalities in the highland should not be
grouped with the towns in the lowland. For Cebu, Commissioner Maambong proposed that they should "balance the area
and population."38

Consistent with Mariano and with the framer deliberations on district apportionment, we stated in Bagabuyo v.
COMELEC39 that:

x x x Undeniably, these figures show a disparity in the population sizes of the districts. The Constitution, however, does
not require mathematical exactitude or rigid equality as a standard in gauging equality of representation. x x x. To ensure
quality representation through commonality of interests and ease of access by the representative to the constituents, all
that the Constitution requires is that every legislative district should comprise, as far as practicable, contiguous, compact
and adjacent territory. (Emphasis supplied).

This 2008 pronouncement is fresh reasoning against the uncompromising stand of petitioner that an additional provincial
legislative district, which does not have at least a 250,000 population is not allowed by the Constitution.

The foregoing reading and review lead to a clear lesson.

Neither in the text nor in the essence of Section 5, Article VI of the Constitution can, the petition find support. And the
formulation of the Ordinance in the implementation of the provision, nay, even the Ordinance itself, refutes the contention
that a population of 250,000 is a constitutional sine qua non for the formation of an additional legislative district in a
province, whose population growth has increased beyond the 1986 numbers.

Translated in the terms of the present case:

1. The Province of Camarines Sur, with an estimated population of 1,693,821 in 2007 is ─ based on the formula
and constant number of 250,000 used by the Constitutional Commission in nationally apportioning legislative
districts among provinces and cities ─ entitled to two (2) districts in addition to the four (4) that it was given in the
1986 apportionment. Significantly, petitioner Aquino concedes this point. 40 In other words, Section 5 of Article VI as
clearly written allows and does not prohibit an additional district for the Province of Camarines Sur, such as that
provided for in Republic Act No. 9786;

2. Based on the pith and pitch of the exchanges on the Ordinance on the protests and complaints against strict
conformity with the population standard, and more importantly based on the final districting in the Ordinance on
considerations other than population, the reapportionment or the recomposition of the first and second legislative
districts in the Province of Camarines Sur that resulted in the creation of a new legislative district is valid even
if the population of the new district is 176,383 and not 250,000 as insisted upon by the petitioners.

3. The factors mentioned during the deliberations on House Bill No. 4264, were:

(a) the dialects spoken in the grouped municipalities;

(b) the size of the original groupings compared to that of the regrouped municipalities;

(c) the natural division separating the municipality subject of the discussion from the reconfigured District
One; and

(d) the balancing of the areas of the three districts resulting from the redistricting of Districts One and
Two.41

Each of such factors and in relation to the others considered together, with the increased population of the erstwhile
Districts One and Two, point to the utter absence of abuse of discretion, much less grave abuse of discretion, 42 that would
warrant the invalidation of Republic Act No. 9716.

To be clear about our judgment, we do not say that in the reapportionment of the first and second legislative districts of
Camarines Sur, the number of inhabitants in the resulting additional district should not be considered. Our ruling is that
population is not the only factor but is just one of several other factors in the composition of the additional district. Such
settlement is in accord with both the text of the Constitution and the spirit of the letter, so very clearly given form in the
Constitutional debates on the exact issue presented by this petition. 1avvphi1

WHEREFORE, the petition is hereby DISMISSED. Republic Act No. 9716 entitled "An Act Reapportioning the
Composition of the First (1st) and Second (2nd) Legislative Districts in the Province of Camarines Sur and Thereby
Creating a New Legislative District From Such Reapportionment" is a VALID LAW. SO ORDERED.

- Alaba v Comelec, 611 SCRA 147 (2010)

This resolves the motion for reconsideration of respondent Commission on Elections (COMELEC) of the Decision dated
25 January 2010.1

The COMELEC grounds its motion on the singular reason, already considered and rejected in the Decision, that
Congress’ reliance on the Certification of Alberto N. Miranda (Miranda), Region III Director, National Statistics Office
(NSO), projecting Malolos City’s population in 2010, is non-justiciable. The COMELEC also calls attention to the other
sources of Malolos City’s population indicators as of 2007 (2007 Census of Population – PMS 3 – Progress Enumeration
Report2) and as of 2008 (Certification of the City of Malolos’ Water District, dated 31 July 2008, 3 and Certification of the
Liga ng Barangay, dated 22 August 2008 4) which Congress allegedly used in enacting Republic Act No. 9591 (RA 9591).
The COMELEC extends its non-justiciability argument to these materials.

We find no reason to grant the motion.

First. It will not do for the COMELEC to insist that the reliability and authoritativeness of the population indicators Congress
used in enacting RA 9591 are non-justiciable. If laws creating legislative districts are unquestionably within the ambit of
this Court’s judicial review power,5 then there is more reason to hold justiciable subsidiary questions impacting on their
constitutionality, such as their compliance with a specific constitutional limitation under Section 5(3), Article VI of the 1987
Constitution that only cities with at least 250,000 constituents are entitled to representation in Congress. To fulfill this
obligation, the Court, of necessity, must inquire into the authoritativeness and reliability of the population indicators
Congress used to comply with the constitutional limitation. Thus, nearly five decades ago, we already rejected claims of
non-justiciability of an apportionment law alleged to violate the constitutional requirement of proportional representation:

It is argued in the motion to reconsider, that since Republic Act 3040 improves existing conditions, this Court could
perhaps, in the exercise of judicial statesmanship, consider the question involved as purely political and therefore non-
justiciable. The overwhelming weight of authority is that district apportionment laws are subject to review by the courts[:]

The constitutionality of a legislative apportionment act is a judicial question, and not one which the court cannot consider
on the ground that it is a political question.

It is well settled that the passage of apportionment acts is not so exclusively within the political power of the legislature as
to preclude a court from inquiring into their constitutionality when the question is properly brought before it.
It may be added in this connection, that the mere impact of the suit upon the political situation does not render it political
instead of judicial.

The alleged circumstance that this statute improves the present set-up constitutes no excuse for approving a
transgression of constitutional limitations, because the end does not justify the means. Furthermore, there is no reason to
doubt that, aware of the existing inequality of representation, and impelled by its sense of duty, Congress will opportunely
approve remedial legislation in accord with the precepts of the Constitution. 6 (Emphasis supplied; internal citations
omitted)

To deny the Court the exercise of its judicial review power over RA 9591 is to contend that this Court has no power "to
determine whether or not there has been a grave abuse of discretion amounting to lack or excess of jurisdiction on the
part of any branch or instrumentality of the Government," a duty mandated under Section 1, Article VIII of the Constitution.
Indeed, if we subscribe to the COMELEC’s theory, this Court would be reduced to rubberstamping laws creating legislative
districts no matter how unreliable and non-authoritative the population indicators Congress used to justify their creation.
There can be no surer way to render meaningless the limitation in Section 5(3), Article VI of the 1987 Constitution. 7

Second. Under Executive Order No. 135 (EO 135), the population indicators Congress used to measure Malolos City’s
compliance with the constitutional limitation are unreliable and non-authoritative. On Miranda’s Certification, (that the
"projected population of the [City] of Malolos will be 254,030 by the year 2010 using the population growth rate of 3.78[%]
between 1995 and 2000"), this fell short of EO 135’s requirements that (a) for intercensal years, the certification should be
based on a set of demographic projections and estimates declared official by the National Statistical and Coordination
Board (NSCB); (b) certifications on intercensal population estimates will be as of the middle of every year; and (c)
certifications based on projections or estimates must be issued by the NSO Administrator or his designated certifying
officer. Further, using Miranda’s own growth rate assumption of 3.78%, Malolos City’s population as of 1 August 2010 will
only be 249,333, below the constitutional threshold of 250,000 (using as base Malolos City’s population as of 1 August
2007 which is 223,069). That Miranda issued his Certification "by authority of the NSO administrator" does not make the
document reliable as it neither makes Miranda the NSO Administrator’s designated certifying officer nor cures the
Certification of its fatal defects for failing to use demographic projections and estimates declared official by the NSCB or
make the projection as of the middle of 2010. 1avvphi1

Nor are the 2007 Census of Population – PMS 3 – Progress Enumeration Report, the Certification of the City of Malolos’
Water District, dated 31 July 2008 and the Certification of the Liga ng Barangay, dated 22 August 2008, reliable because
none of them qualifies as authoritative population indicator under EO 135. The 2007 Census of Population – PMS 3 –
Progress Enumeration Report merely contains preliminary data on the population census of Bulacan which were
subsequently adjusted to reflect actual population as indicated in the 2007 Census results (showing Malolos City’s
population at 223,069). The COMELEC, through the Office of the Solicitor General (OSG), adopts Malolos City’s claim that
the 2007 census for Malolos City was "sloped to make it appear that come Year 2010, the population count for Malolos
would still fall short of the constitutional requirement." 8 This unbecoming attack by the government’s chief counsel on the
integrity of the processes of the government’s census authority has no place in our judicial system. The OSG ought to
know that absent convincing proof of so-called data "sloping," the NSO enjoys the presumption of the regularity in the
performance of its functions.

The Certification of the City of Malolos’ Water District fares no better. EO 135 excludes from its ambit certifications from a
public utility gathered incidentally in the course of pursuing its business. To elevate the water district’s so-called population
census to the level of credibility NSO certifications enjoy is to render useless the existence of NSO. This will allow
population data incidentally gathered by electric, telephone, sewage, and other utilities to enter into legislative processes
even though these private entities are not in the business of generating statistical data and thus lack the scientific training,
experience and competence to handle, collate and process them.

Similarly, the Certification of the Liga ng Barangay is not authoritative because much like the Malolos City Water District,
the Liga ng Barangay is not authorized to conduct population census, much less during off-census years. The non-NSO
entities EO 135 authorizes to conduct population census are local government units (that is, province, city, municipality or
barangay) subject to the prior approval of the NSCB and

under the technical supervision of the NSO from planning to data processing. 9

By presenting these alternative population indicators with their widely divergent population figures, 10 the COMELEC
unwittingly highlighted the danger of relying on non-NSO authorized certifications. EO 135’s stringent standards ensuring
reliability of population census cannot be diluted as these data lie at the core of crucial government decisions and, in this
case, the legislative function of enforcing the constitutional mandate of creating congressional districts in cities with at
least 250,000 constituents.

There can be no doubt on the applicability of EO 135 to test the constitutionality of RA 9591. The COMELEC invoked EO
135 to convince the Court of the credibility and authoritativeness of Miranda’s certificate. 11 It is hardly alien for the Court to
adopt standards contained in a parallel statute to fill gaps in the law in the absence of an express prohibition. 12 Indeed, one
is hard-pressed to find any distinction, statistically speaking, on the reliability of an NSO certification of a city’s population
for purposes of creating its legislative district and for purposes of converting it to a highly-urbanized or an independent
component city.13 Congress itself confirms the wisdom and relevance of EO 135’s paradigm of privileging NSO
certifications by mandating that compliance with the population requirement in the creation and conversion of local
government units shall be proved exclusively by an NSO certification. 14 Unquestionably, representation in Congress is no
less important than the creation of local government units in enhancing our democratic institutions, thus both processes
should be subject to the same stringent standards.
Third. Malolos City is entitled to representation in Congress only if, before the 10 May 2010 elections, it breaches the
250,000 population mark following the mandate in Section 3 of the Ordinance appended to the 1987 Constitution that "any
city whose population may hereafter increase to more than two hundred fifty thousand shall be entitled in the immediately
following election to at least one Member." COMELEC neither alleged nor proved that Malolos City is in compliance with
Section 3 of the Ordinance.

Fourth. Aside from failing to comply with Section 5(3), Article VI of the Constitution on the population requirement, the
creation by RA 9591 of a legislative district for Malolos City, carving the city from the former First Legislative District,
leaves the town of Bulacan isolated from the rest of the geographic mass of that district. 15 This contravenes the
requirement in Section 5(3), Article VI that each legislative district shall "comprise, as far as practicable, contiguous,
compact, and adjacent territory." It is no argument to say, as the OSG does, that it was impracticable for Congress to
create a district with contiguous, compact, and adjacent territory because Malolos city lies at the center of the First
Legislative District. The geographic lay-out of the First Legislative District is not an insuperable condition making
compliance with Section 5(3) impracticable. To adhere to the constitutional mandate, and thus maintain fidelity to its
purpose of ensuring efficient representation, the practicable alternative for Congress was to include the municipality of
Bulacan in Malolos City’s legislative district. Although unorthodox, the resulting contiguous and compact district fulfills the
constitutional requirements of geographic unity and population floor, ensuring efficient representation of the minimum
mass of constituents.

WHEREFORE, the Supplemental Motion for Reconsideration of respondent Commission on Elections dated 22 February
2010 is DENIED WITH FINALITY. Let no further pleadings be allowed.

SO ORDERED.

- Naval v Comelec, 729 SCRA 299 (2014)

A politician thinks of the next election –

a statesman of the next generation.

- James Freeman Clarke, American preacher and author

The Case

A provincial board member cannot be elected and serve for more than three consecutive terms. But then, the Court is now
called upon to resolve the following questions. First.What are the consequences to the provincial board member’s
eligibility to run for the same elective position if the legislative district, which brought him orher to office to serve the first
two consecutive terms, be reapportioned in such a way that 8 out of its 10 town constituencies are carved out and
renamed as another district? Second. Is the provincial board member’s election to the same position for the third and
fourth time, but now in representation ofthe renamed district, a violation of the three-term limit rule?

Before the Court is a Petition for Certiorariwith an Urgent Prayer for the Issuance of a Temporary Restraining Order and a
Writ of Preliminary Injunction  filed under Rule 64 of the Rules of Court to assail the following resolutions of the public
1

respondent Commission on Elections (COMELEC):

(a) Resolution  (first assailed resolution) issued by the Second Division on March 5, 2013, in SPA No. 13-166 (DC),
2

granting the petition filed by Nelson B. Julia (Julia), seeking to cancel the Certificate of Candidacy  (COC) as Member of
3

the Sangguniang Panlalawiganof Camarines Sur (Sanggunian) of Angel G. Naval (Naval), who is allegedly violating the
three-term limit imposed upon elective local officials as provided for in Article X, Section 8  of the 1987 Constitution, and
4

Section 43(b)  of the Local Government Code (LGC); and


5

(b) En BancResolution  (second assailed resolution) issued on June 5, 2013, denying Naval’s Motion for
6

Reconsideration  to the Resolution dated March 5, 2013.


7

Antecedents

From 2004 to 2007 and 2007 to 2010, Naval had been elected and had served as a member of the Sanggunian, Second
District, Province of Camarines Sur.

On October 12, 2009, the President approved Republic Act (R.A.) No. 9716,  which reapportioned the legislative districts
8

in Camarines Sur in the following manner:

[[reference - http://sc.judiciary.gov.ph/pdf/web/viewer.html?file=/jurisprudence/2014/july2014/207851.pdf ]]

Before the Enactment of After the Enactment of


District
R.A. No. 9716 R.A. No. 9716
Libmanan, Minalabac, Del Gallego, Ragay, Lupi,
1st
Pamplona, Pasacao, San Sipocot, Cabusao

Fernando, Del Gallego,

Ragay, Lupi, Sipocot,

Cabusao

Naga City, Pili, Ocampo, Libmanan, Minalabac,


2nd
Camaligan, Canaman, Pamplona, Pasacao, San

Magarao, Bombon, Fernando, Gainza, Milaor

Calabanga,  Gainza,
9

Milaor

Caramoan, Garchitorena, Naga City, Pili, Ocampo,


3rd
Goa, Lagonoy, Presentacion, Camaligan, Canaman,

Sangay, San Jose, Tigaon, Magarao, Bombon,

Tinambac, Siruma Calabanga

Iriga City, Baao, Balatan, Caramoan, Garchitorena,


4th
Bato, Buhi, Bula, Nabua Goa, Lagonoy,

Presentacion, Sangay, San

Jose, Tigaon, Tinambac,

Siruma

 
5th Iriga City, Baao, Balatan, Bato,
Buhi, Bula, Nabua

Notably, 8 out of 10 towns were taken from the old Second District to form the present Third District. The present Second
District is composed of the two remaining towns, Gainza and Milaor, merged with five towns from the old First District.

In the 2010 elections, Naval once again won as among the members of the Sanggunian, Third District. He served until
2013.

In the 2013 elections, Naval ran anewand was re-elected as Member of the Sanggunian, Third District.

Julia was likewise a SanggunianMember candidate from the Third District in the 2013 elections. On October 29, 2012, he
invoked Section 78  of the Omnibus Election Code (OEC) and filed beforethe COMELEC a Verified Petition to Deny Due
10

Course or to Cancel the Certificate of Candidacy  of Naval. Julia posited that Naval had fully served the entire Province of
11

Camarines Sur for three consecutive terms as a member of the Sanggunian, irrespective of the district he had been
elected from. The three-term limit rule’s application is more with reference to the same local elective post, and not
necessarily in connection with an identical territorial jurisdiction. Allowing Naval to run as a Sanggunianmember for the
fourth time is violative of the inflexible three-term limit rule enshrined in the Constitution and the LGC, which must be
strictly construed.
12

The Resolution of the COMELEC Second Division

In the first assailed resolution issued on March 5, 2013, the COMELEC Second Division cancelled Naval’s COC on
grounds stated below:

[W]hen a candidate for public office swears in his COC that he is eligible for the elective posts he seeks, while, in reality,
he knowingly lacks the necessary requirements for eligibility, he commits a false material misrepresentation cognizable
under Section 78 of the [OEC].

xxxx
The Supreme Court[,] in the case of Lonzanida v. [COMELEC][,] detailed the important components of[Article X, Section 8
of the Constitution]:

This Court held that the two conditions for the application of the disqualification must concur: 1) that the official concerned
has been elected for three consecutive terms in the same local government post and 2) that he has fully served three
consecutive terms.It stated:

To recapitulate, the term limit for elective local officials must be taken to refer to the right to be elected as well as the right
to serve in the same elective position.

Consequently, it is not enough that an individual has servedthree consecutive terms in an elective local office[;] he must
also have been electedto the same position for the same number of times before the disqualification can apply. x x x

x x x The first requisite does not only describe a candidate who has been elected for public office for three consecutive
terms. The candidate must have been elected in the samelocal government post. This connotes that the candidate must
have been inthe same elective position serving the same constituency who elected him to office for three consecutive
terms.

xxxx

The three-term limit rule was designed by the framers of the Constitution to prevent the monopoly of power centered only
on a chosen few. The said disqualification was primarily intended to forestall the accumulation of massive political power
by an elective local government official in a given locality in order to perpetuate his tenure in office. The framers also
considered the necessityof the enhancement of the freedom of choice of the electorate by broadening the selection of
would-be elective public officers. By rendering ineligible for public office those who have been elected and served for three
consecutive terms in the same public elective post, the prohibition seeks to infuse new blood in the political arena.

xxxx

x x x [T]he new Third District where [Naval] was elected and has served is composed of the same municipalities
comprising the previous Second District, absent the towns Gainza and [Milaor]. The territorial jurisdiction [Naval] seeks to
serve for the term 2013-2016 is the same as the territorial jurisdiction he previously served. The electorate who voted for
him in 2004, 2007 and 2010 isthe same electorate who shall vote for him come May 13, 2013 Elections. They are the
same group of voters who elected him into office for three consecutive terms.

The resolution of this Commission in the case of Bandillo, et al[.] v. Hernandez (SPA No. 10-078)  cannot be applied inthe
13

case at bar. Hernandez who then hailed from Libmanan belonged to the First District of Camarines Sur. With RepublicAct
9716, Libmanan, Minalabac, Pamplona, Pasacao and San Fernando, all originally belonging to the First District, were
merged with Gainza and Milaor to form the Second District. With the addition of the municipalities of Gainza and Milaor, it
cannot be said that the previous First District became the Second District only by name. The voters of Gainza and
Milaoradded to the electorate of the new Second District formed a different electorate, different from the one which voted
for Hernandez in the 2001, 2004 and 2007 elections. In the case at bar, the municipalities comprising the new Third
District are the same municipalities that consisted of the previous Second [District], absent Milaor and Gainza.

The Supreme Court, in Latasav. [COMELEC], ruled that the conversion of the municipality into a city did not convert the
office of the municipal mayor into a local government post different from the office of the city mayor[.]

x x x x  (Citations omitted)
14

The Resolution of the COMELEC En Banc

In the second assailed resolution issued on June 5, 2013, the COMELEC en bancdenied Naval’s Motion for
Reconsideration to the above. The COMELEC pointed out thatabsent the verification required under Section 3, Rule 19 of
the COMELEC Rules of Procedure, Naval’s motion was instantly dismissible. Nonetheless, the COMELEC proceeded to
discuss the demerits of Naval’s motion, viz:

The conditions for the application of the three-term limit rule are present in the instant case as the records clearly establish
that [Naval] is running for the 4th time for the same government post. To put things in a proper perspective, it is imperative
to review and discuss the salient points in the case of Latasa v. [COMELEC]. The case involves the question of whether or
not a municipal mayor, having been elected and had already served for three (3) consecutive terms, canrun as city mayor
in light of the conversion of the municipality to a city. In applying the three-term limit rule, the Court pointed out that the
conversion of the municipality into a city did not convert the office of the municipal mayor into a local government post
different from the office of the city mayor. The Court took into account the following circumstances: (1) That the territorial
jurisdiction of [the] city was the same as that of the municipality; (2) That the inhabitants were the same group of voters
who elected the municipal mayor for three (3) consecutive terms; and (3) That the inhabitants were the same group of
voters [over] whom he held power and authority as their chief executive for nine years.

Anchoring from the said case, it is therefore clear that the position to which [Naval] has filed his candidacy for the 13 May
2013 x x x Elections is the same position for which he had been elected and had served for the past nine (9) years.

xxxx
x x x The following circumstances establish that the subject posts are one and the same: First, the territorial jurisdictions of
the two (2) districts are the same except for the municipalities of Gainza and Milaor which were excluded by R.A. No.
9716; Second, the inhabitants of the 3rd District of Camarines Sur, where [Naval] is presently running as member of the
[Sanggunian], are the same voters who elected him for the past three (3) consecutive terms; and Lastly, the inhabitants of
the [3rd ] District are the same group of voters whom [Naval] had served as member of the [Sanggunian] representing the
2nd District.

x x x The enactment of R.A. No. 9716 did not convert [Naval’s] post [into one] different from [w]hat he [previously had]. As
correctly ruled by the Commission (Second Division), [Naval] ha[d] already been elected and ha[d] already served inthe
same government post for three consecutive terms, x x x[.]

x x x x.  (Citations omitted)
15

Unperturbed, Naval is now before the Court raising the issues of whether or not the COMELEC gravely erred and ruled
contrary to law and jurisprudence:

I. IN FINDING THAT NAVAL HAD ALREADY SERVED FOR THREE CONSECUTIVE TERMS IN THE SAME
GOVERNMENT POST; 16

II. IN IGNORING THE FACT THAT SANGGUNIAN MEMBERS ARE ELECTED BY LEGISLATIVE DISTRICTS;  and 17

III. WHEN IT RULED THAT THE PROHIBITION CONTEMPLATED BY SECTION 8, ARTICLE X OF THE 1987
CONSTITUTION AND SECTION 43(B) OF THE LGC APPLIES TO NAVAL. 18

The Arguments of the Contending Parties

In support of the instant petition, Naval alleges that the First, Second and Third Legislative Districts of Camarines Sur are
not merely renamed but are composed of new sets of municipalities. With the separation of Gainza and Milaor from the
other eight towns which used to comprise the Second District, the voters from the Third Legislative District are no longer
the same ones as those who had elected him to office in the 2004 and 2007 elections.

Naval further invokes Article 94  of Administrative Order No. 270 prescribing the Implementing Rules and Regulations of
19

the LGC to argue that Sanggunianmembers are elected by districts. Thus, the right to choose representatives in the
Sanggunianpertains to each of the districts. Naval was elected as Sanggunian member in 2004 and 2007 by the Second
District. In 2010 and 2013, it was the Third District, which brought him to office. Essentially then, Naval’s election in 2013
is merely his second term as Sanggunianmember for the Third District.

Naval likewise cites Borja, Jr. v. COMELEC  to point out that for the disqualification on the ground of the three-term limit to
20

apply, it is not enough that an individual has served three consecutive terms in an elective local office, but it is also
required that he or she had been elected to the same position for the same number of times. 21

Naval also assails as erroneous the COMELEC’s interpretations of the rulings in Latasa v. COMELEC  and Bandillo, et al.
22

v. Hernandez.  In Latasa, the Court applied the three-term prohibition only because notwithstanding the conversion of the
23

Municipality of Digos into a city, the mayor was to serve the same territorialjurisdiction and constituents. Naval asserts that
the same does not hold true in his case. Naval further avers that in Bandillo, which finds more application in the instant
petition, the COMELEC ruled that the three-term limit cannot be invoked in a situation where the legislative districts have
been altered. An extraction or an addition both yields a change inthe composition of the voters.

Naval further emphasizes that he garnered the majority of the votes from his constituents, whose will and mandate should
be upheld. Besides, Julia’s counsel already withdrew his appearance, indicating no less than his client’s lack of interest in
still pursuing Naval’s ouster from office.
24

In its Comment,  the Office of the Solicitor General (OSG) seeks the denial of the instant petition. The OSG contends that
25

Naval had been elected and had fully served the same local elective post for three consecutive terms. Naval thus
violatedSection 78 of the OEC when he filed his COC despite knowledge of his ineligibility. Naval’s reliance on Bandillo is
also misplaced since in the said case, two towns were instead added to form a new district. Apparently then, in Bandillo,
there was a new set of voters. The OSG also alleges that Naval is not entitled to the issuance of injunctive reliefs by this
Court. No clear and unmistakable right pertains to Naval and it is his eligibility to be elected as Sanggunianmember for the
Third District which is the issue at hand.

Ruling of the Court

The Court denies the petition.

As the issues are interrelated, they shall be discussed jointly.

The case before this Court is one of first impression. While the contending parties cite Latasa, Lonzanida v.
COMELEC,  Borja,Aldovino, Jr. v. COMELEC,  and Bandillo, which all involve the application of the three-term limit rule,
26 27

the factual and legal circumstances in those cases are different and the doctrinal values therein do not directly address the
issues now at hand.
In Latasa, the issue arose as a result of the conversion of a municipality into a city. The then municipal mayor attempted to
evade the application upon him of the three-term limit rule by arguing that the position of a city mayor was not the same as
the one he previously held. The Court was not convinced and, thus, declared that there was no interruption of the
incumbent mayor’s continuity of service.

In Lonzanida, a candidate ran for the mayoralty post and won in three consecutive elections. While serving his third term,
his opponent filed an election protest. Months before the expiration of the mayor’s third term, he was ousted from office.
He ran again for the same post in the immediately succeeding election. A petition was thereafter filed assailing his
eligibility to run as mayor on the ground of violation of the three-term limit rule. The Court ruled that the mayor could not
beconsidered as having served a full third term. An interruption for any length of time, if due to an involuntary cause, is
enough to break the elected official’s continuity of service.

In Borja, the mayor of Pateros died and was succeeded in office by the vice mayor. In the two immediately succeeding
elections, the latter vied for and won the mayoralty post. When he ran for the same position for the third time, his
disqualification was sought for alleged violation of the three-term limit rule. The Court ruled that whenhe assumed the
position of mayor by virtue of succession, his service should not be treated as one full term. For the disqualification to
apply, the candidate should have been thrice elected for and had served the same post consecutively. In Aldovino,
preventive suspension was imposed upon an elected municipal councilor. The Court ruled that the said suspension did not
interrupt the elective official’s term. Although hewas barred from exercising the functions of the position during the period
of suspension, his continued stay and entitlement tothe office remain unaffected.

In Bandillo, a case decided by the COMELEC, Gainza and Milaor were added to five of the ten towns, which used to
comprise Camarines Sur’s old First District, to form the new Second District. The COMELEC declined to apply the three-
term limit rule against the elected Provincial Board member on the ground that the addition of Gainza and Milaor
distinctively created a new district, with an altered territory and constituency.

In the case before this Court, the task is to determine the application of the three-term limit rule upon local elective officials
in renamed and/or reapportioned districts. In the process of doing so, it is inevitable to discuss the role of elections and the
nature of public office in a democratic and republican state like ours.

The Role of Elections in our

Democratic and Republican State,

and the Restraints Imposed Upon

Those Who Hold Public Office

The Court begins with general and undeniable principles.

The Philippines is a democratic and republican State. Sovereignty resides in the people and all government authority
emanates from them. 28

Then Associate Justice Reynato S. Puno explained the character of a republican state and a public office, viz: A republic
is a representative government, a government run by and for the people. It is not a pure democracy where the people
govern themselves directly. The essence of republicanism is representation and renovation, the selection by the citizenry
of a corps of public functionaries who derive their mandate from the people and act on their behalf, serving for a limited
period only, after which they are replaced or retained, at the option of their principal. Obviously, a republican government
is a responsiblegovernment whose officials hold and discharge their position as a public trust and shall, according to the
Constitution, ‘at all times be accountable to the people’ they are sworn to serve. The purpose of a republican government
it is almost needless to state, is the promotion of the common welfare according to the will of the people
themselves.  (Emphasis ours and italics in the original)
29

In Tolentino v. COMELEC,  Justice Puno likewise characterized the role of the electoral process in the following wise:
30

The electoral process is one of the linchpins of a democratic and republican framework because it isthrough the act of
voting that government by consent is secured. Through the ballot, people express their will on the defining issues of the
day and they are able to choose their leaders in accordance with the fundamental principle of representative democracy
that the people should elect whom they please to govern them. Voting has an important instrumental value in preserving
the viability of constitutional democracy. It has traditionally been taken as a prime indicator of democratic
participation.  (Citations omitted and italics ours)
31

The importance of elections cannottherefore be over emphasized. Thus,

True, election is the expression ofthe sovereign power of the people. In the exercise of suffrage, a free people expects to
achieve the continuity of government and the perpetuation of its benefits. However, inspite of its importance, the privileges
and rights arising from having been elected may be enlarged or restricted by law. x x x.  (Italics ours)
32

Hence, while it is settled that in elections, the first consideration of every democratic polity is to give effect to the
expressed will of the majority, there are limitations tobeing elected to a public office.  Our Constitution and statutes are
33

explicit anent the existence of term limits, the nature of public office, and the guarantee from the State that citizens shall
have equal access to public service.  Section 8, Article X of our Constitution, on term limits, is significantly reiterated by
34

Section 43(b) of the LGC. Moreover, the Court has time and again declared that a public office is a public trust and not a
vested property right.35

The Deliberations of the Members

of the Constitutional Commission

on the Three-Term Limit’s

Application to Local Elective

Officials

Following are entries in the Journal of the Constitutional Commission regarding the exchanges of the members on the
subject of the three-term limit rule imposed on local elective officials: VOTING ON THE TERMS OF LOCAL OFFICIALS

With respect to local officials, Mr. Nolledo, informed that the Committee on Local Governments had not decided on the
term of office for local officials and suggested that the Body decide on the matter.

xxxx

On Mr. Bacani’s inquiry regarding localofficials, Mr. Davide explained that local officials would includethe governor, vice-
governor and the members of the provincial board; the city mayor, city vice-mayor and members of the city board; and the
municipal mayor, municipal vice mayor and members of the municipal council. He stated that barangay officials would be
governed by speciallaw, to which Mr. Nolledo agreed.

xxxx

MOTION TO VOTE ON THE PROPOSALS RELATIVE TO ALTERNATIVE NO. 3

In reply to Mr. Guingona’s query onwhether the Committee had decided on the interpretation of "two reelections", Mr.
Davide suggested that the matter be submitted to a vote.

Thereupon, Mr. Romulo moved for a vote on whether Alternative No. 3 as proposed by Mr. Garcia, would allow a local
official three terms, after which he would not be allowed to seek any reelection; or whether, as interpreted by Mr. Davide, it
would mean that after two successive reelections or a consecutive periodof nine years, he could run for reelection after the
lapse of three years.

xxxx

RESTATEMENT OF THE PROPOSALS

Mr. Garcia reiterated that the local officials could be reelected twice, after which, they would be barred from ever
runningfor reelection.

On the other proposal, Mr. Davide, on behalf ofthe Committee, stated that local officials after two reelections would be
allowed to run for reelection after the lapse of three years.

xxxx

MANIFESTATION OF MR. ROMULO

Upon resumption of session, Mr. Romulomanifested that the Body would proceed to the consideration of two issues on the
term of Representatives and local officials, namely: 1) Alternative No. 1 (no further reelection after a total of three terms),
and 2) Alternative No. 2 (no immediate reelection after three successive terms).

SPONSORSHIP REMARKS OF MR. GARCIA ON ALTERNATIVE NO. 1

Mr. Garcia stated that he was advocating Alternative No. 1 on four grounds: 1) to prevent monopoly of political power
because the country’s history showed that prolonged stay in public office could lead to the creation of entrenched
preservesof political dynasties; 2) to broaden the choiceso that more people could be enlisted to the cause of public
service; 3) no one is indispensable in running the affairs of the countryand that reliance on personalities would be avoided;
and 4) the disqualification from running for reelection after three terms would create a reserve of statesmen both in the
local and national levels.

He added that the turnover in public office after nine years would ensure the introduction of new ideas and approaches.
He stressed that public office would no longer be a preserve of conservatism and tradition, and that public service would
no longer be limited to those directly holding public office, but would also include consultative bodiesorganized by the
people. INQUIRY OF MR. REGALADO
In reply to Mr. Regalado’s query whether the three terms need not be served consecutively, Mr. Garcia answered in the
affirmative.

SPONSORSHIP REMARKS OF MR. MONSOD ON ALTERNATIVE NO. 2

Mr. Monsod stated that while the new Constitution would recognize people power because of a new awareness, a new
kind of voter and a new kind of Filipino, at the same time, it pre-screens the candidates among whom the people would
choose by barring those who would have served for nine years from being reelected. He opined that this would actually
require an additional qualification for office to a certain number of people.

He stressed that, while the stand of the Commission is to create a reserve of statesmen, their future participation is
actually limited to some areas and only for a certain periodof time. He added thatit is not for the Commission to decide on
the future of our countrymen who may have more years ahead of them to serve the country.

xxxx

INQUIRY OF MR. OPLE

xxxx

Thereupon, speaking in support of Mr. Monsod’s manifestation, Mr. Ople expressed apprehension over the Body’s
exercise of some sort of omnipotent power in disqualifying those who will have served their tasks. He opined that the
Commission had already taken steps to prevent the accumulation of powers and prequisites that would permit officials to
stay on indefinitely and to transfer them to members of their families. He opined, however, that perpetual disqualification
would deprive the people of their freedom of choice.He stated that the Body had already succeeded in striking a balance
onpolicies which could ensure a redistribution of opportunities to the people both in terms of political and economic power.
He stated that Philippine politics had been unshackled from the two-party system, which he said was the most critical
support for the perpetuation of political dynasties. Considering that such achievement is already a victory, Mr. Ople stated
that the role of political parties should not be despised because the strength of democracy depends on how strong political
parties are, that a splintering thereof will mean a great loss to the vitality and resiliency of democracy.

Mr. Ople reiterated that he was against perpetual disqualification from office.

x x x x.

MR. GARCIA’S RESPONSE TOMR. OPLE’S STATEMENTS

Mr. Garcia stated that there are two principles involved in Alternative No. 1: 1) the recognition of the ambivalent nature of
political power, and 2) the recognition of alternative forms of public service. He stated that it is important to remember the
lessons learned from the recent past; that public service is service to the people and not an opportunity to accumulate
political power, and that a prolonged stay in public office brings about political dynasties or vested interests. Regarding
political parties, he stated that it will encourage the constant renewal of blood in party leadership, approach, style and
ideas. He opined that this is very healthy for a pluralist and multi-party democracy.

On the recognition of alternative forms of public service, Mr. Garcia stressed that public service could be limited to public
office since many good leaders who were in the streets and in jail fought against the dictatorship. He stressed that public
service would also mean belonging to consultative bodies or people’s councils which brought about new forms of service
and leadership.

REMARKS OF MR. ABUBAKAR

Mr. Abubakar stated that in any democracy the voice of the people is the voice of God.He stated that if the people want to
elect a representative to serve them continuously, the Commission should not arrogate unto itself the right to decide what
the people want. He stated that in the United States, a Senator had served for 30 years.

xxxx

REMARKS OF MS. AQUINO

Ms. Aquino stated that she differs from the views advanced by Mr. Garcia and Ms. Tan, although she stated that they
spoke of the same premises. She stated that she agrees with them that leaders need not be projected and developed
publicly in an election as leaders are better tempered and tested in the various forms of mass struggles and organized
work. She stated that if the people are to be encouraged to have their own sense of responsibility in national leadership,
what ultimately matters is the political determination of the citizenry to chart their own national destiny. She opined that the
Body should allow the people to exercise their own sense of proportion and imbibe the salutary effects of their own
strength to curtail power when it overreaches itself. She stressed that in the final analysis,the Commission cannot legislate
into the Constitution the essence of new politics as it is a chastening experience of learning and unlearning. Adverting to
Mr. Garcia’s statement that politics is an imperfect art, she stated that the Commission could correct politics with all its
imperfections and flaws by a constitutional provision. She opined that perpetual disqualification cannot provide the cure.
She maintained that perpetual disqualification is, at best, a palliative which could also be counter-productive, in the sense
that it could effectively foil the possibilities of realpublic service.
REMARKS OF MR. BACANI

Mr. Bacani stated that when the Body granted the illiterates the right to vote and that proposals were made to empower
the people to engage in the legislative process,the Body presupposed the political maturity of the people. He observed
that in this instance, political maturity is denied with the constitutional bar for reelection.He opined that the Body should
stick to the premise that the people are politically mature.

REJOINDER OF MR. GARCIA

By way of rejoinder to Mr. Bacani’s statements,Mr. Garcia stated that the proposal was basically premisedon the undue
advantage of the incumbent in accumulating power, money, party machine and patronage and not on lack of trust in the
people.

Mr. Garcia stated that politics isnot won by ideals alone but by solid organized work by organizations. He stated that with
three terms, an official would have served the people long enough.

xxxx

VOTING ON THE TWO ALTERNATIVES

Thereafter, the Body proceeded to vote by ballot on the two alternatives.

xxxx

RESULT OF THE VOTING

The result of the voting was as follows:

Alternative No. 1 (no further election after three successive terms) — 17 votes

Alternative No. 2 (no immediate reelection after three successive terms) — 26 votes

With 17 votes in favor of Alternative No. 1 and 26 in favor of Alternative No. 2, the Chair declared Alternative No. 2
approved by the Body.  (Emphasis and italics ours)
36

The Constitution mandates the

strict implementation of the

three-term limit rule.

The Court notes that in the process of drafting the Constitution, the framers thereof had not discussed with specifity the
subject of the three-term limit rule’s application on reapportioned districts.

From the above-cited deliberations, however, the divergent stances of the members of the Constitutional Commission on
the general application of the three-term limit rule show. On one side were those who espoused the stern view that
perpetual disqualification to hold public office after three consecutive terms would ensure that new blood would be infused
into our political system. More choices for the voters would give fuller meaning to our democratic institutions. On the other
side of the fence were those who believed that the imposition of termlimits would be tantamount to squandering the
experience of seasoned public servants and a curtailment of the power of the citizens to elect whoever they want to
remain in office.

In the end, 26 members of the Commission cast their votes in favor of the proposal that no immediate re-election after
three successive terms shall be allowed. On the other hand, 17 members stood pat on their view that there should be no
further reelection after three successive terms.

Clearly, the drafters of our Constitution are in agreement about the possible attendant evils if there would be no limit to re-
election. Notwithstanding their conflicting preferences on whether the term limit would disqualify the elected official
perpetually or temporarily, they decided that only three consecutive elections tothe same position would be allowed.
Thereafter, the public official can once again vie for the same post provided there be a gap of at least one term from his or
her last election. The rule answers the need to prevent the consolidation of political power in the hands of the few, while at
the same time giving to the people the freedom to call back to public service those who are worthy to be called statesmen.

The compromise agreed upon by the drafters of our Constitution was a result of exhaustive deliberations. The required
gap after three consecutive elections is significant. Thus, the rulecannot be taken with a grain of salt. Nothing less than its
strict application is called for.

Ratio legis est anima. 37


"A foolproof yardstick in constitutional construction is the intention underlying the provision under consideration.Thus, it
has been held that the Court in construing a Constitution should bear in mind the object sought to be accomplished by its
adoption, and the evils, if any, sought to be prevented or remedied. A doubtful provision will be examined in the light of the
history of the times, and the condition and circumstances under which the Constitution was framed. The object is to
ascertain the reason which induced the framers of the Constitution to enact the particular provision and the purpose
sought to be accomplished thereby, in order to construe the whole as to make the words consonant to that reason and
calculated to effect that purpose."  In Aldovino, the Court describes the three-term limit rule as inflexible.
38

In Aldovino, a local elective official pleaded exemption from the application of the three-term limit on the ground that there
was an interruption in his service after the penalty of suspension was imposed upon him. Although not in all four
withNaval’s case, there are principles enunciated therein which undeniably hold true, viz:

As worded, the constitutional provision fixes the term of a local elective office and limits an elective official’s stay in office
to no more than three consecutive terms. This is the first branch of the rule embodied in Section 8, Article X.

Significantly, this provision refers to a "term" as a period of time – three years– during which an official has title to office
and can serve. x x x[.]

xxxx

The "limitation" under this first branch of the provision is expressed in the negative—"no such official shall serve for more
than three consecutive terms." This formulation—no more than three consecutive terms—is a clear command suggesting
the existence of an inflexible rule. x x x.

xxxx

This examination of the wording of the constitutional provision and of the circumstances surrounding its formulation
impresses upon us the clear intent to make term limitation a high priority constitutional objective whose terms must be
strictly construed and which cannot be defeated by, nor sacrificed for, values of less than equal constitutional worth. x x x.

xxxx

x x x [T]he Court signalled how zealously it guards the three-term limit rule. Effectively, these cases teach usto strictly
interpret the term limitation rule in favor of limitation rather than its exception.

xxxx

[In] Latasa v. Commission on Electionsx x x[,] [t]he Court said:

This Court reiterates that the framers of the Constitution specifically included an exception to the people’s freedom to
choose those who will govern them in order to avoid the evil of a single person accumulating excessive power over a
particular territorial jurisdiction as a result of a prolonged stay in the same office. x x x.

xxxx

To put it differently although at the risk of repetition, Section 8, Article X—both by structure and substance—fixes an
elective official’s term of office and limits his stay in office to three consecutive terms as an inflexible rule that is stressed,
no less, by citing voluntary renunciation as an example of a circumvention. x x x.  (Citations omitted, italics and emphasis
39

in the original and underscoring ours)

Reapportionment and its Basis

Reapportionment is "the realignment orchange in legislative districts brought about by changes in population and
mandated by the constitutional requirement of equality of representation."  The aim of legislative apportionment is to
40

equalize population and voting power among districts.  The basis for districting shall be the number of the inhabitants of a
41

city or a province and not the number of registered voters therein. 42

R.A. No. 9716 and the Reappor-

tioned Districts of Camarines Sur

Sections 1 to 3 of R.A. No. 9716 provide:

Section 1. The composition of the current First (1st) and Second (2nd) Legislative Districts in the Province of Camarines
Sur is hereby reapportioned in order to create an additional legislative districtto commence in the next national elections
after the effectivity of this Act.

Section 2. In furtherance of the reapportionment mandated by this Act, the municipalities of Libmanan, Minalabac,
Pamplona, Pasacao and San Fernando of the current First (1st) Legislative District are hereby consolidated with the
municipalities of Gainza and Milaor of the current Second (2nd) Legislative District, to comprise the new legislative district
authorized under this Act.

Section 3. The result of the reapportionment described in this Act are summarized as follows:

a) First District – The remaining municipalities in the current First (1st) Legislative District shall continue to be designated
as the First (1st) Legislative District, composed of the following municipalities: Del Gallego, Ragay, Lupi, Sipicot and
Cabusao;

b) Second District – This new legislative districtshall be composed of the municipalities enumerated in Section 2 hereof;

c) Third District – The current Second (2nd) Legislative District shall be renamedas the Third (3rd) Legislative District,
composed of the following: Naga City and the municipalities of Pili, Ocampo, Camaligan, Canaman, Magarao, Bombon
and Calabanga;

d) Fourth District – The current Third (3rd) Legislative District, without any change in its composition, shall be renamedas
the Fourth (4th) Legislative District, composed of the following municipalities: Caramoan, Garchitorena, Goa, Lagonoy,
Presentacion, Sangay, San Jose, Tigaon, Tinambac and Siruma; and

e) Fifth District – The current Fourth (4th) Legislative District, without any change inits composition, shall be renamedas
the Fifth (5th) Legislative District, composed of the following: Iriga City and the municipalities of Baao, Balatan, Bato, Buhi,
Bula and Nabua. (Italics and emphasis ours)

As a result of the reapportionment made by R.A. No. 9716, the old Second District of Camarines Sur, minus only the two
towns of Gainza and Milaor, is renamed as the Third District and now configured as follows: 43

[[reference - http://sc.judiciary.gov.ph/pdf/web/viewer.html?file=/jurisprudence/2014/july2014/207851.pdf ]]

Before the Enactment of After the Enactment of

RA 9716 RA 9716

2 3rd District

nd Population: 439,043

District Naga

Population: 474,899 Pili

Gainza Ocampo

Milaor Canaman

Naga Camaligan

Pili Magarao

Ocampo Bombon

Canaman Calabanga

Camaligan

Magarao

Bombon

Calabanga

R.A. No. 9716 created a new Second

District, but it merely renamed the

other four.

The Court notes that after the reapportionment of the districts in Camarines Sur, the current Third District, which brought
Naval to office in 2010 and 2013, has a population of 35,856 less than that of the old Second District, which elected him in
2004 and 2007. However, the wordings of R.A. No. 9716 indicate the intent of the lawmakers to create a single new
Second District from the merger of the towns from the old First District with Gainza and Milaor. As to the current Third
District, Section 3(c) of R.A. No. 9716 used the word "rename." Although the qualifier "without a change in its composition"
was not found in Section 3(c), unlike in Sections 3(d) and (e), still, what is pervasive isthe clear intent to create a sole new
district in that of the Second, while merely renaming the rest.

The following statutory construction rules surface:

First, the general rule in construing words and phrases used in a statute is that, in the absence of legislative intent to the
contrary, they should be given their plain, ordinary and common usage meaning; the words should be read and
considered intheir natural, ordinary, commonly accepted usage, and without resorting to forced or subtle construction.
Words are presumed to have been employed by the lawmaker in their ordinary and common use and acceptation.
Second, a word of general significance ina statute is to be taken in its ordinary and comprehensive sense, unless it is
shown that the word is intended to be given a different or restricted meaning; what is generally spoken shall be generally
understood and general words shall be understood in a general sense.  (Citations omitted)
44

The Court looks to the language of the document itself in our search for its meaning. 45

In Naval’s case, the words of R.A.No. 9716 plainly state that the new Second Districtis to be created, but the Third
Districtis to be renamed. Verba legis non est recedendum. The terms used in a legal provision to be construed compels
acceptanceand negates the power of the courts to alter it, based on the postulate that the framers mean what they say. 46

The verb createmeans to "make or produce something new."  On the other hand, the verb renamemeans to "give a new
47

name to someone or something."  A complete reading of R.A. No. 9716 yields no logical conclusion other than that the
48

lawmakers intended the old Second District to be merely renamed as the current Third District.

It likewise bears noting that the actual difference in the population of the old Second District from that of the current Third
District amounts to less than 10% of the population of the latter. This numericalfact renders the new Third District as
essentially, although not literally, the same as the old Second District. Hence, while Naval is correct in his argument that
Sanggunianmembers are elected by district, it does not alter the fact that the district which elected him for the third and
fourth time is the same one which brought him to office in 2004 and 2007.

The application upon Naval of the

three-term limit rule does not

undermine the constitutional

requirement to achieve equality of

representation among districts.

The rationale behind reapportionment is the constitutional requirement to achieve equality ofrepresentation among the
districts.  It is with this mindset that the Court should consider Naval’s argument anent having a new set of constituents
49

electing him into office in 2010 and 2013.

Naval’s ineligibility to run, by reason of violation of the three-term limit rule, does not undermine the right toequal
representation of any of the districts in Camarines Sur. With or without him, the renamed Third District, which he labels as
a new set of constituents, would still be represented, albeit by another eligible person.

The presumed competence of the

COMELEC to resolve matters

falling within its jurisdiction is

upheld.

"Time and again, the Court has held that a petition for certiorariagainst actions of the COMELEC is confined only to
instances of grave abuse of discretion amounting to patent and substantial denial of due process, because the COMELEC
is presumed to be most competent in matters falling within its domain." 50

"In a special civil action for certiorari, the burden rests on the petitioner to prove not merelyreversible error, but grave
abuse of discretion amounting to lack or excess of jurisdiction on the part of the public respondent issuing the impugned
order, decision or resolution."  "Grave abuse of discretion arises when a court or tribunal violates the Constitution, the law
51

or existing jurisprudence." 52

In the case at bar, the Court finds the COMELEC’s disquisitions to be amply supported by the Constitution,law and
jurisprudence.
Conclusion

In sum, the Court finds no compelling reason to grant the reliefs prayed for by Naval. For the Court to declare otherwise
would be to create a dangerous precedent unintended by the drafters of our Constitution and of R.A. No. 9716.
Considering that the one-term gap or rest after three consecutive elections is a result of a compromise among the
members of the Constitutional Commission, no cavalier exemptions or exceptions to its application is to be allowed.
Aldovinoaffirms this interpretation. Further, sustaining Naval’s arguments would practically allow him to hold the same
office for 15 years. These are the circumstances the Constitution explicitly intends to avert.

Certainly, the Court accords primacy to upholding the will of the voting public, the real sovereign, soto speak. However, let
all the candidates for public office be reminded that as citizens, we have a commitment to be bound by our Constitution
and laws. Side by side our privileges as citizens are restrictions too.

Einer Elhauge, a faculty member from Harvard Law School, wrote an article entitled "What Term Limits Do That Ordinary
Voting Cannot."  In the article, Greek mythology was tapped to make a tempting analogy. The gist of the story follows.
53

In Odyssey Book XII, the goddess Circe warned Odysseus of the Sirens who seduce all men approaching them with their
voices. Those who fell into the Sirens’ trap never returnedhome to their wives and children. A clever strategy was thus
hatched to secure safe passage for Odysseus and his men. The men were to plug their ears with wax to muffle the songs
of the Sirens. Odysseus, on the other hand, was to be tied to the mast of the ship so he could still listen to the songs,
which may contain clues on how they can get home. When the wind died down,Odysseus heard beautiful voices calling
out to them. The voices were incomparable to anything he had ever heard before. Even whenOdysseus knew that the
irresistible voices were coming from the Sirens, he struggled with all his strength to free himself from the ropes, but was
unable to do so. The voices became fainter as the men continued to row. When the voices can no longer be heard,
Odysseus realized how he had nearly been beguiled. They had made it through safely and Odysseus was untied. It was
their clever plan which kept them all alive.
54

The same lesson holds true in the case before this Court. The drafters of the Constitution recognized the propensity of
public officers to perpetuate themselves in power, hence, the adoption of term limits and a guarantee of every citizen's
equal access to public service. These are the restrictions statesmen should observe for they are intended to help ensure
the continued vitality of our republican institutions.

WHEREFORE, IN VIEW OF THE FOREGOING, the petition is DENIED. The Resolutions dated March 5, 2013 and June
5, 2013 of the Commission on Elections in SPA No. 13-166 (DC) are AFFIRMED.

SO ORDERED.

- Bagabuyo v Comelec, 573 SCRA 290 (2008)

Before us is the petition for certiorari, prohibition, and mandamus,1 with a prayer for the issuance of a temporary
restraining order and a writ of preliminary injunction, filed by Rogelio Bagabuyo (petitioner) to prevent the Commission on
Elections (COMELEC) from implementing Resolution No. 7837 on the ground that Republic Act No. 9371 2 - the law that
Resolution No. 7837 implements - is unconstitutional.

BACKGROUND FACTS

On October 10, 2006, Cagayan de Oro's then Congressman Constantino G. Jaraula filed and sponsored House Bill No.
5859: "An Act Providing for the Apportionment of the Lone Legislative District of the City of Cagayan De Oro." 3 This law
eventually became Republic Act (R.A.) No. 9371. 4 It increased Cagayan de Oro's legislative district from one to two. For
the election of May 2007, Cagayan de Oro's voters would be classified as belonging to either the first or the second
district, depending on their place of residence. The constituents of each district would elect their own representative to
Congress as well as eight members of the Sangguniang Panglungsod.

Section 1 of R.A. No. 9371 apportioned the City's barangays as follows:

Legislative Districts - The lone legislative district of the City of Cagayan De Oro is hereby apportioned to
commence in the next national elections after the effectivity of this Act. Henceforth, barangays Bonbon, Bayabas,
Kauswagan, Carmen, Patag, Bulua, Iponan, Baikingon, San Simon, Pagatpat, Canitoan, Balulang, Lumbia,
Pagalungan, Tagpangi, Taglimao, Tuburan, Pigsag-an, Tumpagon, Bayanga, Mambuaya, Dansulihon,
Tignapoloan and Bisigan shall comprise the first district while barangays Macabalan, Puntod, Consolacion,
Camaman-an, Nazareth, Macasandig, Indahag, Lapasan, Gusa, Cugman, FS Catanico, Tablon, Agusan, Puerto,
Bugo, and Balubal and all urban barangays from Barangay 1 to Barangay 40 shall comprise the second district. 5

On March 13, 2007, the COMELEC en Banc promulgated Resolution No. 78376 implementing R.A. No. 9371.

Petitioner Rogelio Bagabuyo filed the present petition against the COMELEC on March 27, 2007. 7 On 10 April 2008, the
petitioner amended the petition to include the following as respondents: Executive Secretary Eduardo Ermita; the
Secretary of the Department of Budget and Management; the Chairman of the Commission on Audit; the Mayor and the
members of the Sangguniang Panglungsod of Cagayan de Oro City; and its Board of Canvassers. 8
In asking for the nullification of R.A. No. 9371 and Resolution No. 7837 on constitutional grounds, the petitioner argued
that the COMELEC cannot implement R.A. No. 9371 without providing for the rules, regulations and guidelines for the
conduct of a plebiscite which is indispensable for the division or conversion of a local government unit. He prayed for the
issuance of an order directing the respondents to cease and desist from implementing R.A. No. 9371 and COMELEC
Resolution No. 7837, and to revert instead to COMELEC Resolution No. 7801 which provided for a single legislative
district for Cagayan de Oro.

Since the Court did not grant the petitioner's prayer for a temporary restraining order or writ of preliminary injunction, the
May 14 National and Local Elections proceeded according to R.A. No. 9371 and Resolution No. 7837.

The respondent's Comment on the petition, filed through the Office of the Solicitor General, argued that: 1) the petitioner
did not respect the hierarchy of courts, as the Regional Trial Court (RTC) is vested with concurrent jurisdiction over cases
assailing the constitutionality of a statute; 2) R.A. No. 9371 merely increased the representation of Cagayan de Oro City in
the House of Representatives and Sangguniang Panglungsod pursuant to Section 5, Article VI of the 1987 Constitution; 3)
the criteria established under Section 10, Article X of the 1987 Constitution only apply when there is a creation, division,
merger, abolition or substantial alteration of boundaries of a province, city, municipality, or barangay; in this case, no such
creation, division, merger, abolition or alteration of boundaries of a local government unit took place; and 4) R.A. No. 9371
did not bring about any change in Cagayan de Oro's territory, population and income classification; hence, no plebiscite is
required.

The petitioner argued in his reply that: 1) pursuant to the Court's ruling in Del Mar v. PAGCOR,9 the Court may take
cognizance of this petition if compelling reasons, or the nature and importance of the issues raised, warrant the immediate
exercise of its jurisdiction; 2) Cagayan de Oro City's reapportionment under R.A. No. 9371 falls within the meaning of
creation, division, merger, abolition or substantial alteration of boundaries of cities under Section 10, Article X of the
Constitution; 3) the creation, division, merger, abolition or substantial alteration of boundaries of local government units
involve a common denominator - the material change in the political and economic rights of the local government units
directly affected, as well as of the people therein; 4) a voter's sovereign power to decide on who should be elected as the
entire city's Congressman was arbitrarily reduced by at least one half because the questioned law and resolution only
allowed him to vote and be voted for in the district designated by the COMELEC; 5) a voter was also arbitrarily denied his
right to elect the Congressman and the members of the city council for the other legislative district, and 6) government
funds were illegally disbursed without prior approval by the sovereign electorate of Cagayan De Oro City. 10

THE ISSUES

The core issues, based on the petition and the parties' memoranda, can be limited to the following contentious points:

1) Did the petitioner violate the hierarchy of courts rule; if so, should the instant petition be dismissed on this
ground?

2) Does R.A. No. 9371 merely provide for the legislative reapportionment of Cagayan de Oro City, or does it
involve the division and conversion of a local government unit?

3) Does R.A. No. 9371 violate the equality of representation doctrine?

OUR RULING

Except for the issue of the hierarchy of courts rule, we find the petition totally without merit.

The hierarchy of courts principle.

The Supreme Court has original jurisdiction over petitions for certiorari, prohibition, mandamus, quo warranto, and habeas
corpus.11 It was pursuant to this original jurisdiction that the petitioner filed the present petition.

While this jurisdiction is shared with the Court of Appeals 12 and the RTCs,13 a direct invocation of the Supreme Court's
jurisdiction is allowed only when there are special and important reasons therefor, clearly and especially set out in the
petition. Reasons of practicality, dictated by an increasingly overcrowded docket and the need to prioritize in favor of
matters within our exclusive jurisdiction, justify the existence of this rule otherwise known as the "principle of hierarchy of
courts." More generally stated, the principle requires that recourse must first be made to the lower-ranked court exercising
concurrent jurisdiction with a higher court.14

Among the cases we have considered sufficiently special and important to be exceptions to the rule, are petitions
for certiorari, prohibition, mandamus and quo warranto against our nation's lawmakers when the validity of their
enactments is assailed.15 The present petition is of this nature; its subject matter and the nature of the issues raised -
among them, whether legislative reapportionment involves a division of Cagayan de Oro City as a local government unit -
are reasons enough for considering it an exception to the principle of hierarchy of courts. Additionally, the petition assails
as well a resolution of the COMELEC en banc issued to implement the legislative apportionment that R.A. No. 9371
decrees. As an action against a COMELEC en banc resolution, the case falls under Rule 64 of the Rules of Court that in
turn requires a review by this Court via a Rule 65 petition for certiorari.16 For these reasons, we do not see the principle of
hierarchy of courts to be a stumbling block in our consideration of the present case.

The Plebiscite Requirement.


The petitioner insists that R.A. No. 9371 converts and divides the City of Cagayan de Oro as a local government unit, and
does not merely provide for the City's legislative apportionment. This argument essentially proceeds from a
misunderstanding of the constitutional concepts of apportionment of legislative districts and division of local government
units.

Legislative apportionment is defined by Black's Law Dictionary as the determination of the number of representatives
which a State, county or other subdivision may send to a legislative body. 17It is the allocation of seats in a legislative body
in proportion to the population; the drawing of voting district lines so as to equalize population and voting power among
the districts.18 Reapportionment, on the other hand, is the realignment or change in legislative districts brought about by
changes in population and mandated by the constitutional requirement of equality of representation.19

Article VI (entitled Legislative Department) of the 1987 Constitution lays down the rules on legislative apportionment under
its Section 5 which provides:

Sec. 5(1). (1) The House of Representatives shall be composed of not more than two hundred fifty members
unless otherwise fixed by law, who shall be elected from legislative districts apportioned among the provinces,
cities, and the Metropolitan Manila area in accordance with the number of their respective inhabitants, and on the
basis of a uniform and progressive ratio, and those who, as provided by law, shall be elected through a party-list
system of registered national, regional and sectoral parties or organizations.

xxx

(3) Each legislative district shall comprise, as far as practicable, continuous, compact, and adjacent territory. Each
city with a population of at least two hundred fifty thousand, or each province, shall have at least one
representative.

(4) Within three years following the return of every census, the Congress shall make a reapportionment of
legislative districts based on the standards provided in this section.

Separately from the legislative districts that legal apportionment or reapportionment speaks of, are the local government
units (historically and generically referred to as "municipal corporations") that the Constitution itself classified into
provinces, cities, municipalities and barangays.20 In its strict and proper sense, a municipality has been defined as "a body
politic and corporate constituted by the incorporation of the inhabitants of a city or town for the purpose of local
government thereof."21 The creation, division, merger, abolition or alteration of boundary of local government units, i.e., of
provinces, cities, municipalities, and barangays, are covered by the Article on Local Government (Article X). Section 10 of
this Article provides:

No province, city, municipality, or barangay may be created, divided, merged, abolished, or its boundary
substantially altered, except in accordance with the criteria established in the local government code and subject
to approval by a majority of the votes cast in a plebiscite in the political unit directly affected.

Under both Article VI, Section 5, and Article X, Section 10 of the Constitution, the authority to act has been vested in the
Legislature. The Legislature undertakes the apportionment and reapportionment of legislative districts, 22 and likewise acts
on local government units by setting the standards for their creation, division, merger, abolition and alteration of
boundaries and by actually creating, dividing, merging, abolishing local government units and altering their boundaries
through legislation. Other than this, not much commonality exists between the two provisions since they are inherently
different although they interface and relate with one another.

The concern that leaps from the text of Article VI, Section 5 is political representation and the means to make a legislative
district sufficiently represented so that the people can be effectively heard. As above stated, the aim of legislative
apportionment is "to equalize population and voting power among districts." 23 Hence, emphasis is given to the number of
people represented; the uniform and progressive ratio to be observed among the representative districts; and accessibility
and commonality of interests in terms of each district being, as far as practicable, continuous, compact and adjacent
territory. In terms of the people represented, every city with at least 250,000 people and every province (irrespective of
population) is entitled to one representative. In this sense, legislative districts, on the one hand, and provinces and cities,
on the other, relate and interface with each other. To ensure continued adherence to the required standards of
apportionment, Section 5(4) specifically mandates reapportionment as soon as the given standards are met.

In contrast with the equal representation objective of Article VI, Section 5, Article X, Section 10 expressly speaks of how
local government units may be "created, divided, merged, abolished, or its boundary substantially altered." Its concern is
the commencement, the termination, and the modification of local government units' corporate existence and territorial
coverage; and it speaks of two specific standards that must be observed in implementing this concern, namely, the criteria
established in the local government code and the approval by a majority of the votes cast in a plebiscite in the political
units directly affected. Under the Local Government Code (R.A. No. 7160) passed in 1991, the criteria of income,
population and land area are specified as verifiable indicators of viability and capacity to provide services. 24 The division or
merger of existing units must comply with the same requirements (since a new local government unit will come into being),
provided that a division shall not reduce the income, population, or land area of the unit affected to less than the minimum
requirement prescribed in the Code.25

A pronounced distinction between Article VI, Section 5 and, Article X, Section 10 is on the requirement of a plebiscite. The
Constitution and the Local Government Code expressly require a plebiscite to carry out any creation, division, merger,
abolition or alteration of boundary of a local government unit. 26 In contrast, no plebiscite requirement exists under the
apportionment or reapportionment provision. In Tobias v. Abalos,27 a case that arose from the division of the congressional
district formerly covering San Juan and Mandaluyong into separate districts, we confirmed this distinction and the fact that
no plebiscite is needed in a legislative reapportionment. The plebiscite issue came up because one was ordered and held
for Mandaluyong in the course of its conversion into a highly urbanized city, while none was held for San Juan. In
explaining why this happened, the Court ruled that no plebiscite was necessary for San Juan because the objective of the
plebiscite was the conversion of Mandaluyong into a highly urbanized city as required by Article X, Section 10 the Local
Government Code; the creation of a new legislative district only followed as a consequence. In other words, the
apportionment alone and by itself did not call for a plebiscite, so that none was needed for San Juan where only a
reapportionment took place.

The need for a plebiscite under Article X, Section 10 and the lack of requirement for one under Article VI, Section 5 can
best be appreciated by a consideration of the historical roots of these two provisions, the nature of the concepts they
embody as heretofore discussed, and their areas of application.

A Bit of History.

In Macias v. COMELEC,28 we first jurisprudentially acknowledged the American roots of our apportionment provision,
noting its roots from the Fourteenth Amendment29 of the U.S. Constitution and from the constitutions of some American
states. The Philippine Organic Act of 1902 created the Philippine Assembly, 30 the body that acted as the lower house of
the bicameral legislature under the Americans, with the Philippine Commission acting as the upper house. While the
members of the Philippine Commission were appointed by the U.S. President with the conformity of the U.S. Senate, the
members of the Philippine Assembly were elected by representative districts previously delineated under the Philippine
Organic Act of 1902 pursuant to the mandate to apportion the seats of the Philippine Assembly among the provinces as
nearly as practicable according to population. Thus, legislative apportionment first started in our country.

The Jones Law or the Philippine Autonomy Act of 1916 maintained the apportionment provision, dividing the country into
12 senate districts and 90 representative districts electing one delegate each to the House of Representatives. Section 16
of the Act specifically vested the Philippine Legislature with the authority to redistrict the Philippine Islands.

Under the 1935 Constitution, Article VI, Section 5 retained the concept of legislative apportionment together with "district"
as the basic unit of apportionment; the concern was "equality of representation . . . as an essential feature of republican
institutions" as expressed in the leading case of Macias v. COMELEC.31 The case ruled that inequality of representation is
a justiciable, not a political issue, which ruling was reiterated in Montejo v. COMELEC.32 Notably, no issue regarding the
holding of a plebiscite ever came up in these cases and the others that followed, as no plebiscite was required.

Article VIII, Section 2 of the 1973 Constitution retained the concept of equal representation "in accordance with the
number of their respective inhabitants and on the basis of a uniform and progressive ratio" with each district being, as far
as practicable, contiguous, compact and adjacent territory. This formulation was essentially carried over to the 1987
Constitution, distinguished only from the previous one by the presence of party-list representatives. In neither Constitution
was a plebiscite required.

The need for a plebiscite in the creation, division, merger, or abolition of local government units was not constitutionally
enshrined until the 1973 Constitution. However, as early as 1959, R.A. No. 2264 33 required, in the creation of barrios by
Provincial Boards, that the creation and definition of boundaries be "upon petition of a majority of the voters in the areas
affected." In 1961, the Charter of the City of Caloocan (R.A. No. 3278) carried this further by requiring that the "Act shall
take effect after a majority of voters of the Municipality of Caloocan vote in favor of the conversion of their municipality into
a city in a plebiscite." This was followed up to 1972 by other legislative enactments requiring a plebiscite as a condition for
the creation and conversion of local government units as well as the transfer of sitios from one legislative unit to
another.34 In 1973, the plebiscite requirement was accorded constitutional status.

Under these separate historical tracks, it can be seen that the holding of a plebiscite was never a requirement in legislative
apportionment or reapportionment. After it became constitutionally entrenched, a plebiscite was also always identified with
the creation, division, merger, abolition and alteration of boundaries of local government units, never with the concept of
legislative apportionment.

Nature and Areas of Application.

The legislative district that Article VI, Section 5 speaks of may, in a sense, be called a political unit because it is the
basis for the election of a member of the House of Representatives and members of the local legislative body. It is not,
however, a political subdivision through which functions of government are carried out. It can more appropriately be
described as a representative unit that may or may not encompass the whole of a city or a province, but unlike the latter, it
is not a corporate unit. Not being a corporate unit, a district does not act for and in behalf of the people comprising the
district; it merely delineates the areas occupied by the people who will choose a representative in their national affairs.
Unlike a province, which has a governor; a city or a municipality, which has a mayor; and a barangay, which has a punong
barangay, a district does not have its own chief executive. The role of the congressman that it elects is to ensure that the
voice of the people of the district is heard in Congress, not to oversee the affairs of the legislative district. Not being a
corporate unit also signifies that it has no legal personality that must be created or dissolved and has no capacity to act.
Hence, there is no need for any plebiscite in the creation, dissolution or any other similar action on a legislative district.

The local government units, on the other hand, are political and corporate units. They are the territorial and political
subdivisions of the state.35 They possess legal personality on the authority of the Constitution and by action of the
Legislature. The Constitution defines them as entities that Congress can, by law, create, divide, abolish, merge; or whose
boundaries can be altered based on standards again established by both the Constitution and the Legislature. 36 A local
government unit's corporate existence begins upon the election and qualification of its chief executive and a majority of the
members of its Sanggunian.37

As a political subdivision, a local government unit is an "instrumentality of the state in carrying out the functions of
government."38 As a corporate entity with a distinct and separate juridical personality from the State, it exercises special
functions for the sole benefit of its constituents. It acts as "an agency of the community in the administration of local
affairs"39 and the mediums through which the people act in their corporate capacity on local concerns. 40 In light of these
roles, the Constitution saw it fit to expressly secure the consent of the people affected by the creation, division, merger,
abolition or alteration of boundaries of local government units through a plebiscite.

These considerations clearly show the distinctions between a legislative apportionment or reapportionment and the
division of a local government unit. Historically and by its intrinsic nature, a legislative apportionment does not mean, and
does not even imply, a division of a local government unit where the apportionment takes place. Thus, the plebiscite
requirement that applies to the division of a province, city, municipality or barangay under the Local Government Code
should not apply to and be a requisite for the validity of a legislative apportionment or reapportionment.

R.A. No. 9371 and COMELEC Res. No. 7837

R.A. No. 9371 is, on its face, purely and simply a reapportionment legislation passed in accordance with the authority
granted to Congress under Article VI, Section 5(4) of the Constitution. Its core provision - Section 1 - provides:

SECTION 1. Legislative Districts. - The lone legislative district of the City of Cagayan de Oro is hereby apportioned
to commence in the next national elections after the effectivity of this Act. Henceforth, barangays Bonbon,
Bayabas, Kauswagan, Carmen, Patag, Bulua, Iponan, Baikingon, San Simon, Pagatpat, Canitoan, Balulang,
Lumbia, Pagalungan, Tagpangi, Taglimao, Tuburan, Pigsag-an, Tumpagon, Bayanga, Mambuaya, Dansulihon,
Tignapoloan and Bisigan shall comprise the first district while barangays Macabalan, Puntod, Consolacion,
Camaman-an, Nazareth, Macansandig, Indahag, Lapasan, Gusa, Cugman, FS Catanico, Tablon, Agusan, Puerto,
Bugo and Balubal and all urban barangays from Barangay 1 to Barangay 40 shall comprise the second district.

Under these wordings, no division of Cagayan de Oro City as a political and corporate entity takes place or is mandated.
Cagayan de Oro City politically remains a single unit and its administration is not divided along territorial lines. Its territory
remains completely whole and intact; there is only the addition of another legislative district and the delineation of the city
into two districts for purposes of representation in the House of Representatives. Thus, Article X, Section 10 of the
Constitution does not come into play and no plebiscite is necessary to validly apportion Cagayan de Oro City into two
districts.

Admittedly, the legislative reapportionment carries effects beyond the creation of another congressional district in the city
by providing, as reflected in COMELEC Resolution No. 7837, for additional Sangguniang Panglunsod seats to be voted for
along the lines of the congressional apportionment made. The effect on the Sangguniang Panglunsod, however, is not
directly traceable to R.A. No. 9371 but to another law - R.A. No. 6636 41 - whose Section 3 provides:

SECTION 3. Other Cities. - The provision of any law to the contrary notwithstanding the City of Cebu, City of
Davao, and any other city with more than one representative district shall have eight (8) councilors for each
district who shall be residents thereof to be elected by the qualified voters therein, provided that the cities of
Cagayan de Oro, Zamboanga, Bacolod, Iloilo and other cities comprising a representative district shall have twelve
(12) councilors each and all other cities shall have ten (10) councilors each to be elected at large by the qualified
voters of the said cities: Provided, That in no case shall the present number of councilors according to their
charters be reduced.

However, neither does this law have the effect of dividing the City of Cagayan de Oro into two political and corporate units
and territories. Rather than divide the city either territorially or as a corporate entity, the effect is merely to enhance voter
representation by giving each city voter more and greater say, both in Congress and in the Sangguniang Panglunsod.

To illustrate this effect, before the reapportionment, Cagayan de Oro had only one congressman and 12 city council
members citywide for its population of approximately 500,000. 42 By having two legislative districts, each of them with one
congressman, Cagayan de Oro now effectively has two congressmen, each one representing 250,000 of the city's
population. In terms of services for city residents, this easily means better access to their congressman since each one
now services only 250,000 constituents as against the 500,000 he used to represent. The same goes true for
the Sangguniang Panglungsod with its ranks increased from 12 to 16 since each legislative district now has 8 councilors.
In representation terms, the fewer constituents represented translate to a greater voice for each individual city resident in
Congress and in the Sanggunian; each congressman and each councilor represents both a smaller area and fewer
constituents whose fewer numbers are now concentrated in each representative. The City, for its part, now has twice the
number of congressmen speaking for it and voting in the halls of Congress. Since the total number of congressmen in the
country has not increased to the point of doubling its numbers, the presence of two congressman (instead of one) from the
same city cannot but be a quantitative and proportional improvement in the representation of Cagayan de Oro City in
Congress.

Equality of representation.
The petitioner argues that the distribution of the legislative districts is unequal. District 1 has only 93,719 registered voters
while District 2 has 127,071. District 1 is composed mostly of rural barangays while District 2 is composed mostly of
urban barangays.43 Thus, R.A. No. 9371 violates the principle of equality of representation.

A clarification must be made. The law clearly provides that the basis for districting shall be the number of the inhabitants of
a city or a province, not the number of registered voters therein. We settled this very same question in Herrera v.
COMELEC44 when we interpreted a provision in R.A. No. 7166 and COMELEC Resolution No. 2313 that applied to the
Province of Guimaras. We categorically ruled that the basis for districting is the number of inhabitants of the Province of
Guimaras by municipality based on the official 1995 Census of Population as certified to by Tomas P. Africa, Administrator
of the National Statistics Office.

The petitioner, unfortunately, did not provide information about the actual population of Cagayan de Oro City. However, we
take judicial notice of the August 2007 census of the National Statistics Office which shows that barangays comprising
Cagayan de Oro's first district have a total population of 254,644, while the second district has 299,322 residents.
Undeniably, these figures show a disparity in the population sizes of the districts. 45 The Constitution, however, does not
require mathematical exactitude or rigid equality as a standard in gauging equality of representation. 46 In fact, for cities, all
it asks is that "each city with a population of at least two hundred fifty thousand shall have one representative," while
ensuring representation for every province regardless of the size of its population. To ensure quality representation
through commonality of interests and ease of access by the representative to the constituents, all that the Constitution
requires is that every legislative district should comprise, as far as practicable, contiguous, compact, and adjacent territory.
Thus, the Constitution leaves the local government units as they are found and does not require their division, merger or
transfer to satisfy the numerical standard it imposes. Its requirements are satisfied despite some numerical disparity if the
units are contiguous, compact and adjacent as far as practicable.

The petitioner's contention that there is a resulting inequality in the division of Cagayan de Oro City into two districts
because the barangays in the first district are mostly rural barangays while the second district is mostly urban, is largely
unsubstantiated. But even if backed up by proper proof, we cannot question the division on the basis of the difference in
the barangays' levels of development or developmental focus as these are not part of the constitutional standards for
legislative apportionment or reapportionment. What the components of the two districts of Cagayan de Oro would be is a
matter for the lawmakers to determine as a matter of policy. In the absence of any grave abuse of discretion or violation of
the established legal parameters, this Court cannot intrude into the wisdom of these policies. 47

WHEREFORE, we hereby DISMISS the petition for lack of merit. Costs against the petitioner. SO ORDERED.

- Reyes v Comelec, 699 SCRA 522 (2013) and 708 SCRA 197 (2013)

This is a Motion for Reconsideration of the En Bane Resolution of 25 June 2013 which stated that: IN VIEW OF THE
FOREGOING, the instant petition is DISMISSED, finding no grave abuse of discretion on the part of the Commission on
Elections. The 14 May 2013 Resolution of the COMELEC En Banc affirming the 27 March 2013 Resolution of the
COMELEC First Division is upheld."

In her Motion for Reconsideration, petitioner summarizes her submission, thus:

"81. Stated differently, the Petitioner x x x is not asking the Honorable Court to make a determination as regards her
qualifications, she is merely asking the Honorable Court to affirm the jurisdiction of the HRET to solely and exclusively
pass upon such qualifications and to set aside the COMELEC Resolutions for having denied Petitioner her right to due
process and for unconstitutionally adding a qualification not otherwise required by the constitution." 1 (as originally
underscored)

The first part of the summary refers to the issue raised in the petition, which is:

"31. Whether or not Respondent Comelec is without jurisdiction over Petitioner who is duly proclaimed winner and who
has already taken her oath of office for the position of Member of the House of Representatives for the lone congressional
district of Marinduque."2

Tied up and neatened the propositions on the COMELEC-or-HRET jurisdiction go thus: petitioner is a duly proclaimed
winner and having taken her oath of office as member of the House of Representatives, all questions regarding her
qualifications are outside the jurisdiction of the COMELEC and are within the HRET exclusive jurisdiction.

The averred proclamation is the critical pointer to the correctness of petitioner's submission. The crucial question is
whether or not petitioner could be proclaimed on 18 May 2013. Differently stated, was there basis for the proclamation of
petitioner on 18 May 2013?

Dates and events indicate that there was no basis for the proclamation of petitioner on 18 May 2013. Without the
proclamation, the petitioner's oath of office is likewise baseless, and without a precedent oath of office, there can be no
valid and effective assumption of office.

We have clearly stated in our Resolution of 5 June 2013 that:


"More importantly, we cannot disregard a fact basic in this controversy – that before the proclamation of petitioner on 18
May 2013, the COMELEC En Banc had already finally disposed of the issue of petitioner's lack of Filipino citizenship and
residency via its Resolution dated 14 May 2013. After 14 May 2013, there was, before the COMELEC, no longer any
pending case on petitioner's qualifications to run for the position of Member of the House of Representatives. x x x As the
point has obviously been missed by the petitioner who continues to argue on the basis of her due proclamation, the instant
motion gives us the opportunity to highlight the undeniable fact we here repeat that the proclamation which petitioner
secured on 18 May 2013 was WITHOUT ANY BASIS.

1. Four (4) days BEFORE the 18 May 2013 proclamation, or on 14 May 2013, the COMELEC En Banc has
already denied for lack o merit the petitioner's motion to reconsider the decision o the COMELEC First Division
that CANCELLED petitioner's certificate of candidacy.

2. On 18 May 2013, there was already a standing and unquestioned cancellation of petitioner's certificate o
candidacy which cancellation is a definite bar to her proclamation. On 18 May 2003, that bar has not been
removed, there was not even any attempt to remove it.

3. The COMELEC Rules indicate the manner by which the impediment to proclamation may be removed. Rule 18,
Section 13 (b) provides:

"(b) In Special Actions and Special Cases a decision or resolution of the Commission En Bane shall become final
and executory after five (5) days from its promulgation unless restrained by the Supreme Court."

Within that five (5 days, petitioner had the opportunity to go to the Supreme Court for a restraining order that will
remove the immediate effect of the En Banc cancellation of her certificate of candidacy. Within the five (5) days the
Supreme Court may remove the barrier to, and thus allow, the proclamation of petitioner. That did not happen.
Petitioner did not move to have it happen.

It is error to argue that the five days should pass before the petitioner is barred from being proclaimed. Petitioner
lost in the COMELEC as of respondent. Her certificate of candidacy has been ordered cancelled. She could not be
proclaimed because there was a final finding against her by the COMELEC. 3 She needed a restraining order from
the Supreme Court to avoid the final finding. After the five days when the decision adverse to her became
executory, the need for Supreme Court intervention became even more imperative. She would have to base her
recourse on the position that the COMELEC committed grave abuse of discretion in cancelling her certificate of
candidacy and that a restraining order, which would allow her proclamation, will have to be based on irreparable
injury and demonstrated possibility of grave abuse of discretion on the part of the COMELEC. In this case, before
and after the 18 May 2013 proclamation, there was not even an attempt at the legal remedy, clearly available to
her, to permit her proclamation. What petitioner did was to "take the law into her hands" and secure a proclamation
in complete disregard of the COMELEC En Bane decision that was final on 14 May 2013 and final and executory
five days thereafter.

4. There is a reason why no mention about notice was made in Section 13(b) of Rule 18 in the provision that the
COMELEC En Bane or decision "SHALL become FINAL AND EXECUTORY after five days from its promulgation
unless restrained by the Supreme Court." On its own the COMELEC En Bane decision, unrestrained, moves from
promulgation into becoming final and executory. This is so because in Section 5 of Rule 18 it is stated:

Section 5. Promulgation. -The promulgation of a decision or resolutions of the Commission or a division shall be made on
a date previously fixed, of which notice shall be served in advance upon the parties or their attorneys personally or by
registered mail or by telegram.

5. Apart from the presumed notice of the COMELEC En Bane decision on the very date of its promulgation on 14
May 2013, petitioner admitted in her petition before us that she in fact received a copy of the decision on 16 May
20 13.4 On that date, she had absolutely no reason why she would disregard the available legal way to remove the
restraint on her proclamation, and, more than that, to in fact secure a proclamation two days thereafter. The utter
disregard of a final COMELEC En Bane decision and of the Rule stating that her proclamation at that point MUST
be on permission by the Supreme Court is even indicative of bad faith on the part of the petitioner.

6. The indicant is magnified by the fact that petitioner would use her tainted proclamation as the very reason to
support her argument that she could no longer be reached by the jurisdiction of the COMELEC; and that it is the
HRET that has exclusive jurisdiction over the issue of her qualifications for office.

7. The suggestions of bad faith aside, petitioner is in error in the conclusion at which she directs, as well as in her
objective quite obvious from such conclusion. It is with her procured proclamation that petitioner nullifies the
COMELEC's decision, by Division and then En Banc and pre-empts any Supreme Court action on the COMELEC
decision. In other words, petitioner repudiates by her proclamation all administrative and judicial actions thereon,
past and present. And by her proclamation, she claims as acquired the congressional seat that she sought to be a
candidate for. As already shown, the reasons that lead to the impermissibility of the objective are clear. She
cannot sit as Member of the House of Representatives by virtue of a baseless proclamation knowingly taken, with
knowledge of the existing legal impediment.

8. Petitioner, therefore, is in error when she posits that at present it is the HRET which has exclusive jurisdiction
over her qualifications as a Member of the House of Representatives. That the HRET is the sole judge of all
contests relating to the election, returns and qualifications of the Members of the House of Representatives is a
written constitutional provision. It is, however unavailable to petitioner because she is NOT a Member of the House
at present. The COMELEC never ordered her proclamation as the rightful winner in the election for such
membership.5 Indeed, the action for cancellation of petitioner's certificate of candidacy, the decision in which is the
indispensable determinant of the right of petitioner to proclamation, was correctly lodged in the COMELEC, was
completely and fully litigated in the COMELEC and was finally decided by the COMELEC. On and after 14 May
2013, there was nothing left for the COMELEC to do to decide the case. The decision sealed the proceedings in
the COMELEC regarding petitioner's ineligibility as a candidate for Representative of Marinduque. The decision
erected the bar to petitioner's proclamation. The bar remained when no restraining order was obtained by
petitioner from the Supreme Court within five days from 14 May 2013.

9. When petitioner finally went to the Supreme Court on 10 June 2013 questioning the COMELEC First Division
ruling and the 14 May 2013 COMELEC En Bane decision, her baseless proclamation on 18 May 2013 did not by
that fact of promulgation alone become valid and legal. A decision favorable to her by the Supreme Court
regarding the decision of the COMELEC En Bane on her certificate of candidacy was indispensably needed, not to
legalize her proclamation on 18 May 2013 but to authorize a proclamation with the Supreme Court decision as
basis.

10. The recourse taken on 25 June 2013 in the form of an original and special civil action for a writ of Certiorari
through Rule 64 of the Rules of Court is circumscribed by set rules and principles.

a) The special action before the COMELEC which was a Petition to Cancel Certificate of Candidacy was a
SUMMARY PROCEEDING or one heard summarily. The nature of the proceedings is best indicated by the
COMELEC Rule on Special Actions, Rule 23, Section 4 of which states that the Commission may
designate any of its officials who are members of the Philippine Bar to hear the case and to receive
evidence. COMELEC Rule 17 further provides in Section 3 that when the proceedings are authorized to be
summary, in lieu of oral testimonies, the parties may, after due notice, be required to submit their position
paper together with affidavits, counter-affidavits and other documentary evidence; x x x and that this
provision shall likewise apply to cases where the hearing and reception of evidence are delegated by the
Commission or the Division to any of its officials x x x.

b) The special and civil action of Certiorari is defined in the Rules of Court thus:

When any tribunal, board or officer exercising judicial or quasi-judicial functions has acted without or in excess of its or his
jurisdiction, or with grave abuse of discretion amounting to lack or excess of jurisdiction, and there is no appeal, or any
plain, speedy, and adequate remedy in the ordinary course of law, a person aggrieved thereby may file a verified petition
in the proper court, alleging the facts with certainty and praying that judgment be rendered annulling or modifying the
proceedings of such tribunal, board or officer, and granting such incidental reliefs as law and justice may require.

The accepted definition of grave abuse of discretion is: a capricious and whimsical exercise of judgment so patent and
gross as to amount to an evasion of a positive duty or a virtual refusal to perform a duty enjoined by law, as where the
power is exercised in an arbitrary and despotic manner because of passion or hostility. 6

It is the category of the special action below providing the procedural leeway in the exercise of the COMELEC summary
jurisdiction over the case, in conjunction with the limits of the Supreme Court's authority over the FINAL COMELEC ruling
that is brought before it, that defines the way petitioner's submission before the Court should be adjudicated. Thus further
explained, the disposition of 25 June 2013 is here repeated for affirmation:

Petitioner alleges that the COMELEC gravely abused its discretion when it took cognizance of "newly-discovered
evidence" without the same having been testified on and offered and admitted in evidence. She assails the admission of
the blog article of Eli Obligacion as hearsay and the photocopy of the Certification from the Bureau of Immigration. She
likewise contends that there was a violation of her right to due process of law because she was not given the opportunity
to question and present controverting evidence.

Her contentions are incorrect.

It must be emphasized that the COMELEC is not bound to strictly adhere to the technical rules of procedure in the
presentation of evidence. Under Section 2 of Rule I the COMELEC Rules of Procedure shall be liberally construed in order
x x x to achieve just, expeditious and inexpensive determination and disposition of every action and proceeding brought
before the Commission. In view of the fact that the proceedings in a petition to deny due course or to cancel certificate of
candidacy are summary in nature, then the newly discovered evidence was properly admitted by respondent COMELEC.

Furthermore, there was no denial of due process in the case at bar as petitioner was given every opportunity to argue her
case before the COMELEC. From 10 October 2012 when Tan's petition was filed up to 27 March 2013 when the First
Division rendered its resolution, petitioner had a period of five (5) months to adduce evidence. Unfortunately, she did not
avail herself of the opportunity given her.

Also, in administrative proceedings, procedural due process only requires that the party be given the opportunity or right to
be heard. As held in the case of Sahali v. COMELEC:
The petitioners should be reminded that due process does not necessarily mean or require a hearing, but simply an
opportunity or right to be heard. One may be heard, not solely by verbal presentation but also, and perhaps many times
more creditably and predictable than oral argument, through pleadings. In administrative proceedings moreover, technical
rules of procedure and evidence are not strictly applied; administrative process cannot be fully equated with due process
in its strict judicial sense. Indeed, deprivation of due process cannot be successfully invoked where a party was given the
chance to be he rd on his motion for reconsideration. (Emphasis supplied)

As to the ruling that petitioner s ineligible to run for office on the ground of citizenship, the COMELEC First Division,
discoursed as follows:

"x x x for respondent to reacquire her Filipino citizenship and become eligible for public office the law requires that she
must have accomplished the following acts: (1) take the oath of allegiance to the Republic of the Philippines before the
Consul-General of the Philippine Consulate in the USA; and (2) make a personal and sworn renunciation of her American
citizenship before any public officer authorized to administer an oath.

In the case at bar, there s no showing that respondent complied with the aforesaid requirements. Early on in the
proceeding, respondent hammered on petitioner's lack of proof regarding her American citizenship, contending that it is
petitioner's burden to present a case. She, however, specifically denied that she has become either a permanent resident
or naturalized citizen of the USA.

Due to petitioner's submission of newly-discovered evidence thru a Manifestation dated February 7, 2013, however,
establishing the fact that respondent is a holder of an American passport which she continues to use until June 30 2012
petitioner was able to substantiate his allegations. The burden now shifts to respondent to present substantial evidence to
prove otherwise. This, the respondent utterly failed to do, leading to the conclusion inevitable that respondent falsely
misrepresented in her COC that she is a natural-born Filipino citizen. Unless and until she can establish that she had
availed of the privileges of RA 9225 by becoming a dual Filipino-American citizen, and thereafter, made a valid sworn
renunciation of her American citizenship, she remains to be an American citizen and is, therefore, ineligible to run for and
hold any elective public office in the Philippines." (Emphasis in the original.)

Let us look into the events that led to this petition: In moving for the cancellation of petitioner's COC, respondent submitted
records of the Bureau of Immigration showing that petitioner is a holder of a US passport, and that her status is that of a
balikbayan. At this point, the burden of proof shifted to petitioner, imposing upon her the duty to prove that she is a natural-
born Filipino citizen and has not lost the same, or that she has re-acquired such status in accordance with the provisions
of R.A. No. 9225. Aside from the bare allegation that she is a natural-born citizen, however, petitioner submitted no proof
to support such contention. Neither did she submit any proof as to the inapplicability of R.A. No. 9225 to her.

Notably, in her Motion for Reconsideration before the COMELEC En Bane, petitioner admitted that she is a holder of a US
passport, but she averred that she is only a dual Filipino-American citizen, thus the requirements of R.A. No. 9225 do not
apply to her. Still, attached to the said motion is an Affidavit of Renunciation of Foreign Citizenship dated 24 September
2012. Petitioner explains that she attached said Affidavit if only to show her desire and zeal to serve the people and to
comply with rules, even as a superfluity. We cannot, however, subscribe to petitioner's explanation. If petitioner executed
said Affidavit if only to comply with the rules, then it is an admission that R.A. No. 9225 applies to her. Petitioner cannot
claim that she executed it to address the observations by the COMELEC as the assailed Resolutions were promulgated
only in 2013, while the Affidavit was executed in September 2012. 1âwphi1

Moreover, in the present petition, petitioner added a footnote to her oath of office as Provincial Administrator, to this effect:
This does not mean that Petitioner did not, prior to her taking her oath of office as Provincial Administrator, take her oath
of allegiance for purposes of re-acquisition of natural-born Filipino status, which she reserves to present in the proper
proceeding. The reference to the taking of oath of office is in order to make reference to what is already part of the records
and evidence in the present case and to avoid injecting into the records evidence on matters of fact that was not
previously passed upon by Respondent COMELEC. This statement raises a lot of questions -Did petitioner execute an
oath of allegiance for re-acquisition of natural-born Filipino status? If she did, why did she not present it at the earliest
opportunity before the COMELEC? And is this an admission that she has indeed lost her natural-born Filipino status?

To cover-up her apparent lack of an oath of allegiance as required by R.A. No. 9225, petitioner contends that, since she
took her oath of allegiance in connection with her appointment as Provincial Administrator of Marinduque, she is deemed
to have reacquired her status as a natural-born Filipino citizen.

This contention is misplaced. For one, this issue is being presented for the first time before this Court, as it was never
raised before the COMELEC. For another, said oath of allegiance cannot be considered compliance with Sec. 3 of R.A.
No. 9225 as certain requirements have to be met as prescribed by Memorandum Circular No. AFF-04-01, otherwise
known as the Rules Governing Philippine Citizenship under R.A. No. 9225 and Memorandum Circular No. AFF-05-002
(Revised Rules) and Administrative Order No. 91, Series of 2004 issued by the Bureau of Immigration. Thus, petitioner s
oath of office as Provincial Administrator cannot be considered as the oath of allegiance in compliance with R.A. No. 9225.

These circumstances, taken together, show that a doubt was clearly cast on petitioner s citizenship. Petitioner, however,
failed to clear such doubt.7

11. It may need pointing out that there is no conflict between the COMELEC and the HRET insofar as the
petitioner s being a Representative of Marinduque is concerned. The COMELEC covers the matter of petitioner s
certificate of candidacy, and its due course or its cancellation, which are the pivotal conclusions that determines
who can be legally proclaimed. The matter can go to the Supreme Court but not as a continuation of the
proceedings in the COMELEC, which has in fact ended, but on an original action before the Court grounded on
more than mere error of judgment but on error of jurisdiction for grave abuse of discretion. At and after the
COMELEC En Bane decision, there is no longer any certificate cancellation matter than can go to the HRET. In
that sense, the HRET s constitutional authority opens, over the qualification of its MEMBER, who becomes so only
upon a duly and legally based proclamation, the first and unavoidable step towards such membership. The HRET
jurisdiction over the qualification of the Member of the House of Representatives is original and exclusive, and as
such, proceeds de novo unhampered by the proceedings in the COMELEC which, as just stated has been
terminated. The HRET proceedings is a regular, not summary, proceeding. It will determine who should be the
Member of the House. It must be made clear though, at the risk of repetitiveness, that no hiatus occurs in the
representation of Marinduque in the House because there is such a representative who shall sit as the HRET
proceedings are had till termination. Such representative is the duly proclaimed winner resulting from the
terminated case of cancellation of certificate of candidacy of petitioner. The petitioner is not, cannot, be that
representative. And this, all in all, is the crux of the dispute between the parties: who shall sit in the House in
representation of Marinduque, while there is yet no HRET decision on the qualifications of the Member.

12. As finale, and as explained in the discussion just done, no unwarranted haste can be attributed, as the dissent
does so, to the resolution of this petition promulgated on 25 June 2013. It was not done to prevent the exercise by
the HRET of its constitutional duty. Quite the contrary, the speedy resolution of the petition was done to pave the
way for the unimpeded performance by the HRET of its constitutional role. The petitioner can very well invoke the
authority of the HRET, but not as a sitting member of the House of Representatives. 8

The inhibition of this ponente was moved for. The reason for the denial of the motion was contained in a letter to the
members of the Court on the understanding that the matter was internal to the Court. The ponente now seeks the Courts
approval to have the explanation published as it is now appended to this Resolution.

The motion to withdraw petition filed AFTER the Court has acted thereon, is noted. It may well be in order to remind
petitioner that jurisdiction, once acquired, is not lost upon the instance of the parties, but continues until the case is
terminated.9 When petitioner filed her Petition for Certiorari jurisdiction vested in the Court and, in fact, the Court exercised
such jurisdiction when it acted on the petition. Such jurisdiction cannot be lost by the unilateral withdrawal of the petition by
petitioner.

More importantly, the Resolution dated 25 June 2013, being a valid court issuance, undoubtedly has legal consequences.
Petitioner cannot, by the mere expediency of withdrawing the petition, negative and nullify the Court's Resolution and its
legal effects. At this point, we counsel petitioner against trifling with court processes. Having sought the jurisdiction of the
Supreme Court, petitioner cannot withdraw her petition to erase the ruling adverse to her interests. Obviously, she cannot,
as she designed below, subject to her predilections the supremacy of the law.

WHEREFORE, The Motion for Reconsideration is DENIED. The dismissal of the petition is affirmed. Entry of Judgment is
ordered.

SO ORDERED.

b. Party-list System, RA 7941


- BANAT v Comelec, 586 SCRA 210 (2009) and 592 SCRA 294 (2009)

Petitioner in G.R. No. 179271 — Barangay Association for National Advancement and Transparency (BANAT) — in a
petition for certiorari and mandamus,1 assails the Resolution2 promulgated on 3 August 2007 by the Commission on
Elections (COMELEC) in NBC No. 07-041 (PL). The COMELEC’s resolution in NBC No. 07-041 (PL) approved the
recommendation of Atty. Alioden D. Dalaig, Head of the National Board of Canvassers (NBC) Legal Group, to deny the
petition of BANAT for being moot. BANAT filed before the COMELEC En Banc, acting as NBC, a Petition to Proclaim the
Full Number of Party-List Representatives Provided by the Constitution.

The following are intervenors in G.R. No. 179271: Arts Business and Science Professionals (ABS), Aangat Tayo (AT), and
Coalition of Associations of Senior Citizens in the Philippines, Inc. (Senior Citizens).

Petitioners in G.R. No. 179295 — Bayan Muna, Abono, and Advocacy for Teacher Empowerment Through Action,
Cooperation and Harmony Towards Educational Reforms (A Teacher) — in a petition for certiorari with mandamus and
prohibition,3 assails NBC Resolution No. 07-604 promulgated on 9 July 2007. NBC No. 07-60 made a partial proclamation
of parties, organizations and coalitions that obtained at least two percent of the total votes cast under the Party-List
System. The COMELEC announced that, upon completion of the canvass of the party-list results, it would determine the
total number of seats of each winning party, organization, or coalition in accordance with Veterans Federation Party v.
COMELEC5 (Veterans).

Estrella DL Santos, in her capacity as President and First Nominee of the Veterans Freedom Party, filed a motion to
intervene in both G.R. Nos. 179271 and 179295.

The Facts

The 14 May 2007 elections included the elections for the party-list representatives. The COMELEC counted 15,950,900
votes cast for 93 parties under the Party-List System. 6
On 27 June 2002, BANAT filed a Petition to Proclaim the Full Number of Party-List Representatives Provided by the
Constitution, docketed as NBC No. 07-041 (PL) before the NBC. BANAT filed its petition because "[t]he Chairman and the
Members of the [COMELEC] have recently been quoted in the national papers that the [COMELEC] is duty bound to and
shall implement the Veterans ruling, that is, would apply the Panganiban formula in allocating party-list seats." 7 There were
no intervenors in BANAT’s petition before the NBC. BANAT filed a memorandum on 19 July 2007.

On 9 July 2007, the COMELEC, sitting as the NBC, promulgated NBC Resolution No. 07-60. NBC Resolution No. 07-60
proclaimed thirteen (13) parties as winners in the party-list elections, namely: Buhay Hayaan Yumabong (BUHAY), Bayan
Muna, Citizens’ Battle Against Corruption (CIBAC), Gabriela’s Women Party (Gabriela), Association of Philippine Electric
Cooperatives (APEC), A Teacher, Akbayan! Citizen’s Action Party (AKBAYAN), Alagad, Luzon Farmers Party (BUTIL),
Cooperative-Natco Network Party (COOP-NATCCO), Anak Pawis, Alliance of Rural Concerns (ARC), and Abono. We
quote NBC Resolution No. 07-60 in its entirety below:

WHEREAS, the Commission on Elections sitting en banc as National Board of Canvassers, thru its Sub-Committee for
Party-List, as of 03 July 2007, had officially canvassed, in open and public proceedings, a total of fifteen million two
hundred eighty three thousand six hundred fifty-nine (15,283,659) votes under the Party-List System of
Representation, in connection with the National and Local Elections conducted last 14 May 2007;

WHEREAS, the study conducted by the Legal and Tabulation Groups of the National Board of Canvassers reveals that
the projected/maximum total party-list votes cannot go any higher than sixteen million seven hundred twenty three
thousand one hundred twenty-one (16,723,121) votes given the following statistical data:

Projected/Maximum Party-List Votes for May 2007 Elections

i. Total party-list votes already canvassed/tabulated 15,283,659

ii. Total party-list votes remaining uncanvassed/ 1,337,032


untabulated (i.e. canvass deferred)

iii. Maximum party-list votes (based on 100% outcome) 102,430


from areas not yet submitted for canvass (Bogo,
Cebu; Bais City; Pantar, Lanao del Norte; and
Pagalungan, Maguindanao)

Maximum Total Party-List Votes 16,723,121

WHEREAS, Section 11 of Republic Act No. 7941 (Party-List System Act) provides in part:

The parties, organizations, and coalitions receiving at least two percent (2%) of the total votes cast for the party-list system
shall be entitled to one seat each: provided, that those garnering more than two percent (2%) of the votes shall be entitled
to additional seats in proportion to their total number of votes: provided, finally, that each party, organization, or coalition
shall be entitled to not more than three (3) seats.

WHEREAS, for the 2007 Elections, based on the above projected total of party-list votes, the presumptive two percent
(2%) threshold can be pegged at three hundred thirty four thousand four hundred sixty-two (334,462) votes;

WHEREAS, the Supreme Court, in Citizen’s Battle Against Corruption (CIBAC) versus COMELEC, reiterated its ruling
in Veterans Federation Party versus COMELEC adopting a formula for the additional seats of each party, organization or
coalition receving more than the required two percent (2%) votes, stating that the same shall be determined only after all
party-list ballots have been completely canvassed;

WHEREAS, the parties, organizations, and coalitions that have thus far garnered at least three hundred thirty four
thousand four hundred sixty-two (334,462) votes are as follows:

RANK PARTY/ORGANIZATION/ VOTES


COALITION RECEIVED

1 BUHAY 1,163,218

2 BAYAN MUNA 972,730

3 CIBAC 760,260

4 GABRIELA 610,451

5 APEC 538,971

6 A TEACHER 476,036

7 AKBAYAN 470,872
8 ALAGAD 423,076

9 BUTIL 405,052

10 COOP-NATCO 390,029

11 BATAS 386,361

12 ANAK PAWIS 376,036

13 ARC 338,194

14 ABONO 337,046

WHEREAS, except for Bagong Alyansang Tagapagtaguyod ng Adhikaing Sambayanan (BATAS), against which
an URGENT PETITION FOR CANCELLATION/REMOVAL OF REGISTRATION AND DISQUALIFICATION OF PARTY-
LIST NOMINEE (With Prayer for the Issuance of Restraining Order) has been filed before the Commission, docketed as
SPC No. 07-250, all the parties, organizations and coalitions included in the aforementioned list are therefore entitled to at
least one seat under the party-list system of representation in the meantime.

NOW, THEREFORE, by virtue of the powers vested in it by the Constitution, the Omnibus Election Code, Executive Order
No. 144, Republic Act Nos. 6646, 7166, 7941, and other election laws, the Commission on Elections, sitting en banc as
the National Board of Canvassers, hereby RESOLVES to PARTIALLY PROCLAIM, subject to certain conditions set forth
below, the following parties, organizations and coalitions participating under the Party-List System:

1 Buhay Hayaan Yumabong BUHAY

2 Bayan Muna BAYAN MUNA

3 Citizens Battle Against Corruption CIBAC

4 Gabriela Women’s Party GABRIELA

5 Association of Philippine Electric Cooperatives APEC

6 Advocacy for Teacher Empowerment Through A TEACHER


Action, Cooperation and Harmony Towards
Educational Reforms, Inc.

7 Akbayan! Citizen’s Action Party AKBAYAN

8 Alagad ALAGAD

9 Luzon Farmers Party BUTIL

10 Cooperative-Natco Network Party COOP-NATCCO

11 Anak Pawis ANAKPAWIS

12 Alliance of Rural Concerns ARC

13 Abono ABONO

This is without prejudice to the proclamation of other parties, organizations, or coalitions which may later on be established
to have obtained at least two percent (2%) of the total actual votes cast under the Party-List System.

The total number of seats of each winning party, organization or coalition shall be determined pursuant to Veterans
Federation Party versus COMELEC formula upon completion of the canvass of the party-list results.

The proclamation of Bagong Alyansang Tagapagtaguyod ng Adhikaing Sambayanan (BATAS) is hereby deferred until
final resolution of SPC No. 07-250, in order not to render the proceedings therein moot and academic.

Finally, all proclamation of the nominees of concerned parties, organizations and coalitions with pending disputes shall
likewise be held in abeyance until final resolution of their respective cases.

Let the Clerk of the Commission implement this Resolution, furnishing a copy thereof to the Speaker of the House of
Representatives of the Philippines.

SO ORDERED.8 (Emphasis in the original)


Pursuant to NBC Resolution No. 07-60, the COMELEC, acting as NBC, promulgated NBC Resolution No. 07-72, which
declared the additional seats allocated to the appropriate parties. We quote from the COMELEC’s interpretation of
the Veterans formula as found in NBC Resolution No. 07-72:

WHEREAS, on July 9, 2007, the Commission on Elections sitting en banc as the National Board of Canvassers
proclaimed thirteen (13) qualified parties, organization[s] and coalitions based on the presumptive two percent (2%)
threshold of 334,462 votes from the projected maximum total number of party-list votes of 16,723,121, and were thus
given one (1) guaranteed party-list seat each;

WHEREAS, per Report of the Tabulation Group and Supervisory Committee of the National Board of Canvassers, the
projected maximum total party-list votes, as of July 11, 2007, based on the votes actually canvassed, votes canvassed but
not included in Report No. 29, votes received but uncanvassed, and maximum votes expected for Pantar, Lanao del
Norte, is 16,261,369; and that the projected maximum total votes for the thirteen (13) qualified parties, organizations and
coalition[s] are as follows:

  Party-List Projected total number of votes

1 BUHAY 1,178,747

2 BAYAN MUNA 977,476

3 CIBAC 755,964

4 GABRIELA 621,718

5 APEC 622,489

6 A TEACHER 492,369

7 AKBAYAN 462,674

8 ALAGAD 423,190

9 BUTIL 409,298

10 COOP-NATCO 412,920

11 ANAKPAWIS 370,165

12 ARC 375,846

13 ABONO 340,151

WHEREAS, based on the above Report, Buhay Hayaan Yumabong (Buhay) obtained the highest number of votes among
the thirteen (13) qualified parties, organizations and coalitions, making it the "first party" in accordance with Veterans
Federation Party versus COMELEC, reiterated in Citizen’s Battle Against Corruption (CIBAC) versus COMELEC;

WHEREAS, qualified parties, organizations and coalitions participating under the party-list system of representation that
have obtained one guaranteed (1) seat may be entitled to an additional seat or seats based on the formula prescribed by
the Supreme Court in Veterans;

WHEREAS, in determining the additional seats for the "first party", the correct formula as expressed in Veterans, is:

Number of votes of first party Proportion of votes of first


= party relative to total votes for
Total votes for party-list system party-list system

wherein the proportion of votes received by the first party (without rounding off) shall entitle it to additional seats:

Proportion of votes received Additional seats


by the first party

Equal to or at least 6% Two (2) additional seats

Equal to or greater than 4% but less than 6% One (1) additional seat

Less than 4% No additional seat

WHEREAS, applying the above formula, Buhay obtained the following percentage:
1,178,747
= 0.07248 or 7.2%
16,261,369

which entitles it to two (2) additional seats.

WHEREAS, in determining the additional seats for the other qualified parties, organizations and coalitions, the correct
formula as expressed in Veterans and reiterated in CIBAC is, as follows:

No. of votes of
concerned party No. of additional
Additional seats for
= x seats allocated
a concerned party
No. of votes of to first party
first party

WHEREAS, applying the above formula, the results are as follows:

Party List Percentage Additional Seat

BAYAN MUNA 1.65 1

CIBAC 1.28 1

GABRIELA 1.05 1

APEC 1.05 1

A TEACHER 0.83 0

AKBAYAN 0.78 0

ALAGAD 0.71 0

BUTIL 0.69 0

COOP-NATCO 0.69 0

ANAKPAWIS 0.62 0

ARC 0.63 0

ABONO 0.57 0

NOW THEREFORE, by virtue of the powers vested in it by the Constitution, Omnibus Election Code, Executive Order No.
144, Republic Act Nos. 6646, 7166, 7941 and other elections laws, the Commission on Elections en banc sitting as the
National Board of Canvassers, hereby RESOLVED, as it hereby RESOLVES, to proclaim the following parties,
organizations or coalitions as entitled to additional seats, to wit:

Party List Additional Seats

BUHAY 2

BAYAN MUNA 1

CIBAC 1

GABRIELA 1

APEC 1

This is without prejudice to the proclamation of other parties, organizations or coalitions which may later on be established
to have obtained at least two per cent (2%) of the total votes cast under the party-list system to entitle them to one (1)
guaranteed seat, or to the appropriate percentage of votes to entitle them to one (1) additional seat.

Finally, all proclamation of the nominees of concerned parties, organizations and coalitions with pending disputes shall
likewise be held in abeyance until final resolution of their respective cases.

Let the National Board of Canvassers Secretariat implement this Resolution, furnishing a copy hereof to the Speaker of
the House of Representatives of the Philippines.
SO ORDERED.9

Acting on BANAT’s petition, the NBC promulgated NBC Resolution No. 07-88 on 3 August 2007, which reads as follows:

This pertains to the Petition to Proclaim the Full Number of Party-List Representatives Provided by the Constitution filed by
the Barangay Association for National Advancement and Transparency (BANAT).

Acting on the foregoing Petition of the Barangay Association for National Advancement and Transparency (BANAT) party-
list, Atty. Alioden D. Dalaig, Head, National Board of Canvassers Legal Group submitted his comments/observations and
recommendation thereon [NBC 07-041 (PL)], which reads:

COMMENTS / OBSERVATIONS:

Petitioner Barangay Association for National Advancement and Transparency (BANAT), in its Petition to Proclaim the Full
Number of Party-List Representatives Provided by the Constitution prayed for the following reliefs, to wit:

1. That the full number -- twenty percent (20%) -- of Party-List representatives as mandated by Section 5, Article
VI of the Constitution shall be proclaimed.

2. Paragraph (b), Section 11 of RA 7941 which prescribes the 2% threshold votes, should be harmonized with
Section 5, Article VI of the Constitution and with Section 12 of the same RA 7941 in that it should be applicable
only to the first party-list representative seats to be allotted on the basis of their initial/first ranking.

3. The 3-seat limit prescribed by RA 7941 shall be applied; and

4. Initially, all party-list groups shall be given the number of seats corresponding to every 2% of the votes they
received and the additional seats shall be allocated in accordance with Section 12 of RA 7941, that is, in
proportion to the percentage of votes obtained by each party-list group in relation to the total nationwide votes cast
in the party-list election, after deducting the corresponding votes of those which were allotted seats under the 2%
threshold rule. In fine, the formula/procedure prescribed in the "ALLOCATION OF PARTY-LIST SEATS, ANNEX
"A" of COMELEC RESOLUTION 2847 dated 25 June 1996, shall be used for [the] purpose of determining how
many seats shall be proclaimed, which party-list groups are entitled to representative seats and how many of their
nominees shall seat [sic].

5. In the alternative, to declare as unconstitutional Section 11 of Republic Act No. 7941 and that the procedure in
allocating seats for party-list representative prescribed by Section 12 of RA 7941 shall be followed.

R E C O M M E N D A T I O N:

The petition of BANAT is now moot and academic.

The Commission En Banc in NBC Resolution No. 07-60 promulgated July 9, 2007 re "In the Matter of the Canvass of
Votes and Partial Proclamation of the Parties, Organizations and Coalitions Participating Under the Party-List System
During the May 14, 2007 National and Local Elections" resolved among others that the total number of seats of each
winning party, organization or coalition shall be determined pursuant to the Veterans Federation
Party versus COMELEC formula upon completion of the canvass of the party-list results." 1awphi1

WHEREFORE, premises considered, the National Board of Canvassers RESOLVED, as it hereby RESOLVES, to
approve and adopt the recommendation of Atty. Alioden D. Dalaig, Head, NBC Legal Group, to DENY the herein petition
of BANAT for being moot and academic.

Let the Supervisory Committee implement this resolution.

SO ORDERED.10

BANAT filed a petition for certiorari and mandamus assailing the ruling in NBC Resolution No. 07-88. BANAT did not file a
motion for reconsideration of NBC Resolution No. 07-88.

On 9 July 2007, Bayan Muna, Abono, and A Teacher asked the COMELEC, acting as NBC, to reconsider its decision to
use the Veterans formula as stated in its NBC Resolution No. 07-60 because the Veterans formula is violative of the
Constitution and of Republic Act No. 7941 (R.A. No. 7941). On the same day, the COMELEC denied reconsideration
during the proceedings of the NBC.11

Aside from the thirteen party-list organizations proclaimed on 9 July 2007, the COMELEC proclaimed three other party-list
organizations as qualified parties entitled to one guaranteed seat under the Party-List System: Agricultural Sector Alliance
of the Philippines, Inc. (AGAP),12 Anak Mindanao (AMIN),13 and An Waray.14 Per the certification15 by COMELEC, the
following party-list organizations have been proclaimed as of 19 May 2008:

Party-List No. of Seat(s)


1.1 Buhay 3
1.2 Bayan Muna 2
1.3 CIBAC 2
1.4 Gabriela 2
1.5 APEC 2
1.6 A Teacher 1
1.7 Akbayan 1
1.8 Alagad 1
1.9 Butil 1
1.10 Coop-Natco [sic] 1
1.11 Anak Pawis 1
1.12 ARC 1
1.13 Abono 1
1.14 AGAP 1
1.15 AMIN 1

The proclamation of Bagong Alyansang Tagapagtaguyod ng Adhikaing Sambayanan (BATAS), against which an Urgent
Petition for Cancellation/Removal of Registration and Disqualification of Party-list Nominee (with Prayer for the Issuance of
Restraining Order) has been filed before the COMELEC, was deferred pending final resolution of SPC No. 07-250.

Issues

BANAT brought the following issues before this Court:

1. Is the twenty percent allocation for party-list representatives provided in Section 5(2), Article VI of the
Constitution mandatory or is it merely a ceiling?

2. Is the three-seat limit provided in Section 11(b) of RA 7941 constitutional?

3. Is the two percent threshold and "qualifier" votes prescribed by the same Section 11(b) of RA 7941
constitutional?

4. How shall the party-list representatives be allocated? 16

Bayan Muna, A Teacher, and Abono, on the other hand, raised the following issues in their petition:

I. Respondent Commission on Elections, acting as National Board of Canvassers, committed grave abuse of
discretion amounting to lack or excess of jurisdiction when it promulgated NBC Resolution No. 07-60 to implement
the First-Party Rule in the allocation of seats to qualified party-list organizations as said rule:

A. Violates the constitutional principle of proportional representation.

B. Violates the provisions of RA 7941 particularly:

1. The 2-4-6 Formula used by the First Party Rule in allocating additional seats for the "First Party"
violates the principle of proportional representation under RA 7941.

2. The use of two formulas in the allocation of additional seats, one for the "First Party" and another
for the qualifying parties, violates Section 11(b) of RA 7941.

3. The proportional relationships under the First Party Rule are different from those required under
RA 7941;

C. Violates the "Four Inviolable Parameters" of the Philippine party-list system as provided for under the
same case of Veterans Federation Party, et al. v. COMELEC.

II. Presuming that the Commission on Elections did not commit grave abuse of discretion amounting to lack or
excess of jurisdiction when it implemented the First-Party Rule in the allocation of seats to qualified party-list
organizations, the same being merely in consonance with the ruling in Veterans Federations Party, et al. v.
COMELEC, the instant Petition is a justiciable case as the issues involved herein are constitutional in nature,
involving the correct interpretation and implementation of RA 7941, and are of transcendental importance to our
nation.17

Considering the allegations in the petitions and the comments of the parties in these cases, we defined the
following issues in our advisory for the oral arguments set on 22 April 2008:

1. Is the twenty percent allocation for party-list representatives in Section 5(2), Article VI of the Constitution
mandatory or merely a ceiling?

2. Is the three-seat limit in Section 11(b) of RA 7941 constitutional?

3. Is the two percent threshold prescribed in Section 11(b) of RA 7941 to qualify for one seat
constitutional?

4. How shall the party-list representative seats be allocated?

5. Does the Constitution prohibit the major political parties from participating in the party-list elections? If
not, can the major political parties be barred from participating in the party-list elections? 18

The Ruling of the Court

The petitions have partial merit. We maintain that a Philippine-style party-list election has at least four inviolable
parameters as clearly stated in Veterans. For easy reference, these are:

First, the twenty percent allocation — the combined number of all party-list congressmen shall not exceed twenty
percent of the total membership of the House of Representatives, including those elected under the party list;

Second, the two percent threshold — only those parties garnering a minimum of two percent of the total valid
votes cast for the party-list system are "qualified" to have a seat in the House of Representatives;

Third, the three-seat limit — each qualified party, regardless of the number of votes it actually obtained, is entitled
to a maximum of three seats; that is, one "qualifying" and two additional seats;

Fourth, proportional representation— the additional seats which a qualified party is entitled to shall be computed
"in proportion to their total number of votes."19

However, because the formula in Veterans has flaws in its mathematical interpretation of the term "proportional
representation," this Court is compelled to revisit the formula for the allocation of additional seats to party-list
organizations.

Number of Party-List Representatives:


The Formula Mandated by the Constitution

Section 5, Article VI of the Constitution provides:

Section 5. (1) The House of Representatives shall be composed of not more than two hundred and fifty members, unless
otherwise fixed by law, who shall be elected from legislative districts apportioned among the provinces, cities, and the
Metropolitan Manila area in accordance with the number of their respective inhabitants, and on the basis of a uniform and
progressive ratio, and those who, as provided by law, shall be elected through a party-list system of registered national,
regional, and sectoral parties or organizations.

(2) The party-list representatives shall constitute twenty per centum of the total number of representatives including those
under the party-list. For three consecutive terms after the ratification of this Constitution, one-half of the seats allocated to
party-list representatives shall be filled, as provided by law, by selection or election from the labor, peasant, urban poor,
indigenous cultural communities, women, youth, and such other sectors as may be provided by law, except the religious
sector.

The first paragraph of Section 11 of R.A. No. 7941 reads:

Section 11. Number of Party-List Representatives. — The party-list representatives shall constitute twenty per centum
(20%) of the total number of the members of the House of Representatives including those under the party-list.

xxx

Section 5(1), Article VI of the Constitution states that the "House of Representatives shall be composed of not more than
two hundred and fifty members, unless otherwise fixed by law." The House of Representatives shall be composed of
district representatives and party-list representatives. The Constitution allows the legislature to modify the number of the
members of the House of Representatives. 1avvphi1.zw+
Section 5(2), Article VI of the Constitution, on the other hand, states the ratio of party-list representatives to the total
number of representatives. We compute the number of seats available to party-list representatives from the number of
legislative districts. On this point, we do not deviate from the first formula in Veterans, thus:

Number of seats
available to legislative
Number of seats available to
districts
x .20 = party-list representatives
.80

This formula allows for the corresponding increase in the number of seats available for party-list representatives whenever
a legislative district is created by law. Since the 14th Congress of the Philippines has 220 district representatives, there
are 55 seats available to party-list representatives.

220
x .20 = 55
.80

After prescribing the ratio of the number of party-list representatives to the total number of representatives, the
Constitution left the manner of allocating the seats available to party-list representatives to the wisdom of the
legislature.

Allocation of Seats for Party-List Representatives:


The Statutory Limits Presented by the Two Percent Threshold
and the Three-Seat Cap

All parties agree on the formula to determine the maximum number of seats reserved under the Party-List System, as well
as on the formula to determine the guaranteed seats to party-list candidates garnering at least two-percent of the total
party-list votes. However, there are numerous interpretations of the provisions of R.A. No. 7941 on the allocation
of "additional seats" under the Party-List System. Veterans produced the First Party Rule,20 and Justice Vicente V.
Mendoza’s dissent in Veterans presented Germany’s Niemeyer formula21 as an alternative.

The Constitution left to Congress the determination of the manner of allocating the seats for party-list representatives.
Congress enacted R.A. No. 7941, paragraphs (a) and (b) of Section 11 and Section 12 of which provide:

Section 11. Number of Party-List Representatives. — x x x

In determining the allocation of seats for the second vote,22 the following procedure shall be observed:

(a) The parties, organizations, and coalitions shall be ranked from the highest to the lowest based on the number
of votes they garnered during the elections.

(b) The parties, organizations, and coalitions receiving at least two percent (2%) of the total votes cast for the
party-list system shall be entitled to one seat each: Provided, That those garnering more than two percent
(2%) of the votes shall be entitled to additional seats in proportion to their total number of votes: Provided,
finally, That each party, organization, or coalition shall be entitled to not more than three (3) seats.

Section 12. Procedure in Allocating Seats for Party-List Representatives. — The COMELEC shall tally all the votes for the
parties, organizations, or coalitions on a nationwide basis, rank them according to the number of votes received and
allocate party-list representatives proportionately according to the percentage of votes obtained by each party,
organization, or coalition as against the total nationwide votes cast for the party-list system. (Emphasis supplied)

In G.R. No. 179271, BANAT presents two interpretations through three formulas to allocate party-list representative seats.

The first interpretation allegedly harmonizes the provisions of Section 11(b) on the 2% requirement with Section 12 of R.A.
No. 7941. BANAT described this procedure as follows:

(a) The party-list representatives shall constitute twenty percent (20%) of the total Members of the House of
Representatives including those from the party-list groups as prescribed by Section 5, Article VI of the
Constitution, Section 11 (1st par.) of RA 7941 and Comelec Resolution No. 2847 dated 25 June 1996. Since there
are 220 District Representatives in the 14th Congress, there shall be 55 Party-List Representatives. All seats shall
have to be proclaimed.

(b) All party-list groups shall initially be allotted one (1) seat for every two per centum (2%) of the total party-list
votes they obtained; provided, that no party-list groups shall have more than three (3) seats (Section 11, RA
7941).

(c) The remaining seats shall, after deducting the seats obtained by the party-list groups under the immediately
preceding paragraph and after deducting from their total the votes corresponding to those seats, the remaining
seats shall be allotted proportionately to all the party-list groups which have not secured the maximum three (3)
seats under the 2% threshold rule, in accordance with Section 12 of RA 7941. 23

Forty-four (44) party-list seats will be awarded under BANAT’s first interpretation.

The second interpretation presented by BANAT assumes that the 2% vote requirement is declared unconstitutional, and
apportions the seats for party-list representatives by following Section 12 of R.A. No. 7941. BANAT states that the
COMELEC:

(a) shall tally all the votes for the parties, organizations, or coalitions on a nationwide basis;

(b) rank them according to the number of votes received; and,

(c) allocate party-list representatives proportionately according to the percentage of votes obtained by each party,
organization or coalition as against the total nationwide votes cast for the party-list system. 24

BANAT used two formulas to obtain the same results: one is based on the proportional percentage of the votes received
by each party as against the total nationwide party-list votes, and the other is "by making the votes of a party-list with a
median percentage of votes as the divisor in computing the allocation of seats." 25 Thirty-four (34) party-list seats will be
awarded under BANAT’s second interpretation.

In G.R. No. 179295, Bayan Muna, Abono, and A Teacher criticize both the COMELEC’s original 2-4-6 formula and
the Veterans formula for systematically preventing all the party-list seats from being filled up. They claim that both
formulas do not factor in the total number of seats alloted for the entire Party-List System. Bayan Muna, Abono, and A
Teacher reject the three-seat cap, but accept the 2% threshold. After determining the qualified parties, a second
percentage is generated by dividing the votes of a qualified party by the total votes of all qualified parties only. The number
of seats allocated to a qualified party is computed by multiplying the total party-list seats available with the second
percentage. There will be a first round of seat allocation, limited to using the whole integers as the equivalent of the
number of seats allocated to the concerned party-list. After all the qualified parties are given their seats, a second round of
seat allocation is conducted. The fractions, or remainders, from the whole integers are ranked from highest to lowest and
the remaining seats on the basis of this ranking are allocated until all the seats are filled up. 26

We examine what R.A. No. 7941 prescribes to allocate seats for party-list representatives.

Section 11(a) of R.A. No. 7941 prescribes the ranking of the participating parties from the highest to the lowest based on
the number of votes they garnered during the elections.

Table 1. Ranking of the participating parties from the highest to the lowest based on the number of votes garnered during
the elections.27

Votes Votes
Rank Party Rank Party
Garnered Garnered

1 BUHAY 1,169,234 48 KALAHI 88,868

2 BAYAN MUNA 979,039 49 APOI 79,386

3 CIBAC 755,686 50 BP 78,541

4 GABRIELA 621,171 51 AHONBAYAN 78,424

5 APEC 619,657 52 BIGKIS 77,327

6 A TEACHER 490,379 53 PMAP 75,200

7 AKBAYAN 466,112 54 AKAPIN 74,686

8 ALAGAD 423,149 55 PBA 71,544

9 COOP-NATCCO 409,883 56 GRECON 62,220

10 BUTIL 409,160 57 BTM 60,993

11 BATAS 385,810 58 A SMILE 58,717

12 ARC 374,288 59 NELFFI 57,872

13 ANAKPAWIS 370,261 60 AKSA 57,012

14 ABONO 339,990 61 BAGO 55,846

15 AMIN 338,185 62 BANDILA 54,751


16 AGAP 328,724 63 AHON 54,522

17 AN WARAY 321,503 64 ASAHAN MO 51,722

18 YACAP 310,889 65 AGBIAG! 50,837

19 FPJPM 300,923 66 SPI 50,478

20 UNI-MAD 245,382 67 BAHANDI 46,612

21 ABS 235,086 68 ADD 45,624

22 KAKUSA 228,999 69 AMANG 43,062

23 KABATAAN 228,637 70 ABAY PARAK 42,282

24 ABA-AKO 218,818 71 BABAE KA 36,512

25 ALIF 217,822 72 SB 34,835

26 SENIOR CITIZENS 213,058 73 ASAP 34,098

27 AT 197,872 74 PEP 33,938

28 VFP 196,266 75 ABA ILONGGO 33,903

29 ANAD 188,521 76 VENDORS 33,691

30 BANAT 177,028 77 ADD-TRIBAL 32,896

31 ANG KASANGGA 170,531 78 ALMANA 32,255

32 BANTAY 169,801 79 AANGAT KA PILIPINO 29,130

33 ABAKADA 166,747 80 AAPS 26,271

34 1-UTAK 164,980 81 HAPI 25,781

35 TUCP 162,647 82 AAWAS 22,946

36 COCOFED 155,920 83 SM 20,744

37 AGHAM 146,032 84 AG 16,916

38 ANAK 141,817 85 AGING PINOY 16,729

39 ABANSE! PINAY 130,356 86 APO 16,421

40 PM 119,054 87 BIYAYANG BUKID 16,241

41 AVE 110,769 88 ATS 14,161

42 SUARA 110,732 89 UMDJ 9,445

43 ASSALAM 110,440 90 BUKLOD FILIPINA 8,915

44 DIWA 107,021 91 LYPAD 8,471

45 ANC 99,636 92 AA-KASOSYO 8,406

46 SANLAKAS 97,375 93 KASAPI 6,221

47 ABC 90,058 TOTAL 15,950,900

The first clause of Section 11(b) of R.A. No. 7941 states that "parties, organizations, and coalitions receiving at least two
percent (2%) of the total votes cast for the party-list system shall be entitled to one seat each." This clause guarantees a
seat to the two-percenters. In Table 2 below, we use the first 20 party-list candidates for illustration purposes. The
percentage of votes garnered by each party is arrived at by dividing the number of votes garnered by each party by
15,950,900, the total number of votes cast for all party-list candidates.

Table 2. The first 20 party-list candidates and their respective percentage of votes garnered over the total votes for the
party-list.28

Rank Party Votes Votes Garnered over Guaranteed


Garnered Total Votes for Seat
Party-List, in %

1 BUHAY 1,169,234 7.33% 1

2 BAYAN MUNA 979,039 6.14% 1

3 CIBAC 755,686 4.74% 1

4 GABRIELA 621,171 3.89% 1

5 APEC 619,657 3.88% 1

6 A TEACHER 490,379 3.07% 1

7 AKBAYAN 466,112 2.92% 1

8 ALAGAD 423,149 2.65% 1

9 COOP-NATCCO 409,883 2.57% 1

10 BUTIL 409,160 2.57% 1

11 BATAS29 385,810 2.42% 1

12 ARC 374,288 2.35% 1

13 ANAKPAWIS 370,261 2.32% 1

14 ABONO 339,990 2.13% 1

15 AMIN 338,185 2.12% 1

16 AGAP 328,724 2.06% 1

17 AN WARAY 321,503 2.02% 1

  Total     17

18 YACAP 310,889 1.95% 0

19 FPJPM 300,923 1.89% 0

20 UNI-MAD 245,382 1.54% 0

From Table 2 above, we see that only 17 party-list candidates received at least 2% from the total number of votes cast for
party-list candidates. The 17 qualified party-list candidates, or the two-percenters, are the party-list candidates that are
"entitled to one seat each," or the guaranteed seat. In this first round of seat allocation, we distributed 17 guaranteed
seats.

The second clause of Section 11(b) of R.A. No. 7941 provides that "those garnering more than two percent (2%) of the
votes shall be entitled to additional seats in proportion to their total number of votes." This is where petitioners’ and
intervenors’ problem with the formula in Veterans lies. Veterans interprets the clause "in proportion to their total number of
votes" to be in proportion to the votes of the first party. This interpretation is contrary to the express language of R.A.
No. 7941.

We rule that, in computing the allocation of additional seats, the continued operation of the two percent threshold for the
distribution of the additional seats as found in the second clause of Section 11(b) of R.A. No. 7941 is unconstitutional.
This Court finds that the two percent threshold makes it mathematically impossible to achieve the maximum number of
available party list seats when the number of available party list seats exceeds 50. The continued operation of the two
percent threshold in the distribution of the additional seats frustrates the attainment of the permissive ceiling that 20% of
the members of the House of Representatives shall consist of party-list representatives.

To illustrate: There are 55 available party-list seats. Suppose there are 50 million votes cast for the 100 participants in the
party list elections. A party that has two percent of the votes cast, or one million votes, gets a guaranteed seat. Let us
further assume that the first 50 parties all get one million votes. Only 50 parties get a seat despite the availability of 55
seats. Because of the operation of the two percent threshold, this situation will repeat itself even if we increase the
available party-list seats to 60 seats and even if we increase the votes cast to 100 million. Thus, even if the maximum
number of parties get two percent of the votes for every party, it is always impossible for the number of occupied party-list
seats to exceed 50 seats as long as the two percent threshold is present.

We therefore strike down the two percent threshold only in relation to the distribution of the additional seats as found in the
second clause of Section 11(b) of R.A. No. 7941. The two percent threshold presents an unwarranted obstacle to the full
implementation of Section 5(2), Article VI of the Constitution and prevents the attainment of "the broadest possible
representation of party, sectoral or group interests in the House of Representatives." 30
In determining the allocation of seats for party-list representatives under Section 11 of R.A. No. 7941, the following
procedure shall be observed:

1. The parties, organizations, and coalitions shall be ranked from the highest to the lowest based on the number of
votes they garnered during the elections.

2. The parties, organizations, and coalitions receiving at least two percent (2%) of the total votes cast for the party-
list system shall be entitled to one guaranteed seat each.

3. Those garnering sufficient number of votes, according to the ranking in paragraph 1, shall be entitled to
additional seats in proportion to their total number of votes until all the additional seats are allocated.

4. Each party, organization, or coalition shall be entitled to not more than three (3) seats.

In computing the additional seats, the guaranteed seats shall no longer be included because they have already been
allocated, at one seat each, to every two-percenter. Thus, the remaining available seats for allocation as "additional seats"
are the maximum seats reserved under the Party List System less the guaranteed seats. Fractional seats are disregarded
in the absence of a provision in R.A. No. 7941 allowing for a rounding off of fractional seats.

In declaring the two percent threshold unconstitutional, we do not limit our allocation of additional seats in Table 3 below to
the two-percenters. The percentage of votes garnered by each party-list candidate is arrived at by dividing the number of
votes garnered by each party by 15,950,900, the total number of votes cast for party-list candidates. There are two steps
in the second round of seat allocation. First, the percentage is multiplied by the remaining available seats, 38, which is the
difference between the 55 maximum seats reserved under the Party-List System and the 17 guaranteed seats of the two-
percenters. The whole integer of the product of the percentage and of the remaining available seats corresponds to a
party’s share in the remaining available seats. Second, we assign one party-list seat to each of the parties next in rank
until all available seats are completely distributed. We distributed all of the remaining 38 seats in the second round of seat
allocation. Finally, we apply the three-seat cap to determine the number of seats each qualified party-list candidate is
entitled. Thus:

Table 3. Distribution of Available Party-List Seats

Votes
Garnered
Guaranteed Additional (B) plus
over Applying
Seat Seats (C), in
Votes Total the three
Rank Party (First (Second whole
Garnered Votes for seat cap
Round) Round) integers
Party (E)
(B) (C) (D)
List, in %
(A)

1 BUHAY 1,169,234 7.33% 1 2.79 3 N.A.

BAYAN
2 979,039 6.14% 1 2.33 3 N.A.
MUNA

3 CIBAC 755,686 4.74% 1 1.80 2 N.A.

4 GABRIELA 621,171 3.89% 1 1.48 2 N.A.

5 APEC 619,657 3.88% 1 1.48 2 N.A.

6 A Teacher 490,379 3.07% 1 1.17 2 N.A.

7 AKBAYAN 466,112 2.92% 1 1.11 2 N.A.

8 ALAGAD 423,149 2.65% 1 1.01 2 N.A.

COOP-
931 409,883 2.57% 1 1 2 N.A.
NATCCO

10 BUTIL 409,160 2.57% 1 1 2 N.A.

11 BATAS 385,810 2.42% 1 1 2 N.A.

12 ARC 374,288 2.35% 1 1 2 N.A.

13 ANAKPAWIS 370,261 2.32% 1 1 2 N.A.

14 ABONO 339,990 2.13% 1 1 2 N.A.

15 AMIN 338,185 2.12% 1 1 2 N.A.


16 AGAP 328,724 2.06% 1 1 2 N.A.

17 AN WARAY 321,503 2.02% 1 1 2 N.A.

18 YACAP 310,889 1.95% 0 1 1 N.A.

19 FPJPM 300,923 1.89% 0 1 1 N.A.

20 UNI-MAD 245,382 1.54% 0 1 1 N.A.

21 ABS 235,086 1.47% 0 1 1 N.A.

22 KAKUSA 228,999 1.44% 0 1 1 N.A.

23 KABATAAN 228,637 1.43% 0 1 1 N.A.

24 ABA-AKO 218,818 1.37% 0 1 1 N.A.

25 ALIF 217,822 1.37% 0 1 1 N.A.

SENIOR
26 213,058 1.34% 0 1 1 N.A.
CITIZENS

27 AT 197,872 1.24% 0 1 1 N.A.

28 VFP 196,266 1.23% 0 1 1 N.A.

29 ANAD 188,521 1.18% 0 1 1 N.A.

30 BANAT 177,028 1.11% 0 1 1 N.A.

ANG
31 170,531 1.07% 0 1 1 N.A.
KASANGGA

32 BANTAY 169,801 1.06% 0 1 1 N.A.

33 ABAKADA 166,747 1.05% 0 1 1 N.A.

34 1-UTAK 164,980 1.03% 0 1 1 N.A.

35 TUCP 162,647 1.02% 0 1 1 N.A.

36 COCOFED 155,920 0.98% 0 1 1 N.A.

Total 17 55

Applying the procedure of seat allocation as illustrated in Table 3 above, there are 55 party-list representatives from the 36
winning party-list organizations. All 55 available party-list seats are filled. The additional seats allocated to the parties with
sufficient number of votes for one whole seat, in no case to exceed a total of three seats for each party, are shown in
column (D).

Participation of Major Political Parties in Party-List Elections

The Constitutional Commission adopted a multi-party system that allowed all political parties to participate in the
party-list elections. The deliberations of the Constitutional Commission clearly bear this out, thus:

MR. MONSOD. Madam President, I just want to say that we suggested or proposed the party list system because we
wanted to open up the political system to a pluralistic society through a multiparty system. x x x We are for opening up
the system, and we would like very much for the sectors to be there. That is why one of the ways to do that is to
put a ceiling on the number of representatives from any single party that can sit within the 50 allocated under the
party list system. x x x.

xxx

MR. MONSOD. Madam President, the candidacy for the 198 seats is not limited to political parties. My question is this:
Are we going to classify for example Christian Democrats and Social Democrats as political parties? Can they run under
the party list concept or must they be under the district legislation side of it only?

MR. VILLACORTA. In reply to that query, I think these parties that the Commissioner mentioned can field candidates for
the Senate as well as for the House of Representatives. Likewise, they can also field sectoral candidates for the 20
percent or 30 percent, whichever is adopted, of the seats that we are allocating under the party list system.

MR. MONSOD. In other words, the Christian Democrats can field district candidates and can also participate in the party
list system?
MR. VILLACORTA. Why not? When they come to the party list system, they will be fielding only sectoral
candidates.

MR. MONSOD. May I be clarified on that? Can UNIDO participate in the party list system?

MR. VILLACORTA. Yes, why not? For as long as they field candidates who come from the different marginalized
sectors that we shall designate in this Constitution.

MR. MONSOD. Suppose Senator Tañada wants to run under BAYAN group and says that he represents the farmers,
would he qualify?

MR. VILLACORTA. No, Senator Tañada would not qualify.

MR. MONSOD. But UNIDO can field candidates under the party list system and say Juan dela Cruz is a farmer. Who
would pass on whether he is a farmer or not?

MR. TADEO. Kay Commissioner Monsod, gusto ko lamang linawin ito. Political parties, particularly minority political
parties, are not prohibited to participate in the party list election if they can prove that they are also organized
along sectoral lines.

MR. MONSOD. What the Commissioner is saying is that all political parties can participate because it is precisely the
contention of political parties that they represent the broad base of citizens and that all sectors are represented in them.
Would the Commissioner agree?

MR. TADEO. Ang punto lamang namin, pag pinayagan mo ang UNIDO na isang political party, it will dominate the party
list at mawawalang saysay din yung sector. Lalamunin mismo ng political parties ang party list system. Gusto ko lamang
bigyan ng diin ang "reserve." Hindi ito reserve seat sa marginalized sectors. Kung titingnan natin itong 198 seats, reserved
din ito sa political parties.

MR. MONSOD. Hindi po reserved iyon kasi anybody can run there. But my question to Commissioner Villacorta and
probably also to Commissioner Tadeo is that under this system, would UNIDO be banned from running under the party list
system?

MR. VILLACORTA. No, as I said, UNIDO may field sectoral candidates. On that condition alone, UNIDO may be
allowed to register for the party list system.

MR. MONSOD. May I inquire from Commissioner Tadeo if he shares that answer?

MR. TADEO. The same.

MR. VILLACORTA. Puwede po ang UNIDO, pero sa sectoral lines.

xxxx

MR. OPLE. x x x In my opinion, this will also create the stimulus for political parties and mass organizations to seek
common ground. For example, we have the PDP-Laban and the UNIDO. I see no reason why they should not be able to
make common goals with mass organizations so that the very leadership of these parties can be transformed through the
participation of mass organizations. And if this is true of the administration parties, this will be true of others like the Partido
ng Bayan which is now being formed. There is no question that they will be attractive to many mass organizations. In the
opposition parties to which we belong, there will be a stimulus for us to contact mass organizations so that with their
participation, the policies of such parties can be radically transformed because this amendment will create conditions that
will challenge both the mass organizations and the political parties to come together. And the party list system is certainly
available, although it is open to all the parties. It is understood that the parties will enter in the roll of the COMELEC the
names of representatives of mass organizations affiliated with them. So that we may, in time, develop this excellent
system that they have in Europe where labor organizations and cooperatives, for example, distribute themselves either in
the Social Democratic Party and the Christian Democratic Party in Germany, and their very presence there has a
transforming effect upon the philosophies and the leadership of those parties.

It is also a fact well known to all that in the United States, the AFL-CIO always vote with the Democratic Party. But the
businessmen, most of them, always vote with the Republican Party, meaning that there is no reason at all why political
parties and mass organizations should not combine, reenforce, influence and interact with each other so that the very
objectives that we set in this Constitution for sectoral representation are achieved in a wider, more lasting, and more
institutionalized way. Therefore, I support this [Monsod-Villacorta] amendment. It installs sectoral representation as a
constitutional gift, but at the same time, it challenges the sector to rise to the majesty of being elected representatives later
on through a party list system; and even beyond that, to become actual political parties capable of contesting political
power in the wider constitutional arena for major political parties.

x x x 32 (Emphasis supplied)
R.A. No. 7941 provided the details for the concepts put forward by the Constitutional Commission. Section 3 of R.A. No.
7941 reads:

Definition of Terms. (a) The party-list system is a mechanism of proportional representation in the election of
representatives to the House of Representatives from national, regional and sectoral parties or organizations or coalitions
thereof registered with the Commission on Elections (COMELEC). Component parties or organizations of a coalition may
participate independently provided the coalition of which they form part does not participate in the party-list system.

(b) A party means either a political party or a sectoral party or a coalition of parties.

(c) A political party refers to an organized group of citizens advocating an ideology or platform, principles and
policies for the general conduct of government and which, as the most immediate means of securing their
adoption, regularly nominates and supports certain of its leaders and members as candidates for public office.

It is a national party when its constituency is spread over the geographical territory of at least a majority of the
regions. It is a regional party when its constituency is spread over the geographical territory of at least a majority of
the cities and provinces comprising the region.

(d) A sectoral party refers to an organized group of citizens belonging to any of the sectors enumerated in Section
5 hereof whose principal advocacy pertains to the special interests and concerns of their sector,

(e) A sectoral organization refers to a group of citizens or a coalition of groups of citizens who share similar
physical attributes or characteristics, employment, interests or concerns.

(f) A coalition refers to an aggrupation of duly registered national, regional, sectoral parties or organizations for
political and/or election purposes.

Congress, in enacting R.A. No. 7941, put the three-seat cap to prevent any party from dominating the party-list elections.

Neither the Constitution nor R.A. No. 7941 prohibits major political parties from participating in the party-list system. On
the contrary, the framers of the Constitution clearly intended the major political parties to participate in party-list elections
through their sectoral wings. In fact, the members of the Constitutional Commission voted down, 19-22, any permanent
sectoral seats, and in the alternative the reservation of the party-list system to the sectoral groups. 33 In defining a "party"
that participates in party-list elections as either "a political party or a sectoral party," R.A. No. 7941 also clearly intended
that major political parties will participate in the party-list elections. Excluding the major political parties in party-list
elections is manifestly against the Constitution, the intent of the Constitutional Commission, and R.A. No. 7941. This Court
cannot engage in socio-political engineering and judicially legislate the exclusion of major political parties from the party-
list elections in patent violation of the Constitution and the law.

Read together, R.A. No. 7941 and the deliberations of the Constitutional Commission state that major political parties are
allowed to establish, or form coalitions with, sectoral organizations for electoral or political purposes. There should not be
a problem if, for example, the Liberal Party participates in the party-list election through the Kabataang Liberal ng Pilipinas
(KALIPI), its sectoral youth wing. The other major political parties can thus organize, or affiliate with, their chosen sector or
sectors. To further illustrate, the Nacionalista Party can establish a fisherfolk wing to participate in the party-list election,
and this fisherfolk wing can field its fisherfolk nominees. Kabalikat ng Malayang Pilipino (KAMPI) can do the same for the
urban poor.

The qualifications of party-list nominees are prescribed in Section 9 of R.A. No. 7941:

Qualifications of Party-List Nominees. — No person shall be nominated as party-list representative unless he is a natural
born citizen of the Philippines, a registered voter, a resident of the Philippines for a period of not less than one (1) year
immediately preceding the day of the elections, able to read and write, bona fide member of the party or organization
which he seeks to represent for at least ninety (90) days preceding the day of the election, and is at least twenty-five (25)
years of age on the day of the election.

In case of a nominee of the youth sector, he must at least be twenty-five (25) but not more than thirty (30) years of age on
the day of the election. Any youth sectoral representative who attains the age of thirty (30) during his term shall be allowed
to continue until the expiration of his term.

Under Section 9 of R.A. No. 7941, it is not necessary that the party-list organization’s nominee "wallow in poverty,
destitution and infirmity"34 as there is no financial status required in the law. It is enough that the nominee of the sectoral
party/organization/coalition belongs to the marginalized and underrepresented sectors, 35 that is, if the nominee represents
the fisherfolk, he or she must be a fisherfolk, or if the nominee represents the senior citizens, he or she must be a senior
citizen.

Neither the Constitution nor R.A. No. 7941 mandates the filling-up of the entire 20% allocation of party-list representatives
found in the Constitution. The Constitution, in paragraph 1, Section 5 of Article VI, left the determination of the number of
the members of the House of Representatives to Congress: "The House of Representatives shall be composed of not
more than two hundred and fifty members, unless otherwise fixed by law, x x x." The 20% allocation of party-list
representatives is merely a ceiling; party-list representatives cannot be more than 20% of the members of the House of
Representatives. However, we cannot allow the continued existence of a provision in the law which will systematically
prevent the constitutionally allocated 20% party-list representatives from being filled. The three-seat cap, as a limitation to
the number of seats that a qualified party-list organization may occupy, remains a valid statutory device that prevents any
party from dominating the party-list elections. Seats for party-list representatives shall thus be allocated in accordance with
the procedure used in Table 3 above.

However, by a vote of 8-7, the Court decided to continue the ruling in Veterans disallowing major political parties from
participating in the party-list elections, directly or indirectly. Those who voted to continue disallowing major political parties
from the party-list elections joined Chief Justice Reynato S. Puno in his separate opinion. On the formula to allocate party-
list seats, the Court is unanimous in concurring with this ponencia.

WHEREFORE, we PARTIALLY GRANT the petition. We SET ASIDE the Resolution of the COMELEC dated 3 August
2007 in NBC No. 07-041 (PL) as well as the Resolution dated 9 July 2007 in NBC No. 07-60. We declare unconstitutional
the two percent threshold in the distribution of additional party-list seats. The allocation of additional seats under the Party-
List System shall be in accordance with the procedure used in Table 3 of this Decision. Major political parties are
disallowed from participating in party-list elections. This Decision is immediately executory. No pronouncement as to costs.

SO ORDERED.

- Atong Paglaum, Inc. v Comelec, 694 SCRA 477 (2013)

The Cases

These cases constitute 54 Petitions for Certiorari and Petitions for Certiorari and Prohibition1 filed by 52 party-list groups
and organizations assailing the Resolutions issued by the Commission on Elections (COMELEC) disqualifying them from
participating in the 13 May 2013 party-list elections, either by denial of their petitions for registration under the party-list
system, or cancellation of their registration and accreditation as party-list organizations.

This Court resolved to consolidate the 54 petitions in the Resolutions dated 13 November 2012, 2 20 November 2012,3 27
November 2012,4 4 December 2012,5 11 December 2012,6 and 19 February 2013.7

The Facts Pursuant to the provisions of Republic Act No. 7941 (R.A. No. 7941) and COMELEC Resolution Nos. 9366 and
9531, approximately 280 groups and organizations registered and manifested their desire to participate in the 13 May
2013 party-list elections.

  G.R. SPP No. Group Grounds for Denial


No.
A. Via the COMELEC En Banc’s automatic review of the COMELEC
Division’s resolutions approving registration of groups/organizations
Resolution dated 23 November 20128
1 204379 12-099 Alagad ng - The "artists" sector is not
(PLM) Sining (ASIN) considered marginalized and
underrepresented;
- Failure to prove track
record; and
- Failure of the nominees to
qualify under RA 7941 and
Ang Bagong Bayani.
Omnibus Resolution dated 27 November 20129
2 204455 12-041 Manila Teachers - A non-stock savings and
(PLM) Savings and loan association cannot be
Loan considered marginalized and
Association, Inc. underrepresented; and
(Manila - The first and second
Teachers) nominees are not teachers by
profession.
3 204426 12-011 Association of - Failure to show that its
(PLM) Local Athletics members belong to the
Entrepreneurs marginalized; and
and Hobbyists, - Failure of the nominees to
Inc. (ALA-EH) qualify.
Resolution dated 27 November 201210
4 204435 12-057 1 Alliance - Failure of the nominees to
(PLM) Advocating qualify: although registering
Autonomy Party as a regional political party,
(1AAAP) two of the nominees are not
residents of the region; and
four of the five nominees do
not belong to the
marginalized and underrepresented.
Resolution dated 27 November 201211
5 204367 12-104 (PL) Akbay - Failure of the group to show
Kalusugan that its nominees belong to
(AKIN), Inc. the urban poor sector.
Resolution dated 29 November 201212
6 204370 12-011 (PP) Ako An Bisaya - Failure to represent a
(AAB) marginalized sector of
society, despite the formation
of a sectoral wing for the
benefit of farmers of Region
8;
- Constituency has district
representatives;
- Lack of track record in
representing peasants and
farmers; and
- Nominees are neither
farmers nor peasants.
Resolution dated 4 December 201213
7 204436 12-009 Abyan Ilonggo - Failure to show that the
(PP), Party (AI) party represents a
12-165 marginalized and
(PLM) underrepresented sector, as
the Province of Iloilo has
district representatives;
- Untruthful statements in the
memorandum; and
- Withdrawal of three of its
five nominees.
Resolution dated 4 December 201214
8 204485 12-175 (PL) Alliance of - Failure to establish that the
Organizations, group can represent 14
Networks and Associations sectors; - The sectors of
of homeowners’
the Philippines, associations, entrepreneurs
Inc. (ALONA) and cooperatives are not
marginalized and
underrepresented; and
- The nominees do not belong
to the marginalized and
underrepresented.
B. Via the COMELEC En Banc’s review on motion for reconsideration
of the COMELEC Division’s resolutions denying registration of groups
and organizations
Resolution dated 7 November 201215
9 204139 12-127 (PL) Alab ng - Failure to prove track
Mamamahayag record as an organization;
(ALAM) - Failure to show that the
group actually represents the
marginalized and
underrepresented; and
- Failure to establish that the
group can represent all
sectors it seeks to represent.
Resolution dated 7 November 201216
10 204402 12-061 (PP) Kalikasan Party-List - The group reflects an
(KALIKASAN) advocacy for the
environment, and is not
representative of the
marginalized and
underrepresented;
- There is no proof that
majority of its members
belong to the marginalized
and underrepresented;
- The group represents
sectors with conflicting
interests; and
- The nominees do not belong
to the sector which the group
claims to represent.
Resolution dated 14 November 201217
11 204394 12-145 (PL) Association of - Failure to prove
Guard, Utility membership base and track
Helper, Aider, record;
Rider, Driver/ - Failure to present activities
Domestic that sufficiently benefited its
Helper, intended constituency; and
Janitor, Agent - The nominees do not belong
and to any of the sectors which
Nanny of the the group seeks to represent.
Philippines, Inc.
(GUARDJAN)
Resolution dated 5 December 201218
12 204490 12-073 Pilipinas Para sa - Failure to show that the
(PLM) Pinoy (PPP) group represents a
marginalized and
underrepresented sector, as
Region 12 has district
representatives; and
- Failure to show a track
record of undertaking
programs for the welfare of
the sector the group seeks to
represent.

In a Resolution dated 5 December 2012,19 the COMELEC En Banc affirmed the COMELEC Second Division’s resolution to
grant Partido ng Bayan ng Bida’s (PBB) registration and accreditation as a political party in the National Capital Region.
However, PBB was denied participation in the 13 May 2013 party-list elections because PBB does not represent any
"marginalized and underrepresented" sector; PBB failed to apply for registration as a party-list group; and PBB failed to
establish its track record as an organization that seeks to uplift the lives of the "marginalized and underrepresented." 20
These 13 petitioners (ASIN, Manila Teachers, ALA-EH, 1AAAP, AKIN, AAB, AI, ALONA, ALAM, KALIKASAN,
GUARDJAN, PPP, and PBB) were not able to secure a mandatory injunction from this Court. The COMELEC, on 7
January 2013 issued Resolution No. 9604, 21 and excluded the names of these 13 petitioners in the printing of the official
ballot for the 13 May 2013 party-list elections.
Pursuant to paragraph 222 of Resolution No. 9513, the COMELEC En Banc scheduled summary evidentiary hearings to
determine whether the groups and organizations that filed manifestations of intent to participate in the 13 May 2013 party-
list elections have continually complied with the requirements of R.A. No. 7941 and Ang Bagong Bayani-OFW Labor Party
v. COMELEC23 (Ang Bagong Bayani). The COMELEC disqualified the following groups and organizations from
participating in the 13 May 2013 party-list elections:

  G.R. No. SPP Group Grounds for Denial


No.
Resolution dated 10 October 201224
1 203818- 12-154 AKO Bicol Retained registration and
19 (PLM) Political Party accreditation as a political
12-177 (AKB) party, but denied participation
(PLM) in the May 2013 party-list
elections
- Failure to represent any
marginalized and
underrepresented sector;
- The Bicol region already
has representatives in
Congress; and
- The nominees are not
marginalized and
underrepresented.
Omnibus Resolution dated 11 October 201225
2 203766 12-161 Atong Paglaum, Cancelled registration and
(PLM) Inc. (Atong accreditation
Paglaum) - The nominees do not belong
to the sectors which the party
represents; and
- The party failed to file its
Statement of Contributions
and Expenditures for the
2010 Elections.
3 203981 12-187 Association for Cancelled registration and
(PLM) Righteousness accreditation
Advocacy on - Failure to comply, and for
Leadership violation of election laws;
(ARAL) - The nominees do not
represent the sectors which
the party represents; and
- There is doubt that the party
is organized for religious
purposes.
4 204002 12-188 Alliance for Cancelled registration and
(PLM) Rural Concerns accreditation
(ARC) - Failure of the nominees to
qualify; and
- Failure of the party to prove
that majority of its members
belong to the sectors it seeks
to represent.
5 204318 12-220 United Cancelled registration and
(PLM) Movement accreditation
Against Drugs - The sectors of drug
Foundation counsellors and lecturers,
(UNIMAD) veterans and the youth, are
not marginalized and
underrepresented;
- Failure to establish track
record; and
- Failure of the nominees to
qualify as representatives of
the youth and young urban
professionals.
Omnibus Resolution dated 16 October 201226
6 204100 12-196 1-Bro Philippine Cancelled registration
(PLM) Guardians - Failure to define the sector
Brotherhood, it seeks to represent; and
Inc. (1BRO-PGBI) - The nominees do not belong
to a marginalized and
underrepresented sector.
7 204122 12-223 1 Guardians Cancelled registration
(PLM) Nationalist - The party is a military
Philippines, Inc. fraternity;
(1GANAP/ - The sector of community
GUARDIANS) volunteer workers is too
broad to allow for meaningful
representation; and
- The nominees do not appear
to belong to the sector of
community volunteer
workers.
8 20426 12-257 Blessed Cancelled registration
(PLM) Federation of - Three of the seven
Farmers and nominees do not belong to
Fishermen the sector of farmers and
International, fishermen, the sector sought
Inc. (A to be represented; and
BLESSED - None of the nominees are
Party-List) registered voters of Region
XI, the region sought to be
represented.
Resolution dated 16 October 201227
9 203960 12-260 1st Cancelled registration
(PLM) Consumers - The sector of rural energy
Alliance for consumers is not
Rural Energy, marginalized and
Inc. (1-CARE) underrepresented;
- The party’s track record is
related to electric
cooperatives and not rural
energy consumers; and
- The nominees do not belong
to the sector of rural energy
consumers.
Resolution dated 16 October 201228
10 203922 12-201 Association of Cancelled registration and
(PLM) Philippine accreditation
Electric - Failure to represent a
Cooperatives marginalized and
(APEC) underrepresented sector; and
- The nominees do not belong
to the sector that the party
claims to represent.
Resolution dated 23 October 201229
11 204174 12-232 Aangat Tayo Cancelled registration and
(PLM) Party-List Party accreditation
( AT ) - The incumbent
representative in Congress
failed to author or sponsor
bills that are beneficial to the
sectors that the party
represents (women, elderly,
youth, urban poor); and
- The nominees do not belong
to the marginalized sectors
that the party seeks to
represent.
Omnibus Resolution dated 24 October 201230
12 203976 12-288 Alliance for Cancelled registration and
(PLM) Rural and accreditation
Agrarian - The interests of the peasant
Reconstruction, and urban poor sectors that
Inc. (ARARO) the party represents differ;
- The nominees do not belong
to the sectors that the party
seeks to represent;
- Failure to show that three of
the nominees are bona fide
party members; and
- Lack of a Board resolution
to participate in the party-list
elections.
Omnibus Resolution dated 24 October 201231
13 204240 12-279 Agri-Agra na Cancelled registration
(PLM) Reporma Para sa - The party ceased to exist for
Magsasaka ng more than a year immediately
Pilipinas after the May 2010 elections;
Movement - The nominees do not belong
(AGRI) to the sector of peasants and
farmers that the party seeks to
represent;
- Only four nominees were
submitted to the COMELEC;
and
- Failure to show meaningful
activities for its constituency.
14 203936 12-248 Aksyon Cancelled registration
(PLM) Magsasaka-Partido - Failure to show that
Tinig ng majority of its members are
Masa (AKMA-PTM) marginalized and
underrepresented;
- Failure to prove that four of
its nine nominees actually
belong to the farmers sector;
and
- Failure to show that five of
its nine nominees work on
uplifting the lives of the
members of the sector.
15 204126 12-263 Kaagapay ng Cancelled registration
(PLM) Nagkakaisang - The Manifestation of Intent
Agilang and Certificate of Nomination
Pilipinong were not signed by an
Magsasaka appropriate officer of the
(KAP) party;
- Failure to show track record
for the farmers and peasants
sector; and
- Failure to show that
nominees actually belong to
the sector, or that they have
undertaken meaningful
activities for the sector.
16 204364 12-180 Adhikain at Cancelled registration
(PLM) Kilusan ng - Failure to show that
Ordinaryong nominees actually belong to
Tao Para sa the sector, or that they have
Lupa, Pabahay, undertaken meaningful
Hanapbuhay at activities for the sector.
Kaunlaran
(AKO-BAHAY)
17 204141 12-229 The True Cancelled registration
(PLM) Marcos Loyalist - Failure to show that
(for God, majority of its members are
Country and marginalized and
People) underrepresented; and
Association of - Failure to prove that two of
the Philippines, its nominees actually belong
Inc. (BANTAY) to the marginalized and
underrepresented.
18 204408 12-217 Pilipino Cancelled registration
(PLM) Association for - Change of sector (from
Country – Urban urban poor youth to urban
Poor Youth poor) necessitates a new
Advancement application;
and Welfare - Failure to show track record
( PA C YAW ) for the marginalized and
underrepresented;
- Failure to prove that
majority of its members and
officers are from the urban
poor sector; and
- The nominees are not
members of the urban poor
sector.
19 204153 12-277 Pasang Masda Cancelled registration
(PLM) Nationwide - The party represents drivers
Party (PASANG and operators, who may have
MASDA) conflicting interests; and
- Nominees are either
operators or former operators.
20 203958 12-015 Kapatiran ng Cancelled registration
(PLM) mga Nakulong - Failure to prove that
na Walang Sala, na Walang Sala,
Inc. (KAKUSA) Inc. (KAKUSA)
majority of its officers and
members belong to the
marginalized and
underrepresented;
- The incumbent
representative in Congress
failed to author or sponsor
bills that are beneficial to the
sector that the party
represents (persons
imprisoned without proof of
guilt beyond reasonable
doubt);
- Failure to show track record
for the marginalized and
underrepresented; and
- The nominees did not
appear to be marginalized and
underrepresented.
Resolution dated 30 October 201232
21 204428 12-256 Ang Galing Cancelled registration and
(PLM) Pinoy (AG) accreditation
- Failure to attend the
summary hearing;
- Failure to show track record
for the marginalized and
underrepresented; and
- The nominees did not
appear to be marginalized and
underrepresented.
Resolution dated 7 November 201233
22 204094 12-185 Alliance for Cancelled registration and
(PLM) Nationalism and accreditation
Democracy - Failure to represent an
(ANAD) identifiable marginalized and
underrepresented sector;
- Only three nominees were
submitted to the COMELEC;
- The nominees do not
belong to the marginalized
and underrepresented; and
- Failure to submit its
Statement of Contribution
and Expenditures for the
2007 Elections.
Omnibus Resolution dated 7 November 201234
23 204239 12-060 Green Force for Cancelled registration and
(PLM) the Environment accreditation
Sons and - The party is an advocacy
Daughters of group and does not represent
Mother Earth the marginalized and
(GREENFORCE) underrepresented;
- Failure to comply with the
track record requirement; and
- The nominees are not
marginalized citizens.
24 204236 12-254 Firm 24-K Cancelled registration and
(PLM) Association, Inc. accreditation
(FIRM 24-K) - The nominees do not
belong to the sector that the
party seeks to represent
(urban poor and peasants of
the National Capital Region);
- Only two of its nominees
reside in the National Capital
Region; and
- Failure to comply with the
track record requirement.
25 204341 12-269 Action League Cancelled registration and
(PLM) of Indigenous accreditation
Masses (ALIM) - Failure to establish that its
nominees are members of the
indigenous people in the
Mindanao and Cordilleras
sector that the party seeks to
represent;
- Only two of the party’s
nominees reside in the
Mindanao and Cordilleras;
and
- Three of the nominees do
not appear to belong to the
marginalized.
Resolution dated 7 November 201235
26 204358 12-204 Alliance of Cancelled registration
(PLM) Advocates in - The sector it represents is a
Mining specifically defined group
Advancement which may not be allowed
for National registration under the party-list system; and
Progress - Failure to establish that the
(AAMA) nominees actually belong to
the sector.
Resolution dated 7 November 201236
27 204359 12-272 Social Cancelled registration
(PLM) Movement for - The nominees are
Active Reform disqualified from
and representing the sectors that
Transparency the party represents;
(SMART) - Failure to comply with the
track record requirement; and
- There is doubt as to whether
majority of its members are
marginalized and
underrepresented.
Resolution dated 7 November 201237
28 204238 12-173 Alliance of Cancelled registration and
(PLM) Bicolnon Party accreditation
(ABP) - Defective registration and
accreditation dating back to
2010;
- Failure to represent any
sector; and
- Failure to establish that the
nominees are employed in the construction
industry, the
sector it claims to represent.
Resolution dated 7 November 201238
29 204323 12-210 Bayani Party Cancelled registration and
(PLM) List (BAYANI) accreditation
- Failure to prove a track
record of trying to uplift the
marginalized and
underrepresented sector of
professionals; and
- One nominee was declared
unqualified to represent the
sector of professionals.
Resolution dated 7 November 201239
30 204321 12-252 Ang Agrikultura Cancelled registration and
(PLM) Natin Isulong accreditation
(AANI) - Failure to establish a track
record of enhancing the lives
of the marginalized and
underrepresented farmers
which it claims to represent;
and
- More than a majority of the
party’s nominees do not
belong to the farmers sector.
Resolution dated 7 November 201240
31 204125 12-292 Agapay ng Cancelled registration and
(PLM) Indigenous accreditation
Peoples Rights - Failure to prove that its five
Alliance, Inc. nominees are members of the
(A-IPRA) indigenous people sector;
- Failure to prove that its five
nominees actively
participated in the
undertakings of the party; and
- Failure to prove that its five nominees are
bona fide
members.
Resolution dated 7 November 201241
32 204216 12-202 Philippine Cancelled registration and
(PLM) Coconut accreditation
Producers - The party is affiliated with
Federation, Inc. private and government
(COCOFED) agencies and is not
marginalized;
- The party is assisted by the
government in various
projects; and
- The nominees are not
members of the marginalized
sector of coconut farmers and
producers.
Resolution dated 7 November 201242
33 204220 12-238 Abang Lingkod Cancelled registration
(PLM) Party-List - Failure to establish a track
(ABANG record of continuously
LINGKOD) representing the peasant
farmers sector;
- Failure to show that its
members actually belong to
the peasant farmers sector;
and
- Failure to show that its
nominees are marginalized
and underrepresented, have
actively participated in
programs for the
advancement of farmers, and
adhere to its advocacies.
Resolution dated 14 November 201243
34 204158 12-158 Action Cancelled registration and
(PLM) Brotherhood for Active accreditation - Failure to show that the
Dreamers, Inc. party is actually able to
(ABROAD) represent all of the sectors it
claims to represent;
- Failure to show a complete
track record of its activities
since its registration; and
- The nominees are not part
of any of the sectors which
the party seeks to represent.
Resolution dated 28 November 201244
35 204374 12-228 Binhi-Partido ng Cancelled registration and
(PLM) mga Magsasaka accreditation
Para sa mga - The party receives
Magsasaka assistance from the
(BINHI) government through the
Department of Agriculture;
and
- Failure to prove that the
group is marginalized and
underrepresented.
Resolution dated 28 November 201245
36 204356 12-136 Butil Farmers Cancelled registration and
(PLM) Party (BUTIL) accreditation
- Failure to establish that the
agriculture and cooperative
sectors are marginalized and
underrepresented; and
- The party’s nominees
neither appear to belong to
the sectors they seek to
represent, nor to have
actively participated in the
undertakings of the party.
Resolution dated 3 December 201246
37 204486 12-194 1st Cancelled registration and
(PLM) Kabalikat ng accreditation
Bayan - Declaration of untruthful
Ginhawang statements;
Sangkatauhan - Failure to exist for at least
(1st one year; and
KABAGIS) - None of its nominees
belong to the labor,
fisherfolk, and urban poor
indigenous cultural
communities sectors which it
seeks to represent.
Resolution dated 4 December 201247
38 204410 12-198 1-United Cancelled accreditation
(PLM) Transport - The party represents drivers
Koalisyon (1-UTAK) and operators, who may have
conflicting interests; and
- The party’s nominees do not
belong to any marginalized
and underrepresented sector.
Resolution dated 4 December 201248
39 204421, 12-157 Coalition of Cancelled registration
204425 (PLM), Senior Citizens - The party violated election
12-191 in the laws because its nominees
(PLM) Philippines, Inc. had a term-sharing
(SENIOR agreement.
CITIZENS)

These 39 petitioners (AKB, Atong Paglaum, ARAL, ARC, UNIMAD, 1BRO-PGBI, 1GANAP/GUARDIANS, A
BLESSED Party-List, 1-CARE, APEC, AT, ARARO, AGRI, AKMA-PTM, KAP, AKO-BAHAY, BANTAY,
PACYAW, PASANG MASDA, KAKUSA, AG, ANAD, GREENFORCE, FIRM 24-K, ALIM, AAMA, SMART,
ABP, BAYANI, AANI, A-IPRA, COCOFED, ABANG LINGKOD, ABROAD, BINHI, BUTIL, 1st KABAGIS, 1-
UTAK, SENIOR CITIZENS) were able to secure a mandatory injunction from this Court, directing the COMELEC to
include the names of these 39 petitioners in the printing of the official ballot for the 13 May 2013 party-list elections.
Petitioners prayed for the issuance of a temporary restraining order and/or writ of preliminary injunction. This Court
issued Status Quo Ante Orders in all petitions. This Decision governs only the 54 consolidated petitions that were
granted Status Quo Ante Orders, namely:

G.R. No. SPP No. Group


Resolution dated 13 November 2012
203818-19 12-154 AKO Bicol Political Party (AKB)
(PLM)
12-177
(PLM)
203981 12-187 Association for Righteousness Advocacy on
(PLM) Leadership (ARAL)
204002 12-188 Alliance for Rural Concerns (ARC)
(PLM)
203922 12-201 Association of Philippine Electric Cooperatives
(PLM) (APEC)
203960 12-260 1st
(PLM) Consumers Alliance for Rural Energy, Inc.
(1-CARE)
203936 12-248 Aksyon Magsasaka-Partido Tinig ng Masa
(PLM) (AKMA-PTM)
203958 12-015 Kapatiran ng mga Nakulong na Walang Sala,
(PLM) Inc. (KAKUSA)
203976 12-288 Alliance for Rural and Agrarian Reconstruction,
(PLM) Inc. (ARARO)
Resolution dated 20 November 2012
204094 12-185 Alliance for Nationalism and Democracy
(PLM) (ANAD)
204125 12-292 Agapay ng Indigenous Peoples Rights Alliance,
(PLM) Inc. (A-IPRA)
204100 12-196 1-Bro Philippine Guardians Brotherhood, Inc.
(PLM) (1BRO-PGBI)
Resolution dated 27 November 2012
204141 12-229 The True Marcos Loyalist (for God, Country
(PLM) and People) Association of the Philippines, Inc.
(BANTAY)
204240 12-279 Agri-Agra na Reporma Para sa Magsasaka ng
(PLM) Pilipinas Movement (AGRI)
204216 12-202 Philippine Coconut Producers Federation, Inc.
(PLM) (COCOFED)
204158 12-158 Action Brotherhood for Active Dreamer, Inc.
(PLM) (ABROAD)
Resolutions dated 4 December 2012
204122 12-223 1 Guardians Nationalist Philippines, Inc.
(PLM) (1GANAP/GUARDIANS)
203766 12-161 Atong Paglaum, Inc. (Atong Paglaum)
(PLM)
204318 12-220 United Movement Against Drugs Foundation
(PLM) (UNIMAD)
204263 12-257 Blessed Federation of Farmers and Fishermen
(PLM) International, Inc. (A BLESSED Party-List)
204174 12-232 Aangat Tayo Party-List Party (AT)
(PLM)
204126 12-263 Kaagapay ng Nagkakaisang Agilang Pilipinong
(PLM) Magsasaka (KAP)
204364 12-180 Adhikain at Kilusan ng Ordinaryong Tao Para sa
(PLM) Lupa, Pabahay, Hanapbuhay at Kaunlaran
(AKO-BAHAY)
204139 12-127 (PL) Alab ng Mamamahayag (ALAM)
204220 12-238 Abang Lingkod Party-List (ABANG
(PLM) LINGKOD)
204236 12-254 Firm 24-K Association, Inc. (FIRM 24-K)
(PLM)
204238 12-173 Alliance of Bicolnon Party (ABP)
(PLM)
204239 12-060 Green Force for the Environment Sons and
(PLM) Daughters of Mother Earth (GREENFORCE)
204321 12-252 Ang Agrikultura Natin Isulong (AANI)
(PLM)
204323 12-210 Bayani Party List (BAYANI)
(PLM)
204341 12-269 Action League of Indigenous Masses (ALIM)
(PLM)
204358 12-204 Alliance of Advocates in Mining Advancement
(PLM) for National Progress (AAMA)
204359 12-272 Social Movement for Active Reform and
(PLM) Transparency (SMART)
204356 12-136 Butil Farmers Party (BUTIL)
(PLM)
Resolution dated 11 December 2012
204402 12-061 (PL) Kalikasan Party-List (KALIKASAN)
204394 12-145 (PL) Association of Guard, Utility Helper, Aider,
Rider, Driver/Domestic Helper, Janitor, Agent
and Nanny of the Philippines, Inc.
(GUARDJAN)
204408 12-217 Pilipino Association for Country – Urban Poor
(PLM) Youth Advancement and Welfare (PACYAW)
204428 12-256 Ang Galing Pinoy (AG)
(PLM)
204490 12-073 Pilipinas Para sa Pinoy (PPP)
(PLM)
204379 12-099 Alagad ng Sining (ASIN)
(PLM)
204367 12-104 (PL) Akbay Kalusugan (AKIN)
204426 12-011 Association of Local Athletics Entrepreneurs
(PLM) and Hobbyists, Inc. (ALA-EH)
204455 12-041 Manila Teachers Savings and Loan Association,
(PLM) Inc. (Manila Teachers)
204374 12-228 Binhi-Partido ng mga Magsasaka Para sa mga
(PLM) Magsasaka (BINHI)
204370 12-011 (PP) Ako An Bisaya (AAB)
204435 12-057 1 Alliance Advocating Autonomy Party
(PLM) (1AAAP)
204486 12-194 1st Kabalikat ng Bayan Ginhawang
(PLM) Sangkatauhan (1st KABAGIS)
204410 12-198 1-United Transport Koalisyon (1-UTAK)
(PLM)
204421, 12-157 Coalition of Senior Citizens in the Philippines,
204425 (PLM) Inc. (SENIOR CITIZENS)
12-191
(PLM)
204436 12-009 (PP), Abyan Ilonggo Party (AI)
12-165
(PLM)
204485 12-175 (PL) Alliance of Organizations, Networks and
Associations of the Philippines, Inc. (ALONA)
204484 11-002 Partido ng Bayan ng Bida (PBB)
Resolution dated 11 December 2012
204153 12-277 Pasang Masda Nationwide Party (PASANG
(PLM) MASDA)
The Issues

We rule upon two issues: first, whether the COMELEC committed grave abuse of discretion amounting to lack or excess
of jurisdiction in disqualifying petitioners from participating in the 13 May 2013 party-list elections, either by denial of their
new petitions for registration under the party-list system, or by cancellation of their existing registration and accreditation
as party-list organizations; and second, whether the criteria for participating in the party-list system laid down in Ang
Bagong Bayani and Barangay Association for National Advancement and Transparency v. Commission on
Elections49 (BANAT) should be applied by the COMELEC in the coming 13 May 2013 party-list elections.

The Court’s Ruling

We hold that the COMELEC did not commit grave abuse of discretion in following prevailing decisions of this Court in
disqualifying petitioners from participating in the coming 13 May 2013 party-list elections. However, since the Court adopts
in this Decision new parameters in the qualification of national, regional, and sectoral parties under the party-list system,
thereby abandoning the rulings in the decisions applied by the COMELEC in disqualifying petitioners, we remand to the
COMELEC all the present petitions for the COMELEC to determine who are qualified to register under the party-list
system, and to participate in the coming 13 May 2013 party-list elections, under the new parameters prescribed in this
Decision.

The Party-List System

The 1987 Constitution provides the basis for the party-list system of representation. Simply put, the party-list system is
intended to democratize political power by giving political parties that cannot win in legislative district elections a chance to
win seats in the House of Representatives.50 The voter elects two representatives in the House of Representatives: one for
his or her legislative district, and another for his or her party-list group or organization of choice. The 1987 Constitution
provides:

Section 5, Article VI

(1) The House of Representatives shall be composed of not more than two hundred and fifty members, unless
otherwise fixed by law, who shall be elected from legislative districts apportioned among the provinces, cities, and
the Metropolitan Manila area in accordance with the number of their respective inhabitants, and on the basis of a
uniform and progressive ratio, and those who, as provided by law, shall be elected through a party-list system of
registered national, regional, and sectoral parties or organizations.

(2) The party-list representatives shall constitute twenty per centum of the total number of representatives
including those under the party list. For three consecutive terms after the ratification of this Constitution, one-half of
the seats allocated to party-list representatives shall be filled, as provided by law, by selection or election from the
labor, peasant, urban poor, indigenous cultural communities, women, youth, and such other sectors as may be
provided by law, except the religious sector.

Sections 7 and 8, Article IX-C

Sec. 7. No votes cast in favor of a political party, organization, or coalition shall be valid, except for those registered under
the party-list system as provided in this Constitution.

Sec. 8. Political parties, or organizations or coalitions registered under the party-list system, shall not be represented in the
voters’ registration boards, boards of election inspectors, boards of canvassers, or other similar bodies. However, they
shall be entitled to appoint poll watchers in accordance with law.

Commissioner Christian S. Monsod, the main sponsor of the party-list system, stressed that "the party-list system is not
synonymous with that of the sectoral representation."51 The constitutional provisions on the party-list system should be
read in light of the following discussion among its framers:

MR. MONSOD: x x x.

I would like to make a distinction from the beginning that the proposal for the party list system is not synonymous with that
of the sectoral representation. Precisely, the party list system seeks to avoid the dilemma of choice of sectors and who
constitute the members of the sectors. In making the proposal on the party list system, we were made aware of the
problems precisely cited by Commissioner Bacani of which sectors will have reserved seats. In effect, a sectoral
representation in the Assembly would mean that certain sectors would have reserved seats; that they will choose among
themselves who would sit in those reserved seats. And then, we have the problem of which sector because as we will
notice in Proclamation No. 9, the sectors cited were the farmers, fishermen, workers, students, professionals, business,
military, academic, ethnic and other similar groups. So these are the nine sectors that were identified here as "sectoral
representatives" to be represented in this Commission. The problem we had in trying to approach sectoral representation
in the Assembly was whether to stop at these nine sectors or include other sectors. And we went through the exercise in a
caucus of which sector should be included which went up to 14 sectors. And as we all know, the longer we make our
enumeration, the more limiting the law become because when we make an enumeration we exclude those who are not in
the enumeration. Second, we had the problem of who comprise the farmers. Let us just say the farmers and the laborers.
These days, there are many citizens who are called "hyphenated citizens." A doctor may be a farmer; a lawyer may also
be a farmer. And so, it is up to the discretion of the person to say "I am a farmer" so he would be included in that sector.
The third problem is that when we go into a reserved seat system of sectoral representation in the Assembly, we are, in
effect, giving some people two votes and other people one vote. We sought to avoid these problems by presenting a party
list system. Under the party list system, there are no reserved seats for sectors. Let us say, laborers and farmers can form
a sectoral party or a sectoral organization that will then register and present candidates of their party. How do the
mechanics go? Essentially, under the party list system, every voter has two votes, so there is no discrimination. First, he
will vote for the representative of his legislative district. That is one vote. In that same ballot, he will be asked: What party
or organization or coalition do you wish to be represented in the Assembly? And here will be attached a list of the parties,
organizations or coalitions that have been registered with the COMELEC and are entitled to be put in that list. This can be
a regional party, a sectoral party, a national party, UNIDO, Magsasaka or a regional party in Mindanao. One need not be a
farmer to say that he wants the farmers' party to be represented in the Assembly. Any citizen can vote for any party. At the
end of the day, the COMELEC will then tabulate the votes that had been garnered by each party or each organization —
one does not have to be a political party and register in order to participate as a party — and count the votes and from
there derive the percentage of the votes that had been cast in favor of a party, organization or coalition.

When such parties register with the COMELEC, we are assuming that 50 of the 250 seats will be for the party list system.
So, we have a limit of 30 percent of 50. That means that the maximum that any party can get out of these 50 seats is 15.
When the parties register they then submit a list of 15 names. They have to submit these names because these nominees
have to meet the minimum qualifications of a Member of the National Assembly. At the end of the day, when the votes are
tabulated, one gets the percentages. Let us say, UNIDO gets 10 percent or 15 percent of the votes; KMU gets 5 percent; a
women’s party gets 2 1/2 percent and anybody who has at least 2 1/2 percent of the vote qualifies and the 50 seats are
apportioned among all of these parties who get at least 2 1/2 percent of the vote.

What does that mean? It means that any group or party who has a constituency of, say, 500,000 nationwide gets a seat in
the National Assembly. What is the justification for that? When we allocate legislative districts, we are saying that any
district that has 200,000 votes gets a seat. There is no reason why a group that has a national constituency, even if it is a
sectoral or special interest group, should not have a voice in the National Assembly. It also means that, let us say, there
are three or four labor groups, they all register as a party or as a group. If each of them gets only one percent or five of
them get one percent, they are not entitled to any representative. So, they will begin to think that if they really have a
common interest, they should band together, form a coalition and get five percent of the vote and, therefore, have two
seats in the Assembly. Those are the dynamics of a party list system.

We feel that this approach gets around the mechanics of sectoral representation while at the same time making sure that
those who really have a national constituency or sectoral constituency will get a chance to have a seat in the National
Assembly. These sectors or these groups may not have the constituency to win a seat on a legislative district basis. They
may not be able to win a seat on a district basis but surely, they will have votes on a nationwide basis.

The purpose of this is to open the system. In the past elections, we found out that there were certain groups or parties
that, if we count their votes nationwide; have about 1,000,000 or 1,500,000 votes. But they were always third place or
fourth place in each of the districts. So, they have no voice in the Assembly. But this way, they would have five or six
representatives in the Assembly even if they would not win individually in legislative districts. So, that is essentially the
mechanics, the purpose and objectives of the party list system.

BISHOP BACANI: Madam President, am I right in interpreting that when we speak now of party list system though we
refer to sectors, we would be referring to sectoral party list rather than sectors and party list?

MR. MONSOD: As a matter of fact, if this body accepts the party list system, we do not even have to mention sectors
because the sectors would be included in the party list system. They can be sectoral parties within the party list
system.

xxxx

MR. MONSOD. Madam President, I just want to say that we suggested or proposed the party list system because we
wanted to open up the political system to a pluralistic society through a multiparty system. x x x We are for opening up
the system, and we would like very much for the sectors to be there. That is why one of the ways to do that is to
put a ceiling on the number of representatives from any single party that can sit within the 50 allocated under the
party list system. x x x.

xxx

MR. MONSOD. Madam President, the candidacy for the 198 seats is not limited to political parties. My question is
this: Are we going to classify for example Christian Democrats and Social Democrats as political parties? Can
they run under the party list concept or must they be under the district legislation side of it only?

MR. VILLACORTA. In reply to that query, I think these parties that the Commissioner mentioned can field
candidates for the Senate as well as for the House of Representatives. Likewise, they can also field sectoral
candidates for the 20 percent or 30 percent, whichever is adopted, of the seats that we are allocating under the
party list system.

MR. MONSOD. In other words, the Christian Democrats can field district candidates and can also participate in
the party list system?
MR. VILLACORTA. Why not? When they come to the party list system, they will be fielding only sectoral
candidates.

MR. MONSOD. May I be clarified on that? Can UNIDO participate in the party list system?

MR. VILLACORTA. Yes, why not? For as long as they field candidates who come from the different marginalized
sectors that we shall designate in this Constitution.

MR. MONSOD. Suppose Senator Tañada wants to run under BAYAN group and says that he represents the farmers,
would he qualify?

MR. VILLACORTA. No, Senator Tañada would not qualify.

MR. MONSOD. But UNIDO can field candidates under the party list system and say Juan dela Cruz is a farmer. Who
would pass on whether he is a farmer or not?

MR. TADEO. Kay Commissioner Monsod, gusto ko lamang linawin ito. Political parties, particularly minority political
parties, are not prohibited to participate in the party list election if they can prove that they are also organized
along sectoral lines.

MR. MONSOD. What the Commissioner is saying is that all political parties can participate because it is precisely the
contention of political parties that they represent the broad base of citizens and that all sectors are represented in them.
Would the Commissioner agree?

MR. TADEO. Ang punto lamang namin, pag pinayagan mo ang UNIDO na isang political party, it will dominate the party
list at mawawalang saysay din yung sector. Lalamunin mismo ng political parties ang party list system. Gusto ko lamang
bigyan ng diin ang "reserve." Hindi ito reserve seat sa marginalized sectors. Kung titingnan natin itong 198 seats, reserved
din ito sa political parties.

MR. MONSOD. Hindi po reserved iyon kasi anybody can run there. But my question to Commissioner Villacorta and
probably also to Commissioner Tadeo is that under this system, would UNIDO be banned from running under the party list
system?

MR. VILLACORTA. No, as I said, UNIDO may field sectoral candidates. On that condition alone, UNIDO may be
allowed to register for the party list system.

MR. MONSOD. May I inquire from Commissioner Tadeo if he shares that answer?

MR. TADEO. The same.

MR. VILLACORTA. Puwede po ang UNIDO, pero sa sectoral lines.

MR. MONSOD: Sino po ang magsasabi kung iyong kandidato ng UNIDO ay hindi talagang labor leader or isang laborer?
Halimbawa, abogado ito.

MR. TADEO: Iyong mechanics.

MR. MONSOD: Hindi po mechanics iyon because we are trying to solve an inherent problem of sectoral representation.
My question is: Suppose UNIDO fields a labor leader, would he qualify?

MR. TADEO: The COMELEC may look into the truth of whether or not a political party is really organized along a
specific sectoral line. If such is verified or confirmed, the political party may submit a list of individuals who are
actually members of such sectors. The lists are to be published to give individuals or organizations belonging to
such sector the chance to present evidence contradicting claims of membership in the said sector or to question
the claims of the existence of such sectoral organizations or parties. This proceeding shall be conducted by the
COMELEC and shall be summary in character. In other words, COMELEC decisions on this matter are final and
unappealable.52 (Emphasis supplied)

Indisputably, the framers of the 1987 Constitution intended the party-list system to include not only sectoral parties but
also non-sectoral parties. The framers intended the sectoral parties to constitute a part, but not the entirety, of the party-list
system. As explained by Commissioner Wilfredo Villacorta, political parties can participate in the party-list system
"For as long as they field candidates who come from the different marginalized sectors that we shall designate in
this Constitution."53

In fact, the framers voted down, 19-22, a proposal to reserve permanent seats to sectoral parties in the House of
Representatives, or alternatively, to reserve the party-list system exclusively to sectoral parties. As clearly explained by
Justice Jose C. Vitug in his Dissenting Opinion in Ang Bagong Bayani:

The draft provisions on what was to become Article VI, Section 5, subsection (2), of the 1987 Constitution took off from
two staunch positions — the first headed by Commissioner Villacorta, advocating that of the 20 per centum of the total
seats in Congress to be allocated to party-list representatives half were to be reserved to appointees from the
marginalized and underrepresented sectors. The proposal was opposed by some Commissioners. Mr. Monsod expressed
the difficulty in delimiting the sectors that needed representation. He was of the view that reserving seats for the
marginalized and underrepresented sectors would stunt their development into full-pledged parties equipped with electoral
machinery potent enough to further the sectoral interests to be represented. The Villacorta group, on the other hand, was
apprehensive that pitting the unorganized and less-moneyed sectoral groups in an electoral contest would be like placing
babes in the lion's den, so to speak, with the bigger and more established political parties ultimately gobbling them up.
R.A. 7941 recognized this concern when it banned the first five major political parties on the basis of party representation
in the House of Representatives from participating in the party-list system for the first party-list elections held in 1998 (and
to be automatically lifted starting with the 2001 elections). The advocates for permanent seats for sectoral representatives
made an effort towards a compromise — that the party-list system be open only to underrepresented and marginalized
sectors. This proposal was further whittled down by allocating only half of the seats under the party-list system to
candidates from the sectors which would garner the required number of votes. The majority was unyielding. Voting 19-22,
the proposal for permanent seats, and in the alternative the reservation of the party-list system to the sectoral groups, was
voted down. The only concession the Villacorta group was able to muster was an assurance of reserved seats for selected
sectors for three consecutive terms after the enactment of the 1987 Constitution, by which time they would be expected to
gather and solidify their electoral base and brace themselves in the multi-party electoral contest with the more veteran
political groups.54 (Emphasis supplied)

Thus, in the end, the proposal to give permanent reserved seats to certain sectors was outvoted. Instead, the reservation
of seats to sectoral representatives was only allowed for the first three consecutive terms. 55 There can be no doubt
whatsoever that the framers of the 1987 Constitution expressly rejected the proposal to make the party-list system
exclusively for sectoral parties only, and that they clearly intended the party-list system to include both sectoral and non-
sectoral parties.

The common denominator between sectoral and non-sectoral parties is that they cannot expect to win in legislative district
elections but they can garner, in nationwide elections, at least the same number of votes that winning candidates can
garner in legislative district elections. The party-list system will be the entry point to membership in the House of
Representatives for both these non-traditional parties that could not compete in legislative district elections.

The indisputable intent of the framers of the 1987 Constitution to include in the party-list system both sectoral and non-
sectoral parties is clearly written in Section 5(1), Article VI of the Constitution, which states:

Section 5. (1) The House of Representative shall be composed of not more that two hundred and fifty members, unless
otherwise fixed by law, who shall be elected from legislative districts apportioned among the provinces, cities, and the
Metropolitan Manila area in accordance with the number of their respective inhabitants, and on the basis of a uniform and
progressive ratio, and those who, as provided by law, shall be elected through a party-list system of registered
national, regional, and sectoral parties or organizations. (Emphasis supplied)

Section 5(1), Article VI of the Constitution is crystal-clear that there shall be "a party-list system of registered national,
regional, and sectoral parties or organizations." The commas after the words "national," and "regional," separate
national and regional parties from sectoral parties. Had the framers of the 1987 Constitution intended national and regional
parties to be at the same time sectoral, they would have stated "national and regional sectoral parties." They did not,
precisely because it was never their intention to make the party-list system exclusively sectoral.

What the framers intended, and what they expressly wrote in Section 5(1), could not be any clearer: the party-list system
is composed of three different groups, and the sectoral parties belong to only one of the three groups. The text of Section
5(1) leaves no room for any doubt that national and regional parties are separate from sectoral parties.

Thus, the party-list system is composed of three different groups: (1) national parties or organizations; (2) regional
parties or organizations; and (3) sectoral parties or organizations. National and regional parties or organizations
are different from sectoral parties or organizations. National and regional parties or organizations need not be organized
along sectoral lines and need not represent any particular sector.

Moreover, Section 5(2), Article VI of the 1987 Constitution mandates that, during the first three consecutive terms of
Congress after the ratification of the 1987 Constitution, "one-half of the seats allocated to party-list representatives shall
be filled, as provided by law, by selection or election from the labor, peasant, urban poor, indigenous cultural communities,
women, youth, and such other sectors as may be provided by law, except the religious sector." This provision clearly
shows again that the party-list system is not exclusively for sectoral parties for two obvious reasons.

First, the other one-half of the seats allocated to party-list representatives would naturally be open to non-sectoral party-list
representatives, clearly negating the idea that the party-list system is exclusively for sectoral parties representing the
"marginalized and underrepresented." Second, the reservation of one-half of the party-list seats to sectoral parties applies
only for the first "three consecutive terms after the ratification of this Constitution," clearly making the party-list system fully
open after the end of the first three congressional terms. This means that, after this period, there will be no seats reserved
for any class or type of party that qualifies under the three groups constituting the party-list system.

Hence, the clear intent, express wording, and party-list structure ordained in Section 5(1) and (2), Article VI of the
1987 Constitution cannot be disputed: the party-list system is not for sectoral parties only, but also for non-
sectoral parties.
Republic Act No. 7941 or the Party-List System Act, which is the law that implements the party-list system prescribed in
the Constitution, provides:

Section 3. Definition of Terms. (a) The party-list system is a mechanism of proportional representation in the election of
representatives to the House of Representatives from national, regional and sectoral parties or organizations or coalitions
thereof registered with the Commission on Elections (COMELEC). Component parties or organizations of a coalition may
participate independently provided the coalition of which they form part does not participate in the party-list system.

(b) A party means either a political party or a sectoral party or a coalition of parties.

(c) A political party refers to an organized group of citizens advocating an ideology or platform, principles
and policies for the general conduct of government and which, as the most immediate means of securing
their adoption, regularly nominates and supports certain of its leaders and members as candidates for
public office.

It is a national party when its constituency is spread over the geographical territory of at least a majority of the
regions. It is a regional party when its constituency is spread over the geographical territory of at least a majority of
the cities and provinces comprising the region.

(d) A sectoral party refers to an organized group of citizens belonging to any of the sectors enumerated in
Section 5 hereof whose principal advocacy pertains to the special interest and concerns of their sector.

(e) A sectoral organization refers to a group of citizens or a coalition of groups of citizens who share similar
physical attributes or characteristics, employment, interests or concerns.

(f) A coalition refers to an aggrupation of duly registered national, regional, sectoral parties or organizations for
political and/or election purposes. (Emphasis supplied)

Section 3(a) of R.A. No. 7941 defines a "party" as "either a political party or a sectoral party or a coalition of parties."
Clearly, a political party is different from a sectoral party. Section 3(c) of R.A. No. 7941 further provides that a "political
party refers to an organized group of citizens advocating an ideology or platform, principles and policies for the
general conduct of government." On the other hand, Section 3(d) of R.A. No. 7941 provides that a "sectoral
party refers to an organized group of citizens belonging to any of the sectors enumerated in Section 5 hereof whose
principal advocacy pertains to the special interest and concerns of their sector." R.A. No. 7941 provides different
definitions for a political and a sectoral party. Obviously, they are separate and distinct from each other.

R.A. No. 7941 does not require national and regional parties or organizations to represent the "marginalized and
underrepresented" sectors. To require all national and regional parties under the party-list system to represent the
"marginalized and underrepresented" is to deprive and exclude, by judicial fiat, ideology-based and cause-oriented parties
from the party-list system. How will these ideology-based and cause-oriented parties, who cannot win in legislative district
elections, participate in the electoral process if they are excluded from the party-list system? To exclude them from the
party-list system is to prevent them from joining the parliamentary struggle, leaving as their only option the armed struggle.
To exclude them from the party-list system is, apart from being obviously senseless, patently contrary to the clear intent
and express wording of the 1987 Constitution and R.A. No. 7941.

Under the party-list system, an ideology-based or cause-oriented political party is clearly different from a sectoral party. A
political party need not be organized as a sectoral party and need not represent any particular sector. There is no
requirement in R.A. No. 7941 that a national or regional political party must represent a "marginalized and
underrepresented" sector. It is sufficient that the political party consists of citizens who advocate the same ideology or
platform, or the same governance principles and policies, regardless of their economic status as citizens.

Section 5 of R.A. No. 7941 states that "the sectors shall include labor, peasant, fisherfolk, urban poor, indigenous cultural
communities, elderly, handicapped, women, youth, veterans, overseas workers, and professionals."56 The sectors
mentioned in Section 5 are not all necessarily "marginalized and underrepresented." For sure, "professionals" are not by
definition "marginalized and underrepresented," not even the elderly, women, and the youth. However, professionals, the
elderly, women, and the youth may "lack well-defined political constituencies," and can thus organize themselves into
sectoral parties in advocacy of the special interests and concerns of their respective sectors.

Section 6 of R.A. No. 7941 provides another compelling reason for holding that the law does not require national or
regional parties, as well as certain sectoral parties in Section 5 of R.A. No. 7941, to represent the "marginalized and
underrepresented." Section 6 provides the grounds for the COMELEC to refuse or cancel the registration of parties or
organizations after due notice and hearing.

Section 6. Refusal and/or Cancellation of Registration. — The COMELEC may, motu proprio or upon verified complaint of
any interested party, refuse or cancel, after due notice and hearing, the registration of any national, regional or sectoral
party, organization or coalition on any of the following grounds:

(1) It is a religious sect or denomination, organization or association organized for religious purposes;

(2) It advocates violence or unlawful means to seek its goal;


(3) It is a foreign party or organization;

(4) It is receiving support from any foreign government, foreign political party, foundation, organization, whether
directly or through any of its officers or members or indirectly through third parties for partisan election purposes;

(5) It violates or fails to comply with laws, rules or regulations relating to elections;

(6) It declares untruthful statements in its petition;

(7) It has ceased to exist for at least one (1) year; or

(8) It fails to participate in the last two (2) preceding elections or fails to obtain at least two per centum (2%) of the
votes cast under the party-list system in the two (2) preceding elections for the constituency in which it has
registered.

None of the 8 grounds to refuse or cancel registration refers to non-representation of the "marginalized and
underrepresented."

The phrase "marginalized and underrepresented" appears only once in R.A. No. 7941, in Section 2 on Declaration of
Policy.57 Section 2 seeks "to promote proportional representation in the election of representatives to the House of
Representatives through the party-list system," which will enable Filipinos belonging to the "marginalized and
underrepresented sectors, organizations and parties, and who lack well-defined political constituencies," to
become members of the House of Representatives. While the policy declaration in Section 2 of R.A. No. 7941 broadly
refers to "marginalized and underrepresented sectors, organizations and parties," the specific implementing provisions of
R.A. No. 7941 do not define or require that the sectors, organizations or parties must be "marginalized and
underrepresented." On the contrary, to even interpret that all the sectors mentioned in Section 5 are "marginalized and
underrepresented" would lead to absurdities.

How then should we harmonize the broad policy declaration in Section 2 of R.A. No. 7941 with its specific implementing
provisions, bearing in mind the applicable provisions of the 1987 Constitution on the matter?

The phrase "marginalized and underrepresented" should refer only to the sectors in Section 5 that are, by their
nature, economically "marginalized and underrepresented." These sectors are: labor, peasant, fisherfolk, urban poor,
indigenous cultural communities, handicapped, veterans, overseas workers, and other similar sectors. For these sectors,
a majority of the members of the sectoral party must belong to the "marginalized and underrepresented." The
nominees of the sectoral party either must belong to the sector, or must have a track record of advocacy for the
sector represented. Belonging to the "marginalized and underrepresented" sector does not mean one must "wallow in
poverty, destitution or infirmity." It is sufficient that one, or his or her sector, is below the middle class. More specifically,
the economically "marginalized and underrepresented" are those who fall in the low income group as classified by the
National Statistical Coordination Board.58

The recognition that national and regional parties, as well as sectoral parties of professionals, the elderly, women and the
youth, need not be "marginalized and underrepresented" will allow small ideology-based and cause-oriented parties who
lack "well-defined political constituencies" a chance to win seats in the House of Representatives. On the other hand,
limiting to the "marginalized and underrepresented" the sectoral parties for labor, peasant, fisherfolk, urban poor,
indigenous cultural communities, handicapped, veterans, overseas workers, and other sectors that by their nature are
economically at the margins of society, will give the "marginalized and underrepresented" an opportunity to likewise win
seats in the House of Representatives.

This interpretation will harmonize the 1987 Constitution and R.A. No. 7941 and will give rise to a multi-party system where
those "marginalized and underrepresented," both in economic and ideological status, will have the opportunity to send
their own members to the House of Representatives. This interpretation will also make the party-list system honest and
transparent, eliminating the need for relatively well-off party-list representatives to masquerade as "wallowing in poverty,
destitution and infirmity," even as they attend sessions in Congress riding in SUVs.

The major political parties are those that field candidates in the legislative district elections. Major political parties cannot
participate in the party-list elections since they neither lack "well-defined political constituencies" nor represent
"marginalized and underrepresented" sectors. Thus, the national or regional parties under the party-list system are
necessarily those that do not belong to major political parties. This automatically reserves the national and regional
parties under the party-list system to those who "lack well-defined political constituencies," giving them the opportunity to
have members in the House of Representatives.

To recall, Ang Bagong Bayani expressly declared, in its second guideline for the accreditation of parties under the party-
list system, that "while even major political parties are expressly allowed by RA 7941 and the Constitution to participate in
the party-list system, they must comply with the declared statutory policy of enabling ‘Filipino citizens belonging to
marginalized and underrepresented sectors xxx to be elected to the House of Representatives.’ "However, the
requirement in Ang Bagong Bayani, in its second guideline, that "the political party xxx must represent the marginalized
and underrepresented," automatically disqualified major political parties from participating in the party-list system.
This inherent inconsistency in Ang Bagong Bayani has been compounded by the COMELEC’s refusal to register
sectoral wings officially organized by major political parties. BANAT merely formalized the prevailing practice when
it expressly prohibited major political parties from participating in the party-list system, even through their sectoral wings.
Section 11 of R.A. No. 7941 expressly prohibited the "first five (5) major political parties on the basis of party
representation in the House of Representatives at the start of the Tenth Congress" from participating in the May 1988
party-list elections.59 Thus, major political parties can participate in subsequent party-list elections since the
prohibition is expressly limited only to the 1988 party-list elections. However, major political parties should
participate in party-list elections only through their sectoral wings. The participation of major political parties through their
sectoral wings, a majority of whose members are "marginalized and underrepresented" or lacking in "well-defined political
constituencies," will facilitate the entry of the "marginalized and underrepresented" and those who "lack well-defined
political constituencies" as members of the House of Representatives.

The 1987 Constitution and R.A. No. 7941 allow major political parties to participate in party-list elections so as to
encourage them to work assiduously in extending their constituencies to the "marginalized and underrepresented" and to
those who "lack well-defined political constituencies." The participation of major political parties in party-list elections must
be geared towards the entry, as members of the House of Representatives, of the "marginalized and underrepresented"
and those who "lack well-defined political constituencies," giving them a voice in law-making. Thus,to participate in party-
list elections, a major political party that fields candidates in the legislative district elections must organize a sectoral wing,
like a labor, peasant, fisherfolk, urban poor, professional, women or youth wing, that can register under the party-list
system.

Such sectoral wing of a major political party must have its own constitution, by-laws, platform or program of government,
officers and members, a majority of whom must belong to the sector represented. The sectoral wing is in itself an
independent sectoral party, and is linked to a major political party through a coalition. This linkage is allowed by Section 3
of R.A. No. 7941, which provides that "component parties or organizations of a coalition may participate independently (in
party-list elections) provided the coalition of which they form part does not participate in the party-list system."

Section 9 of R.A. No. 7941 prescribes the qualifications of party-list nominees. This provision prescribes a special
qualification only for the nominee from the youth sector.

Section 9. Qualifications of Party-List Nominees. No person shall be nominated as party-list representative unless he is a
natural-born citizen of the Philippines, a registered voter, a resident of the Philippines for a period of not less than one (1)
year immediately preceding the day of the election, able to read and write, a bona fide member of the party or organization
which he seeks to represent for at least ninety (90) days preceding the day of the election, and is at least twenty-five (25)
years of age on the day of the election.

In case of a nominee of the youth sector, he must at least be twenty-five (25) but not more than thirty (30) years of age on
the day of the election.

Any youth sectoral representative who attains the age of thirty (30) during his term shall be allowed to continue in office
until the expiration of his term.
1âwphi1

A party-list nominee must be a bona fide member of the party or organization which he or she seeks to represent. In the
case of sectoral parties, to be a bona fide party-list nominee one must either belong to the sector represented, or
have a track record of advocacy for such sector.

In disqualifying petitioners, the COMELEC used the criteria prescribed in Ang Bagong Bayani and BANAT. Ang Bagong
Bayani laid down the guidelines for qualifying those who desire to participate in the party-list system:

First, the political party, sector, organization or coalition must represent the marginalized and underrepresented
groups identified in Section 5 of RA 7941. x x x

Second, while even major political parties are expressly allowed by RA 7941 and the Constitution to participate in the
party-list system, they must comply with the declared statutory policy of enabling "Filipino citizens belonging to
marginalized and underrepresented sectors x x x to be elected to the House of Representatives." x x x.

xxxx

Third, x x x the religious sector may not be represented in the party-list system. x x x.

xxxx

Fourth, a party or an organization must not be disqualified under Section 6 of RA 7941, which enumerates the grounds for
disqualification as follows:

"(1) It is a religious sect or denomination, organization or association, organized for religious purposes;

(2) It advocates violence or unlawful means to seek its goal;

(3) It is a foreign party or organization;

(4) It is receiving support from any foreign government, foreign political party, foundation, organization, whether
directly or through any of its officers or members or indirectly through third parties for partisan election purposes;
(5) It violates or fails to comply with laws, rules or regulations relating to elections;

(6) It declares untruthful statements in its petition;

(7) It has ceased to exist for at least one (1) year; or

(8) It fails to participate in the last two (2) preceding elections or fails to obtain at least two per centum (2%) of the
votes cast under the party-list system in the two (2) preceding elections for the constituency in which it has
registered."

Fifth, the party or organization must not be an adjunct of, or a project organized or an entity funded or assisted by, the
government. x x x.

xxxx

Sixth, the party must not only comply with the requirements of the law; its nominees must likewise do so. Section 9 of RA
7941 reads as follows:

"SEC 9. Qualifications of Party-List Nominees. - No person shall be nominated as party-list representative unless he is a
natural-born citizen of the Philippines, a registered voter, a resident of the Philippines for a period of not less than one
(1)year immediately preceding the day of the election, able to read and write, a bona fide member of the party or
organization which he seeks to represent for at least ninety (90) days preceding the day of the election, and is at least
twenty-five (25) years of age on the day of the election.

In case of a nominee of the youth sector, he must at least be twenty-five (25) but not more than thirty (30) years of age on
the day of the election. Any youth sectoral representative who attains the age of thirty (30) during his term shall be allowed
to continue in office until the expiration of his term."

Seventh, not only the candidate party or organization must represent marginalized and underrepresented
sectors; so also must its nominees. x x x.

Eighth, x x x the nominee must likewise be able to contribute to the formulation and enactment of appropriate legislation
that will benefit the nation as a whole. (Emphasis supplied)

In 2009, by a vote of 8-7 in BANAT, this Court stretched the Ang Bagong Bayani ruling further. In BANAT, the majority
officially excluded major political parties from participating in party-list elections, 60 abandoning even the lip-service that Ang
Bagong Bayani accorded to the 1987 Constitution and R.A.No. 7941 that major political parties can participate in party-list
elections.

The minority in BANAT, however, believed that major political parties can participate in the party-list system through their
sectoral wings. The minority expressed that "[e]xcluding the major political parties in party-list elections is manifestly
against the Constitution, the intent of the Constitutional Commission, and R.A. No. 7941. This Court cannot engage in
socio-political engineering and judicially legislate the exclusion of major political parties from the party-list elections in
patent violation of the Constitution and the law."61 The experimentations in socio-political engineering have only resulted in
confusion and absurdity in the party-list system. Such experimentations, in clear contravention of the 1987 Constitution
and R.A. No. 7941, must now come to an end.

We cannot, however, fault the COMELEC for following prevailing jurisprudence in disqualifying petitioners. In following
prevailing jurisprudence, the COMELEC could not have committed grave abuse of discretion. However, for the coming 13
May 2013 party-list elections, we must now impose and mandate the party-list system actually envisioned and
authorized under the 1987 Constitution and R.A. No. 7941. In BANAT, this Court devised a new formula in the allocation
of party-list seats, reversing the COMELEC's allocation which followed the then prevailing formula in Ang Bagong Bayani.
In BANAT, however, the Court did not declare that the COMELEC committed grave abuse of discretion. Similarly, even as
we acknowledge here that the COMELEC did not commit grave abuse of discretion, we declare that it would not be in
accord with the 1987 Constitution and R.A. No. 7941 to apply the criteria in Ang Bagong Bayani and BANAT in
determining who are qualified to participate in the coming 13 May 2013 party-list elections. For this purpose, we
suspend our rule62 that a party may appeal to this Court from decisions or orders of the COMELEC only if the COMELEC
committed grave abuse of discretion.

Thus, we remand all the present petitions to the COMELEC. In determining who may participate in the coming 13 May
2013 and subsequent party-list elections, the COMELEC shall adhere to the following parameters:

1. Three different groups may participate in the party-list system: (1) national parties or organizations, (2) regional
parties or organizations, and (3) sectoral parties or organizations.

2. National parties or organizations and regional parties or organizations do not need to organize along sectoral
lines and do not need to represent any "marginalized and underrepresented" sector.

3. Political parties can participate in party-list elections provided they register under the party-list system and do
not field candidates in legislative district elections. A political party, whether major or not, that fields candidates in
legislative district elections can participate in party-list elections only through its sectoral wing that can separately
register under the party-list system. The sectoral wing is by itself an independent sectoral party, and is linked to a
political party through a coalition.

4. Sectoral parties or organizations may either be "marginalized and underrepresented" or lacking in "well-defined
political constituencies." It is enough that their principal advocacy pertains to the special interest and concerns of
their sector. The sectors that are "marginalized and underrepresented" include labor, peasant, fisherfolk, urban
poor, indigenous cultural communities, handicapped, veterans, and overseas workers. The sectors that lack "well-
defined political constituencies" include professionals, the elderly, women, and the youth.

5. A majority of the members of sectoral parties or organizations that represent the "marginalized and
underrepresented" must belong to the "marginalized and underrepresented" sector they represent. Similarly, a
majority of the members of sectoral parties or organizations that lack "well-defined political constituencies" must
belong to the sector they represent. The nominees of sectoral parties or organizations that represent the
"marginalized and underrepresented," or that represent those who lack "well-defined political constituencies,"
either must belong to their respective sectors, or must have a track record of advocacy for their respective sectors.
The nominees of national and regional parties or organizations must be bona-fide members of such parties or
organizations.

6. National, regional, and sectoral parties or organizations shall not be disqualified if some of their nominees are
disqualified, provided that they have at least one nominee who remains qualified.

The COMELEC excluded from participating in the 13 May 2013 party-list elections those that did not satisfy these two
criteria: (1) all national, regional, and sectoral groups or organizations must represent the "marginalized and
underrepresented" sectors, and (2) all nominees must belong to the "marginalized and underrepresented" sector they
represent. Petitioners may have been disqualified by the COMELEC because as political or regional parties they are not
organized along sectoral lines and do not represent the "marginalized and underrepresented." Also, petitioners' nominees
who do not belong to the sectors they represent may have been disqualified, although they may have a track record of
advocacy for their sectors. Likewise, nominees of non-sectoral parties may have been disqualified because they do not
belong to any sector. Moreover, a party may have been disqualified because one or more of its nominees failed to qualify,
even if the party has at least one remaining qualified nominee. As discussed above, the disqualification of petitioners, and
their nominees, under such circumstances is contrary to the 1987 Constitution and R.A. No. 7941.

This Court is sworn to uphold the 1987 Constitution, apply its provisions faithfully, and desist from engaging in socio-
economic or political experimentations contrary to what the Constitution has ordained. Judicial power does not include the
power to re-write the Constitution. Thus, the present petitions should be remanded to the COMELEC not because the
COMELEC committed grave abuse of discretion in disqualifying petitioners, but because petitioners may now possibly
qualify to participate in the coming 13 May 2013 party-list elections under the new parameters prescribed by this Court.

WHEREFORE, all the present 54 petitions are GRANTED. The 13 petitions, which have been granted Status Quo Ante
Orders but without mandatory injunction to include the names of petitioners in the printing of ballots, are remanded to the
Commission on Elections only for determination whether petitioners are qualified to register under the party-list system
under the parameters prescribed in this Decision but they shall not participate in the 13 May 2013 part-list elections. The
41 petitions, which have been granted mandatory injunctions to include the names of petitioners in the printing of ballots,
are remanded to the Commission on Elections for determination whether petitioners are qualified to register under the
party-list system and to participate in the 13 May 2013 party-list elections under the parameters prescribed in this
Decision. The Commission on Elections may conduct summary evidentiary hearings for this purpose. This Decision is
immediately executory.

SO ORDERED.

- Coalition of Asso. of Senior Citizens in the Phil. v Comelec, 201 SCRA 786 (2013)

The present petitions were filed by the two rival factions within the same party-list organization, the Coalition of
Associations of Senior Citizens in the Phil., Inc. (SENIOR CITIZENS) that are now praying for essentially the same reliefs
from this Court.

One group is headed by Godofredo V. Arquiza (Rep. Arquiza), the organization’s incumbent representative in the House
of Representatives. This group shall be hereinafter referred to as the Arquiza Group. The other group is led by Francisco
G. Datol, Jr., the organization’s erstwhile third nominee. This group shall be hereinafter referred to as the Datol Group.

G.R. Nos. 206844-45 is the Extremely Very Urgent Petition for Certiorari (With Prayer for the Forthwith Issuance of a Writ
of Preliminary Injunction and Temporary Restraining Order [TRO] and/or Status Quo Ante Order [SQAO]) 1 filed in the
name of SENIOR CITIZENS by Francisco G. Datol, Jr. For brevity, we shall refer to this petition as the Datol Group’s
petition.

G.R. No. 206982 is the Very Urgent Petition for Certiorari (With Application for a Temporary Restraining Order and Writ of
Preliminary Injunction)2 filed on behalf of SENIOR CITIZENS by Rep. Arquiza. We shall refer to this as the Arquiza
Group’s petition.
The above petitions were filed pursuant to Rule 643 in relation to Rule 654 of the Rules of Court, both assailing the
Omnibus Resolution5 dated May 10, 2013 of the Commission on Elections (COMELEC) En Banc in SPP No. 12-157
(PLM) and SPP No. 12-191 (PLM). Said Resolution disqualified SENIOR CITIZENS from participating in the May 13, 2013
elections and ordered the cancellation of its registration and accreditation as a party-list organization.

THE ANTECEDENTS

On March 16, 2007, the COMELEC En Banc accredited SENIOR CITIZENS as a party-list organization in a
Resolution6 issued on even date in SPP No. 06-026 (PL).

SENIOR CITIZENS participated in the May 14, 2007 elections. However, the organization failed to get the required two
percent (2%) of the total votes cast.7 Thereafter, SENIOR CITIZENS was granted leave to intervene in the case of
Barangay Association for National Advancement and Transparency (BANAT) v. Commission on Elections. 8 In accordance
with the procedure set forth in BANAT for the allocation of additional seats under the party-list system, SENIOR CITIZENS
was allocated one seat in Congress. Rep. Arquiza, then the organization’s first nominee, served as a member of the
House of Representatives.

Subsequently, SENIOR CITIZENS was allowed to participate in the May 10, 2010 elections.

On May 5, 2010, the nominees of SENIOR CITIZENS signed an agreement, entitled Irrevocable Covenant, the relevant
terms of which we quote:

IRREVOCABLE COVENANT

KNOW ALL MEN BY THESE PRESENT

We, in representation of our respective personal capacity, hereby covenant and agree as follows:

ARTICLE I
PARTIES AND PERSONS

1. ATTY. GODOFREDO V. ARQUIZA, of legal age, married, Filipino, and residing at 1881 C.M. Recto Avenue,
Sampaloc, Manila, and representing the Senior Citizens Party-list in my capacity as President with our General
Headquarters at Room 404 West Trade Center, 132 West Avenue, hereinafter referred to as the FIRST PARTY;

2. ATTY. DAVID L. KHO, of legal age, married, Filipino, and residing at 35 Quezon Avenue, Quezon City,
hereinafter referred to as the SECOND PARTY;

3. FRANCISCO G. DATOL, JR., of legal age, married, Filipino, and residing at North Olympus Blk., 3, Lot 15 Ph4
Grieg St., Novaliches, Quezon City, hereinafter referred to as the THIRD PARTY;

4. REMEDIOS D. ARQUIZA, of legal age, married, Filipino, and residing at 1881 C.M. Recto Avenue, Sampaloc,
Manila, hereinafter referred to as the FOURTH PARTY;

5. LINDA GADDI DAVID, of legal age, married, Filipino, and residing at 150 Don Francisco, St. Francis Vil., San
Fernando, Pampanga City (sic) hereinafter referred to as the FIFTH PARTY;

xxxx

ARTICLE III
THE LIST OF CANDIDATES

We agree that official candidates of the SENIOR CITIZENS PARTY-LIST and in the following order shall be:

Name CTC No. Issued at Issued on


1. Godofredo V. Arquiza S.C.I.D.#2615256 Manila 04-02-04
2. David L. Kho 16836192 Quezon City 03-15-09
3. Francisco G. Datol, Jr. 27633197 Quezon City 02-10-10
4. Remedios D. Arquiza S.C.I.D.#50696 Quezon City 01-02-07
5. Linda Gaddi David CCI2009 12306699 Pampanga 01-04-10

ARTICLE IV
SHARING OF POWER

The Nominees agreed and pledged on their legal and personal honor and interest as well as the legal privileges and rights
of the respective party-list offices, under the following circumstances and events:
ELECTION RESULTS

Where only ONE (1) candidate qualifies and is proclaimed, then No. 1 shall assume the Office of Party-list Representative
in CONGRESS from July 1, 2010 to June 30, 2012 and shall relinquish his seat in Congress by the proper and legal acts
and No. 2 shall assume said seat from July 1, 2012 to June 30, 2013;

In the event TWO (2) candidates qualify and are proclaimed, then, No. 1 shall serve for three (3) years, and No. 2 and No.
3 will each serve for one-and-a-half years.

In the event THREE (3) candidates qualify and are proclaimed, then No. 1 shall serve for three years; No. 2 will serve for
two (2) years and afterwards shall relinquish the second seat to No. 4 nominee, who will then serve for one (1) year; No. 3
will occupy the third seat for two (2) years and afterwards shall relinquish said seat on the third year to Nominee 5, who
will serve for the remaining one (1) year.

In Fine:

If only one (1) seat is won If three (3) seats are won:
No. 1 nominee = 2 years
No. 2 nominee = 1 year No. 1 nominee = 3 years
No. 2 nominee = 2 years
If two (2) seats are won No. 3 nominee = 2 years
No. 1 nominee = 3 years No. 4 nominee = 1 year
No. 2 nominee = 1½ years No. 5 nominee = 1 year
No. 3 nominee = 1½ years
All beginning July 1, 2010

SHARING OF RIGHTS
BENEFITS AND PRIVILEGES

That serving incumbent Congress Representative in the event one or more is elected and qualified shall observe proper
sharing of certain benefits by virtue of his position as such, to include among others, appointment of persons in his office,
projects which may redound to the benefits and privileges that may be possible under the law.

The above mentioned parties shall oversee the implementation of this COVENANT.

IN WITNESS WHEREOF, the parties hereto have set their hands this MAY 05 2010 in QUEZON CITY.

(Signed) (Signed)

Godofredo V. Arquiza David L. Kho


S.C.I.D. #2615256 Iss. at Manila CTC#16836192 Iss. at
on 04-02-04 Quezon City on 03-15-09

(Signed) (Signed)

Francisco G. Datol, Jr. Remedios D. Arquiza


CTC#16836192 Iss. at S.C.I.D.#50696 Iss. at
Quezon City on 03-15-09 Quezon City on 01-02-07

(Signed)

Linda Gaddi David


CTC#CCI2009 12306699 Iss. at
San Fernando, Pampanga on 01-04-109

After the conduct of the May 10, 2010 elections, SENIOR CITIZENS ranked second among all the party-list candidates
and was allocated two seats in the House of Representatives. The first seat was occupied by its first nominee, Rep.
Arquiza, while the second was given to its second nominee, David L. Kho (Rep. Kho).

The split among the ranks of SENIOR CITIZENS came about not long after. According to the Datol Group’s petition, the
members of SENIOR CITIZENS held a national convention on November 27, 2010 in order to address "the unfulfilled
commitment of Rep. Arquiza to his constituents."10 Further, a new set of officers and members of the Board of Trustees of
the organization were allegedly elected during the said convention. SENIOR CITIZENS’ third nominee, Francisco G. Datol,
Jr., was supposedly elected as the organization’s Chairman. Thereafter, on November 30, 2010, in an opposite turn of
events, Datol was expelled from SENIOR CITIZENS by the Board of Trustees that were allied with Rep. Arquiza. 11
Thenceforth, the two factions of SENIOR CITIZENS had been engaged in a bitter rivalry as both groups, with their own
sets of officers, claimed leadership of the organization.

The Resignation of Rep. Kho

On December 14, 2011, Rep. Arquiza informed the office of COMELEC Chairman Sixto S. Brillantes, Jr. in a letter 12 dated
December 8, 2011 that the second nominee of SENIOR CITIZENS, Rep. Kho, had tendered his resignation, which was to
take effect on December 31, 2011. The fourth nominee, Remedios D. Arquiza, was to assume the vacant position in view
of the previous expulsion from the organization of the third nominee, Francisco G. Datol, Jr.

The letter of Rep. Arquiza was also accompanied by a petition 13 dated December 14, 2011 in the name of SENIOR
CITIZENS. The petition prayed that the "confirmation and approval of the replacement of Congressman David L. Kho, in
the person of the fourth nominee, Remedios D. Arquiza, due to the expulsion of the third nominee, Francisco G. Datol, Jr.,
be issued immediately in order to pave the way of her assumption into the office." 14 Before the COMELEC, the petition was
docketed as E.M. No. 12-040.

Attached to the petition was the resignation letter 15 of Rep. Kho, which was addressed to the Speaker of the House of
Representatives. The letter stated thus:

THE HONORABLE SPEAKER


House of Representatives
Congress
Republic of the Philippines
Quezon City

Sir:

I am hereby tendering my irrevocable resignation as Representative of the Senior Citizens Party-list in the House of
Representatives, effective December 31, 2011 in the event that only two (2) seats are won by our party-list group; and will
resign on June 30, 2012 in case three seats are won.

As a consequence thereof, the Coalition of Associations of Senior Citizens in the Philippines, Inc. shall nominate my
successor pursuant to law and Rules on the matter.

Please accept my esteem and respect.

Truly yours,

(Signed)
Rep. David L. Kho
Party-list Congressman

Copy furnished:
The Board of Trustees
Coalition of Associations of Senior Citizens in the Philippines, Inc.16

According to the Datol Group, Rep. Kho submitted to them a letter dated December 31, 2011, notifying them of his
resignation in this wise:

December 31, 2011

COALITION OF ASSOCIATIONS OF
SENIOR CITIZENS IN THE PHILS., INC.
Rm. 405, 4th Floor, WTC Building
132 West Avenue, Quezon City

Gentlemen/Ladies:

It is with deepest regret that I inform this esteemed organization of my decision to resign as the party-list nominee for the
House of Representatives this 15th Congress for personal reason already conveyed to you.

Thank you for the opportunity to serve the Senior Citizens of our dear country.

Very truly yours,

(Signed)
DAVID L. KHO17

In the interim, during the pendency of E.M. No. 12-040, COMELEC Resolution No. 9366 18 was promulgated on February
21, 2012. Pertinently, Section 7 of Rule 4 thereof provided that:
SEC. 7. Term sharing of nominees. Filing of vacancy as a result of term sharing agreement among nominees of winning
party-list groups/organizations shall not be allowed.

On March 12, 2012, the Board of Trustees of SENIOR CITIZENS that were allied with Rep. Arquiza issued Board
Resolution No. 003-2012, which pertinently stated thus:

BOARD RESOLUTION NO. 003-2012


Series of 2012

A RESOLUTION RECALLING THE ACCEPTANCE OF THE BOARD IN RESOLUTION NO. 11-0012 OF THE
RESIGNATION OF CONGRESSMAN DAVID L. KHO AND ALLOWING HIM TO CONTINUE REPRESENTING THE
SENIOR CITIZENS PARTY-LIST IN THE HOUSE OF REPRESENTATIVES, ALLOWING HIM TO CONTINUE HIS TERM
AND IMPOSING CERTAIN CONDITIONS ON HIM TO BE PERFORMED WITH THE COALITION;

WHEREAS, the second nominee, Congressman David L. Kho, tendered his resignation as representative of the Senior
Citizens Party-list effective December 31, 2011, x x x;

WHEREAS, the said resignation was accepted by the Board of Trustees in a resolution signed unanimously, in view of the
nature of his resignation, and in view of his determination to resign and return to private life, x x x;

WHEREAS, after much deliberation and consultation, the said nominee changed his mind and requested the Board of
Trustees to reconsider the acceptance, for he also reconsidered his resignation, and requested to continue his term;

WHEREAS, in consideration of all factors affecting the party-list and in view of the forthcoming elections, the Board opted
to reconsider the acceptance, recall the same, and allow Cong. David L. Kho to continue his term;

WHEREAS, the Coalition, in recalling the acceptance of the Board, is however imposing certain conditions on Cong. Kho
to be performed;

NOW THEREFORE, BE IT RESOLVED, AS IT IS HEREBY RESOLVED to recall the acceptance of the resignation of
Congressman David L. Kho in view of his request and change of mind, hence allow him to continue his term subject to
conditions stated above.19

Thereafter, on April 18, 2012, the COMELEC En Banc conducted a hearing on SENIOR CITIZENS’ petition in E.M. No.
12-040. At the hearing, the counsel for SENIOR CITIZENS (Arquiza Group) admitted that Rep. Kho’s tender of resignation
was made pursuant to the agreement entered into by the organization’s nominees. 20 However, said counsel also stated
that the Board of Trustees of the organization reconsidered the acceptance of Rep. Kho’s resignation and the latter was,
instead, to complete his term.21 Also, from the transcript of the hearing, it appears that the Arquiza Group previously
manifested that it was withdrawing its petition, but the same was opposed by the Datol Group and was not acted upon by
the COMELEC.22

On June 27, 2012, the COMELEC En Banc issued a Resolution 23 in E.M. No. 12-040, dismissing the petition of the
SENIOR CITIZENS (Arquiza Group). The pertinent portions of the Resolution stated, thus:

First, resignation of Kho,


pursuant to the party nominees’
term-sharing agreement, cannot
be recognized and be given effect
so as to create a vacancy in the
list and change the order of the
nominees.

Under Section 8 of Republic Act No. 7941, the withdrawal in writing of the nominee of his nomination is one of the three
(3) exemptions to the rule that "no change of names or alteration of the order of nominees shall be allowed after the same
shall have been submitted to the COMELEC." While we can consider the resignation of Rep. Kho as akin to the withdrawal
of his own nomination, we are constrained however NOT to recognize such resignation but only in so far as to change the
order of petitioner’s nominees as submitted to the Commission.

xxxx

Considering that it is an admitted fact that the resignation of Rep. Kho was made by virtue of a prior agreement of the
parties, we resolve and hereby rule that we cannot recognize such arrangement and accordingly we cannot approve the
movement in the order of nominees for being contrary to public policy. The term of office of public officials cannot be made
subject to any agreement of private parties. Public office is not a commodity that can be shared, apportioned or be made
subject of any private agreement. Public office is vested with public interest that should not be reined by individual interest.

In fact, to formalize the policy of disallowing term sharing agreements among party list nominees, the Commission recently
promulgated Resolution No. 9366, which provides:

"SEC. 7. Term sharing of nominees. – Filing of vacancy as a result of term sharing agreement among nominees of winning
party-list groups/organizations shall not be allowed."
Considering all these, we find the term sharing agreement by the nominees of the Senior Citizen’s Party-List null and void.
Any action committed by the parties in pursuit of such term-sharing arrangement—including the resignation of
Congressman David Kho—cannot be recognized and be given effect. Thus, in so far as this Commission is concerned, no
vacancy was created by the resignation of Rep. Kho and there can be no change in the list and order of nominees of the
petitioner party-list.

Second, the expulsion of Datol –


even if proven true – has no effect
in the list and in the order of
nominees, thus Remedios Arquiza
(the fourth nominee) cannot be
elevated as the third nominee.

xxxx

It must be noted that the list and order of nominees, after submission to this Commission, is meant to be permanent. The
legislature in crafting Republic Act No. 7941 clearly deprived the party-list organization of the right to change its nominees
or to alter the order of nominees once the list is submitted to the COMELEC, except for three (3) enumerated instances
such as when: (a) the nominee dies; (b) the nominee withdraws in writing his nomination; or (c) the nominee becomes
incapacitated.

xxxx

Thus, even if the expulsion of Datol in the petitioner party-list were true, the list and order of nominees of the Senior
Citizen’s party-list remains the same in so far as we are concerned as it does not fall under one of the three grounds
mentioned above. Neither does it have an automatic effect on the organization’s representative in the House of
Representatives, for once a party-list nominee is "elected" into office and becomes a member of the House, he is treated
similarly and equally with the regular district representatives. As such, they can only be expelled or suspended upon the
concurrence of the two-thirds of all its Members and never by mere expulsion of a party-list organization.

xxxx

WHEREFORE, there being no vacancy in the list of nominees of the petitioner organization, the instant petition is hereby
DISMISSED for lack of merit. The list and order of nominees of petitioner hereby remains the same as it was submitted to
us there being no legally recognizable ground to cause any changes thereat. 24 (Citation omitted.)

The Datol Group filed A Very Urgent Motion for Reconsideration 25 of the above resolution, but the same remained
unresolved.

The Review of SENIOR CITIZENS’ Registration

Meanwhile, the Datol Group and the Arquiza Group filed their respective Manifestations of Intent to Participate in the
Party-list System of Representation in the May 13, 2013 Elections under the name of SENIOR CITIZENS. 26 The
Manifestation of the Datol Group was docketed as SPP

No. 12-157 (PLM), while that of the Arquiza Group was docketed as SPP No. 12-191 (PLM).

On August 2, 2012, the COMELEC issued Resolution No. 9513, 27 which, inter alia, set for summary evidentiary hearings
by the COMELEC En Banc the review of the registration of existing party-list organizations, which have filed their
Manifestations of Intent to Participate in the Party-list System of Representation in the May 13, 2013 Elections.

The two factions of SENIOR CITIZENS appeared before the COMELEC En Banc on August 24, 2012 and they both
submitted their respective evidence, which established their continuing compliance with the requirements of accreditation
as a party-list organization.28

On December 4, 2012, the COMELEC En Banc issued a Resolution 29 in SPP Nos. 12-157 (PLM) and 12-191 (PLM). By a
vote of 4-3, the COMELEC En Banc ordered the cancellation of the registration of SENIOR CITIZENS. The resolution
explained that:

It shall be recalled that on June 27, 2012, this Commission promulgated its resolution in a petition that involved SENIOR
CITIZENS titled "In Re: Petition for Confirmation of Replacement of Resigned PartyList Nominee" and docketed as EM
No. 12-040. In the process of resolving the issues of said case, this Commission found that SENIOR CITIZENS nominees
specifically nominees David L. Kho and Francisco G. Datol, Jr. have entered into a term-sharing agreement. x x x.

Nominee David Kho’s term as party-list congressman is three (3) years which starts on June 30, 2010 and to end on June
30, 2013 as directed no less than by the Constitution of the Philippines. Section 7, Article VI of the 1987 Constitution
states:

"Sec. 7. The Members of the House of Representatives shall be elected for a term of three years which shall begin, unless
otherwise provided by law, at noon on the thirtieth day of June next following their election."
But following the term-sharing agreement entered into by SENIOR CITIZENS, David Kho’s term starts on June 30, 2010
and ends on December 31, 2011, the date of effectivity of Kho’s resignation. By virtue of the term-sharing agreement, the
term of Kho as member of the House of Representatives is cut short to one year and six months which is merely half of
the three-year term. This is totally opposed to the prescription of the Constitution on the term of a Member of the House of
Representatives. Hence, when confronted with this issue on term sharing done by SENIOR CITIZENS, this Commission
made a categorical pronouncement that such term-sharing agreement must be rejected.

xxxx

From the foregoing, SENIOR CITIZENS failed to comply with Section 7, Article VI of the 1987 Constitution and Section 7,
Rule 4 of Comelec Resolution No. 9366. This failure is a ground for cancellation of registration under Section 6 of Republic
Act No. 7941 which states:

"Section 6. Refusal and/or Cancellation of Registration. – The COMELEC may, motu proprio or upon verified complaint of
any interested party, refuse or cancel, after due notice and hearing, the registration of any national, regional or sectoral
party, organization or coalition on any of the following grounds:

xxxx

(5) It violates or fails to comply with laws, rules or regulations relating to elections;

xxxx

WHEREFORE, premises considered, the Commission RESOLVED, as it hereby RESOLVES, to CANCEL the registration
of Coalition of Associations of Senior Citizens in the Philippines (SENIOR CITIZENS) under the Party-List System of
Representation.

The rival factions of SENIOR CITIZENS challenged the above resolution before this Court by filing their respective
petitions for certiorari. The petition filed by the Datol Group was docketed as G.R. No. 204421, while the petition of the
Arquiza Group was docketed as G.R. No. 204425.

On December 11, 2012, the Court initially granted status quo ante orders on said petitions, directing the COMELEC to
include the name of SENIOR CITIZENS in the printing of official ballots for the May 13, 2013 party-list elections.
Eventually, both petitions were consolidated with the petition in Atong Paglaum, Inc. v. Commission on Elections, which
was docketed as G.R. No. 203766.

On April 2, 2013, the Court promulgated its Decision in Atong Paglaum, which ordered the remand to the COMELEC of
the petitions that have been granted mandatory injunctions to include the names of the petitioners in the printing of ballots.
Following the parameters set forth in the Court’s Decision, the COMELEC was to determine whether said petitioners,
which included the two factions of SENIOR CITIZENS, were qualified to register under the party-list system and to
participate in the May 13, 2013 elections. For this purpose, the Court stated that the COMELEC may conduct summary
evidentiary hearings.

Thereafter, on May 10, 2013, the COMELEC En Banc rendered the assailed Omnibus Resolution in SPP Nos. 12-157
(PLM) and 12-191 (PLM), ruling in this wise:

Guided by these six new parameters [enunciated by the Court in Atong Paglaum, Inc. v. Commission on Elections], as
well as the provisions of the Constitution, Republic Act No. 7941 ("R.A. No. 7941") or the Party-List System Act, and other
pertinent election laws, and after a careful and exhaustive reevaluation of the documents submitted by the petitioners per
their compliance with Resolution No. 9513 ("Res. No. 9513"), the Commission En Banc RESOLVES as follows:

I. SPP Nos. 12-157 (PLM) & 12-191 (PLM) – SENIOR CITIZENS

To DENY the Manifestations of Intent to Participate, and to CANCEL the registration and accreditation, of petitioner Senior
Citizens, for violating laws, rules, and regulations relating to elections pursuant to Section 6 (5) of R.A. No. 7941.

The Commission En Banc finds no cogent reason to reverse its earlier finding in the Resolution for SPP Nos. 12-157
(PLM) & 12-191 (PLM) promulgated on 04 December 2012, in relation to the Resolution for E.M. No. 12-040 promulgated
on 27 June 2012. The sole ground for which the petitioner Senior Citizens was disqualified was because of the term-
sharing agreement between its nominees, which the Commission En Banc found to be contrary to public policy. It will be
noted that this ground is independent of the six parameters in Atong Paglaum, and there is nothing in the doctrine
enunciated in that case which will absolve the petitioner Senior Citizen of what, to the Commission En Banc, is a clear
bastardization of the term of office fixed by Section 7, Article VI of the Constitution as implemented by Section 14 of R.A.
No. 7941, which expressly provides that Members of the House of Representatives, including party-list representatives,
shall be elected for a term of three years. A term, in the legal sense, is a fixed and definite period of time during which an
officer may claim to hold office as a matter of right, a fixed interval after which the several incumbents succeed one
another. Thus, service of the term is for the entire period; it cannot be broken down to accommodate those who are not
entitled to hold the office.

That the term-sharing agreement was made in 2010, while the expression of the policy prohibiting it was promulgated only
in 2012 via Section 7, Rule 4 of Resolution No. 9366 ("Res. No. 9366"), is of no moment. As it was in 2010 as it is now, as
it was in 1987 when the Constitution was ratified and as it was in 1995 when R.A. No. 7941 was enacted into law, the
agreement was and is contrary to public policy because it subjects a Constitutionally-ordained fixed term to hold public
elective office to contractual bargaining and negotiation, and treats the same as though it were nothing more than a
contractual clause, an object in the ordinary course of the commerce of men. To accept this defense will not only open the
floodgates to unscrupulous individuals, but more importantly it will render inutile Section 16 of R.A. No. 7941 which
prescribes the procedure to be taken to fill a vacancy in the available seats for a party-list group or organization. For this
mistake, the petitioner Senior Citizens cannot hide behind the veil of corporate fiction because the corporate veil can be
pierced if necessary to achieve the ends of justice or equity, such as when it is used to defeat public convenience, justify
wrong, or protect fraud. It further cannot invoke the prohibition in the enactment of ex post facto laws under Section 22,
Article III of the Constitution because the guarantee only the retrospectivity of penal laws and definitely, Reso. No. 9366 is
not penal in character.

From the foregoing, the cancellation of the registration and accreditation of the petitioner Senior Citizens is therefore in
order, and consequently, the two Manifestations of Intent to Participate filed with the Commission should be denied.

xxxx

WHEREFORE, the Commission En Banc RESOLVES:

A. To DENY the Manifestations of Intent to Participate, and CANCEL the registration and accreditation, of the following
parties, groups, or organizations:

(1) SPP No. 12-157 (PLM) & SPP No. 12-191 (PLM) – Coalition of Associations of Senior Citizens in the Philippines, Inc.;

xxxx

Accordingly, the foregoing shall be REMOVED from the registry of party-list groups and organizations of the Commission,
and shall NOT BE ALLOWED to PARTICIPATE as a candidate for the Party-List System of Representation for the 13 May
2013 Elections and subsequent elections thereafter. 30 (Citations omitted.)

On May 13, 2013, the elections proceeded. Despite the earlier declaration of its disqualification, SENIOR CITIZENS still
obtained 677,642 votes.

Questioning the cancellation of SENIOR CITIZENS’ registration and its disqualification to participate in the May 13, 2013
elections, the Datol Group and the Arquiza Group filed the instant petitions.

On May 15, 2013, the Datol Group filed a Very2 Urgent Motion to Reiterate Issuance of Temporary Restraining Order
and/or Status Quo Ante Order,31 alleging that the COMELEC had ordered the stoppage of the counting of votes of the
disqualified party-list groups. The Datol Group urged the Court to issue a TRO and/or a status quo ante order during the
pendency of its petition.

Meanwhile, on May 24, 2013, the COMELEC En Banc issued a Resolution, 32 which considered as final and executory its
May 10, 2013 Resolution that cancelled the registration of SENIOR CITIZENS. On even date, the COMELEC En Banc,
sitting as the National Board of Canvassers (NBOC), promulgated NBOC Resolution No. 0006-13, 33 proclaiming fourteen
(14) party-list organizations as initial winners in the party-list elections of May 13, 2013.

The Arquiza Group filed on May 27, 2013 a Supplement to the "Very Urgent Petition for Certiorari," 34 also reiterating its
application for a TROand a writ of preliminary injunction.

On May 28, 2013, the COMELEC En Banc issued NBOC Resolution No. 0008-13, 35 which partially proclaimed the winning
party-list organizations that filled up a total of fifty-three (53) out of the available fifty-eight (58) seats for party-list
organizations.

On May 29, 2013, the Chief Justice issued a TRO,36 which ordered the COMELEC to submit a Comment on the instant
petitions and to cease and desist from further proclaiming the winners from among the party-list candidates in the May 13,
2013 elections.

On June 3, 2013, the Datol Group filed a Most Urgent Motion for Issuance of an Order Directing Respondent to Proclaim
Petitioner Pendente Lite.37

In a Resolution38 dated June 5, 2013, the Court issued an order, which directed the COMELEC to refrain from
implementing the assailed Omnibus Resolution dated May 10, 2013 in SPP No. 12-157 (PLM) and SPP No. 12-191
(PLM), insofar as SENIOR CITIZENS was concerned and to observe the status quo ante before the issuance of the
assailed COMELEC resolution. The Court likewise ordered the COMELEC to reserve the seat(s) intended for SENIOR
CITIZENS, in accordance with the number of votes it garnered in the May 13, 2013 Elections. The Court, however,
directed the COMELEC to hold in abeyance the proclamation insofar as SENIOR CITIZENS is concerned until the instant
petitions are decided. The Most Urgent Motion for Issuance of an Order Directing Respondent to Proclaim Petitioner
Pendente Lite filed by the Datol Group was denied for lack of merit.

On June 7, 2013, the COMELEC, through the Office of the Solicitor General (OSG), filed a Comment 39 on the instant
petitions. In a Resolution40 dated June 10, 2013, the Court required the parties to submit their respective memoranda. On
June 19, 2013, the Arquiza Group filed its Reply41 to the Comment of the COMELEC. Subsequently, the Datol Group and
the Arquiza Group filed their separate memoranda. 42 On the other hand, the OSG manifested43 that it was adopting its
Comment as its memorandum in the instant case.

THE ISSUES

The Datol Group’s memorandum raised the following issues for our consideration:

IV. STATEMENT OF THE ISSUES

4.1

WHETHER OR NOT RESPONDENT COMELEC COMMITTED GRAVE ABUSE OF DISCRETION AMOUNTING TO


LACK OR EXCESS OF JURISDICTION WHEN IT ADDED ANOTHER GROUND (VIOLATION OF PUBLIC POLICY) FOR
CANCELLATION OF REGISTRATION OF A PARTY–LIST GROUP AS PROVIDED UNDER SECTION 6, REPUBLIC
ACT NO. 7941.

4.2

WHETHER OR NOT RESPONDENT COMELEC COMMITTED GRAVE ABUSE OF DISCRETION AMOUNTING TO


LACK OR EXCESS OF JURISDICTION WHEN IT CANCELLED PETITIONER’S CERTIFICATE OF
REGISTRATION/ACCREDITATION WITHOUT DUE PROCESS OF LAW.

4.3

WHETHER OR NOT RESPONDENT COMELEC COMMITTED GRAVE ABUSE OF DISCRETION AMOUNTING TO


LACK OR EXCESS OF JURISDICTION WHEN IT CONCLUDED THAT PETITIONER VIOLATED PUBLIC POLICY ON
TERM SHARING.

4.4

WHETHER OR NOT RESPONDENT COMELEC COMMITTED GRAVE ABUSE OF DISCRETION AMOUNTING TO


LACK OR EXCESS OF JURISDICTION WHEN IT ORDERED THE AUTOMATIC REVIEW BY THE EN BANC OF THE
REGISTRATION/ACCREDITATION GRANTED BY ITS DIVISION, NOTWITHSTANDING THE CONSTITUTIONAL
PROVISION THAT THE EN BANC CAN ONLY REVIEW DECISIONS OF THE DIVISION UPON FILING OF A MOTION
FOR RECONSIDERATION.44 (Citation omitted.)

Upon the other hand, the memorandum of the Arquiza Group brought forward the following arguments:

4.1. Whether or not COMELEC EN BANC RESOLUTION of MAY 10, 2013 is invalid for being contrary to law and
having been issued without or in excess of jurisdiction or in grave abuse of discretion amounting to lack of
jurisdiction?

(1) The Comelec En Banc Resolution of May 10, 2013 was issued pursuant to the directive of the Supreme
Court in Atong Paglaum. Therefore, the SUBSIDIARY ISSUES arising therefrom are:

a. Are there guidelines prescribed in Atong Paglaum to be followed by respondent Comelec in


determining which partylist groups are qualified to participate in party-list elections?

b. If there are these guidelines to be followed, were these adhered to by respondent Comelec?

(2) Is the ground -- the Term-Sharing Agreement between Senior Citizens nominees -- a legal ground to
cancel Senior Citizens’ Certificate of Registration?

4.2. Whether or not COMELEC EN BANC RESOLUTION of MAY 24, 2013 is invalid for being contrary to law and
having been issued without or in excess of jurisdiction or in grave abuse of discretion amounting to lack of
jurisdiction?

(1) The SUBSIDIARY ISSUES are:

a. Is the factual basis thereof valid?

b. Has the Comelec En Banc Resolution of May 20, 2013, in fact, become final and executory?

4.3. Whether or not NATIONAL BOARD of CANVASSERS’ (NBOC) RESOLUTION No. 0006-13 of MAY 24, 2013
is invalid for being contrary to law and having been issued without or in excess of jurisdiction or grave abuse of
discretion amounting to lack of jurisdiction?

(1) The SUBSIDIARY ISSUES are:


a. Is the factual basis thereof valid?

b. Is the total of the party-list votes cast which was made as the basis thereof correct?

c. Has the Justice Carpio Formula prescribed in Banat vs. Comelec been followed?

4.4. Whether or not NBOC RESOLUTION No. 0008-13 of MAY 28, 2013 is invalid for being contrary to law and
having been issued without or in excess of jurisdiction or in grave abuse of discretion amounting to lack of
jurisdiction?

(1) The SUBSIDIARY ISSUES are identical with those of Issue No. 4.3, namely:

a. Is the factual basis thereof valid?

b. Is the total of the party-list votes cast which was made as the basis thereof correct?

c. Has the Justice Carpio Formula prescribed in Banat vs. Comelec been followed?

4.5. What is the cardinal rule in interpreting laws/rules on qualifications and disqualifications of the candidates after
the election where they have received the winning number of votes?

4.6. May the COMELEC En Banc Resolutions of May 10 and 24, 2013 and NBOC Resolutions of May 24 and 28,
2013 be annulled and set aside?45

THE COURT’S RULING

After reviewing the parties’ pleadings, as well as the various resolutions attached thereto, we find merit in the petitioners’
contentions.1âwphi1

SENIOR CITIZENS’ Right to Due Process

First, we shall dispose of the procedural issue. In their petitions, the two rival groups of SENIOR CITIZENS are actually
one in asserting that the organization’s disqualification and cancellation of its registration and accreditation were effected
in violation of its right to due process.

The Arquiza Group argues that no notice and hearing were given to SENIOR CITIZENS for the cancellation of its
registration on account of the term-sharing agreement of its nominees. The Arquiza Group maintains that SENIOR
CITIZENS was summoned only to a single hearing date in the afternoon of August 24, 2012 and the COMELEC’s review
therein focused on the group’s programs, accomplishments, and other related matters. The Arquiza Group asserts that
SENIOR CITIZENS was not advised, before or during the hearing, that the issue of the term-sharing agreement would
constitute a basis for the review of its registration and accreditation.

Likewise, the Datol Group faults the COMELEC for cancelling the registration and accreditation of SENIOR CITIZENS
without giving the latter the opportunity to show that it complied with the parameters laid down in Atong Paglaum. The
Arquiza Group confirms that after the promulgation of Atong Paglaum, the COMELEC conducted summary hearings in
executive sessions, without informing SENIOR CITIZENS. The Arquiza Group says that it filed a "Very Urgent Motion To
Set Case For Hearing Or To Be Included In The Hearing Set On Thursday, May 9, 2013," but its counsel found that
SENIOR CITIZENS was not included in the hearings wherein other party-list groups were heard by the COMELEC. The
Arquiza Group subsequently filed on May 10, 2013 a "2nd Very Urgent Motion To Set Case For Public Hearing," but the
same was also not acted upon. The Arquiza Group alleges that it only found out after the elections that the assailed May
10, 2013 Omnibus Resolution was issued and the Arquiza Group was not actually served a copy thereof.

Section 6 of Republic Act No. 7941 46 provides for the procedure relative to the review of the registration of party-list
organizations, to wit:

SEC. 6. Refusal and/or Cancellation of Registration. – The COMELEC may, motu proprio or upon verified complaint of any
interested party, refuse or cancel, after due notice and hearing, the registration of any national, regional or sectoral party,
organization or coalition on any of the following grounds:

(1) It is a religious sect or denomination, organization or association organized for religious purposes;

(2) It advocates violence or unlawful means to seek its goal;

(3) It is a foreign party or organization;

(4) It is receiving support from any foreign government, foreign political party, foundation, organization, whether
directly or through any of its officers or members or indirectly through third parties for partisan election purposes;

(5) It violates or fails to comply with laws, rules or regulations relating to elections;
(6) It declares untruthful statements in its petition;

(7) It has ceased to exist for at least one (1) year; or

(8) It fails to participate in the last two (2) preceding elections or fails to obtain at least two per centum (2%) of the
votes cast under the party-list system in the two (2) preceding elections for the constituency in which it has
registered.

Unquestionably, the twin requirements of due notice and hearing are indispensable before the COMELEC may properly
order the cancellation of the registration and accreditation of a party-list organization. In connection with this, the Court
lengthily discussed in Mendoza v. Commission on Elections 47 the concept of due process as applied to the COMELEC. We
emphasized therein that:

The appropriate due process standards that apply to the COMELEC, as an administrative or quasi-judicial tribunal, are
those outlined in the seminal case of Ang Tibay v. Court of Industrial Relations, quoted below:

(1) The first of these rights is the right to a hearing, which includes the right of the party interested or affected to
present his own case and submit evidence in support thereof. x x x.

(2) Not only must the party be given an opportunity to present his case and to adduce evidence tending to
establish the rights which he asserts but the tribunal must consider the evidence presented.

(3) While the duty to deliberate does not impose the obligation to decide right, it does imply a necessity which
cannot be disregarded, namely, that of having something to support its decision. A decision with absolutely nothing
to support it is a nullity, a place when directly attached.

(4) Not only must there be some evidence to support a finding or conclusion, but the evidence must be
"substantial." "Substantial evidence is more than a mere scintilla. It means such relevant evidence as a reasonable
mind might accept as adequate to support a conclusion."

(5) The decision must be rendered on the evidence presented at the hearing, or at least contained in the record
and disclosed to the parties affected.

(6) The Court of Industrial Relations or any of its judges, therefore, must act on its or his own independent
consideration of the law and facts of the controversy, and not simply accept the views of a subordinate in arriving
at a decision.

(7) The Court of Industrial Relations should, in all controversial questions, render its decision in such a manner
that the parties to the proceeding can know the various issues involved, and the reasons for the decisions
rendered. The performance of this duty is inseparable from the authority conferred upon it.

These are now commonly referred to as cardinal primary rights in administrative proceedings.

The first of the enumerated rights pertain to the substantive rights of a party at hearing stage of the proceedings. The
essence of this aspect of due process, we have consistently held, is simply the opportunity to be heard, or as applied to
administrative proceedings, an opportunity to explain one’s side or an opportunity to seek a reconsideration of the action
or ruling complained of. A formal or trial-type hearing is not at all times and in all instances essential; in the case of
COMELEC, Rule 17 of its Rules of Procedure defines the requirements for a hearing and these serve as the standards in
the determination of the presence or denial of due process.

The second, third, fourth, fifth, and sixth aspects of the Ang Tibay requirements are reinforcements of the right to a hearing
and are the inviolable rights applicable at the deliberative stage, as the decision-maker decides on the evidence presented
during the hearing. These standards set forth the guiding considerations in deliberating on the case and are the material
and substantial components of decision-making. Briefly, the tribunal must consider the totality of the evidence presented
which must all be found in the records of the case (i.e., those presented or submitted by the parties); the conclusion,
reached by the decision-maker himself and not by a subordinate, must be based on substantial evidence.

Finally, the last requirement, relating to the form and substance of the decision of a quasi-judicial body, further
complements the hearing and decision-making due process rights and is similar in substance to the constitutional
requirement that a decision of a court must state distinctly the facts and the law upon which it is based. As a component of
the rule of fairness that underlies due process, this is the "duty to give reason" to enable the affected person to understand
how the rule of fairness has been administered in his case, to expose the reason to public scrutiny and criticism, and to
ensure that the decision will be thought through by the decision-maker. (Emphases ours, citations omitted.)

In the instant case, the review of the registration of SENIOR CITIZENS was made pursuant to COMELEC Resolution No.
9513 through a summary evidentiary hearing carried out on August 24, 2012 in SPP No. 12-157 (PLM) and SPP No. 12-
191 (PLM). In this hearing, both the Arquiza Group and the Datol Group were indeed given the opportunity to adduce
evidence as to their continuing compliance with the requirements for party-list accreditation. Nevertheless, the due
process violation was committed when they were not apprised of the fact that the term-sharing agreement entered into by
the nominees of SENIOR CITIZENS in 2010 would be a material consideration in the evaluation of the organization’s
qualifications as a party-list group for the May 13, 2013 elections. As it were, both factions of SENIOR CITIZENS were not
able to answer this issue squarely. In other words, they were deprived of the opportunity to adequately explain their side
regarding the term-sharing agreement and/or to adduce evidence, accordingly, in support of their position.

In its Comment48 to the petitions, the COMELEC countered that petitioners were actually given the opportunity to present
their side on the issue of the term-sharing agreement during the hearing on April 18, 2012. 49 Said hearing was allegedly
conducted to determine petitioners’ continuing compliance for accreditation as a party-list organization.

The Court is not persuaded. It is true that during the April 18, 2012 hearing, the rival groups of SENIOR CITIZENS
admitted to the existence of the term-sharing agreement. Contrary to the claim of COMELEC, however, said hearing was
conducted for purposes of discussing the petition of the Arquiza Group in E.M. No. 12-040. To recall, said petition asked
for the confirmation of the replacement of Rep. Kho, who had tendered his resignation effective on December 31, 2011.
More specifically, the transcript of the hearing reveals that the focus thereof was on the petition filed by the Arquiza group
and its subsequent manifestation, praying that the group be allowed to withdraw its petition. Also, during the hearing,
COMELEC Chairman Brillantes did admonish the rival factions of SENIOR CITIZENS about their conflicts and warned
them about the complications brought about by their term-sharing agreement. However, E.M. No. 12-040 was not a
proceeding regarding the qualifications of SENIOR CITIZENS as a party-list group and the issue of whether the term-
sharing agreement may be a ground for disqualification was neither raised nor resolved in that case. Chairman Brillantes’s
remonstration was not sufficient as to constitute a fair warning that the term-sharing agreement would be considered as a
ground for the cancellation of SENIOR CITIZENS’ registration and accreditation.

Furthermore, after the promulgation of Atong Paglaum, which remanded, among other cases, the disqualification cases
involving SENIOR CITIZENS, said organization should have still been afforded the opportunity to be heard on the matter
of the term-sharing agreement, either through a hearing or through written memoranda. This was the proper recourse
considering that the COMELEC was about to arrive at a final determination as to the qualification of SENIOR CITIZENS.
Instead, the COMELEC issued the May 10, 2013 Omnibus Resolution in SPP No. 12-157 (PLM) and SPP No. 12-191
(PLM) without conducting any further proceedings thereon after its receipt of our Decision in Atong Paglaum.

The Prohibition on Term-sharing

The second issue both raised by the petitioners herein constitute the threshold legal issue of the instant cases: whether
the COMELEC committed grave abuse of discretion amounting to lack or excess of jurisdiction when it issued the assailed
Omnibus Resolution, disqualifying and cancelling the registration and accreditation of SENIOR CITIZENS solely on
account of its purported violation of the prohibition against term-sharing.

The Datol Group argues that the public policy prohibiting term-sharing was provided for under Section 7, Rule 4 of
COMELEC Resolution No. 9366, which was promulgated only on February 21, 2012. Hence, the resolution should not be
made to apply retroactively to the case of SENIOR CITIZENS as nothing therein provides for its retroactive effect. When
the term-sharing agreement was executed in 2010, the same was not yet expressly proscribed by any law or resolution.

Furthermore, the Datol Group points out that the mere execution of the Irrevocable Covenant between the nominees of
SENIOR CITIZENS for the 2010 elections should not have been a ground for the cancellation of the organization’s
registration and accreditation because the nominees never actually implemented the agreement.

In like manner, the Arquiza Group vehemently stresses that no term-sharing actually transpired between the nominees of
SENIOR CITIZENS. It explained that whatever prior arrangements were made by the nominees on the term-sharing
agreement, the same did not materialize given that the resignation of Rep. Kho was disapproved by the Board of Trustees
and the members of SENIOR CITIZENS.

Still, granting for the sake of argument that the term-sharing agreement was actually implemented, the Arquiza Group
points out that SENIOR CITIZENS still cannot be held to have violated Section 7 of Resolution No. 9366. The term-sharing
agreement was entered into in 2010 or two years prior to the promulgation of said resolution on February 21, 2012.
Likewise, assuming that the resolution can be applied retroactively, the Arquiza Group contends that the same cannot
affect SENIOR CITIZENS at it already earned a vested right in 2010 as party-list organization.

Article 4 of the Civil Code states that "laws shall have no retroactive effect, unless the contrary is provided." As held in
Commissioner of Internal Revenue v. Reyes,50 "the general rule is that statutes are prospective. However, statutes that are
remedial, or that do not create new or take away vested rights, do not fall under the general rule against the retroactive
operation of statutes." We also reiterated in Lintag and Arrastia v. National Power Corporation 51 that:

It is a well-entrenched principle that statutes, including administrative rules and regulations, operate prospectively unless
the legislative intent to the contrary is manifest by express terms or by necessary implication because the retroactive
application of a law usually divests rights that have already become vested. This is based on the Latin maxim: Lex
prospicit non respicit (the law looks forward, not backward). (Citations omitted.)

True, COMELEC Resolution No. 9366 does not provide that it shall have retroactive effect. Nonetheless, the Court cannot
subscribe to the argument of the Arquiza Group that SENIOR CITIZENS already earned a vested right to its registration as
a party-list organization.

Montesclaros v. Commission on Elections52 teaches that "a public office is not a property right. As the Constitution
expressly states, a ‘Public office is a public trust.’ No one has a vested right to any public office, much less a vested right
to an expectancy of holding a public office." Under Section 2(5), Article IX-C of the Constitution, the COMELEC is
entrusted with the function to "register, after sufficient publication, political parties, organizations, or coalitions which, in
addition to other requirements, must present their platform or program of government." In fulfilling this function, the
COMELEC is duty-bound to review the grant of registration to parties, organizations, or coalitions already registered in
order to ensure the latter’s continuous adherence to the requirements prescribed by law and the relevant rulings of this
Court relative to their qualifications and eligibility to participate in party-list elections.

The Arquiza Group cannot, therefore, object to the retroactive application of COMELEC Resolution No. 9366 on the
ground of the impairment of SENIOR CITIZENS’ vested right.

Be that as it may, even if COMELEC Resolution No. 9366 expressly provided for its retroactive application, the Court finds
that the COMELEC En Banc indeed erred in cancelling the registration and accreditation of SENIOR CITIZENS.

The reason for this is that the ground invoked by the COMELEC En Banc, i.e., the term-sharing agreement among the
nominees of SENIOR CITIZENS, was not implemented. This fact was manifested by the Arquiza Group even during the
April 18, 2012 hearing conducted by the COMELEC En Banc in E.M. No. 12-040 wherein the Arquiza Group manifested
that it was withdrawing its petition for confirmation and approval of Rep. Kho’s replacement. Thereafter, in its Resolution
dated June 27, 2012 in E.M. No. 12-040, the COMELEC En Banc itself refused to recognize the term-sharing agreement
and the tender of resignation of Rep. Kho. The COMELEC even declared that no vacancy was created despite the
execution of the said agreement. Subsequently, there was also no indication that the nominees of SENIOR CITIZENS still
tried to implement, much less succeeded in implementing, the term-sharing agreement. Before this Court, the Arquiza
Group and the Datol Group insist on this fact of non-implementation of the agreement. Thus, for all intents and purposes,
Rep. Kho continued to hold his seat and served his term as a member of the House of Representatives, in accordance
with COMELEC Resolution No. 9366 and the COMELEC En Banc ruling in E.M. No. 12-040. Curiously, the COMELEC is
silent on this point.

Indubitably, if the term-sharing agreement was not actually implemented by the parties thereto, it appears that SENIOR
CITIZENS, as a party-list organization, had been unfairly and arbitrarily penalized by the COMELEC En Banc. Verily, how
can there be disobedience on the part of SENIOR CITIZENS when its nominees, in fact, desisted from carrying out their
agreement? Hence, there was no violation of an election law, rule, or regulation to speak of. Clearly then, the
disqualification of SENIOR CITIZENS and the cancellation of its registration and accreditation have no legal leg to stand
on.

In sum, the due process violations committed in this case and the lack of a legal ground to disqualify the SENIOR
CITIZENS spell out a finding of grave abuse of discretion amounting to lack or excess of jurisdiction on the part of the
COMELEC En Banc. We are, thus, left with no choice but to strike down the assailed Omnibus Resolution dated May 10,
2013 in SPP No. 12-157 (PLM) and SPP No. 12-191 (PLM).

In light of the foregoing discussion, the Court finds no need to discuss the other issues raised by the petitioners. In
particular, the dispute between the rival factions of SENIOR CITIZENS, not being an issue raised here, should be
threshed out in separate proceedings before the proper tribunal having jurisdiction thereon.

Having established that the COMELEC En Banc erred in ordering the disqualification of SENIOR CITIZENS and the
cancellation of its registration and accreditation, said organization is entitled to be proclaimed as one of the winning party-
list organizations in the recently concluded May 13, 2013 elections.

WHEREFORE, the Court hereby rules that:

(1) The Extremely Very Urgent Petition for Certiorari (With Prayer for the Forthwith Issuance of a Writ of
Preliminary Injunction and Temporary Restraining Order [TRO] and/or Status Quo Ante Order [SQAO]) in G.R.
Nos. 206844-45 and the Very Urgent Petition for Certiorari (With Application for a Temporary Restraining Order
and Writ of Preliminary Injunction) in G.R. No. 206982 are GRANTED;

(2) The Omnibus Resolution dated May 10, 2013 of the Commission on Elections En Banc in SPP No. 12-157
(PLM) and SPP No. 12-191 (PLM) is REVERSED and SET ASIDE insofar as Coalition of Associations of Senior
Citizens in the Philippines, Inc. is concerned; and

(3) The Commission on Elections En Bane is ORDERED to PROCLAIM the Coalition of Associations of Senior
Citizens in the Philippines, Inc. as one of the winning party-list organizations during the May 13, 20 13 elections
with the number of seats it may be entitled to based on the total number of votes it garnered during the said
elections.

No costs. SO ORDERED.

- Lico v Comelec, GR 205505, Sep 29, 2015

The pivotal and interrelated issues before Us in this case involve the seemingly elementary matter of the
Commission on Elections' (COMELEC) jurisdiction over the expulsion of a sitting party-list representative:
from the House of Representatives, on the one hand; and from his party-list organization, on the other.

The instant case involves two rival factions of the same party-list organization, the Adhikaing Tinataguyod
ng Kooperatiba (Ating Koop). One group is headed by petitioner Atty. Isidro Q. Lico (the Lico Group), who
represents the organization in the House of Representatives, and the other group by Amparo T. Rimas
(respondents herein, or the Rimas Group).

THE CASE

Before Us is a Petition for Certiorari under Rule 641 in relation to Rule 65,2 seeking to annul the
Resolutions in E.M. No. 12-039 dated 18 July 2012 and 31 January 2013 of the COMELEC.

THE ANTECEDENT FACTS

Ating Koop is a multi-sectoral party-list organization which was registered on 16 November 2009 under
Republic Act (R.A.) No. 7941, also known as the Party-List System Act (Party-List Law).

Under Ating Koop's Constitution and By-Laws, its highest policymaking body is the National Convention.
The Central Committee, however, takes over when the National Convention is not in session. 3

On 30 November 2009, Ating Koop filed its Manifestation of Intent to Participate in the Party-List System
of Representation for the 10 May 2010 Elections. 4 On 6 March 2010, it filed with the COMELEC the list of
its nominees, with petitioner Lico as first nominee and Roberto Mascarina as second nominee.

On 8 December 2010, COMELEC proclaimed Ating Koop as one of the winning party-list groups. 5 Based on
the procedure provided in BANAT Party-List v. COMELEC,6 Ating Koop earned a seat in the House of
Representatives. Petitioner Lico subsequently took his oath of office on 9 December 2010 before the
Secretary-General of the House of Representatives, 7 and thereafter assumed office.

Several months prior to its proclamation as one of the winning party-list organizations, or on 9 June 2010,
Ating Koop issued Central Committee Resolution 2010-01, which incorporated a term-sharing agreement
signed by its nominees.8 Under the agreement, petitioner Lico was to serve as Party-list Representative for
the first year of the three-year term.9

On 14 May 2011, Ating Koop held its Second National Convention, during which it introduced amendments
to its Constitution and By-laws. Among the salient changes was the composition of the Central
Committee,10 which would still be composed of 15 representatives but with five each coming from Luzon,
Visayas and Mindanao (5-5-5 equal representation).11 The amendments likewise mandated the holding of
an election of Central Committee members within six months after the Second National Convention. 12

In effect, the amendments cut short the three-year term of the incumbent members (referred to hereafter
as the Interim Central Committee) of the Central Committee. 13 The Interim Central Committee was
dominated by members of the Rimas Group.

On 5 December 2011, or almost one year after petitioner Lico had assumed office, the Interim Central
Committee expelled him from Ating Koop for disloyalty. 14 Apart from allegations of malversation and graft
and corruption, the Committee cited petitioner Lico's refusal to honor the term-sharing agreement as
factual basis for disloyalty and as cause for his expulsion under Ating Koop's Amended Constitution and
By-laws.15

On 8 December 2011, Congressman Lico filed a Motion for Reconsideration with the Interim Central
Committee,16 which subsequently denied the same in a Resolution dated 29 December 2011. 17

While petitioner Lico's Motion for Reconsideration was pending, the Lico Group held a special meeting in
Cebu City (the Cebu meeting) on 19 December 2011. At the said meeting, new members of the Central
Committee, as well as a new set of officers, were elected. 18 The election was purportedly held for the
purpose of implementing the 5-5-5 equal representation amendment made during the Second National
Convention.19

On 21 January 2012, the Rimas Group held a Special National Convention in Parañaque City 20 (the
Parañaque convention), at which a new Central Committee and a new set of officers were
constituted.21 Members of the Rimas Group won the election and occupied all the corresponding seats.

PROCEEDINGS BEFORE THE COMELEC


SECOND DIVISION

On 16 March 2012, the Rimas Group, claiming to represent Ating Koop, filed with COMELEC a Petition
against petitioner Lico docketed as E.M. No. 12-039. 22 The said Petition, which was subsequently raffled to
the Second Division, prayed that petitioner Lico be ordered to vacate the office of Ating Koop in the House
of Representatives, and for the succession of the second nominee, Roberto Mascarina as Ating Koop's
representative in the House.

The Rimas Group thereafter filed an Amended Petition with the COMELEC on 14 May 2012, this time
impleading not only petitioner Lico but the entire Lico Group. The Amended Petition also prayed that the
COMELEC nullify the election conducted at the Cebu meeting and recognize the Paranaque convention.

In both the Petition and the Amended Petition, the Rimas Group alleged that Ating Koop had expelled
Congressman Lico for acts inimical to the party-list group, such as malversation, graft and corruption, and
that he had "boldly displayed his recalcitrance to honor party commitment to be upright and consistently
honest, thus violating basic principles of the Ating Koop." 23 The Amended Petition stated further that the
Cebu meeting held by the Lico Group violated notice and quorum requirements. 24

In a Resolution dated 18 July 2012,25 the COMELEC Second Division upheld the expulsion of petitioner Lico
from Ating Koop and declared Mascarina as the duly qualified nominee of the party-list group. 26 The
Second Division characterized the issue of the validity of the expulsion of petitioner Lico from Ating Koop
as an intra-party leadership dispute, which it could resolve as an incident of its power to register political
parties.27 chanroblesvirtuallawlibrary

PROCEEDINGS BEFORE THE COMELEC


EN BANC

Consequently, the Lico Group filed a Motion for Reconsideration from the Second Division's Resolution,
which the COMELEC En Banc denied on 31 January 2013. The dispositive portion of its Resolution reads: cralawlawlibrary

WHEREFORE, premises considered, the Commission (En Banc) RESOLVES, as it hereby RESOLVED, to:


a. DISMISS the instant Petition to Expel Respondent Atty. Isidro Q. Lico in the House of Representatives
and to Sanction the Immediate Succession of the Second Nominee of ATING KOOP Party List, Mr. Roberto
C. Mascarina as its Party Representative, for lack of jurisdiction; ChanRoblesVirtualawlibrary

b. UPHOLD the Expulsion of Respondent Atty. Isidro Lico from ATING KOOP Party-list Group; [and]

c. UPHOLD the ATING KOOP Party-list Group represented by its President, Amparo T. Rimas, as the
legitimate Party-list Group accredited by the Commission on Elections, to the exclusion of respondents
Atty. Isidro Q. Lico, Rafael A. Puentespina, Proculo T. Sarmen, Amelito L. Revuelta, William C. Ybanez,
Silverio J. Sanchez, Gloria G. Futalan, Hilario De Guzman, Eugene M. Pabualan, Rodolfo E. Perez, Hipolito
R. Quillan, Mario Arenas, Tirso C. Buenaventura, Lydia B. Tubella, and Jonathan Dequina. 28
chanrobleslaw

In arriving at its Resolution, the COMELEC En Banc held that it had no jurisdiction to expel Congressman
Lico from the House of Representatives, considering that his expulsion from Ating Koop affected his
qualifications as member of the House, and therefore it was the House of Representatives Electoral
Tribunal (HRET) that had jurisdiction over the Petition.

At the same time, the COMELEC upheld the validity of petitioner Lico's expulsion from Ating Koop,
explaining that when the Interim Central Committee ousted him from Ating Koop, the said Committee's
members remained in hold-over capacity even after their terms had expired; 29 and that the COMELEC was
not in a position to substitute its judgment for that of Ating Koop with respect to the cause of the
expulsion.30

Finally, the COMELEC En Banc recognized the Rimas Group as the legitimate representative of Ating Koop
considering that: 1) it found nothing in the records to show that the Lico Group made a valid call for the
special election of Central Committee members as required under the Amended Constitution and By-
Laws;31 2) there is nothing on record indicating that a minimum of 100 attended the Cebu meeting; 32 and
3) the Parañaque convention was in accordance with Ating Koop's Amended Constitution and By-Laws. 33

Hence, this Petition: the Lico Group now comes before Us, praying for a review of the COMELEC
Resolutions.

The Court's Ruling

On the COMELEC's jurisdiction over


the expulsion of a Member of the House
of Representatives from his party-list
organization
We find that while the COMELEC correctly dismissed the Petition to expel petitioner Lico from the House of
Representatives for being beyond its jurisdiction, it nevertheless proceeded to rule upon the validity of his
expulsion from Ating Koop - a matter beyond its purview.

The COMELEC notably characterized the Petition for expulsion of petitioner Lico from the House of
Representatives and for the succession of the second nominee as party-list representative as a
disqualification case. For this reason, the COMELEC dismissed the petition for lack of jurisdiction, insofar as
it relates to the question of unseating petitioner Lico from the House of Representatives.

Section 17, Article VI of the 1987 Constitution 34 endows the HRET with jurisdiction to resolve questions on
the qualifications of members of Congress. In the case of party-list representatives, the HRET acquires
jurisdiction over a disqualification case upon proclamation of the winning party-list group, oath of the
nominee, and assumption of office as member of the House of Representatives. 35 In this case, the
COMELEC proclaimed Ating Koop as a winning party-list group; petitioner Lico took his oath; and he
assumed office in the House of Representatives. Thus, it is the HRET, and not the COMELEC, that has
jurisdiction over the disqualification case.

What We find to be without legal basis, however, is the action of the COMELEC in upholding the validity of
the expulsion of petitioner Lico from Ating Koop, despite its own ruling that the HRET has jurisdiction over
the disqualification issue. These findings already touch upon the qualification requiring a party-list nominee
to be a bona fide member of the party-list group sought to be represented.

The COMELEC justified its Resolution on the merits of the expulsion, by relying on the rule that it can
decide intra-party matters as an incident of its constitutionally granted powers and functions. It cited Lokin
v. COMELEC, where We held that when the resolution of an intra-party controversy is necessary or
incidental to the performance of the constitutionally-granted functions of the COMELEC, the latter can step
in and exercise jurisdiction over the intra-party matter. 36 The Lokin case, however, involved nominees and
not incumbent members of Congress. In the present case, the fact that petitioner Lico was a member of
Congress at the time of his expulsion from Ating Koop removes the matter from the jurisdiction of the
COMELEC.

The rules on intra-party matters and on the jurisdiction of the HRET are not parallel concepts that do not
intersect. Rather, the operation of the rule on intra-party matters is circumscribed by Section 17 of Article
VI of the 1987 Constitution and jurisprudence on the jurisdiction of electoral tribunals. The jurisdiction of
the HRET is exclusive. It is given full authority to hear and decide the cases on any matter touching on
the validity of the title of the proclaimed winner. 37

In the present case, the Petition for petitioner Lico's expulsion from the House of Representatives is
anchored on his expulsion from Ating Koop, which necessarily affects his title as member of Congress. A
party-list nominee must have been, among others, a bona fide member of the party or organization for at
least ninety (90) days preceding the day of the election. Needless to say, bona fide membership in the
party-list group is a continuing qualification. We have ruled that qualifications for public office, whether
elective or not, are continuing requirements. They must be possessed not only at the time of appointment
or election, or of assumption of office, but during the officer's entire tenure.39

This is not the first time that this Court has passed upon the issue of HRET jurisdiction over the
requirements for bona fide membership in a party-list organization. In Abayon v. HRET,40 it was argued
that the petitioners did not belong to the marginalized and under-represented sectors that they should
represent; as such, they could not be properly considered bona fide members of their respective party-list
organizations. The Court held that it was for the HRET to interpret the meaning of the requirement of bona
fide membership in a party-list organization. It reasoned that under Section 17, Article VI of the
Constitution, the HRET is the sole judge of all contests when it comes to qualifications of the
members of the House of Representatives. 41

Consequently, the COMELEC failed to recognize that the issue on the validity of petitioner Lico's expulsion
from Ating Koop is integral to the issue of his qualifications to sit in Congress. This is not merely an error
of law but an error of jurisdiction correctible by a writ of certiorari; 42 the COMELEC should not have
encroached into the expulsion issue, as it was outside its authority to do so.

Distinguished from Reyes v. COMELEC

Our ruling here must be distinguished from Regina Ongsiako Reyes v. Commission on Elections. 43 In that
case, We upheld the disqualification by the COMELEC of petitioner Reyes, even as she was already
proclaimed winner in the elections at the time she filed her petition with the High Court. In doing so, We
rejected the argument that the case fell within the exclusive jurisdiction of the HRET.

In Reyes, the petitioner was proclaimed winner of the 13 May 2013 Elections, and took her oath of office
before the Speaker of the House of Representatives. However, the Court ruled on her qualifications since
she was not yet a member of the House of Representatives: petitioner Reyes had yet to assume office,
the term of which would officially start at noon of 30 June 2013, when she filed a Petition
for Certiorari with Prayer for Temporary Restraining Order and/or Preliminary Injunction and/or Status Quo
Ante Order dated 7 June 2013 assailing the Resolutions ordering the cancellation of her Certificate of
Candidacy. In the present case, all three requirements of proclamation, oath of office, and assumption of
office were satisfied.

Moreover, in Reyes, the COMELEC En Banc Resolution disqualifying petitioner on grounds of lack of Filipino
citizenship and residency had become final and executory when petitioner elevated it to this Court. 44 It
should be mentioned that when petitioner Reyes filed her petition with the Court, the COMELEC En
Banc had, as early as 5 June 2013, already issued a Certificate of Finality over its 14 May 2013 Resolution
disqualifying her. Therefore, there was no longer any pending case on the qualifications of petitioner Reyes
to speak of. Here, the question of whether petitioner Lico remains a member of the House of
Representatives in view of his expulsion from Ating Koop is a subsisting issue.

Finally, in Reyes, We found the question of jurisdiction of the HRET to be a non-issue, since the recourse of
the petitioner to the Court appeared to be a mere attempt to prevent the COMELEC from implementing a
final and executory judgment. We said that the petitioner therein took an inconsistent, if not confusing,
stance, considering that she sought remedy before the Court, and yet asserted that it is the HRET which
had jurisdiction over the case.45 In this case, the question on the validity of petitioner Lico's expulsion from
Ating Koop is a genuine issue that falls within the jurisdiction of the HRET, as it unmistakably affects his
qualifications as party-list representative.

On which group legitimately represents


Ating Koop

We now pass upon the question of which, between the two contending groups, is the legitimate leadership
of Ating Koop.

At the outset, We reject the Lico Group's argument that the COMELEC has no jurisdiction to decide which
of the feuding groups is to be recognized, and that it is the Regional Trial Court which has jurisdiction over
intra-corporate controversies. Indeed, the COMELECs jurisdiction to settle the struggle for leadership
within the party is well established. This power to rule upon questions of party identity and leadership is
exercised by the COMELEC as an incident of its enforcement powers. 46

That being said, We find the COMELEC to have committed grave abuse of discretion in declaring the Rimas
Group as the legitimate set of Ating Koop officers for the simple reason that the amendments to the
Constitution and By-laws of Ating Koop were not registered with the COMELEC. Hence, neither of the
elections held during the Cebu meeting and the Paranaque conference pursuant to the said amendments,
were valid.

Both the Lico Group and the Rimas Group indeed assert that their respective elections were conducted
pursuant to the amendment introduced in the Second National Convention held on 14 May 2011. In
particular, Section 1 of Article VI of Ating Koop's By-laws called for the conduct of an election of Central
Committee members within six months after the Second National Convention. 47

There is no showing, however, that the amendments were actually filed with the COMELEC.

A party-list organization owes its existence to the State and the latter's approval must be obtained through
its agent, the COMELEC. In the 2013 case of Dayao v. COMELEC,48 We declared that it is the State, acting
through the COMELEC, that breathes life to a party-list organization. The implication, therefore, is that the
State, through the COMELEC, is a party to the principal contracts entered into by the party-list
organization and its members - the Constitution and By-laws - such that any amendment to these
contracts would constitute a novation requiring the consent of all the parties involved. An amendment to
the bylaws of a party-list organization should become effective only upon approval by the COMELEC.

Such a prerequisite is analogous to the requirement of filing of the amended by-laws and subsequent
conformity thereto of the Securities and Exchange Commission (SEC) under corporation law. Under the
Corporation Code, an amendment to a by-law provision must be filed with the SEC. The amendment shall
be effective only upon the issuance by the SEC of a certification that it is not inconsistent with the
Corporation Code.49

There being no showing that the amendments on the by-laws of Ating Koop were filed with and
subsequently approved by the COMELEC, any election conducted pursuant thereto may not be considered
valid. Without such requisite proof, neither the Lico Group nor the Rimas Group can claim to be the
legitimate set of officers of Ating Koop.

Even assuming arguendo that the amendment calling for a special election were effective, this Court still
cannot declare any of the feuding groups as the legitimate set of officers considering that the respective
sets of evidence presented were evenly balanced. With respect to the Lico Group's Cebu meeting, the
COMELEC correctly found - and the records bear out - that the notices sent were deficient and that there
was no sufficient proof of quorum. Hence, the Cebu meeting was held to be invalid. On the other hand, the
COMELEC failed to appreciate the fact that the Paranaque convention suffered from the same infirmity, the
records of the said convention, consisting merely of the Minutes thereof, likewise fail to establish due
notice and a quorum.50

Accordingly, as neither group can sufficiently lay claim to legitimacy, the equipoise doctrine comes into
play. This rule provides that when the evidence in an issue of fact is in equipoise, that is, when the
respective sets of evidence of both parties are evenly balanced, the party having the burden of proof fails
in that issue. Since neither party succeeds in making out a case, neither side prevails. The courts are left
with no other option but to leave them as they are. The consequence, therefore, is the dismissal of the
complaint/petition.51

The Rimas Group, being the petitioner before the COMELEC, had the burden of proving that it is the
petitioner, and not the Lico Group, that is the legitimate group. As the evidence of both parties are in
equipoise, the Rimas Group failed to discharge its burden. The COMELEC should have dismissed the
petition of the Rimas Group insofar as it sought to be declared the legitimate group representing Ating
Koop.

Yet, the COMELEC held that the Paranaque convention "appeared to be in conformity" with Ating Koop's
Amended Constitution and By-Laws.52 It should be stressed that the COMELEC did not even substantiate
this conclusion.53

The Court ordinarily refrains from reviewing the COMELEC s appreciation and evaluation of the
evidence.54 But when the COMELECs assessment of the evidence is so grossly unreasonable that it turns
into an error of jurisdiction, the Court is compelled to intervene and correct the error. 55

As seen in the above discussions, neither of the parties was able to establish its legitimacy. The evaluation
of the evidence by the COMELEC in deciding the issue of which group legitimately represents Ating Koop
was therefore grossly unreasonable, which amounts to a jurisdictional error that may be remedied by
certiorari under Rule 65.

The final, and most important question to be addressed is: if neither of the two groups is the legitimate
leadership of Ating Koop, then who is?

We find such legitimate leadership to be the Interim Central Committee, whose members remain as such
in a hold-over capacity.

In Seneres v. COMELEC,56 the validity of the Certificate of Nomination filed by Buhay Party-List through its
President, Roger Robles, was questioned on the ground that his term had expired at the time it was filed.
The Court applied by analogy the default rule in corporation law to the effect that officers and directors of
a corporation hold over after the expiration of their terms until such time as their successors are elected or
appointed.57Señeres ruled that the hold-over principle applies in the absence of a provision in the
constitution or by-laws of the party-list organization prohibiting its application.

In the present case, We have gone through the Constitution and Bylaws of Ating Koop and We do not see
any provision forbidding, either expressly or impliedly, the application of the hold-over rule. Thus, in
accordance with corporation law, the existing Interim Central Committee is still a legitimate entity with full
authority to bind the corporation and to carry out powers despite the lapse of the term of its members on
14 November 2011, since no successors had been validly elected at the time, or since.

WHEREFORE, premises considered, the Petition is GRANTED. The COMELEC En Banc Resolution dated 31
January 2013 and the COMELEC Second Division Resolution dated 18 July 2012 in E.M. No. 12-039 are
hereby ANNULLED and SET ASIDE insofar as it declares valid the expulsion of Congressman Lico from
Ating Koop and it upholds the ATING KOOP Party-list Group represented by its President, Amparo T.
Rimas, as the legitimate Party-list Group.

A new one is entered DECLARING that the legitimate Central Committee and set of officers legitimately
representing Ating Koop are the Interim Central Committee and set of officers prior to the split of Ating
Koop. SO ORDERED.

- Abang Lingkod v Comelec, 708 SCRA 133 (2013)

This is a petition for certiorari under Rule 64 in relation to Rule 65 of the Rules of Court filed by (Abang Lingkod Party-List
ABANG LINGKOD) assailing the Resolution 1 dated May 10, 2013 issued by the Commission on Elections COMELEC) En
Bane in SPP No. 12-238 PLM}, which, alia, affirmed the cancellation of ABANG LINGKOD's registration as a party-list
group.

The Facts

ABANG LINGKOD is a sectoral organization that represents the interests of peasant fanners and fisherfolks, and was
registered under the party-list system on December 22, 2009. It participated in the May 2010 elections, but failed to obtain
the number of votes needed for a seat in the House of Representatives.

On May 31, 2012, ABANG LINGKOD manifested before the COMELEC its intent to participate in the May 2013 elections.
On August 2, 2012, the COMELEC issued Resolution No. 9513, 2 which, inter alia required previously registered party-list
groups that have filed their respective Manifestations of Intent to undergo summary evidentiary hearing for purposes of
determining their continuing compliance with the requirements under Republic Act (R.A.) No. 7941 3 and the guidelines set
forth in Ang Bagong Bayani-OFW Labor Party v. COMELEC. 4

Accordingly, on August 9 2012, the COMELEC issued a Resolution, which set the summary evidentiary hearing of
previously registered party-list groups. The COMELEC scheduled three (3) dates -August 17, 31 and September 3, 2012
-for the summary hearing of ABANG LINGKOD's Manifestation of Intent to enable it to show proof of its continuing
qualification under the party-list system.

On August 16, 2012, ABANG LINGKOD, in compliance with the COMELEC's August 9, 2012 Resolution, filed with the
COMELEC pertinent documents to prove its continuing compliance with the requirements under R.A. No. 7941.

After due proceedings, the COMELEC En Bane in a Resolution dated November 7 2012, cancelled ABANG LINGKOD's
registration as a partylist group. The COMELEC En Bane pointed out that ABANG LINGKOD failed to establish its track
record in uplifting the cause of the marginalized and underrepresented; that it merely offered photographs of some alleged
activities it conducted after the May 2010 elections. The COMELEC En Bane further opined that ABANG LINGKOD failed
to show that its nominees are themselves marginalized and underrepresented or that they have been involved in activities
aimed at improving the plight of the marginalized and underrepresented sectors it claims to represent.

ABANG LINGKOD then filed with this Court a petition 5 for certiorari alleging that the COMELEC gravely abused its
discretion in cancelling its registration under the party-list system. The said petition was consolidated with the separate
petitions filed by fifty-one (51) other party-list groups whose registration were cancelled or who were denied registration
under the party-list system. The said party-list groups, including ABANG LINGKOD, were able to obtain status quo ante
orders from this Court.

On April 2, 2013, the Court, in Atong Paglaum Inc. v. Commission on Elections, 6 laid down new parameters to be
observed by the COMELEC in screening parties, organizations or associations seeking registration and/or accreditation
under the party-list system, viz:

1. Three different groups may participate in the party-list system: (1) national parties or organizations, (2) regional
parties or organizations, and (3) sectoral parties or organizations.

2. National parties or organizations and regional parties or organizations do not need to organize along sectoral
lines and do not need to represent any marginalized and underrepresented sector. 3. Political parties can
participate in party-list elections provided they register under the party-list system and do not field candidates in
legislative district elections. A political party, whether major or not, that fields candidates in legislative district
elections can participate in party-list elections only through its sectoral wing that can separately register under the
party-list system. The sectoral wing is by itself an independent sectoral party, and is linked to a political party
through a coalition.

4. Sectoral parties or organizations may either be "marginalized and underrepresented or lacking in "well-defined
political constituencies." It is enough that their principal advocacy pertains to the special interests and concerns of
their sector. The sectors that are marginalized and underrepresented include labor, peasant, fisherfolk, urban
poor, indigenous cultural communities, handicapped, veterans, and overseas workers. The sectors that lack "well-
defined political constituencies" include professionals, the elderly, women, and the youth.

5. A majority of the members of the sectoral parties or organizations that represent the ''marginalized and
underrepresented must belong to the marginalized and underrepresented sector they represent. Similarly, a
majority of the members of sectoral parties or organizations that lack "well-defined political constituencies" must
belong to the sector they represent. The nominees of sectoral parties or organizations that represent the
"marginalized and underrepresented" or that represent those who lack "well-defined political constituencies," either
must belong to their respective sectors, or must have a track record or advocacy for their respective sectors. The
nominees of national and regional parties or organizations must be bona-fide members of such parties or
organizations.

6. National, regional, and sectoral parties or organizations shall not be disqualified if some of their nominees are
disqualified, provided that they have at least one nominee who remains qualified.
Thus, the Court remanded to the COMELEC the cases of previously registered party-list groups, including that of ABANG
LINGKOD, to determine whether they are qualified under the party-list system pursuant to the new parameters laid down
by the Court and, in the affirmative, be allowed to participate in the May 2013 party-list elections.

On May 10, 2013, the COMELEC issued the herein assailed Resolution, 7 which, inter alia affirmed the cancellation of
ABANG LINGKOD's registration under the party-list system. The COMELEC issued the Resolution dated May 10, 2013
sans any summary evidentiary hearing, citing the proximity of the May 13 2013 elections as the reason therefor.

In maintaining the cancellation of ABANG LINGKOD's registration, the COMELEC held that:

The Commission maintains its position in the previous en bane ruling cancelling the registration of ABANG LINGKOD. To
reiterate, it is not enough that the party-list organization claim representation of the marginalized and underrepresented
because representation is easy to claim and to feign. It is but reasonable to require from groups and organizations
consistent participation and advocacy in the sector it seeks to represent, and not just seasonal and sporadic programs
which are unrelated to its sector.

ABANG LINGKOD submitted pictures showing a seminar held on 10 July 2010, Medical Mission on 11 November 2010,
Disaster Management Training on 21 October 2011, Book-giving on 28 June 2011, and Medical Mission on 1 December
2011.

And as if to insult the Commission, the photographs submitted appear to have been edited to show in the banners that
ABANG LINGKOD participated in the activities. ABANG LINGKOD's name and logo was superimposed on some banners
to feign participation in the activities (Joint Medical Mission, Book-giving).

Under the party-list System Act, a group s registration may be cancelled for declaring unlawful statements in its petition.
Photoshopping images to establish a fact that did not occur is tantamount to declaring unlawful statements. It is on this
ground that the Commission cancels ABANG LINGKOD s registration. 8

On May 12, 2013, ABANG LINGKOD sought a reconsideration of the COMELEC s Resolution dated May 10, 2013.
However, on May 15, 2013, ABANG LINGKOD withdrew the motion for reconsideration it filed with the COMELEC and,
instead, instituted the instant petition9 with this Court, alleging that there may not be enough time for the COMELEC to
pass upon the merits of its motion for reconsideration considering that the election returns were already being canvassed
and consolidated by the COMELEC.

In support of the instant petition, ABANG LINGKOD claims that the COMELEC gravely abused its discretion when it
affirmed the cancellation of its registration sans a summary evidentiary hearing for that purpose, asserting that the
COMELEC should have allowed it to present evidence to prove its qualification as a party-list group pursuant to Atong
Paglaum. It claims that there was no valid justification for the COMELEC to cancel its registration considering that it
complied with the six-point parameters m screening party-list groups laid down in Atong Paglaum.

On the other hand, the COMELEC avers that the instant petition should be dismissed for utter lack of merit. It asserts that
ABANG LINGKOD was not denied due process when the COMELEC affirmed the cancellation of its registration since it
was given every reasonable opportunity to be heard. The COMELEC further claims that it did not abuse its discretion
when it cancelled ABANG LINGKOD’s registration on the ground that it failed to establish a track record in representing
the marginalized and underrepresented. Further, the COMELEC alleges that its finding of facts may not be passed upon
by this Court as the same is supported by substantial evidence.

The Issues

In sum, the issues presented for the Court s resolution are the following: first whether ABANG LINGKOD was denied due
process when the COMELEC affirmed the cancellation of its registration under the patiy-list system sans any summary
evidentiary hearing; and second whether the COMELEC gravely abused its discretion in cancelling ABANG LINGKOD’s
registration under the party-list system.

The Court's Ruling

The petition is meritorious.

First Issue: Due Process

The essence of due process is simply an opportunity to be heard or as applied to administrative or quasi-judicial
proceedings, an opportunity to explain one s side or an opportunity to seek reconsideration of the action or ruling
complained of. A formal or trial type hearing is not at all times and in all instances essential. The requirements are satisfied
when the parties are afforded fair and reasonable opportunity to explain their side of the controversy at hand. What is
frowned upon is the absolute lack of notice or hearing. 10

In the instant case, while the petitioner laments that it was denied due process, the Court finds that the COMELEC had
afforded ABANG LINGKOD sufficient opportunity to present evidence establishing its qualification as a party-list group. It
was notified through Resolution No. 9513 that its registration was to be reviewed by the COMELEC. That ABANG
LINGKOD was able to file its Manifestation of Intent and other pertinent documents to prove its continuing compliance with
the requirements under R.A. No. 7941, which the COMELEC set for summary hearing on three separate dates, belies its
claim that it was denied due process.

There was no necessity for the COMELEC to conduct further summary evidentiary hearing to assess the qualification of
ABANG LINGKOD pursuant to Atong Paglaum. ABANG LINGKOD’s Manifestation of Intent and all the evidence adduced
by it to establish its qualification as a party-list group are already in the possession of the COMELEC. Thus, conducting
further summary evidentiary hearing for the sole purpose of determining ABANG LINGKOD s qualification under the party-
list system pursuant to Atong Paglaum would just be a superfluity.

Contrary to ABANG LINGKOD’s claim, the Court, in Atong Paglaum, did not categorically require the COMELEC to
conduct a summary evidentiary hearing for the purpose of determining the qualifications of the petitioners therein pursuant
to the new parameters for screening party-list groups. The dispositive portion of Atong Paglaum reads:

WHEREFORE, all the present 54 petitions are GRANTED. The 13 petitions, which have been granted Status Quo Ante
Orders but without mandatory injunction to include the names of the petitioners in the printing of ballots, are remanded to
the Commission on Elections only for determination whether petitioners are qualified to register under the party-list system
under the parameters prescribed in this Decision but they shall not participate in the 13 May 2013 party-list elections. The
41 petitions, which have been granted mandatory injunctions to include the names of petitioners in the printing of ballots,
are remanded to the Commission on Elections for determination whether petitioners are qualified to register under the
party-list system and to participate in the 13 May 2013 party-list elections under the parameters prescribed in this
Decision. The Commission on Elections may conduct summary evidentiary hearings for this purpose. This Decision is
immediately executory.

SO ORDERED.11 (Emphasis ours)

Thus, the cases of previously registered party-list groups, including ABANG LINGKOD, were remanded to the COMELEC
so that it may reassess, based on the evidence already submitted by the former, whether they are qualified to participate
in the party-list system pursuant to the new parameters laid down in Atong Paglaum. The Court did not require the
COMELEC to conduct a hearing de novo in reassessing the qualifications of said party-list groups. Nevertheless, the
Court gave the COMELEC the option to conduct further summary evidentiary hearing should it deem appropriate to do so.

The records also disclose that ABANG LINGKOD was able to file with the COMELEC a motion for reconsideration of the
Resolution dated May 10, 2013, negating its claim that it was denied due process. As it has been held, deprivation of due
process cannot be successfully invoked where a party was given a chance to be heard on his motion for reconsideration. 12

Second Issue: Cancellation of

ABANG LINGKOD’s Registration

However, after a careful perusal of the factual antecedents of this case, pinned against the new parameters in screening
party-list groups laid down in Atong Paglaum the Court finds that the COMELEC gravely abused its discretion in cancelling
the registration of ABANG LINGKOD under the party-list system.

The COMELEC affirmed the cancellation of ABANG LINGKOD's registration on the ground that it declared untruthful
statement in its bid for accreditation as a party-list group in the May 2013 elections, pointing out that it deliberately
submitted digitally altered photographs of activities to make it appear that it had a track record in representing the
marginalized and underrepresented. Essentially, ABANG LINGKOD's registration was cancelled on the ground that it
failed to adduce evidence showing its track record in representing the marginalized and underrepresented.

The flaw in the COMELEC's disposition lies in the fact that it insists on requiring party-list groups to present evidence
showing that they have a track record in representing the marginalized and underrepresented.

Track record is a record of past performance often taken as an indicator of likely future performance. 13 As a requirement
imposed by Ang Bagong Bayani for groups intending to participate in the party-list elections, track record pertains to the
actual activities undertaken by groups to uplift the cause of the sector/s, which they represent.

Section 5 of R.A. No. 7941 however provides:

Sec. 5 Registration. Any organized group of persons may register as a party, organization or coalition for purposes of the
party-list system by filing with the COMELEC not later than ninety (90) days before the election a petition verified by its
president or secretary stating its desire to participate in the party-list system as a national, regional or sectoral party or
organization or a coalition of such parties or organizations, attaching thereto its constitution, by-laws, platform or program
of government list of officers, coalition agreement and other relevant information as the COMELEC may require: Provided,
That the sectors shall include labor, peasant, fisherfolk, urban poor, indigenous cultural communities, elderly,
handicapped, women, youth, veterans, overseas workers, and professionals. (Emphasis ours)

R.A. No. 7941 did not require groups intending to register under the party-list system to submit proof of their track record
as a group. The track record requirement was only imposed in Ang Bagong Bayani where the Court held that national,
regional, and sectoral parties or organizations seeking registration under the party-list system must prove through their,
inter alia track record that they truly represent the marginalized and underrepresented, thus:
xxx

In this light, the Court finds it appropriate to lay down the following guidelines, culled from the law and the Constitution, to
assist the Comelec in its work.

First, the political pat1y, sector, organization or coalition must represent the marginalized and underrepresented groups
identified in Secdon 5 of RA 7941. In other words, it must show -- through its constitution, articles of incorporation, bylaws,
history, platform of government and track record -- that it represents and seeks to uplift marginalized and
underrepresented sectors. Verily, majority of its membership should belong to the marginalized and underrepresented.
And it must demonstrate that in a conflict of interests, it has chosen or is likely to choose the interest of such sectors.
(Emphasis ours)

Track record is not the same as the submission or presentation of "constitution, by-laws, platform of government, list of
officers, coalition agreement, and other relevant information as may be required by the COMELEC," which are but mere
pieces of documentary evidence intended to establish that the group exists and is a going concern. The said documentary
evidence presents an abstract of the ideals that national, regional, and sectoral parties or organizations seek to achieve.

This is not merely a matter of semantics; the delineation of what constitutes a track record has certain consequences in a
group's bid for registration under the party-list system. Under Section 5 of R.A. No. 7941, groups intending to register
under the party-list system are not required to submit evidence of their track record; they are merely required to attach to
their verified petitions their "constitution, by-laws, platform of government, list of officers, coalition agreement, and other
relevant information as may be required by the COMELEC."

In Atong Paglaum the Court has modified to a great extent the jurisprudential doctrines on who may register under the
party-list system and the representation of the marginalized and underrepresented. For purposes of registration under the
party-list system, national or regional parties or organizations need not represent any marginalized and underrepresented
sector; that representation of the marginalized and underrepresented is only required of sectoral organizations that
represent the sectors stated under Section 5 of R.A. No. 7941 that are, by their nature, economically marginalized and
underrepresented.

There was no mention that sectoral organizations intending to participate in the party-list elections are still required to
present a track record, viz:

x x x In determining who may participate in the coming 13 May 2013 and subsequent party-list elections, the COMELEC
shall adhere to the following parameters:

xxxx

4. Sectoral parties or organizations may either be marginalized and underrepresented or lacking in well-defined political
constituencies. It is enough that their principal advocacy pertains to the special interests and concerns of their sector. The
sectors that are marginalized and underrepresented include labor, peasant, fisherfolk, urban poor, indigenous cultural
communities, handicapped, veterans, and overseas workers. The sectors that lack well-defined political constituencies''
include professionals, the elderly, women, and the youth. (Emphasis ours)

Contrary to the COMELEC's claim, sectoral parties or organizations, such as ABANG LINGKOD, are no longer required to
adduce evidence showing their track record, i.e. proof of activities that they have undertaken to further the cause of the
sector they represent. Indeed, it is enough that their principal advocacy pertains to the special interest and concerns of
their sector. Otherwise stated, it is sufficient that the ideals represented by the sectoral organizations are geared towards
the cause of the sector/s, which they represent.

If at all, evidence showing a track record in representing the marginalized and underrepresented sectors is only required
from nominees of sectoral parties or organizations that represent the marginalized and underrepresented who do not
factually belong to the sector represented by their party or organization.

Dissenting, my esteemed colleague, Mr. Justice Leonen, however, maintains that parties or organizations intending to
register under the party-list system are still required to present a track record notwithstanding the Court's pronouncement
in Atong Paglaum that the track record that would have to be presented would only differ as to the nature of their
group/organization. He opines that sectoral organizations must prove their links with the marginalized and
underrepresented while national or regional parties or organizations must show that they have been existing as a bona
fide organization.

To submit to the dissent's insistence on varying track records, which are required of those intending to register under the
party-list system, depending on the nature of their group, would result into an absurd and unjust situation. Under the
varying track record requirement, sectoral organizations must present evidence showing their track record in representing
the marginalized and underrepresented, i.e. actual activities conducted by them to further uplift the cause of the sector/s
they represent. On the other hand, national and regional parties or organizations need only prove that they exist as bona
fide organizations which, as the dissent suggests, may be done through the submission of their constitution, by-laws,
platform of government, list of officers, coalition agreement, and other relevant information required by the COMELEC.

However, submission of a group's constitution, by-laws, platform of government, list of officers, coalition agreement, and
other relevant information required by the COMELEC, as explained earlier, is not synonymous with the track record
requirement. In such case, only sectoral organizations would be required to present a track record (actual activities
conducted by them to further the cause of the marginalized and underrepresented); while national and regional
organizations need not present their track record as they are only required to submit documentary evidence showing that
they are bona fide organizations.

There is no logic in treating sectoral organizations differently from national and regional parties or organizations as regards
their bid for registration under the party-list system. The varying track record requirement suggested by the dissent would
unnecessarily put a premium on groups intending to register as national and regional parties or organizations as against
those intending to register as sectoral organizations The imposition of an additional burden on sectoral organizations, i.e.
submission of their track record, would be plainly unjust as it effectively deters the marginalized and underrepresented
sectors from organizing themselves under the party-list system.

Likewise, that there was no explicit reversal of the guidelines in ng Bagong Bayani in tong Paglaum does not mean that
groups intending to register under the party-list system are still required to submit a track record. The track record of
groups intending to register under the party-list system was required under the first guideline of Ang Bagong Bayani for a
very specific purpose to show that the national, regional, and sectoral parties or organizations that would be allowed to
participate in the party-list elections are truly representative of the marginalized and underrepresented sectors It was
necessary-then to require groups seeking registration under the party-list system since representation of the marginalized
and underrepresented, as understood in the context of Ang Bagong Bayani is easy to claim and feign.

There exists no reason to further require groups seeking registration under the party-list system to submit evidence
showing their track record. Pursuant to Atong Paglaum not all groups are required to represent the marginalized and
underrepresented sectors and, accordingly, there is no longer any incentive in merely feigning representation of the
marginalized and underrepresented sectors.

In the case of sectoral organizations, although they are still required to represent the marginalized and underrepresented,
they are likewise not required to show a track record since there would be no reason for them to feign representation of
the marginalized and underrepresented as they can just register as a national or regional party or organization. Thus, the
Court, in Atong Paglaum stated that, for purposes of registration under the party-list system, it is enough that the principal
advocacy of sectoral organizations pertains to the sector/s they represent.

There is thus no basis in law and established jurisprudence to insist that groups seeking registration under the party-list
system still comply with the track record requirement. Indeed, nowhere in R.A. No. 7941 is it mandated that groups
seeking registration thereunder must submit evidence to show their track record as a group.

The dissent likewise suggests that the deceit committed by ABANG LINGKOD goes into its qualification as a party-list
group since it seriously puts in question the existence of ABANG LINGKOD as a group per se and the genuineness of its
representation of the farmers and fisherfolk.

It must be stressed that the COMELEC cancelled ABANG LINGKOD s registration solely on the ground of the lack of its
track record -that it falsely represented, by submitting digitally altered photographs of its supposed activities, that it had a
track record in representing the marginalized and underrepresented. The existence of ABANG LINGKOD as a party-list
group per se and the genuineness of its representation of the farmers and fisherfolks were never raised in the proceedings
before the COMELEC. It would thus be the height of injustice in the Court, in this certiorari action, would scrutinize the
legitimacy of ABANG LINGKOD as a party-list group and the genuineness of its representation of the farmers and
fisherfolk, and affirm the cancellation of its registration, when the issue is limited only to the track record of ABANG
LINGKOD.

Moreover, ABANG LINGKOD had been previously registered as a party-list group, as in fact it participated in the May
2010 party-list elections, and it was able to obtain a sufficient number of votes in the May 2013 party-list elections to obtain
a seat in the House of Representatives. These are circumstances, which clearly indicate that ABANG LINGKOD is indeed
a legitimate party-list group.

ABANG LINGKOD, notwithstanding the cancellation of its registration three days prior to the May 13, 2013 elections, was
able to obtain a total of 260 215 votes out of the 26 722 131 votes that were cast for the party-list, 14 thus entitling it to a
seat in the House of Representatives. This is indicative of the fact that a considerable portion of the electorate considers
ABANG LINGKOD as truly representative of peasant farmers and fisherfolk.

Anent the photographs submitted by ABANG LINGKOD, these only show book-giving and medical missions, which are
activities it conducted. Suffice it to state, however, that said activities do not specifically or directly pertain to the interest or
advocacy espoused by ABANG LINGKOD. As such, the misrepresentation committed by ABANG LINGKOD with regard to
said activities would not necessarily militate against its representation of the farmers and fisherfolk.

Lest it be misunderstood, the Court does not condone the deceit perpetrated by ABANG LINGKOD in connection with its
bid for continued registration under the party-list system. That ABANG LINGKOD, to establish its track record, submitted
photographs that were edited to make it appear that it conducted activities aimed at ameliorating the plight of the sectors it
represents is a factual finding by the COMELEC, which the Court, considering that it is supported by substantial evidence,
will not disturb. The Court does not tolerate ABANG LINGKOD s resort to chicanery and its shabby treatment of the
requirements for registration under the party-list system.
Nevertheless, considering that track record is no longer a requirement, a group’s misrepresentation as to its track record
cannot be used as a ground to deny or cancel its registration -it is no longer material to its qualification under the party-list
system. In this case, ABANG LINGKOD s submission of digitally altered photographs cannot be considered material to its
qualification as a party-list group. Section 6 of R.A. No. 7941, in part, reads:

Sec. 6 Refusal and/or Cancellation o Registration The COMELEC may, motu propio or upon verified complaint of any
interested party, refuse or cancel, after due notice and hearing, the registration of any national, regional or sectoral party,
organization or coalition on any of the following grounds:

xxxx

(6) It declares untruthful statements in its petition;

Declaration of an untruthful statement in a petition for registration, or in any other document pertinent to the registration
and/or accreditation under the party-list system, as a ground for the refusal or cancellation of registration under Section
6(6) of R.A. No. 7941, is akin to material misrepresentation in the certificate of candidacy filed by an individual candidate
under Section 78 of the Omnibus Election Code. Both provisions disallow prospective candidates from participating in an
election for declaring false statements in their eligibility requirements. Section 78 of the Omnibus Election Code reads:

Sec. 78. A verified petition seeking to deny due course to or cancel a certificate of candidacy may be filed by any person
exclusively on the ground that any material misrepresentation contained therein as required under Section 74 hereof is
false. The petition may be filed at any time not later than twenty-five days from the time of the filing of the certificate of
candidacy and shall be decided, after due notice and hearing, not later than fifteen days before the election.

Elucidating on what constitutes material misrepresentation in a certificate of candidacy under Section 78 of the Omnibus
Election Code, the Court, in Lluz v. Commission on Elections, 15 explained that:

From these two cases several conclusions follow. First a misrepresentation in a certificate of candidacy is material when it
refers to a qualification for elective office and affects the candidate s eligibility. x x x Third a misrepresentation of a non-
material fact, or a non-material misrepresentation, is not a ground to deny due course to or cancel a certificate of
candidacy under Section 78. In other words, for a candidate s certificate of candidacy to be denied due course or canceled
by the COMELEC, the fact misrepresented must pertain to a qualification for the office sought by the
candidate.16 (Emphasis ours)

In Velasco v. Commission on Elections,17 the Court further clarified that a false representation under Section 78 of the
Omnibus Election Code, in order to be a ground to deny due course or cancel a certificate of candidacy, must consist of a
deliberate attempt to mislead, misinform, or hide a fact which would otherwise render a candidate ineligible. Thus:

The false representation that [Sections 74 and 78 of the Omnibus Election Code] mention must necessarily pertain to a
material fact, not to a mere innocuous mistake. This is emphasized by the consequences of any material falsity: a
candidate who falsifies a material fact cannot run; if he runs and is elected, cannot serve; in both cases, he or she can be
prosecuted for violation of the election laws. Obviously, these facts are those that refer to a candidate s qualification for
elective office, such as his or her citizenship and residence. The candidate's status as a registered voter similarly falls
under this classification as it is a requirement that, by law (the Local Government Code), must be reflected in the COC.
The reason for this is obvious: the candidate, if he or she wins, will work for and represent the local government under
which he is running.

Separately from the requirement of materiality, a false representation under Section 78 must consist of a deliberate
attempt to mislead, misinform, or hide a fact which would otherwise render a candidate ineligible." In other words, it must
be made with the intention to deceive the electorate as to the would-be candidate's qualifications for public
office.18 (Citation omitted and emphasis ours)

Similarly, a declaration of an untruthful statement in a petition for registration under Section 6(6) of R.A. No. 7941, in order
to be a ground for the refusal and/or cancellation of registration under the party-list system, must pertain to the
qualification of the party, organization or coalition under the party-list system. In order to justify the cancellation or refusal
of registration of a group, there must be a deliberate attempt to mislead, misinform, or hide a fact, which would otherwise
render the group disqualified from participating in the party-list elections.

The digitally altered photographs of activities submitted by ABANG LINGKOD to prove its continuing qualification under
R.A. No. 7941 only pertain to its track record, which, as already discussed, is no longer a requirement under the new
parameters laid down in Atong Paglaum Simply put, they do not affect the qualification of ABANG LINGKOD as a party-list
group and, hence, could not be used as a ground to cancel its registration under the party-list system. Further, the Court
notes that the COMELEC, in its Resolution dated November 7 2012, asserted that ABANG LINGKOD failed to adduce
evidence that would show the track record of its five nominees, composed of a non-government organization worker, an
employee and three farmers, in uplifting the cause of the sector that the group represents.  The COMELEC opined that
1âwphi1

the failure of ABANG LINGKOD to present a track record of its nominees justified the cancellation of its registration as a
party-list group.

The Court does not agree. Assuming arguendo that the nominees of ABANG LINGKOD, as opined by the COMELEC,
indeed do not have track records showing their participation in activities aimed at improving the conditions of the sector
that the group represents, the same would not affect the registration of ABANG LINGKOD as a party-list group.
To stress, in Atong Paglaum the Court pointed out that [t]he nominees of sectoral parties or organizations that represent
the 'marginalized and underrepresented,' or that represent those who lack 'well-defined political constituencies,' either
must belong to their respective sectors or must have a track record o advocacy for their respective sectors. Stated
otherwise, the nominee of a party-list groups may either be: first one who actually belongs to the sector which the party-list
group represents, in which case the track record requirement does not apply; or second one who does not actually belong
to the sector which the party-list group represents but has a track record showing the nominee's active participation in
activities aimed at uplifting the cause of the sector which the group represents."

In the case under consideration, three of the five nominees of ABANG LINGKOD are farmers and, thus, are not required
to present a track record showing their active participation in activities aimed to promote the sector which ABANG
LINGKOD represents, i.e. peasant farmers and fisherfolk. That two of ABANG LINGKOD's nominees do not actually
belong to the sector it represents is immaterial and would not result in the cancellation of ABANG LINGKOD's registration
as a party-list group. This is clear from the sixth parameter laid down by the Court in tong Paglaum which states that
"national, regional and sectoral organizations shall not be disqualified if some of their nominees are disqualified, provided
that they have at least one nominee who remains qualified." At the very least, ABANG LINGKOD has three (3) qualified
nominees, being farmers by occupation.

Indeed, the disqualification of one or some of the nominees of a party-list group should not automatically result in the
disqualification of the group.  Otherwise it would accord the nominees the same significance, which the law holds for the
1avvphi1

party-list groups; it is still the fact that the party-list group satisfied the qualifications of the law that is material to consider.
The disqualification of the nominees must simply be regarded as failure to qualify for an office or position. It should not, in
any way, blemish the qualifications of the party-list group itself with defect. The party-list group must be treated as
separate and distinct from its nominees such that qualifications of the latter must not be considered part and parcel of the
qualifications of the former.

In sum, that ABANG LINGKOD's registration must be cancelled due to its misrepresentation is a conclusion derived from a
simplistic reading of the provisions of R.A. No. 7941 and the import of the Court's disposition in tong Paglaum. Not every
misrepresentation committed by national, regional, and sectoral groups or organizations would merit the denial or
cancellation of their registration under the party-list system. The misrepresentation must relate to their qualification as a
party-list group. In this regard, the COMELEC gravely abused its discretion when it insisted on requiring ABANG
LINGKOD to prove its track record notwithstanding that a group s track record is no longer required pursuant to the Court
s pronouncement in Atong Paglaum

Likewise, upholding the cancellation of ABANG LINGKOD s registration, notwithstanding that it was able to obtain
sufficient number of votes for a legislative seat, would serve no purpose other than to subvert the will of the electorate who
voted to give ABANG LINGKOD the privilege to represent them in the House of Representatives.

WHEREFORE in light of the foregoing disquisitions, the instant petition is hereby GRANTED. The Resolution dated May
10, 2013 issued by the Commission on Elections in SPP Case No. 12-238 (PLM), insofar as it affirmed the cancellation of
ABANG LINGKOD s registration and disallowed it to participate in the May 13, 2013 elections is REVERSED and SET
ASIDE.

The Commission on Elections is hereby ORDERED to PROCLAIM ABANG LINGKOD as one of the winning party-list
groups during the May 13, 2013 elections with the number of seats it may be entitled to based on the total number of votes
it garnered during the said elections.

SO ORDERED.

- Abayon v HRET, 612 SCRA 375 (2010)

These two cases are about the authority of the House of Representatives Electoral Tribunal (HRET) to pass upon the
eligibilities of the nominees of the party-list groups that won seats in the lower house of Congress.

The Facts and the Case

In G.R. 189466, petitioner Daryl Grace J. Abayon is the first nominee of the Aangat Tayo party-list organization that won a
seat in the House of Representatives during the 2007 elections.

Respondents Perfecto C. Lucaban, Jr., Ronyl S. Dela Cruz, and Agustin C. Doroga, all registered voters, filed a petition
for quo warranto with respondent HRET against Aangat Tayo and its nominee, petitioner Abayon, in HRET Case 07-041.
They claimed that Aangat Tayo was not eligible for a party-list seat in the House of Representatives, since it did not
represent the marginalized and underrepresented sectors.

Respondent Lucaban and the others with him further pointed out that petitioner Abayon herself was not qualified to sit in
the House as a party-list nominee since she did not belong to the marginalized and underrepresented sectors, she being
the wife of an incumbent congressional district representative. She moreover lost her bid as party-list representative of the
party-list organization called An Waray in the immediately preceding elections of May 10, 2004.

Petitioner Abayon countered that the Commission on Elections (COMELEC) had already confirmed the status of Aangat
Tayo as a national multi-sectoral party-list organization representing the workers, women, youth, urban poor, and elderly
and that she belonged to the women sector. Abayon also claimed that although she was the second nominee of An Waray
party-list organization during the 2004 elections, she could not be regarded as having lost a bid for an elective office.

Finally, petitioner Abayon pointed out that respondent HRET had no jurisdiction over the petition for quo warranto since
respondent Lucaban and the others with him collaterally attacked the registration of Aangat Tayo as a party-list
organization, a matter that fell within the jurisdiction of the COMELEC. It was Aangat Tayo that was taking a seat in the
House of Representatives, and not Abayon who was just its nominee. All questions involving her eligibility as first
nominee, said Abayon, were internal concerns of Aangat Tayo.

On July 16, 2009 respondent HRET issued an order, dismissing the petition as against Aangat Tayo but upholding its
jurisdiction over the qualifications of petitioner Abayon. 1 The latter moved for reconsideration but the HRET denied the
same on September 17, 2009,2 prompting Abayon to file the present petition for special civil action of certiorari.

In G.R. 189506, petitioner Jovito S. Palparan, Jr. is the first nominee of the Bantay party-list group that won a seat in the
2007 elections for the members of the House of Representatives. Respondents Reynaldo Lesaca, Jr., Cristina Palabay,
Renato M. Reyes, Jr., Erlinda Cadapan, Antonio Flores, and Joselito Ustarez are members of some other party-list
groups.

Shortly after the elections, respondent Lesaca and the others with him filed with respondent HRET a petition
for quo warranto against Bantay and its nominee, petitioner Palparan, in HRET Case 07-040. Lesaca and the others
alleged that Palparan was ineligible to sit in the House of Representatives as party-list nominee because he did not belong
to the marginalized and underrepresented sectors that Bantay represented, namely, the victims of communist rebels,
Civilian Armed Forces Geographical Units (CAFGUs), former rebels, and security guards. Lesaca and the others said that
Palparan committed gross human rights violations against marginalized and underrepresented sectors and organizations.

Petitioner Palparan countered that the HRET had no jurisdiction over his person since it was actually the party-list Bantay,
not he, that was elected to and assumed membership in the House of Representatives. Palparan claimed that he was just
Bantay’s nominee. Consequently, any question involving his eligibility as first nominee was an internal concern of Bantay.
Such question must be brought, he said, before that party-list group, not before the HRET.

On July 23, 2009 respondent HRET issued an order dismissing the petition against Bantay for the reason that the issue of
the ineligibility or qualification of the party-list group fell within the jurisdiction of the COMELEC pursuant to the Party-List
System Act. HRET, however, defended its jurisdiction over the question of petitioner Palparan’s qualifications. 3 Palparan
moved for reconsideration but the HRET denied it by a resolution dated September 10, 2009, 4 hence, the recourse to this
Court through this petition for special civil action of certiorari and prohibition.

Since the two cases raise a common issue, the Court has caused their consolidation.

The Issue Presented

The common issue presented in these two cases is:

Whether or not respondent HRET has jurisdiction over the question of qualifications of petitioners Abayon and Palparan as
nominees of Aangat Tayo and Bantay party-list organizations, respectively, who took the seats at the House of
Representatives that such organizations won in the 2007 elections.

The Court’s Ruling

Petitioners Abayon and Palparan have a common theory: Republic Act (R.A.) 7941, the Party-List System Act, vests in the
COMELEC the authority to determine which parties or organizations have the qualifications to seek party-list seats in the
House of Representatives during the elections. Indeed, the HRET dismissed the petitions for quo warranto filed with it
insofar as they sought the disqualifications of Aangat Tayo and Bantay. Since petitioners Abayon and Palparan were not
elected into office but were chosen by their respective organizations under their internal rules, the HRET has no
jurisdiction to inquire into and adjudicate their qualifications as nominees.

If at all, says petitioner Abayon, such authority belongs to the COMELEC which already upheld her qualification as
nominee of Aangat Tayo for the women sector. For Palparan, Bantay’s personality is so inseparable and intertwined with
his own person as its nominee so that the HRET cannot dismiss the quo warranto action against Bantay without
dismissing the action against him.

But, although it is the party-list organization that is voted for in the elections, it is not the organization that sits as and
becomes a member of the House of Representatives. Section 5, Article VI of the Constitution, 5 identifies who the
"members" of that House are:

Sec. 5. (1). The House of Representatives shall be composed of not more than two hundred and fifty members, unless
otherwise fixed by law, who shall be elected from legislative districts apportioned among the provinces, cities, and the
Metropolitan Manila area in accordance with the number of their respective inhabitants, and on the basis of a uniform and
progressive ratio, and those who, as provided by law, shall be elected through a party-list system of registered national,
regional, and sectoral parties or organizations. (Underscoring supplied)
Clearly, the members of the House of Representatives are of two kinds: "members x x x who shall be elected from
legislative districts" and "those who x x x shall be elected through a party-list system of registered national,
regional, and sectoral parties or organizations." This means that, from the Constitution’s point of view, it is the party-list
representatives who are "elected" into office, not their parties or organizations. These representatives are elected,
however, through that peculiar party-list system that the Constitution authorized and that Congress by law established
where the voters cast their votes for the organizations or parties to which such party-list representatives belong.

Once elected, both the district representatives and the party-list representatives are treated in like manner. They have the
same deliberative rights, salaries, and emoluments. They can participate in the making of laws that will directly benefit
their legislative districts or sectors. They are also subject to the same term limitation of three years for a maximum of three
consecutive terms.

It may not be amiss to point out that the Party-List System Act itself recognizes party-list nominees as "members of the
House of Representatives," thus:

Sec. 2. Declaration of Policy. - The State shall promote proportional representation in the election of representatives to the
House of Representatives through a party-list system of registered national, regional and sectoral parties or organizations
or coalitions thereof, which will enable Filipino citizens belonging to the marginalized and underrepresented sectors,
organizations and parties, and who lack well-defined political constituencies but who could contribute to the formulation
and enactment of appropriate legislation that will benefit the nation as a whole, to become members of the House of
Representatives. Towards this end, the State shall develop and guarantee a full, free and open party system in order to
attain the broadest possible representation of party, sectoral or group interests in the House of Representatives by
enhancing their chances to compete for and win seats in the legislature, and shall provide the simplest scheme possible.
(Underscoring supplied)

As this Court also held in Bantay Republic Act or BA-RA 7941 v. Commission on Elections,6 a party-list representative is in
every sense "an elected member of the House of Representatives." Although the vote cast in a party-list election is a vote
for a party, such vote, in the end, would be a vote for its nominees, who, in appropriate cases, would eventually sit in the
House of Representatives.

Both the Constitution and the Party-List System Act set the qualifications and grounds for disqualification of party-list
nominees. Section 9 of R.A. 7941, echoing the Constitution, states:

Sec. 9. Qualification of Party-List Nominees. – No person shall be nominated as party-list representative unless
he is a natural-born citizen of the Philippines, a registered voter, a resident of the Philippines for a period of not
less than one (1) year immediately preceding the day of the election, able to read and write, bona fide member of
the party or organization which he seeks to represent for at least ninety (90) days preceding the day of the
election, and is at least twenty-five (25) years of age on the day of the election. 1avvphi1

In case of a nominee of the youth sector, he must at least be twenty-five (25) but not more than thirty (30) years of
age on the day of the election. Any youth sectoral representative who attains the age of thirty (30) during his term
shall be allowed to continue until the expiration of his term.

In the cases before the Court, those who challenged the qualifications of petitioners Abayon and Palparan claim that the
two do not belong to the marginalized and underrepresented sectors that they ought to represent. The Party-List System
Act provides that a nominee must be a "bona fide member of the party or organization which he seeks to represent." 7

It is for the HRET to interpret the meaning of this particular qualification of a nominee—the need for him or her to be a
bona fide member or a representative of his party-list organization—in the context of the facts that characterize petitioners
Abayon and Palparan’s relation to Aangat Tayo and Bantay, respectively, and the marginalized and underrepresented
interests that they presumably embody.

Petitioners Abayon and Palparan of course point out that the authority to determine the qualifications of a party-list
nominee belongs to the party or organization that nominated him. This is true, initially. The right to examine the fitness of
aspiring nominees and, eventually, to choose five from among them after all belongs to the party or organization that
nominates them.8 But where an allegation is made that the party or organization had chosen and allowed a disqualified
nominee to become its party-list representative in the lower House and enjoy the secured tenure that goes with the
position, the resolution of the dispute is taken out of its hand.

Parenthetically, although the Party-List System Act does not so state, the COMELEC seems to believe, when it resolved
the challenge to petitioner Abayon, that it has the power to do so as an incident of its authority to approve the registration
of party-list organizations. But the Court need not resolve this question since it is not raised here and has not been argued
by the parties.

What is inevitable is that Section 17, Article VI of the Constitution 9 provides that the HRET shall be the sole judge of all
contests relating to, among other things, the qualifications of the members of the House of Representatives. Since, as
pointed out above, party-list nominees are "elected members" of the House of Representatives no less than the district
representatives are, the HRET has jurisdiction to hear and pass upon their qualifications. By analogy with the cases of
district representatives, once the party or organization of the party-list nominee has been proclaimed and the nominee has
taken his oath and assumed office as member of the House of Representatives, the COMELEC’s jurisdiction over election
contests relating to his qualifications ends and the HRET’s own jurisdiction begins. 10
The Court holds that respondent HRET did not gravely abuse its discretion when it dismissed the petitions for quo
warranto against Aangat Tayo party-list and Bantay party-list but upheld its jurisdiction over the question of the
qualifications of petitioners Abayon and Palparan.

WHEREFORE, the Court DISMISSES the consolidated petitions and AFFIRMS the Order dated July 16, 2009 and
Resolution 09-183 dated September 17, 2009 in HRET Case 07-041 of the House of Representatives Electoral Tribunal
as well as its Order dated July 23, 2009 and Resolution 09-178 dated September 10, 2009 in HRET Case 07-040.

SO ORDERED.

Age, Citizenship, Residence, Literacy and other Qualifications (Legislative Department)


- David vs Poe-Llamanzares, GR 221538, Sep 20, 2016 [SEE PAGE 221]

Incompatible and Prohibited Offices, Article VI , Sections 13, 14


- Liban v Gordon, 593 SCRA 68 (2009) and 639 SCRA 709 (2011)

The Case

This is a petition to declare Senator Richard J. Gordon (respondent) as having forfeited his seat in the Senate.

The Facts

Petitioners Dante V. Liban, Reynaldo M. Bernardo, and Salvador M. Viari (petitioners) filed with this Court a Petition to
Declare Richard J. Gordon as Having Forfeited His Seat in the Senate. Petitioners are officers of the Board of Directors of
the Quezon City Red Cross Chapter while respondent is Chairman of the Philippine National Red Cross (PNRC) Board of
Governors.

During respondent’s incumbency as a member of the Senate of the Philippines, 1 he was elected Chairman of the PNRC
during the 23 February 2006 meeting of the PNRC Board of Governors. Petitioners allege that by accepting the
chairmanship of the PNRC Board of Governors, respondent has ceased to be a member of the Senate as provided in
Section 13, Article VI of the Constitution, which reads:

SEC. 13. No Senator or Member of the House of Representatives may hold any other office or employment in the
Government, or any subdivision, agency, or instrumentality thereof, including government-owned or controlled
corporations or their subsidiaries, during his term without forfeiting his seat. Neither shall he be appointed to any office
which may have been created or the emoluments thereof increased during the term for which he was elected.

Petitioners cite Camporedondo v. NLRC,2 which held that the PNRC is a government-owned or controlled corporation.
Petitioners claim that in accepting and holding the position of Chairman of the PNRC Board of Governors, respondent has
automatically forfeited his seat in the Senate, pursuant to Flores v. Drilon,3 which held that incumbent national legislators
lose their elective posts upon their appointment to another government office.

In his Comment, respondent asserts that petitioners have no standing to file this petition which appears to be an action for
quo warranto, since the petition alleges that respondent committed an act which, by provision of law, constitutes a ground
for forfeiture of his public office. Petitioners do not claim to be entitled to the Senate office of respondent. Under Section 5,
Rule 66 of the Rules of Civil Procedure, only a person claiming to be entitled to a public office usurped or unlawfully held
by another may bring an action for quo warranto in his own name. If the petition is one for quo warranto, it is already
barred by prescription since under Section 11, Rule 66 of the Rules of Civil Procedure, the action should be commenced
within one year after the cause of the public officer’s forfeiture of office. In this case, respondent has been working as a
Red Cross volunteer for the past 40 years. Respondent was already Chairman of the PNRC Board of Governors when he
was elected Senator in May 2004, having been elected Chairman in 2003 and re-elected in 2005.

Respondent contends that even if the present petition is treated as a taxpayer’s suit, petitioners cannot be allowed to raise
a constitutional question in the absence of any claim that they suffered some actual damage or threatened injury as a
result of the allegedly illegal act of respondent. Furthermore, taxpayers are allowed to sue only when there is a claim of
illegal disbursement of public funds, or that public money is being diverted to any improper purpose, or where petitioners
seek to restrain respondent from enforcing an invalid law that results in wastage of public funds.

Respondent also maintains that if the petition is treated as one for declaratory relief, this Court would have no jurisdiction
since original jurisdiction for declaratory relief lies with the Regional Trial Court.

Respondent further insists that the PNRC is not a government-owned or controlled corporation and that the prohibition
under Section 13, Article VI of the Constitution does not apply in the present case since volunteer service to the PNRC is
neither an office nor an employment.

In their Reply, petitioners claim that their petition is neither an action for quo warranto nor an action for declaratory relief.
Petitioners maintain that the present petition is a taxpayer’s suit questioning the unlawful disbursement of funds,
considering that respondent has been drawing his salaries and other compensation as a Senator even if he is no longer
entitled to his office. Petitioners point out that this Court has jurisdiction over this petition since it involves a legal or
constitutional issue which is of transcendental importance.

The Issues

Petitioners raise the following issues:

1. Whether the Philippine National Red Cross (PNRC) is a government- owned or controlled corporation;

2. Whether Section 13, Article VI of the Philippine Constitution applies to the case of respondent who is Chairman
of the PNRC and at the same time a Member of the Senate;

3. Whether respondent should be automatically removed as a Senator pursuant to Section 13, Article VI of the
Philippine Constitution; and

4. Whether petitioners may legally institute this petition against respondent. 4

The substantial issue boils down to whether the office of the PNRC Chairman is a government office or an office in a
government-owned or controlled corporation for purposes of the prohibition in Section 13, Article VI of the Constitution.

The Court’s Ruling

We find the petition without merit.

Petitioners Have No Standing to File this Petition

A careful reading of the petition reveals that it is an action for quo warranto. Section 1, Rule 66 of the Rules of Court
provides:

Section 1. Action by Government against individuals. – An action for the usurpation of a public office, position or franchise
may be commenced by a verified petition brought in the name of the Republic of the Philippines against:

(a) A person who usurps, intrudes into, or unlawfully holds or exercises a public office, position or franchise;

(b) A public officer who does or suffers an act which by provision of law, constitutes a ground for the forfeiture of
his office; or

(c) An association which acts as a corporation within the Philippines without being legally incorporated or without
lawful authority so to act. (Emphasis supplied)

Petitioners allege in their petition that:

4. Respondent became the Chairman of the PNRC when he was elected as such during the First Regular
Luncheon-Meeting of the Board of Governors of the PNRC held on February 23, 2006, the minutes of which is
hereto attached and made integral part hereof as Annex "A."

5. Respondent was elected as Chairman of the PNRC Board of Governors, during his incumbency as a Member of
the House of Senate of the Congress of the Philippines, having been elected as such during the national elections
last May 2004.

6. Since his election as Chairman of the PNRC Board of Governors, which position he duly accepted, respondent
has been exercising the powers and discharging the functions and duties of said office, despite the fact that he is
still a senator.

7. It is the respectful submission of the petitioner[s] that by accepting the chairmanship of the Board of Governors
of the PNRC, respondent has ceased to be a Member of the House of Senate as provided in Section 13, Article VI
of the Philippine Constitution, x x x

xxxx

10. It is respectfully submitted that in accepting the position of Chairman of the Board of Governors of the PNRC
on February 23, 2006, respondent has automatically forfeited his seat in the House of Senate and, therefore, has
long ceased to be a Senator, pursuant to the ruling of this Honorable Court in the case of FLORES, ET AL. VS.
DRILON AND GORDON, G.R. No. 104732, x x x

11. Despite the fact that he is no longer a senator, respondent continues to act as such and still performs the
powers, functions and duties of a senator, contrary to the constitution, law and jurisprudence.

12. Unless restrained, therefore, respondent will continue to falsely act and represent himself as a senator or
member of the House of Senate, collecting the salaries, emoluments and other compensations, benefits and
privileges appertaining and due only to the legitimate senators, to the damage, great and irreparable injury of the
Government and the Filipino people.5 (Emphasis supplied)

Thus, petitioners are alleging that by accepting the position of Chairman of the PNRC Board of Governors, respondent has
automatically forfeited his seat in the Senate. In short, petitioners filed an action for usurpation of public office against
respondent, a public officer who allegedly committed an act which constitutes a ground for the forfeiture of his public
office. Clearly, such an action is for quo warranto, specifically under Section 1(b), Rule 66 of the Rules of Court.

Quo warranto is generally commenced by the Government as the proper party plaintiff. However, under Section 5, Rule 66
of the Rules of Court, an individual may commence such an action if he claims to be entitled to the public office allegedly
usurped by another, in which case he can bring the action in his own name. The person instituting quo warranto
proceedings in his own behalf must claim and be able to show that he is entitled to the office in dispute, otherwise the
action may be dismissed at any stage. 6 In the present case, petitioners do not claim to be entitled to the Senate office of
respondent. Clearly, petitioners have no standing to file the present petition.

Even if the Court disregards the infirmities of the petition and treats it as a taxpayer’s suit, the petition would still fail on the
merits.

PNRC is a Private Organization Performing Public Functions

On 22 March 1947, President Manuel A. Roxas signed Republic Act No. 95, 7 otherwise known as the PNRC Charter. The
PNRC is a non-profit, donor-funded, voluntary, humanitarian organization, whose mission is to bring timely, effective, and
compassionate humanitarian assistance for the most vulnerable without consideration of nationality, race, religion, gender,
social status, or political affiliation.8 The PNRC provides six major services: Blood Services, Disaster Management, Safety
Services, Community Health and Nursing, Social Services and Voluntary Service. 9

The Republic of the Philippines, adhering to the Geneva Conventions, established the PNRC as a voluntary organization
for the purpose contemplated in the Geneva Convention of 27 July 1929. 10 The Whereas clauses of the PNRC Charter
read:

WHEREAS, there was developed at Geneva, Switzerland, on August 22, 1864, a convention by which the nations of the
world were invited to join together in diminishing, so far lies within their power, the evils inherent in war;

WHEREAS, more than sixty nations of the world have ratified or adhered to the subsequent revision of said convention,
namely the "Convention of Geneva of July 29 [sic], 1929 for the Amelioration of the Condition of the Wounded and Sick of
Armies in the Field" (referred to in this Charter as the Geneva Red Cross Convention);

WHEREAS, the Geneva Red Cross Convention envisages the establishment in each country of a voluntary organization
to assist in caring for the wounded and sick of the armed forces and to furnish supplies for that purpose;

WHEREAS, the Republic of the Philippines became an independent nation on July 4, 1946 and proclaimed its adherence
to the Geneva Red Cross Convention on February 14, 1947, and by that action indicated its desire to participate with the
nations of the world in mitigating the suffering caused by war and to establish in the Philippines a voluntary organization
for that purpose as contemplated by the Geneva Red Cross Convention;

WHEREAS, there existed in the Philippines since 1917 a Charter of the American National Red Cross which must be
terminated in view of the independence of the Philippines; and

WHEREAS, the volunteer organizations established in the other countries which have ratified or adhered to the Geneva
Red Cross Convention assist in promoting the health and welfare of their people in peace and in war, and through their
mutual assistance and cooperation directly and through their international organizations promote better understanding and
sympathy among the peoples of the world. (Emphasis supplied)

The PNRC is a member National Society of the International Red Cross and Red Crescent Movement (Movement), which
is composed of the International Committee of the Red Cross (ICRC), the International Federation of Red Cross and Red
Crescent Societies (International Federation), and the National Red Cross and Red Crescent Societies (National
Societies). The Movement is united and guided by its seven Fundamental Principles:

1. HUMANITY – The International Red Cross and Red Crescent Movement, born of a desire to bring assistance
without discrimination to the wounded on the battlefield, endeavors, in its international and national capacity, to
prevent and alleviate human suffering wherever it may be found. Its purpose is to protect life and health and to
ensure respect for the human being. It promotes mutual understanding, friendship, cooperation and lasting peace
amongst all peoples.

2. IMPARTIALITY – It makes no discrimination as to nationality, race, religious beliefs, class or political opinions. It
endeavors to relieve the suffering of individuals, being guided solely by their needs, and to give priority to the most
urgent cases of distress.

3. NEUTRALITY – In order to continue to enjoy the confidence of all, the Movement may not take sides in
hostilities or engage at any time in controversies of a political, racial, religious or ideological nature.
4. INDEPENDENCE – The Movement is independent. The National Societies, while auxiliaries in the humanitarian
services of their governments and subject to the laws of their respective countries, must always maintain their
autonomy so that they may be able at all times to act in accordance with the principles of the Movement.

5. VOLUNTARY SERVICE – It is a voluntary relief movement not prompted in any manner by desire for gain.

6. UNITY – There can be only one Red Cross or one Red Crescent Society in any one country. It must be open to
all. It must carry on its humanitarian work throughout its territory.

7. UNIVERSALITY – The International Red Cross and Red Crescent Movement, in which all Societies have equal
status and share equal responsibilities and duties in helping each other, is worldwide. (Emphasis supplied)

The Fundamental Principles provide a universal standard of reference for all members of the Movement. The PNRC, as a
member National Society of the Movement, has the duty to uphold the Fundamental Principles and ideals of the
Movement. In order to be recognized as a National Society, the PNRC has to be autonomous and must operate in
conformity with the Fundamental Principles of the Movement. 11

The reason for this autonomy is fundamental. To be accepted by warring belligerents as neutral workers during
international or internal armed conflicts, the PNRC volunteers must not be seen as belonging to any side of the armed
conflict. In the Philippines where there is a communist insurgency and a Muslim separatist rebellion, the PNRC cannot be
seen as government-owned or controlled, and neither can the PNRC volunteers be identified as government personnel or
as instruments of government policy. Otherwise, the insurgents or separatists will treat PNRC volunteers as enemies
when the volunteers tend to the wounded in the battlefield or the displaced civilians in conflict areas.

Thus, the PNRC must not only be, but must also be seen to be, autonomous, neutral and independent in order to conduct
its activities in accordance with the Fundamental Principles. The PNRC must not appear to be an instrument or agency
that implements government policy; otherwise, it cannot merit the trust of all and cannot effectively carry out its mission as
a National Red Cross Society.12 It is imperative that the PNRC must be autonomous, neutral, and independent in relation
to the State.

To ensure and maintain its autonomy, neutrality, and independence, the PNRC cannot be owned or controlled by the
government. Indeed, the Philippine government does not own the PNRC. The PNRC does not have government assets
and does not receive any appropriation from the Philippine Congress. 13 The PNRC is financed primarily by contributions
from private individuals and private entities obtained through solicitation campaigns organized by its Board of Governors,
as provided under Section 11 of the PNRC Charter:

SECTION 11. As a national voluntary organization, the Philippine National Red Cross shall be financed primarily by
contributions obtained through solicitation campaigns throughout the year which shall be organized by the Board of
Governors and conducted by the Chapters in their respective jurisdictions. These fund raising campaigns shall be
conducted independently of other fund drives by other organizations. (Emphasis supplied)

The government does not control the PNRC. Under the PNRC Charter, as amended, only six of the thirty members of the
PNRC Board of Governors are appointed by the President of the Philippines. Thus, twenty-four members, or four-fifths
(4/5), of the PNRC Board of Governors are not appointed by the President. Section 6 of the PNRC Charter, as amended,
provides:

SECTION 6. The governing powers and authority shall be vested in a Board of Governors composed of thirty members,
six of whom shall be appointed by the President of the Philippines, eighteen shall be elected by chapter delegates in
biennial conventions and the remaining six shall be selected by the twenty-four members of the Board already chosen. x x
x.

Thus, of the twenty-four members of the PNRC Board, eighteen are elected by the chapter delegates of the PNRC, and
six are elected by the twenty-four members already chosen — a select group where the private sector members have
three-fourths majority. Clearly, an overwhelming majority of four-fifths of the PNRC Board are elected or chosen by the
private sector members of the PNRC.

The PNRC Board of Governors, which exercises all corporate powers of the PNRC, elects the PNRC Chairman and all
other officers of the PNRC. The incumbent Chairman of PNRC, respondent Senator Gordon, was elected, as all PNRC
Chairmen are elected, by a private sector-controlled PNRC Board four-fifths of whom are private sector members of the
PNRC. The PNRC Chairman is not appointed by the President or by any subordinate government official.

Under Section 16, Article VII of the Constitution,14 the President appoints all officials and employees in the Executive
branch whose appointments are vested in the President by the Constitution or by law. The President also appoints those
whose appointments are not otherwise provided by law. Under this Section 16, the law may also authorize the "heads of
departments, agencies, commissions, or boards" to appoint officers lower in rank than such heads of departments,
agencies, commissions or boards.15 In Rufino v. Endriga,16 the Court explained appointments under Section 16 in this wise:

Under Section 16, Article VII of the 1987 Constitution, the President appoints three groups of officers. The first group
refers to the heads of the Executive departments, ambassadors, other public ministers and consuls, officers of the armed
forces from the rank of colonel or naval captain, and other officers whose appointments are vested in the President by the
Constitution. The second group refers to those whom the President may be authorized by law to appoint. The third group
refers to all other officers of the Government whose appointments are not otherwise provided by law.

Under the same Section 16, there is a fourth group of lower-ranked officers whose appointments Congress may by law
vest in the heads of departments, agencies, commissions, or boards. x x x

xxx

In a department in the Executive branch, the head is the Secretary. The law may not authorize the Undersecretary, acting
as such Undersecretary, to appoint lower-ranked officers in the Executive department. In an agency, the power is vested
in the head of the agency for it would be preposterous to vest it in the agency itself. In a commission, the head is the
chairperson of the commission. In a board, the head is also the chairperson of the board. In the last three situations, the
law may not also authorize officers other than the heads of the agency, commission, or board to appoint lower-ranked
officers.

xxx

The Constitution authorizes Congress to vest the power to appoint lower-ranked officers specifically in the "heads" of the
specified offices, and in no other person. The word "heads" refers to the chairpersons of the commissions or boards and
not to their members, for several reasons.

The President does not appoint the Chairman of the PNRC. Neither does the head of any department, agency,
commission or board appoint the PNRC Chairman. Thus, the PNRC Chairman is not an official or employee of the
Executive branch since his appointment does not fall under Section 16, Article VII of the Constitution. Certainly, the PNRC
Chairman is not an official or employee of the Judiciary or Legislature. This leads us to the obvious conclusion that the
PNRC Chairman is not an official or employee of the Philippine Government. Not being a government official or employee,
the PNRC Chairman, as such, does not hold a government office or employment.

Under Section 17, Article VII of the Constitution,17 the President exercises control over all government offices in the
Executive branch. If an office is legally not under the control of the President, then such office is not part of the
Executive branch. In Rufino v. Endriga,18 the Court explained the President’s power of control over all government offices
as follows:

Every government office, entity, or agency must fall under the Executive, Legislative, or Judicial branches, or must belong
to one of the independent constitutional bodies, or must be a quasi-judicial body or local government unit. Otherwise, such
government office, entity, or agency has no legal and constitutional basis for its existence.

The CCP does not fall under the Legislative or Judicial branches of government. The CCP is also not one of the
independent constitutional bodies. Neither is the CCP a quasi-judicial body nor a local government unit. Thus, the CCP
must fall under the Executive branch. Under the Revised Administrative Code of 1987, any agency "not placed by law or
order creating them under any specific department" falls "under the Office of the President."

Since the President exercises control over "all the executive departments, bureaus, and offices," the President necessarily
exercises control over the CCP which is an office in the Executive branch. In mandating that the President "shall have
control of all executive . . . offices," Section 17, Article VII of the 1987 Constitution does not exempt any executive office —
one performing executive functions outside of the independent constitutional bodies — from the President’s power of
control. There is no dispute that the CCP performs executive, and not legislative, judicial, or quasi-judicial functions.

The President’s power of control applies to the acts or decisions of all officers in the Executive branch. This is true whether
such officers are appointed by the President or by heads of departments, agencies, commissions, or boards. The power of
control means the power to revise or reverse the acts or decisions of a subordinate officer involving the exercise of
discretion.

In short, the President sits at the apex of the Executive branch, and exercises "control of all the executive departments,
bureaus, and offices." There can be no instance under the Constitution where an officer of the Executive branch is outside
the control of the President. The Executive branch is unitary since there is only one President vested with executive power
exercising control over the entire Executive branch. Any office in the Executive branch that is not under the control of the
President is a lost command whose existence is without any legal or constitutional basis. (Emphasis supplied)

An overwhelming four-fifths majority of the PNRC Board are private sector individuals elected to the PNRC Board by the
private sector members of the PNRC. The PNRC Board exercises all corporate powers of the PNRC. The PNRC is
controlled by private sector individuals. Decisions or actions of the PNRC Board are not reviewable by the President. The
President cannot reverse or modify the decisions or actions of the PNRC Board. Neither can the President reverse or
modify the decisions or actions of the PNRC Chairman. It is the PNRC Board that can review, reverse or modify the
decisions or actions of the PNRC Chairman. This proves again that the office of the PNRC Chairman is a private office,
not a government office. 1avvphi1

Although the State is often represented in the governing bodies of a National Society, this can be justified by the need for
proper coordination with the public authorities, and the government representatives may take part in decision-making
within a National Society. However, the freely-elected representatives of a National Society’s active members must remain
in a large majority in a National Society’s governing bodies. 19
The PNRC is not government-owned but privately owned. The vast majority of the thousands of PNRC members are
private individuals, including students. Under the PNRC Charter, those who contribute to the annual fund campaign of the
PNRC are entitled to membership in the PNRC for one year. Thus, any one between 6 and 65 years of age can be a
PNRC member for one year upon contributing ₱35, ₱100, ₱300, ₱500 or ₱1,000 for the year. 20 Even foreigners, whether
residents or not, can be members of the PNRC. Section 5 of the PNRC Charter, as amended by Presidential Decree No.
1264,21 reads:

SEC. 5. Membership in the Philippine National Red Cross shall be open to the entire population in the Philippines
regardless of citizenship. Any contribution to the Philippine National Red Cross Annual Fund Campaign shall entitle the
contributor to membership for one year and said contribution shall be deductible in full for taxation purposes.

Thus, the PNRC is a privately owned, privately funded, and privately run charitable organization. The PNRC is not a
government-owned or controlled corporation.

Petitioners anchor their petition on the 1999 case of Camporedondo v. NLRC, 22 which ruled that the PNRC is a
government-owned or controlled corporation. In ruling that the PNRC is a government-owned or controlled corporation, the
simple test used was whether the corporation was created by its own special charter for the exercise of a public function or
by incorporation under the general corporation law. Since the PNRC was created under a special charter, the Court then
ruled that it is a government corporation. However, the Camporedondo ruling failed to consider the definition of a
government-owned or controlled corporation as provided under Section 2(13) of the Introductory Provisions of the
Administrative Code of 1987:

SEC. 2. General Terms Defined. – x x x

(13) Government-owned or controlled corporation refers to any agency organized as a stock or non-stock corporation,
vested with functions relating to public needs whether governmental or proprietary in nature, and owned by the
Government directly or through its instrumentalities either wholly, or where applicable as in the case of stock corporations,
to the extent of at least fifty-one (51) percent of its capital stock: Provided, That government-owned or controlled
corporations may be further categorized by the Department of the Budget, the Civil Service Commission, and the
Commission on Audit for purposes of the exercise and discharge of their respective powers, functions and responsibilities
with respect to such corporations.(Boldfacing and underscoring supplied)

A government-owned or controlled corporation must be owned by the government, and in the case of a stock corporation,
at least a majority of its capital stock must be owned by the government. In the case of a non-stock corporation, by
analogy at least a majority of the members must be government officials holding such membership by appointment or
designation by the government. Under this criterion, and as discussed earlier, the government does not own or control
PNRC.

The PNRC Charter is Violative of the Constitutional Proscription against the Creation of Private Corporations by Special
Law

The 1935 Constitution, as amended, was in force when the PNRC was created by special charter on 22 March 1947.
Section 7, Article XIV of the 1935 Constitution, as amended, reads:

SEC. 7. The Congress shall not, except by general law, provide for the formation, organization, or regulation of private
corporations, unless such corporations are owned or controlled by the Government or any subdivision or instrumentality
thereof.

The subsequent 1973 and 1987 Constitutions contain similar provisions prohibiting Congress from creating private
corporations except by general law. Section 1 of the PNRC Charter, as amended, creates the PNRC as a "body corporate
and politic," thus:

SECTION 1. There is hereby created in the Republic of the Philippines a body corporate and politic to be the voluntary
organization officially designated to assist the Republic of the Philippines in discharging the obligations set forth in the
Geneva Conventions and to perform such other duties as are inherent upon a National Red Cross Society. The national
headquarters of this Corporation shall be located in Metropolitan Manila. (Emphasis supplied)

In Feliciano v. Commission on Audit, 23 the Court explained the constitutional provision prohibiting Congress from creating
private corporations in this wise:

We begin by explaining the general framework under the fundamental law. The Constitution recognizes two classes of
corporations. The first refers to private corporations created under a general law. The second refers to government-owned
or controlled corporations created by special charters. Section 16, Article XII of the Constitution provides:

Sec. 16. The Congress shall not, except by general law, provide for the formation, organization, or regulation of private
corporations. Government-owned or controlled corporations may be created or established by special charters in the
interest of the common good and subject to the test of economic viability.

The Constitution emphatically prohibits the creation of private corporations except by general law applicable to all citizens.
The purpose of this constitutional provision is to ban private corporations created by special charters, which historically
gave certain individuals, families or groups special privileges denied to other citizens.
In short, Congress cannot enact a law creating a private corporation with a special charter. Such legislation would be
unconstitutional. Private corporations may exist only under a general law. If the corporation is private, it must necessarily
exist under a general law. Stated differently, only corporations created under a general law can qualify as private
corporations. Under existing laws, the general law is the Corporation Code, except that the Cooperative Code governs the
incorporation of cooperatives.

The Constitution authorizes Congress to create government-owned or controlled corporations through special charters.
Since private corporations cannot have special charters, it follows that Congress can create corporations with special
charters only if such corporations are government-owned or controlled. 24 (Emphasis supplied)

In Feliciano, the Court held that the Local Water Districts are government-owned or controlled corporations since they
exist by virtue of Presidential Decree No. 198, which constitutes their special charter. The seed capital assets of the Local
Water Districts, such as waterworks and sewerage facilities, were public property which were managed, operated by or
under the control of the city, municipality or province before the assets were transferred to the Local Water Districts. The
Local Water Districts also receive subsidies and loans from the Local Water Utilities Administration (LWUA). In fact, under
the 2009 General Appropriations Act,25 the LWUA has a budget amounting to ₱400,000,000 for its subsidy
requirements.26 There is no private capital invested in the Local Water Districts. The capital assets and operating
funds of the Local Water Districts all come from the government, either through transfer of assets, loans, subsidies or the
income from such assets or funds.

The government also controls the Local Water Districts because the municipal or city mayor, or the provincial governor,
appoints all the board directors of the Local Water Districts. Furthermore, the board directors and other personnel of the
Local Water Districts are government employees subject to civil service laws and anti-graft laws. Clearly, the Local Water
Districts are considered government-owned or controlled corporations not only because of their creation by special charter
but also because the government in fact owns and controls the Local Water Districts.

Just like the Local Water Districts, the PNRC was created through a special charter. However, unlike the Local Water
Districts, the elements of government ownership and control are clearly lacking in the PNRC. Thus, although the PNRC is
created by a special charter, it cannot be considered a government-owned or controlled corporation in the absence of the
essential elements of ownership and control by the government. In creating the PNRC as a corporate entity, Congress
was in fact creating a private corporation. However, the constitutional prohibition against the creation of private
corporations by special charters provides no exception even for non-profit or charitable corporations. Consequently, the
PNRC Charter, insofar as it creates the PNRC as a private corporation and grants it corporate powers, 27 is void for being
unconstitutional. Thus, Sections 1,28 2,29 3,30 4(a),31 5,32 6,33 7,34 8,35 9,36 10,37 11,38 12,39 and 1340 of the PNRC Charter, as
amended, are void.

The other provisions41 of the PNRC Charter remain valid as they can be considered as a recognition by the State that the
unincorporated PNRC is the local National Society of the International Red Cross and Red Crescent Movement, and thus
entitled to the benefits, exemptions and privileges set forth in the PNRC Charter. The other provisions of the PNRC
Charter implement the Philippine Government’s treaty obligations under Article 4(5) of the Statutes of the International
Red Cross and Red Crescent Movement, which provides that to be recognized as a National Society, the Society must be
"duly recognized by the legal government of its country on the basis of the Geneva Conventions and of the national
legislation as a voluntary aid society, auxiliary to the public authorities in the humanitarian field."

In sum, we hold that the office of the PNRC Chairman is not a government office or an office in a government-owned or
controlled corporation for purposes of the prohibition in Section 13, Article VI of the 1987 Constitution. However, since the
PNRC Charter is void insofar as it creates the PNRC as a private corporation, the PNRC should incorporate under the
Corporation Code and register with the Securities and Exchange Commission if it wants to be a private corporation.

WHEREFORE, we declare that the office of the Chairman of the Philippine National Red Cross is not a government office
or an office in a government-owned or controlled corporation for purposes of the prohibition in Section 13, Article VI of the
1987 Constitution. We also declare that Sections 1, 2, 3, 4(a), 5, 6, 7, 8, 9, 10, 11, 12, and 13 of the Charter of the
Philippine National Red Cross, or Republic Act No. 95, as amended by Presidential Decree Nos. 1264 and 1643, are
VOID because they create the PNRC as a private corporation or grant it corporate powers.

SO ORDERED.

Discipline of members, Article VI , Section 16 (3)


- Pobre v Defensor-Santiago, 597 SCRA 1 (2009)

In his sworn letter/complaint dated December 22, 2006, with enclosures, Antero J. Pobre invites the Court’s attention to
the following excerpts of Senator Miriam Defensor-Santiago’s speech delivered on the Senate floor:

x x x I am not angry. I am irate. I am foaming in the mouth. I am homicidal. I am suicidal. I am humiliated, debased,
degraded. And I am not only that, I feel like throwing up to be living my middle years in a country of this nature. I am
nauseated. I spit on the face of Chief Justice Artemio Panganiban and his cohorts in the Supreme Court, I am no longer
interested in the position [of Chief Justice] if I was to be surrounded by idiots. I would rather be in another environment but
not in the Supreme Court of idiots x x x.
To Pobre, the foregoing statements reflected a total disrespect on the part of the speaker towards then Chief Justice
Artemio Panganiban and the other members of the Court and constituted direct contempt of court. Accordingly, Pobre
asks that disbarment proceedings or other disciplinary actions be taken against the lady senator.

In her comment on the complaint dated April 25, 2007, Senator Santiago, through counsel, does not deny making the
aforequoted statements. She, however, explained that those statements were covered by the constitutional provision on
parliamentary immunity, being part of a speech she delivered in the discharge of her duty as member of Congress or its
committee. The purpose of her speech, according to her, was to bring out in the open controversial anomalies in
governance with a view to future remedial legislation. She averred that she wanted to expose what she believed "to be an
unjust act of the Judicial Bar Council [JBC]," which, after sending out public invitations for nomination to the soon to-be
vacated position of Chief Justice, would eventually inform applicants that only incumbent justices of the Supreme Court
would qualify for nomination. She felt that the JBC should have at least given an advanced advisory that non-sitting
members of the Court, like her, would not be considered for the position of Chief Justice.

The immunity Senator Santiago claims is rooted primarily on the provision of Article VI, Section 11 of the Constitution,
which provides: "A Senator or Member of the House of Representative shall, in all offenses punishable by not more than
six years imprisonment, be privileged from arrest while the Congress is in session. No member shall be questioned nor
be held liable in any other place for any speech or debate in the Congress or in any committee thereof." Explaining
the import of the underscored portion of the provision, the Court, in Osmeña, Jr. v. Pendatun, said:

Our Constitution enshrines parliamentary immunity which is a fundamental privilege cherished in every legislative
assembly of the democratic world. As old as the English Parliament, its purpose "is to enable and encourage a
representative of the public to discharge his public trust with firmness and success" for "it is indispensably necessary that
he should enjoy the fullest liberty of speech and that he should be protected from resentment of every one, however,
powerful, to whom the exercise of that liberty may occasion offense." 1

As American jurisprudence puts it, this legislative privilege is founded upon long experience and arises as a means of
perpetuating inviolate the functioning process of the legislative department. Without parliamentary immunity, parliament, or
its equivalent, would degenerate into a polite and ineffective debating forum. Legislators are immune from deterrents to
the uninhibited discharge of their legislative duties, not for their private indulgence, but for the public good. The privilege
would be of little value if they could be subjected to the cost and inconvenience and distractions of a trial upon a
conclusion of the pleader, or to the hazard of a judgment against them based upon a judge’s speculation as to the
motives.2

This Court is aware of the need and has in fact been in the forefront in upholding the institution of parliamentary immunity
and promotion of free speech. Neither has the Court lost sight of the importance of the legislative and oversight functions
of the Congress that enable this representative body to look diligently into every affair of government, investigate and
denounce anomalies, and talk about how the country and its citizens are being served. Courts do not interfere with the
legislature or its members in the manner they perform their functions in the legislative floor or in committee rooms. Any
claim of an unworthy purpose or of the falsity and mala fides of the statement uttered by the member of the Congress
does not destroy the privilege.3 The disciplinary authority of the assembly 4 and the voters, not the courts, can properly
discourage or correct such abuses committed in the name of parliamentary immunity. 5

For the above reasons, the plea of Senator Santiago for the dismissal of the complaint for disbarment or disciplinary action
is well taken. Indeed, her privilege speech is not actionable criminally or in a disciplinary proceeding under the Rules of
Court. It is felt, however, that this could not be the last word on the matter.

The Court wishes to express its deep concern about the language Senator Santiago, a member of the Bar, used in her
speech and its effect on the administration of justice. To the Court, the lady senator has undoubtedly crossed the limits of
decency and good professional conduct. It is at once apparent that her statements in question were intemperate and
highly improper in substance. To reiterate, she was quoted as stating that she wanted "to spit on the face of Chief Justice
Artemio Panganiban and his cohorts in the Supreme Court," and calling the Court a "Supreme Court of idiots."

The lady senator alluded to In Re: Vicente Sotto.6 We draw her attention to the ensuing passage in Sotto that she should
have taken to heart in the first place:

x x x [I]f the people lose their confidence in the honesty and integrity of this Court and believe that they cannot expect
justice therefrom, they might be driven to take the law into their own hands, and disorder and perhaps chaos would be the
result.
1avvphi1

No lawyer who has taken an oath to maintain the respect due to the courts should be allowed to erode the people’s faith in
the judiciary. In this case, the lady senator clearly violated Canon 8, Rule 8.01 and Canon 11 of the Code of Professional
Responsibility, which respectively provide:

Canon 8, Rule 8.01.––A lawyer shall not, in his professional dealings, use language which is abusive, offensive or
otherwise improper.

Canon 11.––A lawyer shall observe and maintain the respect due to the courts and to the judicial officers and should insist
on similar conduct by others.
Senator/Atty. Santiago is a cut higher than most lawyers. Her achievements speak for themselves. She was a former
Regional Trial Court judge, a law professor, an oft-cited authority on constitutional and international law, an author of
numerous law textbooks, and an elected senator of the land. Needless to stress, Senator Santiago, as a member of the
Bar and officer of the court, like any other, is duty-bound to uphold the dignity and authority of this Court and to maintain
the respect due its members. Lawyers in public service are keepers of public faith and are burdened with the higher
degree of social responsibility, perhaps higher than their brethren in private practice. 7 Senator Santiago should have
known, as any perceptive individual, the impact her statements would make on the people’s faith in the integrity of the
courts.

As Senator Santiago alleged, she delivered her privilege speech as a prelude to crafting remedial legislation on the JBC.
This allegation strikes the Court as an afterthought in light of the insulting tenor of what she said. We quote the passage
once more:

x x x I am not angry. I am irate. I am foaming in the mouth. I am homicidal. I am suicidal. I am humiliated, debased,
degraded. And I am not only that, I feel like throwing up to be living my middle years in a country of this nature. I am
nauseated. I spit on the face of Chief Justice Artemio Panganiban and his cohorts in the Supreme Court, I am no longer
interested in the position [of Chief Justice] if I was to be surrounded by idiots. I would rather be in another environment but
not in the Supreme Court of idiots x x x. (Emphasis ours.)

A careful re-reading of her utterances would readily show that her statements were expressions of personal anger and
frustration at not being considered for the post of Chief Justice. In a sense, therefore, her remarks were outside the pale of
her official parliamentary functions. Even parliamentary immunity must not be allowed to be used as a vehicle to ridicule,
demean, and destroy the reputation of the Court and its magistrates, nor as armor for personal wrath and disgust.
Authorities are agreed that parliamentary immunity is not an individual privilege accorded the individual members of the
Parliament or Congress for their personal benefit, but rather a privilege for the benefit of the people and the institution that
represents them.

To be sure, Senator Santiago could have given vent to her anger without indulging in insulting rhetoric and offensive
personalities.

Lest it be overlooked, Senator Santiago’s outburst was directly traceable to what she considered as an "unjust act" the
JBC had taken in connection with her application for the position of Chief Justice. But while the JBC functions under the
Court’s supervision, its individual members, save perhaps for the Chief Justice who sits as the JBC’s ex-
officio chairperson,8 have no official duty to nominate candidates for appointment to the position of Chief Justice. The
Court is, thus, at a loss to understand Senator Santiago’s wholesale and indiscriminate assault on the members of the
Court and her choice of critical and defamatory words against all of them.

At any event, equally important as the speech and debate clause of Art. VI, Sec. 11 of the Constitution is Sec. 5(5) of Art.
VIII of the Constitution that provides:

Section 5. The Supreme Court shall have the following powers:

xxxx

(5) Promulgate rules concerning the protection and enforcement of constitutional rights, pleading, practice, and procedure
in all courts, the admission to the practice of the law, the Integrated Bar, and legal assistance to the underprivileged.
(Emphasis ours.)

The Court, besides being authorized to promulgate rules concerning pleading, practice, and procedure in all courts,
exercises specific authority to promulgate rules governing the Integrated Bar with the end in view that the integration of the
Bar will, among other things:

(4) Shield the judiciary, which traditionally cannot defend itself except within its own forum, from the assaults that politics
and self interest may level at it, and assist it to maintain its integrity, impartiality and independence;

xxxx

(11) Enforce rigid ethical standards x x x.9

In Re: Letter Dated 21 February 2005 of Atty. Noel S. Sorreda, 10 we reiterated our pronouncement in Rheem of the
Philippines v. Ferrer11 that the duty of attorneys to the courts can only be maintained by rendering no service involving any
disrespect to the judicial office which they are bound to uphold. The Court wrote in Rheem of the Philippines:

x x x As explicit is the first canon of legal ethics which pronounces that "[i]t is the duty of a lawyer to maintain towards the
Courts a respectful attitude, not for the sake of the temporary incumbent of the judicial office, but for the maintenance of its
supreme importance." That same canon, as a corollary, makes it peculiarly incumbent upon lawyers to support the courts
against "unjust criticism and clamor." And more. The attorney’s oath solemnly binds him to a conduct that should be "with
all good fidelity x x x to the courts."

Also, in Sorreda, the Court revisited its holding in Surigao Mineral Reservation Board v. Cloribel 12 that:
A lawyer is an officer of the courts; he is, "like the court itself, an instrument or agency to advance the ends of justice." His
duty is to uphold the dignity and authority of the courts to which he owes fidelity, "not to promote distrust in the
administration of justice." Faith in the courts, a lawyer should seek to preserve. For, to undermine the judicial edifice "is
disastrous to the continuity of government and to the attainment of the liberties of the people." Thus has it been said of a
lawyer that "[a]s an officer of the court, it is his sworn and moral duty to help build and not destroy unnecessarily that high
esteem and regard towards the courts so essential to the proper administration of justice." 13

The lady senator belongs to the legal profession bound by the exacting injunction of a strict Code. Society has entrusted
that profession with the administration of the law and dispensation of justice. Generally speaking, a lawyer holding a
government office may not be disciplined as a member of the Bar for misconduct committed while in the discharge of
official duties, unless said misconduct also constitutes a violation of his/her oath as a lawyer. 14

Lawyers may be disciplined even for any conduct committed in their private capacity, as long as their misconduct reflects
their want of probity or good demeanor, 15 a good character being an essential qualification for the admission to the practice
of law and for continuance of such privilege. When the Code of Professional Responsibility or the Rules of Court speaks of
"conduct" or "misconduct," the reference is not confined to one’s behavior exhibited in connection with the performance of
lawyers’ professional duties, but also covers any misconduct, which––albeit unrelated to the actual practice of their
profession––would show them to be unfit for the office and unworthy of the privileges which their license and the law
invest in them.16

This Court, in its unceasing quest to promote the people’s faith in courts and trust in the rule of law, has consistently
exercised its disciplinary authority on lawyers who, for malevolent purpose or personal malice, attempt to obstruct the
orderly administration of justice, trifle with the integrity of courts, and embarrass or, worse, malign the men and women
who compose them. We have done it in the case of former Senator Vicente Sotto in Sotto, in the case of Atty. Noel
Sorreda in Sorreda, and in the case of Atty. Francisco B. Cruz in Tacordan v. Ang 17 who repeatedly insulted and
threatened the Court in a most insolent manner.

The Court is not hesitant to impose some form of disciplinary sanctions on Senator/Atty. Santiago for what otherwise
would have constituted an act of utter disrespect on her part towards the Court and its members. The factual and legal
circumstances of this case, however, deter the Court from doing so, even without any sign of remorse from her. Basic
constitutional consideration dictates this kind of disposition.

We, however, would be remiss in our duty if we let the Senator’s offensive and disrespectful language that definitely
tended to denigrate the institution pass by. It is imperative on our part to re-instill in Senator/Atty. Santiago her duty to
respect courts of justice, especially this Tribunal, and remind her anew that the parliamentary non-accountability thus
granted to members of Congress is not to protect them against prosecutions for their own benefit, but to enable them, as
the people’s representatives, to perform the functions of their office without fear of being made responsible before the
courts or other forums outside the congressional hall. 18 It is intended to protect members of Congress against government
pressure and intimidation aimed at influencing the decision-making prerogatives of Congress and its members.

The Rules of the Senate itself contains a provision on Unparliamentary Acts and Language that enjoins a Senator from
using, under any circumstance, "offensive or improper language against another Senator or against any public
institution."19 But as to Senator Santiago’s unparliamentary remarks, the Senate President had not apparently called her
to order, let alone referred the matter to the Senate Ethics Committee for appropriate disciplinary action, as the Rules
dictates under such circumstance.20 The lady senator clearly violated the rules of her own chamber. It is unfortunate that
her peers bent backwards and avoided imposing their own rules on her.

Finally, the lady senator questions Pobre’s motives in filing his complaint, stating that disciplinary proceedings must be
undertaken solely for the public welfare. We cannot agree with her more. We cannot overstress that the senator’s use of
intemperate language to demean and denigrate the highest court of the land is a clear violation of the duty of respect
lawyers owe to the courts.21

Finally, the Senator asserts that complainant Pobre has failed to prove that she in fact made the statements in question.
Suffice it to say in this regard that, although she has not categorically denied making such statements, she has
unequivocally said making them as part of her privilege speech. Her implied admission is good enough for the Court.

WHEREFORE, the letter-complaint of Antero J. Pobre against Senator/Atty. Miriam Defensor-Santiago is, conformably to
Art. VI, Sec. 11 of the Constitution, DISMISSED. SO ORDERED.

Powers of Congress
1. Legislative
- Makalintal v Comelec, GR 157013, Jul 10, 2003

Before the Court is a petition for certiorari and prohibition filed by Romulo B. Macalintal, a member of the Philippine Bar,
seeking a declaration that certain provisions of Republic Act No. 9189 (The Overseas Absentee Voting Act of 2003)1 suffer
from constitutional infirmity. Claiming that he has actual and material legal interest in the subject matter of this case in
seeing to it that public funds are properly and lawfully used and appropriated, petitioner filed the instant petition as a
taxpayer and as a lawyer.

The Court upholds the right of petitioner to file the present petition.
R.A. No. 9189, entitled, "An Act Providing for A System of Overseas Absentee Voting by Qualified Citizens of the
Philippines Abroad, Appropriating Funds Therefor, and for Other Purposes," appropriates funds under Section 29 thereof
which provides that a supplemental budget on the General Appropriations Act of the year of its enactment into law shall
provide for the necessary amount to carry out its provisions. Taxpayers, such as herein petitioner, have the right to
restrain officials from wasting public funds through the enforcement of an unconstitutional statute. 2 The Court has held that
they may assail the validity of a law appropriating public funds 3 because expenditure of public funds by an officer of the
State for the purpose of executing an unconstitutional act constitutes a misapplication of such funds. 4

The challenged provision of law involves a public right that affects a great number of citizens. The Court has adopted the
policy of taking jurisdiction over cases whenever the petitioner has seriously and convincingly presented an issue of
transcendental significance to the Filipino people. This has been explicitly pronounced in Kapatiran ng mga Naglilingkod
sa Pamahalaan ng Pilipinas, Inc. vs. Tan,5 where the Court held:

Objections to taxpayers’ suit for lack of sufficient personality standing, or interest are, however, in the main
procedural matters. Considering the importance to the public of the cases at bar, and in keeping with the Court’s
duty, under the 1987 Constitution, to determine whether or not the other branches of government have kept
themselves within the limits of the Constitution and the laws and that they have not abused the discretion given to
them, the Court has brushed aside technicalities of procedure and has taken cognizance of these petitions. 6

Indeed, in this case, the Court may set aside procedural rules as the constitutional right of suffrage of a considerable
number of Filipinos is involved.

The question of propriety of the instant petition which may appear to be visited by the vice of prematurity as there are no
ongoing proceedings in any tribunal, board or before a government official exercising judicial, quasi-judicial or ministerial
functions as required by Rule 65 of the Rules of Court, dims in light of the importance of the constitutional issues raised by
the petitioner. In Tañada vs. Angara,7 the Court held:

In seeking to nullify an act of the Philippine Senate on the ground that it contravenes the Constitution, the petition
no doubt raises a justiciable controversy. Where an action of the legislative branch is seriously alleged to have
infringed the Constitution, it becomes not only the right but in fact the duty of the judiciary to settle the dispute.
"The question thus posed is judicial rather than political. The duty (to adjudicate) remains to assure that the
supremacy of the Constitution is upheld." Once a "controversy as to the application or interpretation of
constitutional provision is raised before this Court (as in the instant case), it becomes a legal issue which the Court
is bound by constitutional mandate to decide."

In another case of paramount impact to the Filipino people, it has been expressed that it is illogical to await the adverse
consequences of the law in order to consider the controversy actual and ripe for judicial resolution. 8 In yet another case,
the Court said that:

. . . despite the inhibitions pressing upon the Court when confronted with constitutional issues, it will not hesitate to
declare a law or act invalid when it is convinced that this must be done. In arriving at this conclusion, its only
criterion will be the Constitution and God as its conscience gives it in the light to probe its meaning and discover its
purpose. Personal motives and political considerations are irrelevancies that cannot influence its decisions.
Blandishment is as ineffectual as intimidation, for all the awesome power of the Congress and Executive, the Court
will not hesitate "to make the hammer fall heavily," where the acts of these departments, or of any official, betray
the people’s will as expressed in the Constitution . . .9

The need to consider the constitutional issues raised before the Court is further buttressed by the fact that it is now more
than fifteen years since the ratification of the 1987 Constitution requiring Congress to provide a system for absentee voting
by qualified Filipinos abroad. Thus, strong reasons of public policy demand that the Court resolves the instant
petition10 and determine whether Congress has acted within the limits of the Constitution or if it had gravely abused the
discretion entrusted to it.11

The petitioner raises three principal questions:

A. Does Section 5(d) of Rep. Act No. 9189 allowing the registration of voters who are immigrants or permanent
residents in other countries by their mere act of executing an affidavit expressing their intention to return to the
Philippines, violate the residency requirement in Section 1 of Article V of the Constitution?

B. Does Section 18.5 of the same law empowering the COMELEC to proclaim the winning candidates for national
offices and party list representatives including the President and the Vice-President violate the constitutional
mandate under Section 4, Article VII of the Constitution that the winning candidates for President and the Vice-
President shall be proclaimed as winners by Congress?

C. May Congress, through the Joint Congressional Oversight Committee created in Section 25 of Rep. Act No.
9189, exercise the power to review, revise, amend, and approve the Implementing Rules and Regulations that the
Commission on Elections shall promulgate without violating the independence of the COMELEC under Section 1,
Article IX-A of the Constitution?

The Court will resolve the questions in seriatim.


A. Does Section 5(d) of Rep. Act No. 9189 violate Section 1, Article V of the 1987 Constitution of the Republic of
the Philippines?

Section 5(d) provides:

Sec. 5. Disqualifications. – The following shall be disqualified from voting under this Act:

.........

d) An immigrant or a permanent resident who is recognized as such in the host country, unless he/she executes,
upon registration, an affidavit prepared for the purpose by the Commission declaring that he/she shall resume
actual physical permanent residence in the Philippines not later than three (3) years from approval of his/her
registration under this Act. Such affidavit shall also state that he/she has not applied for citizenship in another
country. Failure to return shall be cause for the removal of the name of the immigrant or permanent resident from
the National Registry of Absentee Voters and his/her permanent disqualification to vote in absentia.

Petitioner posits that Section 5(d) is unconstitutional because it violates Section 1, Article V of the 1987 Constitution which
requires that the voter must be a resident in the Philippines for at least one year and in the place where he proposes to
vote for at least six months immediately preceding an election. Petitioner cites the ruling of the Court in Caasi vs. Court of
Appeals12 to support his claim. In that case, the Court held that a "green card" holder immigrant to the United States is
deemed to have abandoned his domicile and residence in the Philippines.

Petitioner further argues that Section 1, Article V of the Constitution does not allow provisional registration or a promise by
a voter to perform a condition to be qualified to vote in a political exercise; 13 that the legislature should not be allowed to
circumvent the requirement of the Constitution on the right of suffrage by providing a condition thereon which in effect
amends or alters the aforesaid residence requirement to qualify a Filipino abroad to vote. 14 He claims that the right of
suffrage should not be granted to anyone who, on the date of the election, does not possess the qualifications provided for
by Section 1, Article V of the Constitution.

Respondent COMELEC refrained from commenting on this issue. 15

In compliance with the Resolution of the Court, the Solicitor General filed his comment for all public respondents. He
contraposes that the constitutional challenge to Section 5(d) must fail because of the absence of clear and unmistakable
showing that said provision of law is repugnant to the Constitution. He stresses: All laws are presumed to be constitutional;
by the doctrine of separation of powers, a department of government owes a becoming respect for the acts of the other
two departments; all laws are presumed to have adhered to constitutional limitations; the legislature intended to enact a
valid, sensible, and just law.

In addition, the Solicitor General points out that Section 1, Article V of the Constitution is a verbatim reproduction of those
provided for in the 1935 and the 1973 Constitutions. Thus, he cites Co vs. Electoral Tribunal of the House of
Representatives16 wherein the Court held that the term "residence" has been understood to be synonymous with "domicile"
under both Constitutions. He further argues that a person can have only one "domicile" but he can have two residences,
one permanent (the domicile) and the other temporary; 17 and that the definition and meaning given to the term residence
likewise applies to absentee voters. Invoking Romualdez-Marcos vs. COMELEC18 which reiterates the Court’s ruling in
Faypon vs. Quirino,19 the Solicitor General maintains that Filipinos who are immigrants or permanent residents abroad may
have in fact never abandoned their Philippine domicile. 20

Taking issue with the petitioner’s contention that "green card" holders are considered to have abandoned their Philippine
domicile, the Solicitor General suggests that the Court may have to discard its ruling in Caasi vs. Court of Appeals21 in so
far as it relates to immigrants and permanent residents in foreign countries who have executed and submitted their
affidavits conformably with Section 5(d) of R.A. No. 9189. He maintains that through the execution of the requisite
affidavits, the Congress of the Philippines with the concurrence of the President of the Republic had in fact given these
immigrants and permanent residents the opportunity, pursuant to Section 2, Article V of the Constitution, to manifest that
they had in fact never abandoned their Philippine domicile; that indubitably, they would have formally and categorically
expressed the requisite intentions, i.e., "animus manendi" and "animus revertendi;" that Filipino immigrants and permanent
residents abroad possess the unquestionable right to exercise the right of suffrage under Section 1, Article V of the
Constitution upon approval of their registration, conformably with R.A. No. 9189. 22

The seed of the present controversy is the interpretation that is given to the phrase, "qualified citizens of the Philippines
abroad" as it appears in R.A. No. 9189, to wit:

SEC. 2. Declaration of Policy. – It is the prime duty of the State to provide a system of honest and orderly overseas
absentee voting that upholds the secrecy and sanctity of the ballot. Towards this end, the State ensures equal opportunity
to all qualified citizens of the Philippines abroad in the exercise of this fundamental right.

SEC. 3. Definition of Terms. – For purposes of this Act:

a) "Absentee Voting" refers to the process by which qualified citizens of the Philippines abroad,
exercise their right to vote;

. . . (Emphasis supplied)
f) "Overseas Absentee Voter" refers to a citizen of the Philippines who is qualified to register and
vote under this Act, not otherwise disqualified by law, who is abroad on the day of elections. (Emphasis
supplied)

SEC. 4. Coverage. – All citizens of the Philippines abroad, who are not otherwise disqualified by law, at
least eighteen (18) years of age on the day of elections, may vote for president, vice-president, senators and
party-list representatives. (Emphasis supplied)

in relation to Sections 1 and 2, Article V of the Constitution which read:

SEC. 1. Suffrage may be exercised by all citizens of the Philippines not otherwise disqualified by law, who are at
least eighteen years of age, and who shall have resided in the Philippines for at least one year and in the place
wherein they propose to vote for at least six months immediately preceding the election. No literacy, property, or
other substantive requirement shall be imposed on the exercise of suffrage.

SEC. 2. The Congress shall provide a system for securing the secrecy and sanctity of the ballot as well as a
system for absentee voting by qualified Filipinos abroad.

. . . . . . . . . (Emphasis supplied)

Section 1, Article V of the Constitution specifically provides that suffrage may be exercised by (1) all citizens of the
Philippines, (2) not otherwise disqualified by law, (3) at least eighteen years of age, (4) who are residents in the
Philippines for at least one year and in the place where they propose to vote for at least six months immediately preceding
the election. Under Section 5(d) of R.A. No. 9189, one of those disqualified from voting is an immigrant or permanent
resident who is recognized as such in the host country unless he/she executes an affidavit declaring that he/she shall
resume actual physical permanent residence in the Philippines not later than three years from approval of his/her
registration under said Act.

Petitioner questions the rightness of the mere act of execution of an affidavit to qualify the Filipinos abroad who are
immigrants or permanent residents, to vote. He focuses solely on Section 1, Article V of the Constitution in ascribing
constitutional infirmity to Section 5(d) of R.A. No. 9189, totally ignoring the provisions of Section 2 empowering Congress
to provide a system for absentee voting by qualified Filipinos abroad.

A simple, cursory reading of Section 5(d) of R.A. No. 9189 may indeed give the impression that it contravenes Section 1,
Article V of the Constitution. Filipino immigrants and permanent residents overseas are perceived as having left and
abandoned the Philippines to live permanently in their host countries and therefore, a provision in the law enfranchising
those who do not possess the residency requirement of the Constitution by the mere act of executing an affidavit
expressing their intent to return to the Philippines within a given period, risks a declaration of unconstitutionality. However,
the risk is more apparent than real.

The Constitution is the fundamental and paramount law of the nation to which all other laws must conform and in
accordance with which all private rights must be determined and all public authority administered. 23 Laws that do not
conform to the Constitution shall be stricken down for being unconstitutional.

Generally, however, all laws are presumed to be constitutional. In Peralta vs. COMELEC, the Court said:

. . . An act of the legislature, approved by the executive, is presumed to be within constitutional limitations. The
responsibility of upholding the Constitution rests not on the courts alone but on the legislature as well. The
question of the validity of every statute is first determined by the legislative department of the government itself. 24

Thus, presumption of constitutionality of a law must be overcome convincingly:

. . . To declare a law unconstitutional, the repugnancy of that law to the Constitution must be clear and
unequivocal, for even if a law is aimed at the attainment of some public good, no infringement of constitutional
rights is allowed. To strike down a law there must be a clear showing that what the fundamental law condemns or
prohibits, the statute allows it to be done.25

As the essence of R.A. No. 9189 is to enfranchise overseas qualified Filipinos, it behooves the Court to take a holistic view
of the pertinent provisions of both the Constitution and R.A. No. 9189. It is a basic rule in constitutional construction that
the Constitution should be construed as a whole. In Chiongbian vs. De Leon,26 the Court held that a constitutional
provision should function to the full extent of its substance and its terms, not by itself alone, but in conjunction with all other
provisions of that great document. Constitutional provisions are mandatory in character unless, either by express
statement or by necessary implication, a different intention is manifest. 27 The intent of the Constitution may be drawn
primarily from the language of the document itself. Should it be ambiguous, the Court may consider the intent of its
framers through their debates in the constitutional convention. 28

R.A. No. 9189 was enacted in obeisance to the mandate of the first paragraph of Section 2, Article V of the Constitution
that Congress shall provide a system for voting by qualified Filipinos abroad. It must be stressed that Section 2 does not
provide for the parameters of the exercise of legislative authority in enacting said law. Hence, in the absence of
restrictions, Congress is presumed to have duly exercised its function as defined in Article VI (The Legislative Department)
of the Constitution.
To put matters in their right perspective, it is necessary to dwell first on the significance of absentee voting. The concept of
absentee voting is relatively new. It is viewed thus:

The method of absentee voting has been said to be completely separable and distinct from the regular system of
voting, and to be a new and different manner of voting from that previously known, and an exception to the
customary and usual manner of voting. The right of absentee and disabled voters to cast their ballots at an
election is purely statutory; absentee voting was unknown to, and not recognized at, the common law.

Absentee voting is an outgrowth of modern social and economic conditions devised to accommodate those
engaged in military or civil life whose duties make it impracticable for them to attend their polling places on the day
of election, and the privilege of absentee voting may flow from constitutional provisions or be conferred by
statutes, existing in some jurisdictions, which provide in varying terms for the casting and reception of ballots by
soldiers and sailors or other qualified voters absent on election day from the district or precinct of their residence.

Such statutes are regarded as conferring a privilege and not a right, or an absolute right. When the legislature
chooses to grant the right by statute, it must operate with equality among all the class to which it is
granted; but statutes of this nature may be limited in their application to particular types of elections. The
statutes should be construed in the light of any constitutional provisions affecting registration and
elections, and with due regard to their texts prior to amendment and to predecessor statutes and the decisions
thereunder; they should also be construed in the light of the circumstances under which they were
enacted; and so as to carry out the objects thereof, if this can be done without doing violence to their provisions
and mandates. Further, in passing on statutes regulating absentee voting, the court should look to the
whole and every part of the election laws, the intent of the entire plan, and reasons and spirit of their
adoption, and try to give effect to every portion thereof.29 (Emphasis supplied)

Ordinarily, an absentee is not a resident and vice versa; a person cannot be at the same time, both a resident and an
absentee.30 However, under our election laws and the countless pronouncements of the Court pertaining to elections, an
absentee remains attached to his residence in the Philippines as residence is considered synonymous with domicile.

In Romualdez-Marcos,31 the Court enunciated:

Article 50 of the Civil Code decrees that "[f]or the exercise of civil rights and the fulfillment of civil obligations, the
domicile of natural persons is their place of habitual residence." In Ong vs. Republic, this court took the concept of
domicile to mean an individual’s "permanent home," "a place to which, whenever absent for business or for
pleasure, one intends to return, and depends on facts and circumstances in the sense that they disclose intent."
Based on the foregoing, domicile includes the twin elements of "the fact of residing or physical presence in a fixed
place" and animus manendi, or the intention of returning there permanently.

Residence, in its ordinary conception, implies the factual relationship of an individual to a certain place. It is the
physical presence of a person in a given area, community or country. The essential distinction between residence
and domicile in law is that residence involves the intent to leave when the purpose for which the resident has taken
up his abode ends. One may seek a place for purposes such as pleasure, business, or health. If a person’s intent
be to remain, it becomes his domicile; if his intent is to leave as soon as his purpose is established it is residence.
It is thus, quite perfectly normal for an individual to have different residences in various places. However, a person
can only have a single domicile, unless, for various reasons, he successfully abandons his domicile in favor of
another domicile of choice. In Uytengsu vs. Republic, we laid this distinction quite clearly:

"There is a difference between domicile and residence. ‘Residence’ is used to indicate a place of abode,
whether permanent or temporary; ‘domicile’ denotes a fixed permanent residence to which, when absent,
one has the intention of returning. A man may have a residence in one place and a domicile in
another. Residence is not domicile, but domicile is residence coupled with the intention to remain for an
unlimited time. A man can have but one domicile for the same purpose at any time, but he may have
numerous places of residence. His place of residence is generally his place of domicile, but it is not by any
means necessarily so since no length of residence without intention of remaining will constitute domicile."

For political purposes the concepts of residence and domicile are dictated by the peculiar criteria of political laws.
As these concepts have evolved in our election law, what has clearly and unequivocally emerged is the fact
that residence for election purposes is used synonymously with domicile.32 (Emphasis supplied)

Aware of the domiciliary legal tie that links an overseas Filipino to his residence in this country, the framers of the
Constitution considered the circumstances that impelled them to require Congress to establish a system for overseas
absentee voting, thus:

MR. OPLE. With respect to Section 1, it is not clear whether the right of suffrage, which here has a residential
restriction, is not denied to citizens temporarily residing or working abroad. Based on the statistics of several
government agencies, there ought to be about two million such Filipinos at this time. Commissioner Bernas had
earlier pointed out that these provisions are really lifted from the two previous Constitutions of 1935 and 1973, with
the exception of the last paragraph. They could not therefore have foreseen at that time the phenomenon now
described as the Filipino labor force explosion overseas.
According to government data, there are now about 600,000 contract workers and employees, and although the
major portions of these expatriate communities of workers are to be found in the Middle East, they are scattered in
177 countries in the world.

In a previous hearing of the Committee on Constitutional Commissions and Agencies, the Chairman of the
Commission on Elections, Ramon Felipe, said that there was no insuperable obstacle to making effective the right
of suffrage for Filipinos overseas. Those who have adhered to their Filipino citizenship notwithstanding strong
temptations are exposed to embrace a more convenient foreign citizenship. And those who on their own or under
pressure of economic necessity here, find that they have to detach themselves from their families to work in other
countries with definite tenures of employment. Many of them are on contract employment for one, two, or three
years. They have no intention of changing their residence on a permanent basis, but are technically disqualified
from exercising the right of suffrage in their countries of destination by the residential requirement in Section 1
which says:

Suffrage shall be exercised by all citizens of the Philippines not otherwise disqualified by law, who are
eighteen years of age or over, and who shall have resided in the Philippines for at least one year and in
the place wherein they propose to vote for at least six months preceding the election.

I, therefore, ask the Committee whether at the proper time they might entertain an amendment that will make this
exercise of the right to vote abroad for Filipino citizens an effective, rather than merely a nominal right under this
proposed Constitution.

FR. BERNAS. Certainly, the Committee will consider that. But more than just saying that, I would like to make a
comment on the meaning of "residence" in the Constitution because I think it is a concept that has been discussed
in various decisions of the Supreme Court, particularly in the case of Faypon vs. Quirino, a 1954 case which dealt
precisely with the meaning of "residence" in the Election Law. Allow me to quote:

A citizen may leave the place of his birth to look for greener pastures, as the saying goes, to improve his
lot and that, of course, includes study in other places, practice of his avocation, reengaging in business.
When an election is to be held, the citizen who left his birthplace to improve his lot may decide to return to
his native town, to cast his ballot, but for professional or business reasons, or for any other reason, he may
not absent himself from the place of his professional or business activities.

So, they are here registered as voters as he has the qualifications to be one, and is not willing to give up or
lose the opportunity to choose the officials who are to run the government especially in national elections.
Despite such registration, the animus revertendi to his home, to his domicile or residence of origin has not
forsaken him.

This may be the explanation why the registration of a voter in a place other than his residence of origin has not been
deemed sufficient to consider abandonment or loss of such residence of origin.

In other words, "residence" in this provision refers to two residence qualifications: "residence" in the Philippines
and "residence" in the place where he will vote. As far as residence in the Philippines is concerned, the word
"residence" means domicile, but as far as residence in the place where he will actually cast his ballot is concerned,
the meaning seems to be different. He could have a domicile somewhere else and yet he is a resident of a place
for six months and he is allowed to vote there. So that there may be serious constitutional obstacles to absentee
voting, unless the vote of the person who is absent is a vote which will be considered as cast in the place
of his domicile.

MR. OPLE. Thank you for citing the jurisprudence.

It gives me scant comfort thinking of about two million Filipinos who should enjoy the right of suffrage, at least a
substantial segment of these overseas Filipino communities. The Committee, of course, is aware that when this
Article of the Constitution explicitly and unequivocally extends the right of effective suffrage to Filipinos abroad, this
will call for a logistical exercise of global proportions. In effect, this will require budgetary and administrative
commitments on the part of the Philippine government, mainly through the COMELEC and the Ministry of Foreign
Affairs, and perhaps, a more extensive elaboration of this mechanism that will be put in place to make effective the
right to vote. Therefore, seeking shelter in some wise jurisprudence of the past may not be sufficient to
meet the demands of the right of suffrage for Filipinos abroad that I have mentioned. But I want to thank the
Committee for saying that an amendment to this effect may be entertained at the proper
time. . . . . . . . . . 33 (Emphasis supplied)

Thus, the Constitutional Commission recognized the fact that while millions of Filipinos reside abroad principally for
economic reasons and hence they contribute in no small measure to the economic uplift of this country, their voices are
marginal insofar as the choice of this country’s leaders is concerned.

The Constitutional Commission realized that under the laws then existing and considering the novelty of the system of
absentee voting in this jurisdiction, vesting overseas Filipinos with the right to vote would spawn constitutional problems
especially because the Constitution itself provides for the residency requirement of voters:
MR. REGALADO. Before I act on that, may I inquire from Commissioner Monsod if the term "absentee voting" also
includes transient voting; meaning, those who are, let us say, studying in Manila need not go back to their places
of registration, for instance, in Mindanao, to cast their votes.

MR. MONSOD. I think our provision is for absentee voting by Filipinos abroad.

MR. REGALADO. How about those people who cannot go back to the places where they are registered?

MR. MONSOD. Under the present Election Code, there are provisions for allowing students and military people
who are temporarily in another place to register and vote. I believe that those situations can be covered by the
Omnibus Election Code. The reason we want absentee voting to be in the Constitution as a mandate to the
legislature is that there could be inconsistency on the residence rule if it is just a question of legislation
by Congress. So, by allowing it and saying that this is possible, then legislation can take care of the
rest.34 (Emphasis supplied)

Thus, Section 2, Article V of the Constitution came into being to remove any doubt as to the inapplicability of the residency
requirement in Section 1. It is precisely to avoid any problems that could impede the implementation of its pursuit to
enfranchise the largest number of qualified Filipinos who are not in the Philippines that the Constitutional Commission
explicitly mandated Congress to provide a system for overseas absentee voting.

The discussion of the Constitutional Commission on the effect of the residency requirement prescribed by Section 1,
Article V of the Constitution on the proposed system of absentee voting for qualified Filipinos abroad is enlightening:

MR. SUAREZ. May I just be recognized for a clarification. There are certain qualifications for the exercise of the
right of suffrage like having resided in the Philippines for at least one year and in the place where they propose to
vote for at least six months preceding the elections. What is the effect of these mandatory requirements on the
matter of the exercise of the right of suffrage by the absentee voters like Filipinos abroad?

THE PRESIDENT. Would Commissioner Monsod care to answer?

MR. MONSOD. I believe the answer was already given by Commissioner Bernas, that the domicile requirements
as well as the qualifications and disqualifications would be the same.

THE PRESIDENT. Are we leaving it to the legislature to devise the system?

FR. BERNAS. I think there is a very legitimate problem raised there.

THE PRESIDENT. Yes.

MR. BENGZON. I believe Commissioner Suarez is clarified.

FR. BERNAS. But I think it should be further clarified with regard to the residence requirement or the place where
they vote in practice; the understanding is that it is flexible. For instance, one might be a resident of Naga or
domiciled therein, but he satisfies the requirement of residence in Manila, so he is able to vote in Manila.

MR. TINGSON. Madam President, may I then suggest to the Committee to change the word "Filipinos" to
QUALIFIED FILIPINO VOTERS. Instead of "VOTING BY FILIPINOS ABROAD," it should be QUALIFIED
FILIPINO VOTERS. If the Committee wants QUALIFIED VOTERS LIVING ABROAD, would that not satisfy the
requirement?

THE PRESIDENT. What does Commissioner Monsod say?

MR. MONSOD. Madam President, I think I would accept the phrase "QUALIFIED FILIPINOS ABROAD" because
"QUALIFIED" would assume that he has the qualifications and none of the disqualifications to vote.

MR. TINGSON. That is right. So does the Committee accept?

FR. BERNAS. "QUALIFIED FILIPINOS ABROAD"?

THE PRESIDENT. Does the Committee accept the amendment?

MR. REGALADO. Madam President.

THE PRESIDENT. Commissioner Regalado is recognized.

MR. REGALADO. When Commissioner Bengzon asked me to read my proposed amendment, I specifically stated
that the National Assembly shall prescribe a system which will enable qualified citizens, temporarily absent from
the Philippines, to vote. According to Commissioner Monsod, the use of the phrase "absentee voting" already took
that into account as its meaning. That is referring to qualified Filipino citizens temporarily abroad.
MR. MONSOD. Yes, we accepted that. I would like to say that with respect to registration we will leave it up to
the legislative assembly, for example, to require where the registration is. If it is, say, members of the
diplomatic corps who may be continuously abroad for a long time, perhaps, there can be a system of
registration in the embassies. However, we do not like to preempt the legislative assembly.

THE PRESIDENT. Just to clarify, Commissioner Monsod’s amendment is only to provide a system.

MR. MONSOD. Yes.

THE PRESIDENT. The Commissioner is not stating here that he wants new qualifications for these absentee
voters.

MR. MONSOD. That is right. They must have the qualifications and none of the disqualifications.

THE PRESIDENT. It is just to devise a system by which they can vote.

MR. MONSOD. That is right, Madam President.35 (Emphasis supplied)

Clearly therefrom, the intent of the Constitutional Commission is to entrust to Congress the responsibility of devising a
system of absentee voting. The qualifications of voters as stated in Section 1 shall remain except for the residency
requirement. This is in fact the reason why the Constitutional Commission opted for the term qualified Filipinos abroad with
respect to the system of absentee voting that Congress should draw up. As stressed by Commissioner Monsod, by the
use of the adjective qualified with respect to Filipinos abroad, the assumption is that they have the "qualifications and none
of the disqualifications to vote." In fine-tuning the provision on absentee voting, the Constitutional Commission discussed
how the system should work:

MR. SUAREZ. For clarification purposes, we just want to state for the record that in the case of qualified Filipino
citizens residing abroad and exercising their right of suffrage, they can cast their votes for the candidates in the
place where they were registered to vote in the Philippines. So as to avoid any complications, for example, if they
are registered in Angeles City, they could not vote for a mayor in Naga City.

In other words, if that qualified voter is registered in Angeles City, then he can vote only for the local and national
candidates in Angeles City. I just want to make that clear for the record.

MR. REGALADO. Madam President.

THE PRESIDENT. What does Commissioner Regalado say?

MR. REGALADO. I just want to make a note on the statement of Commissioner Suarez that this envisions
Filipinos residing abroad. The understanding in the amendment is that the Filipino is temporarily abroad. He may
not be actually residing abroad; he may just be there on a business trip. It just so happens that the day before the
elections he has to fly to the United States, so he could not cast his vote. He is temporarily abroad, but not residing
there. He stays in a hotel for two days and comes back. This is not limited only to Filipinos temporarily
residing abroad. But as long as he is temporarily abroad on the date of the elections, then he can fall
within the prescription of Congress in that situation.

MR. SUAREZ. I thank the Commissioner for his further clarification. Precisely, we need this clarification on record.

MR. MONSOD. Madam President, to clarify what we mean by "temporarily abroad," it need not be on very
short trips. One can be abroad on a treaty traders visa. Therefore, when we talk about registration, it is possible
that his residence is in Angeles and he would be able to vote for the candidates in Angeles, but Congress or the
Assembly may provide the procedure for registration, like listing one’s name, in a registry list in the
embassy abroad. That is still possible under the system.

FR. BERNAS. Madam President, just one clarification if Commissioner Monsod agrees with this.

Suppose we have a situation of a child of a diplomatic officer who reaches the voting age while living abroad and
he has never registered here. Where will he register? Will he be a registered voter of a certain locality in the
Philippines?

MR. MONSOD. Yes, it is possible that the system will enable that child to comply with the registration
requirements in an embassy in the United States and his name is then entered in the official registration book in
Angeles City, for instance.

FR. BERNAS. In other words, he is not a registered voter of Los Angeles, but a registered voter of a locality here.

MR. MONSOD. That is right. He does not have to come home to the Philippines to comply with the registration
procedure here.

FR. BERNAS. So, he does not have to come home.


MR. BENGZON. Madam President, the Floor Leader wishes to inquire if there are more clarifications needed from
the body.

Also, the Floor Leader is happy to announce that there are no more registered Commissioners to propose
amendments. So I move that we close the period of amendments. 36 (Emphasis supplied)

It is clear from these discussions of the members of the Constitutional Commission that they intended to enfranchise as
much as possible all Filipino citizens abroad who have not abandoned their domicile of origin. The Commission even
intended to extend to young Filipinos who reach voting age abroad whose parents’ domicile of origin is in the Philippines,
and consider them qualified as voters for the first time.

It is in pursuance of that intention that the Commission provided for Section 2 immediately after the residency requirement
of Section 1. By the doctrine of necessary implication in statutory construction, which may be applied in construing
constitutional provisions,37 the strategic location of Section 2 indicates that the Constitutional Commission provided for an
exception to the actual residency requirement of Section 1 with respect to qualified Filipinos abroad. The same
Commission has in effect declared that qualified Filipinos who are not in the Philippines may be allowed to vote even
though they do not satisfy the residency requirement in Section 1, Article V of the Constitution.

That Section 2 of Article V of the Constitution is an exception to the residency requirement found in Section 1 of the same
Article was in fact the subject of debate when Senate Bill No. 2104, which became R.A. No. 9189, was deliberated upon
on the Senate floor, thus:

Senator Arroyo. Mr. President, this bill should be looked into in relation to the constitutional provisions. I think the
sponsor and I would agree that the Constitution is supreme in any statute that we may enact.

Let me read Section 1, Article V, of the Constitution entitled, "Suffrage." It says:

Section 1. Suffrage may be exercised by all citizens of the Philippines not otherwise disqualified by law,
who are at least eighteen years of age, and who shall have resided in the Philippines for at least one year
and in the place wherein they propose to vote for at least six months immediately preceding the election.

Now, Mr. President, the Constitution says, "who shall have resided in the Philippines." They are permanent
immigrants. They have changed residence so they are barred under the Constitution. This is why I asked whether
this committee amendment which in fact does not alter the original text of the bill will have any effect on this?

Senator Angara. Good question, Mr. President. And this has been asked in various fora. This is in compliance
with the Constitution. One, the interpretation here of "residence" is synonymous with "domicile."

As the gentleman and I know, Mr. President, "domicile" is the intent to return to one’s home. And the fact that a
Filipino may have been physically absent from the Philippines and may be physically a resident of the
United States, for example, but has a clear intent to return to the Philippines, will make him qualified as a
resident of the Philippines under this law.

This is consistent, Mr. President, with the constitutional mandate that we – that Congress – must provide a
franchise to overseas Filipinos.

If we read the Constitution and the suffrage principle literally as demanding physical presence, then there
is no way we can provide for offshore voting to our offshore kababayan, Mr. President.

Senator Arroyo. Mr. President, when the Constitution says, in Section 2 of Article V, it reads: "The Congress shall
provide a system for securing the secrecy and sanctity of the ballot as well as a system for absentee voting by
qualified Filipinos abroad."

The key to this whole exercise, Mr. President, is "qualified." In other words, anything that we may do or
say in granting our compatriots abroad must be anchored on the proposition that they are qualified.
Absent the qualification, they cannot vote. And "residents" (sic) is a qualification.

I will lose votes here from permanent residents so-called "green-card holders", but the Constitution is the
Constitution. We cannot compromise on this. The Senate cannot be a party to something that would affect or
impair the Constitution.

Look at what the Constitution says – "In the place wherein they propose to vote for at least six months immediately
preceding the election."

Mr. President, all of us here have run (sic) for office.

I live in Makati. My neighbor is Pateros where Senator Cayetano lives. We are separated only by a creek. But one
who votes in Makati cannot vote in Pateros unless he resides in Pateros for six months. That is how restrictive our
Constitution is. I am not talking even about the Election Code. I am talking about the Constitution.
As I have said, if a voter in Makati would want to vote in Pateros, yes, he may do so. But he must do so, make the
transfer six months before the election, otherwise, he is not qualified to vote.

That is why I am raising this point because I think we have a fundamental difference here.

Senator Angara. It is a good point to raise, Mr. President. But it is a point already well-debated even in the
constitutional commission of 1986. And the reason Section 2 of Article V was placed immediately after the
six-month/one-year residency requirement is to demonstrate unmistakably that Section 2 which
authorizes absentee voting is an exception to the six-month/one-year residency requirement. That is the
first principle, Mr. President, that one must remember.

The second reason, Mr. President, is that under our jurisprudence – and I think this is so well-entrenched that one
need not argue about it – "residency" has been interpreted as synonymous with "domicile."

But the third more practical reason, Mr. President, is, if we follow the interpretation of the gentleman, then
it is legally and constitutionally impossible to give a franchise to vote to overseas Filipinos who do not
physically live in the country, which is quite ridiculous because that is exactly the whole point of this
exercise – to enfranchise them and empower them to vote.38 (Emphasis supplied)

Accordingly, Section 4 of R.A. No. 9189 provides for the coverage of the absentee voting process, to wit:

SEC. 4. Coverage. – All citizens of the Philippines abroad, who are not otherwise disqualified by law, at least
eighteen (18) years of age on the day of elections, may vote for president, vice-president, senators and party-list
representatives.

which does not require physical residency in the Philippines; and Section 5 of the assailed law which enumerates those
who are disqualified, to wit:

SEC. 5. Disqualifications. – The following shall be disqualified from voting under this Act:

a) Those who have lost their Filipino citizenship in accordance with Philippine laws;

b) Those who have expressly renounced their Philippine citizenship and who have pledged allegiance to a foreign
country;

c) Those who have committed and are convicted in a final judgment by a court or tribunal of an offense punishable
by imprisonment of not less than one (1) year, including those who have committed and been found guilty of
Disloyalty as defined under Article 137 of the Revised Penal Code, such disability not having been removed by
plenary pardon or amnesty: Provided, however, That any person disqualified to vote under this subsection shall
automatically acquire the right to vote upon expiration of five (5) years after service of sentence; Provided, further,
That the Commission may take cognizance of final judgments issued by foreign courts or tribunals only on the
basis of reciprocity and subject to the formalities and processes prescribed by the Rules of Court on execution of
judgments;

d) An immigrant or a permanent resident who is recognized as such in the host country, unless he/she executes,
upon registration, an affidavit prepared for the purpose by the Commission declaring that he/she shall resume
actual physical permanent residence in the Philippines not later than three (3) years from approval of his/her
registration under this Act. Such affidavit shall also state that he/she has not applied for citizenship in another
country. Failure to return shall be cause for the removal of the name of the immigrant or permanent resident from
the National Registry of Absentee Voters and his/her permanent disqualification to vote in absentia.

e) Any citizen of the Philippines abroad previously declared insane or incompetent by competent authority in the
Philippines or abroad, as verified by the Philippine embassies, consulates or foreign service establishments
concerned, unless such competent authority subsequently certifies that such person is no longer insane or
incompetent.

As finally approved into law, Section 5(d) of R.A. No. 9189 specifically disqualifies an immigrant or permanent
resident who is "recognized as such in the host country" because immigration or permanent residence in another country
implies renunciation of one’s residence in his country of origin. However, same Section allows an immigrant and
permanent resident abroad to register as voter for as long as he/she executes an affidavit to show that he/she has not
abandoned his domicile in pursuance of the constitutional intent expressed in Sections 1 and 2 of Article V that "all citizens
of the Philippines not otherwise disqualified by law" must be entitled to exercise the right of suffrage and, that Congress
must establish a system for absentee voting; for otherwise, if actual, physical residence in the Philippines is required, there
is no sense for the framers of the Constitution to mandate Congress to establish a system for absentee voting.

Contrary to the claim of petitioner, the execution of the affidavit itself is not the enabling or enfranchising act. The affidavit
required in Section 5(d) is not only proof of the intention of the immigrant or permanent resident to go back and resume
residency in the Philippines, but more significantly, it serves as an explicit expression that he had not in fact abandoned
his domicile of origin. Thus, it is not correct to say that the execution of the affidavit under Section 5(d) violates the
Constitution that proscribes "provisional registration or a promise by a voter to perform a condition to be qualified to vote in
a political exercise."
To repeat, the affidavit is required of immigrants and permanent residents abroad because by their status in their host
countries, they are presumed to have relinquished their intent to return to this country; thus, without the affidavit, the
presumption of abandonment of Philippine domicile shall remain.

Further perusal of the transcripts of the Senate proceedings discloses another reason why the Senate required the
execution of said affidavit. It wanted the affiant to exercise the option to return or to express his intention to return to his
domicile of origin and not to preempt that choice by legislation. Thus:

Senator Villar. Yes, we are going back.

It states that: "For Filipino immigrants and those who have acquired permanent resident status abroad," a
requirement for the registration is the submission of "a Sworn Declaration of Intent to Return duly sworn before any
Philippine embassy or consulate official authorized to administer oath…"

Mr. President, may we know the rationale of this provision? Is the purpose of this Sworn Declaration to include
only those who have the intention of returning to be qualified to exercise the right of suffrage? What if the Filipino
immigrant has no purpose of returning? Is he automatically disbarred from exercising this right to suffrage?

Senator Angara. The rationale for this, Mr. President, is that we want to be expansive and all-inclusive in
this law. That as long as he is a Filipino, no matter whether he is a green-card holder in the U.S. or not, he
will be authorized to vote. But if he is already a green-card holder, that means he has acquired permanent
residency in the United States, then he must indicate an intention to return. This is what makes for the
definition of "domicile." And to acquire the vote, we thought that we would require the immigrants and the green-
card holders . . . Mr. President, the three administration senators are leaving, maybe we may ask for a vote
[Laughter].

Senator Villar. For a merienda, Mr. President.

Senator Angara. Mr. President, going back to the business at hand. The rationale for the requirement that an
immigrant or a green-card holder should file an affidavit that he will go back to the Philippines is that, if he is
already an immigrant or a green-card holder, that means he may not return to the country any more and that
contradicts the definition of "domicile" under the law.

But what we are trying to do here, Mr. President, is really provide the choice to the voter. The voter, after
consulting his lawyer or after deliberation within the family, may decide "No, I think we are risking our permanent
status in the United States if we file an affidavit that we want to go back." But we want to give him the
opportunity to make that decision. We do not want to make that decision for him. 39 (Emphasis supplied)

The jurisprudential declaration in Caasi vs. Court of Appeals that green card holders are disqualified to run for any elective
office finds no application to the present case because the Caasi case did not, for obvious reasons, consider the absentee
voting rights of Filipinos who are immigrants and permanent residents in their host countries.

In the advent of The Overseas Absentee Voting Act of 2003 or R.A. 9189, they may still be considered as a "qualified
citizen of the Philippines abroad" upon fulfillment of the requirements of registration under the new law for the purpose of
exercising their right of suffrage.

It must be emphasized that Section 5(d) does not only require an affidavit or a promise to "resume actual physical
permanent residence in the Philippines not later than three years from approval of his/her registration," the Filipinos
abroad must also declare that they have not applied for citizenship in another country. Thus, they must return to the
Philippines; otherwise, their failure to return "shall be cause for the removal" of their names "from the National Registry of
Absentee Voters and his/her permanent disqualification to vote in absentia."

Thus, Congress crafted a process of registration by which a Filipino voter permanently residing abroad who is at least
eighteen years old, not otherwise disqualified by law, who has not relinquished Philippine citizenship and who has not
actually abandoned his/her intentions to return to his/her domicile of origin, the Philippines, is allowed to register and vote
in the Philippine embassy, consulate or other foreign service establishments of the place which has jurisdiction over the
country where he/she has indicated his/her address for purposes of the elections, while providing for safeguards to a clean
election.

Thus, Section 11 of R.A. No. 9189 provides:

SEC. 11. Procedure for Application to Vote in Absentia. –

11.1. Every qualified citizen of the Philippines abroad whose application for registration has been approved,
including those previously registered under Republic Act No. 8189, shall, in every national election, file with the
officer of the embassy, consulate or other foreign service establishment authorized by the Commission, a sworn
written application to vote in a form prescribed by the Commission. The authorized officer of such embassy,
consulate or other foreign service establishment shall transmit to the Commission the said application to vote
within five (5) days from receipt thereof. The application form shall be accomplished in triplicate and submitted
together with the photocopy of his/her overseas absentee voter certificate of registration.
11.2. Every application to vote in absentia may be done personally at, or by mail to, the embassy, consulate or
foreign service establishment, which has jurisdiction over the country where he/she has indicated his/her address
for purposes of the elections.

11.3. Consular and diplomatic services rendered in connection with the overseas absentee voting processes shall
be made available at no cost to the overseas absentee voter.

Contrary to petitioner’s claim that Section 5(d) circumvents the Constitution, Congress enacted the law prescribing a
system of overseas absentee voting in compliance with the constitutional mandate. Such mandate expressly requires that
Congress provide a system of absentee voting that necessarily presupposes that the "qualified citizen of the Philippines
abroad" is not physically present in the country. The provisions of Sections 5(d) and 11 are components of the system of
overseas absentee voting established by R.A. No. 9189. The qualified Filipino abroad who executed the affidavit is
deemed to have retained his domicile in the Philippines. He is presumed not to have lost his domicile by his physical
absence from this country. His having become an immigrant or permanent resident of his host country does not
necessarily imply an abandonment of his intention to return to his domicile of origin, the Philippines. Therefore, under the
law, he must be given the opportunity to express that he has not actually abandoned his domicile in the Philippines by
executing the affidavit required by Sections 5(d) and 8(c) of the law.

Petitioner’s speculative apprehension that the implementation of Section 5(d) would affect the credibility of the elections is
insignificant as what is important is to ensure that all those who possess the qualifications to vote on the date of the
election are given the opportunity and permitted to freely do so. The COMELEC and the Department of Foreign Affairs
have enough resources and talents to ensure the integrity and credibility of any election conducted pursuant to R.A. No.
9189.

As to the eventuality that the Filipino abroad would renege on his undertaking to return to the Philippines, the penalty of
perpetual disenfranchisement provided for by Section 5(d) would suffice to serve as deterrence to non-compliance with
his/her undertaking under the affidavit.

Petitioner argues that should a sizable number of "immigrants" renege on their promise to return, the result of the elections
would be affected and could even be a ground to contest the proclamation of the winning candidates and cause further
confusion and doubt on the integrity of the results of the election. Indeed, the probability that after an immigrant has
exercised the right to vote, he shall opt to remain in his host country beyond the third year from the execution of the
affidavit, is not farfetched. However, it is not for this Court to determine the wisdom of a legislative exercise. As expressed
in Tañada vs. Tuvera,40 the Court is not called upon to rule on the wisdom of the law or to repeal it or modify it if we find it
impractical.

Congress itself was conscious of said probability and in fact, it has addressed the expected problem. Section 5(d) itself
provides for a deterrence which is that the Filipino who fails to return as promised stands to lose his right of suffrage.
Under Section 9, should a registered overseas absentee voter fail to vote for two consecutive national elections, his name
may be ordered removed from the National Registry of Overseas Absentee Voters.

Other serious legal questions that may be raised would be: what happens to the votes cast by the qualified voters abroad
who were not able to return within three years as promised? What is the effect on the votes cast by the non-returnees in
favor of the winning candidates? The votes cast by qualified Filipinos abroad who failed to return within three years shall
not be invalidated because they were qualified to vote on the date of the elections, but their failure to return shall be cause
for the removal of the names of the immigrants or permanent residents from the National Registry of Absentee Voters and
their permanent disqualification to vote in absentia.

In fine, considering the underlying intent of the Constitution, the Court does not find Section 5(d) of R.A. No. 9189 as
constitutionally defective.

B. Is Section 18.5 of R.A. No. 9189 in relation to Section 4 of the same Act in contravention of Section 4, Article VII
of the Constitution?

Section 4 of R.A. No. 9189 provides that the overseas absentee voter may vote for president, vice-president, senators and
party-list representatives.

Section 18.5 of the same Act provides:

SEC. 18. On-Site Counting and Canvassing. –

.........

18. 5 The canvass of votes shall not cause the delay of the proclamation of a winning candidate if the outcome of
the election will not be affected by the results thereof. Notwithstanding the foregoing, the Commission is
empowered to order the proclamation of winning candidates despite the fact that the scheduled election has
not taken place in a particular country or countries, if the holding of elections therein has been rendered impossible
by events, factors and circumstances peculiar to such country or countries, in which events, factors and
circumstances are beyond the control or influence of the Commission. (Emphasis supplied)
Petitioner claims that the provision of Section 18.5 of R.A. No. 9189 empowering the COMELEC to order the proclamation
of winning candidates insofar as it affects the canvass of votes and proclamation of winning candidates for president and
vice-president, is unconstitutional because it violates the following provisions of paragraph 4, Section 4 of Article VII of the
Constitution:

SEC. 4 . . .

The returns of every election for President and Vice-President, duly certified by the board of canvassers of each
province or city, shall be transmitted to the Congress, directed to the President of the Senate. Upon receipt of the
certificates of canvass, the President of the Senate shall, not later than thirty days after the day of the election,
open all the certificates in the presence of the Senate and the House of Representatives in joint public session,
and the Congress, upon determination of the authenticity and due execution thereof in the manner provided by
law, canvass the votes.

The person having the highest number of votes shall be proclaimed elected, but in case two or more shall have an
equal and highest number of votes, one of them shall forthwith be chosen by the vote of a majority of all the
Members of both Houses of the Congress, voting separately.

The Congress shall promulgate its rules for the canvassing of the certificates.

...

which gives to Congress the duty to canvass the votes and proclaim the winning candidates for president and vice-
president.

The Solicitor General asserts that this provision must be harmonized with paragraph 4, Section 4, Article VII of the
Constitution and should be taken to mean that COMELEC can only proclaim the winning Senators and party-list
representatives but not the President and Vice-President. 41

Respondent COMELEC has no comment on the matter.

Indeed, the phrase, proclamation of winning candidates, in Section 18.5 of R.A. No. 9189 is far too sweeping that it
necessarily includes the proclamation of the winning candidates for the presidency and the vice-presidency.

Section 18.5 of R.A. No. 9189 appears to be repugnant to Section 4, Article VII of the Constitution only insofar as said
Section totally disregarded the authority given to Congress by the Constitution to proclaim the winning candidates for the
positions of president and vice-president.

In addition, the Court notes that Section 18.4 of the law, to wit:

18.4. . . . Immediately upon the completion of the canvass, the chairman of the Special Board of Canvassers shall
transmit via facsimile, electronic mail, or any other means of transmission equally safe and reliable the Certificates
of Canvass and the Statements of Votes to the Commission, . . . [Emphasis supplied]

clashes with paragraph 4, Section 4, Article VII of the Constitution which provides that the returns of every election for
President and Vice-President shall be certified by the board of canvassers to Congress.

Congress could not have allowed the COMELEC to usurp a power that constitutionally belongs to it or, as aptly stated by
petitioner, to encroach "on the power of Congress to canvass the votes for president and vice-president and the power to
proclaim the winners for the said positions." The provisions of the Constitution as the fundamental law of the land should
be read as part of The Overseas Absentee Voting Act of 2003 and hence, the canvassing of the votes and the
proclamation of the winning candidates for president and vice-president for the entire nation must remain in the hands of
Congress.

C. Are Sections 19 and 25 of R.A. No. 9189 in violation of Section 1, Article IX-A of the Constitution?

Petitioner avers that Sections 19 and 25 of R.A. No. 9189 violate Article IX-A (Common Provisions) of the Constitution, to
wit:

Section 1. The Constitutional Commissions, which shall be independent, are the Civil Service Commission, the
Commission on Elections, and the Commission on Audit. (Emphasis supplied)

He submits that the creation of the Joint Congressional Oversight Committee with the power to review, revise, amend and
approve the Implementing Rules and Regulations promulgated by the COMELEC, R.A. No. 9189 intrudes into the
independence of the COMELEC which, as a constitutional body, is not under the control of either the executive or
legislative departments of government; that only the COMELEC itself can promulgate rules and regulations which may be
changed or revised only by the majority of its members; and that should the rules promulgated by the COMELEC violate
any law, it is the Court that has the power to review the same via the petition of any interested party, including the
legislators.
It is only on this question that respondent COMELEC submitted its Comment. It agrees with the petitioner that Sections 19
and 25 of R.A. No. 9189 are unconstitutional. Like the petitioner, respondent COMELEC anchors its claim of
unconstitutionality of said Sections upon Section 1, Article IX-A of the Constitution providing for the independence of the
constitutional commissions such as the COMELEC. It asserts that its power to formulate rules and regulations has been
upheld in Gallardo vs. Tabamo, Jr.42 where this Court held that the power of the COMELEC to formulate rules and
regulations is implicit in its power to implement regulations under Section 2(1) of Article IX-C 43 of the Constitution.
COMELEC joins the petitioner in asserting that as an independent constitutional body, it may not be subject to interference
by any government instrumentality and that only this Court may review COMELEC rules and only in cases of grave abuse
of discretion.

The COMELEC adds, however, that another provision, vis-à-vis its rule-making power, to wit:

SEC. 17. Voting by Mail. –

17.1. For the May, 2004 elections, the Commission shall authorize voting by mail in not more than three (3)
countries, subject to the approval of the Congressional Oversight Committee. Voting by mail may be allowed
in countries that satisfy the following conditions:

a) Where the mailing system is fairly well-developed and secure to prevent occasion for fraud;

b) Where there exists a technically established identification system that would preclude multiple or proxy voting;
and

c) Where the system of reception and custody of mailed ballots in the embassies, consulates and other foreign
service establishments concerned are adequate and well-secured.

Thereafter, voting by mail in any country shall be allowed only upon review and approval of the Joint
Congressional Oversight Committee . . . . . . . . . (Emphasis supplied)

is likewise unconstitutional as it violates Section 1, Article IX-A mandating the independence of constitutional
commissions.

The Solicitor General takes exception to his prefatory statement that the constitutional challenge must fail and agrees with
the petitioner that Sections 19 and 25 are invalid and unconstitutional on the ground that there is nothing in Article VI of
the Constitution on Legislative Department that would as much as imply that Congress has concurrent power to enforce
and administer election laws with the COMELEC; and by the principles of exclusio unius est exclusio alterius and
expressum facit cessare tacitum, the constitutionally enumerated powers of Congress circumscribe its authority to the
exclusion of all others.

The parties are unanimous in claiming that Sections 19, 25 and portions of Section 17.1 are unconstitutional. Thus, there
is no actual issue forged on this question raised by petitioner.

However, the Court finds it expedient to expound on the role of Congress through the Joint Congressional Oversight
Committee (JCOC) vis-à-vis the independence of the COMELEC, as a constitutional body.

R.A. No. 9189 created the JCOC, as follows:

SEC. 25. Joint Congressional Oversight Committee. – A Joint Congressional Oversight Committee is hereby
created, composed of the Chairman of the Senate Committee on Constitutional Amendments, Revision of Codes
and Laws, and seven (7) other Senators designated by the Senate President, and the Chairman of the House
Committee on Suffrage and Electoral Reforms, and seven (7) other Members of the House of Representatives
designated by the Speaker of the House of Representatives: Provided, That, of the seven (7) members to be
designated by each House of Congress, four (4) should come from the majority and the remaining three (3) from
the minority.

The Joint Congressional Oversight Committee shall have the power to monitor and evaluate the
implementation of this Act. It shall review, revise, amend and approve the Implementing Rules and
Regulations promulgated by the Commission. (Emphasis supplied)

SEC. 19. Authority of the Commission to Promulgate Rules. – The Commission shall issue the necessary rules
and regulations to effectively implement the provisions of this Act within sixty (60) days from the effectivity of this
Act. The Implementing Rules and Regulations shall be submitted to the Joint Congressional Oversight
Committee created by virtue of this Act for prior approval.

. . . . . . . . . (Emphasis supplied)

Composed of Senators and Members of the House of Representatives, the Joint Congressional Oversight Committee
(JCOC) is a purely legislative body. There is no question that the authority of Congress to "monitor and evaluate the
implementation" of R.A. No. 9189 is geared towards possible amendments or revision of the law itself and thus, may be
performed in aid of its legislation.
However, aside from its monitoring and evaluation functions, R.A. No. 9189 gives to the JCOC the following functions: (a)
to "review, revise, amend and approve the Implementing Rules and Regulations" (IRR) promulgated by the COMELEC
[Sections 25 and 19]; and (b) subject to the approval of the JCOC [Section 17.1], the voting by mail in not more than three
countries for the May 2004 elections and in any country determined by COMELEC.

The ambit of legislative power under Article VI of the Constitution is circumscribed by other constitutional provisions. One
such provision is Section 1 of Article IX-A of the 1987 Constitution ordaining that constitutional commissions such as the
COMELEC shall be "independent."

Interpreting Section 1, Article X of the 1935 Constitution providing that there shall be an independent COMELEC, the
Court has held that "[w]hatever may be the nature of the functions of the Commission on Elections, the fact is that the
framers of the Constitution wanted it to be independent from the other departments of the Government." 44 In an earlier
case, the Court elucidated:

The Commission on Elections is a constitutional body. It is intended to play a distinct and important part in our
scheme of government. In the discharge of its functions, it should not be hampered with restrictions that would be
fully warranted in the case of a less responsible organization. The Commission may err, so may this court also. It
should be allowed considerable latitude in devising means and methods that will insure the accomplishment of the
great objective for which it was created – free, orderly and honest elections. We may not agree fully with its choice
of means, but unless these are clearly illegal or constitute gross abuse of discretion, this court should not interfere.
Politics is a practical matter, and political questions must be dealt with realistically – not from the standpoint of
pure theory. The Commission on Elections, because of its fact-finding facilities, its contacts with political
strategists, and its knowledge derived from actual experience in dealing with political controversies, is in a
peculiarly advantageous position to decide complex political questions. 45 (Emphasis supplied)

The Court has no general powers of supervision over COMELEC which is an independent body "except those specifically
granted by the Constitution," that is, to review its decisions, orders and rulings. 46 In the same vein, it is not correct to hold
that because of its recognized extensive legislative power to enact election laws, Congress may intrude into the
independence of the COMELEC by exercising supervisory powers over its rule-making authority.

By virtue of Section 19 of R.A. No. 9189, Congress has empowered the COMELEC to "issue the necessary rules and
regulations to effectively implement the provisions of this Act within sixty days from the effectivity of this Act." This
provision of law follows the usual procedure in drafting rules and regulations to implement a law – the legislature grants an
administrative agency the authority to craft the rules and regulations implementing the law it has enacted, in recognition of
the administrative expertise of that agency in its particular field of operation. 47 Once a law is enacted and approved, the
legislative function is deemed accomplished and complete. The legislative function may spring back to Congress relative
to the same law only if that body deems it proper to review, amend and revise the law, but certainly not to approve, review,
revise and amend the IRR of the COMELEC.

By vesting itself with the powers to approve, review, amend, and revise the IRR for The Overseas Absentee Voting Act of
2003, Congress went beyond the scope of its constitutional authority. Congress trampled upon the constitutional mandate
of independence of the COMELEC. Under such a situation, the Court is left with no option but to withdraw from its usual
reticence in declaring a provision of law unconstitutional.

The second sentence of the first paragraph of Section 19 stating that "[t]he Implementing Rules and Regulations shall be
submitted to the Joint Congressional Oversight Committee created by virtue of this Act for prior approval," and the second
sentence of the second paragraph of Section 25 stating that "[i]t shall review, revise, amend and approve the
Implementing Rules and Regulations promulgated by the Commission," whereby Congress, in both provisions, arrogates
unto itself a function not specifically vested by the Constitution, should be stricken out of the subject statute for
constitutional infirmity. Both provisions brazenly violate the mandate on the independence of the COMELEC.

Similarly, the phrase, "subject to the approval of the Congressional Oversight Committee" in the first sentence of Section
17.1 which empowers the Commission to authorize voting by mail in not more than three countries for the May, 2004
elections; and the phrase, "only upon review and approval of the Joint Congressional Oversight Committee" found in the
second paragraph of the same section are unconstitutional as they require review and approval of voting by mail in any
country after the 2004 elections. Congress may not confer upon itself the authority to approve or disapprove the countries
wherein voting by mail shall be allowed, as determined by the COMELEC pursuant to the conditions provided for in
Section 17.1 of R.A. No. 9189. 48 Otherwise, Congress would overstep the bounds of its constitutional mandate and intrude
into the independence of the COMELEC.

During the deliberations, all the members of the Court agreed to adopt the separate opinion of Justice Reynato S. Puno as
part of the ponencia on the unconstitutionality of Sections 17.1, 19 and 25 of R.A. No. 9189 insofar as they relate to the
creation of and the powers given to the Joint Congressional Oversight Committee.

WHEREFORE, the petition is partly GRANTED. The following portions of R.A. No. 9189 are declared VOID for
being UNCONSTITUTIONAL:

a) The phrase in the first sentence of the first paragraph of Section 17.1, to wit: "subject to the approval of the
Joint Congressional Oversight Committee;"
b) The portion of the last paragraph of Section 17.1, to wit: "only upon review and approval of the Joint
Congressional Oversight Committee;"

c) The second sentence of the first paragraph of Section 19, to wit: "The Implementing Rules and Regulations
shall be submitted to the Joint Congressional Oversight Committee created by virtue of this Act for prior
approval;" and

d) The second sentence in the second paragraph of Section 25, to wit: "It shall review, revise, amend and
approve the Implementing Rules and Regulations promulgated by the Commission" of the same law;

for being repugnant to Section 1, Article IX-A of the Constitution mandating the independence of constitutional
commission, such as COMELEC.

The constitutionality of Section 18.5 of R.A. No. 9189 is UPHELD with respect only to the authority given to the COMELEC
to proclaim the winning candidates for the Senators and party-list representatives but not as to the power to canvass the
votes and proclaim the winning candidates for President and Vice-President which is lodged with Congress under Section
4, Article VII of the Constitution.

The constitutionality of Section 5(d) is UPHELD.Pursuant to Section 30 of R.A. No. 9189, the rest of the provisions of said
law continues to be in full force and effect. SO ORDERED.

- Abakada Guro Party List v Purisima, GR 166715, Aug 14, 2008

This petition for prohibition1 seeks to prevent respondents from implementing and enforcing Republic Act (RA) 9335 2 (Attrition
Act of 2005).

RA 9335 was enacted to optimize the revenue-generation capability and collection of the Bureau of Internal Revenue (BIR) and
the Bureau of Customs (BOC). The law intends to encourage BIR and BOC officials and employees to exceed their revenue
targets by providing a system of rewards and sanctions through the creation of a Rewards and Incentives Fund (Fund) and a
Revenue Performance Evaluation Board (Board). 3 It covers all officials and employees of the BIR and the BOC with at least six
months of service, regardless of employment status.4

The Fund is sourced from the collection of the BIR and the BOC in excess of their revenue targets for the year, as determined by
the Development Budget and Coordinating Committee (DBCC). Any incentive or reward is taken from the fund and allocated to
the BIR and the BOC in proportion to their contribution in the excess collection of the targeted amount of tax revenue. 5

The Boards in the BIR and the BOC are composed of the Secretary of the Department of Finance (DOF) or his/her
Undersecretary, the Secretary of the Department of Budget and Management (DBM) or his/her Undersecretary, the Director
General of the National Economic Development Authority (NEDA) or his/her Deputy Director General, the Commissioners of the
BIR and the BOC or their Deputy Commissioners, two representatives from the rank-and-file employees and a representative
from the officials nominated by their recognized organization. 6

Each Board has the duty to (1) prescribe the rules and guidelines for the allocation, distribution and release of the Fund; (2) set
criteria and procedures for removing from the service officials and employees whose revenue collection falls short of the target;
(3) terminate personnel in accordance with the criteria adopted by the Board; (4) prescribe a system for performance evaluation;
(5) perform other functions, including the issuance of rules and regulations and (6) submit an annual report to Congress. 7

The DOF, DBM, NEDA, BIR, BOC and the Civil Service Commission (CSC) were tasked to promulgate and issue the
implementing rules and regulations of RA 9335,8 to be approved by a Joint Congressional Oversight Committee created for such
purpose.9

Petitioners, invoking their right as taxpayers filed this petition challenging the constitutionality of RA 9335, a tax reform
legislation. They contend that, by establishing a system of rewards and incentives, the law "transform[s] the officials and
employees of the BIR and the BOC into mercenaries and bounty hunters" as they will do their best only in consideration of such
rewards. Thus, the system of rewards and incentives invites corruption and undermines the constitutionally mandated duty of
these officials and employees to serve the people with utmost responsibility, integrity, loyalty and efficiency.

Petitioners also claim that limiting the scope of the system of rewards and incentives only to officials and employees of the BIR
and the BOC violates the constitutional guarantee of equal protection. There is no valid basis for classification or distinction as to
why such a system should not apply to officials and employees of all other government agencies.

In addition, petitioners assert that the law unduly delegates the power to fix revenue targets to the President as it lacks a
sufficient standard on that matter. While Section 7(b) and (c) of RA 9335 provides that BIR and BOC officials may be dismissed
from the service if their revenue collections fall short of the target by at least 7.5%, the law does not, however, fix the revenue
targets to be achieved. Instead, the fixing of revenue targets has been delegated to the President without sufficient standards. It
will therefore be easy for the President to fix an unrealistic and unattainable target in order to dismiss BIR or BOC personnel.

Finally, petitioners assail the creation of a congressional oversight committee on the ground that it violates the doctrine of
separation of powers. While the legislative function is deemed accomplished and completed upon the enactment and approval of
the law, the creation of the congressional oversight committee permits legislative participation in the implementation and
enforcement of the law.
In their comment, respondents, through the Office of the Solicitor General, question the petition for being premature as there is
no actual case or controversy yet. Petitioners have not asserted any right or claim that will necessitate the exercise of this
Court’s jurisdiction. Nevertheless, respondents acknowledge that public policy requires the resolution of the constitutional issues
involved in this case. They assert that the allegation that the reward system will breed mercenaries is mere speculation and does
not suffice to invalidate the law. Seen in conjunction with the declared objective of RA 9335, the law validly classifies the BIR
and the BOC because the functions they perform are distinct from those of the other government agencies and instrumentalities.
Moreover, the law provides a sufficient standard that will guide the executive in the implementation of its provisions. Lastly, the
creation of the congressional oversight committee under the law enhances, rather than violates, separation of powers. It ensures
the fulfillment of the legislative policy and serves as a check to any over-accumulation of power on the part of the executive and
the implementing agencies.

After a careful consideration of the conflicting contentions of the parties, the Court finds that petitioners have failed to overcome
the presumption of constitutionality in favor of RA 9335, except as shall hereafter be discussed.

Actual Case And Ripeness

An actual case or controversy involves a conflict of legal rights, an assertion of opposite legal claims susceptible of judicial
adjudication.10 A closely related requirement is ripeness, that is, the question must be ripe for adjudication. And a constitutional
question is ripe for adjudication when the governmental act being challenged has a direct adverse effect on the individual
challenging it.11 Thus, to be ripe for judicial adjudication, the petitioner must show a personal stake in the outcome of the case or
an injury to himself that can be redressed by a favorable decision of the Court. 12

In this case, aside from the general claim that the dispute has ripened into a judicial controversy by the mere enactment of the
law even without any further overt act,13 petitioners fail either to assert any specific and concrete legal claim or to demonstrate
any direct adverse effect of the law on them. They are unable to show a personal stake in the outcome of this case or an injury
to themselves. On this account, their petition is procedurally infirm.

This notwithstanding, public interest requires the resolution of the constitutional issues raised by petitioners. The grave nature of
their allegations tends to cast a cloud on the presumption of constitutionality in favor of the law. And where an action of the
legislative branch is alleged to have infringed the Constitution, it becomes not only the right but in fact the duty of the judiciary to
settle the dispute.14

Accountability of
Public Officers

Section 1, Article 11 of the Constitution states:

Sec. 1. Public office is a public trust. Public officers and employees must at all times be accountable to the people, serve
them with utmost responsibility, integrity, loyalty, and efficiency, act with patriotism, and justice, and lead modest lives.

Public office is a public trust. It must be discharged by its holder not for his own personal gain but for the benefit of the public for
whom he holds it in trust. By demanding accountability and service with responsibility, integrity, loyalty, efficiency, patriotism and
justice, all government officials and employees have the duty to be responsive to the needs of the people they are called upon to
serve.

Public officers enjoy the presumption of regularity in the performance of their duties. This presumption necessarily obtains in
favor of BIR and BOC officials and employees. RA 9335 operates on the basis thereof and reinforces it by providing a system of
rewards and sanctions for the purpose of encouraging the officials and employees of the BIR and the BOC to exceed their
revenue targets and optimize their revenue-generation capability and collection. 15

The presumption is disputable but proof to the contrary is required to rebut it. It cannot be overturned by mere conjecture or
denied in advance (as petitioners would have the Court do) specially in this case where it is an underlying principle to advance a
declared public policy.

Petitioners’ claim that the implementation of RA 9335 will turn BIR and BOC officials and employees into "bounty hunters and
mercenaries" is not only without any factual and legal basis; it is also purely speculative.

A law enacted by Congress enjoys the strong presumption of constitutionality. To justify its nullification, there must be a clear
and unequivocal breach of the Constitution, not a doubtful and equivocal one. 16 To invalidate RA 9335 based on petitioners’
baseless supposition is an affront to the wisdom not only of the legislature that passed it but also of the executive which
approved it.

Public service is its own reward. Nevertheless, public officers may by law be rewarded for exemplary and exceptional
performance. A system of incentives for exceeding the set expectations of a public office is not anathema to the concept of
public accountability. In fact, it recognizes and reinforces dedication to duty, industry, efficiency and loyalty to public service of
deserving government personnel.

In United States v. Matthews,17 the U.S. Supreme Court validated a law which awards to officers of the customs as well as other
parties an amount not exceeding one-half of the net proceeds of forfeitures in violation of the laws against smuggling.
Citing Dorsheimer v. United States,18 the U.S. Supreme Court said:

The offer of a portion of such penalties to the collectors is to stimulate and reward their zeal and industry in detecting
fraudulent attempts to evade payment of duties and taxes.
In the same vein, employees of the BIR and the BOC may by law be entitled to a reward when, as a consequence of their zeal in
the enforcement of tax and customs laws, they exceed their revenue targets. In addition, RA 9335 establishes safeguards to
ensure that the reward will not be claimed if it will be either the fruit of "bounty hunting or mercenary activity" or the product of the
irregular performance of official duties. One of these precautionary measures is embodied in Section 8 of the law:

SEC. 8. Liability of Officials, Examiners and Employees of the BIR and the BOC. –  The officials, examiners, and
employees of the [BIR] and the [BOC] who violate this Act or who are guilty of negligence, abuses or acts of
malfeasance or misfeasance or fail to exercise extraordinary diligence in the performance of their duties shall be held
liable for any loss or injury suffered by any business establishment or taxpayer as a result of such violation, negligence,
abuse, malfeasance, misfeasance or failure to exercise extraordinary diligence.

Equal Protection

Equality guaranteed under the equal protection clause is equality under the same conditions and among persons similarly
situated; it is equality among equals, not similarity of treatment of persons who are classified based on substantial differences in
relation to the object to be accomplished.19 When things or persons are different in fact or circumstance, they may be treated in
law differently. In Victoriano v. Elizalde Rope Workers’ Union,20 this Court declared:

The guaranty of equal protection of the laws is not a guaranty of equality in the application of the laws upon all citizens
of the [S]tate. It is not, therefore, a requirement, in order to avoid the constitutional prohibition against inequality, that
every man, woman and child should be affected alike by a statute. Equality of operation of statutes does not mean
indiscriminate operation on persons merely as such, but on persons according to the circumstances surrounding them. It
guarantees equality, not identity of rights. The Constitution does not require that things which are different in fact
be treated in law as though they were the same. The equal protection clause does not forbid discrimination as
to things that are different. It does not prohibit legislation which is limited either in the object to which it is
directed or by the territory within which it is to operate.

The equal protection of the laws clause of the Constitution allows classification. Classification in law, as in the other
departments of knowledge or practice, is the grouping of things in speculation or practice because they agree with one
another in certain particulars. A law is not invalid because of simple inequality. The very idea of classification is that of
inequality, so that it goes without saying that the mere fact of inequality in no manner determines the matter of
constitutionality. All that is required of a valid classification is that it be reasonable, which means that the
classification should be based on substantial distinctions which make for real differences, that it must be
germane to the purpose of the law; that it must not be limited to existing conditions only; and that it must apply
equally to each member of the class. This Court has held that the standard is satisfied if the classification or
distinction is based on a reasonable foundation or rational basis and is not palpably arbitrary.

In the exercise of its power to make classifications for the purpose of enacting laws over matters within its jurisdiction,
the state is recognized as enjoying a wide range of discretion. It is not necessary that the classification be based on
scientific or marked differences of things or in their relation. Neither is it necessary that the classification be made with
mathematical nicety. Hence, legislative classification may in many cases properly rest on narrow distinctions, for the
equal protection guaranty does not preclude the legislature from recognizing degrees of evil or harm, and legislation is
addressed to evils as they may appear.21 (emphasis supplied)

The equal protection clause recognizes a valid classification, that is, a classification that has a reasonable foundation or rational
basis and not arbitrary.22 With respect to RA 9335, its expressed public policy is the optimization of the revenue-generation
capability and collection of the BIR and the BOC.23 Since the subject of the law is the revenue- generation capability and
collection of the BIR and the BOC, the incentives and/or sanctions provided in the law should logically pertain to the said
agencies. Moreover, the law concerns only the BIR and the BOC because they have the common distinct primary function of
generating revenues for the national government through the collection of taxes, customs duties, fees and charges.

The BIR performs the following functions:

Sec. 18. The Bureau of Internal Revenue. – The Bureau of Internal Revenue, which shall be headed by and subject to
the supervision and control of the Commissioner of Internal Revenue, who shall be appointed by the President upon the
recommendation of the Secretary [of the DOF], shall have the following functions:

(1) Assess and collect all taxes, fees and charges and account for all revenues collected;

(2) Exercise duly delegated police powers for the proper performance of its functions and duties;

(3) Prevent and prosecute tax evasions and all other illegal economic activities;

(4) Exercise supervision and control over its constituent and subordinate units; and

(5) Perform such other functions as may be provided by law. 24

xxx       xxx       xxx (emphasis supplied)

On the other hand, the BOC has the following functions:


Sec. 23. The Bureau of Customs. – The Bureau of Customs which shall be headed and subject to the management and
control of the Commissioner of Customs, who shall be appointed by the President upon the recommendation of the
Secretary[of the DOF] and hereinafter referred to as Commissioner, shall have the following functions:

(1) Collect custom duties, taxes and the corresponding fees, charges and penalties;

(2) Account for all customs revenues collected;

(3) Exercise police authority for the enforcement of tariff and customs laws;

(4) Prevent and suppress smuggling, pilferage and all other economic frauds within all ports of entry;

(5) Supervise and control exports, imports, foreign mails and the clearance of vessels and aircrafts in all ports of entry;

(6) Administer all legal requirements that are appropriate;

(7) Prevent and prosecute smuggling and other illegal activities in all ports under its jurisdiction;

(8) Exercise supervision and control over its constituent units;

(9) Perform such other functions as may be provided by law. 25

xxx       xxx       xxx (emphasis supplied)

Both the BIR and the BOC are bureaus under the DOF. They principally perform the special function of being the
instrumentalities through which the State exercises one of its great inherent functions – taxation. Indubitably, such substantial
distinction is germane and intimately related to the purpose of the law. Hence, the classification and treatment accorded to the
BIR and the BOC under RA 9335 fully satisfy the demands of equal protection.

Undue Delegation

Two tests determine the validity of delegation of legislative power: (1) the completeness test and (2) the sufficient standard test.
A law is complete when it sets forth therein the policy to be executed, carried out or implemented by the delegate. 26 It lays down
a sufficient standard when it provides adequate guidelines or limitations in the law to map out the boundaries of the delegate’s
authority and prevent the delegation from running riot. 27 To be sufficient, the standard must specify the limits of the delegate’s
authority, announce the legislative policy and identify the conditions under which it is to be implemented. 28

RA 9335 adequately states the policy and standards to guide the President in fixing revenue targets and the implementing
agencies in carrying out the provisions of the law. Section 2 spells out the policy of the law:

SEC. 2. Declaration of Policy. – It is the policy of the State to optimize the revenue-generation capability and collection
of the Bureau of Internal Revenue (BIR) and the Bureau of Customs (BOC) by providing for a system of rewards and
sanctions through the creation of a Rewards and Incentives Fund and a Revenue Performance Evaluation Board in the
above agencies for the purpose of encouraging their officials and employees to exceed their revenue targets.

Section 4 "canalized within banks that keep it from overflowing"29 the delegated power to the President to fix revenue targets:

SEC. 4. Rewards and Incentives Fund. – A Rewards and Incentives Fund, hereinafter referred to as the Fund, is hereby
created, to be sourced from the collection of the BIR and the BOC in excess of their respective revenue targets of the
year, as determined by the Development Budget and Coordinating Committee (DBCC) , in the following
percentages:

Excess of Collection of the Percent (%) of the Excess Collection to Accrue


Excess the Revenue Targets to the Fund

30% or below – 15%

More than 30% – 15% of the first 30% plus 20% of the
remaining excess

The Fund shall be deemed automatically appropriated the year immediately following the year when the revenue
collection target was exceeded and shall be released on the same fiscal year.

Revenue targets shall refer to the original estimated revenue collection expected of the BIR and the BOC for a
given fiscal year as stated in the Budget of Expenditures and Sources of Financing (BESF) submitted by the
President to Congress. The BIR and the BOC shall submit to the DBCC the distribution of the agencies’ revenue
targets as allocated among its revenue districts in the case of the BIR, and the collection districts in the case of the
BOC.

xxx       xxx       xxx (emphasis supplied)


Revenue targets are based on the original estimated revenue collection expected respectively of the BIR and the BOC for a
given fiscal year as approved by the DBCC and stated in the BESF submitted by the President to Congress. 30 Thus, the
determination of revenue targets does not rest solely on the President as it also undergoes the scrutiny of the DBCC.

On the other hand, Section 7 specifies the limits of the Board’s authority and identifies the conditions under which officials and
employees whose revenue collection falls short of the target by at least 7.5% may be removed from the service:

SEC. 7. Powers and Functions of the Board. –  The Board in the agency shall have the following powers and functions:

xxx       xxx       xxx

(b) To set the criteria and procedures for removing from service officials and employees whose revenue collection
falls short of the target by at least seven and a half percent (7.5%), with due consideration of all relevant factors
affecting the level of collection as provided in the rules and regulations promulgated under this Act, subject to civil
service laws, rules and regulations and compliance with substantive and procedural due process: Provided, That
the following exemptions shall apply:

1. Where the district or area of responsibility is newly-created, not exceeding two years in operation, as has no
historical record of collection performance that can be used as basis for evaluation; and

2. Where the revenue or customs official or employee is a recent transferee in the middle of the period under
consideration unless the transfer was due to nonperformance of revenue targets or potential nonperformance of
revenue targets: Provided, however, That when the district or area of responsibility covered by revenue or
customs officials or employees has suffered from economic difficulties brought about by natural calamities
or force majeure or economic causes as may be determined by the Board, termination shall be considered only
after careful and proper review by the Board.

(c) To terminate personnel in accordance with the criteria adopted in the preceding paragraph: Provided, That such
decision shall be immediately executory: Provided, further, That the application of the criteria for the separation of
an official or employee from service under this Act shall be without prejudice to the application of other relevant
laws on accountability of public officers and employees, such as the Code of Conduct and Ethical Standards of
Public Officers and Employees and the Anti-Graft and Corrupt Practices Act;

xxx       xxx       xxx (emphasis supplied)

Clearly, RA 9335 in no way violates the security of tenure of officials and employees of the BIR and the BOC. The guarantee of
security of tenure only means that an employee cannot be dismissed from the service for causes other than those provided by
law and only after due process is accorded the employee. 31 In the case of RA 9335, it lays down a reasonable yardstick for
removal (when the revenue collection falls short of the target by at least 7.5%) with due consideration of all relevant factors
affecting the level of collection. This standard is analogous to inefficiency and incompetence in the performance of official duties,
a ground for disciplinary action under civil service laws.32 The action for removal is also subject to civil service laws, rules and
regulations and compliance with substantive and procedural due process.

At any rate, this Court has recognized the following as sufficient standards: "public interest," "justice and equity," "public
convenience and welfare" and "simplicity, economy and welfare."33 In this case, the declared policy of optimization of the
revenue-generation capability and collection of the BIR and the BOC is infused with public interest.

Separation Of Powers

Section 12 of RA 9335 provides:

SEC. 12. Joint Congressional Oversight Committee. – There is hereby created a Joint Congressional Oversight
Committee composed of seven Members from the Senate and seven Members from the House of Representatives. The
Members from the Senate shall be appointed by the Senate President, with at least two senators representing the
minority. The Members from the House of Representatives shall be appointed by the Speaker with at least two members
representing the minority. After the Oversight Committee will have approved the implementing rules and regulations
(IRR) it shall thereafter become functus officio and therefore cease to exist.

The Joint Congressional Oversight Committee in RA 9335 was created for the purpose of approving the implementing rules and
regulations (IRR) formulated by the DOF, DBM, NEDA, BIR, BOC and CSC. On May 22, 2006, it approved the said IRR. From
then on, it became functus officio and ceased to exist. Hence, the issue of its alleged encroachment on the executive function of
implementing and enforcing the law may be considered moot and academic.

This notwithstanding, this might be as good a time as any for the Court to confront the issue of the constitutionality of the Joint
Congressional Oversight Committee created under RA 9335 (or other similar laws for that matter).

The scholarly discourse of Mr. Justice (now Chief Justice) Puno on the concept of congressional oversight  in Macalintal v.
Commission on Elections34 is illuminating:

Concept and bases of congressional oversight

Broadly defined, the power of oversight embraces all activities undertaken by Congress to enhance its
understanding of and influence over the implementation of legislation it has enacted. Clearly, oversight
concerns post-enactment measures undertaken by Congress: (a) to monitor bureaucratic compliance with
program objectives, (b) to determine whether agencies are properly administered, (c) to eliminate executive
waste and dishonesty, (d) to prevent executive usurpation of legislative authority, and (d) to assess executive
conformity with the congressional perception of public interest.

The power of oversight has been held to be intrinsic in the grant of legislative power itself and integral to the checks and
balances inherent in a democratic system of government. x x x x x x x x x

Over the years, Congress has invoked its oversight power with increased frequency to check the perceived "exponential
accumulation of power" by the executive branch. By the beginning of the 20 th century, Congress has delegated an
enormous amount of legislative authority to the executive branch and the administrative agencies. Congress, thus, uses
its oversight power to make sure that the administrative agencies perform their functions within the authority delegated
to them. x x x x x x x x x

Categories of congressional oversight functions

The acts done by Congress purportedly in the exercise of its oversight powers may be divided into three categories,
namely: scrutiny, investigation and supervision.

a. Scrutiny

Congressional scrutiny implies a lesser intensity and continuity of attention to administrative operations. Its


primary purpose is to determine economy and efficiency of the operation of government activities. In the
exercise of legislative scrutiny, Congress may request information and report from the other branches of
government. It can give recommendations or pass resolutions for consideration of the agency involved.

xxx       xxx       xxx

b. Congressional investigation

While congressional scrutiny is regarded as a passive process of looking at the facts that are readily
available, congressional investigation involves a more intense digging of facts. The power of Congress to
conduct investigation is recognized by the 1987 Constitution under section 21, Article VI, xxx       xxx       xxx

c. Legislative supervision

The third and most encompassing form by which Congress exercises its oversight power is thru legislative supervision.
"Supervision" connotes a continuing and informed awareness on the part of a congressional committee
regarding executive operations in a given administrative area. While both congressional scrutiny and investigation
involve inquiry into past executive branch actions in order to influence future executive branch
performance, congressional supervision allows Congress to scrutinize the exercise of delegated law-making authority,
and permits Congress to retain part of that delegated authority.

Congress exercises supervision over the executive agencies through its veto power. It typically utilizes veto provisions
when granting the President or an executive agency the power to promulgate regulations with the force of law. These
provisions require the President or an agency to present the proposed regulations to Congress, which retains a "right" to
approve or disapprove any regulation before it takes effect. Such legislative veto provisions usually provide that a
proposed regulation will become a law after the expiration of a certain period of time, only if Congress does not
affirmatively disapprove of the regulation in the meantime. Less frequently, the statute provides that a proposed
regulation will become law if Congress affirmatively approves it.

Supporters of legislative veto stress that it is necessary to maintain the balance of power between the legislative and the
executive branches of government as it offers lawmakers a way to delegate vast power to the executive branch or to
independent agencies while retaining the option to cancel particular exercise of such power without having to pass new
legislation or to repeal existing law. They contend that this arrangement promotes democratic accountability as it
provides legislative check on the activities of unelected administrative agencies. One proponent thus explains:

It is too late to debate the merits of this delegation policy: the policy is too deeply embedded in our law and
practice. It suffices to say that the complexities of modern government have often led Congress-whether by
actual or perceived necessity- to legislate by declaring broad policy goals and general statutory standards,
leaving the choice of policy options to the discretion of an executive officer. Congress articulates legislative
aims, but leaves their implementation to the judgment of parties who may or may not have participated in or
agreed with the development of those aims. Consequently, absent safeguards, in many instances the reverse of
our constitutional scheme could be effected: Congress proposes, the Executive disposes. One safeguard, of
course, is the legislative power to enact new legislation or to change existing law. But without some means of
overseeing post enactment activities of the executive branch, Congress would be unable to determine whether
its policies have been implemented in accordance with legislative intent and thus whether legislative intervention
is appropriate.

Its opponents, however, criticize the legislative veto as undue encroachment upon the executive prerogatives. They
urge that any post-enactment measures undertaken by the legislative branch should be limited to scrutiny and
investigation; any measure beyond that would undermine the separation of powers guaranteed by the
Constitution. They contend that legislative veto constitutes an impermissible evasion of the President’s veto authority
and intrusion into the powers vested in the executive or judicial branches of government. Proponents counter that
legislative veto enhances separation of powers as it prevents the executive branch and independent agencies from
accumulating too much power. They submit that reporting requirements and congressional committee investigations
allow Congress to scrutinize only the exercise of delegated law-making authority. They do not allow Congress to review
executive proposals before they take effect and they do not afford the opportunity for ongoing and binding expressions
of congressional intent. In contrast, legislative veto permits Congress to participate prospectively in the approval or
disapproval of "subordinate law" or those enacted by the executive branch pursuant to a delegation of authority by
Congress. They further argue that legislative veto "is a necessary response by Congress to the accretion of policy
control by forces outside its chambers." In an era of delegated authority, they point out that legislative veto "is the most
efficient means Congress has yet devised to retain control over the evolution and implementation of its policy as
declared by statute."

In Immigration and Naturalization Service v. Chadha, the U.S. Supreme Court resolved the validity of legislative
veto provisions. The case arose from the order of the immigration judge suspending the deportation of Chadha
pursuant to § 244(c)(1) of the Immigration and Nationality Act. The United States House of Representatives passed a
resolution vetoing the suspension pursuant to § 244(c)(2) authorizing either House of Congress, by resolution, to
invalidate the decision of the executive branch to allow a particular deportable alien to remain in the United States. The
immigration judge reopened the deportation proceedings to implement the House order and the alien was ordered
deported. The Board of Immigration Appeals dismissed the alien’s appeal, holding that it had no power to declare
unconstitutional an act of Congress. The United States Court of Appeals for Ninth Circuit held that the House was
without constitutional authority to order the alien’s deportation and that § 244(c)(2) violated the constitutional doctrine on
separation of powers.

On appeal, the U.S. Supreme Court declared § 244(c)(2) unconstitutional. But the Court shied away from the issue of
separation of powers and instead held that the provision violates the presentment clause and bicameralism. It held that
the one-house veto was essentially legislative in purpose and effect. As such, it is subject to the procedures set out in
Article I of the Constitution requiring the passage by a majority of both Houses and presentment to the President. x x x x
xxxxx

Two weeks after the Chadha decision, the Court upheld, in memorandum decision, two lower court decisions
invalidating the legislative veto provisions in the Natural Gas Policy Act of 1978 and the Federal Trade Commission
Improvement Act of 1980. Following this precedence, lower courts invalidated statutes containing legislative veto
provisions although some of these provisions required the approval of both Houses of Congress and thus met the
bicameralism requirement of Article I. Indeed, some of these veto provisions were not even exercised. 35 (emphasis
supplied)

In Macalintal, given the concept and configuration of the power of congressional oversight and considering the nature and
powers of a constitutional body like the Commission on Elections, the Court struck down the provision in RA 9189 (The
Overseas Absentee Voting Act of 2003) creating a Joint Congressional Committee. The committee was tasked not only to
monitor and evaluate the implementation of the said law but also to review, revise, amend and approve the IRR promulgated by
the Commission on Elections. The Court held that these functions infringed on the constitutional independence of the
Commission on Elections.36

With this backdrop, it is clear that congressional oversight is not unconstitutional  per se, meaning, it neither necessarily
constitutes an encroachment on the executive power to implement laws nor undermines the constitutional separation of powers.
Rather, it is integral to the checks and balances inherent in a democratic system of government. It may in fact even enhance the
separation of powers as it prevents the over-accumulation of power in the executive branch.

However, to forestall the danger of congressional encroachment "beyond the legislative sphere," the Constitution imposes two
basic and related constraints on Congress.37 It may not vest itself, any of its committees or its members with either executive or
judicial power.38 And, when it exercises its legislative power, it must follow the "single, finely wrought and exhaustively
considered, procedures" specified under the Constitution, 39 including the procedure for enactment of laws and presentment.

Thus, any post-enactment congressional measure such as this should be limited to scrutiny and investigation. In particular,
congressional oversight must be confined to the following:

(1) scrutiny based primarily on Congress’ power of appropriation and the budget hearings conducted in connection with
it, its power to ask heads of departments to appear before and be heard by either of its Houses on any matter pertaining
to their departments and its power of confirmation 40 and

(2) investigation and monitoring41 of the implementation of laws pursuant to the power of Congress to conduct inquiries
in aid of legislation.42

Any action or step beyond that will undermine the separation of powers guaranteed by the Constitution. Legislative vetoes fall in
this class.

Legislative veto is a statutory provision requiring the President or an administrative agency to present the proposed
implementing rules and regulations of a law to Congress which, by itself or through a committee formed by it, retains a "right" or
"power" to approve or disapprove such regulations before they take effect. As such, a legislative veto in the form of a
congressional oversight committee is in the form of an inward-turning delegation designed to attach a congressional leash (other
than through scrutiny and investigation) to an agency to which Congress has by law initially delegated broad powers. 43 It
radically changes the design or structure of the Constitution’s diagram of power as it entrusts to Congress a direct role in
enforcing, applying or implementing its own laws.44

Congress has two options when enacting legislation to define national policy within the broad horizons of its legislative
competence.45 It can itself formulate the details or it can assign to the executive branch the responsibility for making necessary
managerial decisions in conformity with those standards. 46 In the latter case, the law must be complete in all its essential terms
and conditions when it leaves the hands of the legislature. 47 Thus, what is left for the executive branch or the concerned
administrative agency when it formulates rules and regulations implementing the law is to fill up details (supplementary rule-
making) or ascertain facts necessary to bring the law into actual operation (contingent rule-making). 48

Administrative regulations enacted by administrative agencies to implement and interpret the law which they are entrusted to
enforce have the force of law and are entitled to respect. 49 Such rules and regulations partake of the nature of a statute 50 and are
just as binding as if they have been written in the statute itself. As such, they have the force and effect of law and enjoy the
presumption of constitutionality and legality until they are set aside with finality in an appropriate case by a competent
court.51 Congress, in the guise of assuming the role of an overseer, may not pass upon their legality by subjecting them to its
stamp of approval without disturbing the calculated balance of powers established by the Constitution. In exercising discretion to
approve or disapprove the IRR based on a determination of whether or not they conformed with the provisions of RA 9335,
Congress arrogated judicial power unto itself, a power exclusively vested in this Court by the Constitution.

Considered Opinion of
Mr. Justice Dante O. Tinga

Moreover, the requirement that the implementing rules of a law be subjected to approval by Congress as a condition for their
effectivity violates the cardinal constitutional principles of bicameralism and the rule on presentment. 52

Section 1, Article VI of the Constitution states:

Section 1. The legislative power shall be vested in the Congress of the Philippines which shall consist of a
Senate and a House of Representatives, except to the extent reserved to the people by the provision on initiative and
referendum. (emphasis supplied)

Legislative power (or the power to propose, enact, amend and repeal laws) 53 is vested in Congress which consists of two
chambers, the Senate and the House of Representatives. A valid exercise of legislative power requires the act of both
chambers. Corrollarily, it can be exercised neither solely by one of the two chambers nor by a committee of either or both
chambers. Thus, assuming the validity of a legislative veto, both a single-chamber legislative veto and a congressional
committee legislative veto are invalid.

Additionally, Section 27(1), Article VI of the Constitution provides:

Section 27. (1) Every bill passed by the Congress shall, before it becomes a law, be presented to the President. If
he approves the same, he shall sign it, otherwise, he shall veto it and return the same with his objections to the House
where it originated, which shall enter the objections at large in its Journal and proceed to reconsider it. If, after such
reconsideration, two-thirds of all the Members of such House shall agree to pass the bill, it shall be sent, together with
the objections, to the other House by which it shall likewise be reconsidered, and if approved by two-thirds of all the
Members of that House, it shall become a law. In all such cases, the votes of each House shall be determined
by yeas or nays, and the names of the members voting for or against shall be entered in its Journal. The President shall
communicate his veto of any bill to the House where it originated within thirty days after the date of receipt thereof;
otherwise, it shall become a law as if he had signed it. (emphasis supplied)

Every bill passed by Congress must be presented to the President for approval or veto. In the absence of presentment to the
President, no bill passed by Congress can become a law. In this sense, law-making under the Constitution is a joint act of the
Legislature and of the Executive. Assuming that legislative veto is a valid legislative act with the force of law, it cannot take effect
without such presentment even if approved by both chambers of Congress.

In sum, two steps are required before a bill becomes a law. First, it must be approved by both Houses of Congress. 54 Second, it
must be presented to and approved by the President. 55 As summarized by Justice Isagani Cruz56 and Fr. Joaquin G. Bernas,
S.J.57, the following is the procedure for the approval of bills:

A bill is introduced by any member of the House of Representatives or the Senate except for some measures that must
originate only in the former chamber.

The first reading involves only a reading of the number and title of the measure and its referral by the Senate President
or the Speaker to the proper committee for study.

The bill may be "killed" in the committee or it may be recommended for approval, with or without amendments,
sometimes after public hearings are first held thereon. If there are other bills of the same nature or purpose, they may all
be consolidated into one bill under common authorship or as a committee bill.

Once reported out, the bill shall be calendared for second reading. It is at this stage that the bill is read in its entirety,
scrutinized, debated upon and amended when desired. The second reading is the most important stage in the passage
of a bill.

The bill as approved on second reading is printed in its final form and copies thereof are distributed at least three days
before the third reading. On the third reading, the members merely register their votes and explain them if they are
allowed by the rules. No further debate is allowed.

Once the bill passes third reading, it is sent to the other chamber, where it will also undergo the three readings. If there
are differences between the versions approved by the two chambers, a conference committee 58 representing both
Houses will draft a compromise measure that if ratified by the Senate and the House of Representatives will then be
submitted to the President for his consideration.

The bill is enrolled when printed as finally approved by the Congress, thereafter authenticated with the signatures of the
Senate President, the Speaker, and the Secretaries of their respective chambers… 59

The President’s role in law-making.

The final step is submission to the President for approval. Once approved, it takes effect as law after the required
publication.60

Where Congress delegates the formulation of rules to implement the law it has enacted pursuant to sufficient standards
established in the said law, the law must be complete in all its essential terms and conditions when it leaves the hands of the
legislature. And it may be deemed to have left the hands of the legislature when it becomes effective because it is only upon
effectivity of the statute that legal rights and obligations become available to those entitled by the language of the statute.
Subject to the indispensable requisite of publication under the due process clause, 61 the determination as to when a law takes
effect is wholly the prerogative of Congress.62 As such, it is only upon its effectivity that a law may be executed and the executive
branch acquires the duties and powers to execute the said law. Before that point, the role of the executive branch, particularly of
the President, is limited to approving or vetoing the law. 63

From the moment the law becomes effective, any provision of law that empowers Congress or any of its members to play any
role in the implementation or enforcement of the law violates the principle of separation of powers and is thus unconstitutional.
Under this principle, a provision that requires Congress or its members to approve the implementing rules of a law after it has
already taken effect shall be unconstitutional, as is a provision that allows Congress or its members to overturn any directive or
ruling made by the members of the executive branch charged with the implementation of the law.

Following this rationale, Section 12 of RA 9335 should be struck down as unconstitutional. While there may be similar provisions
of other laws that may be invalidated for failure to pass this standard, the Court refrains from invalidating them wholesale but will
do so at the proper time when an appropriate case assailing those provisions is brought before us. 64

The next question to be resolved is: what is the effect of the unconstitutionality of Section 12 of RA 9335 on the other provisions
of the law? Will it render the entire law unconstitutional? No.

Section 13 of RA 9335 provides:

SEC. 13. Separability Clause. – If any provision of this Act is declared invalid by a competent court, the remainder of this
Act or any provision not affected by such declaration of invalidity shall remain in force and effect.

In Tatad v. Secretary of the Department of Energy,65 the Court laid down the following rules:

The general rule is that where part of a statute is void as repugnant to the Constitution, while another part is valid, the
valid portion, if separable from the invalid, may stand and be enforced. The presence of a separability clause in a statute
creates the presumption that the legislature intended separability, rather than complete nullity of the statute. To justify
this result, the valid portion must be so far independent of the invalid portion that it is fair to presume that the legislature
would have enacted it by itself if it had supposed that it could not constitutionally enact the other. Enough must remain to
make a complete, intelligible and valid statute, which carries out the legislative intent. x x x

The exception to the general rule  is that when the parts of a statute are so mutually dependent and connected, as
conditions, considerations, inducements, or compensations for each other, as to warrant a belief that the legislature
intended them as a whole, the nullity of one part will vitiate the rest. In making the parts of the statute dependent,
conditional, or connected with one another, the legislature intended the statute to be carried out as a whole and would
not have enacted it if one part is void, in which case if some parts are unconstitutional, all the other provisions thus
dependent, conditional, or connected must fall with them.

The separability clause of RA 9335 reveals the intention of the legislature to isolate and detach any invalid provision from the
other provisions so that the latter may continue in force and effect. The valid portions can stand independently of the invalid
section. Without Section 12, the remaining provisions still constitute a complete, intelligible and valid law which carries out the
legislative intent to optimize the revenue-generation capability and collection of the BIR and the BOC by providing for a system
of rewards and sanctions through the Rewards and Incentives Fund and a Revenue Performance Evaluation Board.

To be effective, administrative rules and regulations must be published in full if their purpose is to enforce or implement existing
law pursuant to a valid delegation. The IRR of RA 9335 were published on May 30, 2006 in two newspapers of general
circulation66 and became effective 15 days thereafter.67 Until and unless the contrary is shown, the IRR are presumed valid and
effective even without the approval of the Joint Congressional Oversight Committee.

WHEREFORE, the petition is hereby PARTIALLY GRANTED. Section 12 of RA 9335 creating a Joint Congressional Oversight
Committee to approve the implementing rules and regulations of the law is declared UNCONSTITUTIONAL and
therefore NULL and VOID. The constitutionality of the remaining provisions of RA 9335 is UPHELD. Pursuant to Section 13 of
RA 9335, the rest of the provisions remain in force and effect.

SO ORDERED.

- Balag v Senate Committee on Public Order, GR 234608, Jul 3, 2018 (Contempt power of Congress)
This is a petition for certiorari and prohibition with prayer for issuance of a temporary restraining order (TRO) and/or writ of
preliminary injunction seeking to annul, set aside and enjoin the implementation of Senate P.S. Resolution (SR) No.
504  and the October 18, 2017 Order  (Contempt Order)of the Senate Committee on Public Order and Dangerous Drugs
1 2

citing Arvin Balag (petitioner) in contempt.

The Antecedents

On September 17, 2017, Horacio Tomas T. Castillo III (Horacio Ill),   a first year law student of the University of Sto.
3

Tomas (UST), died allegedly due to hazing conducted by the Aegis Juris Fraternity (AJ Fraternity) of the same university.

On September 19, 2017, SR No. 504,  was filed by Senator Juan Miguel Zubiri (Senator Zubiri)  condemning the death of
4 5

Horacio III and directing the appropriate Senate Committee to conduct an investigation, in aid of legislation, to hold those
responsible accountable.

On September 20, 2017, SR No. 510, entitled: "A Resolution Directing the Appropriate Senate Committees to Conduct An
Inquiry, In Aid of Legislation, into the Recent Death of Horacio Tomas Castillo III Allegedly Due to Hazing-Related
Activities" was filed by Senator Paolo Benigno Aquino IV. 6

On the same day, the Senate Committee on Public Order and Dangerous Drugs chaired by Senator Panfilo
Lacson (Senator Lacson) together with the Committees on Justice and Human Rights and Constitutional Amendment and
Revision of Codes, invited petitioner and several other persons to the Joint Public Hearing on September 25, 2017 to
discuss and deliberate the following: Senate Bill Nos. 27,  199,  223,  1161,  1591,  and SR No. 504.
7 8 9 10 11

Petitioner, however, did not attend the hearing scheduled on September 25, 2017. Nevertheless, John Paul Solano, a
member of AJ Fraternity, Atty. Nilo T. Divina, Dean of UST Institute of Civil Law and Arthur Capili, UST Faculty Secretary,
attended the hearing and were questioned by the senate committee members.

On the same date, Spouses Carmina T. Castillo and Horacio M. Castillo, Jr. (Spouses Castillo), parents of Horacio III, filed
a Criminal Complaint  for Murder and violation of Section 4 of Republic Act (R.A.) No. 8049,   before the Department of
12 13

Justice (DOJ) against several members of the AJ Fraternity, including petitioner. On October 9, 2017, Spouses Castillo
filed a Supplemental Complaint-Affidavit   before the DOJ citing the relevant transcripts of stenographic notes during the
14

September 25, 2017 Senate Hearing.

On October 11, 2017, Senator Lacson as Chairman of Senate Committee on Public Order and Dangerous Drugs, and as
approved by Senate President Aquilino Pimentel III, issued a Subpoena Ad Testifzcandum   addressed to petitioner
15

directing him to appear before the committee and to testify as to the subject matter under inquiry.   Another Subpoena Ad
16

Testifzcandum   was issued on October 17, 2017, which was received by petitioner on the same day, requiring him to
17

attend the legislative hearing on October 18, 2017.

On said date, petitioner attended the senate hearing. In the course of the proceedings, at around 11:29 in the morning,
Senator Grace Poe (Senator Poe) asked petitioner if he was the president of AJ Fraternity but he refused to answer the
question and invoked his right against self-incrimination. Senator Poe repeated the question but he still refused to answer.
Senator Lacson then reminded him to answer the question because it was a very simple question, otherwise, he could be
cited in contempt. Senator Poe retorted that petitioner might still be clinging to the supposed "Code of Silence" in his
alleged text messages to his fraternity. She manifested that petitioner's signature appeared on the application for
recognition of the AJ Fraternity and on the organizational sheet, indicating that he was the president. Petitioner, again,
invoked his right against self-incrimination. Senator Poe then moved to cite him in contempt, which was seconded by
Senators Joel Villanueva (Senator Villanueva) and Zubiri. Senator Lacson ruled that the motion was properly seconded,
hence, the Senate Sergeant-at-arms was ordered to place petitioner in detention after the committee hearing. Allegedly,
Senator Lacson threatened to order the detention of petitioner in Pasay City Jail under the custody of the Senate
Sergeant-at-arms and told him not to be evasive because he would be merely affirming school records.

A few minutes later, at around 12:09 in the afternoon, Senators Lacson and Poe gave petitioner another chance to purge
himself of the contempt charge. Again, he was asked the same question twice and each time he refused to answer.  18

Thereafter, around 1: 19 in the afternoon, Senator Villanueva inquired from petitioner whether he knew whose decision it
was to bring Horacio III to the Chinese General Hospital instead of the UST Hospital. Petitioner apologized for his earlier
statement and moved for the lifting of his contempt. He admitted that he was a member of the AJ Fraternity but he was not
aware as to who its president was because, at that time, he was enrolled in another school.

Senator Villanueva repeated his question to petitioner but the latter, again, invoked his right against self-incrimination.
Petitioner reiterated his plea that the contempt order be lifted because he had already answered the question regarding
his membership in the AJ Fraternity. Senator Villanueva replied that petitioner's contempt would remain. Senator Lacson
added that he had numerous opportunities to answer the questions of the committee but he refused to do so. Thus,
petitioner was placed under the custody of the Senate Sergeant-at-arms. The Contempt Order reads:

RE: PRIVILEGE SPEECH OF SEN. JUAN MIGUEL ZUBIRI ON THE DEATH OF HORA TIO "A TIO" CASTILLO III DUE
TO HAZING DELIVERED ON 20 SEPTEMBER 2017;

PS RES. NO. 504: RESOLUTION CONDEMNING IN THE STRONGEST SENSE THE DEATH OF FRESHMAN LAW
STUDENT HORA TIO TOMAS CASTILLO III AND DIRECTING THE APPROPRIATE SENA TE COMMITTEES TO
CONDUCT AN INVESTIGATION, IN AID OF LEGISLATION, TO HOLD ACCOUNTABLE THOSE RESPONSIBLE FOR
THIS SENSELESS ACT (SEN. ZUBIRI); AND

SENATE BILLS NOS. 27, 199, 223, 1161, AND 1591.

xxx

For testifying falsely and evasively before the Committee on [October 18, 2017] and thereby delaying, impeding and
obstructing the inquiry into the death of Horacio "Atio" Castillo III. Thereupon the motion of Senator Grace Poe and
seconded by Senator Joel Villanueva and Senator Juan Miguel Zubiri, the Committee hereby cites MR. ARVIN BALAG in
contempt and ordered arrested and detained at the Office of the Sergeant-at-Arms until such time that he gives his true
testimony, or otherwise purges himself of that contempt.

The Sergeant-at-Arms is hereby directed to carry out and implement this Order and make a return hereof within twenty-
four (24) hours from its enforcement.

SO ORDERED. 19

Hence, this petition.

ISSUE

WHETHER RESPONDENT SENATE COMMITTEES ACTED WITH GRAVE ABUSE OF DISCRETION IN CONDUCTING
THE LEGISLATIVE INQUIRY AND CITING PETITIONER IN CONTEMPT.

Petitioner chiefly argues that the legislative inquiry conducted by respondent committees was not in aid of legislation;
rather, it was in aid of prosecution. He posits that the purpose of SR No. 504 was to hold accountable those responsible
for the senseless act of killing Horacio III, and not to aid legislation. Petitioner underscores that the transcripts during the
September 25, 2017 committee hearing were used in the criminal complaint filed against him, which bolsters that the said
hearings were in aid of prosecution. He insists that the senate hearings would violate his right to due process and would
pre-empt the findings of the DOJ with respect to the criminal complaint filed against him.

Petitioner also asserts that he properly invoked his right against self-incrimination as the questions propounded by Senator
Poe regarding the officers, particularly the presidency of the AJ Fraternity, were incriminating because the answer thereto
involves an element of the crime of hazing. Despite the questions being incriminating, he, nonetheless, answered them by
admitting that he was a member of the AJ Fraternity but he did not know of its current president because he transferred to
another school. He adds that his right to equal protection of laws was violated because the other resource persons who
refused to answer the questions of the Senate committees were not cited in contempt.

Finally, petitioner prays for the issuance of TRO and/or writ of preliminary injunction because the Senate illegally enforced
and executed SR No. 504 and the Contempt Order, which caused him grave and irreparable injury as he was deprived of
his liberty without due process of law. He contends that respondents did not exercise their power of contempt judiciously
and with restraint.

In their Comment,   respondents, through the Office of the Senate Legal Counsel, countered that the purpose of the
20

hearing was to re-examine R.A. No. 8049; that several documents showed that the legislative hearing referred to Senate
Bill Nos. 27, 199, 223, 1161, and 1591; that the statement of the senators during the hearing demonstrated that the
legislative inquiry was conducted in aid of legislation; and that the Senate Rules of Procedure Governing Inquiries in Aid of
Legislation (Senate Rules) were duly published.

Respondents emphasized that petitioner was first asked on October 18, 2017, around 11:29 in the morning, whether he
was the president of the AJ Fraternity, based on school records, and he denied it; he was asked again at 12:09 in the
afternoon whether he was the president of the AJ Fraternity but he still refused to answer the question; at 1:19 in the
afternoon, he admitted that he was a member of the fraternity but still he refused to say whether or not he was the
president, only saying that he is already studying in another school. On November 6, 2017, at the resumption of the
hearing, petitioner was still unresponsive. According to respondents, these acts were contemptuous and were valid
reasons to cite petitioner in contempt.

Respondents highlighted that there were numerous documents showing that petitioner was the president of the AJ
Fraternity but he continually refused to answer. They added that petitioner cannot purge himself of contempt by continually
lying.

Further, respondents underscored that the question propounded to petitioner was not incriminating because an admission
that he was an officer of the AJ Fraternity would not automatically make him liable under R.A. No. 8049. They emphasized
that the Senate respected petitioner's right to due process because the hearing was conducted in aid of legislation; that
the senators explained why he would be cited in contempt; that he was given several chances to properly purge himself
from contempt; and that no incriminating question was asked. Respondents concluded that there was no violation of
petitioner's right to equal protection of laws because the other resource persons did not invoke their right against self-
incrimination when asked if they were the officers of the AJ Fraternity.
Respondents likewise explained that the legislative inquiry in aid of legislation may still continue in spite of any pending
criminal or administrative cases or investigation. They countered that the actions for certiorari and prohibition were not
proper because there were existing remedies that petitioner could have availed of, particularly: a motion to reverse the
contempt charge filed within 7 days under Section 18 of the Senate Rules; and a petition for habeas corpus as petitioner
ultimately would seek for his release from detention.

Finally, respondents asserted that the recourse for the issuance of TRO and/or writ of preliminary injunction was not
proper because petitioner was actually asking to be freed from detention, and this was contemplated under a status quo
ante order. For invoking the wrong remedy, respondents concluded that a TRO and/or writ of preliminary injunction should
not be issued.

In its Resolution  dated December 12, 2017, the Court ordered in the interim the immediate release of petitioner pending
21

resolution of the instant petition.

In its Manifestation  dated February 20, 2018, respondents stated that on January 23, 2018, the Committees on Public
22

Order and Dangerous Drugs and Justice and Human Rights jointly submitted Committee Report Nos. 232 and 233
recommending that Senate Bill No. 1662 be approved in substitution of Senate Bill Nos. 27, 199, 223, 1161, 1591, and
1609. The said committee reports were approved by the majority of their members.  On February 12, 2018, the Senate
23

passed on 3rd reading Senate Bill No. 1662, entitled: An Act Amending Republic Act No. 8049 to Strengthen the Law on
Hazing and Regulate Other Forms of Initiation Rites of Fraternities, Sororities, and Other Organizations, Providing
Penalties Therefor, and for Other Purposes, with its short title as "Anti-Hazing Act of 2018."

The Court's Ruling

The petition is moot and academic.

The existence of an actual case or controversy is a necessary condition precedent to the court's exercise of its power of
adjudication. An actual case or controversy exists when there is a conflict of legal rights or an assertion of opposite legal
claims between the parties that is susceptible or ripe for judicial resolution. In the negative, a justiciable controversy must
neither be conjectural nor moot and academic. There must be a definite and concrete dispute touching on the legal
relations of the parties who have adverse legal interests. The reason is that the issue ceases to be justiciable when a
controversy becomes moot and academic; otherwise, the court would engage in rendering an advisory opinion on what
the law would be upon a hypothetical state of facts.  24

In this case, the Court finds that there is no more justiciable controversy. Petitioner essentially alleges that respondents
unlawfully exercised their power of contempt and that his detention was invalid. As discussed earlier, in its resolution
dated December 12, 2017, the Court ordered in the interim the immediate release of petitioner pending resolution of the
instant petition. Thus, petitioner was no longer detained under the Senate's authority.

Then, on January 23, 2018, the Committees on Public Order and Dangerous Drugs and Justice and Human Rights jointly
adopted Committee Report Nos. 232 and 233 and submitted the same to the Senate. Committee Report No. 232 referred
to the findings of respondent committees in the inquiry conducted in aid of legislation; while Committee Report No. 233
referred to the recommendation that Senate Bill No. 1662 be approved in substitution of Senate Bill Nos. 27, 199, 223,
1161, 1591, and 1609. On February 12, 2018, the Senate passed on 3rd reading Senate Bill No. 1662.

Evidently, respondent committees have terminated their legislative inquiry upon the approval of Committee Report Nos.
232 and 233 by the majority of its members. The Senate even went further by approving on its 3rd reading the proposed
bill, Senate Bill No. 1662, the result of the inquiry in aid of legislation. As the legislative inquiry ends, the basis for the
detention of petitioner likewise ends.

Accordingly, there is no more justiciable controversy regarding respondents' exercise of their constitutional power to
conduct inquiries in aid of legislation, their power of contempt, and the validity of petitioner's detention. Indeed, the petition
has become moot and academic.

Nevertheless, there were occasions in the past when the Court passed upon issues although supervening events had
rendered those petitions moot and academic. After all, the moot and academic principle is not a magical formula that can
automatically dissuade the courts from resolving a case. Courts will decide cases, otherwise moot and academic.   This 25

Court may assume jurisdiction over a case that has been rendered moot and academic by supervening events when any
of the following instances are present:

(1) Grave constitutional violations;

(2) Exceptional character of the case;

(3) Paramount public interest;

(4) The case presents an opportunity to guide the bench, the bar, and the public; or

(5) The case is capable of repetition yet evading review. 26


In David v. Arroyo,  several petitions assailed the constitutionality of the declaration of a state of national emergency by
27

then President Gloria Macapagal-Arroyo. During the pendency of the suits, the said declaration was lifted. However, the
Court still decided the cases on the merits because the issues involved a grave violation of the Constitution and it affected
public interest.

Similarly, in Republic v. Principalia Management and Personnel Consultants, Inc.,   the controversy therein was whether
28

the Regional Trial Court (RTC) had jurisdiction over an injunction complaint filed against the Philippine Overseas
Employment Administration (POEA) regarding the cancellation of the respondent's license. The respondent then argued
that the case was already moot and academic because it had continuously renewed its license with the POEA. The Court
ruled that although the case was moot and academic, it could still pass upon the main issue for the guidance of both bar
and bench, and because the said issue was capable of repetition.

Recently, in Regulus Development, Inc. v. Dela Cruz,   the issue therein was moot and academic due to the redemption of
29

the subject property by the respondent. However, the Court ruled that it may still entertain the jurisdictional issue of
whether the RTC had equity jurisdiction in ordering the levy of the respondent's property since it posed a situation capable
of repetition yet evading judicial review.

In this case, the petition presents a critical and decisive issue that must be addressed by Court: what is the duration of the
detention for a contempt ordered by the Senate?

This issue must be threshed out as the Senate's exercise of its power of contempt without a definite period is capable of
repetition. Moreover, the indefinite detention of persons cited in contempt impairs their constitutional right to liberty. Thus,
paramount public interest requires the Court to determine such issue to ensure that the constitutional rights of the persons
appearing before a legislative inquiry of the Senate are protected.

The contempt order issued against petitioner simply stated that he would be arrested and detained until such time that he
gives his true testimony, or otherwise purges himself of the contempt. It does not provide any definite and concrete period
of detention. Neither does the Senate Rules specify a precise period of detention when a person is cited in contempt.

Thus, a review of the Constitution and relevant laws and jurisprudence must be conducted to determine whether there is a
limitation to the period of detention when the Senate exercises its power of contempt during inquiries in aid of legislation.

Period of imprisonment for contempt


during inquiries in aid of legislation

The contempt power of the legislature under our Constitution is sourced from the American system.   A study of foreign
30

jurisprudence reveals that the Congress' inherent power of contempt must have a limitation. In the 1821 landmark case
of Anderson v. Dunn,   the Supreme Court of the United States (SCOTUS) held that although the offense committed
31

under the inherent power of contempt by Congress may be undefinable, it is justly contended that the punishment need
not be indefinite. It held that as the legislative body ceases to exist from the moment of its adjournment or periodical
dissolution, then it follows that imprisonment under the contempt power of Congress must terminate with adjournment.

As the US Congress was restricted of incarcerating an erring witnesses beyond their adjournment under its inherent power
of contempt, it enacted a statutory law that would fix the period of imprisonment under legislative contempt. Section 102 of
the Revised Statutes, enacted on January 24, 1857, provided that the penalty of imprisonment for legislative contempt
was a fine of not more than one thousand dollars nor less than one hundred dollars, and imprisonment in a common jail
for not less than one (1) month nor more than twelve (12) months.  The legislative contempt under the statutes must be
32

initiated for criminal prosecution and it must heard before the courts in order to convict the contumacious witness.  33

The case of In re Chapman  involved the constitutionality of the statutory power of contempt of the US Congress. There,
34

the SCOTUS ruled that the said statute was valid because Congress, by enacting this law, simply sought to aid each of
the Houses in the discharge of its constitutional functions.

Subsequently, in Jurney v. MacCracken,   the SCOTUS clarified that the power of either Houses of Congress to punish for
35

contempt was not impaired by the enactment of the 1857 statute. The said law was enacted, not because the power of
both Houses to punish for a past contempt was doubted, but because imprisonment limited to the duration of the session
was not considered sufficiently drastic as a punishment for contumacious witnesses. The purpose of the statutory
contempt was merely to supplement the inherent power of contempt by providing for additional punishment. On June 22,
1938, Section 102 of the Revised Statutes was codified in Section 192, Title II of the U.S. Code. 36

In our jurisdiction, the period of the imprisonment for contempt by Congress was first discussed in Lopez v. De Los
Reyes  (Lopez). In that case, on September 16, 1930, the petitioner therein was cited in contempt by the House of
37

Representatives for physically attacking their member. However, the assault occurred during the Second Congress, which
adjourned on November 8, 1929. The Court ruled therein that there was no valid exercise of the inherent power of
contempt because the House of Representatives already adjourned when it declared the petitioner in contempt.

It was held therein that imprisonment for a term not exceeding the session of the deliberative body in which the contempt
occurred was the limit of the authority to deal directly by way of contempt, without criminal prosecution. Citing foreign
jurisprudence, it was thoroughly discussed therein that the power of contempt was limited to imprisonment during the
session of the legislative body affected by the contempt. The Court also discussed the nature of Congress' inherent power
of contempt as follows:
x x x We have said that the power to find in contempt rests fundamentally on the power of self-preservation. That
is true even of contempt of court where the power to punish is exercised on the preservative and not on the vindictive
principle. Where more is desired, where punishment as such is to be imposed, a criminal prosecution must be
brought, and in all fairness to the culprit, he must have thrown around him all the protections afforded by the Bill
of Rights. Proceeding a step further, it is evident that, while the legislative power is perpetual, and while one of the bodies
composing the legislative power disappears only every three years, yet the sessions of that body mark new beginnings
and abrupt endings, which must be respected.  (emphases supplied)
38

The Court ruled therein that if the House of Representatives desires to punish the person cited in contempt beyond its
adjournment, then criminal prosecution must be brought. In that instance, the said person shall be given an opportunity to
defend himself before the courts.

Then came Arnault v. Nazareno  (Arnault), where the Senate's power of contempt was discussed. In that case, the Court
39

held that the Senate "is a continuing body and which does not cease to exist upon the periodical dissolution of Congress
or of the House of Representatives. There is no limit as to time [with] the Senate's power to punish for contempt in cases
where that power may constitutionally be exerted xxx"  It was ruled therein that had contempt been exercised by the
40

House of Representatives, the contempt could be enforced until the final adjournment of the last session of the said
Congress. 41

Notably, Arnault gave a distinction between the Senate and the House of Representatives' power of contempt. In the
former, since it is a continuing body, there is no time limit in the exercise of its power to punish for contempt; on the other
hand, the House of Representatives, as it is not a continuing body, has a limit in the exercise of its power to punish for
contempt, which is on the final adjournment of its last session. In the same case, the Court addressed the possibility that
the Senate might detain a witness for life, to wit:

As against the foregoing conclusion it is argued for the petitioner that the power may be abusively and oppressively
exerted by the Senate which might keep the witness in prison for life. But we must assume that the Senate will not be
disposed to exert the power beyond its proper bounds. And if, contrary to this assumption, proper limitations are
disregarded, the portals of this Court are always open to those whose rights might thus be transgressed.  42

Further, the Court refused to limit the period of imprisonment under the power of contempt of the Senate because
"[l]egislative functions may be performed during recess by duly constituted committees charged with the duty of performing
investigations or conducting hearings relative to any proposed legislation. To deny to such committees the power
of inquiry with process to enforce it would be to defeat the very purpose for which that power is recognized in the
legislative body as an essential and appropriate auxiliary to its legislative function. xxx."
43

Later, in Neri v. Senate  (Neri), the Court clarified the nature of the Senate as continuing body:
44

On the nature of the Senate as a "continuing body", this Court sees fit to issue a clarification. Certainly, there is no debate
that the Senate as an institution is "continuing'', as it is not dissolved as an entity with each national election or change in
the composition of its members. However, in the conduct of its day-to-day business the Senate of each Congress acts
separately and independently of the Senate of the Congress before it. The Rules of the Senate itself confirms this when it
states:

RULE XLIV
UNFINISHED BUSINESS

SEC. 123. Unfinished business at the end of the session shall be taken up at the next session in the same status.

All pending matters and proceedings shall terminate upon the expiration of one (1) Congress, but may be taken by the
succeeding Congress as if present for the first time.

Undeniably from the foregoing, all pending matters and proceedings, i.e., unpassed bills and even legislative
investigations, of the Senate of a particular Congress are considered terminated upon the expiration of that Congress and
it is merely optional on the Senate of the succeeding Congress to take up such unfinished matters, not in the same status,
but as if presented for the first time. The logic and practicality of such a rule is readily apparent considering that the Senate
of the succeeding Congress (which will typically have a different composition as that of the previous Congress) should not
be bound by the acts and deliberations of the Senate of which they had no part. If the Senate is a continuing body even
with respect to the conduct of its business, then pending matters will not be deemed terminated with the expiration of one
Congress but will, as a matter of course, continue into the next Congress with the same status. 45

Based on the above-pronouncement, the Senate is a continuing institution. However, in the conduct of its day-to-day
business, the Senate of each Congress acts separately and independently of the Senate of the Congress before it. Due to
the termination of the business of the Senate during the expiration of one (1) Congress, all pending matters and
proceedings, such as unpassed bills and even legislative investigations, of the Senate are considered terminated
upon the expiration of that Congress and it is merely optional on the Senate of the succeeding Congress to take up
such unfinished matters, not in the same status, but as if presented for the first time.

The termination of the Senate's business and proceedings after the expiration of Congress was utilized by the Court in
ruling that the Senate needs to publish its rules for its legislative inquiries in each Congress. The pronouncement
in Neri was reiterated in Garcillano v. House of Representatives  and Romero II v. Estrada.
46 47
The period of detention under
the Senate's inherent power of
contempt is not indefinite.

The Court finds that there is a genuine necessity to place a limitation on the period of imprisonment that may be imposed
by the Senate pursuant to its inherent power of contempt during inquiries in aid of legislation. Section 21, Article VI of the
Constitution states that Congress, in conducting inquiries in aid of legislation, must respect the rights of persons appearing
in or affected therein. Under Arnault, however, a witness or resource speaker cited in contempt by the Senate may be
detained indefinitely due to its characteristic as a continuing body. The said witness may be detained for a day, a month, a
year, or even for a lifetime depending on the desire of the perpetual Senate. Certainly, in that case, the rights of persons
appearing before or affected by the legislative inquiry are in jeopardy. The constitutional right to liberty that every citizen
enjoys certainly cannot be respected when they are detained for an indefinite period of time without due process of law.

As discussed in Lopez, Congress' power of contempt rests solely upon the right of self-preservation and does not extend
to the infliction of punishment as such. It is a means to an end and not the end itself.   Even arguendo that detention under
48

the legislative's inherent power of contempt is not entirely punitive in character because it may be used by Congress only
to secure information from a recalcitrant witness or to remove an obstruction, it is still a restriction to the liberty of the said
witness. It is when the restrictions during detention are arbitrary and purposeless that courts will infer intent to punish.
Courts will also infer intent to punish even if the restriction seems to be related rationally to the alternative purpose if the
restriction appears excessive in relation to that purpose.   An indefinite and unspecified period of detention will amount to
49

excessive restriction and will certainly violate any person's right to liberty.

Nevertheless, it is recognized that the Senate's inherent power of contempt is of utmost importance. A legislative body
cannot legislate wisely or effectively in the absence of information respecting the conditions which the legislations are
intended to affect or change. Mere requests for such information are often unavailing, and also that information which is
volunteered is not always accurate or complete; so some means of compulsion is essential to obtain what is needed
through the power of contempt during legislative inquiry.  While there is a presumption of regularity that the Senate will not
50

gravely abuse its power of contempt, there is still a lingering and unavoidable possibility of indefinite imprisonment of
witnesses as long as there is no specific period of detention, which is certainly not contemplated and envisioned by the
Constitution.

Thus, the Court must strike a balance between the interest of the Senate and the rights of persons cited in contempt
during legislative inquiries.  The balancing of interest requires that the Court take a conscious and detailed consideration
1avvphi1

of the interplay of interests observable in a given situation or type of situation. These interests usually consist in the
exercise by an individual of his basic freedoms on the one hand, and the government's promotion of fundamental public
interest or policy objectives on the other.51

The Court finds that the period of imprisonment under the inherent power of contempt by the Senate during
inquiries in aid of legislation should only last until the termination of the legislative inquiry under which the said
power is invoked. In Arnault, it was stated that obedience to its process may be enforced by the Senate Committee if the
subject of investigation before it was within the range of legitimate legislative inquiry and the proposed testimony called
relates to that subject.   Accordingly, as long as there is a legitimate legislative inquiry, then the inherent power of
52

contempt by the Senate may be properly exercised. Conversely, once the said legislative inquiry concludes, the exercise
of the inherent power of contempt ceases and there is no more genuine necessity to penalize the detained witness.

Further, the Court rules that the legislative inquiry of the Senate terminates on two instances:

First, upon the approval or disapproval of the Committee Report. Sections 22 and 23 of Senate Rules state:

Sec. 22. Report of Committee. Within fifteen (15) days after the conclusion of the inquiry, the Committee shall meet
to begin the consideration of its Report.

The Report shall be approved by a majority vote of all its members. Concurring and dissenting reports may likewise be
made by the members who do not sign the majority report within seventy-two (72) hours from the approval of the report.
The number of members who sign reports concurring in the conclusions of the Committee Report shall be taken into
account in determining whether the Report has been approved by a majority of the members: Provided, That the vote of a
member who submits both a concurring and dissenting opinion shall not be considered as part of the majority unless he
expressly indicates his vote for the majority position.

The Report, together with any concurring and/or dissenting opinions, shall be filed with the Secretary of the
Senate, who shall include the same in the next Order of Business.

Sec. 23. Action on Report. The Report, upon inclusion in the Order of Business, shall be referred to the Committee on
Rules for assignment in the Calendar. (emphases supplied)

As gleaned above, the Senate Committee is required to issue a Committee Report after the conduct of the legislative
inquiry. The importance of the Committee Report is highlighted in the Senate Rules because it mandates that the
committee begin the consideration of its Report within fifteen (15) days from the conclusion of the inquiry. The said
Committee Report shall then be approved by a majority vote of all its members; otherwise, it is disapproved. The said
Report shall be the subject matter of the next order of business, and it shall be acted upon by the Senate. Evidently, the
Committee Report is the culmination of the legislative inquiry. Its approval or disapproval signifies the end of such
legislative inquiry and it is now up to the Senate whether or not to act upon the said Committee Report in the succeeding
order of business. At that point, the power of contempt simultaneously ceases and the detained witness should be
released. As the legislative inquiry ends, the basis for the detention of the recalcitrant witness likewise ends.

Second, the legislative inquiry of the Senate also terminates upon the expiration of one (1) Congress. As stated in Neri, all
pending matters and proceedings, such as unpassed bills and even legislative investigations, of the Senate are
considered terminated upon the expiration of that Congress and it is merely optional on the Senate of the succeeding
Congress to take up such unfinished matters, not in the same status, but as if presented for the first time. Again, while the
Senate is a continuing institution, its proceedings are terminated upon the expiration of that Congress at the final
adjournment of its last session. Hence, as the legislative inquiry ends upon that expiration, the imprisonment of the
detained witnesses likewise ends.

In Arnault, there have been fears that placing a limitation on the period of imprisonment pursuant to the Senate's power of
contempt would "deny to it an essential and appropriate means for its performance."  Also, in view of the limited period of
53

imprisonment, "the Senate would have to resume the investigation at the next and succeeding sessions and repeat the
contempt proceedings against the witness until the investigation is completed xxx." 54

The Court is of the view that these fears are insufficient to permit an indefinite or an unspecified period of imprisonment
under the Senate's inherent power of contempt. If Congress believes that there is a necessity to supplement its power of
contempt by extending the period of imprisonment beyond the conduct of its legislative inquiry or beyond its final
adjournment of the last session, then it can enact a law or amend the existing law that penalizes the refusal of a witness to
testify or produce papers during inquiries in aid of legislation. The charge of contempt by Congress shall be tried before
the courts, where the contumacious witness will be heard. More importantly, it shall indicate the exact penalty of the
offense, which may include a fine and/or imprisonment, and the period of imprisonment shall be specified therein. This
constitutes as the statutory power of contempt, which is different from the inherent power of contempt.

Congress' statutory power of contempt has been recognized in foreign jurisdictions as reflected in the cases of In re
Chapman and Jurney v. MacCracken. Similarly, in this jurisdiction, the statutory power of contempt of Congress was also
acknowledged in Lopez. It was stated therein that in cases that if Congress seeks to penalize a person cited in contempt
beyond its adjournment, it must institute a criminal proceeding against him. When his case is before the courts, the culprit
shall be afforded all the rights of the accused under the Constitution. He shall have an opportunity to defend himself before
he can be convicted and penalized by the State.

Notably, there is an existing statutory provision under Article 150 of the Revised Penal Code, which penalizes the refusal
of a witness to answer any legal inquiry before Congress, to wit:

Art. 150. Disobedience to summons issued by the National Assembly, its committees or subcommittees, by the
Constitutional Commissions, its committees, subcommittees or divisions. - The penalty of arresto mayor or a fine ranging
from two hundred to one thousand pesos, or both such fine and imprisonment shall be imposed upon any person who,
having been duly summoned to attend as a witness before the National Assembly, (Congress), its special or standing
committees and subcommittees, the Constitutional Commissions and its committees, subcommittees, or divisions, or
before any commission or committee chairman or member authorized to summon witnesses, refuses, without legal
excuse, to obey such summons, or being present before any such legislative or constitutional body or official, refuses to
be sworn or placed under affirmation or to answer any legal inquiry or to produce any books, papers, documents,
or records in his possession, when required by them to do so in the exercise of their functions. The same penalty
shall be imposed upon any person who shall restrain another from attending as a witness, or who shall induce
disobedience to a summon or refusal to be sworn by any such body or official. (emphasis and underscoring supplied)

Verily, the said law may be another recourse for the Senate to exercise its statutory power of contempt. The period of
detention provided therein is definite and is not limited by the period of the legislative inquiry. Of course, the enactment of
a new law or the amendment of the existing law to augment its power of contempt and to extend the period of
imprisonment shall be in the sole discretion of Congress.

Moreover, the apprehension in Arnault - that the Senate will be prevented from effectively conducting legislative hearings
during recess - shall be duly addressed because it is expressly provided herein that the Senate may still exercise its power
of contempt during legislative hearings while on recess provided that the period of imprisonment shall only last until the
termination of the legislative inquiry, specifically, upon the approval or disapproval of the Committee Report. Thus, the
Senate's inherent power of contempt is still potent and compelling even during its recess. At the same time, the rights of
the persons appearing are respected because their detention shall not be indefinite.

In fine, the interests of the Senate and the witnesses appearing in its legislative inquiry are balanced.  The Senate can
1âwphi1

continuously and effectively exercise its power of contempt during the legislative inquiry against recalcitrant witnesses,
even during recess. Such power can be exercised by the Senate immediately when the witness performs a contemptuous
act, subject to its own rules and the constitutional rights of the said witness.

In fine, the interests of the Senate and the witnesses appearing in its legislative inquiry are balanced. The Senate can
continuously and effectively exercise its power of contempt during the legislative inquiry against recalcitrant witnesses,
even during recess. Such power can be exercised by the Senate immediately when the witness performs a contemptuous
act, subject to its own rules and the constitutional rights of the said witness.
In addition, if the Congress decides to extend the period of imprisonment for the contempt committed by a witness beyond
the duration of the legislative inquiry, then it may file a criminal case under the existing statute or enact a new law to
increase the definite period of imprisonment.

WHEREFORE, the petition is DENIED for being moot and academic. However, the period of imprisonment under the
inherent power of contempt of the Senate during inquiries in aid of legislation should only last until the termination of the
legislative inquiry.

The December 12, 2017 Resolution of the Court ordering the temporary release of Arvin R. Balag from detention is hereby
declared FINAL.

SO ORDERED.

Passage of laws
-Imbong v Ochoa, 721 SCRA 146 (2014) [SEE PAGE 138]

Non-legislative
- Belgica v Ochoa, Jr., GR 208566, Nov 19, 2013 (Cross border transfer) [SEE PART 1]
- Araullo v Aquino III, GR 209287, Jul 1, 2014 (Augmentation Power; Impoundment)

For resolution are the consolidated petitions assailing the constitutionality of the Disbursement Acceleration
Program(DAP), National Budget Circular (NBC) No. 541, and related issuances of the Department of Budget and
Management (DBM) implementing the DAP.

At the core of the controversy is Section 29(1) of Article VI of the 1987 Constitution, a provision of the fundamental law
that firmly ordains that "[n]o money shall be paid out of the Treasury except in pursuance of an appropriation made by
law." The tenor and context of the challenges posed by the petitioners against the DAP indicate that the DAP contravened
this provision by allowing the Executive to allocate public money pooled from programmed and unprogrammed funds of its
various agencies in the guise of the President exercising his constitutional authority under Section 25(5) of the 1987
Constitution to transfer funds out of savings to augment the appropriations of offices within the Executive Branch of the
Government. But the challenges are further complicated by the interjection of allegations of transfer of funds to agencies
or offices outside of the Executive.

Antecedents

What has precipitated the controversy?

On September 25, 2013, Sen. Jinggoy Ejercito Estrada delivered a privilege speech in the Senate of the Philippines to
reveal that some Senators, including himself, had been allotted an additional ₱50 Million each as "incentive" for voting in
favor of the impeachment of Chief Justice Renato C. Corona.

Responding to Sen. Estrada’s revelation, Secretary Florencio Abad of the DBM issued a public statement entitled Abad:
Releases to Senators Part of Spending Acceleration Program,  explaining that the funds released to the Senators had
1

been part of the DAP, a program designed by the DBM to ramp up spending to accelerate economic expansion. He
clarified that the funds had been released to the Senators based on their letters of request for funding; and that it was not
the first time that releases from the DAP had been made because the DAP had already been instituted in 2011 to ramp up
spending after sluggish disbursements had caused the growth of the gross domestic product (GDP) to slow down. He
explained that the funds under the DAP were usually taken from (1) unreleased appropriations under Personnel
Services;  (2) unprogrammed funds; (3) carry-over appropriations unreleased from the previous year; and (4) budgets for
2

slow-moving items or projects that had been realigned to support faster-disbursing projects.

The DBM soon came out to claim in its website  that the DAP releases had been sourced from savings generated by the
3

Government, and from unprogrammed funds; and that the savings had been derived from (1) the pooling of unreleased
appropriations, like unreleased Personnel Services  appropriations that would lapse at the end of the year, unreleased
4

appropriations of slow-moving projects and discontinued projects per zero based budgeting findings;  and (2) the
5

withdrawal of unobligated allotments also for slow-moving programs and projects that had been earlier released to the
agencies of the National Government.

The DBM listed the following as the legal bases for the DAP’s use of savings,  namely: (1) Section 25(5), Article VI of the
6

1987 Constitution, which granted to the President the authority to augment an item for his office in the general
appropriations law; (2) Section 49 (Authority to Use Savings for Certain Purposes) and Section 38 (Suspension of
Expenditure Appropriations), Chapter 5, Book VI of Executive Order (EO) No. 292 (Administrative Code of 1987); and (3)
the General Appropriations Acts (GAAs) of 2011, 2012 and 2013, particularly their provisions on the (a) use of savings; (b)
meanings of savings and augmentation; and (c) priority in the use of savings.

As for the use of unprogrammed funds under the DAP, the DBM cited as legal bases the special provisions on
unprogrammed fund contained in the GAAs of 2011, 2012 and 2013.
The revelation of Sen. Estrada and the reactions of Sec. Abad and the DBM brought the DAP to the consciousness of the
Nation for the first time, and made this present controversy inevitable. That the issues against the DAP came at a time
when the Nation was still seething in anger over Congressional pork barrel – "an appropriation of government spending
meant for localized projects and secured solely or primarily to bring money to a representative’s district"  – excited the
7

Nation as heatedly as the pork barrel controversy.

Nine petitions assailing the constitutionality of the DAP and the issuances relating to the DAP were filed within days of
each other, as follows: G.R. No. 209135 (Syjuco), on October 7, 2013; G.R. No. 209136 (Luna), on October 7, 2013; G.R.
No. 209155 (Villegas),  on October 16, 2013; G.R. No. 209164 (PHILCONSA), on October 8, 2013; G.R. No. 209260
8

(IBP), on October 16, 2013; G.R. No. 209287 (Araullo), on October 17, 2013; G.R. No. 209442 (Belgica), on October 29,
2013; G.R. No. 209517 (COURAGE), on November6, 2013; and G.R. No. 209569 (VACC), on November 8, 2013.

In G.R. No. 209287 (Araullo), the petitioners brought to the Court’s attention NBC No. 541 (Adoption of Operational
Efficiency Measure – Withdrawal of Agencies’ Unobligated Allotments as of June 30, 2012), alleging that NBC No. 541,
which was issued to implement the DAP, directed the withdrawal of unobligated allotments as of June 30, 2012 of
government agencies and offices with low levels of obligations, both for continuing and current allotments.

In due time, the respondents filed their Consolidated Comment through the Office of the Solicitor General (OSG).

The Court directed the holding of oral arguments on the significant issues raised and joined.

Issues

Under the Advisory issued on November 14, 2013, the presentations of the parties during the oral arguments were limited
to the following, to wit:

Procedural Issue:

A. Whether or not certiorari, prohibition, and mandamus are proper remedies to assail the constitutionality and validity of
the Disbursement Acceleration Program (DAP), National Budget Circular (NBC) No. 541, and all other executive
issuances allegedly implementing the DAP. Subsumed in this issue are whether there is a controversy ripe for judicial
determination, and the standing of petitioners.

Substantive Issues:

B. Whether or not the DAP violates Sec. 29, Art. VI of the 1987 Constitution, which provides: "No money shall be paid out
of the Treasury except in pursuance of an appropriation made by law."

C. Whether or not the DAP, NBC No. 541, and all other executive issuances allegedly implementing the DAP violate Sec.
25(5), Art. VI of the 1987 Constitution insofar as:

(a)They treat the unreleased appropriations and unobligated allotments withdrawn from government agencies as
"savings" as the term is used in Sec. 25(5), in relation to the provisions of the GAAs of 2011, 2012 and 2013;

(b)They authorize the disbursement of funds for projects or programs not provided in the GAAs for the Executive
Department; and

(c)They "augment" discretionary lump sum appropriations in the GAAs.

D. Whether or not the DAP violates: (1) the Equal Protection Clause, (2) the system of checks and balances, and (3) the
principle of public accountability enshrined in the 1987 Constitution considering that it authorizes the release of funds upon
the request of legislators.

E. Whether or not factual and legal justification exists to issue a temporary restraining order to restrain the implementation
of the DAP, NBC No. 541, and all other executive issuances allegedly implementing the DAP.

In its Consolidated Comment, the OSG raised the matter of unprogrammed funds in order to support its argument
regarding the President’s power to spend. During the oral arguments, the propriety of releasing unprogrammed funds to
support projects under the DAP was considerably discussed. The petitioners in G.R. No. 209287 (Araullo) and G.R. No.
209442 (Belgica) dwelled on unprogrammed funds in their respective memoranda. Hence, an additional issue for the oral
arguments is stated as follows:

F. Whether or not the release of unprogrammed funds under the DAP was in accord with the GAAs.

During the oral arguments held on November 19, 2013, the Court directed Sec. Abad to submit a list of savings brought
under the DAP that had been sourced from (a) completed programs; (b) discontinued or abandoned programs; (c) unpaid
appropriations for compensation; (d) a certified copy of the President’s directive dated June 27, 2012 referred to in NBC
No. 541; and (e) all circulars or orders issued in relation to the DAP.
9

In compliance, the OSG submitted several documents, as follows:


(1) A certified copy of the Memorandum for the President dated June 25, 2012 (Omnibus Authority to Consolidate
Savings/Unutilized Balances and their Realignment); 10

(2) Circulars and orders, which the respondents identified as related to the DAP, namely:

a. NBC No. 528 dated January 3, 2011 (Guidelines on the Release of Funds for FY 2011);

b. NBC No. 535 dated December 29, 2011 (Guidelines on the Release of Funds for FY 2012);

c. NBC No. 541 dated July 18, 2012 (Adoption of Operational Efficiency Measure – Withdrawal of
Agencies’ Unobligated Allotments as of June 30, 2012);

d. NBC No. 545 dated January 2, 2013 (Guidelines on the Release of Funds for FY 2013);

e. DBM Circular Letter No. 2004-2 dated January 26, 2004 (Budgetary Treatment of
Commitments/Obligations of the National Government);

f. COA-DBM Joint Circular No. 2013-1 dated March 15, 2013 (Revised Guidelines on the Submission of
Quarterly Accountability Reports on Appropriations, Allotments, Obligations and Disbursements);

g. NBC No. 440 dated January 30, 1995 (Adoption of a Simplified Fund Release System in the
Government).

(3) A breakdown of the sources of savings, including savings from discontinued projects and unpaid appropriations
for compensation from 2011 to 2013

On January 28, 2014, the OSG, to comply with the Resolution issued on January 21, 2014 directing the respondents to
submit the documents not yet submitted in compliance with the directives of the Court or its Members, submitted several
evidence packets to aid the Court in understanding the factual bases of the DAP, to wit:

(1) First Evidence Packet  – containing seven memoranda issued by the DBM through Sec. Abad, inclusive of
11

annexes, listing in detail the 116 DAP identified projects approved and duly signed by the President, as follows:

a. Memorandum for the President dated October 12, 2011 (FY 2011 Proposed Disbursement Acceleration
Program (Projects and Sources of Funds);

b. Memorandum for the President dated December 12, 2011 (Omnibus Authority to Consolidate
Savings/Unutilized Balances and its Realignment);

c. Memorandum for the President dated June 25, 2012 (Omnibus Authority to Consolidate
Savings/Unutilized Balances and their Realignment);

d. Memorandum for the President dated September 4, 2012 (Release of funds for other priority projects
and expenditures of the Government);

e. Memorandum for the President dated December 19, 2012 (Proposed Priority Projects and Expenditures
of the Government);

f. Memorandum for the President dated May 20, 2013 (Omnibus Authority to Consolidate
Savings/Unutilized Balances and their Realignment to Fund the Quarterly Disbursement Acceleration
Program); and

g. Memorandum for the President dated September 25, 2013 (Funding for the Task Force Pablo
Rehabilitation Plan).

(2) Second Evidence Packet  – consisting of 15 applications of the DAP, with their corresponding Special
12

Allotment Release Orders (SAROs) and appropriation covers;

(3) Third Evidence Packet  – containing a list and descriptions of 12 projects under the DAP;
13

(4) Fourth Evidence Packet  – identifying the DAP-related portions of the Annual Financial Report (AFR) of the
14

Commission on Audit for 2011 and 2012;

(5) Fifth Evidence Packet  – containing a letter of Department of Transportation and Communications(DOTC) Sec.
15

Joseph Abaya addressed to Sec. Abad recommending the withdrawal of funds from his agency, inclusive of
annexes; and

(6) Sixth Evidence Packet  – a print-out of the Solicitor General’s visual presentation for the January 28, 2014 oral
16

arguments.
On February 5, 2014,  the OSG forwarded the Seventh Evidence Packet,  which listed the sources of funds brought under
17 18

the DAP, the uses of such funds per project or activity pursuant to DAP, and the legal bases thereof.

On February 14, 2014, the OSG submitted another set of documents in further compliance with the Resolution dated
January 28, 2014, viz:

(1) Certified copies of the certifications issued by the Bureau of Treasury to the effect that the revenue collections
exceeded the original revenue targets for the years 2011, 2012 and 2013, including collections arising from sources not
considered in the original revenue targets, which certifications were required for the release of the unprogrammed funds
as provided in Special Provision No. 1 of Article XLV, Article XVI, and Article XLV of the 2011, 2012 and 2013 GAAs; and
(2) A report on releases of savings of the Executive Department for the use of the Constitutional Commissions and other
branches of the Government, as well as the fund releases to the Senate and the Commission on Elections (COMELEC).

RULING

I.

Procedural Issue:

a) The petitions under Rule 65 are proper remedies

All the petitions are filed under Rule 65 of the Rules of Court, and include applications for the issuance of writs of
preliminary prohibitory injunction or temporary restraining orders. More specifically, the nature of the petitions is
individually set forth hereunder, to wit:

G.R. No. 209135 (Syjuco) Certiorari, Prohibition and Mandamus


G.R. No. 209136 (Luna) Certiorariand Prohibition
G.R. No. 209155 (Villegas) Certiorariand Prohibition
G.R. No. 209164 (PHILCONSA) Certiorariand Prohibition
G.R. No. 209260 (IBP) Prohibition
G.R. No. 209287 (Araullo) Certiorariand Prohibition
G.R. No. 209442 (Belgica) Certiorari
G.R. No. 209517 (COURAGE) Certiorari and Prohibition
G.R. No. 209569 (VACC) Certiorari and Prohibition

The respondents submit that there is no actual controversy that is ripe for adjudication in the absence of adverse claims
between the parties;  that the petitioners lacked legal standing to sue because no allegations were made to the effect that
19

they had suffered any injury as a result of the adoption of the DAP and issuance of NBC No. 541; that their being
taxpayers did not immediately confer upon the petitioners the legal standing to sue considering that the adoption and
implementation of the DAP and the issuance of NBC No. 541 were not in the exercise of the taxing or spending power of
Congress;  and that even if the petitioners had suffered injury, there were plain, speedy and adequate remedies in the
20

ordinary course of law available to them, like assailing the regularity of the DAP and related issuances before the
Commission on Audit (COA) or in the trial courts. 21

The respondents aver that the special civil actions of certiorari and prohibition are not proper actions for directly assailing
the constitutionality and validity of the DAP, NBC No. 541, and the other executive issuances implementing the DAP. 22

In their memorandum, the respondents further contend that there is no authorized proceeding under the Constitution and
the Rules of Court for questioning the validity of any law unless there is an actual case or controversy the resolution of
which requires the determination of the constitutional question; that the jurisdiction of the Court is largely appellate; that for
a court of law to pass upon the constitutionality of a law or any act of the Government when there is no case or
controversy is for that court to set itself up as a reviewer of the acts of Congress and of the President in violation of the
principle of separation of powers; and that, in the absence of a pending case or controversy involving the DAP and NBC
No. 541, any decision herein could amount to a mere advisory opinion that no court can validly render. 23

The respondents argue that it is the application of the DAP to actual situations that the petitioners can question either in
the trial courts or in the COA; that if the petitioners are dissatisfied with the ruling either of the trial courts or of the COA,
they can appeal the decision of the trial courts by petition for review on certiorari, or assail the decision or final order of the
COA by special civil action for certiorari under Rule 64 of the Rules of Court. 24

The respondents’ arguments and submissions on the procedural issue are bereft of merit.

Section 1, Article VIII of the 1987 Constitution expressly provides:


Section 1. The judicial power shall be vested in one Supreme Court and in such lower courts as may be established by
law.

Judicial power includes the duty of the courts of justice to settle actual controversies involving rights which are legally
demandable and enforceable, and to determine whether or not there has been a grave abuse of discretion amounting to
lack or excess of jurisdiction on the part of any branch or instrumentality of the Government.

Thus, the Constitution vests judicial power in the Court and in such lower courts as may be established by law. In creating
a lower court, Congress concomitantly determines the jurisdiction of that court, and that court, upon its creation, becomes
by operation of the Constitution one of the repositories of judicial power.  However, only the Court is a constitutionally
25

created court, the rest being created by Congress in its exercise of the legislative power.

The Constitution states that judicial power includes the duty of the courts of justice not only "to settle actual controversies
involving rights which are legally demandable and enforceable" but also "to determine whether or not there has been a
grave abuse of discretion amounting to lack or excess of jurisdiction on the part of any branch or instrumentality of the
Government." It has thereby expanded the concept of judicial power, which up to then was confined to its traditional ambit
of settling actual controversies involving rights that were legally demandable and enforceable.

The background and rationale of the expansion of judicial power under the 1987 Constitution were laid out during the
deliberations of the 1986 Constitutional Commission by Commissioner Roberto R. Concepcion (a former Chief Justice of
the Philippines) in his sponsorship of the proposed provisions on the Judiciary, where he said:–

The Supreme Court, like all other courts, has one main function: to settle actual controversies involving conflicts of rights
which are demandable and enforceable. There are rights which are guaranteed by law but cannot be enforced by a judicial
party. In a decided case, a husband complained that his wife was unwilling to perform her duties as a wife. The Court said:
"We can tell your wife what her duties as such are and that she is bound to comply with them, but we cannot force her
physically to discharge her main marital duty to her husband. There are some rights guaranteed by law, but they are so
personal that to enforce them by actual compulsion would be highly derogatory to human dignity." This is why the first part
of the second paragraph of Section 1 provides that: Judicial power includes the duty of courts to settle actual controversies
involving rights which are legally demandable or enforceable…

The courts, therefore, cannot entertain, much less decide, hypothetical questions. In a presidential system of government,
the Supreme Court has, also, another important function. The powers of government are generally considered divided into
three branches: the Legislative, the Executive and the Judiciary. Each one is supreme within its own sphere and
independent of the others. Because of that supremacy power to determine whether a given law is valid or not is vested in
courts of justice.

Briefly stated, courts of justice determine the limits of power of the agencies and offices of the government as well as
those of its officers. In other words, the judiciary is the final arbiter on the question whether or not a branch of government
or any of its officials has acted without jurisdiction or in excess of jurisdiction, or so capriciously as to constitute an abuse
of discretion amounting to excess of jurisdiction or lack of jurisdiction. This is not only a judicial power but a duty to pass
judgmenton matters of this nature.

This is the background of paragraph 2 of Section 1, which means that the courts cannot hereafter evade the duty to settle
matters of this nature, by claiming that such matters constitute a political question. (Bold emphasis supplied) 26

Upon interpellation by Commissioner Nolledo, Commissioner Concepcion clarified the scope of judicial power in the
following manner:–

MR. NOLLEDO. x x x

The second paragraph of Section 1 states: "Judicial power includes the duty of courts of justice to settle actual
controversies…" The term "actual controversies" according to the Commissioner should refer to questions which are
political in nature and, therefore, the courts should not refuse to decide those political questions. But do I understand it
right that this is restrictive or only an example? I know there are cases which are not actual yet the court can assume
jurisdiction. An example is the petition for declaratory relief.

May I ask the Commissioner’s opinion about that?

MR. CONCEPCION. The Supreme Court has no jurisdiction to grant declaratory judgments.

MR. NOLLEDO. The Gentleman used the term "judicial power" but judicial power is not vested in the Supreme Court
alone but also in other lower courts as may be created by law.

MR. CONCEPCION. Yes.

MR. NOLLEDO. And so, is this only an example?

MR. CONCEPCION. No, I know this is not. The Gentleman seems to identify political questions with jurisdictional
questions. But there is a difference.
MR. NOLLEDO. Because of the expression "judicial power"?

MR. CONCEPCION. No. Judicial power, as I said, refers to ordinary cases but where there is a question as to whether the
government had authority or had abused its authority to the extent of lacking jurisdiction or excess of jurisdiction, that is
not a political question. Therefore, the court has the duty to decide. 27

Our previous Constitutions equally recognized the extent of the power of judicial review and the great responsibility of the
Judiciary in maintaining the allocation of powers among the three great branches of Government. Speaking for the Court
in Angara v. Electoral Commission,  Justice Jose P. Laurel intoned:
28

x x x In times of social disquietude or political excitement, the great landmarks of the Constitution are apt to be forgotten or
marred, if not entirely obliterated. In cases of conflict, the judicial department is the only constitutional organ which can be
called upon to determine the proper allocation of powers between the several department and among the integral or
constituent units thereof.

xxxx

The Constitution is a definition of the powers of government. Who is to determine the nature, scope and extent of such
powers? The Constitution itself has provided for the instrumentality of the judiciary as the rational way. And when the
judiciary mediates to allocate constitutional boundaries, it does not assert any superiority over the other department; it
does not in reality nullify or invalidate an act of the legislature, but only asserts the solemn and sacred obligation assigned
to it by the Constitution to determine conflicting claims of authority under the Constitution and to establish for the parties in
an actual controversy the rights which that instrument secures and guarantees to them. This is in truth all that is involved
in what is termed "judicial supremacy" which properly is the power of judicial review under the Constitution. x x x 29

What are the remedies by which the grave abuse of discretion amounting to lack or excess of jurisdiction on the part of
any branch or instrumentality of the Government may be determined under the Constitution?

The present Rules of Court uses two special civil actions for determining and correcting grave abuse of discretion
amounting to lack or excess of jurisdiction. These are the special civil actions for certiorari and prohibition, and both are
governed by Rule 65. A similar remedy of certiorari exists under Rule 64, but the remedy is expressly applicable only to
the judgments and final orders or resolutions of the Commission on Elections and the Commission on Audit.

The ordinary nature and function of the writ of certiorari in our present system are aptly explained in Delos Santos v.
Metropolitan Bank and Trust Company: 30

In the common law, from which the remedy of certiorari evolved, the writ of certiorari was issued out of Chancery, or the
King’s Bench, commanding agents or officers of the inferior courts to return the record of a cause pending before them, so
as to give the party more sure and speedy justice, for the writ would enable the superior court to determine from an
inspection of the record whether the inferior court’s judgment was rendered without authority. The errors were of such a
nature that, if allowed to stand, they would result in a substantial injury to the petitioner to whom no other remedy was
available. If the inferior court acted without authority, the record was then revised and corrected in matters of law. The writ
of certiorari was limited to cases in which the inferior court was said to be exceeding its jurisdiction or was not proceeding
according to essential requirements of law and would lie only to review judicial or quasi-judicial acts.

The concept of the remedy of certiorari in our judicial system remains much the same as it has been in the common law.
In this jurisdiction, however, the exercise of the power to issue the writ of certiorari is largely regulated by laying down the
instances or situations in the Rules of Court in which a superior court may issue the writ of certiorari to an inferior court or
officer. Section 1, Rule 65 of the Rules of Court compellingly provides the requirements for that purpose, viz:

xxxx

The sole office of the writ of certiorari is the correction of errors of jurisdiction, which includes the commission of grave
abuse of discretion amounting to lack of jurisdiction. In this regard, mere abuse of discretion is not enough to warrant the
issuance of the writ. The abuse of discretion must be grave, which means either that the judicial or quasi-judicial power
was exercised in an arbitrary or despotic manner by reason of passion or personal hostility, or that the respondent judge,
tribunal or board evaded a positive duty, or virtually refused to perform the duty enjoined or to act in contemplation of law,
such as when such judge, tribunal or board exercising judicial or quasi-judicial powers acted in a capricious or whimsical
manner as to be equivalent to lack of jurisdiction.31

Although similar to prohibition in that it will lie for want or excess of jurisdiction, certiorari is to be distinguished from
prohibition by the fact that it is a corrective remedy used for the re-examination of some action of an inferior tribunal, and is
directed to the cause or proceeding in the lower court and not to the court itself, while prohibition is a preventative remedy
issuing to restrain future action, and is directed to the court itself.  The Court expounded on the nature and function of the
32

writ of prohibition in Holy Spirit Homeowners Association, Inc. v. Defensor: 33

A petition for prohibition is also not the proper remedy to assail an IRR issued in the exercise of a quasi-legislative
function. Prohibition is an extraordinary writ directed against any tribunal, corporation, board, officer or person, whether
exercising judicial, quasi-judicial or ministerial functions, ordering said entity or person to desist from further proceedings
when said proceedings are without or in excess of said entity’s or person’s jurisdiction, or are accompanied with grave
abuse of discretion, and there is no appeal or any other plain, speedy and adequate remedy in the ordinary course of law.
Prohibition lies against judicial or ministerial functions, but not against legislative or quasi-legislative functions. Generally,
the purpose of a writ of prohibition is to keep a lower court within the limits of its jurisdiction in order to maintain the
administration of justice in orderly channels. Prohibition is the proper remedy to afford relief against usurpation of
jurisdiction or power by an inferior court, or when, in the exercise of jurisdiction in handling matters clearly within its
cognizance the inferior court transgresses the bounds prescribed to it by the law, or where there is no adequate remedy
available in the ordinary course of law by which such relief can be obtained. Where the principal relief sought is to
invalidate an IRR, petitioners’ remedy is an ordinary action for its nullification, an action which properly falls under the
jurisdiction of the Regional Trial Court. In any case, petitioners’ allegation that "respondents are performing or threatening
to perform functions without or in excess of their jurisdiction" may appropriately be enjoined by the trial court through a writ
of injunction or a temporary restraining order.

With respect to the Court, however, the remedies of certiorari and prohibition are necessarily broader in scope and reach,
and the writ of certiorari or prohibition may be issued to correct errors of jurisdiction committed not only by a tribunal,
corporation, board or officer exercising judicial, quasi-judicial or ministerial functions but also to set right, undo and restrain
any act of grave abuse of discretion amounting to lack or excess of jurisdiction by any branch or instrumentality of the
Government, even if the latter does not exercise judicial, quasi-judicial or ministerial functions. This application is
expressly authorized by the text of the second paragraph of Section 1, supra.

Thus, petitions for certiorari and prohibition are appropriate remedies to raise constitutional issues and to review and/or
prohibit or nullify the acts of legislative and executive officials.
34

Necessarily, in discharging its duty under Section 1, supra, to set right and undo any act of grave abuse of discretion
amounting to lack or excess of jurisdiction by any branch or instrumentality of the Government, the Court is not at all
precluded from making the inquiry provided the challenge was properly brought by interested or affected parties. The
Court has been thereby entrusted expressly or by necessary implication with both the duty and the obligation of
determining, in appropriate cases, the validity of any assailed legislative or executive action. This entrustment is consistent
with the republican system of checks and balances. 35

Following our recent dispositions concerning the congressional pork barrel, the Court has become more alert to discharge
its constitutional duty. We will not now refrain from exercising our expanded judicial power in order to review and
determine, with authority, the limitations on the Chief Executive’s spending power.

b) Requisites for the exercise of the


power of judicial review were
complied with

The requisites for the exercise of the power of judicial review are the following, namely: (1) there must bean actual case or
justiciable controversy before the Court; (2) the question before the Court must be ripe for adjudication; (3) the person
challenging the act must be a proper party; and (4) the issue of constitutionality must be raised at the earliest opportunity
and must be the very litis mota of the case. 36

The first requisite demands that there be an actual case calling for the exercise of judicial power by the Court.  An actual
37

case or controversy, in the words of Belgica v. Executive Secretary Ochoa: 38

x x x is one which involves a conflict of legal rights, an assertion of opposite legal claims, susceptible of judicial resolution
as distinguished from a hypothetical or abstract difference or dispute. In other words, "[t]here must be a contrariety of legal
rights that can be interpreted and enforced on the basis of existing law and jurisprudence." Related to the requirement of
an actual case or controversy is the requirement of "ripeness," meaning that the questions raised for constitutional scrutiny
are already ripe for adjudication. "A question is ripe for adjudication when the act being challenged has had a direct
adverse effect on the individual challenging it. It is a prerequisite that something had then been accomplished or
performed by either branch before a court may come into the picture, and the petitioner must allege the existence of an
immediate or threatened injury to itself as a result of the challenged action." "Withal, courts will decline to pass upon
constitutional issues through advisory opinions, bereft as they are of authority to resolve hypothetical or moot questions."

An actual and justiciable controversy exists in these consolidated cases. The incompatibility of the perspectives of the
parties on the constitutionality of the DAP and its relevant issuances satisfy the requirement for a conflict between legal
rights. The issues being raised herein meet the requisite ripeness considering that the challenged executive acts were
already being implemented by the DBM, and there are averments by the petitioners that such implementation was
repugnant to the letter and spirit of the Constitution. Moreover, the implementation of the DAP entailed the allocation and
expenditure of huge sums of public funds. The fact that public funds have been allocated, disbursed or utilized by reason
or on account of such challenged executive acts gave rise, therefore, to an actual controversy that is ripe for adjudication
by the Court.

It is true that Sec. Abad manifested during the January 28, 2014 oral arguments that the DAP as a program had been
meanwhile discontinued because it had fully served its purpose, saying: "In conclusion, Your Honors, may I inform the
Court that because the DAP has already fully served its purpose, the Administration’s economic managers have
recommended its termination to the President. x x x." 39

The Solicitor General then quickly confirmed the termination of the DAP as a program, and urged that its termination had
already mooted the challenges to the DAP’s constitutionality, viz:
DAP as a program, no longer exists, thereby mooting these present cases brought to challenge its constitutionality. Any
constitutional challenge should no longer be at the level of the program, which is now extinct, but at the level of its prior
applications or the specific disbursements under the now defunct policy. We challenge the petitioners to pick and choose
which among the 116 DAP projects they wish to nullify, the full details we will have provided by February 5. We urge this
Court to be cautious in limiting the constitutional authority of the President and the Legislature to respond to the dynamic
needs of the country and the evolving demands of governance, lest we end up straight jacketing our elected
representatives in ways not consistent with our constitutional structure and democratic principles. 40

A moot and academic case is one that ceases to present a justiciable controversy by virtue of supervening events, so that
a declaration thereon would be of no practical use or value. 41

The Court cannot agree that the termination of the DAP as a program was a supervening event that effectively mooted
these consolidated cases. Verily, the Court had in the past exercised its power of judicial review despite the cases being
rendered moot and academic by supervening events, like: (1) when there was a grave violation of the Constitution; (2)
when the case involved a situation of exceptional character and was of paramount public interest; (3) when the
constitutional issue raised required the formulation of controlling principles to guide the Bench, the Bar and the public; and
(4) when the case was capable of repetition yet evading review. 42

Assuming that the petitioners’ several submissions against the DAP were ultimately sustained by the Court here, these
cases would definitely come under all the exceptions. Hence, the Court should not abstain from exercising its power of
judicial review.

Did the petitioners have the legal standing to sue?

Legal standing, as a requisite for the exercise of judicial review, refers to "a right of appearance in a court of justice on a
given question."  The concept of legal standing, or locus standi, was particularly discussed in De Castro v. Judicial and
43

Bar Council,  where the Court said:


44

In public or constitutional litigations, the Court is often burdened with the determination of the locus standi of the
petitioners due to the ever-present need to regulate the invocation of the intervention of the Court to correct any official
action or policy in order to avoid obstructing the efficient functioning of public officials and offices involved in public service.
It is required, therefore, that the petitioner must have a personal stake in the outcome of the controversy, for, as indicated
in Agan, Jr. v. Philippine International Air Terminals Co., Inc.:

The question on legal standing is whether such parties have "alleged such a personal stake in the outcome of the
controversy as to assure that concrete adverseness which sharpens the presentation of issues upon which the court so
largely depends for illumination of difficult constitutional questions." Accordingly, it has been held that the interest of a
person assailing the constitutionality of a statute must be direct and personal. He must be able to show, not only that the
law or any government act is invalid, but also that he sustained or is in imminent danger of sustaining some direct injury as
a result of its enforcement, and not merely that he suffers thereby in some indefinite way. It must appear that the person
complaining has been or is about to be denied some right or privilege to which he is lawfully entitled or that he is about to
be subjected to some burdens or penalties by reason of the statute or act complained of.

It is true that as early as in 1937, in People v. Vera, the Court adopted the direct injury test for determining whether a
petitioner in a public action had locus standi. There, the Court held that the person who would assail the validity of a
statute must have "a personal and substantial interest in the case such that he has sustained, or will sustain direct injury
as a result." Vera was followed in Custodio v. President of the Senate, Manila Race Horse Trainers’ Association v. De la
Fuente, Anti-Chinese League of the Philippines v. Felix, and Pascual v. Secretary of Public Works.

Yet, the Court has also held that the requirement of locus standi, being a mere procedural technicality, can be waived by
the Court in the exercise of its discretion. For instance, in 1949, in Araneta v. Dinglasan, the Court liberalized the approach
when the cases had "transcendental importance." Some notable controversies whose petitioners did not pass the direct
injury test were allowed to be treated in the same way as in Araneta v. Dinglasan.

In the 1975 decision in Aquino v. Commission on Elections, this Court decided to resolve the issues raised by the petition
due to their "far reaching implications," even if the petitioner had no personality to file the suit. The liberal approach of
Aquino v. Commission on Elections has been adopted in several notable cases, permitting ordinary citizens, legislators,
and civic organizations to bring their suits involving the constitutionality or validity of laws, regulations, and rulings.

However, the assertion of a public right as a predicate for challenging a supposedly illegal or unconstitutional executive or
legislative action rests on the theory that the petitioner represents the public in general. Although such petitioner may not
be as adversely affected by the action complained against as are others, it is enough that he sufficiently demonstrates in
his petition that he is entitled to protection or relief from the Court in the vindication of a public right.

Quite often, as here, the petitioner in a public action sues as a citizen or taxpayer to gain locus standi. That is not
surprising, for even if the issue may appear to concern only the public in general, such capacities nonetheless equip the
petitioner with adequate interest to sue. In David v. Macapagal-Arroyo, the Court aptly explains why:

Case law in most jurisdiction snow allows both "citizen" and "taxpayer" standing in public actions. The distinction was first
laid down in Beauchamp v. Silk, where it was held that the plaintiff in a taxpayer’s suit is in a different category from the
plaintiff in a citizen’s suit. In the former, the plaintiff is affected by the expenditure of public funds, while in the latter, he is
but the mere instrument of the public concern. As held by the New York Supreme Court in People ex rel Case v. Collins:
"In matter of mere public right, however…the people are the real parties…It is at least the right, if not the duty, of every
citizen to interfere and see that a public offence be properly pursued and punished, and that a public grievance be
remedied." With respect to taxpayer’s suits, Terr v. Jordan held that "the right of a citizen and a taxpayer to maintain an
action in courts to restrain the unlawful use of public funds to his injury cannot be denied."
45

The Court has cogently observed in Agan, Jr. v. Philippine International Air Terminals Co., Inc.  that "[s]tanding is a
46

peculiar concept in constitutional law because in some cases, suits are not brought by parties who have been personally
injured by the operation of a law or any other government act but by concerned citizens, taxpayers or voters who actually
sue in the public interest."

Except for PHILCONSA, a petitioner in G.R. No. 209164, the petitioners have invoked their capacities as taxpayers who,
by averring that the issuance and implementation of the DAP and its relevant issuances involved the illegal disbursements
of public funds, have an interest in preventing the further dissipation of public funds. The petitioners in G.R. No. 209287
(Araullo) and G.R. No. 209442 (Belgica) also assert their right as citizens to sue for the enforcement and observance of
the constitutional limitations on the political branches of the Government. 47

On its part, PHILCONSA simply reminds that the Court has long recognized its legal standing to bring cases upon
constitutional issues.  Luna, the petitioner in G.R. No. 209136, cites his additional capacity as a lawyer. The IBP, the
48

petitioner in G.R. No. 209260, stands by "its avowed duty to work for the rule of law and of paramount importance of the
question in this action, not to mention its civic duty as the official association of all lawyers in this country."
49

Under their respective circumstances, each of the petitioners has established sufficient interest in the outcome of the
controversy as to confer locus standi on each of them.

In addition, considering that the issues center on the extent of the power of the Chief Executive to disburse and allocate
public funds, whether appropriated by Congress or not, these cases pose issues that are of transcendental importance to
the entire Nation, the petitioners included. As such, the determination of such important issues call for the Court’s exercise
of its broad and wise discretion "to waive the requirement and so remove the impediment to its addressing and resolving
the serious constitutional questions raised."50

II.
Substantive Issues

1.
Overview of the Budget System

An understanding of the Budget System of the Philippines will aid the Court in properly appreciating and justly resolving
the substantive issues.

a) Origin of the Budget System

The term "budget" originated from the Middle English word bouget that had derived from the Latin word bulga (which
means bag or purse). 51

In the Philippine setting, Commonwealth Act (CA) No. 246 (Budget Act) defined "budget" as the financial program of the
National Government for a designated fiscal year, consisting of the statements of estimated receipts and expenditures for
the fiscal year for which it was intended to be effective based on the results of operations during the preceding fiscal
years. The term was given a different meaning under Republic Act No. 992 (Revised Budget Act) by describing the budget
as the delineation of the services and products, or benefits that would accrue to the public together with the estimated unit
cost of each type of service, product or benefit.  For a forthright definition, budget should simply be identified as the
52

financial plan of the Government,  or "the master plan of government."


53 54

The concept of budgeting has not been the product of recent economies. In reality, financing public goals and activities
was an idea that existed from the creation of the State.  To protect the people, the territory and sovereignty of the State,
55

its government must perform vital functions that required public expenditures. At the beginning, enormous public
expenditures were spent for war activities, preservation of peace and order, security, administration of justice, religion, and
supply of limited goods and services.  In order to finance those expenditures, the State raised revenues through taxes and
56

impositions.  Thus, budgeting became necessary to allocate public revenues for specific government functions.  The
57 58

State’s budgeting mechanism eventually developed through the years with the growing functions of its government and
changes in its market economy.

The Philippine Budget System has been greatly influenced by western public financial institutions. This is because of the
country’s past as a colony successively of Spain and the United States for a long period of time. Many aspects of the
country’s public fiscal administration, including its Budget System, have been naturally patterned after the practices and
experiences of the western public financial institutions. At any rate, the Philippine Budget System is presently guided by
two principal objectives that are vital to the development of a progressive democratic government, namely: (1) to carry on
all government activities under a comprehensive fiscal plan developed, authorized and executed in accordance with the
Constitution, prevailing statutes and the principles of sound public management; and (2) to provide for the periodic review
and disclosure of the budgetary status of the Government in such detail so that persons entrusted by law with the
responsibility as well as the enlightened citizenry can determine the adequacy of the budget actions taken, authorized or
proposed, as well as the true financial position of the Government. 59

b) Evolution of the Philippine Budget System

The budget process in the Philippines evolved from the early years of the American Regime up to the passage of the
Jones Law in 1916. A Budget Office was created within the Department of Finance by the Jones Law to discharge the
budgeting function, and was given the responsibility to assist in the preparation of an executive budget for submission to
the Philippine Legislature. 60

As early as under the 1935 Constitution, a budget policy and a budget procedure were established, and subsequently
strengthened through the enactment of laws and executive acts.  EO No. 25, issued by President Manuel L. Quezon on
61

April 25, 1936, created the Budget Commission to serve as the agency that carried out the President’s responsibility of
preparing the budget.  CA No. 246, the first budget law, went into effect on January 1, 1938 and established the Philippine
62

budget process. The law also provided a line-item budget as the framework of the Government’s budgeting system,  with63

emphasis on the observance of a "balanced budget" to tie up proposed expenditures with existing revenues.

CA No. 246 governed the budget process until the passage on June 4, 1954 of Republic Act (RA) No. 992,whereby
Congress introduced performance-budgeting to give importance to functions, projects and activities in terms of expected
results.  RA No. 992 also enhanced the role of the Budget Commission as the fiscal arm of the Government.
64 65

The 1973 Constitution and various presidential decrees directed a series of budgetary reforms that culminated in the
enactment of PD No. 1177 that President Marcos issued on July30, 1977, and of PD No. 1405, issued on June 11, 1978.
The latter decree converted the Budget Commission into the Ministry of Budget, and gave its head the rank of a Cabinet
member.

The Ministry of Budget was later renamed the Office of Budget and Management (OBM) under EO No. 711. The OBM
became the DBM pursuant to EO No. 292 effective on November 24, 1989.

c) The Philippine Budget Cycle 66

Four phases comprise the Philippine budget process, specifically: (1) Budget Preparation; (2) Budget Legislation; (3)
Budget Execution; and (4) Accountability. Each phase is distinctly separate from the others but they overlap in the
implementation of the budget during the budget year.

c.1.Budget Preparation 67

The budget preparation phase is commenced through the issuance of a Budget Call by the DBM. The Budget Call
contains budget parameters earlier set by the Development Budget Coordination Committee (DBCC) as well as policy
guidelines and procedures to aid government agencies in the preparation and submission of their budget proposals. The
Budget Call is of two kinds, namely: (1) a National Budget Call, which is addressed to all agencies, including state
universities and colleges; and (2) a Corporate Budget Call, which is addressed to all government-owned and -controlled
corporations (GOCCs) and government financial institutions (GFIs).

Following the issuance of the Budget Call, the various departments and agencies submit their respective Agency Budget
Proposals to the DBM. To boost citizen participation, the current administration has tasked the various departments and
agencies to partner with civil society organizations and other citizen-stakeholders in the preparation of the Agency Budget
Proposals, which proposals are then presented before a technical panel of the DBM in scheduled budget hearings wherein
the various departments and agencies are given the opportunity to defend their budget proposals. DBM bureaus thereafter
review the Agency Budget Proposals and come up with recommendations for the Executive Review Board, comprised by
the DBM Secretary and the DBM’s senior officials. The discussions of the Executive Review Board cover the prioritization
of programs and their corresponding support vis-à-vis the priority agenda of the National Government, and their
implementation.

The DBM next consolidates the recommended agency budgets into the National Expenditure Program (NEP)and a Budget
of Expenditures and Sources of Financing (BESF). The NEP provides the details of spending for each department and
agency by program, activity or project (PAP), and is submitted in the form of a proposed GAA. The Details of Selected
Programs and Projects is the more detailed disaggregation of key PAPs in the NEP, especially those in line with the
National Government’s development plan. The Staffing Summary provides the staffing complement of each department
and agency, including the number of positions and amounts allocated.

The NEP and BESF are thereafter presented by the DBM and the DBCC to the President and the Cabinet for further
refinements or reprioritization. Once the NEP and the BESF are approved by the President and the Cabinet, the DBM
prepares the budget documents for submission to Congress. The budget documents consist of: (1) the President’s Budget
Message, through which the President explains the policy framework and budget priorities; (2) the BESF, mandated by
Section 22, Article VII of the Constitution,  which contains the macroeconomic assumptions, public sector context,
68

breakdown of the expenditures and funding sources for the fiscal year and the two previous years; and (3) the NEP.

Public or government expenditures are generally classified into two categories, specifically: (1) capital expenditures or
outlays; and (2) current operating expenditures. Capital expenditures are the expenses whose usefulness lasts for more
than one year, and which add to the assets of the Government, including investments in the capital of government-owned
or controlled corporations and their subsidiaries.  Current operating expenditures are the purchases of goods and services
69

in current consumption the benefit of which does not extend beyond the fiscal year.  The two components of current
70

expenditures are those for personal services (PS), and those for maintenance and other operating expenses(MOOE).

Public expenditures are also broadly grouped according to their functions into: (1) economic development expenditures
(i.e., expenditures on agriculture and natural resources, transportation and communications, commerce and industry, and
other economic development efforts);  (2) social services or social development expenditures (i.e., government outlay on
71

education, public health and medicare, labor and welfare and others);  (3) general government or general public services
72

expenditures (i.e., expenditures for the general government, legislative services, the administration of justice, and for
pensions and gratuities);  (4) national defense expenditures (i.e., sub-divided into national security expenditures and
73

expenditures for the maintenance of peace and order);  and (5) public debt.
74 75

Public expenditures may further be classified according to the nature of funds, i.e., general fund, special fund or bond
fund.76

On the other hand, public revenues complement public expenditures and cover all income or receipts of the government
treasury used to support government expenditures. 77

Classical economist Adam Smith categorized public revenues based on two principal sources, stating: "The revenue which
must defray…the necessary expenses of government may be drawn either, first from some fund which peculiarly belongs
to the sovereign or commonwealth, and which is independent of the revenue of the people, or, secondly, from the revenue
of the people."  Adam Smith’s classification relied on the two aspects of the nature of the State: first, the State as a juristic
78

person with an artificial personality, and, second, the State as a sovereign or entity possessing supreme power. Under the
first aspect, the State could hold property and engage in trade, thereby deriving what is called its quasi private income or
revenues, and which "peculiarly belonged to the sovereign." Under the second aspect, the State could collect by imposing
charges on the revenues of its subjects in the form of taxes. 79

In the Philippines, public revenues are generally derived from the following sources, to wit: (1) tax revenues(i.e.,
compulsory contributions to finance government activities); 80 (2) capital revenues(i.e., proceeds from sales of fixed
capital assets or scrap thereof and public domain, and gains on such sales like sale of public lands, buildings and other
structures, equipment, and other properties recorded as fixed assets); 81 (3) grants(i.e., voluntary contributions and aids
given to the Government for its operation on specific purposes in the form of money and/or materials, and do not require
any monetary commitment on the part of the recipient);  (4) extraordinary income(i.e., repayment of loans and advances
82

made by government corporations and local governments and the receipts and shares in income of the Banko Sentral ng
Pilipinas, and other receipts);  and (5) public borrowings(i.e., proceeds of repayable obligations generally with interest
83

from domestic and foreign creditors of the Government in general, including the National Government and its political
subdivisions).84

More specifically, public revenues are classified as follows: 85

General Income Specific Income


1. Subsidy Income from National 1. Income Taxes
Government 2. Property Taxes
2. Subsidy from Central Office 3. Taxes on Goods and Services
3. Subsidy from Regional 4. Taxes on International Trade and
Office/Staff Bureaus Transactions
4. Income from Government 5. Other Taxes 6.Fines and Penalties-Tax Revenue
Services
7. Other Specific Income
5. Income from Government
Business Operations
6. Sales Revenue
7. Rent Income
8. Insurance Income
9. Dividend Income
10. Interest Income
11. Sale of Confiscated Goods and
Properties
12. Foreign Exchange (FOREX)
Gains
13. Miscellaneous Operating and
Service Income
14. Fines and Penalties-Government
Services and Business Operations
15. Income from Grants and
Donations
c.2. Budget Legislation 86

The Budget Legislation Phase covers the period commencing from the time Congress receives the President’s Budget,
which is inclusive of the NEPand the BESF, up to the President’s approval of the GAA. This phase is also known as the
Budget Authorization Phase, and involves the significant participation of the Legislative through its deliberations.

Initially, the President’s Budget is assigned to the House of Representatives’ Appropriations Committee on First Reading.
The Appropriations Committee and its various Sub-Committees schedule and conduct budget hearings to examine the
PAPs of the departments and agencies. Thereafter, the House of Representatives drafts the General Appropriations Bill
(GAB). 87

The GABis sponsored, presented and defended by the House of Representatives’ Appropriations Committee and Sub-
Committees in plenary session. As with other laws, the GAB is approved on Third Reading before the House of
Representatives’ version is transmitted to the Senate.88

After transmission, the Senate conducts its own committee hearings on the GAB. To expedite proceedings, the Senate
may conduct its committee hearings simultaneously with the House of Representatives’ deliberations. The Senate’s
Finance Committee and its Sub-Committees may submit the proposed amendments to the GAB to the plenary of the
Senate only after the House of Representatives has formally transmitted its version to the Senate. The Senate version of
the GAB is likewise approved on Third Reading. 89

The House of Representatives and the Senate then constitute a panel each to sit in the Bicameral Conference Committee
for the purpose of discussing and harmonizing the conflicting provisions of their versions of the GAB. The "harmonized"
version of the GAB is next presented to the President for approval.  The President reviews the GAB, and prepares the
90

Veto Message where budget items are subjected to direct veto,  or are identified for conditional implementation.
91

If, by the end of any fiscal year, the Congress shall have failed to pass the GAB for the ensuing fiscal year, the GAA for
the preceding fiscal year shall be deemed re-enacted and shall remain in force and effect until the GAB is passed by the
Congress. 92

c.3. Budget Execution 93

With the GAA now in full force and effect, the next step is the implementation of the budget. The Budget Execution Phase
is primarily the function of the DBM, which is tasked to perform the following procedures, namely: (1) to issue the
programs and guidelines for the release of funds; (2) to prepare an Allotment and Cash Release Program; (3) to release
allotments; and (4) to issue disbursement authorities.

The implementation of the GAA is directed by the guidelines issued by the DBM. Prior to this, the various departments and
agencies are required to submit Budget Execution Documents(BED) to outline their plans and performance targets by
laying down the physical and financial plan, the monthly cash program, the estimate of monthly income, and the list of
obligations that are not yet due and demandable.

Thereafter, the DBM prepares an Allotment Release Program (ARP)and a Cash Release Program (CRP).The ARP sets a
limit for allotments issued in general and to a specific agency. The CRP fixes the monthly, quarterly and annual
disbursement levels.

Allotments, which authorize an agency to enter into obligations, are issued by the DBM. Allotments are lesser in scope
than appropriations, in that the latter embrace the general legislative authority to spend. Allotments may be released in two
forms – through a comprehensive Agency Budget Matrix (ABM),  or, individually, by SARO.
94 95

Armed with either the ABM or the SARO, agencies become authorized to incur obligations  on behalf of the Government
96

in order to implement their PAPs. Obligations may be incurred in various ways, like hiring of personnel, entering into
contracts for the supply of goods and services, and using utilities.

In order to settle the obligations incurred by the agencies, the DBM issues a disbursement authority so that cash may be
allocated in payment of the obligations. A cash or disbursement authority that is periodically issued is referred to as a
Notice of Cash Allocation (NCA),  which issuance is based upon an agency’s submission of its Monthly Cash Program
97

and other required documents. The NCA specifies the maximum amount of cash that can be withdrawn from a
government servicing bank for the period indicated. Apart from the NCA, the DBM may issue a Non-Cash Availment
Authority(NCAA) to authorize non-cash disbursements, or a Cash Disbursement Ceiling(CDC) for departments with
overseas operations to allow the use of income collected by their foreign posts for their operating requirements.

Actual disbursement or spending of government funds terminates the Budget Execution Phase and is usually
accomplished through the Modified Disbursement Scheme under which disbursements chargeable against the National
Treasury are coursed through the government servicing banks.

c.4. Accountability 98

Accountability is a significant phase of the budget cycle because it ensures that the government funds have been
effectively and efficiently utilized to achieve the State’s socio-economic goals. It also allows the DBM to assess the
performance of agencies during the fiscal year for the purpose of implementing reforms and establishing new policies.
An agency’s accountability may be examined and evaluated through (1) performance targets and outcomes; (2) budget
accountability reports; (3) review of agency performance; and (4) audit conducted by the Commission on Audit(COA).

2.

Nature of the DAP as a fiscal plan

a. DAP was a program designed to


promote economic growth

Policy is always a part of every budget and fiscal decision of any Administration.  The national budget the Executive
99

prepares and presents to Congress represents the Administration’s "blueprint for public policy" and reflects the
Government’s goals and strategies.  As such, the national budget becomes a tangible representation of the programs of
100

the Government in monetary terms, specifying therein the PAPs and services for which specific amounts of public funds
are proposed and allocated.  Embodied in every national budget is government spending.
101 102

When he assumed office in the middle of 2010, President Aquino made efficiency and transparency in government
spending a significant focus of his Administration. Yet, although such focus resulted in an improved fiscal deficit of 0.5% in
the gross domestic product (GDP) from January to July of 2011, it also unfortunately decelerated government project
implementation and payment schedules.  The World Bank observed that the Philippines’ economic growth could be
103

reduced, and potential growth could be weakened should the Government continue with its underspending and fail to
address the large deficiencies in infrastructure.  The economic situation prevailing in the middle of 2011 thus paved the
104

way for the development and implementation of the DAP as a stimulus package intended to fast-track public spending and
to push economic growth by investing on high-impact budgetary PAPs to be funded from the "savings" generated during
the year as well as from unprogrammed funds.  In that respect, the DAP was the product of "plain executive policy-
105

making" to stimulate the economy by way of accelerated spending.  The Administration would thereby accelerate
106

government spending by: (1) streamlining the implementation process through the clustering of infrastructure projects of
the Department of Public Works and Highways (DPWH) and the Department of Education (DepEd),and (2) front loading
PPP-related projects  due for implementation in the following year.
107 108

Did the stimulus package work?

The March 2012 report of the World Bank,  released after the initial implementation of the DAP, revealed that the DAP
109

was partially successful. The disbursements under the DAP contributed 1.3 percentage points to GDP growth by the fourth
quarter of 2011.  The continued implementation of the DAP strengthened growth by 11.8% year on year while
110

infrastructure spending rebounded from a 29% contraction to a 34% growth as of September 2013. 111

The DAP thus proved to be a demonstration that expenditure was a policy instrument that the Government could use to
direct the economies towards growth and development.  The Government, by spending on public infrastructure, would
112

signify its commitment of ensuring profitability for prospective investors.  The PAPs funded under the DAP were chosen
113

for this reason based on their: (1) multiplier impact on the economy and infrastructure development; (2) beneficial effect on
the poor; and (3) translation into disbursements. 114

b. History of the implementation of


the DAP, and sources of funds
under the DAP

How the Administration’s economic managers conceptualized and developed the DAP, and finally presented it to the
President remains unknown because the relevant documents appear to be scarce.

The earliest available document relating to the genesis of the DAP was the memorandum of October 12,2011 from Sec.
Abad seeking the approval of the President to implement the proposed DAP. The memorandum, which contained a list of
the funding sources for ₱72.11 billion and of the proposed priority projects to be funded,  reads:
115

MEMORANDUM FOR THE PRESIDENT

xxxx

SUBJECT: FY 2011 PROPOSED DISBURSEMENT ACCELERATION PROGRAM (PROJECTS AND SOURCES OF


FUNDS)

DATE: OCTOBER 12, 2011

Mr. President, this is to formally confirm your approval of the Disbursement Acceleration Program totaling ₱72.11 billion.
We are already working with all the agencies concerned for the immediate execution of the projects therein.

A. Fund Sources for the Acceleration Program

Fund Sources Amount Description Action


(In million
Requested
Php)

FY 2011 30,000 Unreleased Personnel Declare as


Unreleased Services (PS) savings and
Personal appropriations which approve/
Services (PS) will lapse at the end of authorize its use
Appropriations FY 2011 but may be for the 2011
pooled as savings and Disbursement
realigned for priority Acceleration
programs that require Program
immediate funding

FY 2011 482 Unreleased  


Unreleased appropriations (slow
Appropriations moving projects and
programs for
discontinuance)

FY 2010 12,336 Supported by the GFI Approve and


Unprogrammed Dividends authorize its use
Fund for the 2011
Disbursement
Acceleration
Program

FY 2010 21,544 Unreleased With prior


Carryover appropriations (slow approval from
Appropriation moving projects and the President in
programs for November 2010
discontinuance) and to declare as
savings from Zero-based Budgeting savings and with
Initiative authority to use
for priority
projects

FY 2011 Budget 7,748 FY 2011 Agency For information


items for Budget items that can
realignment be realigned within the
agency to fund new fast
disbursing projects
DPWH-3.981 Billion
DA – 2.497 Billion
DOT – 1.000 Billion
DepEd – 270 Million

TOTAL 72.110    

B. Projects in the Disbursement Acceleration Program

(Descriptions of projects attached as Annex A)

GOCCs and GFIs

Agency/Project Allotment
(SARO and NCA Release) (in Million Php)

1. LRTA: Rehabilitation of LRT 1 and 2 1,868

2. NHA: 11,050

a. Resettlement of North Triangle residents to 450


Camarin A7
b. Housing for BFP/BJMP 500
c. On-site development for families living 10,000
along dangerous
d. Relocation sites for informal settlers 100
along Iloilo River and its tributaries

3. PHIL. HEART CENTER: Upgrading of 357


ageing physical plant and medical equipment

75
4. CREDIT INFO CORP: Establishment of
centralized credit information system

100
5. PIDS: purchase of land to relocate the PIDS
office and building construction

400
6. HGC: Equity infusion for credit insurance
and mortgage guaranty operations of HGC

7. PHIC: Obligations incurred (premium 1,496


subsidy for indigent families) in January-June
2010, booked for payment in Jul[y] – Dec
2010. The delay in payment is due to the
delay in the certification of the LGU
counterpart. Without it, the NG is obliged to
pay the full amount.

8. Philpost: Purchase of foreclosed property. 644


Payment of Mandatory Obligations, (GSIS,
PhilHealth, ECC), Franking Privilege

10,000
9. BSP: First equity infusion out of Php 40B
capitalization under the BSP Law

280
10. PCMC: Capital and Equipment Renovation

11. LCOP: 105


a. Pediatric Pulmonary Program
b. Bio-regenerative Technology Program 35
(Stem-Cell Research – subject to legal
review and presentation) 70

570
12. TIDCORP: NG Equity infusion

TOTAL 26,945

NGAs/LGUs

Agency/Project Allotment
(SARO) Cash
(In Million Requirement
Php) (NCA)

13. DOF-BIR: NPSTAR


centralization of data    
processing and others (To be    
synchronized with GFMIS    
activities) 758 758

14. COA: IT infrastructure


program and hiring of    
additional litigational experts 144 144

15. DND-PAF: On Base Housing


Facilities and Communication    
Equipment 30 30

16. DA: 2,959 2,223


a. Irrigation, FMRs and
Integrated Community Based Multi-Species    
Hatchery and Aquasilvi    
Farming 1,629 1,629
b. Mindanao Rural
Development Project 919 183
c. NIA Agno River Integrated
Irrigation Project 411 411

17. DAR: 1,293 1,293


a. Agrarian Reform
Communities Project 2 1,293 132
b. Landowners Compensation 5,432

18. DBM: Conduct of National


Survey of    
Farmers/Fisherfolks/Ips 625 625

19. DOJ: Operating requirements


of 50 investigation agents and    
15 state attorneys 11 11

20. DOT: Preservation of the Cine


Corregidor Complex 25 25

21. OPAPP: Activities for Peace


Process (PAMANA- Project    
details: budget breakdown,    
implementation plan, and    
conditions on fund release    
attached as Annex B) 1,819 1,819

22. DOST 425 425


a. Establishment of National
Meterological and Climate    
Center 275 275
b. Enhancement of Doppler
Radar Network for National    
Weather Watch, Accurate    
Forecasting and Flood Early    
Warning 190 190

23. DOF-BOC: To settle the


principal obligations with    
PDIC consistent with the    
agreement with the CISS and    
SGS 2,800 2,800

24. OEO-FDCP: Establishment of


the National Film Archive and    
local cinematheques, and other    
local activities 20 20

25. DPWH: Various infrastructure


projects 5,500 5,500

26. DepEd/ERDT/DOST: Thin


Client Cloud Computing    
Project 270 270

27. DOH: Hiring of nurses and


midwives 294 294

28. TESDA: Training Program in


partnership with BPO industry    
and other sectors 1,100 1,100

29. DILG: Performance Challenge


Fund (People Empowered    
Community Driven    
Development with DSWD and    
NAPC) 250 50

30. ARMM: Comprehensive Peace


and Development Intervention 8,592 8,592

31. DOTC-MRT: Purchase of


additional MRT cars 4,500 -

32. LGU Support Fund 6,500 6,500


33. Various Other Local Projects 6,500 6,500

34. Development Assistance to the


Province of Quezon 750 750

TOTAL 45,165 44,000

C. Summary

  Fund Sources
Identified for Allotments Cash
Approval for Release Requirements for
(In Million Release in FY
Php) 2011

Total 72,110 72,110 70,895

GOCCs 26,895 26,895

NGAs/LGUs 45,165 44,000

For His Excellency’s Consideration

(Sgd.) FLORENCIO B. ABAD

[/] APPROVED

[ ] DISAPPROVED

(Sgd.) H.E. BENIGNO S. AQUINO, III

OCT 12, 2011

The memorandum of October 12, 2011 was followed by another memorandum for the President dated December 12,
2011  requesting omnibus authority to consolidate the savings and unutilized balances for fiscal year 2011. Pertinent
116

portions of the memorandum of December 12, 2011 read:

MEMORANDUM FOR THE PRESIDENT

xxxx

SUBJECT: Omnibus Authority to Consolidate Savings/Unutilized Balances and its Realignment

DATE: December 12, 2011

This is to respectfully request for the grant of Omnibus Authority to consolidate savings/unutilized balances in FY 2011
corresponding to completed or discontinued projects which may be pooled to fund additional projects or expenditures.

In addition, Mr. President, this measure will allow us to undertake projects even if their implementation carries over to
2012 without necessarily impacting on our budget deficit cap next year.

BACKGROUND

1.0 The DBM, during the course of performance reviews conducted on the agencies’ operations, particularly on the
implementation of their projects/activities, including expenses incurred in undertaking the same, have identified
savings out of the 2011 General Appropriations Act. Said savings correspond to completed or discontinued
projects under certain departments/agencies which may be pooled, for the following:

1.1 to provide for new activities which have not been anticipated during preparation of the budget;

1.2 to augment additional requirements of on-going priority projects; and

1.3 to provide for deficiencies under the Special Purpose Funds, e.g., PDAF, Calamity Fund, Contingent
Fund

1.4 to cover for the modifications of the original allotment class allocation as a result of on-going priority
projects and implementation of new activities
2.0 x x x x

2.1 x x x

2.2 x x x

ON THE UTILIZATION OF POOLED SAVINGS

3.0 It may be recalled that the President approved our request for omnibus authority to pool savings/unutilized
balances in FY 2010 last November 25, 2010.

4.0 It is understood that in the utilization of the pooled savings, the DBM shall secure the corresponding
approval/confirmation of the President. Furthermore, it is assured that the proposed realignments shall be within
the authorized Expenditure level.

5.0 Relative thereto, we have identified some expenditure items that may be sourced from the said pooled
appropriations in FY 2010 that will expire on December 31, 2011 and appropriations in FY 2011 that may be
declared as savings to fund additional expenditures.

5.1 The 2010 Continuing Appropriations (pooled savings) is proposed to be spent for the projects that we
have identified to be immediate actual disbursements considering that this same fund source will expire on
December 31, 2011.

5.2 With respect to the proposed expenditure items to be funded from the FY 2011 Unreleased
Appropriations, most of these are the same projects for which the DBM is directed by the Office of the
President, thru the Executive Secretary, to source funds.

6.0 Among others, the following are such proposed additional projects that have been chosen given their multiplier
impact on economy and infrastructure development, their beneficial effect on the poor, and their translation into
disbursements. Please note that we have classified the list of proposed projects as follows:

7.0 x x x

FOR THE PRESIDENT’S APPROVAL

8.0 Foregoing considered, may we respectfully request for the President’s approval for the following:

8.1 Grant of omnibus authority to consolidate FY 2011 savings/unutilized balances and its realignment;
and

8.2 The proposed additional projects identified for funding.

For His Excellency’s consideration and approval.

(Sgd.)

[/] APPROVED

[ ] DISAPPROVED

(Sgd.) H.E. BENIGNO S. AQUINO, III

DEC 21, 2011

Substantially identical requests for authority to pool savings and to fund proposed projects were contained in various other
memoranda from Sec. Abad dated June 25, 2012,  September 4, 2012,  December 19, 2012,  May 20, 2013,  and
117 118 119 120

September 25, 2013.  The President apparently approved all the requests, withholding approval only of the proposed
121

projects contained in the June 25, 2012 memorandum, as borne out by his marginal note therein to the effect that the
proposed projects should still be "subject to further discussions."122

In order to implement the June25, 2012 memorandum, Sec. Abad issued NBC No. 541 (Adoption of Operational Efficiency
Measure – Withdrawal of Agencies’ Unobligated Allotments as of June 30, 2012),  reproduced herein as follows:
123

NATIONAL BUDGET CIRCULAR No. 541

July 18, 2012

TO: All Heads of Departments/Agencies/State Universities and Colleges and other Offices of the National Government,
Budget and Planning Officers; Heads of Accounting Units and All Others Concerned
SUBJECT : Adoption of Operational Efficiency Measure – Withdrawal of Agencies’ Unobligated Allotments as of June 30,
2012

1.0 Rationale

The DBM, as mandated by Executive Order (EO) No. 292 (Administrative Code of 1987), periodically reviews and
evaluates the departments/agencies’ efficiency and effectiveness in utilizing budgeted funds for the delivery of services
and production of goods, consistent with the government priorities.

In the event that a measure is necessary to further improve the operational efficiency of the government, the President is
authorized to suspend or stop further use of funds allotted for any agency or expenditure authorized in the General
Appropriations Act. Withdrawal and pooling of unutilized allotment releases can be effected by DBM based on authority of
the President, as mandated under Sections 38 and 39, Chapter 5, Book VI of EO 292.

For the first five months of 2012, the National Government has not met its spending targets. In order to accelerate
spending and sustain the fiscal targets during the year, expenditure measures have to be implemented to optimize the
utilization of available resources.

Departments/agencies have registered low spending levels, in terms of obligations and disbursements per initial review of
their 2012 performance. To enhance agencies’ performance, the DBM conducts continuous consultation meetings and/or
send call-up letters, requesting them to identify slow-moving programs/projects and the factors/issues affecting their
performance (both pertaining to internal systems and those which are outside the agencies’ spheres of control). Also, they
are asked to formulate strategies and improvement plans for the rest of 2012.

Notwithstanding these initiatives, some departments/agencies have continued to post low obligation levels as of end of
first semester, thus resulting to substantial unobligated allotments.

In line with this, the President, per directive dated June 27, 2012 authorized the withdrawal of unobligated allotments of
agencies with low levels of obligations as of June 30, 2012, both for continuing and current allotments. This measure will
allow the maximum utilization of available allotments to fund and undertake other priority expenditures of the national
government.

2.0 Purpose

2.1 To provide the conditions and parameters on the withdrawal of unobligated allotments of agencies as of June
30, 2012 to fund priority and/or fast-moving programs/projects of the national government;

2.2 To prescribe the reports and documents to be used as bases on the withdrawal of said unobligated allotments;
and

2.3 To provide guidelines in the utilization or reallocation of the withdrawn allotments.

3.0 Coverage

3.1 These guidelines shall cover the withdrawal of unobligated allotments as of June 30, 2012 of all national
government agencies (NGAs) charged against FY 2011 Continuing Appropriation (R.A. No.10147) and FY 2012
Current Appropriation (R.A. No. 10155), pertaining to:

3.1.1 Capital Outlays (CO);

3.1.2 Maintenance and Other Operating Expenses (MOOE) related to the implementation of programs and
projects, as well as capitalized MOOE; and

3.1.3 Personal Services corresponding to unutilized pension benefits declared as savings by the agencies
concerned based on their updated/validated list of pensioners.

3.2 The withdrawal of unobligated allotments may cover the identified programs, projects and activities of the
departments/agencies reflected in the DBM list shown as Annex A or specific programs and projects as may be
identified by the agencies.

4.0 Exemption

These guidelines shall not apply to the following:

4.1 NGAs

4.1.1 Constitutional Offices/Fiscal Autonomy Group, granted fiscal autonomy under the Philippine
Constitution; and
4.1.2 State Universities and Colleges, adopting the Normative Funding allocation scheme i.e., distribution
of a predetermined budget ceiling.

4.2 Fund Sources

4.2.1 Personal Services other than pension benefits;

4.2.2 MOOE items earmarked for specific purposes or subject to realignment conditions per General
Provisions of the GAA:

• Confidential and Intelligence Fund;

• Savings from Traveling, Communication, Transportation and Delivery, Repair and Maintenance,
Supplies and Materials and Utility which shall be used for the grant of Collective Negotiation
Agreement incentive benefit;

• Savings from mandatory expenditures which can be realigned only in the last quarter after taking
into consideration the agency’s full year requirements, i.e., Petroleum, Oil and Lubricants, Water,
Illumination, Power Services, Telephone, other Communication Services and Rent.

4.2.3 Foreign-Assisted Projects (loan proceeds and peso counterpart);

4.2.4 Special Purpose Funds such as: E-Government Fund, International Commitments Fund, PAMANA,
Priority Development Assistance Fund, Calamity Fund, Budgetary Support to GOCCs and Allocation to
LGUs, among others;

4.2.5 Quick Response Funds; and

4.2.6 Automatic Appropriations i.e., Retirement Life Insurance Premium and Special Accounts in the
General Fund.

5.0 Guidelines

5.1 National government agencies shall continue to undertake procurement activities notwithstanding the
implementation of the policy of withdrawal of unobligated allotments until the end of the third quarter, FY 2012.
Even without the allotments, the agency shall proceed in undertaking the procurement processes (i.e.,
procurement planning up to the conduct of bidding but short of awarding of contract) pursuant to GPPB Circular
Nos. 02-2008 and 01-2009 and DBM Circular Letter No. 2010-9.

5.2 For the purpose of determining the amount of unobligated allotments that shall be withdrawn, all
departments/agencies/operating units (OUs) shall submit to DBM not later than July 30, 2012, the following budget
accountability reports as of June 30, 2012;

• Statement of Allotments, Obligations and Balances (SAOB);

• Financial Report of Operations (FRO); and

• Physical Report of Operations.

5.3 In the absence of the June 30, 2012 reports cited under item 5.2 of this Circular, the agency’s latest report
available shall be used by DBM as basis for withdrawal of allotment. The DBM shall compute/approximate the
agency’s obligation level as of June 30 to derive its unobligated allotments as of same period. Example: If the
March 31 SAOB or FRO reflects actual obligations of P 800M then the June 30 obligation level shall approximate
to ₱1,600 M (i.e., ₱800 M x 2 quarters).

5.4 All released allotments in FY 2011 charged against R.A. No. 10147 which remained unobligated as of June 30,
2012 shall be immediately considered for withdrawal. This policy is based on the following considerations:

5.4.1 The departments/agencies’ approved priority programs and projects are assumed to be
implementation-ready and doable during the given fiscal year; and

5.4.2 The practice of having substantial carryover appropriations may imply that the agency has a slower-
than-programmed implementation capacity or agency tends to implement projects within a two-year
timeframe.

5.5. Consistent with the President’s directive, the DBM shall, based on evaluation of the reports cited above and
results of consultations with the departments/agencies, withdraw the unobligated allotments as of June 30, 2012
through issuance of negative Special Allotment Release Orders (SAROs).
5.6 DBM shall prepare and submit to the President, a report on the magnitude of withdrawn allotments. The report
shall highlight the agencies which failed to submit the June 30 reports required under this Circular.

5.7 The withdrawn allotments may be:

5.7.1 Reissued for the original programs and projects of the agencies/OUs concerned, from which the
allotments were withdrawn;

5.7.2 Realigned to cover additional funding for other existing programs and projects of the agency/OU; or

5.7.3 Used to augment existing programs and projects of any agency and to fund priority programs and
projects not considered in the 2012 budget but expected to be started or implemented during the current
year.

5.8 For items 5.7.1 and 5.7.2 above, agencies/OUs concerned may submit to DBM a Special Budget Request
(SBR), supported with the following:

5.8.1 Physical and Financial Plan (PFP);

5.8.2 Monthly Cash Program (MCP); and

5.8.3 Proof that the project/activity has started the procurement processes i.e., Proof of Posting and/or
Advertisement of the Invitation to Bid.

5.9 The deadline for submission of request/s pertaining to these categories shall be until the end of the third
quarter i.e., September 30, 2012. After said cut-off date, the withdrawn allotments shall be pooled and form part of
the overall savings of the national government.

5.10 Utilization of the consolidated withdrawn allotments for other priority programs and projects as cited under
item 5.7.3 of this Circular, shall be subject to approval of the President. Based on the approval of the President,
DBM shall issue the SARO to cover the approved priority expenditures subject to submission by the agency/OU
concerned of the SBR and supported with PFP and MCP.

5.11 It is understood that all releases to be made out of the withdrawn allotments (both 2011 and 2012 unobligated
allotments) shall be within the approved Expenditure Program level of the national government for the current
year. The SAROs to be issued shall properly disclose the appropriation source of the release to determine the
extent of allotment validity, as follows:

• For charges under R.A. 10147 – allotments shall be valid up to December 31, 2012; and

• For charges under R.A. 10155 – allotments shall be valid up to December 31, 2013.

5.12 Timely compliance with the submission of existing BARs and other reportorial requirements is reiterated for
monitoring purposes.

6.0 Effectivity

This circular shall take effect immediately.

(Sgd.) FLORENCIO B. ABAD


Secretary

As can be seen, NBC No. 541 specified that the unobligated allotments of all agencies and departments as of June 30,
2012 that were charged against the continuing appropriations for fiscal year 2011 and the 2012 GAA (R.A. No. 10155)
were subject to withdrawal through the issuance of negative SAROs, but such allotments could be either: (1) reissued for
the original PAPs of the concerned agencies from which they were withdrawn; or (2) realigned to cover additional funding
for other existing PAPs of the concerned agencies; or (3) used to augment existing PAPs of any agency and to fund
priority PAPs not considered in the 2012 budget but expected to be started or implemented in 2012. Financing the other
priority PAPs was made subject to the approval of the President. Note here that NBC No. 541 used terminologies like
"realignment" and "augmentation" in the application of the withdrawn unobligated allotments.

Taken together, all the issuances showed how the DAP was to be implemented and funded, that is — (1) by declaring
"savings" coming from the various departments and agencies derived from pooling unobligated allotments and
withdrawing unreleased appropriations; (2) releasing unprogrammed funds; and (3) applying the "savings" and
unprogrammed funds to augment existing PAPs or to support other priority PAPs.

c. DAP was not an appropriation


measure; hence, no appropriation
law was required to adopt or to
implement it
Petitioners Syjuco, Luna, Villegas and PHILCONSA state that Congress did not enact a law to establish the DAP, or to
authorize the disbursement and release of public funds to implement the DAP. Villegas, PHILCONSA, IBP, Araullo, and
COURAGE observe that the appropriations funded under the DAP were not included in the 2011, 2012 and 2013 GAAs.
To petitioners IBP, Araullo, and COURAGE, the DAP, being actually an appropriation that set aside public funds for public
use, should require an enabling law for its validity. VACC maintains that the DAP, because it involved huge allocations that
were separate and distinct from the GAAs, circumvented and duplicated the GAAs without congressional authorization
and control.

The petitioners contend in unison that based on how it was developed and implemented the DAP violated the mandate of
Section 29(1), Article VI of the 1987 Constitution that "[n]o money shall be paid out of the Treasury except in pursuance of
an appropriation made by law."

The OSG posits, however, that no law was necessary for the adoption and implementation of the DAP because of its
being neither a fund nor an appropriation, but a program or an administrative system of prioritizing spending; and that the
adoption of the DAP was by virtue of the authority of the President as the Chief Executive to ensure that laws were
faithfully executed.

We agree with the OSG’s position.

The DAP was a government policy or strategy designed to stimulate the economy through accelerated spending. In the
context of the DAP’s adoption and implementation being a function pertaining to the Executive as the main actor during
the Budget Execution Stage under its constitutional mandate to faithfully execute the laws, including the GAAs, Congress
did not need to legislate to adopt or to implement the DAP. Congress could appropriate but would have nothing more to do
during the Budget Execution Stage. Indeed, appropriation was the act by which Congress "designates a particular fund, or
sets apart a specified portion of the public revenue or of the money in the public treasury, to be applied to some general
object of governmental expenditure, or to some individual purchase or expense."  As pointed out in Gonzales v.
124

Raquiza:  ‘"In a strict sense, appropriation has been defined ‘as nothing more than the legislative authorization prescribed
125

by the Constitution that money may be paid out of the Treasury,’ while appropriation made by law refers to ‘the act of the
legislature setting apart or assigning to a particular use a certain sum to be used in the payment of debt or dues from the
State to its creditors.’"
126

On the other hand, the President, in keeping with his duty to faithfully execute the laws, had sufficient discretion during the
execution of the budget to adapt the budget to changes in the country’s economic situation.  He could adopt a plan like
127

the DAP for the purpose. He could pool the savings and identify the PAPs to be funded under the DAP. The pooling of
savings pursuant to the DAP, and the identification of the PAPs to be funded under the DAP did not involve appropriation
in the strict sense because the money had been already set apart from the public treasury by Congress through the GAAs.
In such actions, the Executive did not usurp the power vested in Congress under Section 29(1), Article VI of the
Constitution.

3.
Unreleased appropriations and withdrawn
unobligated allotments under the DAP
were not savings, and the use of such
appropriations contravened Section 25(5),
Article VI of the 1987 Constitution.

Notwithstanding our appreciation of the DAP as a plan or strategy validly adopted by the Executive to ramp up spending to
accelerate economic growth, the challenges posed by the petitioners constrain us to dissect the mechanics of the actual
execution of the DAP. The management and utilization of the public wealth inevitably demands a most careful scrutiny of
whether the Executive’s implementation of the DAP was consistent with the Constitution, the relevant GAAs and other
existing laws.

a. Although executive discretion


and flexibility are necessary in
the execution of the budget, any
transfer of appropriated funds
should conform to Section 25(5),
Article VI of the Constitution

We begin this dissection by reiterating that Congress cannot anticipate all issues and needs that may come into play once
the budget reaches its execution stage. Executive discretion is necessary at that stage to achieve a sound fiscal
administration and assure effective budget implementation. The heads of offices, particularly the President, require
flexibility in their operations under performance budgeting to enable them to make whatever adjustments are needed to
meet established work goals under changing conditions.  In particular, the power to transfer funds can give the President
128

the flexibility to meet unforeseen events that may otherwise impede the efficient implementation of the PAPs set by
Congress in the GAA.

Congress has traditionally allowed much flexibility to the President in allocating funds pursuant to the GAAs,  particularly
129

when the funds are grouped to form lump sum accounts.  It is assumed that the agencies of the Government enjoy more
130

flexibility when the GAAs provide broader appropriation items.  This flexibility comes in the form of policies that the
131

Executive may adopt during the budget execution phase. The DAP – as a strategy to improve the country’s economic
position – was one policy that the President decided to carry out in order to fulfill his mandate under the GAAs.
Denying to the Executive flexibility in the expenditure process would be counterproductive. In Presidential Spending
Power,  Prof. Louis Fisher, an American constitutional scholar whose specialties have included budget policy, has
132

justified extending discretionary authority to the Executive thusly:

[T]he impulse to deny discretionary authority altogether should be resisted. There are many number of reasons why
obligations and outlays by administrators may have to differ from appropriations by legislators. Appropriations are made
many months, and sometimes years, in advance of expenditures. Congress acts with imperfect knowledge in trying to
legislate in fields that are highly technical and constantly undergoing change. New circumstances will develop to make
obsolete and mistaken the decisions reached by Congress at the appropriation stage. It is not practicable for Congress to
adjust to each new development by passing separate supplemental appropriation bills. Were Congress to control
expenditures by confining administrators to narrow statutory details, it would perhaps protect its power of the purse but it
would not protect the purse itself. The realities and complexities of public policy require executive discretion for the sound
management of public funds.

xxxx

x x x The expenditure process, by its very nature, requires substantial discretion for administrators. They need to exercise
judgment and take responsibility for their actions, but those actions ought to be directed toward executing congressional,
not administrative policy. Let there be discretion, but channel it and use it to satisfy the programs and priorities established
by Congress.

In contrast, by allowing to the heads of offices some power to transfer funds within their respective offices, the Constitution
itself ensures the fiscal autonomy of their offices, and at the same time maintains the separation of powers among the
three main branches of the Government. The Court has recognized this, and emphasized so in Bengzon v. Drilon,  viz: 133

The Judiciary, the Constitutional Commissions, and the Ombudsman must have the independence and flexibility needed
in the discharge of their constitutional duties. The imposition of restrictions and constraints on the manner the independent
constitutional offices allocate and utilize the funds appropriated for their operations is anathema to fiscal autonomy and
violative not only of the express mandate of the Constitution but especially as regards the Supreme Court, of the
independence and separation of powers upon which the entire fabric of our constitutional system is based.

In the case of the President, the power to transfer funds from one item to another within the Executive has not been the
mere offshoot of established usage, but has emanated from law itself. It has existed since the time of the American
Governors-General.  Act No. 1902 (An Act authorizing the Governor-General to direct any unexpended balances of
134

appropriations be returned to the general fund of the Insular Treasury and to transfer from the general fund moneys which
have been returned thereto), passed on May 18, 1909 by the First Philippine Legislature,  was the first enabling law that
135

granted statutory authority to the President to transfer funds. The authority was without any limitation, for the Act explicitly
empowered the Governor-General to transfer any unexpended balance of appropriations for any bureau or office to
another, and to spend such balance as if it had originally been appropriated for that bureau or office.

From 1916 until 1920, the appropriations laws set a cap on the amounts of funds that could be transferred, thereby limiting
the power to transfer funds. Only 10% of the amounts appropriated for contingent or miscellaneous expenses could be
transferred to a bureau or office, and the transferred funds were to be used to cover deficiencies in the appropriations also
for miscellaneous expenses of said bureau or office.

In 1921, the ceiling on the amounts of funds to be transferred from items under miscellaneous expenses to any other item
of a certain bureau or office was removed.

During the Commonwealth period, the power of the President to transfer funds continued to be governed by the GAAs
despite the enactment of the Constitution in 1935. It is notable that the 1935 Constitution did not include a provision on the
power to transfer funds. At any rate, a shift in the extent of the President’s power to transfer funds was again experienced
during this era, with the President being given more flexibility in implementing the budget. The GAAs provided that the
power to transfer all or portions of the appropriations in the Executive Department could be made in the "interest of the
public, as the President may determine." 136

In its time, the 1971 Constitutional Convention wanted to curtail the President’s seemingly unbounded discretion in
transferring funds.  Its Committee on the Budget and Appropriation proposed to prohibit the transfer of funds among the
137

separate branches of the Government and the independent constitutional bodies, but to allow instead their respective
heads to augment items of appropriations from savings in their respective budgets under certain limitations.  The clear
138

intention of the Convention was to further restrict, not to liberalize, the power to transfer appropriations.  Thus, the
139

Committee on the Budget and Appropriation initially considered setting stringent limitations on the power to augment, and
suggested that the augmentation of an item of appropriation could be made "by not more than ten percent if the original
item of appropriation to be augmented does not exceed one million pesos, or by not more than five percent if the original
item of appropriation to be augmented exceeds one million pesos."  But two members of the Committee objected to the
140

₱1,000,000.00 threshold, saying that the amount was arbitrary and might not be reasonable in the future. The Committee
agreed to eliminate the ₱1,000,000.00 threshold, and settled on the ten percent limitation. 141

In the end, the ten percent limitation was discarded during the plenary of the Convention, which adopted the following final
version under Section 16, Article VIII of the 1973 Constitution, to wit:
(5) No law shall be passed authorizing any transfer of appropriations; however, the President, the Prime Minister, the
Speaker, the Chief Justice of the Supreme Court, and the heads of Constitutional Commissions may by law be authorized
to augment any item in the general appropriations law for their respective offices from savings in other items of their
respective appropriations.

The 1973 Constitution explicitly and categorically prohibited the transfer of funds from one item to another, unless
Congress enacted a law authorizing the President, the Prime Minister, the Speaker, the Chief Justice of the Supreme
Court, and the heads of the Constitutional omissions to transfer funds for the purpose of augmenting any item from
savings in another item in the GAA of their respective offices. The leeway was limited to augmentation only, and was
further constricted by the condition that the funds to be transferred should come from savings from another item in the
appropriation of the office.
142

On July 30, 1977, President Marcos issued PD No. 1177, providing in its Section 44 that:

Section 44. Authority to Approve Fund Transfers. The President shall have the authority to transfer any fund appropriated
for the different departments, bureaus, offices and agencies of the Executive Department which are included in the
General Appropriations Act, to any program, project, or activity of any department, bureau or office included in the General
Appropriations Act or approved after its enactment.

The President shall, likewise, have the authority to augment any appropriation of the Executive Department in the General
Appropriations Act, from savings in the appropriations of another department, bureau, office or agency within the
Executive Branch, pursuant to the provisions of Article VIII, Section 16 (5) of the Constitution.

In Demetria v. Alba, however, the Court struck down the first paragraph of Section 44 for contravening Section 16(5)of the
1973 Constitution, ruling:

Paragraph 1 of Section 44 of P.D. No. 1177 unduly over-extends the privilege granted under said Section 16. It empowers
the President to indiscriminately transfer funds from one department, bureau, office or agency of the Executive
Department to any program, project or activity of any department, bureau or office included in the General Appropriations
Act or approved after its enactment, without regard as to whether or not the funds to be transferred are actually savings in
the item from which the same are to be taken, or whether or not the transfer is for the purpose of augmenting the item to
which said transfer is to be made. It does not only completely disregard the standards set in the fundamental law, thereby
amounting to an undue delegation of legislative powers, but likewise goes beyond the tenor thereof. Indeed, such
constitutional infirmities render the provision in question null and void.
143

It is significant that Demetria was promulgated 25 days after the ratification by the people of the 1987 Constitution, whose
Section 25(5) of Article VI is identical to Section 16(5), Article VIII of the 1973 Constitution, to wit:

Section 25. x x x

xxxx

5) No law shall be passed authorizing any transfer of appropriations; however, the President, the President of the Senate,
the Speaker of the House of Representatives, the Chief Justice of the Supreme Court, and the heads of Constitutional
Commissions may, by law, be authorized to augment any item in the general appropriations law for their respective offices
from savings in other items of their respective appropriations.

xxxx

The foregoing history makes it evident that the Constitutional Commission included Section 25(5), supra, to keep a tight
rein on the exercise of the power to transfer funds appropriated by Congress by the President and the other high officials
of the Government named therein. The Court stated in Nazareth v. Villar: 144

In the funding of current activities, projects, and programs, the general rule should still be that the budgetary amount
contained in the appropriations bill is the extent Congress will determine as sufficient for the budgetary allocation for the
proponent agency. The only exception is found in Section 25 (5), Article VI of the Constitution, by which the President, the
President of the Senate, the Speaker of the House of Representatives, the Chief Justice of the Supreme Court, and the
heads of Constitutional Commissions are authorized to transfer appropriations to augmentany item in the GAA for their
respective offices from the savings in other items of their respective appropriations. The plain language of the
constitutional restriction leaves no room for the petitioner’s posture, which we should now dispose of as untenable.

It bears emphasizing that the exception in favor of the high officials named in Section 25(5), Article VI of the Constitution
limiting the authority to transfer savings only to augment another item in the GAA is strictly but reasonably construed as
exclusive. As the Court has expounded in Lokin, Jr. v. Commission on Elections:

When the statute itself enumerates the exceptions to the application of the general rule, the exceptions are strictly but
reasonably construed. The exceptions extend only as far as their language fairly warrants, and all doubts should be
resolved in favor of the general provision rather than the exceptions. Where the general rule is established by a statute
with exceptions, none but the enacting authority can curtail the former. Not even the courts may add to the latter by
implication, and it is a rule that an express exception excludes all others, although it is always proper in determining the
applicability of the rule to inquire whether, in a particular case, it accords with reason and justice.
The appropriate and natural office of the exception is to exempt something from the scope of the general words of a
statute, which is otherwise within the scope and meaning of such general words. Consequently, the existence of an
exception in a statute clarifies the intent that the statute shall apply to all cases not excepted. Exceptions are subject to the
rule of strict construction; hence, any doubt will be resolved in favor of the general provision and against the exception.
Indeed, the liberal construction of a statute will seem to require in many circumstances that the exception, by which the
operation of the statute is limited or abridged, should receive a restricted construction.

Accordingly, we should interpret Section 25(5), supra, in the context of a limitation on the President’s discretion over the
appropriations during the Budget Execution Phase.

b. Requisites for the valid transfer of


appropriated funds under Section
25(5), Article VI of the 1987
Constitution

The transfer of appropriated funds, to be valid under Section 25(5), supra, must be made upon a concurrence of the
following requisites, namely:

(1) There is a law authorizing the President, the President of the Senate, the Speaker of the House of
Representatives, the Chief Justice of the Supreme Court, and the heads of the Constitutional Commissions to
transfer funds within their respective offices;

(2) The funds to be transferred are savings generated from the appropriations for their respective offices; and (3)
The purpose of the transfer is to augment an item in the general appropriations law for their respective offices.

b.1. First Requisite–GAAs of 2011 and


2012 lacked valid provisions to
authorize transfers of funds under
the DAP; hence, transfers under the
DAP were unconstitutional

Section 25(5), supra, not being a self-executing provision of the Constitution, must have an implementing law for it to be
operative. That law, generally, is the GAA of a given fiscal year. To comply with the first requisite, the GAAs should
expressly authorize the transfer of funds.

Did the GAAs expressly authorize the transfer of funds?

In the 2011 GAA, the provision that gave the President and the other high officials the authority to transfer funds was
Section 59, as follows:

Section 59. Use of Savings. The President of the Philippines, the Senate President, the Speaker of the House of
Representatives, the Chief Justice of the Supreme Court, the Heads of Constitutional Commissions enjoying fiscal
autonomy, and the Ombudsman are hereby authorized to augment any item in this Act from savings in other items of their
respective appropriations.

In the 2012 GAA, the empowering provision was Section 53, to wit:

Section 53. Use of Savings. The President of the Philippines, the Senate President, the Speaker of the House of
Representatives, the Chief Justice of the Supreme Court, the Heads of Constitutional Commissions enjoying fiscal
autonomy, and the Ombudsman are hereby authorized to augment any item in this Act from savings in other items of their
respective appropriations.

In fact, the foregoing provisions of the 2011 and 2012 GAAs were cited by the DBM as justification for the use of savings
under the DAP. 145

A reading shows, however, that the aforequoted provisions of the GAAs of 2011 and 2012 were textually unfaithful to the
Constitution for not carrying the phrase "for their respective offices" contained in Section 25(5), supra. The impact of the
phrase "for their respective offices" was to authorize only transfers of funds within their offices (i.e., in the case of the
President, the transfer was to an item of appropriation within the Executive). The provisions carried a different phrase ("to
augment any item in this Act"), and the effect was that the 2011 and 2012 GAAs thereby literally allowed the transfer of
funds from savings to augment any item in the GAAs even if the item belonged to an office outside the Executive. To that
extent did the 2011 and 2012 GAAs contravene the Constitution. At the very least, the aforequoted provisions cannot be
used to claim authority to transfer appropriations from the Executive to another branch, or to a constitutional commission.

Apparently realizing the problem, Congress inserted the omitted phrase in the counterpart provision in the 2013 GAA, to
wit:

Section 52. Use of Savings. The President of the Philippines, the Senate President, the Speaker of the House of
Representatives, the Chief Justice of the Supreme Court, the Heads of Constitutional Commissions enjoying fiscal
autonomy, and the Ombudsman are hereby authorized to use savings in their respective appropriations to augment actual
deficiencies incurred for the current year in any item of their respective appropriations.
Even had a valid law authorizing the transfer of funds pursuant to Section 25(5), supra, existed, there still remained two
other requisites to be met, namely: that the source of funds to be transferred were savings from appropriations within the
respective offices; and that the transfer must be for the purpose of augmenting an item of appropriation within the
respective offices.

b.2. Second Requisite – There were


no savings from which funds
could be sourced for the DAP
Were the funds used in the DAP actually savings?

The petitioners claim that the funds used in the DAP — the unreleased appropriations and withdrawn unobligated
allotments — were not actual savings within the context of Section 25(5), supra, and the relevant provisions of the GAAs.
Belgica argues that "savings" should be understood to refer to the excess money after the items that needed to be funded
have been funded, or those that needed to be paid have been paid pursuant to the budget.  The petitioners posit that
146

there could be savings only when the PAPs for which the funds had been appropriated were actually implemented and
completed, or finally discontinued or abandoned. They insist that savings could not be realized with certainty in the middle
of the fiscal year; and that the funds for "slow-moving" PAPs could not be considered as savings because such PAPs had
not actually been abandoned or discontinued yet.  They stress that NBC No. 541, by allowing the withdrawn funds to be
147

reissued to the "original program or project from which it was withdrawn," conceded that the PAPs from which the
supposed savings were taken had not been completed, abandoned or discontinued. 148

The OSG represents that "savings" were "appropriations balances," being the difference between the appropriation
authorized by Congress and the actual amount allotted for the appropriation; that the definition of "savings" in the GAAs
set only the parameters for determining when savings occurred; that it was still the President (as well as the other officers
vested by the Constitution with the authority to augment) who ultimately determined when savings actually existed
because savings could be determined only during the stage of budget execution; that the President must be given a wide
discretion to accomplish his tasks; and that the withdrawn unobligated allotments were savings inasmuch as they were
clearly "portions or balances of any programmed appropriation…free from any obligation or encumbrances which are (i)
still available after the completion or final discontinuance or abandonment of the work, activity or purpose for which the
appropriation is authorized…"

We partially find for the petitioners.

In ascertaining the meaning of savings, certain principles should be borne in mind. The first principle is that Congress
wields the power of the purse. Congress decides how the budget will be spent; what PAPs to fund; and the amounts of
money to be spent for each PAP. The second principle is that the Executive, as the department of the Government tasked
to enforce the laws, is expected to faithfully execute the GAA and to spend the budget in accordance with the provisions of
the GAA.  The Executive is expected to faithfully implement the PAPs for which Congress allocated funds, and to limit the
149

expenditures within the allocations, unless exigencies result to deficiencies for which augmentation is authorized, subject
to the conditions provided by law. The third principle is that in making the President’s power to augment operative under
the GAA, Congress recognizes the need for flexibility in budget execution. In so doing, Congress diminishes its own power
of the purse, for it delegates a fraction of its power to the Executive. But Congress does not thereby allow the Executive to
override its authority over the purse as to let the Executive exceed its delegated authority. And the fourth principle is that
savings should be actual. "Actual" denotes something that is real or substantial, or something that exists presently in fact,
as opposed to something that is merely theoretical, possible, potential or hypothetical. 150

The foregoing principles caution us to construe savings strictly against expanding the scope of the power to augment. It is
then indubitable that the power to augment was to be used only when the purpose for which the funds had been allocated
were already satisfied, or the need for such funds had ceased to exist, for only then could savings be properly realized.
This interpretation prevents the Executive from unduly transgressing Congress’ power of the purse.

The definition of "savings" in the GAAs, particularly for 2011, 2012 and 2013, reflected this interpretation and made it
operational, viz:

Savings refer to portions or balances of any programmed appropriation in this Act free from any obligation or
encumbrance which are: (i) still available after the completion or final discontinuance or abandonment of the work, activity
or purpose for which the appropriation is authorized; (ii) from appropriations balances arising from unpaid compensation
and related costs pertaining to vacant positions and leaves of absence without pay; and (iii) from appropriations balances
realized from the implementation of measures resulting in improved systems and efficiencies and thus enabled agencies
to meet and deliver the required or planned targets, programs and services approved in this Act at a lesser cost.

The three instances listed in the GAAs’ aforequoted definition were a sure indication that savings could be generated only
upon the purpose of the appropriation being fulfilled, or upon the need for the appropriation being no longer existent.

The phrase "free from any obligation or encumbrance" in the definition of savings in the GAAs conveyed the notion that
the appropriation was at that stage when the appropriation was already obligated and the appropriation was already
released. This interpretation was reinforced by the enumeration of the three instances for savings to arise, which showed
that the appropriation referred to had reached the agency level. It could not be otherwise, considering that only when the
appropriation had reached the agency level could it be determined whether (a) the PAP for which the appropriation had
been authorized was completed, finally discontinued, or abandoned; or (b) there were vacant positions and leaves of
absence without pay; or (c) the required or planned targets, programs and services were realized at a lesser cost because
of the implementation of measures resulting in improved systems and efficiencies.
The DBM declares that part of the savings brought under the DAP came from "pooling of unreleased appropriations such
as unreleased Personnel Services appropriations which will lapse at the end of the year, unreleased appropriations of
slow moving projects and discontinued projects per Zero-Based Budgeting findings."

The declaration of the DBM by itself does not state the clear legal basis for the treatment of unreleased or unalloted
appropriations as savings.

The fact alone that the appropriations are unreleased or unalloted is a mere description of the status of the items as
unalloted or unreleased. They have not yet ripened into categories of items from which savings can be generated.
Appropriations have been considered "released" if there has already been an allotment or authorization to incur
obligations and disbursement authority. This means that the DBM has issued either an ABM (for those not needing
clearance), or a SARO (for those needing clearance), and consequently an NCA, NCAA or CDC, as the case may be.
Appropriations remain unreleased, for instance, because of noncompliance with documentary requirements (like the
Special Budget Request), or simply because of the unavailability of funds. But the appropriations do not actually reach the
agencies to which they were allocated under the GAAs, and have remained with the DBM technically speaking. Ergo,
unreleased appropriations refer to appropriations with allotments but without disbursement authority.

For us to consider unreleased appropriations as savings, unless these met the statutory definition of savings, would
seriously undercut the congressional power of the purse, because such appropriations had not even reached and been
used by the agency concerned vis-à-vis the PAPs for which Congress had allocated them. However, if an agency has
unfilled positions in its plantilla and did not receive an allotment and NCA for such vacancies, appropriations for such
positions, although unreleased, may already constitute savings for that agency under the second instance.

Unobligated allotments, on the other hand, were encompassed by the first part of the definition of "savings" in the GAA,
that is, as "portions or balances of any programmed appropriation in this Act free from any obligation or encumbrance."
But the first part of the definition was further qualified by the three enumerated instances of when savings would be
realized. As such, unobligated allotments could not be indiscriminately declared as savings without first determining
whether any of the three instances existed. This signified that the DBM’s withdrawal of unobligated allotments had
disregarded the definition of savings under the GAAs.

Justice Carpio has validly observed in his Separate Concurring Opinion that MOOE appropriations are deemed divided
into twelve monthly allocations within the fiscal year; hence, savings could be generated monthly from the excess or
unused MOOE appropriations other than the Mandatory Expenditures and Expenditures for Business-type Activities
because of the physical impossibility to obligate and spend such funds as MOOE for a period that already lapsed.
Following this observation, MOOE for future months are not savings and cannot be transferred.

The DBM’s Memorandum for the President dated June 25, 2012 (which became the basis of NBC No. 541) stated:

ON THE AUTHORITY TO WITHDRAW UNOBLIGATED ALLOTMENTS

5.0 The DBM, during the course of performance reviews conducted on the agencies’ operations, particularly on the
implementation of their projects/activities, including expenses incurred in undertaking the same, have been
continuously calling the attention of all National Government agencies (NGAs) with low levels of obligations as of
end of the first quarter to speedup the implementation of their programs and projects in the second quarter.

6.0 Said reminders were made in a series of consultation meetings with the concerned agencies and with call-up
letters sent.

7.0 Despite said reminders and the availability of funds at the department’s disposal, the level of financial
performance of some departments registered below program, with the targeted obligations/disbursements for the
first semester still not being met.

8.0 In order to maximize the use of the available allotment, all unobligated balances as of June 30, 2012, both for
continuing and current allotments shall be withdrawn and pooled to fund fast moving programs/projects.

9.0 It may be emphasized that the allotments to be withdrawn will be based on the list of slow moving projects to
be identified by the agencies and their catch up plans to be evaluated by the DBM.

It is apparent from the foregoing text that the withdrawal of unobligated allotments would be based on whether the
allotments pertained to slow-moving projects, or not. However, NBC No. 541 did not set in clear terms the criteria for the
withdrawal of unobligated allotments, viz:

3.1. These guidelines shall cover the withdrawal of unobligated allotments as of June 30, 2012 ofall national
government agencies (NGAs) charged against FY 2011 Continuing Appropriation (R.A. No. 10147) and FY 2012
Current Appropriation (R.A. No. 10155), pertaining to:

3.1.1 Capital Outlays (CO);

3.1.2 Maintenance and Other Operating Expenses (MOOE) related to the implementation of programs and
projects, as well as capitalized MOOE; and
3.1.3 Personal Services corresponding to unutilized pension benefits declared as savings by the agencies
concerned based on their undated/validated list of pensioners.

A perusal of its various provisions reveals that NBC No. 541 targeted the "withdrawal of unobligated allotments of
agencies with low levels of obligations"  "to fund priority and/or fast-moving programs/projects."  But the fact that the
151 152

withdrawn allotments could be "[r]eissued for the original programs and projects of the agencies/OUs concerned, from
which the allotments were withdrawn"  supported the conclusion that the PAPs had not yet been finally discontinued or
153

abandoned. Thus, the purpose for which the withdrawn funds had been appropriated was not yet fulfilled, or did not yet
cease to exist, rendering the declaration of the funds as savings impossible.

Worse, NBC No. 541 immediately considered for withdrawal all released allotments in 2011 charged against the 2011
GAA that had remained unobligated based on the following considerations, to wit:

5.4.1 The departments/agencies’ approved priority programs and projects are assumed to be implementation-
ready and doable during the given fiscal year; and

5.4.2 The practice of having substantial carryover appropriations may imply that the agency has a slower-than-
programmed implementation capacity or agency tends to implement projects within a two-year timeframe.

Such withdrawals pursuant to NBC No. 541, the circular that affected the unobligated allotments for continuing and current
appropriations as of June 30, 2012, disregarded the 2-year period of availability of the appropriations for MOOE and
capital outlay extended under Section 65, General Provisions of the 2011 GAA, viz:

Section 65. Availability of Appropriations. — Appropriations for MOOE and capital outlays authorized in this Act shall be
available for release and obligation for the purpose specified, and under the same special provisions applicable thereto,
for a period extending to one fiscal year after the end of the year in which such items were appropriated: PROVIDED, That
appropriations for MOOE and capital outlays under R.A. No. 9970 shall be made available up to the end of FY 2011:
PROVIDED, FURTHER, That a report on these releases and obligations shall be submitted to the Senate Committee on
Finance and the House Committee on Appropriations.

and Section 63 General Provisions of the 2012 GAA, viz:

Section 63. Availability of Appropriations. — Appropriations for MOOE and capital outlays authorized in this Act shall be
available for release and obligation for the purpose specified, and under the same special provisions applicable thereto,
for a period extending to one fiscal year after the end of the year in which such items were appropriated: PROVIDED, That
a report on these releases and obligations shall be submitted to the Senate Committee on Finance and the House
Committee on Appropriations, either in printed form or by way of electronic document. 154

Thus, another alleged area of constitutional infirmity was that the DAP and its relevant issuances shortened the period of
availability of the appropriations for MOOE and capital outlays.

Congress provided a one-year period of availability of the funds for all allotment classes in the 2013 GAA (R.A. No.
10352), to wit:

Section 63. Availability of Appropriations.— All appropriations authorized in this Act shall be available for release and
obligation for the purposes specified, and under the same special provisions applicable thereto, until the end of FY 2013:
PROVIDED, That a report on these releases and obligations shall be submitted to the Senate Committee on Finance and
House Committee on Appropriations, either in printed form or by way of electronic document.

Yet, in his memorandum for the President dated May 20, 2013, Sec. Abad sought omnibus authority to consolidate
savings and unutilized balances to fund the DAP on a quarterly basis, viz:

7.0 If the level of financial performance of some department will register below program, even with the availability
of funds at their disposal, the targeted obligations/disbursements for each quarter will not be met. It is important to
note that these funds will lapse at the end of the fiscal year if these remain unobligated.

8.0 To maximize the use of the available allotment, all unobligated balances at the end of every quarter, both for
continuing and current allotments shall be withdrawn and pooled to fund fast moving programs/projects.

9.0 It may be emphasized that the allotments to be withdrawn will be based on the list of slow moving projects to
be identified by the agencies and their catch up plans to be evaluated by the DBM.

The validity period of the affected appropriations, already given the brief Lifes pan of one year, was further shortened to
only a quarter of a year under the DBM’s memorandum dated May 20, 2013.

The petitioners accuse the respondents of forcing the generation of savings in order to have a larger fund available for
discretionary spending. They aver that the respondents, by withdrawing unobligated allotments in the middle of the fiscal
year, in effect deprived funding for PAPs with existing appropriations under the GAAs. 155
The respondents belie the accusation, insisting that the unobligated allotments were being withdrawn upon the instance of
the implementing agencies based on their own assessment that they could not obligate those allotments pursuant to the
President’s directive for them to spend their appropriations as quickly as they could in order to ramp up the economy. 156

We agree with the petitioners.

Contrary to the respondents’ insistence, the withdrawals were upon the initiative of the DBM itself. The text of NBC No.
541 bears this out, to wit:

5.2 For the purpose of determining the amount of unobligated allotments that shall be withdrawn, all
departments/agencies/operating units (OUs) shall submit to DBM not later than July 30, 2012, the following budget
accountability reports as of June 30, 2012;

• Statement of Allotments, Obligation and Balances (SAOB);

• Financial Report of Operations (FRO); and

• Physical Report of Operations.

5.3 In the absence of the June 30, 2012 reports cited under item 5.2 of this Circular, the agency’s latest report available
shall be used by DBM as basis for withdrawal of allotment. The DBM shall compute/approximate the agency’s obligation
level as of June 30 to derive its unobligated allotments as of same period. Example: If the March 31 SAOB or FRO reflects
actual obligations of P 800M then the June 30 obligation level shall approximate to ₱1,600 M (i.e., ₱800 M x 2 quarters).

The petitioners assert that no law had authorized the withdrawal and transfer of unobligated allotments and the pooling of
unreleased appropriations; and that the unbridled withdrawal of unobligated allotments and the retention of appropriated
funds were akin to the impoundment of appropriations that could be allowed only in case of "unmanageable national
government budget deficit" under the GAAs,  thus violating the provisions of the GAAs of 2011, 2012 and 2013 prohibiting
157

the retention or deduction of allotments.158

In contrast, the respondents emphasize that NBC No. 541 adopted a spending, not saving, policy as a last-ditch effort of
the Executive to push agencies into actually spending their appropriations; that such policy did not amount to an
impoundment scheme, because impoundment referred to the decision of the Executive to refuse to spend funds for
political or ideological reasons; and that the withdrawal of allotments under NBC No. 541 was made pursuant to Section
38, Chapter 5, Book VI of the Administrative Code, by which the President was granted the authority to suspend or
otherwise stop further expenditure of funds allotted to any agency whenever in his judgment the public interest so
required.

The assertions of the petitioners are upheld. The withdrawal and transfer of unobligated allotments and the pooling of
unreleased appropriations were invalid for being bereft of legal support. Nonetheless, such withdrawal of unobligated
allotments and the retention of appropriated funds cannot be considered as impoundment.

According to Philippine Constitution Association v. Enriquez:  "Impoundment refers to a refusal by the President, for
159

whatever reason, to spend funds made available by Congress. It is the failure to spend or obligate budget authority of any
type." Impoundment under the GAA is understood to mean the retention or deduction of appropriations. The 2011 GAA
authorized impoundment only in case of unmanageable National Government budget deficit, to wit:

Section 66. Prohibition Against Impoundment of Appropriations. No appropriations authorized under this Act shall be
impounded through retention or deduction, unless in accordance with the rules and regulations to be issued by the DBM:
PROVIDED, That all the funds appropriated for the purposes, programs, projects and activities authorized under this Act,
except those covered under the Unprogrammed Fund, shall be released pursuant to Section 33 (3), Chapter 5, Book VI of
E.O. No. 292.

Section 67. Unmanageable National Government Budget Deficit. Retention or deduction of appropriations authorized in
this Act shall be effected only in cases where there is an unmanageable national government budget deficit.

Unmanageable national government budget deficit as used in this section shall be construed to mean that (i) the actual
national government budget deficit has exceeded the quarterly budget deficit targets consistent with the full-year target
deficit as indicated in the FY 2011 Budget of

Expenditures and Sources of Financing submitted by the President and approved by Congress pursuant to Section 22,
Article VII of the Constitution, or (ii) there are clear economic indications of an impending occurrence of such condition, as
determined by the Development Budget Coordinating Committee and approved by the President.

The 2012 and 2013 GAAs contained similar provisions.

The withdrawal of unobligated allotments under the DAP should not be regarded as impoundment because it entailed only
the transfer of funds, not the retention or deduction of appropriations.
Nor could Section 68 of the 2011 GAA (and the similar provisions of the 2012 and 2013 GAAs) be applicable. They
uniformly stated:

Section 68. Prohibition Against Retention/Deduction of Allotment. Fund releases from appropriations provided in this Act
shall be transmitted intact or in full to the office or agency concerned. No retention or deduction as reserves or overhead
shall be made, except as authorized by law, or upon direction of the President of the Philippines. The COA shall ensure
compliance with this provision to the extent that sub-allotments by agencies to their subordinate offices are in conformity
with the release documents issued by the DBM.

The provision obviously pertained to the retention or deduction of allotments upon their release from the DBM, which was
a different matter altogether. The Court should not expand the meaning of the provision by applying it to the withdrawal of
allotments.

The respondents rely on Section 38, Chapter 5, Book VI of the Administrative Code of 1987 to justify the withdrawal of
unobligated allotments. But the provision authorized only the suspension or stoppage of further expenditures, not the
withdrawal of unobligated allotments, to wit:

Section 38. Suspension of Expenditure of Appropriations.- Except as otherwise provided in the General Appropriations Act
and whenever in his judgment the public interest so requires, the President, upon notice to the head of office concerned, is
authorized to suspend or otherwise stop further expenditure of funds allotted for any agency, or any other expenditure
authorized in the General Appropriations Act, except for personal services appropriations used for permanent officials and
employees.

Moreover, the DBM did not suspend or stop further expenditures in accordance with Section 38, supra, but instead
transferred the funds to other PAPs.

It is relevant to remind at this juncture that the balances of appropriations that remained unexpended at the end of the
fiscal year were to be reverted to the General Fund.  This was the mandate of Section 28, Chapter IV, Book VI of the
1âwphi1

Administrative Code, to wit:

Section 28. Reversion of Unexpended Balances of Appropriations, Continuing Appropriations.- Unexpended balances of
appropriations authorized in the General Appropriation Act shall revert to the unappropriated surplus of the General Fund
at the end of the fiscal year and shall not thereafter be available for expenditure except by subsequent legislative
enactment: Provided, that appropriations for capital outlays shall remain valid until fully spent or reverted: provided,
further, that continuing appropriations for current operating expenditures may be specifically recommended and approved
as such in support of projects whose effective implementation calls for multi-year expenditure commitments: provided,
finally, that the President may authorize the use of savings realized by an agency during given year to meet non-recurring
expenditures in a subsequent year.

The balances of continuing appropriations shall be reviewed as part of the annual budget preparation process and the
preparation process and the President may approve upon recommendation of the Secretary, the reversion of funds no
longer needed in connection with the activities funded by said continuing appropriations.

The Executive could not circumvent this provision by declaring unreleased appropriations and unobligated allotments as
savings prior to the end of the fiscal year.

b.3. Third Requisite – No funds from


savings could be transferred under
the DAP to augment deficient items
not provided in the GAA

The third requisite for a valid transfer of funds is that the purpose of the transfer should be "to augment an item in the
general appropriations law for the respective offices." The term "augment" means to enlarge or increase in size, amount,
or degree.160

The GAAs for 2011, 2012 and 2013 set as a condition for augmentation that the appropriation for the PAP item to be
augmented must be deficient, to wit: –

x x x Augmentation implies the existence in this Act of a program, activity, or project with an appropriation, which upon
implementation, or subsequent evaluation of needed resources, is determined to be deficient. In no case shall a non-
existent program, activity, or project, be funded by augmentation from savings or by the use of appropriations otherwise
authorized in this Act.

In other words, an appropriation for any PAP must first be determined to be deficient before it could be augmented from
savings. Note is taken of the fact that the 2013 GAA already made this quite clear, thus:

Section 52. Use of Savings. The President of the Philippines, the Senate President, the Speaker of the House of
Representatives, the Chief Justice of the Supreme Court, the Heads of Constitutional Commissions enjoying fiscal
autonomy, and the Ombudsman are hereby authorized to use savings in their respective appropriations to augment actual
deficiencies incurred for the current year in any item of their respective appropriations.
As of 2013, a total of ₱144.4 billion worth of PAPs were implemented through the DAP. 161

Of this amount ₱82.5 billion were released in 2011 and ₱54.8 billion in 2012.  Sec. Abad has reported that 9% of the total
162

DAP releases were applied to the PAPs identified by the legislators. 163

The petitioners disagree, however, and insist that the DAP supported the following PAPs that had not been covered with
appropriations in the respective GAAs, namely:

(i) ₱1.5 billion for the Cordillera People’s Liberation Army;

(ii) ₱1.8 billion for the Moro National Liberation Front;

(iii) ₱700 million for assistance to Quezon Province; 164

(iv) ₱50 million to ₱100 (million) each to certain senators; 165

(v) ₱10 billion for the relocation of families living along dangerous zones under the National Housing Authority;

(vi) ₱10 billion and ₱20 billion equity infusion under the Bangko Sentral;

(vii) ₱5.4 billion landowners’ compensation under the Department of Agrarian Reform;

(viii) ₱8.6 billion for the ARMM comprehensive peace and development program;

(ix) ₱6.5 billion augmentation of LGU internal revenue allotments

(x) ₱5 billion for crucial projects like tourism road construction under the Department of Tourism and the
Department of Public Works and Highways;

(xi) ₱1.8 billion for the DAR-DPWH Tulay ng Pangulo;

(xii) ₱1.96 billion for the DOH-DPWH rehabilitation of regional health units; and

(xiii) ₱4 billion for the DepEd-PPP school infrastructure projects. 166

In refutation, the OSG argues that a total of 116 DAP-financed PAPs were implemented, had appropriation covers, and
could properly be accounted for because the funds were released following and pursuant to the standard practices
adopted by the DBM.  In support of its argument, the OSG has submitted seven evidence packets containing
167

memoranda, SAROs, and other pertinent documents relative to the implementation and fund transfers under the DAP. 168

Upon careful review of the documents contained in the seven evidence packets, we conclude that the "savings" pooled
under the DAP were allocated to PAPs that were not covered by any appropriations in the pertinent GAAs.

For example, the SARO issued on December 22, 2011 for the highly vaunted Disaster Risk, Exposure, Assessment and
Mitigation (DREAM) project under the Department of Science and Technology (DOST) covered the amount of ₱1.6
Billion,  broken down as follows:
169

APPROPRIATION PARTICULARS AMOUNT


CODE AUTHORIZED

A.03.a.01.a Generation of new knowledge and technologies


and research capability building in priority areas
identified as strategic to National Development
Personnel Services
Maintenance and Other Operating Expenses P 43,504,024
Capital Outlays 1,164,517,589
391,978,387
P 1,600,000,000

the pertinent provision of the 2011 GAA (R.A. No. 10147) showed that Congress had appropriated only ₱537,910,000 for
MOOE, but nothing for personnel services and capital outlays, to wit:

Personnel Maintenance Capital TOTAL


Services and Other Outlays
Operating
Expenditures
III. Operations
a. Funding Assistance to Science 177,406,000 1,887,365,000 49,090,000 2,113,861,000
and Technology Activities
1. Central Office 1,554,238,000 1,554,238,000
a. Generation of new
knowledge and
technologies and research
capability building in
priority areas identified as
strategic to National
Development 537,910,000 537,910,000

Aside from this transfer under the DAP to the DREAM project exceeding by almost 300% the appropriation by Congress
for the program Generation of new knowledge and technologies and research capability building in priority areas identified
as strategic to National Development, the Executive allotted funds for personnel services and capital outlays. The
Executive thereby substituted its will to that of Congress. Worse, the Executive had not earlier proposed any amount for
personnel services and capital outlays in the NEP that became the basis of the 2011 GAA. 170

It is worth stressing in this connection that the failure of the GAAs to set aside any amounts for an expense category
sufficiently indicated that Congress purposely did not see fit to fund, much less implement, the PAP concerned. This
indication becomes clearer when even the President himself did not recommend in the NEP to fund the PAP. The
consequence was that any PAP requiring expenditure that did not receive any appropriation under the GAAs could only be
a new PAP, any funding for which would go beyond the authority laid down by Congress in enacting the GAAs. That
happened in some instances under the DAP.

In relation to the December 22, 2011 SARO issued to the Philippine Council for Industry, Energy and Emerging
Technology Research and Development (DOST-PCIEETRD)171 for Establishment of the Advanced Failure Analysis
Laboratory, which reads:

APPROPRIATION PARTICULARS AMOUNT


CODE AUTHORIZED

Development, integration and coordination of the


A.02.a National Research System for Industry, Energy and
Emerging Technology and Related Fields
Capital Outlays P 300,000,000

the appropriation code and the particulars appearing in the SARO did not correspond to the program specified in the GAA,
whose particulars were Research and Management Services(inclusive of the following activities: (1) Technological and
Economic Assessment for Industry, Energy and Utilities; (2) Dissemination of Science and Technology Information; and
(3) Management of PCIERD Information System for Industry, Energy and Utilities. Even assuming that Development,
integration and coordination of the National Research System for Industry, Energy and Emerging Technology and Related
Fields– the particulars stated in the SARO – could fall under the broad program description of Research and Management
Services– as appearing in the SARO, it would nonetheless remain a new activity by reason of its not being specifically
stated in the GAA. As such, the DBM, sans legislative authorization, could not validly fund and implement such PAP under
the DAP.

In defending the disbursements, however, the OSG contends that the Executive enjoyed sound discretion in implementing
the budget given the generality in the language and the broad policy objectives identified under the GAAs;  and that the
172

President enjoyed unlimited authority to spend the initial appropriations under his authority to declare and utilize
savings,  and in keeping with his duty to faithfully execute the laws.
173

Although the OSG rightly contends that the Executive was authorized to spend in line with its mandate to faithfully execute
the laws (which included the GAAs), such authority did not translate to unfettered discretion that allowed the President to
substitute his own will for that of Congress. He was still required to remain faithful to the provisions of the GAAs, given that
his power to spend pursuant to the GAAs was but a delegation to him from Congress. Verily, the power to spend the
public wealth resided in Congress, not in the Executive.  Moreover, leaving the spending power of the Executive
174

unrestricted would threaten to undo the principle of separation of powers. 175

Congress acts as the guardian of the public treasury in faithful discharge of its power of the purse whenever it deliberates
and acts on the budget proposal submitted by the Executive.  Its power of the purse is touted as the very foundation of its
176

institutional strength,  and underpins "all other legislative decisions and regulating the balance of influence between the
177

legislative and executive branches of government."  Such enormous power encompasses the capacity to generate money
178

for the Government, to appropriate public funds, and to spend the money.  Pertinently, when it exercises its power of the
179

purse, Congress wields control by specifying the PAPs for which public money should be spent.

It is the President who proposes the budget but it is Congress that has the final say on matters of appropriations.  For this
180

purpose, appropriation involves two governing principles, namely: (1) "a Principle of the Public Fisc, asserting that all
monies received from whatever source by any part of the government are public funds;" and (2) "a Principle of
Appropriations Control, prohibiting expenditure of any public money without legislative authorization."  To conform with
181

the governing principles, the Executive cannot circumvent the prohibition by Congress of an expenditure for a PAP by
resorting to either public or private funds.  Nor could the Executive transfer appropriated funds resulting in an increase in
182

the budget for one PAP, for by so doing the appropriation for another PAP is necessarily decreased. The terms of both
appropriations will thereby be violated.

b.4 Third Requisite – Cross-border


augmentations from savings were
prohibited by the Constitution

By providing that the President, the President of the Senate, the Speaker of the House of Representatives, the Chief
Justice of the Supreme Court, and the Heads of the Constitutional Commissions may be authorized to augment any item
in the GAA "for their respective offices," Section 25(5), supra, has delineated borders between their offices, such that
funds appropriated for one office are prohibited from crossing over to another office even in the guise of augmentation of a
deficient item or items. Thus, we call such transfers of funds cross-border transfers or cross-border augmentations.

To be sure, the phrase "respective offices" used in Section 25(5), supra, refers to the entire Executive, with respect to the
President; the Senate, with respect to the Senate President; the House of Representatives, with respect to the Speaker;
the Judiciary, with respect to the Chief Justice; the Constitutional Commissions, with respect to their respective
Chairpersons.

Did any cross-border transfers or augmentations transpire?

During the oral arguments on January 28, 2014, Sec. Abad admitted making some cross-border augmentations, to wit:

JUSTICE BERSAMIN:

Alright, the whole time that you have been Secretary of Department of Budget and Management, did the Executive
Department ever redirect any part of savings of the National Government under your control cross border to another
department?

SECRETARY ABAD:

Well, in the Memos that we submitted to you, such an instance, Your Honor

JUSTICE BERSAMIN:

Can you tell me two instances? I don’t recall having read your material.

SECRETARY ABAD:

Well, the first instance had to do with a request from the House of Representatives. They started building their e-library in
2010 and they had a budget for about 207 Million but they lack about 43 Million to complete its 250 Million requirements.
Prior to that, the COA, in an audit observation informed the Speaker that they had to continue with that construction
otherwise the whole building, as well as the equipments therein may suffer from serious deterioration. And at that time,
since the budget of the House of Representatives was not enough to complete 250 Million, they wrote to the President
requesting for an augmentation of that particular item, which was granted, Your Honor. The second instance in the Memos
is a request from the Commission on Audit. At the time they were pushing very strongly the good governance programs of
the government and therefore, part of that is a requirement to conduct audits as well as review financial reports of many
agencies. And in the performance of that function, the Commission on Audit needed information technology equipment as
well as hire consultants and litigators to help them with their audit work and for that they requested funds from the
Executive and the President saw that it was important for the Commission to be provided with those IT equipments and
litigators and consultants and the request was granted, Your Honor.

JUSTICE BERSAMIN:

These cross border examples, cross border augmentations were not supported by appropriations…

SECRETARY ABAD:

They were, we were augmenting existing items within their… (interrupted)

JUSTICE BERSAMIN:

No, appropriations before you augmented because this is a cross border and the tenor or text of the Constitution is quite
clear as far as I am concerned. It says here, "The power to augment may only be made to increase any item in the
General Appropriations Law for their respective offices." Did you not feel constricted by this provision?

SECRETARY ABAD:
Well, as the Constitution provides, the prohibition we felt was on the transfer of appropriations, Your Honor. What we
thought we did was to transfer savings which was needed by the Commission to address deficiency in an existing item in
both the Commission as well as in the House of Representatives; that’s how we saw…(interrupted)

JUSTICE BERSAMIN:

So your position as Secretary of Budget is that you could do that?

SECRETARY ABAD:

In an extreme instances because…(interrupted)

JUSTICE BERSAMIN:

No, no, in all instances, extreme or not extreme, you could do that, that’s your feeling.

SECRETARY ABAD:

Well, in that particular situation when the request was made by the Commission and the House of Representatives, we felt
that we needed to respond because we felt…(interrupted). 183

The records show, indeed, that funds amounting to ₱143,700,000.00 and ₱250,000,000.00 were transferred under the
DAP respectively to the COA  and the House of Representatives.  Those transfers of funds, which constituted cross-
184 185

border augmentations for being from the Executive to the COA and the House of Representatives, are graphed as
follows:
186

AMOUNT
DATE (In thousand pesos)
OFFICE PURPOSE
RELEASED Reserve Releases
Imposed
Commission on IT Infrastructure Program and 11/11/11   143,700
Audit hiring of additional litigation
experts
Congress – Completion of the construction of 07/23/12 207,034 250,000
House of the Legislative Library and (Savings of HOR)
Representatives Archives Building/Congressional
e-library

The respondents further stated in their memorandum that the President "made available" to the "Commission on Elections
the savings of his department upon [its] request for funds…"  This was another instance of a cross-border augmentation.
187

The respondents justified all the cross-border transfers thusly:

99. The Constitution does not prevent the President from transferring savings of his department to another department
upon the latter’s request, provided it is the recipient department that uses such funds to augment its own appropriation. In
such a case, the President merely gives the other department access to public funds but he cannot dictate how they shall
be applied by that department whose fiscal autonomy is guaranteed by the Constitution. 188

In the oral arguments held on February 18, 2014, Justice Vicente V. Mendoza, representing Congress, announced a
different characterization of the cross-border transfers of funds as in the nature of "aid" instead of "augmentation," viz:

HONORABLE MENDOZA:

The cross-border transfers, if Your Honors please, is not an application of the DAP. What were these cross-border
transfers? They are transfers of savings as defined in the various General Appropriations Act. So, that makes it similar to
the DAP, the use of savings. There was a cross-border which appears to be in violation of Section 25, paragraph 5 of
Article VI, in the sense that the border was crossed. But never has it been claimed that the purpose was to augment a
deficient item in another department of the government or agency of the government. The cross-border transfers, if Your
Honors please, were in the nature of [aid] rather than augmentations. Here is a government entity separate and
independent from the Executive Department solely in need of public funds. The President is there 24 hours a day, 7 days
a week. He’s in charge of the whole operation although six or seven heads of government offices are given the power to
augment. Only the President stationed there and in effect in-charge and has the responsibility for the failure of any part of
the government. You have election, for one reason or another, the money is not enough to hold election. There would be
chaos if no money is given as an aid, not to augment, but as an aid to a department like COA. The President is
responsible in a way that the other heads, given the power to augment, are not. So, he cannot very well allow this, if Your
Honor please. 189
JUSTICE LEONEN:

May I move to another point, maybe just briefly. I am curious that the position now, I think, of government is that some
transfers of savings is now considered to be, if I’m not mistaken, aid not augmentation. Am I correct in my hearing of your
argument?

HONORABLE MENDOZA:

That’s our submission, if Your Honor, please.

JUSTICE LEONEN:

May I know, Justice, where can we situate this in the text of the Constitution? Where do we actually derive the concepts
that transfers of appropriation from one branch to the other or what happened in DAP can be considered a said? What
particular text in the Constitution can we situate this?

HONORABLE MENDOZA:

There is no particular provision or statutory provision for that matter, if Your Honor please. It is drawn from the fact that the
Executive is the executive in-charge of the success of the government.

JUSTICE LEONEN:

So, the residual powers labelled in Marcos v. Manglapus would be the basis for this theory of the government?

HONORABLE MENDOZA:

Yes, if Your Honor, please.

JUSTICE LEONEN:

A while ago, Justice Carpio mentioned that the remedy is might be to go to Congress. That there are opportunities and
there have been opportunities of the President to actually go to Congress and ask for supplemental budgets?

HONORABLE MENDOZA:

If there is time to do that, I would say yes.

JUSTICE LEONEN:

So, the theory of aid rather than augmentation applies in extra-ordinary situation?

HONORABLE MENDOZA:

Very extra-ordinary situations.

JUSTICE LEONEN:

But Counsel, this would be new doctrine, in case?

HONORABLE MENDOZA:

Yes, if Your Honor please. 190

Regardless of the variant characterizations of the cross-border transfers of funds, the plain text of Section 25(5), supra,
disallowing cross border transfers was disobeyed. Cross-border transfers, whether as augmentation, or as aid, were
prohibited under Section 25(5), supra.

4.
Sourcing the DAP from unprogrammed
funds despite the original revenue targets
not having been exceeded was invalid

Funding under the DAP were also sourced from unprogrammed funds provided in the GAAs for 2011, 2012,and 2013. The
respondents stress, however, that the unprogrammed funds were not brought under the DAP as savings, but as separate
sources of funds; and that, consequently, the release and use of unprogrammed funds were not subject to the restrictions
under Section 25(5), supra.
The documents contained in the Evidence Packets by the OSG have confirmed that the unprogrammed funds were
treated as separate sources of funds. Even so, the release and use of the unprogrammed funds were still subject to
restrictions, for, to start with, the GAAs precisely specified the instances when the unprogrammed funds could be released
and the purposes for which they could be used.

The petitioners point out that a condition for the release of the unprogrammed funds was that the revenue collections must
exceed revenue targets; and that the release of the unprogrammed funds was illegal because such condition was not
met.191

The respondents disagree, holding that the release and use of the unprogrammed funds under the DAP were in
accordance with the pertinent provisions of the GAAs. In particular, the DBM avers that the unprogrammed funds could be
availed of when any of the following three instances occur, to wit: (1) the revenue collections exceeded the original
revenue targets proposed in the BESFs submitted by the President to Congress; (2) new revenues were collected or
realized from sources not originally considered in the BESFs; or(3) newly-approved loans for foreign assisted projects
were secured, or when conditions were triggered for other sources of funds, such as perfected loan agreements for
foreign-assisted projects.  This view of the DBM was adopted by all the respondents in their Consolidated Comment.
192 193

The BESFs for 2011, 2012 and 2013 uniformly defined "unprogrammed appropriations" as appropriations that provided
standby authority to incur additional agency obligations for priority PAPs when revenue collections exceeded targets, and
when additional foreign funds are generated.  Contrary to the DBM’s averment that there were three instances when
194

unprogrammed funds could be released, the BESFs envisioned only two instances. The third mentioned by the DBM – the
collection of new revenues from sources not originally considered in the BESFs – was not included. This meant that the
collection of additional revenues from new sources did not warrant the release of the unprogrammed funds. Hence, even if
the revenues not considered in the BESFs were collected or generated, the basic condition that the revenue collections
should exceed the revenue targets must still be complied with in order to justify the release of the unprogrammed funds.

The view that there were only two instances when the unprogrammed funds could be released was bolstered by the
following texts of the Special Provisions of the 2011 and 2012 GAAs, to wit:

2011 GAA

1. Release of Fund. The amounts authorized herein shall be released only when the revenue collections exceed the
original revenue targets submitted by the President of the Philippines to Congress pursuant to Section 22, Article VII of the
Constitution, including savings generated from programmed appropriations for the year: PROVIDED, That collections
arising from sources not considered in the aforesaid original revenue targets may be used to cover releases from
appropriations in this Fund: PROVIDED, FURTHER, That in case of newly approved loans for foreign-assisted projects,
the existence of a perfected loan agreement for the purpose shall be sufficient basis for the issuance of a SARO covering
the loan proceeds: PROVIDED, FURTHERMORE, That if there are savings generated from the programmed
appropriations for the first two quarters of the year, the DBM may, subject to the approval of the President, release the
pertinent appropriations under the Unprogrammed Fund corresponding to only fifty percent (50%) of the said savings net
of revenue shortfall: PROVIDED, FINALLY, That the release of the balance of the total savings from programmed
appropriations for the year shall be subject to fiscal programming and approval of the President.

2012 GAA

1. Release of the Fund. The amounts authorized herein shall be released only when the revenue collections exceed the
original revenue targets submitted by the President of the Philippines to Congress pursuant to Section 22, Article VII of the
Constitution: PROVIDED, That collections arising from sources not considered in the aforesaid original revenue targets
may be used to cover releases from appropriations in this Fund: PROVIDED, FURTHER, That in case of newly approved
loans for foreign-assisted projects, the existence of a perfected loan agreement for the purpose shall be sufficient basis for
the issuance of a SARO covering the loan proceeds.

As can be noted, the provisos in both provisions to the effect that "collections arising from sources not considered in the
aforesaid original revenue targets may be used to cover releases from appropriations in this Fund" gave the authority to
use such additional revenues for appropriations funded from the unprogrammed funds. They did not at all waive
compliance with the basic requirement that revenue collections must still exceed the original revenue targets.

In contrast, the texts of the provisos with regard to additional revenues generated from newly-approved foreign loans were
clear to the effect that the perfected loan agreement would be in itself "sufficient basis" for the issuance of a SARO to
release the funds but only to the extent of the amount of the loan. In such instance, the revenue collections need not
exceed the revenue targets to warrant the release of the loan proceeds, and the mere perfection of the loan agreement
would suffice.

It can be inferred from the foregoing that under these provisions of the GAAs the additional revenues from sources not
considered in the BESFs must be taken into account in determining if the revenue collections exceeded the revenue
targets. The text of the relevant provision of the 2013 GAA, which was substantially similar to those of the GAAs for 2011
and 2012, already made this explicit, thus:

1. Release of the Fund. The amounts authorized herein shall be released only when the revenue collections exceed the
original revenue targets submitted by the President of the Philippines to Congress pursuant to Section 22, Article VII of the
Constitution, including collections arising from sources not considered in the aforesaid original revenue target, as certified
by the BTr: PROVIDED, That in case of newly approved loans for foreign-assisted projects, the existence of a perfected
loan agreement for the purpose shall be sufficient basis for the issuance of a SARO covering the loan proceeds.

Consequently, that there were additional revenues from sources not considered in the revenue target would not be
enough. The total revenue collections must still exceed the original revenue targets to justify the release of the
unprogrammed funds (other than those from newly-approved foreign loans).

The present controversy on the unprogrammed funds was rooted in the correct interpretation of the phrase "revenue
collections should exceed the original revenue targets." The petitioners take the phrase to mean that the total revenue
collections must exceed the total revenue target stated in the BESF, but the respondents understand the phrase to refer
only to the collections for each source of revenue as enumerated in the BESF, with the condition being deemed complied
with once the revenue collections from a particular source already exceeded the stated target.

The BESF provided for the following sources of revenue, with the corresponding revenue target stated for each source of
revenue, to wit:

TAX REVENUES

Taxes on Net Income and Profits


Taxes on Property
Taxes on Domestic Goods and Services

General Sales, Turnover or VAT


Selected Excises on Goods

Selected Taxes on Services


Taxes on the Use of Goods or Property or Permission to Perform Activities
Other Taxes
Taxes on International Trade and Transactions

NON-TAX REVENUES

Fees and Charges


BTR Income

Government Services
Interest on NG Deposits
Interest on Advances to Government Corporations
Income from Investments

Interest on Bond Holdings

Guarantee Fee
Gain on Foreign Exchange
NG Income Collected by BTr

Dividends on Stocks
NG Share from Airport Terminal Fee
NG Share from PAGCOR Income
NG Share from MIAA Profit

Privatization
Foreign Grants

Thus, when the Court required the respondents to submit a certification from the Bureau of Treasury (BTr) to the effect
that the revenue collections had exceeded the original revenue targets,  they complied by submitting certifications from
195

the BTr and Department of Finance (DOF) pertaining to only one identified source of revenue – the dividends from the
shares of stock held by the Government in government-owned and controlled corporations.

To justify the release of the unprogrammed funds for 2011, the OSG presented the certification dated March 4, 2011
issued by DOF Undersecretary Gil S. Beltran, as follows:

This is to certify that under the Budget for Expenditures and Sources of Financing for 2011, the programmed income from
dividends from shares of stock in government-owned and controlled corporations is 5.5 billion.

This is to certify further that based on the records of the Bureau of Treasury, the National Government has recorded
dividend income amounting to ₱23.8 billion as of 31 January 2011. 196

For 2012, the OSG submitted the certification dated April 26, 2012 issued by National Treasurer Roberto B. Tan, viz:
This is to certify that the actual dividend collections remitted to the National Government for the period January to March
2012 amounted to ₱19.419 billion compared to the full year program of ₱5.5 billion for 2012. 197

And, finally, for 2013, the OSG presented the certification dated July 3, 2013 issued by National Treasurer Rosalia V. De
Leon, to wit:

This is to certify that the actual dividend collections remitted to the National Government for the period January to May
2013 amounted to ₱12.438 billion compared to the full year program of ₱10.0  billion for 2013.
198

Moreover, the National Government accounted for the sale of the right to build and operate the NAIA expressway
amounting to ₱11.0 billion in June 2013. 199

The certifications reflected that by collecting dividends amounting to ₱23.8 billion in 2011, ₱19.419 billion in 2012, and
₱12.438 billion in 2013 the BTr had exceeded only the ₱5.5 billion in target revenues in the form of dividends from stocks
in each of 2011 and 2012, and only the ₱10 billion in target revenues in the form of dividends from stocks in 2013.

However, the requirement that revenue collections exceed the original revenue targets was to be construed in light of the
purpose for which the unprogrammed funds were incorporated in the GAAs as standby appropriations to support
additional expenditures for certain priority PAPs should the revenue collections exceed the resource targets assumed in
the budget or when additional foreign project loan proceeds were realized. The unprogrammed funds were included in the
GAAs to provide ready cover so as not to delay the implementation of the PAPs should new or additional revenue sources
be realized during the year.  Given the tenor of the certifications, the unprogrammed funds were thus not yet supported
200

by the corresponding resources. 201

The revenue targets stated in the BESF were intended to address the funding requirements of the proposed programmed
appropriations. In contrast, the unprogrammed funds, as standby appropriations, were to be released only when there
were revenues in excess of what the programmed appropriations required. As such, the revenue targets should be
considered as a whole, not individually; otherwise, we would be dealing with artificial revenue surpluses. The requirement
that revenue collections must exceed revenue target should be understood to mean that the revenue collections must
exceed the total of the revenue targets stated in the BESF. Moreover, to release the unprogrammed funds simply because
there was an excess revenue as to one source of revenue would be an unsound fiscal management measure because it
would disregard the budget plan and foster budget deficits, in contravention of the Government’s surplus budget policy. 202

We cannot, therefore, subscribe to the respondents’ view.

5.
Equal protection, checks and balances,
and public accountability challenges

The DAP is further challenged as violative of the Equal Protection Clause, the system of checks and balances, and the
principle of public accountability.

With respect to the challenge against the DAP under the Equal Protection Clause,  Luna argues that the implementation
203

of the DAP was "unfair as it [was] selective" because the funds released under the DAP was not made available to all the
legislators, with some of them refusing to avail themselves of the DAP funds, and others being unaware of the availability
of such funds. Thus, the DAP practised "undue favoritism" in favor of select legislators in contravention of the Equal
Protection Clause.

Similarly, COURAGE contends that the DAP violated the Equal Protection Clause because no reasonable classification
was used in distributing the funds under the DAP; and that the Senators who supposedly availed themselves of said funds
were differently treated as to the amounts they respectively received.

Anent the petitioners’ theory that the DAP violated the system of checks and balances, Luna submits that the grant of the
funds under the DAP to some legislators forced their silence about the issues and anomalies surrounding the DAP.
Meanwhile, Belgica stresses that the DAP, by allowing the legislators to identify PAPs, authorized them to take part in the
implementation and execution of the GAAs, a function that exclusively belonged to the Executive; that such situation
constituted undue and unjustified legislative encroachment in the functions of the Executive; and that the President
arrogated unto himself the power of appropriation vested in Congress because NBC No. 541 authorized the use of the
funds under the DAP for PAPs not considered in the 2012 budget.

Finally, the petitioners insist that the DAP was repugnant to the principle of public accountability enshrined in the
Constitution,  because the legislators relinquished the power of appropriation to the Executive, and exhibited a reluctance
204

to inquire into the legality of the DAP.

The OSG counters the challenges, stating that the supposed discrimination in the release of funds under the DAP could
be raised only by the affected Members of Congress themselves, and if the challenge based on the violation of the Equal
Protection Clause was really against the constitutionality of the DAP, the arguments of the petitioners should be directed
to the entitlement of the legislators to the funds, not to the proposition that all of the legislators should have been given
such entitlement.
The challenge based on the contravention of the Equal Protection Clause, which focuses on the release of funds under
the DAP to legislators, lacks factual and legal basis. The allegations about Senators and Congressmen being unaware of
the existence and implementation of the DAP, and about some of them having refused to accept such funds were
unsupported with relevant data. Also, the claim that the Executive discriminated against some legislators on the ground
alone of their receiving less than the others could not of itself warrant a finding of contravention of the Equal Protection
Clause. The denial of equal protection of any law should be an issue to be raised only by parties who supposedly suffer it,
and, in these cases, such parties would be the few legislators claimed to have been discriminated against in the releases
of funds under the DAP. The reason for the requirement is that only such affected legislators could properly and fully bring
to the fore when and how the denial of equal protection occurred, and explain why there was a denial in their situation.
The requirement was not met here. Consequently, the Court was not put in the position to determine if there was a denial
of equal protection. To have the Court do so despite the inadequacy of the showing of factual and legal support would be
to compel it to speculate, and the outcome would not do justice to those for whose supposed benefit the claim of denial of
equal protection has been made.

The argument that the release of funds under the DAP effectively stayed the hands of the legislators from conducting
congressional inquiries into the legality and propriety of the DAP is speculative. That deficiency eliminated any need to
consider and resolve the argument, for it is fundamental that speculation would not support any proper judicial
determination of an issue simply because nothing concrete can thereby be gained. In order to sustain their constitutional
challenges against official acts of the Government, the petitioners must discharge the basic burden of proving that the
constitutional infirmities actually existed.  Simply put, guesswork and speculation cannot overcome the presumption of the
205

constitutionality of the assailed executive act.

We do not need to discuss whether or not the DAP and its implementation through the various circulars and memoranda
of the DBM transgressed the system of checks and balances in place in our constitutional system. Our earlier expositions
on the DAP and its implementing issuances infringing the doctrine of separation of powers effectively addressed this
particular concern.

Anent the principle of public accountability being transgressed because the adoption and implementation of the DAP
constituted an assumption by the Executive of Congress’ power of appropriation, we have already held that the DAP and
its implementing issuances were policies and acts that the Executive could properly adopt and do in the execution of the
GAAs to the extent that they sought to implement strategies to ramp up or accelerate the economy of the country.

6.
Doctrine of operative fact was applicable

After declaring the DAP and its implementing issuances constitutionally infirm, we must now deal with the consequences
of the declaration.

Article 7 of the Civil Code provides:

Article 7. Laws are repealed only by subsequent ones, and their violation or non-observance shall not be excused by
disuse, or custom or practice to the contrary.

When the courts declared a law to be inconsistent with the Constitution, the former shall be void and the latter shall
govern.

Administrative or executive acts, orders and regulations shall be valid only when they are not contrary to the laws or the
Constitution.

A legislative or executive act that is declared void for being unconstitutional cannot give rise to any right or
obligation.  However, the generality of the rule makes us ponder whether rigidly applying the rule may at times be
206

impracticable or wasteful. Should we not recognize the need to except from the rigid application of the rule the instances in
which the void law or executive act produced an almost irreversible result?

The need is answered by the doctrine of operative fact. The doctrine, definitely not a novel one, has been exhaustively
explained in De Agbayani v. Philippine National Bank: 207

The decision now on appeal reflects the orthodox view that an unconstitutional act, for that matter an executive order or a
municipal ordinance likewise suffering from that infirmity, cannot be the source of any legal rights or duties. Nor can it
justify any official act taken under it. Its repugnancy to the fundamental law once judicially declared results in its being to
all intents and purposes a mere scrap of paper. As the new Civil Code puts it: ‘When the courts declare a law to be
inconsistent with the Constitution, the former shall be void and the latter shall govern.’ Administrative or executive acts,
orders and regulations shall be valid only when they are not contrary to the laws of the Constitution. It is understandable
why it should be so, the Constitution being supreme and paramount. Any legislative or executive act contrary to its terms
cannot survive.

Such a view has support in logic and possesses the merit of simplicity. It may not however be sufficiently realistic. It does
not admit of doubt that prior to the declaration of nullity such challenged legislative or executive act must have been in
force and had to be complied with. This is so as until after the judiciary, in an appropriate case, declares its invalidity, it is
entitled to obedience and respect. Parties may have acted under it and may have changed their positions. What could be
more fitting than that in a subsequent litigation regard be had to what has been done while such legislative or executive
act was in operation and presumed to be valid in all respects. It is now accepted as a doctrine that prior to its being
nullified, its existence as a fact must be reckoned with. This is merely to reflect awareness that precisely because the
judiciary is the governmental organ which has the final say on whether or not a legislative or executive measure is valid, a
period of time may have elapsed before it can exercise the power of judicial review that may lead to a declaration of nullity.
It would be to deprive the law of its quality of fairness and justice then, if there be no recognition of what had transpired
prior to such adjudication.

In the language of an American Supreme Court decision: ‘The actual existence of a statute, prior to such a determination
[of unconstitutionality], is an operative fact and may have consequences which cannot justly be ignored. The past cannot
always be erased by a new judicial declaration. The effect of the subsequent ruling as to invalidity may have to be
considered in various aspects, with respect to particular relations, individual and corporate, and particular conduct, private
and official.’"

The doctrine of operative fact recognizes the existence of the law or executive act prior to the determination of its
unconstitutionality as an operative fact that produced consequences that cannot always be erased, ignored or
disregarded. In short, it nullifies the void law or executive act but sustains its effects. It provides an exception to the
general rule that a void or unconstitutional law produces no effect.  But its use must be subjected to great scrutiny and
208

circumspection, and it cannot be invoked to validate an unconstitutional law or executive act, but is resorted to only as a
matter of equity and fair play.  It applies only to cases where extraordinary circumstances exist, and only when the
209

extraordinary circumstances have met the stringent conditions that will permit its application.

We find the doctrine of operative fact applicable to the adoption and implementation of the DAP. Its application to the DAP
proceeds from equity and fair play. The consequences resulting from the DAP and its related issuances could not be
ignored or could no longer be undone.

To be clear, the doctrine of operative fact extends to a void or unconstitutional executive act. The term executive act is
broad enough to include any and all acts of the Executive, including those that are quasi legislative and quasi-judicial in
nature. The Court held so in Hacienda Luisita, Inc. v. Presidential Agrarian Reform Council: 210

Nonetheless, the minority is of the persistent view that the applicability of the operative fact doctrine should be limited to
statutes and rules and regulations issued by the executive department that are accorded the same status as that of a
statute or those which are quasi-legislative in nature. Thus, the minority concludes that the phrase ‘executive act’ used in
the case of De Agbayani v. Philippine National Bank refers only to acts, orders, and rules and regulations that have the
force and effect of law. The minority also made mention of the Concurring Opinion of Justice Enrique Fernando in
Municipality of Malabang v. Benito, where it was supposedly made explicit that the operative fact doctrine applies to
executive acts, which are ultimately quasi-legislative in nature.

We disagree. For one, neither the De Agbayani case nor the Municipality of Malabang case elaborates what ‘executive
act’ mean. Moreover, while orders, rules and regulations issued by the President or the executive branch have fixed
definitions and meaning in the Administrative Code and jurisprudence, the phrase ‘executive act’ does not have such
specific definition under existing laws. It should be noted that in the cases cited by the minority, nowhere can it be found
that the term ‘executive act’ is confined to the foregoing. Contrarily, the term ‘executive act’ is broad enough to encompass
decisions of administrative bodies and agencies under the executive department which are subsequently revoked by the
agency in question or nullified by the Court.

A case in point is the concurrent appointment of Magdangal B. Elma (Elma) as Chairman of the Presidential Commission
on Good Government (PCGG) and as Chief Presidential Legal Counsel (CPLC) which was declared unconstitutional by
this Court in Public Interest Center, Inc. v. Elma. In said case, this Court ruled that the concurrent appointment of Elma to
these offices is in violation of Section 7, par. 2, Article IX-B of the 1987 Constitution, since these are incompatible offices.
Notably, the appointment of Elma as Chairman of the PCGG and as CPLC is, without a question, an executive act. Prior to
the declaration of unconstitutionality of the said executive act, certain acts or transactions were made in good faith and in
reliance of the appointment of Elma which cannot just be set aside or invalidated by its subsequent invalidation.

In Tan v. Barrios, this Court, in applying the operative fact doctrine, held that despite the invalidity of the jurisdiction of the
military courts over civilians, certain operative facts must be acknowledged to have existed so as not to trample upon the
rights of the accused therein. Relevant thereto, in Olaguer v. Military Commission No. 34, it was ruled that ‘military
tribunals pertain to the Executive Department of the Government and are simply instrumentalities of the executive power,
provided by the legislature for the President as Commander-in-Chief to aid him in properly commanding the army and
navy and enforcing discipline therein, and utilized under his orders or those of his authorized military representatives.’

Evidently, the operative fact doctrine is not confined to statutes and rules and regulations issued by the executive
department that are accorded the same status as that of a statute or those which are quasi-legislative in nature.

Even assuming that De Agbayani initially applied the operative fact doctrine only to executive issuances like orders and
rules and regulations, said principle can nonetheless be applied, by analogy, to decisions made by the President or the
agencies under the executive department. This doctrine, in the interest of justice and equity, can be applied liberally and in
a broad sense to encompass said decisions of the executive branch. In keeping with the demands of equity, the Court can
apply the operative fact doctrine to acts and consequences that resulted from the reliance not only on a law or executive
act which is quasi-legislative in nature but also on decisions or orders of the executive branch which were later nullified.
This Court is not unmindful that such acts and consequences must be recognized in the higher interest of justice, equity
and fairness.
Significantly, a decision made by the President or the administrative agencies has to be complied with because it has the
force and effect of law, springing from the powers of the President under the Constitution and existing laws. Prior to the
nullification or recall of said decision, it may have produced acts and consequences in conformity to and in reliance of said
decision, which must be respected. It is on this score that the operative fact doctrine should be applied to acts and
consequences that resulted from the implementation of the PARC Resolution approving the SDP of HLI. (Bold
underscoring supplied for emphasis)

In Commissioner of Internal Revenue v. San Roque Power Corporation,  the Court likewise declared that "for the
211

operative fact doctrine to apply, there must be a ‘legislative or executive measure,’ meaning a law or executive issuance."
Thus, the Court opined there that the operative fact doctrine did not apply to a mere administrative practice of the Bureau
of Internal Revenue, viz:

Under Section 246, taxpayers may rely upon a rule or ruling issued by the Commissioner from the time the rule or ruling is
issued up to its reversal by the Commissioner or this Court. The reversal is not given retroactive effect. This, in essence, is
the doctrine of operative fact. There must, however, be a rule or ruling issued by the Commissioner that is relied upon by
the taxpayer in good faith. A mere administrative practice, not formalized into a rule or ruling, will not suffice because such
a mere administrative practice may not be uniformly and consistently applied. An administrative practice, if not formalized
as a rule or ruling, will not be known to the general public and can be availed of only by those with informal contacts with
the government agency.

It is clear from the foregoing that the adoption and the implementation of the DAP and its related issuances were executive
acts.  The DAP itself, as a policy, transcended a merely administrative practice especially after the Executive, through the
1avvphi1

DBM, implemented it by issuing various memoranda and circulars. The pooling of savings pursuant to the DAP from the
allotments made available to the different agencies and departments was consistently applied throughout the entire
Executive. With the Executive, through the DBM, being in charge of the third phase of the budget cycle – the budget
execution phase, the President could legitimately adopt a policy like the DAP by virtue of his primary responsibility as the
Chief Executive of directing the national economy towards growth and development. This is simply because savings could
and should be determined only during the budget execution phase.

As already mentioned, the implementation of the DAP resulted into the use of savings pooled by the Executive to finance
the PAPs that were not covered in the GAA, or that did not have proper appropriation covers, as well as to augment items
pertaining to other departments of the Government in clear violation of the Constitution. To declare the implementation of
the DAP unconstitutional without recognizing that its prior implementation constituted an operative fact that produced
consequences in the real as well as juristic worlds of the Government and the Nation is to be impractical and unfair.
Unless the doctrine is held to apply, the Executive as the disburser and the offices under it and elsewhere as the
recipients could be required to undo everything that they had implemented in good faith under the DAP. That scenario
would be enormously burdensome for the Government. Equity alleviates such burden.

The other side of the coin is that it has been adequately shown as to be beyond debate that the implementation of the
DAP yielded undeniably positive results that enhanced the economic welfare of the country. To count the positive results
may be impossible, but the visible ones, like public infrastructure, could easily include roads, bridges, homes for the
homeless, hospitals, classrooms and the like. Not to apply the doctrine of operative fact to the DAP could literally cause
the physical undoing of such worthy results by destruction, and would result in most undesirable wastefulness.

Nonetheless, as Justice Brion has pointed out during the deliberations, the doctrine of operative fact does not always
apply, and is not always the consequence of every declaration of constitutional invalidity. It can be invoked only in
situations where the nullification of the effects of what used to be a valid law would result in inequity and injustice;  but
212

where no such result would ensue, the general rule that an unconstitutional law is totally ineffective should apply.

In that context, as Justice Brion has clarified, the doctrine of operative fact can apply only to the PAPs that can no longer
be undone, and whose beneficiaries relied in good faith on the validity of the DAP, but cannot apply to the authors,
proponents and implementors of the DAP, unless there are concrete findings of good faith in their favor by the proper
tribunals determining their criminal, civil, administrative and other liabilities.

WHEREFORE, the Court PARTIALLY GRANTS the petitions for certiorari and prohibition; and DECLARES the following
acts and practices under the Disbursement Acceleration Program, National Budget Circular No. 541 and related executive
issuances UNCONSTITUTIONAL for being in violation of Section 25(5), Article VI of the 1987 Constitution and the
doctrine of separation of powers, namely:

(a) The withdrawal of unobligated allotments from the implementing agencies, and the declaration of the withdrawn
unobligated allotments and unreleased appropriations as savings prior to the end of the fiscal year and without
complying with the statutory definition of savings contained in the General Appropriations Acts;

(b) The cross-border transfers of the savings of the Executive to augment the appropriations of other offices
outside the Executive; and

(c) The funding of projects, activities and programs that were not covered by any appropriation in the General
Appropriations Act.
The Court further DECLARES VOID the use of unprogrammed funds despite the absence of a certification by the National
Treasurer that the revenue collections exceeded the revenue targets for non-compliance with the conditions provided in
the relevant General Appropriations Acts.

SO ORDERED.

You might also like